Está en la página 1de 769

, ,

Quinta edicin

Jess Fraile Mora


Catedrtico de Electrotecnia E.T.S.I. Caminos, Canales y Puertos

U.P.M.

i '-li' U

',l.~RTE'-" J....., ~1'-\ O.. : ::Me ~ L_.


~raw ""ii' Interainericana de Vanell S,A

Me Hill

Graw
MADRID BUENOS AIRES CARACAS GUATEMALA LISBOA MXICO NUEVA YORK PANAM. SAN JUAN SANTAF DE BOGOT SANTIAGO so PAULO AUCKLAND HAMBURGO LONDRES MILN MONTREAL NUEVA DELHI PARs SAN FRANCISCO SIDNEY SINGAPUR STo LOUIS TOKIO TORONTO

MAQUINAS ELECTRICAS. Quinta edicin


No est permitida la reproduccin total o parcial de este libro, ni su tratamiento informtico, ni la transmisin de ninguna forma o por cualquier medio, ya sea electrnico, mecnico , por fotocopia, por registro u otros mtodos, sin el permiso previo y por escrito de los titulares del Copyright. DERECHOS RESERVADOS 2003, respecto de la quinta edicin en espaol, por McGRAW-HILLIINTERAMERICANA DE ESPANA, S. A. U. Edificio Valrealty, l ." planta Basauri, 17 28023 Aravaca (Madrid)

ISBN: 84-481-3913-5 Depsito legal: M. 22.399-2003 Editora: Concepcin Femndez Madrid Asist, Editorial: Amelia Nieva Diseno de cubierta: Desing Master DIMA Preimpresin: MonoComp, S. A. Impreso en EDIGRAFOS, S. A.

IMPRESO EN ESPANA - PRINTED IN SPAIN

Contenido

Acerca del autor

..

111

....

..

..

..

..

111

....

111

..

..

111

111

.......

111

XIII

Prlogo . . . . . . . . . . . . . . . . . . . . . . . . . . . . . . . . . . . . . . . . . . . . . . . . . . . . . . . . . . . . . . . . . . Agradecimien tos . . . . . . . . . . . . . . . . . . . . . . . . . . . . . . . . . . . . . . . . . . . . . . . . . . . . . . . . .. Captulo 1: CIRCUITOS MAGNTICOS Y CONVERSIN DE ENERGA


1.1. Introduccin
111 111 111 111 111 111 " 111 , ~ .

xv
XiX

1.2. Materiales magnticos . . . . . . . . . . . . . . . . . . . . . . . . . . . . . . . . . . . . . . . . . . . 1 2.1. Diamagnetismo . 1.2.2. Paramagnetismo . 1.2.3. Ferromagnetismo y ciclo de histresis . 1.3. Leyes de los circuitos magnticos . ", .,;. .. E nergla y coenergla magneuca . . . . . . . . . . . . . . . . . . . . . . . . . . . . . . . . . . . . . . . .. 14 1.5. Prdidas de energa en los ncleos ferromagnticos . 1.5.1 . Prdidas por hi stresis . . . . . . . . . . . . . . . . . . . . Consecuencias tecnolgicas . 1.6. Circuitos magnticos excitados con corriente alterna . 1.6.1. Generalidades............................................... .. 1.6.2. Circuito elctrico equivalente de una bobina con de hierro alimentada
con c.a 1.6.3. ,. Corriente de excitacin en una bobina con ncleo de hierro alimentada
t t

1 3

3
4 4

8
19

1.5.2. 1.5.3.

Prdidas por corrientes de Foucault

23 23 25

27
30 30 33 35
39

It

..

con c.a

..

..

..

1.7. Conversin de energa en sistemas magnticos con movimiento de traslacin.


Electroimanes ... ..... _... ....... . ... ... . ... . . . ... ... . . . . . ........
I .. .. ..

1.8. Conversin de energa en sistemas magnticos con movimiento de rotacin. M. ] ". . quinas e ectncas rotativas . 1.8.1. Sistemas magnticos de rotacin alimentados con una sola fuente. Motores de reluctancia. . . . . . . . . . . . . . . . . . . . . . . . . . . . . . . . . . . . . . . . . . . . . . ] .8.2. Sistemas magnticos de rotacin alimentados con dos fuentes. . . . . . . . Problemas Biografas. . . . . . . . . . . . . . . . . . . . . . . . . . . . . . . . . . . . . . . . . . . . . . . . . . . . . . . . . . . . . . . . . Referencias ......
I 11 , .. .. .. f .. 4 " 4 I lO ...... I

55 55
60

75 83 85 87 87
88

Captulo 2: PRINCIPIOS GENERALES DE LAS MQUINAS ELCTRICAS. . . .

24 l. 2.2.

Introduccin

"

Elementos bsicos de las mquinas elctricas. . . . . . . . . . . . . . . . . . . . . . . . . . . .

vi 2.3. 2 .4. 2.5. 2.6.

C01VTEJ.VIDO
.

Colector de delgas y colector de anillos. . . . . . . . . . . . . . . . . . . . . . . . . . . . . . . . De vanad os . . ... . . . . . . . . . . . . . . . ... .., ... ... ... ... . . ..... " ...................,"" ....'"" ..'"..". " Prdidas y calentamiento.. . . . . . . . . . . . . . . . . . . . . . . . . . . . . . . . . . . . . . . . . . .. Potencia asignada o nominal. Tipos de servicio. . . . . . . . . . . . . . . . . . . . . . . . .. 2.7. Rendimiento . . . . . . . . . . . . . . . . . . . . . . . . . . . . . . . . . . . . . . . . . . . . . . . . . . . . . . .. 2.8.. F.m.m. y campo magntico en el entrehierro de una mquina elctrica. . . . .. 2.8.1. Campo magntico y f.m.m. producida por un devanado concentrado de paso diametral . . . . . . . . . . . . . . . . . . . . . . . . . . . . . . . . . . . . . . . . . . . . 2.8.2. F.m.m. producida por un devanado' distribuido . . . . . . . . . . . . . . . . . .. 2.8.3. F.m.m. producida por un devanado trifsico. Campos giratorios. Teorema de Ferraris . . . . " ... . ... . . . . .." ". , .... " . "..... '"....... ... ... ..... ..... " 2.8.4. Relacin entre un campo alternativo y un campo giratorio. Teorema de Leb lanc . . . . . . . . . . . . . . . . . . . . . . . . . . . . . . . . . . . . . . . . . . . . . . . .. 2.9. F.e.m. inducida en un devanado de una mquina elctrica ..... ,. . . . . . . . . .. . .' \ 2.9.1. Generalldades............. _. . . . . . . . . . . . . . . . . . . . . . . . . . . . . . . . . 2.9.2. Factores que afectan a la f.e.m. inducida en un devanado. . . . . . . .. 2.9.3. Armnicos de f.e.m.: origen y eliminacin. . . . . . . . . . . . . . . . . . . . .. 2.10. Par electromagntico en las mquinas elctricas. . . . . . . . . . . . . . . . . . . . . . . .. 2.11 ~ Clasificacin general de las mquinas elctricas. . . . . . . . . . . . . . . . . . . . . . . .. 2.12. Anlisis cualitativo de las principales mquinas elctricas . . . . . . . . . . . . . . . .. 2.12.1. Transformadores . . . . . . . . . . . . . . . . . . . . . . . . . . . . . . . . . . . . . . . . . . .. 2.12.2. Mquinas sncronas................... . . . . . . . . . . . . . . . . . . . . . .. 2.12.3. Mquinas de c.c. . . . . . . . . . . . . . . . . . . . . . . . . . . . . . . . . . . . . . . . . . . .. 2.12.4. Mquinas asncronas o de induccin. . . . . . . . . . . . . . . . . . . . . . . . . .. 2.12.5. Motores de c.a. de colector. Motores universales................. Problemas: .. Biografas. . . . . . . . . . . . . . . . . . . . . . . . . . . . . . . . . . . . . . . . . . . . . . . . . . . . . . . . . . . . . . . .. Referencias "
41 .. .. .. .. " lo .. .. .. .. 41 .... 41 .. .. 111 , .... , ....... " 111 .. ... lo .. .. .. .. .. ..

92 97 100 103

107
110 111 117 120 125 126 126 130 135

138
142 145 145 146 147 149 151 152

157
159 161

Captulo 3: TRANSFORMADORES. . . . . . . . . . . . . . . . . . . . . . . . . . . . . . . . . . . . . . . .. 3.1.

Introduccin........................................................ 161 3.2. Principales aspectos constructivos. . . . . . . . . . . . . . . . . . . . . . . . . . . . . . . . . . . . . . 162 3.. 3. Principio de funcionamiento de un transformador ideal. . . . . . . . . . . . . . . . . .. 172 3.4. Funcionamiento de un transformador real. . . . . . . . . . . . . . . . . . . . . . . . . . . . . .. 178 3.5. Circuito equivalente de un transformador , . . . . . . . . . . . . . . . . . . . . . . . . . . . . . . 182 3.6. Ensayos del transformador. . . . . . . . . . . . . . . . . . . . . . . . . . . . . . . . . . . . . . . . . . . . 187 3.6.1. Ensayo de vaco. . . . . . . . . . . . . . . . . . . . . . . . . . . . . . . . . . . . . . . . . . .. 188 3.6.2. Ensayo de cortocircuito. . . . . . . . . . . . . . . . . . . . . . . . . . . . . . . . . . . . . . 190 3.7. Cada de tensin en un transformador. . . . . . . . . . . . . . . . . . . . . . . . . . . . . . . . .. 195 3.8. Prdidas y rendimiento de .un transformador ... ~. . . . . . . . . . . . . . . . . . . . . . . .. 200 3.9. Corriente de excitacin o de vaco de un transformador. Armnicos de la co. de vaco " . ... . . . . . . . . . . ... . mente 202 3.10. Corriente de conexin de un transformador . 203 3.11. Transformadores trifsicos. . . . . . . . . . . . . . . . . . . . . . . . . . . . . . . . . . . . . . . . . . . . 204 3.11.1. Generalidades............................................... 204 3.11.2. Armnicos en las corrientes de excitacin de transformadores trifsiI " " 11

11

11

cos ........
3.11.3.

ti

..

..

..

..

..

..

..

..

..

..

..

..

..

..

..

..

..

..

..

..

..

..

..

..

..

..

...

210
216

Conexiones de los transformadores

trifsicos. . . . . . . . . . . . . . . . . . ..

CONTENIDO
,
<

VII

, t ,

3.12. 3.13. 3.14.

3.15.

3.16.

Acoplamiento en paralelo de transformadores. . . . . . . . . . . . . . . . . . . . . . . . . .. Autotransformadores . . . . . . . . . . . . . . . . . . . . . . . . . . . . . . . . . . . . . . . . . . . . . . . .. Transformadores con tomas. . . . . . . . . . . . . . . . . . . . . . . . . . . . . . . . . . . . . . . . . .. 3.14.1. Tomas de regulacin. . . . . . . . . . . . . . . . . . . . . . . . . . . . . . . . . . . . . . . .. 3. 14.2. Elementos de conmutacin. . . . . . . . . . . . . . . . . . . . . . . . . . . . . . . . . . .. Transformadores de medida. . . . . . . . . . . . . . . . . . . . . . . . . . . . . . . . . . . . . . . . . .. 3.15.1. Transformadores de tensin. . . . . . . . . . . . . . . . . . . . . . . . . . . . . . . . . .. 3.15.2. Transformadores de corriente........................ . . . . . . . .. Apndice: Transformaciones especiales. . . . . . . . . . . . . . . . . . . . . . . . . . . . . . . .. 3.16.1. Transformacin trifsica a bifsica y viceversa. Conexi6n Scott . . .. 3.16.2. Transformacin trifsica a hexafsica . . . . . . . . . . . . . . . . . . . . . . . . ..
I .. " .. .. .. " .. .. .. " " " .. .. .. .. " .. " ....

227 231 232 233 234 236 236 238 241 241 243

Pro b 1emas . . . " . ... ... . . " . . . " ... .." " . . . ... ... . .
I " " .. .. ..

244
_

Biografas. . . . . . . . . . . . . . . . . . . . . . . . . . . . . . . . . . . . . . . . . . . . . . . . . . . . . . . . . . . . . . . .. Referencias .." "..... . ..". ... ..


.. .. .. .. .. .. .. .. ,. .. .. .. "

254 257 259 259 261 264 270 278 278 280
281

, ,
<

Captulo 4: MQUINAS ASNCRONAS O DE INDUCCIN. . . . . . . . . . . . . . . . .. Introduccin -........ Aspectos constructivos............................................... Principio de funcionamientt).......................................... Circuito equivalente del motor asncrono. . . . . . . . . . . . . . . . . . . . . . . . . . . . . .. Ensayos del motor asncrono. . . . . . . . . . . . . . . . . . . . . . . . . . . . . . . . . . . . . . . . .. 4.5.1. Ensayo de vaco o de rotor libre. . . . . . . . . . . . . . . . . . . . . . . . . . . . . .. 4.5.2. Ensayo de cortocircuito o de rotor bloqueado. . . . . . . . . . . . . . . . . . .. 4.6. Balance de potencias. . . . . . . . . . . . . . . . . . . . . . . . . . . . . . . . . . . . . . . . . . . . . . . .. 4.7. Par de rotacin . . . . . . . . . . . . . . . . . . . . . . . . . . . . . . . . . . . . . . . . . . . . . . . . . . . . .. 4.7.1. Generalidades............................................... 4.7.2. Tipos de funcionamiento de la mquina asncrona. . . . . . . . . . . . . . .. 4.R. Diagrama del crculo , . . . . . . . . . . . . . . . . . . . . . . . . . . . . . . . . . . . . . . . . .. 4.8.1. Introduccin................... . . . . . . . . . . . . . . . . . . . . . . . . . . . .. 4.8.2. Deduccin del diagrama circular. . . . . . . . . . . . . . . . . . . . . . . . . . . . . .. 4.8.3. Eleccin de las escalas en el diagrama del crculo. . . . . . . . . . . . . . .. 4.9. Arranque........................................................... 4.9.1. Arranque de los motores en j aula de ardi 11a. . . . . . . . . . . . . . . . . . . .. 4.9.2. Arranque de los motores de rotor bobinado. . . . . . . . . . . . . . . . . . . .. 4.10. Motores de doble jaula de ardilla. . . . . . . . . . . . . . . . . . . . . . . . . . . . . . . . . . . . .. 4.11. Regulacin de velocidad . . . . . . . . . . . . . . . . . . . . . . . . . . . . . . . . . . . . . . . . . . . . .. 4.11.1. Regulacin por variacin del nmero de polos. . . . . . . . . . . . . . . . . .. 4.11.2. Regulacin por variacin del deslizamiento. . . . . . . . . . . . . . . . . . . .. 4.11.3. Regulacin por variacin en la frecuencia ~. 4.12. Dinmica del motor asncrono. . . . . . . . . . . . . . . . . . . . . . . . . . . . . . . . . . . . . . . .. 4.12.1 . Generalidades............................................... 4.12.2. Tiempo de arranque de un motor asncrono. . . . . . . . . . . . . . . . . . . .. 4.12.3. Prdidas de energa en rgimen dinmico " 4.13. Motor de induccin monofsico " 4.13.1. Principio de funcionamiento........... . . . . . . . . . . . . . . . . . . . . . .. 4.13.2. Circuito equivalente. . . . . . . . . . . . . . . . . . . . . . . . . . . . . . . . . . . . . . . . . 4.13.3. Arranque de los motores de induccin monofsicos. . . . . . . . . . . . .. 4.1. 4.2. 4.3. 4.4. 4.5.

, . >

,
l
,

, ,

l
J ~

I
I

289 289 293 310 310 311 315 316 317 320 325 327 328 330 330

, ,
'.

331
331 333 334 337 337 339 341

,
i

, ,

VIII

CONTENIDO

Funcionamiento del motor asncrono trifsico alimentado con tensiones desequili bradas . . . . . . . . . . . . . . . . . . . . . . . . . . . . . . . . . . . . . . . . . . . . . . . . . . . . . . . .. 4.15. Mquinas asncronas especiales. . . . . . . . . . . . . . . . . . . . . . . . . . . . . . . . . . . . . . .. 4.15.1. Regulador de induccin...................................... 4.15.2. Selsyns . . . . . . . . . . . . . . . . . . . . . . . . . . . . . . . . . . . . . . . . . . . . . . . . . . . .. 4.15.3. Motor de induccin lineal.. . . . . . . . . . . . . . . . . . . . . . . . . . . . . . . . . .. 4.16. Apndice: El par de rotacin de un motor asncrono desde el punto de vista " . flSICO . Prob lemas . . . . . . . . . . . . . . . . . . . . . . . . . . . . . . . . . .'. . . . . . . . . . . . . . . . . . . . . . . . . . . . . . Biografas . Referencias
111 I 11 111 111 111 111 111 111 111 111 111 11

4.14.

349 358 358 359 360 362 367 . 376 378

Captulo 5: MAQUINAS SINCRONAS


5.1. 5.2. 5.3. 5.4.
111 11 ' 111 111 111 111 111 111

"

,
111

.
111

381 381 383 386 387 387 390 394 398 398 399 402 410

5.5. 5.6.

5.7. 5.8. 5.9.

5.10. 5.11. 5.12.

Introduccin ... Aspectos constructivos. . . . . . . . . . . . . . . . . . . . . . . . . . . . . . . . . . . . . . . . . . . . . .. Sistemas de excitacin . . . . . . . . . . . . . . . . . . . . . . . . . . . . . . . . . . . . . . . . . . . . . .. Principio de funcionamiento de un alternador. . . . . . . . . . . . . . . . . . . . . . . . . . .. 5.4.1. Funcionamiento en vaco. . . . . . . . . . . . . . . . . . . . . . . . . . . . . . . . . . .. 5.4.2. Funcionamiento en carga. Reaccin de inducido. . . . . . . . . . . . . . .. Diagrama fasorial de un alternador. Regulacin de tensin. . . . . . . . . . . . . . .. Anlisis lineal de la mquina sncrona: el circuito equivalente. . . . . . . . . . . .. 5.6.1. Generalidades 5.6.2. Mtodo de Behn-Eschenburg. Impedancia sncrona ..... 5.6.3. Caracterstica de vaco y cortocircuito de la mquina sncrona. Determinacin de la impedancia sncrona. . . . . . . . . . . . . . . . . . . . . . . . . . .. Anlisis no lineal de la mquina sncrona: Mtodo de Potier o del f.d.p, nulo. Clculo de la regulacin. Regulacin de tensin en las mquinas sncronas de polos salientes. Teora de las dos reacciones Funcionamiento de un alternador en una red aislada ,. 5.9.1. Generalidades . . . . . . . . . . . . . . . . . . . . . . . . . . . . . . . . . . . . . . . . . . . . . .. 5.9.2. Funcionamiento del regulador de velocidad. . . . . . . . . . . . . . . . . . . .. Acoplamiento de un alternador a la red. . . . . . . . . . . . . . . . . . . . . . . . . . . . . . . .. Potencia activa y reactiva desarrollada por una mquina sncrona acoplada a una red de potencia infinita. . . . . . . . . . . . . . . . . . . . . . . . . . . . . . . . . . . . . . . . . .. Funcionamiento de una mquina sncrona conectada a una red de potencia infiI I I I

415 420 420 422 428


433 436 436 438 445 452 455 462 466 473 477

ni ta . . . . . . . . . . . . . . . . . . . . . . . . . . . . . . . . . . . . . . . . . . . . . . . . . . . . . . . . . . . . . . ..
5.12.1. Efectos de la variacin de excitacin. . . . . . . . . . . . . . . . . . . . . . . . . .. 5.12.2. Efectos de la variacin del par primario (regulador de velocidad) .. 5.13. Funcionamiento en paralelo de alternadores de potencias similares. . . . . . . .. 5.14. Motor sncrono: Caractersticas y aplicaciones. . . . . . . . . . . . . . . . . . . . . . . . . .. 5.15. Diagrama de lmites de funcionamiento de una mquina sncrona. . . . . . . . .. 5.16. Transitorio de cortocircuito de una mquina sncrona. . . . . . . . . . . . . . . . . . . .. Problemas Biografas. . . . . . . . . . . . . . . . . . . . . . . . . . . . . . . . . . . . . . . . . . . . . . . . . . . . . . . . . . . . . . . .. Referencias . . ... . ... . . . . . ... . ... . . . ..
411 411 .. .. .. 411 .. .. .. .. .. 411 ..

CONTENIDO

IX

Captulo 6: MQUINAS DE CORRIENTE CONTINUA

479

6.1. Introduccin............................................................ 6.2. Aspectos constructivos............................................... 6.3. Principio de funcionamiento..... . . . . . . . . . . . . . . . . . . . . . . . . . . . . . . . . . . . .. 6.4. Reaccin del inducido. . . . . . . . . . . . . . . . . . . . . . . . . . . . . . . . . . . . . . . . . . . . . . .. 6.5. Conmutacin........................................................ 6.6. Generadores de C.C.: Aspectos generales. . . . . . . . . . . . . . . . . . . . . . . . . . . . . . .. 6.7. Generadores de c .C.: Caractersticas de servicio. . . . . . . . . . . . . . . . . . . . . . . . .. 6.8. Motores de C.C.: Aspectos generales. . . . . .. . . . . . . . . . . . . . . . . . . . . . . . . . . . .. 6.9. Motores de C.C.: Caractersticas de funcionamiento. . . . . . . . . . . . . . . . . . . . . .. 6.9.1. Motores de c.c. con excitacin independiente y derivacin. Sistema Ward -Leonard . . ... . . ... . . 6.9.2. Motores de c.c. con excitacin serie. . . . . . . . . . . . . . . . . . . . . . . . . . . .. 6.9.3. Motores de c.c. con excitacin compuesta. . . . . . . . . . . . . . . . . . . . . . .. 6.10. Motor de c .C.: Mtodos de frenado. . . . . . . . . . . . . . . . . . . . . . . . . . . . . . . . . . . .. 6.11. Funcionamiento de una mquina de c.c. en cuatro cuadrantes. . . . . . . . . . . . .. 6.12. Motor monofsico de c.a. con colector de delgas : . . . . . . .. Problemas , . . . ... . ,. " ......... ... ... ... . Biografas . . . . . . . . . . . . . . . . . . . . . . . . . . . . . . . . . . . . . . . . . . . . . . . . . . . . . . . . . . . . . . . .. Referencias ,.
p " '" .. .. .. .. .. ... .. .. .. .. .. ... .. .. .. .. .. .. .. ....

479 480 483 488 496 501 504 514 517


519

.lo

..

..

"

..

..

lo

"

..

..

..

..

..

..

..

..

525 528 532 538 540 543 546 549 551

Captulo 7: ACeION AMIENTOS ELCTRICOS.

. . . . . . . . . . . . . . . . . . . . . . . . . . . ..
.. .. .. .. .. .. .. .. .. .. .. .. .. .. .. .. .. .. .. ..

7 .1. 7.2.

7.3. 7.4.

7.5.

Introduccin " "... Dispositivos semiconductores de potencia . 7. 2.1. Introduccin................................................ 7.2.2. Diodos rectificadores . 7.2.3. Tiristor o rectificador controlado . 7.2.4. Tiristor de apagado por puerta (GTO) . 7.2.5. Transistor bipolar de unin (BJT) . 7.2.6. Transistor MOSFET . 7.2.7. Transistor bipolar de puerta aislada IGBT . 7.2.8. Tiristor MCT . 7.2.9. Funcionamiento ideal de los dispositivos semiconductores . Convertidores electrnicos de potencia . 7.3.1. Introduccin . Rectificadores " " . 7 .4.1. Rectificador monofsico media onda con carga resistiva . 7.4.2. Rectificador monofsico media onda (onda completa) con carga inductiva , 7.4.3. Rectificador monofsico de doble onda con carga resistiva . 7.4.4. Rectificador monfsico de doble onda (onda completa) con carga inductiva , 7.4.5. Rectificadores trifsicos...................................... Rectificadores controlados............................................ 7.5 .1. Introduccin................................................ 7.5.2. Principio de funcionamiento de un rectificador con control de fase.
t .. ..
111

551 552
552 555 557

560 561 562 563


564

564 565
565
568

568

572 578
582 585 592 592 594

If

..

CONTENIDO

7.6.

7.7.

7.8.

Convertidor monofsico en puente completo con carga inductiva. .. Convertidor trifsico en puente completo. . . . . . . . . . . . . . . . . . . . . .. Reguladores de corriente alterna. . . . . . . . . . . . . . . . . . . . . . . . . . . . . . . . . . . . . .. 7.6.1. Introduccin :.. 7.6 .. 2. Regulador con control de fase y carga resistiva. . . . . . . . . . . . . . . . .. 7.6.3. Regulador con control de fase y carga inductiva. . . . . . . . . . . . . . . .. 7.6.4. Regulador on-off............................................. Convertidores c.c. a c.c. (choppers o recortadores) . . . . . . . . . . . . . . . . . . . . . .. 7.7.1. Chopper directo o reductor de tensin . . . . . . . . . . . . . . . . . . . . . . . . .. 7.7.2. Chopper inverso o elevador de tensin. . . . . . . . . . . . . . . . . . . . . . . .. 7.7.3. Choppers de dos y cuatro cuadrantes. . . . . . . . . . . . . . . . . . . . . . . . . .. Convertidores c.c. a c.a. (onduladores o inversores) ... " . . . . . . . . . . . . . . . . .. 7.8.1. Inversores monofsicos. . . . . . . . . . . . . . . . . . . . . . . . . . . . . . . . . . . . . .. .7.8.2. Inversor trifsico en puente . . . . . . . . . . . . . . . . . . . . . . . . . . . . J . . . . ..

7.5.3. 7.5.4.

595
604

613 613 613 614 616 617 618

622
623 626 626 628

7.9. 7.10.

7.8.3. Control de la tensin de salida en un inversor. . . . . . . . . . . . . . . . . .. Convertidores c.a. a c.a. (cicloconvertidores) . . . . . . . . . . . . . . . . . . . . . . . . . . .. Accionamientos elctricos............................................ 7.10.1. Generalidades...............................................
7.10.2. Funcionamiento en cuatro cuadrantes. . . . . . . . . . . . . . . . . . . . . . . . . .. 7. 10.3. Dinmica de la combinacin motor-carga. Estabilidad . . . . . . . . . . .. Accionamientos elctricos con motores de c.c. . . . . . . . . . . . . . . . . . . . . . . . . . .. 7.11.1. Generalidades............................................... 7.11.2. Regulacin de la velocidad de motores de c.C. por medio de rectificadores controlados. . . . . . . . . . . . . . . . . . . . . . . . . . . . . . . . . . . . . . . . . . .. 7.11.3. Regulacin de la velocidad de motores de c.c. por medio de choppers. 7.11.4. Regulacin de motores de c.c. mediante realimentacin. . . . . . . . . .. Accionamientos elctricos con motores de c.a. asncronos. . . . . . . . . . . . . . . ..

629 636' 637


637

639
641 644

7.11.

644
646 652

7.12.

7.12.1.
7.12.2. 7.12.3. 7.12.4.

Introduccin......................... esttor ..... ....... . . . .


ti .. .. ..

. . . . . . . . . . . . . . . . . . . . . ..
... .. .. .. .. .. .. .. .. .. .. ....

655 658 658


660 661 673 676 681 692 695 696

Regulacin de velocidad por control de la tensin de lnea aplicada al Regulacin de velocidad por control de la tensin y frecuencia de lnea. Control escalar . . . . . . . . . . . . . . . . . . . . . . . . . . . . . . . . . . . . . . . .. Regulacin de velocidad por control esttico de una resistencia adicional en el rotor. . . . . . . . . . . . . . . . . . . . . . . . . . . . . . . . . . . . . . . . . . .. Regulacin de velocidad por recuperacin de la potencia de desliza-

7.12.5.
7.12.6.
7.13.

miento.

11

11

..

..

..

..

..

Control vectorial de motores asncronos. . . . . . . . . . . . . . . . . . . . . . .. Accionamientos elctricos con motores de c.a. sncronos. . . . . . . . . . . . . . . . .. 7.13.1. Regulacin de velocidad de motores sncronos en lazo abierto. . . .. 7.13.2. Regulacin de velocidad de motores sncronos en lazo cerrado. Motor sncrono autopilotado........................................
11 " 6. 11 .. .. ..

Problemas
Biografas

700
705
...

~. . . . . ..
11 .. .. .. .. .. 11 .. 11 .. .. .. .. .. .. .. .. .. .. 11 .. .. .. .. .. ..

Referencias ... . .

708
.
711 711

Apndice 1: MAQUINAS ELECTRICAS: ASPECTOS HISTORICOS


l. Los orgenes . . . . . . . . . . . . ... . . . . . . . . . . . . . . . . . . . . . . . . . . . . . . . . . . . . . . . . . . . ..

~,

CONTENIDO

XI

2. Generadores 2. l. Generadores de c.c. o dinamos 2.2. Generadores de c.a. (alternadores) 3. Motores 3.1. Motores de c.c " 3.2. Motores asncronos o de induccin 3.3. M otores sncronos................................................ 3.4. Motores especiales de c.a 3.5. Motores. especiales de C.c. y otros motores 4. Transformadores . 5. Desarrollos tecnolgicos en la construccin de mquinas elctricas 6. Las mquinas elctricas y la electrnica de potencia 6.1. Desarrollo de componentes electrnicos 6.2. Control electrnico de mquinas elctricas 6.2.1. Regulacin de velocidad de motores de c.c 6.2.2. Regulacin de velocidad de motores de c.a Referencias . . . ..... . . . ... . . . . . . . . . . . . . . . . . . . . . . . . . . . . . . . . . . . . . .
I I ...... " ...

ti

"

..

..

..

..

..

..

..

..

..

712 . 712
..

.
I I .. .. .. ,

715

l'

..

"

717

l'

..

..

..

..

..

..

..

..

..

..

..

..

..

..

o.

l'

"

"

717 . 718 720 . 720 . 721 722 . 724


725

727 . 727 . 728


.

. .

725

730

Apndice 2: REPASO DE SERIES DE FOURIER. . . . . . . . . . . . . . . . . . . . . . . . . . . .. 1. Introduccin........................................................... 2. Funcin peridica...................................................... 3. Series trigonomtricas de Fourier . . . . . . . . . . . . . . . . . . . . . . . . . . . . . . . . . . . . . . . .. 4. Ortogonalidad del sistemas trigonomtrico. . . . . . . . . . . . . . . . . . . . . . . . . . . . . . . .. 5. Evaluacin de los coeficientes de Fourier . . . . . . . . . . . . . . . . . . . . . . . . . . . . . . . . .. 6. Simetra de la funcin f(t) . . . . . . . . . . . . . . . . . . . . . . . . . . . . . . . . . . . . . . . . . . . . . .. 7. Coeficientes de Fourier de ondas simtricas. . . . . . . . . . . . . . . . . . . . . . . . . . . . . . ..

733 733 733 734 735 735 736 737

Apndice 3: EL SISTEMA POR UNIDAD. . . . . . . . . . . . . . . . . . . . . . . . . . . . . . . . . . .. Magnitudes normalizadas. El sistema por unidad. . . . . . . . . . . . . . . . . . . . . . . . . .. 2. Cambios de base. . . . . . . . . . . . . . . . . . . . . . . . . . . . . . . . . . . . . . . . . . . . . . . . . . . . . .. 3. Sistemas trifsicos. Anlisis por unidad. . . . . . . . . . . . . . . . . . . . . . . . . . . . . . . . . ..
l.
~

743 743 746 749


753 757

Indice alfa btico . . . . . . . . . . . . . . . . . . . . . . . . . . . . . . . . . . . . . . . . . . . . . . . . . . . . . . . . ..


,

Indice bogrco . . . . . . . . . . . . . . . . . . . . . . . . . . . . . . . . . . . . . . . . . . . . . . . . . . . . . . . . ..

"

Acerca del autor

Natural de Ayerbe (Huesca), Perito Industrial, Rama Elctrica, por la Escuela Tcnica de Peritos Industriales de Zaragoza, 1965 (en la actualidad: E.U. de Ingeniera Tcnica Industrial). Ingeniero de Telecomunicacin, Rama Electrnica, por la E.T.S. de Ingenieros de Telecomunicacin de Madrid, 1970. Doctor Ingeniero de Telecomunicacin por la Universidad Politcnica de Madrid, 1974. Licenciado en Ciencias, Seccin de Fsicas, por la Universidad Complutense de Madrid, 1976. Maestro de Laboratorio de Electrotecnia de la E.T.S. de Ingenieros de Telecomunicacin de Madrid, 1967-71. Profesor Encargado de Curso y de Clases Prcticas de Electrotecnia en la Escuela anterior, 1970-72. Protesor Encargado de Laboratorio de Electrotecnia de la E.T.S. de Ingenieros de Telecomunicacin de Madrid, 1972-74. Profesor Adjunto de Laboratorio de Electrotecnia en la misma Escuela, 1974-75. Catedrtico de Electrotecnia de la Escuela Universitaria de Ingeniera Tcnica de Obras Pblicas de Madrid, 1972-78. Profesor Adjunto de Mquinas Elctricas de la E.T.S. de Ingenieros Industriales de Madrid, 1975-78. Catedrtico de Electrotecnia de la E.T.S. de Ingenieros de Caminos, Canales y Puertos de la Universidad de Santander, 1978-80. Catedrtico de Electrotecnia de la E.T.S. de Ingenieros de Caminos, Canales y Puertos de la Universidad Politcnica de Madrid, desde 1980, continuando en la actualidad. Profesor Encargado de la asignatura Instrumentacin y Control de la Carrera de 2. Ciclo de Ingeniero de Materiales de la UPM durante los Cursos 1995 a 1998. Director del Departamento de Energtica de la Universidad de Santander, 1978-80. Secretario General de la Universidad de Santander, 1979-80. Secretario de la E.T.S. de Ingenieros de Caminos, Canales y Puertos de Madrid, 1981-82, Subdirector de Investigacin y Doctorado de la E.T.S. de Ingenieros de Caminos de Madrid, 1983-86. Member del IEEE (Institute 01 Electrical and Electronic Engineersi desde 1972, recibiendo el grado de Senior Member en 1985. Director del Departamento de Ingeniera Civil: Hidrulica y Energtica de la UPM, 1994-2003. Premio de la Fundacin General de la Universidad Politcnica de Madrid a la labor docente desarrollada por un profesor en su vida acadmica, ao 1991. '" Autor de diversos libros de texto y artculos en el Area de Ingeniera Elctrica. Premio de la Fundacin General de la Universidad Politcnica de Madrid al mejor libro de texto escrito por un profesor de la UPM por el libro Electromagnetismo y Circuitos Elctricos, ao 1993. Ha impartido gran nmero de Seminarios y Cursos de Doctorado en diversas Universidades espaolas. Tambin ha dirigido o participado en numerosos Cursos de Formacin y de Reciclado para diversas Empresas e Instituciones. La labor investigadora desarrollada incluye los temas de estabilidad de sistemas elctricos de potencia; comportamiento transitorio de mquinas sncronas; regulacin electrnica de velocidad de motores de induccin trifsicos; sistemas de almacenamiento de energa elctrica mediante bobinas superconductoras SMES y tambin sobre historia de la ingeniera elctrica.
XIII

La excelente acogida que se ha venido dispensando a las cuatro primeras ediciones de este libro de Mquinas Elctricas, nos ha animado a realizar esta quinta edicin bajo los auspicios de la editorial McGraw-HilVInteramericana de Espaa. El texto trata d, los principios y aplicaciones de las mquinas elctricas que todo ingeniero, cualquiera que sea su especialidad de origen, emplear a lo largo de toda su vida profesional, merced a su aplicacin en las diferentes fases de los procesos productivos. El libro es el fruto de ms de treinta aos de experiencia en la enseanza de las mquinas elctricas dentro de los Cursos de Electrotecnia impartidos por el autor en diferentes Escuelas Tcnicas Espaolas de Ingeniera Superior y de Ingeniera Tcnica. En la actualidad, gran parte del texto es utilizado por el autor para explicar la segunda parte de la asignatura Electricidad y Electrotecnia en la E.T.S. de Ingenieros de Caminos, Canales y Puertos de la Universidad Politcnica de Madrid. El Captulo 1 se refiere a los circuitos magnticos y a los principios bsicos de la conversin de energa en el que se explican las leyes de los circuitos magnticos, las prdidas en los materiales ferromagnticos y tambin se deducen las expresiones de las fuerzas en sistemas de traslacin, y las del par en sistemas de rotacin. Se incluyen ejemplos de aplicacin en los que se calculan los pares de diversas mquinas elctricas desde un punto de vista unificado. El Captulo 2 se dedica a los principios generales de las mquinas elctricas. Se explican los conceptos bsicos comunes a las mquinas tales como: esttor*, rotor, inductor, inducido, tipos de colectores, devanados, etc. Se analizan las f.m.m.s. producidas por diferentes configuraciones de devanados, haciendo especial hincapi en el teorema de Ferraris que constituye el principio de funcionamiento de las mquinas de c.a. Se estudia de una forma general la generacin de f.e.m. en una mquina elctrica, explicando tambin la creacin del par electromagntico desde el punto de vista de interaccin de las f.m.m.s. del esttor y rotor. El captulo finaliza con un anlisis cualitativo de las principales mquinas elctricas, de tal modo que el estudiante pueda identificarlas con facilidad y conozca desde el principio las mquinas existentes y su funcionamiento bsico . El Captulo 3 se refiere a los transformadores. Se detallan los aspectos constructivos y el principio de funcionamiento del transformador ideal. A continuacin se sealan los efectos reales que tienen lugar en esta mquina y se desarrollan los circuitos equivalentes del transformador de potencia, junto con los ensayos que se precisan para determinar los parmetros que intervienen en el circuito final. Este esquema organizativo es el que se sigue en el libro para estudiar el funcionamiento de todas las mquinas elctricas.

En la ltima edicin del ao 2001 del Diccionario de la Real Academia Espaola de la Lengua, se ha incluido por fin este trmino y con acento ortografico o tilde. Se hace de este modo justicia a una palabra pronunciada de este modo desde hace ms de 100 aos por los ingenieros elctricos y que se consideraba por los eruditos como un argot.

!
1

, ,
I ,
, ,

xvi

PRLOGO

El Captulo 4 est dedicado a las mquinas asncronas o de induccin. Fundamentalmente el tema se refiere a los motores trifsicos, en el que se explican el principio de funcionamiento, circuito equivalente y ensayos necesarios para la determinacin del mismo. Se estudia el balance de potencias en estas mquinas y se determina la ecuacin general del par de rotacin. Se analiza ms tarde el proceso de arranque de los motores trifsicos y los diversos mtodos existentes para lograrlo. Se explica tambin el motor de induccin monofsico y se describen algunas mquinas asncronas especiales como: el regulador de induccin, los ejes elctricos y el motor de induccin lineal. El Captulo 5 se refiere a las mquinas sncronas, que en su funcionamiento como generador es la mquina principal por antonomasia en las centrales elctricas. Se explica el funcionamiento del alternador en vaco y en carga, destacando el fenmeno de la reaccin del inducido y la obtencin del circuito equivalente. Se estudia el rgimen de funcionamiento de un alternador tanto en una red aislada como conectado a una red de potencia infinita. Se analiza el motor sncrono y tambin el diagrama de lmites de funcionamiento de una mqui" na smcrona, El Captulo 6 se dedica a las mquinas de c.c.; se inicia el tema explicando los aspectos constructivos y el principio de funcionamiento que incluye el fenmeno de la reaccin de inducido y la conmutacin en estas mquinas. Se estudia el funcionamiento de la mquina en rgimen generador y motor; en este caso se explica con detalle la regulacin de velocidad que ofrecen, haciendo una referencia especial al sistema de regulacin Ward-Leonard. Tambin se explican los mtodos de frenado de los motores de e.e. por procedimientos clsicos y el funcionamiento de la .mquina de c.c. en cuatro cuadrantes. El Captulo 7 se dedica a los accionamientos elctricos cuya importancia es cada vez mayor en la industria moderna. Es una materia interdisciplinar entre el mundo de la electrnica y el de la electrotecnia. Se explica de una forma sencilla el principio de funcionamiento de los semiconductores, describiendo los diversos tipos de convertidores electrnicos utilizados en la regulacin de las mquinas elctricas. Se describen luego los accionamientos elctricos tanto con motores de c.c. como con motores de c.a. En ambos casos se incluyen notas informativas de su aplicacin especfica a la traccin elctrica espaola, por lo que ser de gran inters para aquellos ingenieros que desarrollan su labor en los talleres de mantenimiento de ferrocarriles elctricos y trenes metropolitanos. Se han incluido en el1ibro tres apndices; el primero explica el desarrollo histrico de las mquinas elctricas. En un principio el autor quera incorporar este apndice como captulo 0, a modo de introduccin del libro, pero se consider ms tarde que era demasiado especializado para empezar el texto y por ello se ha dejado en este lugar. Esperamos fervientemente que este apndice sea de utilidad para los profesores de ingeniera elctrica. Es opinin del autor, que los profesores de Universidad estamos obligados a dar una formacin ms humanstica a los alumnos y que debemos ensearles cmo ha ido progresando la ciencia y la tecnologa, y el porqu se ha avanzado en una direccin y no en la otra, porque esto ayuda a conocer las relaciones entre las diferentes ramas cientficas. A guisa de ejemplo, podemos indicar que el desarrollo de la electrnica no hubiera sido posible sin un adelanto suficiente de la electrotecnia, y de un modo anlogo, el progreso de los ordenadores nicamente se puede justificar por un avance espectacular de la electrnica. En el Apndice 2 del libro se hace un repaso de las series de Fourier, cuyo conocimiento es necesario para estudiar los armnicos de campo en el entrehierro de las mquinas elctricas y tambin los armnicos de tensin producidos por los convertidores electrnicos utilizados en los diversos accionamientos elctricos. En el Apndice 3 se estudia el sistema por unidad que representa una normalizacin que se emplea frecuentemente en el anlisis de sistemas

PRLOGO

XVII

elctricos de potencia y que en el caso de las mquinas elctricas constituye un mtodo muy til para comparar los parmetros de mquinas de diferentes potencias nominales. En cada captulo del libro se han incluido una gran variedad de ejemplos de aplicacin con su solucin completa, facilitando por una parte la labor del profesor que lo utilice en su asignatura, ya que podr dedicar menos tiempo a la tediosa manipulacin numrica y ms a las deducciones bsicas. Por otra parte, estos ejercicios facilitan el autoaprendizaje del alumno, ya que cada nuevo concepto que se introduce, va seguido de unos ejemplos de aplicacin que le servirn para comprender mejor la teora presentada, lo qlle permite afianzar las ideas de un modo progresivo sin dejar lagunas en la interpretacin de los conceptos implicados. Al final de cada captulo se han incluido entre veinte y treinta problemas en los que se da nicamente la respuesta final. Con ello se pretende ayudar al profesor en la bsqueda de nuevos problemas para realizar en clase y facilitar el trabajo del alumno, para que pueda comprobar su propio progreso y nivel de conocimientos. Teniendo en cuenta los ejemplos resueltos a lo largo de cada captulo y los problemas finales, el libro contiene cerca de trescientos problemas que facilitan el aprendizaje de esta materia. En cada captulo se incluye una amplia bibliografa de ampliacin de los temas estudiados en la leccin, que puede ser til para aquellos estudiantes que deseen una mayor profundizacin de la teora. Representan la mayor parte de las referencias que ha utilizado el autor en la redaccin del libro. Tambin se incluyen biografas de cientficos, ingenieros y profesores que han contribuido directa o indirectamente al desarrollo de la ingeniera elctrica. Ha sido una tarea bastante ardua encontrar algunas de las biografas, pero se ha hecho un gran esfuerzo para que el estudiante conozca los protagonistas de la historia elctrica, a quienes con esta semblanza el autor rinde su homenaje. La lectura de estas biograffas suavizar la lec-tura del libro y ayudar al alumno a conocer las aportaciones ms importantes que realizaron estos ingenieros, a los que la humanidad les debe respeto y gratitud.

Agradecimientos

Deseo hacer patente mi agradecimiento a aquellas personas que en algn momento de mi vida causaron un gran impacto en mi formacin. En primer lugar los recuerdos se dirigen a mis padres: mis primeros y mejores maestros, su ejemplo de dedicacin y esfuerzo han sido para m la gua de mi vida. A los profesores que supieron inculcarme la pasin por esta asignatura: D. Valentn Abada LaJana (E.U. de Ingeniera Tcnica Industrial de Zaragoza); D. Guillerl110 Herranz Acero y D. Miguel Aguilar Fernndez (E.T.S. de Ingenieros de Telecomunicacin de Madrid); D. ngel Alonso Rodrguez (E.T.S. de Ingenieros Industriales de Madrid). Quisiera tambin agradecer a mis compaeros y colaboradores, en los Centros donde he impartido docencia, por la ayuda prestada en cada momento y por la amistad que nos ha unido desde entonces. En orden cronolgico me refiero a: D. Luis Santamara Gago, D. Eugenio Bertoln Gmez y D. Jos Mara Prez Martnez, de la E.U. de Ingeniera Tcnica de Obras Pblicas de Madrid. A D. Javier Sanz Feito, D. Luis Serrano Iribarnegaray, D. Angel Molina Martn Urda y D. Manuel Femndez Florez de la E.T.S. de Ingenieros Industriales de Madrid, los dos primeros destinados actualmente como Catedrticos de Ingeniera Elctrica en las Universidades Carlos 111 de Madrid y Politcnica de Valencia respectivamente. A D. Jos Antonio Gurrutxaga Ruiz de la E.T.S. de Ingenieros de Caminos, Canales y Puertos de Santander. A los profesores: D. Jos Romn Wilhelmi Ayza, D. Pedro Garca Gutirrez, D. Antonio Ruiz Mateo, D. Jos Angel Snchez Fernndez, D. Luis Arvalo Muoz, D. Jess Fraile Ardanuy, Da. Nieves Herrero Martnez, D. Joaqun de Gregorio Saavedra y al personal del Laboratorio de Electrotecnia: D. Enrique Amau Lzaro, D. Carmelo Hernnz Benndez y Da. Cristina Gordillo Iracheta, todos ellos pertenecientes a la E.T.S. de Ingenieros de Caminos, Canales y Puertos de Madrid. Aprovecho la oportunidad para dar las gracias colectivamente a muchos profesores de Ingeniera Elctrica de la Universidad espaola por las muy tiles sugerencias recibidas, demasiado numerosas para agradecerlas individualmente, aunque no por ello menos apreciadas. Es enorme tambin la deuda de gratitud a mis alumnos, por ayudarme con sus preguntas a buscar mejores ideas para escribir mejor y hacer ms asequible la transmisin de informacin. A ellos va dirigido especialmente este libro, para que encuentren menos dificultades en el estudio de la asignatura. Representan el pblico de ese gran teatro que son las aulas, donde el actor desea ser escuchado con inters y cario. Su actitud favorable y respetuosa en todo momento me proporcionaron el impulso suficiente para llevar a cabo esta ardua tarea. Deseo agradecer de un modo especial a mi esposa e hijos la paciencia mostrada durante la preparacin del libro y por comprender mi vocacin docente, por su aliento y comprensin y a quienes esta obra les ha quitado muchsimas horas de convivencia. En particular, quiero agradecer a mi hijo y colaborador Jess Fraile Ardanuy por sus sugerencias y crticas en el tratamiento de algunas partes del texto y por su gran ayuda en la correccin de erratas. Esta quinta edicin se publica bajo el patrocinio de la editorial Mcflraw-Hilf/Interamericana de Espaa y se debe a la perseverancia de la editora universitaria Concepcin Femndez que ha dedicado un gran esfuerzo y entusiasmo para que la obra se confeccionara en un tiempo rcord y con una excelente presentacin.
l' l'

XIX

CAPTULO

Circuitos magnticos y conversin de energa

1.1. INTRODUCCIN
En los circuitos elctricos, la conexin existente entre los elementos pasivos se realiza por medio de materiales conductores que obligan a que la corriente elctrica siga determinados recorridos, obedeciendo las leyes de Kirchhoff. Cuando se trata de estudiar las mquinas elctricas, electroimanes y otros dispositivos electromagnticos, se plantea un problema similar de canalizar y concentrar altas densidades de flujo magntico, en las regiones donde se necesita, lo cual se logra por medio de materiales ferromagnticos. Un circuito magntico est formado generalmente por una estructura de hierro, sobre la que se arrollan una o ms bobinas por las que circulan corrientes, que dan lugar a los flujos que aparecen en el sistema. El clculo riguroso de los flujos producidos es generalmente muy difcil y para una determinacin precisa sera necesario emplear correctamente las ecuaciones de Maxwell y la ayuda de un calculador de tipo analgico o digital (ordenador); sin embargo, las reglas de los circuitos magnticos que se estudian en este captulo permiten resolver el problema de una forma aproximada y la mayor parte de las veces suficiente para las aplicaciones que se dan en la Electrotecnia. El comportamiento de un circuito magntico viene determinado fundamentalmente por el carcter solenoidal de las lneas de induccin magntica (div B = O)y por el hecho de que en los materiales ferromagnticos la permeabilidad es elevada y muy superior a la del vaco (11 110)' Estas condiciones corresponden, en el caso de circuitos elctricos, a la consideracin de que la densidad de corriente J es solenoidal, es decir, div J = O(lo cual es siempre cierto, en todos aquellos puntos en donde no existan almacenamientos de carga), ya que la conductividad (J de un conductor es muy elevada frente a la de un aislador o dielctrico. Esta similitud hace que puedan aplicarse a los circuitos magnticos todos los teoremas de redes analizados en un curso de teora de circuitos elctricos, aunque la resolucin es algo ms compleja, debido al carcter no lineal del ncleo ferromagntico. En este captulo se estudian los diferentes tipos de materiales magnticos, haciendo especial hincapi en los ferromagnticos por su importancia como elemento estructural bsico de las mquinas elctricas. Se analizan las leyes de los circuitos magnticos y se explican las analogas con los circuitos elctricos, definiendo los conceptos de reluctancia y permeancia como conceptos anlogos a la resistencia y conductancia. Se desarrollan las expresiones de la energa y coenerga magntica y ms tarde se calculan las expresiones de las prdidas en el hierro: por histresis y por corrientes de Foucault, dando a continuacin una serie de ideas constructivas sobre las chapas magnticas y su composicin qumica. Se explican tambin los 1

MQUINAS ELCTRICAS

circuitos magnticos excitados con bobinas alimentadas por c.a. y se comparan los fenmenos que tienen lugar con los que se presentan en las bobinas con ncleo de hierro alimentadas con c.c., se desarrolla el circuito equivalente de una bobina con ncleo de hierro y se determina la forma de onda de la corriente de excitacin. El captulo finaliza analizando la conversin de energa en sistemas magnticos dotados de movimiento de traslacin, dando detalles del funcionamiento de electroimanes, rels y contactores y su importancia en los automatismos elctricos. Tambin se estudian los fenmenos de conversin de energa en sistemas de rotacin, lo que permite una introduccin a los motores de reluctancia y a las mquinas elctricas, comprendiendo la generacin de par en las mismas y los fenmenos energticos puestos en juego.

1.2. MATERIALES MAGNTICOS


Como ya se conoce de un curso bsico de electromagnetismo, las propiedades magnticas macroscpicasde un material lineal, homogneo e istropo se definen en funcin del valor de la susceptibilidad magntica Xm' que es un coeficiente adimensional que expresa la proporcionalidad entre la magnetizacin o imanacin M y la intensidad del campo magntico H de acuerdo con la ecuacin:
M=XmH [Nm] (1.1)

como quiera adems que la induccin magntica B est relacionada con los campos H y M por: B = J.10 (H + M) teniendo en cuenta (1.1) resulta:
B = J.10 (H + Xm H)

[Teslas]

(1.2)

= J.10(l

+ Xm) H = J.10 J.1,. H = J.1H

(1.3)

donde J.1 representa la permeabilidad magntica del medio (J.1 = /-Lo J.1r) Y J.1r la permeabilidad relativa, que a su vez es igual a 1 + X",; J.10 es la permeabilidad del vaco y que en unidades SI vale 4n 10-7 Hlm. De acuerdo con el valor de J.1r' los materiales se clasifican en: Diamagnticos: si J.1r ;:::::: 1 Paramagnticos: si J.1r ;:::::: 1 Ferromagnticos: si J.1r 1
(Xm es del orden de _10-5) (X", es del orden de

+ l0-3)

(Xm tiene un valor elevado)

Para comprender el comportamiento magntico microscpico de un material es preciso recurrir a la mecnica cuntica. Sin embargo, se puede dar una descripcin cualitativa de los fenmenos magnticos en base al modelo atmico clsico de Bohr-Sommerfeld. De acuerdo con este modelo podemos suponer que el tomo est formado por un ncleo central fijo que contiene protones y neutrones alrededor del cual giran los electrones describiendo rbitas cerradas que pueden considerarse como circuitos elctricos. Cada uno de estos circuitos origina un momento magntico dipolar m (que es el producto de la corriente por la superficie de espira del circuito), que va asociado a un momento angular L o momento de la cantidad de movimiento (L = mrico, siendo m la masa del electrn, r el radio de su rbita y w la velocidad angular de giro). Hay que tener en cuenta tambin que el electrn gira sobre s mismo (espn del electrn), lo que da lugar a un mayor momento angular y a un momento magntico dipolar adicional que se incorpora al tomo. Al efecto anterior se denomina interaccin espn-orbital

CAPTULO

l.

CIRCUITOS

MAGNTICOS

Y CONVERSIN

DE ENERGA

(o enlace L-S), gracias a la cual el momento orbital de los electrones se enlaza con su momento magntico de espn, formando el momento magntico total del tomo.

1.2.1. Diamagnetismo
En un material diamagntico, el momento magntico neto debido a los movimientos orbitales de los electrones y a sus espines en cualquier tomo particular es cero en ausencia de campo magntico externo. Al aplicar un campo exterior de induccin B, aparecer una fuerza sobre los electrones orbitales de acuerdo con la frmula de Lorentz:
Fm=q(uxB)

[NewtonJ

(l.4)

donde q es la carga del electrn y u la velocidad de los mismos. La fuerza provoca un cambio en la velocidad angular de los electrones (hay que tener en cuenta que la fuerza centrpeta que surge como consecuencia del movimiento del electrn alrededor del ncleo es muy superior a las fuerzas que actan sobre el electrn por parte de los campos exteriores, y por este motivo los radios de las rbitas no varan al colocar el tomo en un campo exterior y solamente se modifica la velocidad angular de los electrones). El cambio en esta velocidad se denomina frecuencia de Larmor. Como consecuencia del cambio en la velocidad, se modifica el valor de la corriente electrnica equivalente, lo que da lugar a la creacin de un momento magntico neto. En definitiva, ste es un proceso de imanacin inducida, que de acuerdo con la ley de Faraday-Lenz representa un momento magntico inducido que se opone siempre al campo aplicado,reduciendo de este modo el valor de la induccin. El efecto macroscpico del proceso es equivalente a una imanacin negativa que se puede describir por medio de una susceptibilidad magntica Xm negativa del orden de _10-5. El bismuto, el cobre, plomo, plata y oro presentan estos efectos. El diamagnetismo se debe principalmente al movimiento orbital de los electrones dentro de un tomo y est presente en todos los materiales. En la mayora de ellos el efecto es muy dbil y es por lo que a veces este fenmeno queda enmascarado por otros ms fuertes, como as ocurre en los materiales paramagnticosy ferromagnticos,que se estudiarn ms adelante. Los materiales diamagnticos no presentan magnetismo remanente, lo que significa que el momento magntico inducido desaparece cuando se anula el campo exterior aplicado. El valor de Xm en los materiales diamagnticos es independiente de la temperatura, y este fenmeno, que fue descubierto experimentalmenteen 1895por Pierre Curie,justifica el hecho de que el movimiento de Larmor de los electrones se establece muy pronto y tanto el movimiento trmico como las colisiones entre tomos no modifican la frecuencia de Larmor.

1.2.2. Paramagnetismo
En algunos materiales, los momentos magnticos debidos a los movimientos de los electrones, orbital y de espn, no se cancelan completamente y los tomos y molculas tienen un momento magntico neto. Al aplicar un campo magntico externo, adems de producirse un efecto diamagntico dbil, el campo tiende a alinear los momentos magnticos moleculares en el sentido del mismo, lo que provoca un aumento de induccin. El efecto macroscpico es entonces equivalente a una imanacin positiva, es decir, a una susceptibilidadmagntica positiva. El proceso de alineamiento es considerablementecontrarrestadopor las vibraciones trmicas aleatorias del material. Hay poca interaccin coherente entre tomos y por ello el aumento de la induccin es bastante reducido, siendo Xm del orden de 1O~3. Los materiales que presentan este comportamiento se denominan paramagnticos, destacando entre ellos: aluminio, magnesio, titanio y wolframio.El paramagnetismose produce fundamentalmentepor los momentosdipolares mag-

MQUINAS ELCTRICAS

nticos de los espines de los electrones. Las fuerzas de alineamiento del campo actuando sobre los dipolos moleculares son contrarrestadas por la distorsin que produce la agitacin trmica. Al contrario que el diamagnetismo,que es independiente de la temperatura,el efecto paramagntico s que depende de ella, siendo ms fuerte a bajas temperaturas, cuando hay menos agitacin trmica. La susceptibilidad paramagntica sigue la ley de Curie:
(1.5)

en la que e es una constante y T la temperatura absoluta. A la temperatura ambiente el valor anterior es, como se ha mencionado antes, del orden de 10-3, es decir, del orden de cien veces la susceptibilidad diamagntica. Esto significa que en las sustancias paramagnticas se puede prescindir del efecto diamagntico debido a su bajo valor.

1.2.3. Ferromagnetismo y ciclo de histresis


El tipo ms importante de magnetismo (en cuanto a sus aplicaciones tecnlogicas se refiere) lo presentan los materiales ferromagnticos. Reciben esta denominacin aquellas sustancias que tienen imanacionesgrandes aun en presencia de campos magnticos muy dbiles. A la temperatura ambiente y por encima de ella * slo tres elementos, hierro, cobalto y nquel, son ferromagnticos (tambin lo son los elementos de las tierras raras: gadolnio y dysprosio). Casi todas las aleaciones y compuestos ferromagnticos contienen uno o ms de estos tres elementos o de manganeso, que pertenecen al mismo grupo de elementos de transicin en la tabla peridica. (Debe destacarse, sin embargo, que los aceros inoxidables con 18 por 100 de cromo y 8 por
100 de nquel, as como el acero al manganeso, con 14 por 100 de Mn, no son ferromagnticos). La facilidad de imanacin de estas sustancias procede de las fuerzas mecnico-cunti-

cas, que tienden a alinear paralelamente entre s a los espines atmicos prximos, aun en ausencia de un campo magntico aplicado (estas fuerzas de intercambio que alinean los espines adyacentes se conocen como interaccin espn-espn y dependen crticamente de la distancia entre los tomos). La citada alineacin no se produce en todo el volumen del material, sino que se encuentra por zonas, denominadas dominios magnticos, los cuales pueden tener volmenes comprendidos entre 10-6 y 10-2 crrr', conteniendo entre 109 y 1015 tomos. La razn por la cual los materiales ferromagnticos forman dominios es compleja; la capa electrnica 3d del tomo est parcialmente completa, como se muestra en la Figura 1.1 para el caso del hierro (existen cuatro espines no apareados), lo cual es una condicin para que exista paramagnetismo. Sin embargo, la separacin entre los tomos de los materiales ferromagnticos es tal que las fuerzas de intercambio cunticas que se producen hacen que los espines de los electrones de estos tomos se alineen paralelamente (interaccin positiva). Cuando una muestra de material ferromagntico se coloca dentro de un campo magntico, los dominios tienden a alinearse, de tal forma que sus campos magnticos se suman al campo externo, resultando un campo total ms fuerte. Este efecto puede observarse por medio de la curva de la Figura 1.2, que relaciona la induccin B resultante en funcin de la intensidad de campo magntico H.

* El ferromagnetismo es una propiedad que depende de la temperatura, y para cada material ferromagntico existe un valor, denominado temperatura de Curie, por encima del cual el material se hace paramagntico. Este fenmeno ocurre cuando el movimiento trmico es suficientemente grande para vencer las fuerzas de alineacin. Para el hierro, la temperatura de Curie es de 770 C.

CAPTULO l.

CIRCUITOS MAGNTICOS y CONVERSIN DE ENERGA

K n=1

M 3

N 4

\\ \\\ \
I 3 I 3 5 1 3 13 I

+- Espines +

'+"

+- Espines - ~

)) ))) )
Is 2s 2p 3s 3p 3d 4s

Figura 1.1. Estructura atmica del hierro.

Inicialmente, la muestra se encuentra en un estado magnticamente neutro, debido a que los dominios tienen alineaciones orientadas al azar, resultando un momento magntico total nulo. Al aplicar una intensidad de campo magntico (o excitacin magntica) definida por un HM (Fig. 1.2) de pequeo valor, se produce un desplazamiento de las paredes que separan los dominios, ensanchando stos, a costa de los que estn orientados menos favorablemente, los cuales se contraen. Este crecimiento es reversible, y si se elimina el campo HA exterior, la densidad de flujo tambin desaparece. Si se va elevando el valor de H, los dominios continan aumentando de volumen, a la par que van producindose rotaciones bruscas para que sus momentos magnticos sigan la direccin ms prxima a H. Este movimiento es irreversible, y si deja de aplicarse la excitacin magntica, permanece la alineacin de los dominios que han rotado. Si se sigue incrementando el valor de H, el proceso de alineacin contina gradualmente, extendindose simultneamente a los dominios (caso anterior) y a los mo-

..

_--_ .... ---_ ... _---_ ...... ----_ ......

Crecimiento reversible

Figura 1.2. Curva de imanacin del hierro.

MQUINAS ELCTRICAS

mentas magnticos dentro de los mismos, de tal forma que cuando los dominios estn alineados totalmente se dice que el material se ha saturado, resultando una permeabilidad relativa unidad. La curva dibujada en la Figura 1.2se denomina curva de imanacin de la muestra y en la Figura 1.3 se representan algunas formas de curvas de magnetizacin (o imanacin) para diversos materiales empleados en la construccin de mquinas elctricas. Se observa que la chapa magntica * posee mejores cualidades magnticas que el hierro fundido o que el acero fundido, ya que para la misma excitacin magntica H se consiguen inducciones ms elevadas, lo que supone un volumen menor del material. Para resolver ejercicios prcticos o estudiar con ayuda de un ordenador un circuito magntico es ms conveniente utilizar una expresin analtica que relacione B con H. Una ecuacin tpica debida a Froelich es:
aH B=-l+bH (1.6)

que eligiendo unos valores adecuados para las constantes a y b, pueden aproximarse a las curvas de magnetizacin de los materiales reales. [En muchos de los problemas propuestos a lo largo de este captulo se utilizar la expresin (l.6) para definir las diferentes curvas de magnetizacin, lo cual da una mayor agilidad a los clculos; hay que resaltar que en los casos reales debern utilizarse las curvas de magnetizacin que proporciona el fabricante]. Hay que destacar que la relacin B = f(H) en estas curvas no es lineal, lo que indica que la permeabilidad del material definida por:
Ji =H

(1.7)

depender del valor de la excitacin magntica que se aplique.


B (Teslas)

2,0

1.8 1.6
1,4 1,2 1,0

...... ~

/
_,/'

-.-~ 00 ~

Chapa magntica

r-

Acero fundido

I
'(

./
1/

0,8 0.6
0,4 0.2

I
./

V
~

Hierro fundido

t r-

1/

I
~

g g g 8 g g g 8 8 x 8 g x 8 8 H (A. v/m)
~ ~ N

~ ~ ~ ~ ~ ~ ~ ~ ~ ~ ~

Figura 1.3. Curvas de imanacin de diversos materiales.

En el apartado 1.5 se explicar la constitucin

y se justificar

la existencia de las chapas magnticas.

CAPTULO

l.

CIRCUITOS

MAGNTICOS

y CONVERSIN

DE ENERGA

Realmente, el valor de B que se produce en un material ferromagntico debido a una determinada excitacin magntica H no es una funcin uniforme como se indica en la Figura 1.3, sino que depende adems de la historia del material. Para observar este fenmeno, consideremos que la muestra ferromagntica se introduce dentro de una bobina como indica la Figura l.4a. En la Figura l.4b se muestra la curva B = f(H) que se obtiene al aplicar excitaciones magnticas H de diferente magnitud y signo. Se parte del material desmagnetizado indicado por el punto a de la Figura l.4b, y se aplica un campo H creciente introduciendo en la bobina una corriente, p. ej., en la direccin indicada en la Figura l.4a, hasta que se alcanza el punto b. Cuando se hace disminuir H, se reduce el valor de B, pero segn un camino diferente. Al volver H a cero persiste una cierta magnetizacin (punto e). Al valor de B en este punto se le conoce con el nombre de magnetismo o induccin remanente y constituye el estado de magnetizacin permanente de la muestra. El punto d determina el campo coercitivo, que es el campo opuesto que resulta necesario aplicar para desmagnetizar la muestra (por inversin en el sentido de la corriente de la bobina de la Figura 1.4a). Si continuamos hasta el punto e y despus invertimos el sentido de cambio de H, llegaremos a formar una curva cerrada denominada ciclo de histresis * . La causa de este ciclo es la dificultad que presenta el desplazar las paredes de los dominios. Las imperfeccionesdel cristal tienden a fijar las paredes, que como consecuencia no se mueven suavemente con el campo aplicado. Esta histresis, que en algunos materiales resulta muy grande, es la que permite la existencia de imanes permanentes muy potentes. (La condicin esencial de todo imn permanente es tener una estabilidad perfecta, es decir, guardar mucho tiempo sin variacin sus propiedades magnticas. Debe, pues, tener un magnetismo remanente intenso y una fuerza coercitiva importante. Para imantar una barra se enrollan a su alrededor espiras conductorasrepartidasregularmentepor toda su superficie y se hace circular durante un tiempo una corrientecontinua intensa. La evolucin de los imanes permanentesha sido considerable en los ltimos cincuenta aos. Al principio se utilizaba nicamente acero al carbono; actualmente se emplean aleaciones especiales a base de hierro, nquel, cobalto e incluso con
Muestra ferromagntica B

Bobina

+
a) Bobina con ncleo de hierro
b)

Ciclos de histresis

Figura 1.4. Ciclo de histresis.

* El trmino histresis procede del griego y significa retraso, indicando con ello el retardo en la imanacin de un material respecto al campo aplicado. Por ejemplo, cuand H es positivo y alcanza el valor cero, B es todava positiva en el valor remanente B" o cuando B llega a cero, entonces H tiene ya un valor negativo y representa el campo coercitivo He'

MQUINAS ELCTRICAS

Tabla 1.1. Parmetros de la curva de histresis para diversos materiales

Hierro Hierro al silicio Hierro al silicio Permalloy Mumetal

99,9 Fe 4 Si; 96 Fe 3,3 Si; 96,7 Fe 45 Ni; 54 Fe 75 Ni; 2 Cr; 5 Mn; 18 Fe

5.000 7.000 10.000 25.000 110.000

80 48 16 24 2,4

2,15 1,97 2 1,6 0,72

10 59 50 50 60

elementos de las tierras raras.) Las sustancias ferromagnticas con mucha histresis se llaman duras, mientras que las que presentan poca se denominan blandas o dulces. Se observa en la Figura 1.4b que para un valor de H corresponden varios de B, lo que matemticamente expresa una funcin multiforme y que indica, como ya se adelantaba antes, que el estado magntico depende de la historia del material, es decir, depende de los estados magnticos anteriores. Hay, sin embargo, una curva B(H) perfectamente determinada, y es la que se obtiene uniendo los vrtices de los ciclos correspondientes a diversos Hmx aplicados (Fig. 1.4b), lo que da origen a la curva de magnetizacin de la sustancia indicada en la Figura l.3 para diversos materiales. En la Tabla 1.1 se muestran algunos valores caractersticos de la curva de histresis (y algunos otros parmetros) para diversos materiales empleados en la Tecnologa Elctrica.

1.3. LEYES DE LOS CIRCUITOS MAGNTICOS


La descripcin exacta del campo magntico requiere el uso de las ecuaciones de Maxwell y el conocimiento de las relaciones entre la induccin B y la intensidad del campo magntico" en el medio en el que se establecen los campos. Como quiera que en lo concerniente a las mquinas elctricas las frecuencias de las seales puestas en juego son bajas, se pueden emplear con suficiente exactitud las aproximaciones que implican la utilizacin de lo que en electromagnetismo se denomina campo cuasiestacionario. En definitiva, se puede despreciar la corriente de desplazamiento en las ecuaciones de Maxwell, siendo por consiguiente vlidas las relaciones magnetostticas siguientes: div B

=O

; rot H

=J

; B

= /1"

(1.8)

Recurdese de un curso de electromagnetismo que la primera relacin anterior indica la imposibilidad fsica de poder aislar los polos magnticos, representa de otro modo una forma elegante de justificar el carcter solenoidal de las lneas de induccin B (las lneas de campo magntico son cerradas, sin principio ni fin). La segunda ecuacin (1.8) es la ley de Ampre en forma diferencial y que en forma integral se convierte en:

fy " . di =

J . ds

Li

= Ni = ,'7

[A.v]

(1.9)

que indica que la circulacin del campo magntico" a lo largo de un camino cerrado y es igual a la suma de corrientes que atraviesan cualquier superficie S apoyada en el camino. Si existen N espiras llevando cada una la corriente i, la suma de corrientes ser igual al produc-

CAPTULO

l.

CIRCUITOS MAGNTICOS Y CONVERSIN DE ENERGA

to Ni. Este producto tiene gran importancia en el estudio de las mquinas elctricas y se denomina fuerza magnetomotri: .7 (de un modo abreviado, f.m.m.) y que se mide en una unidad til para el ingeniero denominada amperivuelta (A. v). La f.m.m. es la causa de que se establezca un campo magntico en un circuito, de un modo anlogo al de fuerza electromotriz (f.e.m.) que es la causa, en un circuito elctrico, de que se establezca una corriente elctrica. En la mayora de las situaciones prcticas que se suelen dar en el estudio de las mquinas elctricas, el camino 'Y elegido para aplicar la ley de Ampre (1.9) coincide con la trayectoria media seguida por las lneas de campo magntico H; por otro lado, si el material es homogneo e istropo, la magnitud de H es la misma en todo el recorrido, de ah que (1.9) se transforme en la ecuacin escalar siguiente: (1.10) en la que representa la longitud magntica media de las lneas de H. Si el recinto y no es atravesado por ninguna corriente, la ecuacin (1.9) nos indica que el campo magntico es entonces irrotacional y que por consiguiente procede del gradiente de un campo escalar denominado potencial magntico U, es decir: rot H

= O=JH

Jy

. di = O = H

=-

gradU

[A.v/m]

(l.ll)

El potencial magntico U es anlogo al potencial escalar elctrico V. La ltima ecuacin (1.8) representa la relacin existente entre los campos B y H Y que se denomina permeabilidad. En los materiales homogneos e istropos se cumple la relacin modular:

B = J1H

[T]

(1.l2)

ya que B y H son uniformes y los campos vectoriales correspondientes tienen la misma direccin y sentido. En los materiales ferromagnticos, J1 tiene un valor elevado y no es uniforme, lo que significa que su magnitud depende del mdulo de H. Para los dems materiales, sean aislantes o conductores elctricos, la permeabilidad es prcticamente la del vaco : J10 = 4n . 10-7

[H/m]

Otro concepto que se debe recordar es el de flujo magntico <l>que atraviesa un rea S, que viene definido por: [Wb] (1.13)

y que en unidades S.I. se mide en Webers. En la prctica, la induccin magntica es prcticamente constante en la seccin transversal de los ncleos ferromagnticos y adems tiene la misma direccin que la superficie, y por ello (1.13) se transforma en: <l>=BS [Wb] (1.l4)

De este modo, si se tienen en cuenta las expresiones (1.10), (1.12) y (1.14), resulta:

.7= He = -

Be ='<l>- e
J1 J1S

[A.v]

(1.l5)

10

MQUINAS ELCTRICAS

Si se denomina reluctancia magntica 9l a:


(1.16)

la ecuacin (1.15) se puede escribir: [A.v]


(1.17)

que es una expresin fundamental para el estudio de los circuitos magnticos y que se denomina ley de Hopkinson, o ley de Ohm de los circuitos magnticos, por su analoga con la ley de Ohm de las redes elctricas: e = iR

[V]

(1.18)

Como se deduce de las expresiones anteriores, existe una gran analoga entre los circuitos elctricos y magnticos que hacen que puedan estudiarse los circuitos magnticos con las mismas tcnicas desarrolladas en el anlisis de los circuitos elctricos. Pero antes de dar a conocer todas las analogas existentes entre ambos tipos de circuitos, conviene destacar que el circuito magntico difiere del circuito elctrico en varios aspectos, que hacen difcil el que se pueda llegar al mismo grado de precisin en los clculos de estructuras magnticas que en los clculos de circuitos elctricos. La corriente elctrica se considera que se limita a un camino definido (el hilo conductor); el aire circundante y los soportes aislantes del hilo tienen una resistencia muy elevada, de manera que las corrientes de dispersin que escapan del hilo son casi siempre despreciables comparadas con la corriente que pasa por dicho hilo. Pero no se conoce ningn aislante para el flujo magntico; de hecho, el propio aire es un conductor magntico relativamente bueno; por lo tanto, es imposible sealar a las lneas de campo magntico caminos definidos como los que se establecen para las corrientes elctricas. Por ejemplo, en el circuito magntico de la Figura 1.5 se. observa que del flujo total producido por la bobina $" parte se dispersa por el aire: $ d Y otra parte que denominaremos flujo til $u atraviesa el ncleo de tal forma que se denomina coeficiente de dispersin o de Hopkinson v al cociente :
$, $u + $d $d v=-= =1+$" $u

e,

(1.19)

El flujo de dispersin oscila entre ellO y el 30 por 100del flujo til, por lo que el coeficiente de Hopkinson vara entre v = 1,1 a 1,3. Este coeficiente tiene gran importancia en el anlisis de los circuitos magnticos de las mquinas elctricas.

Entrehierro

Figura 1.5. Dispersin magntica en la bobina. Expansin del campo magntico en el entrehierro.

CAPTULO

/.

CIRCUITOS

MAGNTICOS

Y CONVERSIN DE ENERGA

11

Otro efecto a consideraren los circuitosmagnticoses la expansinque ofrecen las lneas de campo, al circular el flujo por espacios de aire, denominadosentrehierros, como se indica en la Figura 1.5,lo que hace que se incrementeel rea efectiva de circulacindel flujo en los mismos respectoa la superticiegeomtricareal. En el desarrollode este captulo se considerar,mientras no se diga lo contrario,que la dispersiny expansin de las lneas de campo son despreciables. Una vez hechas todas estas consideraciones y para comprender ms plenamente todas las analogas entre los circuitos elctricos y magnticos, se van a considerar los esquemas de las Figuras 1.6a y b. En la Figura 1.6a se ha representado un circuito elctrico formado por un conductor de conductividad (J, longitud e y seccin uniforme S, alimentado por una pila de f.e.m. e. En la Figura 1.6b se muestra un circuito magntico de permitividad u, longitud e y seccin uniforme S, alimentado por una bobina de f.m.m.. 7= Ni. En el circuito elctrico, el campo elctrico no conservativo de la pila E, produce una d.d.p. en bornes, que a su vez provoca un campo elctrico E en todos los puntos del conductor, dando lugar segn la ley de Ohm a una densidad de corriente J = (JE, cumplindose las relaciones bsicas siguientes:
a)
b) e)

Fuerza electromotriz: e =

l EN . di Jy

Principio de continuidad: div J = O Ley de Ohm diferencial: J = (JE Corriente elctrica: i =

d)
e)

J . ds
s

D.d.p. entre dos puntos: Vl2 = V - \! =

rE.
.r =

di

En el circuito magntico, la f.m.rn, Ni provoca la aparicin de un campo magntico H a lo largo de todo el circuito magntico, que da lugar a una induccin B = uli, cumplindose las relaciones bsicas siguientes:
a) b)

Fuerza magnetomotriz (ley de Ampre): Carcter solenoidal de B: div B = O Relacin del medio: B = flH

f.

H . di

e)

----------------------

~'\

+
Pila ~J
I

!H
s
N

Bobina

lB
I I

'_ - - --- - - - - --- ---- ----_.'


I
a) Circuito elctrico

,-----------------------, I
b) Circuito magntico

I I

Figura 1.6. Analoga circuito elctrico-circuito magntico.

12

MQUINAS ELCTRICAS

d)
e)

Flujo magntico: <1> =

Ss B . ds

D.d.p. magntico: Vi2 = Vi - V2 =

(l.20) H . di (1.21)

Comparando las ecuaciones (1.20) y (1.21) podemos establecer las analogas mostradas en la Tabla 1.2. Se observa que la f.m.m.. 7= Ni en el circuito magntico cumple la misma funcin que la f.e.m. e en el circuito elctrico, la induccin B es anloga a la densidad de corriente J, la permeabilidad ji es anloga a la conductividad a, el campo magntico H es anlogo al campo elctrico E, el flujo magntico <1> es anlogo a la corriente elctrica i y el potencial magntico Ves anlogo al potencial elctrico V. La tabla de analogas anterior puede ampliarse a magnitudes ms tiles para el ingeniero. As, resulta ms prctico emplear conceptos de relaciones entre tensiones y corrientes que de campos. Sabemos, por ejemplo, que el principio de continuidad de la corriente en los circuitos elctricos conduce al primer lema de Kirchhoff: div J = O
=;>

f
f

J . ds = O

=;>

Li=O

(1.22)

que nos indica que la suma de corrientes que llegan a un nudo es igual a cero. De un modo equivalente, teniendo en cuenta que en los circuitos magnticos el flujo es anlogo a la corriente de los circuitos elctricos, se cumplir en un nudo magntico: div B = O
=;>

B . ds = O

=;>

L <1>

=O

(1.23)

ecuacin que representa el primer lema de Kirchhoff aplicado a los circuitos magnticos: la suma de flujos que llegan a un nudo magntico es igual a cero. Por otro lado, la ley de Ohm en forma diferencial: J = aE, se convierte en forma integral:
e = Ri

(1.24)

donde R es la resistencia del circuito, que en funcin de la longitud e, seccin S y conductividad a vale:

R=--

1e aS

(1.25)

Tabla 1.2. Parmetros equivalentes entre los circuitos elctrico y magntico

e:

f.e.m. [V]

J: densidad de corriente [A/m]


a: conductividad ES/m] E: campo elctrico [V/m] i: corriente elctrica [A] V: potencial elctrico [V]

9: f.m.m. [A.v] induccin [T] permeabilidad [H/m] /1: H: campo magntico [Av/m] <1>: flujo magntico [Wb] U: potencial magntico [A.v]
B:

CAPTULO l.

CIRCUITOS MAGNTICOS Y CONVERSIN DE ENERGA

13

Para los circuitos magnticos, la ecuacin equivalente a (1.24) es la ley de Hopkinson que ya se determin en (1.17): (1.26) donde la reluctancia ~se defini en (1.16): (1.27) el lector comprobar las analogas entre las expresiones de la resistencia elctrica (1.25) Yla reluctancia magntica (1.27). Segn (1.26), la unidad de reluctancia magntica es el cociente de A.vlWb, que es la inversa del henrio, es decir, H-'. El inverso de la reluctancia magntica se denomina permeancia ~ = 1/!)ir y su unidad es el henrio (H). En la prctica de los circuitos elctricos, la ley de Ohm se convierte en el segundo lema de Kirchhoff:

Le

L Ri

(1.28)

y de un modo anlogo, en circuitos magnticos, la ley de Hopkinson (1.26) se transforma en: (1.29) que indica que en un circuito magntico la suma de f.m.m.s. en una malla es igual a la suma de cadas de tensiones magnticas, representadas por la suma de los productos de las reluctancias por los flujos. Las ecuaciones (1.23) y (1.29) son la base del clculo de las estructuras magnticas. En la Tabla 1.3 se han representado estas ecuaciones a modo de sntesis y su comparacin con las ecuaciones de los circuitos elctricos. Tambin se muestran las leyes de asociacin de reluctancias, que son anlogas a las de asociacin de resistencias. De lo que antecede se deduce que un circuito magntico puede resolverse, al menos a primera vista, como si se tratara de un circuito elctrico, con las analogas presentadas en las Tablas 1.2 y 1.3. En realidad, la resolucin es algo ms compleja porque hay una diferencia esencial que hemos ocultado al lector, que hace que el clculo no sea tan directo. Efectivamente, si en la resolucin de un problema de circuitos magnticos deseamos determinar las reluctancias de las diferentes partes del circuito con ayuda de la ecuacin (1.27) para ms tarde poder aplicar la ley de Hopkinson o su generalizacin en la forma del segundo lema de Kirchhoff, se caer enseguida en la cuenta de que el problema no es obvio, ya que para
Tabla 1.3. Leyes equivalentes circuito elctrico-circuito magntico

Primer lema de Kirchhoff: L i = O Segundo lema de Kirchhoff: L e = Resistencia: R


ie = -as - en]

L Ri

Primer lema de Kirchhoff: L q, = O Segundo lema de Kirchhoff: L :JT = L Reluctancia


[;f

[;f q,

=- ;.S

[Wl]

Resistencias en serie: RT = L R; Resistencias en paralelo: l RT

Reluctancia en serie: .9lr = L 9P,


l

=L-

Reluctancias en paralelo:

R;

9lr = L 9P,

14

MQUINAS ELCTRICAS

determinar (i? se necesita saber el valor de 11 = 11011,., pero la permeabilidad relativa no se puede conocer hasta que no se conozca H o B, que es lo que en definitiva se desea calcular. En los problemas en los que se parte de una induccin conocida, el clculo de la f.m.m. se realiza con ayuda de (1.29), donde suele ser ms prctico sustituir el segundo miembro por el que se indica en la siguiente ecuacin:

.7= U= He

(1.30)

Ahora bien, en el caso de que el dato de referencia sea la f.m.m., la nica forma de resolver el problema es por un procedimiento iterativo de ensayo y error; es decir, el clculo comienza eligiendo un valor de B y determinando la f.m.m. necesaria, que se compara con la real aplicada, a continuacin se modifica el valor de B anterior hacia arriba o hacia abajo para que la f.m.m. se acerque al valor original y as sucesivamente. Con un poco de prctica, se resuelve el problema con dos o tres iteraciones a lo sumo. EJEMPLO DE APLICACIN 1.1 El ncleo central del circuito magntico de la Figura 1.7 est bobinado con 800 espiras. El material es acerofundido con un valor de la permeabilidad relativa 11, = 1.000. Calcular la corriente i que debe aplicarse a la bobina para obtener en el entrehierro unflujo de 1 mWb.

(1

= 700 mm; s = 2.000 mm2)


Figura 1.7.

Ncleos laterales

SOLUCIN El circuito elctrico equivalente es el indicado en la Figura 1.8, donde 01'1 indica la reluctancia de los ncleos laterales, !i?c indica la reluctancia del ncleo central y .!?e es la reluctancia del entrehierro. Los valores respectivos son: rl'1 = -1
11, 110 s =

700 10-3 = 27,85 Ht [H-'J 1.000 . 4n . 10 7 . 2.000 . 10 6 [W1J ;


!'e =

?, = 4,97 lO"

19,9 104 [W1J

donde se ha tenido en cuenta para el clculo de Y?e que la permeabilidad relativa del aire es igual a la unidad (obsrvese que la reluctancia del entrehierro es prcticamente cuatro veces la reluctancia del ncleo magntico central, de lo que se deduce la conveniencia de limitar al mximo los espacios de aire en los circuitos magnticos, para reducir lo ms posible la f.m.m. de la bobina).

CAPTULO l.

CIRCUITOS MAGNTICOS Y CONVERSIN DE ENERGA

15

$/2

Figura 1.9.

Figura 1.8.

Las reluctancias lente:

9{,

de la Figura 1.8 estn en paralelo, resultando una reluctancia equiva-

(f
<

EQ

= (j', 2 =

13' 92 . 104

[H-'J

dando lugar al circuito de la Figura 1.9. En esta figura, aplicando el segundo lema de Kirchhoff resulta:

L Ni

L <D.

(/1. = 10-3 (4,97 + 19,9 + 13,92)' lCt = 387,9 A.v

387,9 de donde: i = 800 = 0,485 A

EJEMPLO DE APLICACIN 1.2 Resolver el problema anterior supuesto que la curva de magnetizacin del acero fundido viene expresada por la ecuacin:
B =-' ---,;1 + 10-3 H

1 8 10-3 H

B: Teslas;

H: Av/m

SOLUCIN Debido a la simetra del circuito y de acuerdo con la Figura 1.8, el flujo en las columnas laterales vale la mitad que en la columna central, es decir: <D -3 <D, = "2 = 0,5 . 10 . Wb Como la seccin lateral es igual a 2.000 mrrr', la induccin en estas columnas valdr: B, = <D 05 10-i = 2.00' 10-6 = 0,25 Teslas
3

que llevando a la curva de magnetizacin del material da un valor de H,: 0,25 =' 18.10-3 H 3' 1 + 10-' H,

= H, = 161,29 Av/m

El ncleo tiene doble flujo y doble seccin que las columnas laterales, por lo que se deduce idntico valor de la induccin y en consecuencia de la excitacin H, es decir: Hc=H,
= 161,29 A. v/m

16

MQUINAS ELCTRICAS

En cuanto al entrehierro, se cumplir: B = 0,25 Teslas => H = - =


e

B 0,25 J1.o 4n. 10

1,99' 105
7

Aplicando en segundo lema de Kirchhoff a la malla 1 del circuito de la Figura 1.8, de acuerdo con la frmula se obtiene:

NI = 161,29' 0,7 + 161,29' 0,25 + 1,99' 105 10-3 = 352,23 A.v

=> I

= 352,23 = O 44 A
800 '

EJEMPLO DE APLICACIN 1.3


La Figura 1.10 representa un circuito magntico realizado con un material ferromagntico

cuya curva de imanacin est expresada por la ecuacin:


2 10-3 H

B = 1 + 10-3 H

B: Teslas;

H: Av/m

El entrehierro es de 1mm, la longitud media magntica de la estructura es de 1 m y la seccin transversal es uniforme de valor: 20 cm'. Calcular la induccin magntica en el entrehierro.

_j_
lmm

s = 20 cm2

Figura 1.10. SOLUCIN La f.m.m. de la bobina es: tJT = Ni = 1.000' 0,5 = 500 A.v Vamos a considerar una induccin de prueba en el entrehierro de 1 Tesla. Al ser un circuito magntico en serie, la induccin anterior ser la que existir tambin en la estructura ferromagntica. En la Figura 1.11 se muestra el circuito elctrico equivalente. El campo en el entrehierro ser:

4n' 10-

---'7

= 7,96 . 10 Av/m

De este modo la d.d.p. magntica entre 2 y 3 ser:

U23 = Ha la = 7,96 . 105 . 1 . 10-3 = 796 Av


que ya supera con exceso el valor de la f.m.m. de la bobina, que es de 500 Av. Probamos entonces con un valor menor de la induccin, por ejemplo 0,5 Teslas. En este caso el campo Ha ser: 0,5 H -_ Ba a J1.o 4n . 10-7 3 "98 105 A .v/ m

CAPTULO l.

C1RCUlTOS MAGNTICOS y CONVERS16N DE ENERGA

17

y por consiguiente, la diferencia de potencial magntico en el entrehierro ser:

V23 = Ha la = 3,98 . 105. l . 10-3 = 398 A.v


.<71 Fe

.---.....I\M,----,

+
.7= 500A.v

!
9Ie

L..------ .... 3
Figura 1.11. A continuacin se calcular la d.d.p. en el hierro. Como quiera que la induccin en el hierro es tambin de 0,5 Teslas, en la curva de imanacin se tendr: 0,5 = 2 10-3 H

1 + 10-' . H

H = 333,33 Av/m

que corresponde a una d.d.p. magntica: VI2 = HLFe = 333,33 . 1 = 333,33 A.v lo que requerir una f.m.m. total en la bobina: .7= VI2 + V23 = 333,33 + 398 = 731,33 A.v/m que es superior al valor de 500 Av que impone el enunciado, lo que demuestra que la induccin real es an ms baja. Si se prueba ahora con una induccin de 0,3 Teslas, el lector puede comprobar, siguiendo el proceso sealado, que se obtiene:

Ha = 2,39' 105 Av/m;


es decir:

H = 176,5 Av/m VI2 = 176,5 A.v

V23 = 239 Av;

y por consiguiente ;7= VI2 + V23 = 415,5 A.v, que es inferior a los 500 Av de la bobina.
Habr que hacer un nuevo intento, elevando el valor de la induccin. El lector puede comprobar que eligiendo B = 0,36 Teslas se obtiene una f.m.m. de 506 Av ~ 500 Av, por lo que se puede tomar el valor de B = 0,36 Teslas como solucin del problema.

EJEMPLO DE APLICACIN 1.4 El esquema de la Figura 1.12 corresponde al circuito magntico de una mquina elctrica rotativa. Existen dos ncleos polares sobre los que se colocan unos devanados de excitacin cuya misin es crear el flujo que atraviese al inducido, y ste va devanado con un bobinado (no indicado en la figura) donde se genera la fe.m. de la mquina al girar el inducido movido por una energa mecnica externa. Las dimensiones son las indicadas en la Figura 1.12, Y los materiales con los que est construida la mquina son: Ncleos polares, inducido: chapa magntica. Culata:acerofundido. Las curvas de magnetizacin de estos materiales son las indicadas en la Figura 1.3. Calcular el nmero de Amperios-vuelta por polo necesarios para producir una induccin en

18

MQUINAS ELCTRICAS

el entrehierro de 1 Tesla, supuesto que el coeficiente de Hopkinson entre los polos y el inducido vale 1,15.
Ncleo polar (25 cm; 1.000 rnrn-) Culata (150 cm; 450 rnm-)

Bobinas de excitacin

Entrehierro (0,6 mm; 1.200 rnrn-)

polar

Figura 1.12.

SOLUCiN Para resolver este problema se seguir el mismo desarrollo que en los ejemplos anteriores, calculando la d.d.p. magntica necesaria en cada parte de la mquina. En la Figura 1.13 se muestra el circuito elctrico equivalente. Obsrvese que debido a la dispersin del flujo magntico al pasar ste del esttor al rotor, el flujo en los polos es v<l>, siendo <l> el flujo en el entrehierro, y de este modo resulta:
a)

ENTREHIERRO. La induccin en esta zona vale 1 Tesla, lo que equivale a un campo magntico He:

H=e

Be
Po

l 5 = 7,96 . 10' Av/m 4n' 10


--:------"-'7

De este modo las diferencias de potenciales magnticos en cada entrehierro sern:


Ve = VCD

VEF

= He l, = 7,96

. 105. 0,6 . 10-3

= 477,6

A.v

<I? culata

.(J{ culata

Figura 1.13.

CAPTULO

l.

CIRCUITOS

MAGNTICOS

Y CONVERSIN

DE ENERGA

19

b) INDUCIDO: El flujo que atraviesa el entrehierro vale: <1>, = <1> = B Se

1 1.200,

10-6

1,2.10-3

Wb

Al llegar al inducido se divide en dos partes iguales: <1> = <1>/2 = 0,6 . 10-3 Wb, lo que corresponde a una induccin en el ncleo:
B

=~ =

<1> 06 . 10-3 5~0' 10-6

1,2 Teslas

lo que equivale en la curva de chapa magntica de la Figura 1.3 a un campo magntico aproximado H = 200 A.v/m. Como la longitud de cada parte del inducido es de 50 cm, se tendr una d.d.p. UIJEdada por: V = VOE = H t = 200 . 0,5 = lOOA.v.
e)

NCLEOS POLARES: El flujo en los polos, teniendo en cuenta la dispersin, es igual a 1,15 veces el flujo en el entrehierro, es decir:
<l>
l'

= v<1> = 1",15 . I 2 . 10-3 = 1 38 . 10-3 Wb e

=>

B
p

= __E = ' s" 1.000'

<1>

l 38 . 10-3

IQ

= 1' 38 Teslas = 100 A.v.

lo que equivale en la curva de chapa magntica de la Figura 1.3 a un campo magntico


Hp = 400 Av/m. Como la longitud de cada polo es igual a 25 cm, se tendrn las

siguientes diferencias de potencial magntico: VI' = VAB d)

= VCH = 400

. 0,25

CULATA: El flujo que circula por cada una de las partes de la culata es la mitad del que atraviesa los polos, es decir:
ffi

'Ve

= <l>1' = 2

1,38 . 10-3 2'

= O 69

.10-3 Wb

=> Be

<1\ 0,69' =S = 450.

10-3 10-6

=, I 53 T es1as

por lo que el campo He necesario, que se obtiene en la curva de imanacin del acero fundido de la Figura 1.3, vale: H,. = 2.200 A.v/m, de este modo la d.d.p. magntica en cada parte de la culata ser:
Ve

VHA

= He l.> 2.200

. 1,5

= 3.300

A.v

Para calcular la f.m.m. total, obsrvese que en la Figura 1.13 se cumple: 2.7= VAB + Veo + VOE + VEF + VCH + VHA
=>

2.7= 2 Ve + V + 2V" + Ve

que al sustituir valores da lugar a una f.m.m. por polo:


=r.:
.J' -

2477,6 + 100 + 2.100 + 3.300 _ 2 2 - .278 A.v

1.4. ENERGA Y COENERGA MAGNTICA


Considrese una bobina de N espiras arrollada en un ncleo ferromagntico, tal como se muestra en la Figura 1.14a, que se conecta a una fuente de tensin variable v(t); como consecuencia de ello se establecer una corriente i(t) en la bobina que producir un flujo variable <1>(t) en el ncleo. De acuerdo con la ley de Faraday, el flujo anterior crear una f.e.m. inducida

20

MQUINAS ELCTRICAS
R fmetros e=--

d<l> v(t) dt

i(t)

e=_Nd<l> dt b)

+
S
R

+
d<l> v(t) e=+dt

i(t)

e)

a)
<l>(t)

Figura 1.14. F.e.m. y f.c.e.m. inducida en una bobina con ncleo de hierro.

en cada una de las espiras del devanado, dando lugar a una f.e.m. total, que puede considerarse bien como una elevacin de tensin en el sentido de la corriente (vase Fig. 1. 14b) de valor:
d<D e=-N-

dt

(1.31)

o bien como una cada de tensin en el sentido de la corriente (vase Fig 1.14c), denominndose entonces fuerza contraelectromotriz (abreviadamente f.c.e.m.) cuya magnitud es:
d<D e=+N-

dt

(1.32)

Las dos formas anteriores de expresar la misma ley (ley de Faraday-Lenz) suele ser motivo de confusin entre los estudiantes, no habiendo razn alguna a este dislate. Para ilustrar ms eficazmente la aplicacin de la ley de Faraday, en la parte central superior de la Figura 1.14 se ha aislado una espira del devanado para ver el sentido de la f.e.m. y corriente inducida. Se observa que la polaridad de la f.e.m. inducida es tal que produce una corriente (si la espira est cerrada) que da lugar a un flujo inducido en el mismo sentido del flujo inductor <D(t), y es por ello que se hace necesario incluir el signo menos en la expresin de la f.e.m., para tener en cuenta la ley de Lenz, de oposicin al cambio de flujo. En la Figura 1.14b se muestra el circuito elctrico equivalente de la bobina, donde se observa que el sentido de elevacin de la f.e.m. coincide con el sentido de la corriente (se ha considerado que la bobina tiene una resistencia R). En el caso de la espira dibujada en la parte central inferior de la Figura 1.14, se observa que la polaridad de la f.e.m. inducida es contraria a la considerada en el caso anterior; en esta situacin se produce una corriente en la espira, que da lugar a un flujo inducido que se opone al flujo principal <D(t); es por ello por lo que no es necesario incluir el signo menos en la expresin de la f.e.m., puesto que con la polaridad asignada se obtiene un flujo inducido que es antagnico al principal. En la Figura 1.14c se ha representado el circuito equivalente de la bobina correspondiente, en el que se observa que ahora el sentido de elevacin de tensin de la f.e.m. es contrario a la corriente, y de ah la justificacin de la denominacin antes indicada de fuerza contraelectromotriz: Una vez hechas estas aclaraciones, vamos a calcular el balance energtico que se produce en el circuito. Si aplicamos el segundo lema de Kirchhoff a la red de la Figura 1.14c resulta:
d<D v = Ri + N-

dt

(1.33)

CAPTULO l.

CIRCUITOS MAGNTICOS Y CONVERSIN DE ENERGA

21

donde las expresiones v, i y <1> son funciones del tiempo, aunque no se hagan constar explcitamente en la ecuacin anterior. Si en (1.33) se multiplican ambos miembros por i dt resulta: vi dt = R 2 dt + Ni d<1> o expresado de otro modo: (1.35) donde:
dWe: diferencial de energa elctrica que entra al circuito. dWR: diferencial de energa disipada en la resistencia R de la bobina por efecto Joule. dW",: diferencial de energa suministrada al campo magntico (diferencial de energa (1.34)

magntica). La ecuacin (1.35) representa el balance energtico del circuito o simplemente la ley de conservacin de la energa. El trmino dWm se puede escribir:
dWm

= Ni

d<1>=

ir d<1>

(1.36)

donde ,'7 representa la f.m.m. de la bobina. Si suponemos que en el instante inicial (t = O) el flujo en el ncleo es nulo y la corriente es cero, y si se incrementan estos valores hasta unas magnitudes finales <1> e i, se tendr una energa magntica total suministrada al ncleo magntico por la fuente:
(1.37)

La ecuacin anterior indica que cuando se incrementa el campo magntico asociado con un ncleo, la energa fluye de la fuente al campo. As pues, esta energa es almacenada por el campo magntico tanto tiempo como el flujo se mantenga en el mismo valor, en nuestro caso <1>. Si se considera que la curva de imanacin del material ferromagntico del ncleo es la mostrada en la Figura 1.15, que es anloga a la indicada en la Figura 1.2, pero en la que ahora (Fig. 1.15) se ha representado en abscisas la f.m.m. en vez del campo H, y en ordenadas el flujo <1> en vez de la induccin B, entonces la energa magntica Wm de la expresin (1.37) vendr expresada por el rea comprendida entre la curva de imanacin y el eje de flujos (rea sombreada horizontalmente). En la teora de los circuitos magnticos es interesante definir una magnitud denominada coenerga y que responde a la ecuacin: W~=

rT <1> dtT Jo

(1.38)

que es el rea comprendida entre la curva de imanacin y el eje de f.m.m.s. (era sombreada verticalmente en la Fig. 1.15). La coenerga no tiene un significado fsico directo pero es de gran utilidad para el clculo de fuerzas en los dispositivos electromagnticos. Obsrvese que la suma de la energa ms la coenerga magntica es el rea del rectngulo ;7<1> de la Figura 1.15.

22

MQUINAS ELCTRICAS

.T

Figura 1.15. Energa y coenerga magntica.

Las expresiones (1.37) y (1.38) pueden tambin definirse en funcin de los campos magnticos H y B. Si se considera el esquema de la Figura 1.14, en el que el ncleo tiene una seccin uniforme S y es la longitud magntica media (longitud geomtrica media), si se suponen uniformes los campos magnticos, se podr escribir:
<l>=BS ,/=H d <l> =S dB dtT=dH

(1.39)

De este modo la expresin de la energa magntica almacenada (1.37) se convertir en:

w'n =

I>
W
m

.7d<l> = vol,

H dB

(1.40)

donde vol = S representa el volumen del ncleo ferromagntico. La energa almacenada por unidad de volumen, y que se denomina densidad de energia magntica, valdr entonces:

w, =-= vol

I
O

HdB

(1.41)

De un modo anlogo, teniendo en cuenta (1.38) y (1.39) se obtiene una densidad de coenerga magntica: (1.42)

que tienen unas interpretaciones grficas similares a las de la Figura 1.15, si se representan ahora el campo magntico H en abscisas y la induccin B en ordenadas. Cuando la curva de imanacin del ncleo se considera lineal, los resultados precedentes dan lugar a expresiones muy simples. En primer lugar, es fcil darse cuenta en la Figura 1.15 que si la curva de imanacin es una recta, entonces coinciden los valores numricos de la energa y coenerga que corresponden a tringulos rectngulos cuyos catetos son <l> y .7, y de este modo (1.37) y (1.38) admiten las versiones siguientes:
(l.43)

CAPTULO l.

CIRCUITOS MAGNTICOS Y CONVERSIN DE ENERGA

23

donde se ha tenido en cuenta la ley de Hopkinson (1.26). De un modo anlogo coinciden tambin las expresiones de las densidades de energa y coenergia (1.41) y (1.42):
W'"

= w'" = 2 HB = 2 --; = 2' JlH

,1

1 B2

(1.44)

La ecuacin (1.43) puede expresarse tambin en funcin de la inductancia L de la bobina. Recurdese que la inductancia en un medio lineal viene definida por el cociente:

L=Ny teniendo en cuenta la ley de Hopkinson:

<l> i

(1.45)

.7= .0' <l> (l.45) se transforma en:


L

(1.46)

= N2 Ni = N2 -.7!7' = =r:
l
/"T2

<l>

<l>

N2

(1.47)

y por consiguiente (1.43) admite la siguiente versin: W'" = W,;, =

2' L

N2

= 2' L i2

(1.48)

que el lector recordar de un Curso de Teora de Circuitos.

1.5. PRDIDAS DE ENERGA EN LOS NCLEOS FERROMAGNTICOS


Cuando se reducen los campos magnticos asociados con ncleos ferromagnticos, parte de la energa almacenada es devuelta a la fuente. Sin embargo, parte de la energa almacenada se pierde irremediablemente en el ncleo en forma de calor. Esta prdida de energa es debida a dos causas: a) caracterstica de histresis del material (prdidas por histresis) y b) corrientes inducidas en el ncleo (prdidas por corrientes parsitas o corrientes de Foucault). Tambin se tienen prdidas de energa en ncleos sujetos a imanaciones y desimanaciones cclicas por medio de excitaciones peridicas.

1.5.1. Prdidas por histresis


Supngase que el ncleo ferromagntico mostrado en la Figura I.l4a es excitado por una bobina alimentada por una fuente de variacin peridica (en particular por una tensin sinusoidal) y que el ciclo de histresis del material magntico es el que se muestra en la Figura 1.16. Supngase que la amplitud del campo magntico vara entre +H Y -Hm, correspondiendo a variaciones de la induccin entre +Bm Y -;-Bm. Si se considera inicialmente que la induccin en el ncleo vara desde -Br (punto a) hasta Bm (punto e) siguiendo el tramo de curva abe, se tendr un aumento de induccin en el ncleo, 10 que corresponde a una

24

MQUINAS ELCTRICAS

B
d

Ciclo de histresis

Figura 1.16. reas del ciclo de histresis.

energa absorbida por el campo magntico y almacenada durante esta parte del ciclo, que de acuerdo con (1.40) valdr: Wac

= vol JB",
-8,.

H dB

= vol

WI

(1.49)

La integral W. de la expresin anterior representar, de acuerdo con lo indicado en el epgrafe anterior, el rea de la superficie abcdea de la Figura 1.16. Si se considera ahora que la induccin se reduce desde Bm (punto e) hasta B, (punto e), siguiendo el tramo ce de la curva de histresis, entonces resultar una energa devuelta a la fuente (red) durante esta parte del ciclo porque es negativa, y cuyo valor es: (1.50) El rea cdec de la Figura 1.16representar la densidad de energa correspondiente, que es el valor W2 de la parte integral de (1.50). Es evidente entonces que si se somete al ncleo a una induccin creciente entre -Br Y Bm siguiendo el camino abe y luego a otra induccin decreciente entre Bm Y B; siguiendo el camino ce, la superficie resultante abcea de la Figura 1.16 representar la densidad de energa absorbida por el ncleo ferrromagntico en esta excitacin Cclicay que no es devuelta a la red, sino que es disipada en el ncleo en forma de calor. Es evidente, segn se muestra en la Figura 1.16, que el rea ms clara corresponde a la mitad del ciclo de histresis y representa la diferencia de energas W2 - W l. Parece lgico, de acuerdo con la conclusin anterior, que si las variaciones de campo se producen entre Hm correspondiendo a variaciones de induccin Bm> la energa total disipada en el ncleo en forma de calor en este ciclo completo, y que designaremos por WH' ser: WH = (vol) pH dB (1.51)

donde la integral curvilnea se extiende a todo el ciclo de histresis, por lo que el resultado de la integral representar el rea que encierra el ciclo de histresis, que segn (1.51) significar la energa perdida por histresis por ciclo y por unidad de volumen del material magntico.

CAPTULO l.

CIRCUITOS MAGNTICOS Y CONVERSIN DE ENERGA

25

En la prctica, es conveniente hablar de prdida de energa por segundo en el ncleo, es decir, de potencia perdida por histresis. Si el nmero de ciclos de imanacin completa esf (donde f representa la frecuencia de la tensin de alimentacin a la bobina), entonces la potencia perdida ser: PH = fWH = f(vol)

fH

dB =f(vol) (rea del ciclo)

(1.52)

La ecuacin anterior es independiente de la forma de onda de la fuente de alimentacin, depende nicamente de la amplitud de la induccin, la frecuencia de la fuente (red) y la naturaleza del material magntico (rea del ciclo). Experimentalmente, C. P. Steinmetz propuso en 1892 una frmula emprica para definir el clculo de (1.52) y que viene expresada por la ecuacin: PH = kHf(vol) B~ (1.53) Los valores de kH (denominado coeficiente de Steinmetz) y IX (denominado exponente de Steinmetz) dependen de la naturaleza del ncleo ferromagntico. El exponente IX vara entre 1,5Y2,5, siendo un valor frecuente IX = 1,6, mientras que kH vara en el caso de acero al silicio entre 100 y 200.

1.5.2. Prdidas por corrientes de Foucault


Considrese el esquema de la Figura 1.17a, donde se muestra una bobina arrollada sobre un ncleo de hierro macizo. Al alimentar la bobina con corriente alterna se producir, de acuerdo con la ley de Ampre, un campo magntico alterno de induccin B, = Bm cos on que atravesar toda la masa de hierro en el sentido del eje Z (eje de la bobina).

e) Detalle de una chapa magntica

Figura 1.17. Corrientes de Foucault.

26

MQUINAS ELCTRICAS

De acuerdo con la ley de Faraday, aparecern en el material unas f.e.m.s. inducidas que darn lugar a unas corrientes parsitas que circularn por el material. Tngase en cuenta que el hierro es conductor de la electricidad, y aunque su conductividad es pequea en comparacin con la del cobre, las f.e.m.s. inducidas provocarn corrientes de circulacin por la masa del hierro. Estas corrientes, denominadas corrientes de Foucault (eddy currents o corrientes de torbellino en la bibliografa inglesa), se han sealado en la Figura 1.17a por medio de crculos concntricos en planos perpendiculares al flujo inductor y cuyo sentido de circulacin es tal, que el flujo producido por estas corrientes se opone (ley de Lenz) al flujo inductor de la bobina. Estas corrientes pueden originar grandes prdidas de potencia, con el consiguiente calentamiento de los ncleos. Para prevenir estas prdidas, el hierro empleado en los circuitos magnticos suele estar laminado, en forma de chapas magnticas de pequeo espesor, tal como se seala en la Figura 1.17b. El plano de las chapas es paralelo al flujo, por lo que las corrientes parsitas quedan confinadas a trayectorias de seccin transversal pequea. Consideremos una de estas chapas de dimensiones transversales a x b (donde a b) y profundidad L, tal como se seala en la Figura 1.17c, que es atravesada por el campo magntico B, = Bm cos on. Suponiendo que el campo es uniforme en la seccin transversal de la chapa, el flujo que atraviesa la espira sombreada de la Figura 1.17e es:
<l> = 2by Bm cos wt (1.54)

ya que la superficie cerrada de la espira es 2by. Por la ley de Faraday, la f.e.m. inducida tiene un valor modular dado por: e = 2wby Bm sen cot
(1.55)

Tomando una longitud unidad en la direccin del eje Z, la f.e.m. anterior produce una corriente de alrededor de la espira indicada, cuya resistencia vale: R=(J

2b

dy

(1.56)

donde se ha tenido en cuenta que a b y que la conductividad del material es (J. La potencia instantnea en la espira ser: ( 1.57) que corresponde a un valor medio:
(1.58)

y a una potencia disipada total:


(1.59)

lo que representa una potencia disipada por unidad de volumen:


PF/vol = n2pB;na2(j/6 = kd2B~a2(j (1.60)

donde se ha llamado k, a n2/6 y se ha tenido en cuenta que el volumen es: vol = abL = ab, al haber considerado una profundidad unidad.

CAPTULO l.

C1RCUITOS MAGNTICOS y CONVERSIN DE ENERGA

27

Wlkg

f =

50 Hz

f =

50Hz

2 ,O ,8 ,6
4

Chapa magntica laminada en cahente

,2
I,0 0, 8 0, 4

,6

lo'
~

I 1 J /
_.,I.

I I

I
flchapa magntical laminada en fro

./

0, 2
v

_"
0,6 0,8 1,0 1,2 1,4 1,6 1,8 B en Teslas

0,2

0,4

Figura 1.18. Curvas de prdidas en el hierro.

Las ecuaciones anteriores son vlidas solamente para valores de las frecuencias tales que la distribucin del campo magntico no est afectada por las propias corrientes parsitas. Cuando la frecuencia es elevada, el flujo que atraviesa las chapas no se distribuye uniformemente y se deben utilizar chapas ms delgadas. De las ecuaciones (1.53) y (1.60) observamos que las prdidas totales en el hierro son: (1.61) siendo vol el volumen de hierro. En la prctica, el fabricante de material magntico, suministra unas curvas donde se muestran estas prdidas totales en funcin de B, a frecuencia constante. En la Figura 1.18 puede verse un ejemplo de ello para chapas magnticas laminadas en fro o en caliente. Nota prctica: Existen aplicaciones de las prdidas por corrientes de Foucault. Considrese un disco metlico girando; si se acerca el polo de un imn, el disco disminuye su velocidad y se detiene rpidamente, al ser frenado por la accin de las corrientes de Foucault que se inducen en el disco. Este procedimiento de frenado se utiliza para amortiguar las agujas de ciertos aparatos de medida, en el frenado del disco de un contador de energa elctrica y en la automocin se emplea como un freno adicional en los camiones.

1.5.3. Consecuencias tecnolgicas


Para reducir las prdidas en el hierro de las mquinas elctricas, se deduce de todo lo anterior que deben emplearse chapas magnticas de pequeo espesor y baja conductividad (es decir, alta resistividad) y que tengan adems un ciclo de histresis pequeo. Las chapas magnticas se caracterizan, fundamentalmente, por contener silicio en la proporcin de 4 o 5 por 100. Esta adicin de silicio ha constituido un progreso considerable, ya que ha tenido como efecto la disminucin de las prdidas por corrientes de Foucault (a consecuencia del aumento de la resistividad, que alcanza de 5 a 6 veces la del hierro ordinario, como indica la Tabla 1.1). La incorporacin del silicio presenta, sin embargo, inconvenientes desde el punto de vista mecnico, ya que el hierro se vuelve duro y quebradizo. La calidad magntica de una chapa est influida no slo por la composicin qumica del hierro que la constituye, sino tambin por los procedimientos de fabricacin, particularmente por el sistema de laminado y los tratamientos trmicos. Antiguamente se empleaba un tipo melaminado en caliente y las chapas tenan unas prdidas al Tesla y 50 Hz que oscilaban entre 1,1y 1,5W/kg (vase Fig. 1.18);modernamente

28

MQUINAS ELCTRICAS

se emplea el laminado en fro, que tiene por efecto alinear las redes cristalinas que se producen en la aleacin hierro-silicio (malla del tipo cbico centrado), por medio de una deformacin plstica, y por ello esta clase de lmina de acero recibe tambin el nombre de chapa de cristales o granos orientados. Este tipo de chapa no contiene ms de 3 a 3,5 por 100 de silicio y se obtiene partiendo de un hierro ms puro an que el de las chapas ordinarias (laminado en caliente), sobre todo en lo concerniente al carbono, que desciende a 0,005 en vez de 0,04 por 100. Para eliminar las tensiones internas que se producen en las chapas de grano orientado, debe procederse, despus de la laminacin, a un recocido en atmsfera no oxidante (normalmente de hidrgeno); las prdidas en estas chapas varan segn sea la direccin del flujo respecto a la orientacin del grano, y para la direccin del laminado vienen a valer de 0,3 a 0,6 W/kg (a 1 Tesla y 50 Hz), pero estos valores se triplican cuando el flujo forma 90 con la alineacin de los cristales. Para reducir las prdidas por corrientes de Foucault, aparte de disminuir el espesor de las chapas (actualmente se construyen con espesores comprendidos entre 0,3 y 0,5 mm), es preciso intercalar entre las mismas un aislante. Al principio se utilizaba el papel como aislante, y para ello se pegaba por una cara de la chapa una hoja de papel fino de 0,02 a 0,03 mm de espesor, pero ste se alteraba fcilmente por el calor; posteriormente se utiliz un barniz de silicato sdico que recubra las dos caras, con un espesor total de 2 x 0,005 = 0,01 mm; ltimamente las chapas de grano orientado vienen preparadas mediante un tratamiento termoqumico especial, conocido con el nombre comercial de carlite, que crea una pelcula aislante extremadamente delgada (0,001 mm) cuya adherencia e inalterabilidad al calor son notables.

EJEMPLO DE APLICACIN 1.5 Unmaterialferromagntico se ha sometido a tres ensayos con diferentesfrecuencias e inducciones, dando lugar a las prdidas totales en el hierro mostradas en la siguiente tabla:

2 3

50 50 100

1
1,5

2 4 5

Calcular: a) Prdidas por histresis y por corrientes de Foucault en cada uno de los ensayos. b) Valor del exponente IX de Steinmetz. SOLUCIN
Designando con los subndices 1, 2 Y 3 las prdidas en cada ensayo resulta:

(a)
donde PH indica la prdida de potencia por histresis y PF las prdidas por corrientes de Foucault. Teniendo en cuenta asimismo que estas prdidas obedecen, de acuerdo con (1.61), a las ecuaciones generales siguientes:

CAPTULO 1.

CIRCUITOS MAGNTICOS Y CONVERSIN DE ENERGA

29

en las que M y N representan parmetros constantes, se podrn escribir las siguientes relaciones:

(b)
Si evitamos utilizar la primera ecuacin (b), la primera y tercera ecuacin (a) nos da: PHI + PFI =2 ; 2 PHI + 4 PFI =5 que da lugar a los siguientes valores: PHI = 1,5 Wjkg ; PFl

= 0,5 Wjkg

; PH3= 3 Wjkg

PF3

= 2 Wjkg

Ahora bien, de las relaciones, (b) obtenemos tambin: Pn = 1,52 PFI = 1,125 Wjkg ; PH2 = 4 - 1,125 = 2,875 Wjkg

y teniendo en cuenta la primera ecuacin (b) resultar:


-=--=-

PHI 1,5 PH2 2,875 1.6

i.s-

::::> ct ~

1,60

EJEMPLO

DE APLICACIN

Las prdidas en el hierro de una muestra de materialferromagntico son de 1.000 Wa 50 Hz. Cuando se aumenta la frecuencia hasta 100 Hz, manteniendo la induccin constante, las prdidas totales correspondientes han sido de 2.500 W. Calcular las prdidas por histresis y por corrientes de Foucault para ambas frecuencias. SOLUCIN Al ser la induccin constante, las ecuaciones de las prdidas admiten las siguientes expresiones: PH=Mf; dando lugar a las prdidas totales en el hierro: PFe = Mf+ Nf2 PF=NP

que corresponden a unas prdidas por unidad de frecuencia: Pdf=M+Nf


y al aplicar la ecuacin anterior a los datos del problema resulta:

1.000 --=M+50N 50 A 50 Hz: PHI = Mf , A lOOHz: PH2 = Mf2

2.500 ; --=M+ lOO

lOO N

=>

M= 15

N = 0,10

lo que da lugar a la distribucin de prdidas siguiente:

= 15 . 50 = 750 W; P = N fi = 0,10 . 50 = 250 W = 15 . 100 = 1.500 W; Pn = N f; = 0,10' 100 = 1.000 W


Fl 2 2

30

MQUINAS ELCTRICAS

1.6. CIRCUITOS MAGNTICOS EXCITADOS CON CORRIENTE ALTERNA 1.6.1. Generalidades


En el epgrafe 1.3se han estudiado las leyes de los circuitos magnticos,observando la analoga existente con los circuitos elctricos. La ley de Hopkinson expresada por la ecuacin:
<l> =-

.7
(J'

(1.62)

define la relacin bsica entre las magnitudes: <l> (flujo), .7 (f.m.m.) y tI? (reluctancia). Si se considera el circuito magntico de la Figura 1.19,donde se muestra una bobina de N espiras, de resistencia elctrica total R, arrollada sobre un ncleo de seccin uniforme S y longitud magntica media e, al aplicar una tensin de alimentacin de c.c. a la bobina, se producir, de acuerdo con la ley de Ohm, una corriente 1 = V/R, que dar lugar a una f.m.m. ,7= Ni, Yque segn sea el valor de la reluctancia del circuito magntico determinar el flujo resultante <l> = /T/(/{ En la Figura 1.20 se muestra la sucesin de efectos que tiene lugar. Est claro que cuando la bobina se alimenta con una excitacin de c.c., la corriente es funcin directa de la tensin aplicada, pero es absolutamente independiente de la naturaleza y caractersticas magnticas del material que constituye el ncleo. Si, para ser ms explcitos, se considera el sistema magntico mostrado en la Figura 1.19 y se aumenta la reluctancia del circuito magntico (un procedimiento sera, por ejemplo, practicar un entrehierro en el ncleo), entonces el flujo magntico se reducir pero no habr cambio en la corriente absorbida por la bobina. Supngase ahora que la bobina de la Figura 1.19 se alimenta con una tensin de c.a. senoidal:
v(t) =

J2 V cos cot

(1.63)

donde V expresa el valor eficaz de la tensin alterna aplicada y w = 2nf la pulsacin de la misma. En este caso se producir una corriente de circulacin (t) que provocar un flujo <l>(t) en el ncleo. Este flujo variable dar lugar a una f.e.m. inducida en la bobina, de tal

Figura 1.19. Bobina con ncleo de hierro.

CAPTULO

/.

CIRCUITOS

MAGNTICOS

Y CONVERSIN

DE ENERGA

31

Figura 1.20. Sucesin de efectos en una bobina alimentada con c.c.

modo que si se aplica el segundo lema de Kirchhoff al circuito elctrico de la Figura 1.19 se cumplir de acuerdo con (1.33):
v=Ri +N-

d<D dt

(1.64)

Suponiendo que la cada de tensin en la resistencia de la bobina es pequea en comparacin con la f.e.m. inducida, la ecuacin (l.64) se puede escribir:
v=N-

d<D dt

(1.65)

de donde se deduce el valor del flujo <D(t):


<D(l)

= 2_
N

Iv . dt = j2 Nw

V sen ou

(1.66)

La constante de integracin es nula siempre que se considere que en t = O no existe magnetismo remanente en el ncleo. La ecuacin (1.66) puede escribirse en la forma clsica: <D(t) = <l>m sen tot = <Dm cos (wt - 90) donde el flujo mximo <Dmvale: j2v <Dm=-Nw y teniendo en cuenta que w = 2nf, la relacin (1.68) se puede escribir: 2n v= j2fN<Dm=4,44fN<Dm (1.69) (1.68) (1.67)

Debe destacarse que en la ecuacin anterior la tensin est expresada en valor eficaz, mientras que el flujo est definido por su valor mximo. Otro hecho a destacar, comparando (1.63) Y(1.67), es que el flujo se retrasa 90 respecto a la tensin aplicada a la bobina. Sin embargo, lo ms importante que hay que resaltar aqu es que la tensin de alimentacin y su frecuencia imponen el valor que va a tener el flujo en el ncleo [de acuerdo con la ecuacin (1.69)], por lo que segn sea el valor de la reluctancia del circuito magntico se tendr, de acuerdo con la ley de Hopkinson, una corriente absorbida por la bobina. En la Figura 1.21 se muestra la sucesin de efectos que tiene lugar. Debe quedar claro, por tanto, que cuando la bobina se alimenta con una excitacin de c.a., el flujo es funcin directa de la magnitud y frecuencia de la tensin aplicada, pero es absolutamente independiente de la naturaleza y caractersticas magnticas del material que constituye el ncleo.

32

MQUINAS ELCTRICAS

V=4,44fN$m

iT=NI

Figura 1.21. Sucesinde efectosen una bobinaalimentadacon c.c. De un modo anlogo al estudiado anteriormente, si se considera el sistema magntico mostrado en la Figura 1.19,en el que ahora se alimenta la bobina con c.a. y se aumenta la reluctancia del circuito magntico(por haber practicado,por ejemplo,un entrehierroen el ncleo),entonces no habr ninguna modificacinen el flujo magntico,pero la bobina absorberms corriente de la red para poder mantener el flujo constante en el valor que le impone la tensin aplicada. Es importante que el lector distinga a la perfeccin el comportamiento de una bobina con ncleo de hierro segn se alimente con c.c. o con c.a., ya que le permitir comprender ms fcilmente el funcionamiento de las mquinas elctricas y de muchos dispositivos electromagnticos.
Nota prctica:

1. Si se dispone de un catlogo de caractersticas tcnicas de contactores (vase en el epgrafe 1.7 el concepto de contactor, que es en definitiva un electroimn alimentado con c.a.), se observar que la potencia o corriente absorbida en el momento de la conexin es muy superior a la que consume en rgimen permanente, es decir, al cabo de un cierto tiempo en el que se produce el cierre de la armadura mvil sobre la fija. Este efecto se debe a que inicialmente la reluctancia del circuito magntico es elevada, ya que la armadura mvil est separada de la fija por un gran entrehierro de aire; sin embargo, la corriente de mantenimiento o permanente es reducida debido a que en esta situacin la armadura mvil queda pegada sobre la fija (entrehierro despreciable). Una avera relativamente frecuente en los contactores utilizados en las maniobras de los equipos elctricos de obras (p. ej., en los motores de gras, montacargas, hormigoneras, etc.) es cuando se introduce en el entrehierro del contactor alguna partcula de grava que impide el cierre completo del contactor, por lo que la corriente absorbida por la bobina del mismo es elevada, lo que provocar la destruccin de la bobina por calentamiento. 2. Si una mquina elctrica de c.a. se conecta por error a una tensin ms elevada que la nominal (p. ej., si es de 220 V Y se conecta a 380 V), de acuerdo con (1.69) se producir un flujo veces (que es el cociente 380/220) el nominal; como quiera que el material magntico suele disearse por el constructor en el codo de la curva de imanacin (que corresponde a 1,401,5 teslas para la chapa magntica de la Fig. 1.3), se observa en esta figura que el campo magntico H necesario, que es proporcional a la corriente absorbida, aumenta enormemente siguiendo un valor asinttico a la curva de imanacin. Es por ello que el devanado se quemar en muy poco tiempo. 3. Si se desmonta un motor elctrico de c.a. y se separa el rotor del esttor (por ejemplo, cuando se rebobinan motores elctricos), debe tenerse sumo cuidado de no aplicar como tensin de prueba del esttor su valor nominal, porque entonces se quemar el devanado, lo que se justifica por el hecho de que al quitar el rotor, la reluctancia del motor es muy elevada y el devanado absorber una gran corriente de la red. Debe probarse el bobinado con tensin reducida del orden de 1/5 a 1/10 de la nominal.

J3

CAPTULO

l.

CIRCUITOS MAGNTICOS y CONVERSIN DE ENERGA

33

1.6.2. Circuito elctrico equivalente de una bobina con ncleo de hierro alimentada con e.a,
Acabamos de demostrar en el epgrafe anterior que una bobina alimentada con c.a. da lugar a un flujo funcin de la magnitud y frecuencia de la tensin y que adems, segn las expresiones (1.63) y (1.67), el flujo se retrasa de la tensin un ngulo de 90. En la Figura 1.22 se muestran los fasores correspondientes, habiendo tomado la tensin como referencia. Es evidente, segn se ha demostrado, que si el flujo en el ncleo es independiente de la naturaleza del material magntico, los efectos de saturacin, histresis, etc., debern tener alguna influencia en la corriente absorbida. Nuestro objetivo ahora es intentar buscar las relaciones analticas que unen a la tensin con la corriente para obtener circuitos elctricos equivalentes que permitan analizar un sistema magntico excitado con c.a. con todo el potencial que nos presenta la teora de circuitos. En un principio, para facilitar los clculos, se va a considerar que el circuito magntico es lineal, lo que equivale a suponer que el sistema tiene una permeabilidad constante. En el epgrafe 1.6.3 se ampliarn los conceptos correspondientes para tener en cuenta la no linealidad que presenta la curva de imanacin de un material ferromagntico real, Para determinar los circuitos equivalentes de una bobina con ncleo de hierro es preciso considerar dos situaciones: a) que el ncleo no tenga prdidas en el hierro y b) que el ncleo tenga prdidas.

a) Ncleo sin prdidas


Si consideramos que el ncleo magntico no tiene prdidas y suponemos tambin despreciable la resistencia de la bobina, en esta situacin la potencia activa absorbida (por la bobina) de la red ser nula. De acuerdo con la ley de Hopkinson, se tendr:
<J>

,ry!/'

: Niexc = -f- = .t -f- S


.tS

(l.70)

donde se ha llamado i.; a la corriente de excitacin instantnea que circula por el devanado, f a la longitud magntica media, S a la seccin transversal del ncleo de la Figura 1.19 y .t la permeabilidad, que suponemos constante. Teniendo en cuenta la relacin (1.65) se puede escribir:
v=N-=----

d<J> .tN2S diexc dt f dt

(l.71)

Q-y-------.v

Figura 1.22. Fasoresde tensiny flujomagntico.

34

MQUINAS ELCTRICAS

que comparando con la tensin en una bobina de coeficiente de autoinduccin L, llevando una corriente i exc:
diexc v=Ldt

(1.72)

indica que L viene expresado por: ( 1.73) lo cual quiere decir que el circuito equivalente de una bobina con ncleo de hierro (Fig. 1.23a) puede representarse por una autoinduccin (Fig. 1.23b) cuya magnitud se expresa por (1.73), resultando el diagrama fasorial de la Figura 1.23c, donde se observa que lexc va en fase con el flujo como indica (1.71) o (1.72), 10 que est de acuerdo con el concepto de que la potencia activa absorbida es nula (por no existir prdidas en el sistema). b)

Ncleo con prdidas

En el caso de que el ncleo tenga prdidas en el hierro, la corriente de excitacin Iexc no formar 90 con la tensin, ya que la potencia activa absorbida de la red debe vencer esas prdidas, de tal forma que si denominamos q>v al ngulo que forman V e lexc Y PFe a las prdidas en el hierro, se cumplir: (1.74) y el diagrama fasorial del sistema ser el indicado en la Figura 1.24, donde puede observarse que lexc tiene dos componentes, una IFe llamada componente de prdidas en el hierro y otra I~ llamada corriente magnetizante, que vienen expresadas por: (l.75) La identidad vectorial (1.75c) y el diagrama fasorial de la Figura 1.24 permite obtener el llamado circuito equivalente de una bobina con ncleo de hierro, indicado en la Figura 1.25b. En el nudo A de este circuito vemos que se cumple la ecuacin (1.75c). La corriente IFe debe pasar por una resistencia RFe, denominada resistencia de prdidas en el hierro, ya que segn indica la Figura 1.24, IFe va en fase con la tensin (circuito resistivo), mientras que la corriente lit debe pasar por una reactancia X~, llamada reactancia magnetizani exc(t)

PFe=

---+

+
ver) v(t) L
<1>

a)

b)

e)

Figura 1.23. Circuito equivalente de una bobina con ncleo de hierro sin prdidas.

CAPTULO

1.

CIRCUITOS MAGNTICOS Y CONVERSIN DE ENERGA

35

Figura 1.24. Diagrama fasorial de una bobina con ncleo real.

te, pues de la Figura 1.24 se observa que /1' se retrasa 90 respecto de la tensin (circuito inductivo). Los valores de RFe Y\, sern: RFe

=-

XII

= -/

V
11

(1.76)

/Fe

Las prdidas RFJie indicarn las prdidas en el ncleo del sistema magntico de la Figura 1.25a, mientras que la corriente /11 expresa, al igual que en el caso del ncleo sin prdidas, la corriente necesaria para magnetizar el material.

1.6.3. Corriente de excitacin en una bobina con ncleo de hierro alimentada con c.a.
En el epgrafe anterior, la determinacin de la corriente de excitacin se ha realizado suponiendo un circuito magntico lineal, de permeabilidad constante, lo que ha permitido obtener expresiones simples que relacionan la tensin con la corriente, o el flujo con la corriente. De hecho la linealidad implica que si la tensin aplicada es senoidal, son tambin senoidales las formas de onda de flujos y corrientes. En la prctica, la curva de imanacin de un material ferromagntico es no lineal y de hecho el punto de trabajo normal en las mquinas elctricas est en el codo de la curva de magnetizacin del material, lo que ejerce gran influencia en la forma de la curva de la corriente de excitacin, que va a dejar de ser senoidal y teniendo que recurrir para su determinacin a soluciones grficas por ser imposible utilizar tcnicas analtiIexc

--+

+
V
V(I)

RFe

IFe b)

XII

= Lw

a)

Figura 1.25. Circuito equivalente de una bobina con ncleo de hierro con prdidas.

36

MQUINAS ELCTRICAS

casoDe un modo anlogo al efectuado en 1.6.2, se va a determinar la forma de la corriente de excitacin, considerando dos casos: a) Ncleo sin prdidas, b) Ncleo con prdidas.

a) Ncleo sin prdidas


La relacin en este caso, entre el flujo Il> y la corriente de excitacin lexc' se obtiene grficamente de la curva de magnetizacin del material, donde en vez de emplear, como se indicaba en la Figura 1.3, el eje de ordenadas para inducciones B, se utiliza la magnitud proporcional Il> = BS, y donde en el eje de abscisas se empleaba H = Nlej se emplea ahora lexc' En la Figura 1.26 se muestra este clculo grfico: en la a) se muestra la curva de magnetizacin del materialll> = f(iex)' en la Figura 1.26b se observa la forma senoidal de la tensin aplicada y la del flujo retrasado 90 respecto a V, como requiere la ecuacin (1.71). Al punto A de la curva b) de flujo le corresponde el punto A' en la curva de corrientes en virtud de la correspondencia Il>-iexc; al punto B de la curva de flujo le corresponde Bien la de corriente, y as sucesivamente hasta obtener la forma completa de la curva de la corriente de excitacin del ncleo. Se observa que la forma no es senoidal y por desarrollo en serie de Fourier puede demostrarse que aparecen armnicos impares: 1, 3, 5, etc. En la Figura 1.27a se muestra la curva acampanada de la corriente, que aparece como suma de una onda fundamental y un tercer armnico. En la Figura 1.27b se muestra la curva v(t) y la corriente iexc(t), que van desfasadas 90.

ver)

<1>(t)

Perodo T

a) b)

Primer armnico de la corriente de excitacin

G'
e)

Figura 1.26. Deformacin de la corriente de excitacin de una bobina con ncleo sin prdidas.

CAPTULO J.

CIRCUITOS MAGNTICOS Y CONVERSIN DE ENERGA

37

v(t) l~r armnico

o
a)

3.erarmnico

b) b)

Figura 1.27.

a)

Corriente de vaco y sus armnicos.

Ondas de tensin y corriente.

b) Ncleo con prdidas


Suponiendo que el ncleo tenga nicamente prdidas por histresis, se obtiene la composicin grfica de la Figura 1.28, donde se ha superpuesto la curva i.: con la del flujo para observar que aparte de la deformacin de la curva de vaco de la corriente, sta va desfasada del flujo debido a las prdidas ncleo. Puede demostrarse que la existencia de las prdidas por corrientes de Foucault hace que se ensanche ms el ciclo de prdidas obligando a un nuevo desfase de las curvas de t.: y <1>, lo cual est en correspondencia con el diagrama vectorial de la Figura 1.24, donde en la cons-

de tensin v(t)

<lI(t)

: ~

,. I !

iexc:(tA

Figura 1.28. Deformacin de la corriente de excitacin de una bobina con ncleo con prdidas.

38

MQUINAS ELCTRICAS

truccin se ha supuesto que iexc es una senoide equivalente a la curva de vaco real, indicando con ello una onda sinusoidal que al circular por la bobina de excitacin produzca las mismas prdidas que la corriente real. Es necesario puntualizar que la representacin fasorial es solamente vlida para dibujar magnitudes senoidales y por ello el diagrama fasorial de la Figura 1.24 y el circuito equivalente correspondiente a que da lugar (Fig. 1.25b) es correcto cuando se considera el circuito magntico lineal. En el caso de tener en cuenta la no linealidad, la corriente de excitacin de la Figura 1.24 representara, como se acaba de indicar, una onda senoidal equivalente, que tendra un valor eficaz igual a la raz cuadrada de la suma de los cuadrados de los valores eficaces de la corriente fundamental y sus armnicos. EJEMPLO DE APLICACIN 1.7 Considerar el ncleo magntico de la Figura 1.19, donde la longitud de la trayectoria magntica media es de 50 cm y la seccin del ncleo es de 10 cm'. El nmero de espiras es 300 y la tensin eficaz aplicada es 150/ voltios. La resistencia de la bobina se supone despreciable y la curva de magnetizacin del material responde a la expresin:

fi

l 8 10-2 H B = ; + 10-2 H

B: Teslas; H: Av/m

Calcular: a) Las corrientes i.; 1" e t.; y el ngulo de desfase ({J". b) Parmetros RFe Y X~ del circuito equivalente de la bobina. Datos: La frecuencia de la tensin es de 50 Hz y las prdidas en el hierro con la tensin aplicada son de 20 W. SOLUCIN
a)

De acuerdo con la expresin (1.69), el valor del flujo mximo es: <D =
m

v
4,44 fN

150/)2' = 1 59 .10-3 Wb 4,44' 50300 '

que corresponde a una densidad de flujo Bm:


B = -"'- = '
111

<D S

1 59 10-3 = 1 59 Teslas 10'10-4 '

que llevando a la curva de imanacin del material, se obtiene:


l 8 10-2 H Bm = 1,59 = 1'+ 10-2 H:,~
=>

H", = 757 Av/m

Suponiendo que la curva de Hm fuera sinusoidal, el valor eficaz de H sera:


H

n;

.j:

::; =

757 2

= 535,28 Av/m

Ycomo H = N1/f, quedara: 1" = N =

He

535,28' 0,5 300 = 0,9 A

CAPTULO 1.

CIRCUITOS MAGNTICOS Y CONVERSIN DE ENERGA

39

Por otra parte, las prdidas en el hierro son de 20 W y de acuerdo con (1.74) se tiene: 150 20= J2/exc.COSepv=

J"2/

150

Fe

=>

/Fe=0,19A

De acuerdo con el diagrama fasorial de la Figura 1.24,se cumple:

/exc = JI;e + /~ = )0,192 + 0,92 = 0,92 A


=>

=>

0,19 cos ep. = - = -- = 021 " /1' 0,92 '

i:

=>

epI"= 78,08

b)

Los valores de RFe YXp pueden obtenerse de las ecuaciones (l.76):


R
Fe

V =-=
/Fe

150/)2 =55824Q 0,19 '

V 150/)2 . X =-= , l' /1' 0,9

117,85Q

1.7. CONVERSIN DE ENERGA EN SISTEMAS MAGNTICOS CON MOVIMIENTO DE TRASLACIN. ELECTROIMANES


En el epgrafe 1.4 se ha demostrado que un campo magntico almacena energa; por otro lado, un campo magntico tambin ejerce fuerzas mecnicas en las estructuras o partes de las estructuras asociadas con l. Estas dos propiedades hacen que se utilice el campo magntico como un enlace eficaz entre las partes elctricas y mecnicas de muchos dispositivos electromecnicos y en particular de las mquinas elctricas que son convertidores electromecnicos de la energa. En este epgrafe y en el siguiente se van a explicar los principios bsicos de la conversin electromecnica de la energa, cuyo fundamento, como sistema fsico que es, est basado en el principio general de la conservacin de la energa. Nuestro objetivo va a ser aplicar esta ley a sistemas que utilizan el campo magntico como medio de enlace en la conversin. Considrese, para iniciar nuestro estudio, el sistema magntico dotado de movimiento de traslacin indicado en la Figura 1.29,que tiene un devanado de excitacin de N espiras arrolladas sobre una armadura fija.
x

Armadura fija

Figura 1.29. Sistema electromecnico de traslacin.

40

MQUINAS ELCTRICAS

Existe una armadura mvil que tiene un solo grado de libertad (su movimiento se restringe al plano horizontal). Cuando circula una corriente por la bobina de excitacin, se establece un flujo magntico en el ncleo que provoca una fuerza de atraccin sobre la armadura mvil, 10 que reduce el entrehierro central, con la consiguiente disminucin en la reluctancia del circuito magntico y la variacin subsiguiente en la energa magntica almacenada. En la Figura 1.29 se han sealado los sentidos positivos de referencia, tanto en lo que concierne al terminal elctrico, tensin y corriente, como al terminal mecnico, fuerza y desplazamiento. Si la armadura mvil se desliza hacia la izquierda desde una posicin inicial x = x hasta una posicin final Xz = x - dx, el principio de conservacin de la energa aplicado al sistema nos dar la ecuacin: .
dWe = dWm + dWmec + d~

(1.77)

Yen la expresin anterior se ha denominado:


dW.: cambio en la energa elctrica absorbida de la red. dW;lI: cambio en la energa magntica almacenada en el sistema. dWmec: cambio en la energa mecnica debida al movimiento de la armadura mvil. d~J: cambio en la energa perdida.

La ecuacin anterior representa la ley de conservacin de la energa aplicada a un sistema motor, es decir, a una transformacin de energa elctrica en mecnica. En el supuesto de considerar despreciables las prdidas en el sistema, la ecuacin (1.77) se transforma en: (1.78) Las prdidas inherentes al sistema son: a) prdidas por efecto Joule en la bobina, b) prdidas en el hierro en el ncleo ferromagntico: histresis y corrientes de Foucault, y e) prdidas mecnicas de rozamiento de la armadura mvil. Vamos a analizar a continuacin cada uno de los trminos mostrados en la ecuacin (1.78). Comenzamos con la energa magntica. En la Figura 1.30 se han representado las dos curvas de imanacin del sistema magntico de la Figura 1.29, correspondientes a las dos posiciones de la armadura mvil: x = x y x2 = X - dx. Es evidente que existe una curva de imanacin para cada una de las posiciones de la armadura mvil, o de otro modo, que la curva de imanacin depende de la distancia x. Si se considera, por ejemplo, un flujo constante, definido por <1> en la Figura 1.30,para la posicin inicial x = x se necesita una f.m.m.. 9;", mientras que para la posicin final x2 = x - dx se
Menor Mayor

Figura 1.30. Energas en un sistema electromecnico no lineal. Funcionamiento a flujo constante.

CAPTULO l.

CIRCUITOS MAGNTICOS Y CONVERSIN DE ENERGA

41

necesita una f.m.m. ,~, que es menor que 9';", ya que corresponde a un menor entrehierro, es decir, a una reluctancia menor, lo que est de acuerdo con la ley de Hopkinson:
<1>

:7 =. /7" = <I>.9'
9[. '

(1.79)

Se observa en la ecuacin anterior que si <1> es constante, a menor reluctancia corresponde menor f.m.m. Se puede razonar tambin de otro modo: si se considera una f.m.m. constante el flujo es tanto mayor cuanto menor es la reluctancia del circuito magntico (menor entrehieITa). De ah que en las curvas de imanacin de la Figura 1.30, la curva ms alta corresponda a un menor entrehierro (es decir, la armadura mvil se ha acercado a la armadura fija). De acuerdo con el epigrafe lA (vase Fig. 1.15 y ecuacin 1.37), si suponemos una situacin inicial con un entrehierro x] = x y un flujo en el ncleo <1>] (punto a de la Fig. 1.30), la energa magntica almacenada vendr expresada por el rea oaco, Cuando se mueve la armadura mvil, la posicin de sta cambia desde XI a x2 La localizacin del nuevo punto de trabajo (nuevoestado) del sistema depende de cmo se ha efectuado el cambio de x] a x2 Existen dos formas bsicas en la prctica de la ingeniera elctrica que tienen inters: a)

El movimiento se realiza a flujo constante


considerando que la tiempo a cambiar al de la red ser cero. alctrica absorbida

Esta situacin se logra ajustando la corriente durante el movimiento, o traslacin del ncleo es suficientemente rpida como para que no le d flujo durante la transicin. En este caso la energa elctrica absorbida Tngase en cuenta en la Figura 1.29 y ecuacin (1.78) que la energa durante la traslacin vale: dWe

= v i dt = N -

d<l> dt

i dt

=N

i d<l>

(1.80)

y al no existir variacin de flujo durante el movimiento indicar que dWe = 0, es decir, no hay aportacin de energa elctrica. Por consiguiente, la ecuacin (1.78) se transformar en:
(1.81 ) es decir: (1.82) lo que se expresa diciendo que el trabajo mecnico se realiza a expensas de la reduccin en la energa magntica almacenada. Si el flujo permanece constante en la transicin de la armadura mvil, el nuevo estado de equilibriocorresponder en la Figura 1.30 al punto b, para el cual la energa magntica almacenadaen este estado final viene expresada por el rea obco, De este modo se ha producido unareduccin de la energa magntica durante la traslacin, que viene expresada en la Figura 1.30por el rea rayada oabo y que teniendo en cuenta (1.82) ser igual al trabajo mecnicodesarrollado. Si la fuerza de atraccin se designa por f, el trabajo mecnico producido valdr! . dx, por lo que la expresin de la fuerza en funcin de la energa magntica almacenada,teniendo en cuenta (1.82), ser:

awm] I=> [ -

ax

(1.83)
<I>=cte

42

MQUINAS ELCTRICAS

En la ecuacin anterior la derivada parcial indica que al depender la energa magntica almacenada del espesor del entrehierro y de otras variables, la derivacin debe hacerse respecto a x, considerando constante el flujo. De acuerdo con (1.83), la fuerza mecnica sobre la armadura mvil tiende a reducir la energa almacenada en el circuito magntico, y como quiera que sta se reduce cuando disminuye el entrehierro, el sentido de la fuerza que se ejerce sobre la armadura mvil de la Figura 1.29 es siempre de atraccin. Si las curvas de imanacin de la Figura 1.30 son lneas rectas (lo que ocurre en la prctica cuando los entrehierros son grandes), entonces, de acuerdo con (1.43), la expresin de la energa magntica almacenada ser:
W = - (I? <1>2 m 2'

(1.84)

por lo que segn (1.83) dar lugar a la fuerza:


f= --

d9' <1>2 _' 2 dx 1

(1.85)

De acuerdo con esta expresin, lafuerza sobre la armadura mvil tendr el sentido de reducir la reluctancia del circuito magntico. No habr fuerza en otras direcciones en las que el movimiento no produzca cambio en la reluctancia magntica.

b) El movimiento se realiza con corriente constante


Esta situacin se produce en la prctica si el movimiento de la armadura mvil es suficientemente lento. Si se parte de la posicin inicial mostrada en la Figura 1.31 por el punto a, definido por la f.m.m. ,7;" y flujo <1>]' el nuevo estado de equilibrio (si se mantiene la corriente constante o, lo que es lo mismo, si es constante la f.m.m.) corresponder al punto e, para el cual el flujo tiene un valor <1>2' Al existir un cambio de flujo en el sistema, existir, de acuerdo con (1.80), un cambio en la energa elctrica de entrada a la bobina durante la transicin. Para poder determinar el sentido geomtrico de la energa mecnica desarrollada en esta situacin
Menor entrehierro ~ __ x2=x-dx Mayor

d ,7

Figura 1.31. Energas en un sistema electromecnico no lineal. Funcionamiento a corriente constante,

CAPTULO

l.

CIRCUITOS

MAGNTICOS

Y CONVERSIN

DE ENERGA

43

es convenienteemplear el concepto de coenerga definido en el epgrafe 1.4. Tngase en cuentaque el principio de conservacin de la energa expresado en (1.78) nos da:
dWe ;;: d"-":lI + dWmec ;;: N i dCD ;;: .7 dCD

(1.86)

dondeel ltimotrmino representa, segn (1.80), el cambio en la energa elctrica de entrada. Comoquiera adems que de acuerdo con la Figura 1.15 se cumple: (1.87) dondeW~,representa la coenerga magntica, al diferenciar la ecuacin anterior resultar:
dWm + dW;', ;;: .:dCD + CD d.7

(1.88)

Yllevandoel valor de dWm de (1.88) a (1.86) se obtiene:


(.7 dCD + CD d.7 - dW;'') + dW;lIec = .7dCD

(1.89)

Simplificando la ecuacin anterior y teniendo en cuenta que d.? = Oen la transicin (debido a quela f.m.m.se mantiene constante), resulta: (1.90) loqueindicaque el trabajo mecnico se realiza ahora a expensas del aumento en la coenerga magntica almacenada. En el caso de la Figura 1.31, Y teniendo en cuenta el significado geomtrico de la coenerga mostrado en la Figura 1.15, la coenerga inicial corresponde al reaodao, mientras que la coenerga final corresponder al rea odeo, por lo que el cambioen la coenerga (valor final menos el inicial) vendr expresado por el rea rayada oaeo y que segn (1.90) define tambin el trabajo mecnico desarrollado en el movimiento. Como quieraque el trabajo anterior es igual af . dx, resultar una expresin para la fuerza:

CW~J I= + [ -",OX
m

(1.91)

;=cte

Sise considerael sistema lineal, la coenerga vendr expresada segn (1.43) por:
1 ,72 W' =--

2 y?

(1.92)

y al llevar(1.92) a (1.91) resulta:

f="2]

d
dx

(1Cj?) ;;:"2''/1

~O

di/'
dx

( 1.93)

dondez" = 1/(i( expresa la denominada permeancia del circuito magntico, anloga a la conductancia de los circuitos elctricos. A veces es ms interesante expresar la ecuacin anterior en funcinde la inductancia del circuito. Si se tiene en cuenta entonces (1.48), resultar: 1 ., dL i= --2
d.x

(1.94)

ecuacin quees ms til desde el punto de vista de la teora de circuitos y que significa que la fuerza tiende a incrementar el valor de la inductancia L. Esta expresin se conoce en la bibliografa francesa como frmula de Picou.

44

MQUINAS ELCTRICAS

En la mayora de las situaciones prcticas se suele considerar que se trabaja con sistemas lineales, por lo que las expresiones (1.85), (1.93) Y(1.94) son equivalentes y por consiguiente redundantes. El sentido de la fuerza corresponde en cada caso a reducir la reluctancia (1.94) si la corriente es constante. El sentido es siempre de atraccin. La aplicacin ms importante de la fuerza magntica en la ingeniera elctrica est en los electroimanes. Para grandes potencias los electroimanes se emplean para levantar vigas de hierro, viruta, chatarra, etc.; en otros casos, al actuar sobre unas zapatas se pueden emplear como frenos elctricos, embragues, electroimanes, etc. En potencias menores los electroimanes constituyen la base de los rels y contactores, en los que la corriente en una bobina hace que se produzca una atraccin sobre una armadura mvil en oposicin a la fuerza antagonista de un muelle. Los rels se utilizan en instalaciones de semforos, en sistemas de control automtico y se han empleado hasta fechas muy recientes en las centrales telefnicas tipo rotary (hoy da estas centrales funcionan mediante sistemas digitales controlados por ordenador). Cuando el rel permite activar una carga trifsica se denomina contactor, y este dispositivo es la base de los automatismos para el control de motores elctricos y otros tipos de instalaciones. La gran ventaja de los rels y contactores estriba en que actuando sobre las pequeas corrientes absorbidas por las bobinas de su circuito magntico se pueden controlar mediante el cierre o la apertura de su armadura mvil otros circuitos que consumen intensidades mayores (accin de rele-vo). En la Figura 1.32 se muestra el esquema bsico de un rel. Un aspecto importante a considerar en los electroimanes (rels, contactares, etc.) es el tipo de alimentacin a la bobina, que puede hacerse con c.c. o c.a. Supngase el circuito . magntico de la Figura 1.32, en el que se desprecia la reluctancia del hierro frente a la del entrehierro. En el supuesto de que el movimiento de la armadura se realice a flujo constante, la fuerza magntica, de acuerdo con (1.85), tendr la siguiente expresin:
f=--$

1 2

d9l dx

(1.95)

lo que significa que la fuerza tiende a reducir la reluctancia del circuito magntico. Si se denomina x el espesor del entrehierro, S la seccin del mismo y ..to la permeabilidad del aire, la reluctancia del sistema magntico que se limita a la reluctancia del entrehierro valdr: 9&=..taS x

(1.96)

Armadura mvil

Armadura fija

Figura 1.32. Principio de funcionamiento de un electroimn.

CAPTULO l.

CIRCUITOS MAGNTICOS Y CONVERSIN DE ENERGA

45

y al sustituiren (1.95) resulta una expresin para la fuerza: 1


2

f=--cD

floS

(1.97)

Comose ha indicado antes, el significado del signo - (menos) es que la fuerza tiende a reducirel entrehierro y se produce en cada uno de los posibles entrehierros que tenga el circuito. Es evidenteen la expresin anterior que si la bobina se alimenta con c.c. el flujo tendr unvalorindependientedel tiempo, lo que provocar una fuerza, segn (1.97), que no depender deltiempo.Ahorabien, si la bobina se alimenta con c.a., el flujo ser alterno y en consecuencia lafuerzadependerdel tiempo. Si se parte, por ejemplo, de un flujo de la forma: cD = cDm sen cot Lafuerzade atraccin se obtiene al sustituir (1.98) en (1.97), resultando ser:
f( t )

(1.98)

= --cD,~

2J1oS

sen2 tt

= --cD,;,

(1

4J1oS

cos 2) cot

(1.99)

Enla Figura 1.33 se ha representado la evolucin con el tiempo del flujo y de la fuerza instantnea. Como indica la expresin (1.99), la fuerza electromagntica en un electroimn alimentado por c.a. tiene una frecuencia doble que la de alimentacin pasando por un valor ceroa otrofmx' Como quiera que, en general, la armadura de los mecanismos electromagnticosestconstantementesometida a la accin de la fuerza antagonista de un muelle fanl' o al pesodelsistemamvil (vase Fig. 1.34), en los intrvalos de tiempo en los que f(t) < la armadura se separa de los polos, mientras que cuando f(t) > fanl' la armadura mvil queda atrada por la fija. Este hecho es un inconveniente y es totalmente inadmisible, ya que provocala vibracinde la armadura del electroimn, deformando los polos y provocando ruidos intensos. El medio ms eficaz para evitar la vibracin de la armadura es colocar unas espiras cortocircuitadas sobre los polos del electroimn, denominadas espiras de sombra. En la Figura 1.34puede observarse la colocacin de una de estas espiras, en un circuito magntico deunsoloentrehierro,y se observa que la fuerza antagonista est producida por el peso mg de laarmadura mvil.

t.:

v(t)

<1>(1)

wt

Figura 1.33. Curva de fuerza magntica en un electroimn alimentado con c.a.

46

MQUINAS ELCTRICAS

Figura 1.34. Electroimn con espira de sombra. Detalle de los flujos.

El flujo <D creado por la bobina del electroimn se divide en dos partes <DI y <D2; el flujo <DI pasa por la espira cortocircuitada e induce una f.e.m. que produce una corriente en la misma, creando su propio flujo <Dec' de tal forma que el flujo total que atraviesa la parte del polo abrazada por la espira (<DI - <DC(.) y el flujo de la parte del polo no abrazada (<D2 + <DeJ estn desfasadas un ngulo CI.. Si estos flujos vienen expresados por:
<Dltotal <D2total

= <DI - <Dec = <Dml cos (ou = <D2 + <Dec = <Dm2 cos iot

+ x) (1.100)

la fuerza resultante en el entrehierro, de acuerdo con (1.99), ser de la forma: (1.101) cuya curva de variacin con el tiempo se indica en la Figura 1.35. Esta fuerza vara entref.nny fmx sin pasar por cero. Sifmn > f."" no existirn vibraciones en la armadura. Normalmente la superficie abrazada por la espira suele variar entre el 75 y el 80 por 100 de la seccin total del polo, de esta forma se minimizan las prdidas por efecto Joule en la espira y se obtiene una fuerza ptima en el entrehierro. Ancdota: Si se quitan las espiras de sombra de un contactor, se notar una fuerte vibracin en el mismo, y si ste se utiliza para la puesta en marcha de un motor, se pueden provocar
1(1)
hOla]

oo

Figura 1.35. Fuerzas componentes en un electroimn con espira de sombra.

CAPTULO J.

CIRCUITOS MAGNTICOS Y CONVERSiN DE ENERGA

47

fuertes corrientes de cierre y apertura en el circuito principal, que normalmente hacen actuar los cortocircuitos fusibles de proteccin. sta era una broma que hacan los antiguos maestros industriales a los peritos o ingenieros tcnicos que se incorporaban al taller elctrico de la empresa y que stos transmitan luego a los nuevos ingenieros. Era desconcertante repasar continuamente el circuito del automatismo del motor y comprobar que era correcto y sin embargo el conjunto funcionaba mal. [La ausencia de las espiras de sombra eran las culpables de tal desaguisado! Qu mala sombra tena la cosa!

EJEMPLO DE APLICACIN 1.8


La Figura 1.36 muestra el circuito magntico de un electroimn cuya bobina tiene 1.000 espiras.La seccin transversal de todas las trayectorias magnticas es de 10 cm', Se desprecia la reluctancia del hierro y la dispersin magntica en el entrehierro. Si se hace circular por la bobina una corriente continua de 10 A, calcular para las separaciones x = 2 cm y x = 1 cm las siguientes magnitudes: 1) Flujo e induccin magntica en el entrehierro; 2) inductanciade la bobina; 3) energa y densidad de energa magntica en el entrehierro; 4)fuerza que acta sobre la armadura mvil; 5) si la armadura mvil se 11}ueve muy lentamentedesde x = 2 cm a x = 1 cm, determinar: a) cambio en la energa magntica almacenada;b) energa elctrica suministrada por lafuente de alimentacin, suponiendo despreciable laresistenciaelctrica de la bobina y el rozamiento de la armadura mvil; c) trabajo mecnico realizado,comprobando el balance energtico del sistema; 6) contestar a la pregunta 5 si se suponeque el movimiento de la armadura mvil es lo suficientemente rpido para que el flujo total no cambie durante la traslacin.

SOLUCIN 1. La reluctancia del circuito magntico se limita a la reluctancia del entrehierro, cuyo
valor para cada espesor del mismo es: x = 2 cm x = 1 cm
=>
(/(1

x =~ =
poS xl,

2 10-2 4n . 10- . 10-' 10-2 . 10- . 10-'


7 3 7 3

= 15,92 . 10 A.vfWb

=>

.' ..rJ?o -

=~ = poS 4n

= 7,96'

10 A.vfWb

Armadura fija

Figura 1.36.

48

MQUINAS ELCTRICAS

La f.m.m. aplicada a la bobina es ;/' pondientes sern: .


<1> = -.- = -------:

= Ni = 10
. <1>

A.v, por lo que los flujos corres-

.7

104

91.[

15,92' 106

628
,

10-4 Wb

== 7,96. .9[2

.7

104

106

= 1256 '

10-4Wb

que corresponden a unas inducciones (B = <1>/S):

B
2.

= 0,628

Teslas

; B2

1,256 Teslas

La inductancia de la bobina es, segn (lA 7): L=-=-(j'

N2

N2 !loS
X

que al sustituir valores da lugar en cada caso a unas inductancias:

L = 0,0628 Henrios
3.

; L2 = 0,1256 Henrios

La energa magntica es, segn (lA8), igual a:

que teniendo en cuenta el apartado anterior corresponde a los valores: Wml

= 3,14

Julios

; Wm2

= 6,28

Julios

Como quiera que el volumen del entrehiero es en cada caso: VI = Sx = 10-3 2 . 10-2 = 2 . 10-5 rrr'

; V2 = SX2 = 10-3 1 . 10-2 = 1 . 10-5 nr'

se obtienen unas densidades de energa magntica (energa por unidad de volumen):

wm

= ~V

= _'2 10-5

314

1 57 105 J/m3
'

; wm2 = _W_m2
V2

= _6_,2_8_= 628.
1 . 10-5 '

105 J/m3

4.

cuyos valores pueden obtenerse tambin aplicando la ecuacin (lA4), como puede comprobar fcilmente el lector. De acuerdo con (1.94), se tiene:

1 .2 dL 1=-1 2

dx

y teniendo en cuenta la expresin de la inductancia determinada en el apartado 2 resulta: dL dx ---=-X

N2!loS

L 1 .2 L ~ 1=--1 x 2 x

que para x = 2 cm y x = 1 cm nos da unos valores de la fuerza:

1II1 = - 102--2
2

0,0628

2 10-

= 157 Newton

l i2 1 -- -2

10

0,1256 1 . 10-2

628 Newton

CAPTULO l.

CIRCUITOS MAGNTICOS Y CONVERSIN DE ENERGA

49

5.

El lector puede llegar tambin a los mismos resultados anteriores aplicando la expresin (1.97). a) Si la armadura se mueve muy lentamente, la traslacin se realizar a corriente constante. El cambio en la energa magntica almacenada, de acuerdo con los resultados obtenidos en el apartado 3, ser:

Aw'n
b)

= Wm2 -

Wml

= 6,28

- 3,14 = 3,14 Julios

La energa elctrica suministrada por la fuente vendr expresada por la integracin de (1.86), dando lugar a: We = .7(<1>2 - <1>1)= 104 (12,56' 10-4
-

6,28'

10-4) = 6,28 Julios

e)

El trabajo mecnico desarrollado ser:

w'"ec =
. es decir:

f f
f dx =
1

- -1 2 -dL dx = JI.2
2 dx
t-,

- - 2 dL = - - 2 (Lz - L1) 2 2

Wmec = - IOz (0,1256 - 0,0628) = 3,14 Julios 2 y de este modo se cumple el principio de conservacin de la energa (1.86), que aplicado a este caso nos da: We = AWm + AWmec

6,28 = 3,14 + 3,14 Julios

Se puede dar una interpretacin del trabajo mecnico desarrollado en funcin de la variacin en la coenerga magntica. Tngase en cuenta que segn (1.87) se cumple:

W:', = .7<1> -

w,,,
-

por lo que las coenergas magnticas para cada valor del entrehierro sern: W~I = .7<1>1 - W,,,I = 104. 6,28 . 10-4 W~z = .7<1>2 - Wm2 = 104
.

3,14 = 3,14 Julios 6,28 = 6,28 Julios

12,56 . 10-4

que coinciden con las energas magnticas calculadas en el apartado 3, en virtud de la linealidad del circuito magntico. De este modo el cambio en la coenerga magntica ser: AW~ = W~2 - W~I = 6,28 - 3,14 = 3,14 Julios que coincide con el trabajo mecnico Wmec calculado anteriormente, la condicin (1.90): Wmec = AW:', = 3,14 Julios Es decir, el trabajo mecnico se ha realizado a expensas del aumento en la coenerga magntica almacenada en el sistema. 6. a) Si el movimiento se realiza a flujo constante, la energa magntica almacenada responder a la expresin (1.84) W =-&<1>
m

lo que confirma

1 2

1 x =--<1> 2 110 S

50

MQUINAS ELCTRICAS

donde el flujo es un parmetro constante y que para XI = 2 cm vale, segn el apartado 1:


<l> = <l> I = 6,28 . 10-4 Wb

De este modo las energas magnticas almacenadas para Xl = 2 cm y X2 = 1 cm son:


W
I

=-

1 2 10-2 7 3 (6,28 . 10-4)2 = 3,14 Julios 2 4n: . 10- . 10- .

Wm2 = -

1 1 . 10-2 7 3 (6,28 . 10-4)2 = 1,57 Julios 2 4n: . 10- . 10-

y como consecuencia de ello el cambio en la energa magntica almacenada ser: dW,,, = Wm2
b)
-

Wml = -1,57 Julios

10 que implica una reduccin en la energa magntica almacenada.

Si el flujo es constante, de acuerdo con (1.80) no habr energa elctrica suministrada por la fuente, es decir, We = O. c) El trabajo mecnico desarrollado se obtendr integrando (1.85):
W

mee

= f dx = - <l> 2
1 2

f f
ee

df7t
dx

dx

= .'/1 - <l> 2
I

ff//

2"

d9?

=- 2 - \V

",,2

(9? - 9? )
2 I

es decir:
Wm = - - (6,28' 10-4)2(7,96' 106) = +1,57 Julios

y el principio de conservacin de la energa nos dar:

que sustituyendo valores es:

o = -1,57

+ 1,57 = O

lo que indica que el trabajo mecnico se ha realizado a expensas de la reduccin en la energa magntica almacenada. Es instructivo para el lector que desarrolle los apartados 5 y 6 de este ejemplo, construyendo las Figuras 1.31 y 1.30, respectivamente, para cada situacin. Tngase en cuenta que las relaciones flujo-f.m.m. representadas en estas figuras sern ahora lneas rectas en virtud de considerar la reluctancia del hierro despreciable. EJEMPLO DE APLICACIN 1.9: REL ELECTROMAGNTICO En la Figura 1.37 se muestra el mecanismo de un rel electromagntico. Al aplicar una corriente continua a la bobina, se produce la atraccin de la armadura mvil que cierra los contactos a y a' haciendo funcionar una carga de mayor consumo que la necesaria por la bobina del rel, lo que permite controlar grandes intensidades de cargas por actuacin sobre intensidadespequeas necesarias para la excitacin de la bobina. Si la corriente que circula por la bobina es igual a 20 mA, y se desprecia la j.m.m. necesaria para el hierro, calcular:

CAPTULO

J.

CIRCUITOS MAGNTICOS y CONVERSIN DE ENERGA

51

a) Fuerza y coeficiente de autoinduccin de la bobina cuando el entrehierro x es igual a 3 mm. b) dem cuando x = 3,6 mm. c) Si la resistencia de la bobina es igual a 100 n, calcular la tensin que es necesario aplicar a la misma para mantener constante la corriente, cuando la armadura se mueve entre las dos posiciones (el tiempo necesario para esta traslacin es de 1J ms).

RED
a a'

Armadura mvil

Armadura fija

Figura 1.37.

Rel electromagntico.

SOLUCIN
a)

La fuerza producida a corriente constante ser: f= + y como quiera que: :T=Ni se tendr: f=[Ir

[ OW'] OX
_m

= -o =cte

OX 2

(1 ) = -ox o (1 7.. - _'9i' <1>2 x

2)

2 [/(

bx

b (1-N
2

2.2 1 -

/lOS) =--N
x 2

2 ,2 1 -

/loS
X2

cuando x = 3 mm, y teniendo en cuenta que

S=
se obtiene:

n d2

= -_

n 402

= 1.256 mm" ; Ni = 6,000' 20 . 10-3 = 120 A.v

1 4n . 10-7 . 1.256 . 10-6 f= - 1202 32 = 1,263 Newton

(3' 10- )

52

MQUINAS ELCTRICAS

Como quiera que el coeficiente de autoinduccin de la bobina vale:


<p gT/r7l L=N-=N--=-=N N2
9(,
2

!loS -

se obtiene:
L = 6.0002 b)

4n . 10-7

1.256 10-6
3

3 10-'

= 18,94 Henrios

Aplicando los mismos conceptos que en el apartado anterior se obtiene:


L = 15,783 H ; f= 0,877 Newton

c) La tensin aplicada a la bobina ser: v

= Ri + N -

d<P d = Ri + - (Li) dt dt

= Ri + L -

di dL +idt dt

Como quiera que no hay variacin de la corriente, se tendr: v

= Ri

+ i I1L I1t

1.000 . 20 . 10-3 + 20 . 10-3 18,94 - 15,783 = 25 74 V 11 . 10-3 '

EJEMPLO DE APLICACIN 1.10: CONTACTOR ELECTROMAGNTICO

En la Figura 1.38 se muestra el circuito magntico de un dispositivo electromecnico denominado contactor. El sistema consiste en un ncleo ferromagntico en forma de E, cuya seccin central lleva el devanado de excitacin, y tiene doble superficie que las secciones laterales. Se tiene una pestaa P que limita el espesor del entrehierro a un valor adecuado. Existen unos contactos m y n que se cierran al aplicar a la bobina una excitacin de c.a. dando alimentacin a una carga externa. Considerando las dimensiones indicadas en la Figura 1.38, que el entrehierro es de 0,5 cm y que se aplica a la bobina una tensin del tipo v= 220 cos wt con unafrecuencia de 50 Hz, calcular, en el supuesto de que se desprecie la reluctancia del hierro y que la resistencia elctrica de la bobina es de 5 Q: a) Coeficiente

fi

= 1.500 espiras

RED CARGA

Figura 1.38. Contactor electromagntico.

CAPTULO l.

CIRCUITOS MAGNTICOS Y CONVERSIN DE ENERGA

53

de autoinduccin de la bobina. b) Corriente instantnea que circula por la bobina. c) Expresin instantnea del flujo y de la induccin en el ncleo central. d) Expresin de la fuerza instantneaejercida sobre la armadura mvil. e) Comprobar que el valor del flujo obtenido en el apartado. c) partiendo del estudio magntico del sistema no coincide con el obtenido aplicandola expresin (1.69), a qu se debe esta diferencia?
SOLUCiN
a)

El circuitoequivalente elctrico es el indicado en la Figura 1.39. Las reluctancias de los entrehierros valen:
.{/'

ya que SI = S/2. El flujo en la columna central, teniendo en cuenta el circuito de la Figura 1.39b, ser:
Ni <1>=---f/? + [/( /2 Ni
-

2e

ro

JI

resultando un valor para el coeficiente de autoinduccin de la bobina, dado por: L


b)

= N -i = N2

<1>

J1 S
_0_

1.5002

4n . 10-7 4 . 10-4

2 . 0,5 . 10-2

0,113 H

Para obtener el valor de la corriente, es preciso calcular antes la reactancia e impedancia del circuito elctrico. La reactancia ser:
XL = L(J) = 0,113 . 2n . 50 = 35,53

y como la resistencia de la bobina es igual a 5 Z

= R + jXL = 5 + j35,53 = -Z = 35,88 L


V

n, la impedancia compleja valdr: = 35,88 L 81,99

La corriente que circula por la bobina ser: I 220 L 0 = 6 13 L -81 ,99 81,99'

donde se ha tomado como referencia la tensin aplicada.

<ll/2

<ll/2

!
al Figura 1.39. bl

54

MQUINAS ELCTRICAS

La expresin instantnea de la corriente ser, en consecuencia: i=

,j2 . 6,13 cos

(wt - 81,99)

e) El flujo en la seccin central vale, segn lo calculado en el apartado a): /1oSNi 4n' 10-7. 4 10-4 1.500 6,13 4 <D = -= = 4 62 . 10- Wb 2e 2 . 0,5 . 10-3 ' que corresponde a un valor instantneo: . <D = ,j2 . 4,62 . 10-4 cos (wt - 81,99) = 6,53 . 10-4cos (wt - 81,99) lo que indica una densidad de flujo en la columna central: <D B = - = 1,64 cos (wt - 81,99) Teslas S En las columnas laterales la densidad de flujo es la misma, pues existe un flujo mitad con seccin mitad que en la columna central. La fuerza en cada entrehierro ser de la forma:

d)

f=-que al aplicar a nuestro sistema da:

1 B2S

2 /10

I-> ~ B2S
2 /10

+ 2(~ B2S1)
2 /10

ya que existen dos secciones laterales. Pero como quiera que S, = S/2, resulta una fuerza total:

cuyo valor es: fT = 856,64 cos" (wt - 81,99)


e)

El valor del flujo instantneo, de acuerdo con el apartado

e),

era:

<D = 6,53 . 10-4 cos (wt - 81,99) que corresponde a un valor mximo: <Dm = 6,53 . 10-4Wb Si se aplica directamente (1.69) se obtiene:

v = 4,44 fN<D
<D =
m

siendo V la tensin aplicada eficaz, y de aqu se deduce el valor mximo del flujo: 220 =660,104,44 . 50 . 1.500 '
4

Wb

CAPTULO l.

CIRCUITOS MAGNTICOS y CONVERSIN DE ENERGA

55

que no coincide con el valor anterior. Esto se debe a que la expresin (1.69) es aproximada y que se supona que la resistencia del devanado era despreciable. La expresin correcta es la (1.64), que volvemos a escribir: dcD v = Ri + Ndt o en forma compleja (llamando E al valor complejo de N dcD/dt), queda: E.= V - RI = 220 LO - 5 6,13 L -81,99 = 217,85 L +8,01 o en forma instantnea:

v~

Lo'

dcD 217,85 cos (wt + 8,01) = Ndt

de donde se deduce, integrando: cDm =

j2 . 217,85 sen (wt + 8,010) j2.


wN

217,85 wN

cos (wt - 81,99)


.

y teniendo en cuenta que w = 2nf = 314 rad/s y N = 1.500 espiras, resulta:


cD
m

= j2.

217,85 314. 1.500

= 653
'

10-4 Wb
e).

que coincide exactamente con el valor de flujo obtenido en el apartado

1.8. CONVERSIN DE ENERGA EN SISTEMAS MAGNTICOS CON MOVIMIENTO DE ROTACIN. MQUINAS ELCTRICAS ROTA TIV AS
En el epgrafe anterior se ha analizado la conversin de energa en sistemas magnticos excitadospor una sola fuente y que disponen de una armadura mvil dotada de movimiento de traslacin, lo que ha permitido explicar el funcionamiento de los electroimanes. En general, las mquinas elctricas son dispositivos electromagnticos dotados de movimientode rotacin. Para analizar la conversin de energa en estos sistemas vamos a considerar las dos situaciones que frecuentemente se presentan en la prctica: a) que el sistema magnticodisponga de una sola fuente de alimentacin, b) que el sistema magntico disponga de varias fuentes de alimentacin, normalmente dos.

1.8.1. Sistemas magnticos de rotacin alimentados con una sola fuente. Motores de reluctancia
Considreseel sistema mostrado en la Figura 1.40, que es la versin rotativa del sistema de traslacinmostrado en la Figura 1.29. La parte fija de este convertidor se denomina esttor y la parte mvil recibe el nombre de rotor. Sobreel esttor est arrollada una bobina de N espiras conectada a una red de v(t) voltios. Enla Figura 1.40 se muestran los respectivos ejes magnticos (ejes de simetra) del esttor y delrotor, que forman un ngulo entre s, De un modo anlogo al estudiado en el epgrafe

56

MQUINAS ELCTRICAS

Posicin del rotor en t = O


... --_~It_ Eje

magntico del estator Eje d

Figura 1.40. Motor de reluctancia variable.

anterior aparecern fuerzas en el sistema mvil que tendern a producir un movimiento de rotacin. Si se considera que el giro se produce a flujo constante, la ecuacin (1.82) representar el principio de conservacin de la energa aplicado al sistema: (1.102) Ahora bien, si se denomina T al par desarrollado y de al ngulo girado por el rotor, el primer miembro de la ecuacin anterior ser igual a:
dWmec = T de

(1.103)

de donde se deduce el valor del par:


T--

UWm] -

ue

(1.104)
CI>=cte

que es una ecuacin anloga a (1.83) aplicada a sistemas de rotacin. Si el sistema se considera lineal, lo que equivale a suponer que la nica reluctancia existente es la del entrehierro, entonces se cumplir:
I W = - 9?<l>2 m 2'

(1.105)

valor que llevado a (1.104) nos da:


T=--<l>

1 2

drA
de

(1.106)

lo que indica que el par acta en el sentido de reducir la reluctancia entre el esttor y el rotor, que en trminos prcticos significa que el par tiende a alinear el eje magntico del rotor con el del esttor. De una forma similar a la analizada en los movimientos de traslacin, si el

CAPTULO /.

CIRCUITOS MAGNTICOS Y CONVERSIN DE ENERGA

57

movimiento se produce a corriente constante (f.m.m. constante), (1.94) se transforman, respectivamente, en: 1 T=-.722

las ecuaciones

(1.93) y

di/'
de

T=

-1--

.0

dL
de

(1.107)

en las que !/'y L representan, respectivamente, la permeancia del circuito magntico y la inductancia de la bobina. Es fcil demostrar, observando la Figura 1.40, que tanto la reluctancia como la permeancia y la inductancia es funcin doble del ngulo () que forman el esttor con el rotor. Desde el punto de vista de la teora de circuitos resulta ms til trabajar con parmetros de inductancia que con valores de reluctancias o permeancias, por lo que ser ms prctico emplear la ltima expresin del par (1.107). Por consiguiente, vamos a demostrar nicamente la evolucin de la inductancia L con el ngulo para poder determinar el sentido del par que se ejerce sobre el rotor. De acuerdo con la definicin de inductancia (1.45) y (l.47), su valor ser mximo cuando el rotor est alineado con el eje del esttor (denominado eje directo), ya que corresponde a una posicin de mximo flujo en el entrehierro o mnima reluctancia del mismo, y denominando Ld a este valor se tendr:

L(e

= O) = L(e = n) = Ld

(1.108)

Anlogamente, la inductancia ser mnima cuando el rotor forme 90 (eje cuadratura) con el eje del esttor. Denominando Lq a este valor se tendr: L(e

= nl2) = ue =3n/2) = Lq

(1.109) que evoluciona entre LJ

En la Figura 1.41 se muestra la variacin de L con y Lq siguiendo una ley senoidal:

e, que se considera

L(e) = LJ + L2 cos 2e en f) = O se cumplir:

(1.110)

(1.111) yen e = nl2: (1.112)


L(e)

e
o
n/2 n 3n/2 2n

Figura 1.41. Variacin de la inductancia con la posicin del rotor.

58

MQUINAS ELCTRICAS

lo que permite identificar a L, y L2 en funcin de las inductancias Ld y Lq como: Ld + Lq . L _ LJ - Lq L '=--2-' 22


y llevando la expresin (1.110) a la ecuacin del par resultar: (1.1 13)

1 , d T = - - - (L, + L2 cos 2e)

dO

(1.114)

es decir: T = - i2L2 sen 2e = - 2 _J --q sen 20 2


L -L

(1.115)

Es evidente de la expresin anterior que si Ld = Lq no se desarrolla ningn par. Esta situacin se produce cuando el rotor es cilndrico y por consiguiente el entrehierro es uniforme. Cuando el rotor es asimtrico, como es el caso de la Figura 1.40, existe un par expresado por (1.115). Si se considera que la corriente de alimentacin es constante (tipo c.c.) y se supone que el rotor se desplaza de la posicin horizontal un ngulo O en direccin contraria a las agujas del reloj (posicin mostrada en la Fig. lAO), el par (1.115) es un par restaurador orientado en el sentido de las agujas del reloj. Si en el proceso de regresar el rotor a la posicin horizontal, debido a su momento, sobrepasa esta posicin, el campo magntico ejerce entonces un par en sentido contrario a las agujas del reloj. En otras palabras, si la corriente es constante, el par que se ejerce sobre el rotor no es unidireccional y est orientado en sentido contrario al que el rotor tiende a moverse. Es por ello que el rotor permanecer estacionario en posicin horizontal (mnima reluctancia); cualquier modificacin de la posicin del rotor producir una oscilacin sobre el eje horizontal hasta que se pare finalmente en esta posicin. Supngase ahora que el rotor se mueve (accionado en principio por un motor exterior) a una velocidad wm; entonces la posicin del rotor, de acuerdo con el esquema de la Figura lAO, ser: (1.116) donde O = + b representa la posicin del rotor en esttor con una corriente senoidal:
jet) = 1m cos
t

= O. Si en esta situacin se alimenta el


(l.l17)

on

entonces, de acuerdo con (1.115), se producir un par: T=

i2L2 sen 2e =

I~ cos? cot L2 sen 2(wmt + b)

(1.118)

y teniendo en cuenta que:


2 1 + cos 2wt cos wt=---2

(1.119)

por lo que el par se puede escribir: T = - - 1;,L2 sen 2(wmt + b)(l + cos 2wt) 2
1

(1.120)

CAPTULO

l.

CIRCUITOS

MAGNTICOS

Y CONVERSIN DE ENERGA

59

y sabiendo de trigonometra que:

1 sen a cos b = - [sen (a + b) + sen (a - b)J


2

(1.l21)

resulta finalmente:
T = - -/~,Lo 2" -, {sen 2(w m t +

6) + 2 sen 2[(w m + w)t + 6J + 6J}


(1.122)

+ - sen 2[(w", - w)t +

se observa en la expresin anterior que el par medio es cero para velocidades wm #- w. Sin embargo si el rotor gira a la velocidad co, (wm = co), denominada velocidad de sincronismo, el par medio es:
Tmed = - 1

2n o

2n

1 I,;,L2 sen 26 T d(wt) = - 4

(1.123)

lo que demuestra que si se verifica:

Iwml

Iwl

=>

wm = w

(1.124)

seproducir un par medio de rotacin, para los dos sentidos de giro que cumplan (1.124). Esta velocidadde rotacin se denomina velocidad de sincronismo, lo que significa que la velocidad mecnica de rotacin wm en rad/s coincide con la pulsacin o frecuencia angular w de la alimentacin.ste es el principio en que se basan los motores de reluctancia. El motor tiende a mantener su sentido de giro aun despus de que se desconecte el motor primario. En la Figura 1.42 se muestra la representacin del par medio (1.123) en funcin del ngulo6, que se denomina ngulo de par. Para 6 < Oel par medio es positivo, lo que significa, de acuerdo con la Figura 1.40, que acta en el sentido de rotacin de la mquina, que trabajaentonces como motor. Si se considera el motor ideal (sin prdidas) y que no existe par resistente,entonces la mquina girar inicialmente con 6 = O,y al arrastrar en este caso una cargamecnica, se necesitar que la mquina genere un par motor. Como en la situacin anteriorel par motor era cero, se producir un rgimen dinmico en el que se reducir la velocidad,

GENERADOR

MOTOR

,
!ZONA ESTABLE

Figura 1.42. Variacin del par electromagntico desarrollado por un motor de reluctancia.

60

MQUINAS ELCTRICAS

y como resultado de ello el ngulo [J comenzar a tomar valores negativos y desarrollar un par motor definido por (1.123) que equilibrar al par resistente y la mquina volver a girar a la velocidad de sincronismo. Conforme el par resistente aumente, el ngulo [J se har cada vez ms negativo; esta situacin puede llegar hasta que el valor de [J sea igual a - n/4, ya que entonces el par desarrollado alcanza su valor mximo; si el par resistente es superior a este valor, la mquina se saldr del sincronismo y se acabar parando. De este modo la zona estable como motor est comprendida entre O y - n/4, ya que en esta zona a medida que aumenta el par resistente tambin aumenta el par motor debido a que [J toma valores (absolutos) mayores; sin embargo, si se sobrepasa el punto A, un aumento del par resistente no se ve compensado por un aumento del par motor. Si [J > O,el par desarrollado es negativo, lo que significa, de acuerdo con la Figura 1.40, que acta en sentido contrario al giro de la mquina, y trabajando entonces como generador, el dispositivo mecnico conectado al eje debe suministrar par y potencia al rotor. Hay entonces un flujo de potencia elctrica de la mquina a la red. De un modo anlogo al estudiado en el comportamiento como motor, la zona estable como generador est comprendida entre Oy + n/4. La aplicacin ms conocida de los motores de reluctancia est en los relojes elctricos, y como quiera que no producen par de arranque, se inicia el giro dando un impulso mecnico al rotor por encima de la velocidad de sincronismo, de tal forma que se alcanza un estado estable al pasar por la velocidad de sincronismo, manteniendo esta velocidad.

1.8.2. Sistemas magnticos de rotacin alimentados con dos fuentes


Estos sistemas son los que normalmente se encuentran en las mquinas elctricas convencionales. Considrese el esquema de la Figura 1.43, que muestra un sistema magntico con dos alimentaciones tanto en el esttor como en el rotor.
Eje magntico del rotor Posicin del rotor en /=0 Eje magntico del estator

Figura 1.43. Sistema electromagntico doblemente alimentado.

CAPTULO l.

CIRCUITOS MAGNTICOS Y CONVERSIN DE ENERGA

61

Si se considera el sistema magntico lineal y recordando la teora de los circuitos acoplados,los flujos magnticos totales 1/11 y 1/12 que atraviesan los devanados 1 y 2 estarn expresados por las siguientes ecuaciones:
1/11 = Lllil + LI2i2 = NI<1>1 ;
1/12

= L2Jil + L22i2 = N2<1>2

(1. 125a)

que en forma matricial se escribe:

I/II} = [LII { I/Iz LZI

LI2]
LZ2

{~I} Iz

= {I/I} = {L}{i}

(1.125b)

donde {I/I} es el vector de flujos, [L] es la matriz de inductancias de los devanados en la que 22 es el correspondiente del devanado2 y LIZ =. L21 es el coeficiente de induccin mutua entre ambos devanados, y finalmente {i} es el vector de las corrientes que circulan por los arrollamientos. Los flujos magnticos <1>1 y <1>2 son los flujos que atraviesan cada una de las espiras de los devanados del esttor y del rotor, respectivamente. De acuerdo con esta nomenclatura, la energa magntica total almacenada (que es igual a la coenerga por ser el sistema lineal) se obtendr aplicando (1.43) a cada uno de los arrollamientos, lo que da lugar a:

LII representa el coeficiente de autoinduccin del devanado 1, L

(1.126) ya que: (1.127)


y teniendo en cuenta (1.125), la ecuacin (1.126) se transforma en:

(1.128) y de un modo anlogo a (1.91) puede demostrarse que aparece un par en el rotor de valor: T=+
y al sustituir (1.128) en (1.129) resulta:

VW;',] [ - ve i=cte
.2

(1.129)

T o en notacin matricial:
T = - [il

=-1

2
1

dL11 --+-1 de

dLZ2 .. dLI2 --+11-de 1 2 de

(1.130a)

1 2

iz] -

v [L 11 ve L21

12

L22

][i]
.1 12

T= -1 2

v {[L]{i}} {ir ve

(1.130b)

donde {ir es el vector traspuesto de {i}. Como era de esperar, las ecuaciones anteriores (1.130a)o (I.l30b) son una generalizacin del resultado (1.107) que se obtuvo para sistemas alimentadoscon una sola fuente. Los dos primeros sumandos de (1.130a) representan pares de reluctanciadebidas a las inductancias propias de cada devanado; el tercer sumando representael par que se obtiene cuando vara la inductancia mutua entre ambos devanados. En

62

MQUINAS ELCTRICAS

general, para una disposicin magntica como la que se indica en la Figura 1.43, y de acuerdo con lo desarrollado en el epgrage 1.81, las expresiones de L" y L22 sern de la forma:
(1.131)

mientras que el coeficiente de induccin mutua depender del ngulo que forman entre s los ejes magnticos del esttor y del rotor:
(1.132)

lo que era lgico suponer teniendo en cuenta la propia definicin del coeficiente de induccin mutua, como flujo que llega a un devanado producido por el otro. Cuando los ejes estn alineados el flujo ser mximo, es decir, todo el flujo que produce un devanado atravesar el otro; cuando los ejes formen 90 no habr flujo que saliendo de un devanado atraviese el otro. En general, para una posicin arbitraria deber aplicarse la ecuacin (1.132), que es vlida sean cuales sean las disposiciones magnticas del esttor y del rotor. No ocurre as con las expresiones (1.131), ya que la evolucin de las inductancias propias o coeficientes de autoinduccin en funcin de depender de las simetras magnticas de las estructuras del esttor y rotor. Por ejemplo, si ambas son cilndricas, L" y L22 sern constantes y no dependern de e porque al moverse el rotor ninguno de los devanados apreciar ningn cambio de reluctancia. Si el esttor tiene salientes magnticos y el rotor es cilndrico, entonces L" ser constante y L22 vendr expresado por la segunda ecuacin (1.131), y a la inversa, si el esttor es cilndrico y el rotor tiene polos salientes, entonces L22 ser constante y L" tendr la forma indicada en

(1.131).

En el caso de que se desee relacionar las tensiones aplicadas con las corrientes en cada devanado deber aplicarse el 2. lema de Kirchhoff a cada arrollamiento. Si se denomina R, y R2 a las resistencias respectivas de los devanados, teniendo en cuenta (1.125) resultar: v,
=

dtf, R' d (L',,1, R 11,+ = ,1, + -

dt

dt

L''212 )

dtf2 R' d (L'2111+L'2212 ) v2= R212+-= 212+dt dt

(1.133a)

o en notacin matricial:

{ Vv2,} = [ROlO] R2

{il}
i2

+ di L2,

[L'I

L22

LI2J[i'J
. 12

=>

{v} = [R]{i} + -d [L]{i}


dt

(1.133b)

En el caso genrico de que existan ms de dos devanados (excitacin mltiple), el estudio correspondiente es anlogo al aqu expuesto, teniendo en cuenta que entonces la ecuacin (1.128) de la energa almacenada ser de la forma:

"-':n = w;" =
o en notacin matricial:
Wm = W;" =

L L;j i;ij
;=, j='

(I.134a)

2 {i}'[L]{i}

(l.134b)

Las notaciones matriciales son expresiones ms convenientes cuando se desea realizar un estudio generalizado de las mquinas elctricas.

CAPTULO

l.

CIRCUITOS MAGNTICOS y CONVERSIN DE ENERGA

63

EJEMPLO DE APLICACIN J.II: MOTORES MONOFSICOS SNCRONOS Y ASNCRONOS


Considrese la mquina con simetra cilndrica mostrada en la Figura 1.44. Los valores de las inductancias de los devanados son de la forma: LJJ = a (constante); Ln = b (constante); LI2 = Lm cos 0, en donde O est medido a partir de la posicin de referencia en sentido contrario a las agujas del reloj. a) Si el rotor se alimenta con una corriente continua i, = IJ y se anlica al esttor una c.a. de la forma i, = I_, cos cot, determinar la expresin del par instantneo y medio desarrollado si el rotor gira a una velocidad angular wm = w. Este montaje describe el comportamiento de un motor sncrono monofsico de polos lisos o rotor cilndrico. b) Si se aplican corrientes alternas a ambos devanados de la forma:

y el rotor gira a una velocidad angular wm' cul debe ser la relacin entre

W J' w2 Y wm para que se produzca un par neto en el rotor? Cul ser la expresin del par medio en esas condiciones? Este montaje describe el comportamiento de un motor asncrono monofsico.

Eje magntico del rotor Posicin del rotor en 1=0


I--C:>---()--. Eje magntico

del estator

t
SOLUCIN
a)

VI

Figura 1.44. Motor asncrono monofsico. Motor sncrono con rotor de polos lisos (cilndrico).

De acuerdo con (1.130), y teniendo en cuenta que LJJ y L22 son constantes, se tendr un par: ..
1112

T=

dLI2 dO = IJ/m2 cos cot (-Lm sen e)

y como quiera, adems, que

e = w",t

+ 6, resultar:
cos cot sen (wt + b)

T = - Lmllm2

64

MQUINAS ELCTRICAS

que haciendo una transformacin trigonomtrica se convierte en un par instantneo:

L"l/m?- [sen (2wt + 6) + sen 6] T = - __ 2


Ahora bien, si se tiene en cuenta que el valor medio del trmino dependiente del tiempo es cero, el valor medio del par resultante ser:
Tmed

= -I

2n: o

2n

Td(wt)

Lm/llm2 = - -2

sen 6

La expresin anterior indica: 1) existe un par medio distinto de cero que es proporcional a sen 6, 2) la mquina no puede arrancar por s misma pero puede desarrollar un par neto si gira a la velocidad wm = W (velocidad de sincronismo), para lo cual se

b)

requiere la accin de un motor primario externo que impulse el rotor hasta la velocidad de sincronismo; en ese momento, aunque se desconecte el motor externo, la mquina podr seguir girando por s misma. Si se aplican corrientes alternas a ambos devanados, al aplicar (1.130) se obtendr un par instantneo:
T=
(1m I

sen wlt)(lm2

sen w2t)( -Lm sen 8)

y teniendo en cuenta que 8 = w",t + 6, despus de una transformacin trigonomtrica resulta:

es decir:
T=

y aplicando a cada sumando contenido entre los corchetes la transformacin trigonomtrica (1.121) se obtiene:

Si se elige la velocidad del rotor de tal modo que se cumpla:


Iwml = IWI
-

w21

=>

wm

= (wl

w2)

(a)

se obtiene tanto para el signo positivo de wm como para el signo negativo un par medio resultante distinto de cero de valor:

(b)
La condicin (a) indica que la velocidad de rotacin est relacionada directamente con las frecuencias de las corrientes que circulan por los devanados del esttor y del rotor.

CAPTULO I.

CIRCUITOS MAGNTICOS Y CONVERSI6N DE ENERGA

65

Como se sealar ms adelante en el Captulo 2, el cociente entre las frecuencias del rotor y del esttor se denomina deslizamiento y se representa por la letra s, es decir:
s=-=--=-

w2 w,

2nf2

f2

Ztt],

1;

de donde se deduce que la expresin


wm

(a)

se puede escribir as:

= (w, -

(2)

= w,(l

- s)

(c)

La descripcin anterior explica el comportamiento de un motor asncrono o de induccin monofsico que produce un par medio resultante proporcional a sen (j. La mquina no puede arrancar por s misma y tiene que ser impulsada por un medio mecnico exterior en el sentido de rotacin que se desee. Realmente, por el devanado del rotor no se introduce ninguna corriente externa, la corriente i] se obtiene por induccin del campo magntico del esttor estando el bobinado del rotor en cortocircuito. El deslizamiento de estos motores suele estar comprendido entre el 10 y el 20 por 100,de ah que segn (c) haya que impulsar el rotor hasta velocidadesdel orden de 80 al 90 por 100de tu/ para obtener un par neto en el eje. En el Captulo 4 se explicar con detalle el funcionamiento de este tipo de motores. EJEMPLO DE APLICACIN 1.I2: MOTOR MONOFSICO SNCRONO y DE RELUCTANCIA En la Figura 1.45 se muestra una mquina elctrica denominada de polos salientes (situados en el rotor): L"

= a (constante)

; L22 = La + Lb cos 2e

; L'2

= Lm cos e

en donde est medido a partir de la posicin de referencia en sentido contrario al movimiento de las agujas del reloj. Las resistencias de los devanados son despreciables: a) Si el rotor se alimenta con una corriente continua i/ = 1/ Y se aplica al esttor una c.a. de la forma iz = 1m2 cos cot, cul deber ser la velocidaddel rotorpara que se obtenga unpar neto en el eje delmismo?Determinar a continuacin el par medio resultante. Este montaje describe el comportamientode un motor sncrono monofsico de polos salientes. b) Responder a la pregunta
Eje magntico del rotor

Posicin del rotor en t= O

- .... ---iIIH~

Eje magntico del estator

Figura 1.45. Motor sncrono de polos salientes. Motor de reluctancia.

66

MQUINAS ELCTRICAS

anterior si el devanado del rotor se deja abierto (i I = O) y la corriente del esttor sigue siendo la misma. Este montaje describe el comportamiento de un motor sincrono de reluctancia

variable. SOLUCIN
a)

De acuerdo con (1.130), la expresin del par desarrollado ser igual a: 1 T= - ii(-2Lb sen 2B) + ili2(-Lm sen fJ)

que al sustituir los valores de i; iz y

e nos da lugar a:

T = - 1~2Lb cos? on sen 2(mmt + )- II/m2Lm cos tot sen (mmt + ) y teniendo en cuenta que:
2 1 + cos Zcat cos mt=---2

resulta:

y al aplicar la transformacin trigonomtrica (1.121) se convierte en: 1~2 T= --Lb 2 { sen 2(mmt+ 6) +1 [sen 2(mt+ mmt+ 6)] +1 [sen 2(-mt+ 2 2
I 1m2 Lm{ sen (cot + mnl + ) + sen (-mt -2

mmt+ )]} -

+ mmt + 6)}

Cada trmino de la ecuacin anterior es una funcin senoidal del tiempo, por lo que el valor medio del par ser nulo. Para que se obtenga un par medio diferente de cero, el rotor deber girar a una velocidad:
Imml = Iml
=>

mm = m

ya que en esta situacin se obtiene un par: 1. Para mm = m :


12 Lb [ sen 2(cot + 6) + 1 sen 2(2mt + 6) + 1 sen 26] T = - ..!'!l:.

222

- 2. Para mm = - m:

1 1m2
1

Lm [sen (2mt + )+ sen )]

1~2Lb [ sen 2(- cot + 6) + -1 sen 2 + -1 sen 2(- 2mt + 6)] T=- 2 2 2 - 111m2

Lm [sen + sen (-2mt + 6)]

CAPTULO l.

CIRCUITOS MAGNTICOS y CONVERSI6N DE ENERGA

67

que en ambos casos da lugar a un par medio:

b)

La expresin anterior indica que el par medio se compone de dos trminos: el primero representa el par de reluctancia que se debe a la forma de polos salientes que tiene el rotor y que no depende de la corriente que circula por este devanado, y el segundo representa el par de excitacin que es funcin de la corriente rotrica. La mquina no puede arrancar por s misma pero desarrolla un par neto cuando gira a velocidad (1)m = (1). En esta situacin solamente se produce un par de reluctancia de valor medio: T = _...!!!3....J'. sen 2<5
ni

/2 L

siempre que el rotor gire a la velocidad de sincronismo demostrado en el apartado anterior.

(1)m

(1),

como ya se ha

EJEMPLO DE APLICACIN 1.13: MOTOR BIFSICO SNCRONO Considresela mquina con simetracilndricamostrada en la Figura /.46. El esttor tiene dos devanadosdesfasados 9{)0 en el espacio(se dice entoncesque estn situados en cuadratura),que se alimentanpor medio de dos corrientesalternas desfasadas en el tiempo 9{)0, es decir:

El rotor lleva incorporado un devanado 2, siendo las inductancias de los devanados las siguientes:

a) Si el rotor gira a una velocidad angular (1)m Y se alimenta con una c.c. de valor i2 = /2' cul deber ser el valor de (1)m para que se obtenga un par neto en el rotor? Qu par medio desarrolla la mquina? Este montaje describe el comportamiento de un motor sncrono bifsico de polos lisos.

Posicin del rotor en t= O

Eje a del estator

Figura 1.46. Motor sncrono bifsico de polos lisos.

68

MQUINAS ELCTRICAS

b)

Si el rotor gira a una velocidad angular wm y se aplica al devanado del mismo una corriente alterna de valor i2 = 1m2COS wi, para qu velocidad del rotor se obtiene un par medio distinto de cero? Cul es el valor del par medio correspondiente? Las corrientes del esttor son las definidas previamente.

SOLUCIN
a)

El par responder a la expresin genrica matricial (1.130b), teniendo en cuenta que existen tres devanados que interaccionan entre s. y as se obtiene:

T=

2 {ia

[Laa
ib i2} :e

t.;
Lbb

i;

LZa LZb

que al desarrollar conduce a la siguiente expresin: 1 .2 dLaa 1.2 dLbb 1.2 dLzz .. dLab .. dLa2 dLb2 T=-I --+-1 --+-1 --+11 --+11 --+11-2 a de 2 b de 2 Z de ti b de a 2 de b 2 de que teniendo en cuenta que existen inductancias que no dependen de
.. dLa2 .. dLbz T = lal2 de + Ibl2 de

e, se obtiene:

al sustituir los valores resulta un par instantneo:


T = IZlml cos wlt (-Lm sen e) + 121mlsen wlt (Lm cos e)

es decir:
T= ImJ2Lm (-sen ecos WII + cos e sen w/) = Iml/2Lm sen (Wll - e)

y como quiera que e = wmt + b, se convierte en:


T = ImJ2Lm sen (wlt - wmt - b)

(a)

si el rotor gira a una velocidad wm = wi' denominada velocidad de sincronismo, se obtiene un par instantneo que no depende del tiempo:

(b)
Lo anterior explica elfuncionamiento de un motor sncrono de polos lisos. La mquina no puede arrancar por s misma, ya que cuando wm = Oel par medio es nulo; sin embargo, se obtiene un par neto resultante si la mquina gira a la velocidad de sincronismo wm = WI. Estos motores requieren un accionamiento o motor primario externo que impulse el rotor hasta la velocidad de sincronismo; alcanzada esta velocidad el rotor se enclava magnticamente con el denominado campo giratorio del esttor de tal modo que aunque se desconecte el motor externo, la mquina podr seguir girando por s misma. Es importante que el lector se d cuenta de que si wm = - wi' el par medio resultante es, segn la expresin (a), igual a cero. A diferencia con el motor sncrono monofsico (vase ejemplo de aplicacin 1.11), que produce un par neto a velocidades wm = wi' el motor bifsico (yen general polifsico) tiene un sentido de rotacin definido. Esto se debe, como se demostrar en el Captulo 2, a que el deva-

CAPTULO l.

CIRCUITOS MAGNTICOS Y CONVERSI6N DE ENERGA

69

b)

nado bifsico, trifsico, etc., produce un campo magntico giratorio que arrastra el rotor como si estuviera enclavado mecnicamente con l. El ngulo b de la Figura 1.46 representa entonces la diferencia de fase espacial entre el eje del campo magntico giratorio que se produce en el esttor con el eje magntico del rotor. Para unas determinadas corrientes Imle 12, la expresin (b) nos indica que el par motor producido es proporcional al sen b. El par resistente se equilibra con el par motor para un determinado ngulo b, y si en esta situacin el par resistente aumenta se elevar el ngulo b hasta un valor mximo de 90; por encima de este valor, el par, segn (b), vuelve a disminuir, y al no poder arrastrar la carga resistente el motor sncrono pierde su sincronismo, es decir, wm deja de ser igual a wI' por lo que el valor del par medio se hace cero. En esta situacin el par resistente del motor obligar a reducir la velocidad del rotor hasta que finalmente se pare. El par resultante, de un modo anlogo al calculado en el apartado anterior, ser igual a: .. dLa2
T=
lall lbl2

dLb2
de

de +

que al sustituir valores nos da: T = Im/m2Lm y teniendo en cuenta que cos w2t sen (wlt - e)

e = wnl + b, resulta:

T = Im/m2Lm cos w2t sen (wlt - wmt - b) que haciendo una transformacin trigonomtrica da lugar a:

si wm = wl w2' se obtiene un par medio de valor:

Obsrvese tambin que a diferencia con el caso anterior, si wm = wl' el par medio es cero. La velocidad a la que puede girar el motor depende de las frecuencias del rotor y el esttor y la mquina no puede arrancar por s misma, ya que para wm = O el par medio es cero. EJEMPLO DE APLICACIN CON ROTOR MONOFSICO 1.14: MOTOR ASNCRONO BIFSICO

Considresela mquina con simetra cilndrica mostrada en la Figura 1.47. Al igual que en el ejemplo anterior, el esttor tiene dos devanados desfasados 90 en el espacio que se alimentan con corrientes bifsicas:

El rotor lleva incorporado un devanado 2 de resistencia Rz ohmios, siendo las inductancias de los devanados las siguientes: Laa = Lbb = LI ; L22 = L2
;

La2 = Lm cos

e ;

Lb2 = Lm sen

e ;

Lab =

70

MQUINAS ELCTRICAS

Eje magntico del rotor

Posicin del rotor


ent=O

I--O---c~-

Eje a

del estator

Figura 1.47. Motor asncrono bifsico.

a) Si el rotor gira a velocidad angular wm' calcular la expresin de laf.e.m. inducida en el mismo en circuito abierto. b) Si se cortocircuita el devanado del rotor, determinar la corriente que circular por este devanado en rgimen permanente. c) Calcular en la situacin del apartado anterior el par medio que se produce en el rotor y la potencia mecnica desarrollada por la mquina. SOLUCIN
a)

La f.e.m. inducida en el rotor ser: e --2 -

dl/l2
dt

donde 1/12 representa el flujo total concatenado por el devanado del rotor. De un modo anlogo a (1.125), el flujo anterior ser igual a:

donde se ha tenido en cuenta la propiedad conmutativa de los coeficientes de induccin mutua y que i2 = O al estar el rotor abierto. De este modo la f.e.m. inducida en el rotor ser:
(a)

donde los valores de las derivadas, teniendo en cuenta que () = wmt + dLa2

(j,

son:

dt

=-

dt

[Lm cos (wmt + (j)]

=-

wmL", sen (wmt + (j)

-dt =

dLb2

m m

L cos (w"t,

+ (j)

y al sustituir los valores anteriores en la ecuacin (a) y haciendo algunas transformaciones trigonomtricas se obtiene: .

e2 = Ln'/",I (WI - wm) sen [(WI - wm)t - b]

CAPTULO 1.

CIRCUITOS MAGNTICOS Y CONVERSIN DE ENERGA

71

Obsrvese en la expresin anterior que si el motor girase a la velocidad de sincronismo (mm = mi) no se producir f.e.m, en este devanado. La pulsacin de la f.e.m. vale m2:

Si se denominan ni a las r.p.m. del rotor y I1 y 12 a las frecuencias del esttor y del rotor, respectivamente, la ltima ecuacin se puede escribir as: 2nl2

::

n 2nll - 2n -

60

=>

12 = II - -

60

expresin que relaciona la frecuencia del rotor con la del esttor y la velocidad de giro. Al estudiar motores asncronos (vase Captulo 4) se acostumbra a definir el concepto de deslizamiento del motor s como cociente de las frecuencias esttor/rotor:

y de este modo se puede escribir:

que al sustituir en la expresin de la f.e.m. e2 da lugar a:


e2 =

Lm1ml

sml

sen (sml1

- b)

que se puede escribir de la forma


e2 = sEm2

sen

(m21 - b)

donde se ha llamado Em2 Y m2 a:

Em2 representa la f.e.m. mxima inducida en el rotor cuando s = 1, es decir, cuando mm = O (rotor parado).
b)

Al cortocircuitar el rotor se producir una corriente de circulacin por el mismo que se obtendr aplicando el 2. lema de Kirchhoff a este circuito, resultando la siguiente ecuacin diferencial:

(b)
Para determinar la corriente instantnea i2 de rgimen permanente, que es la solucin particular de la ecuacin diferencial anterior, se resolver la ecuacin fasorial (en valores mximos y tomando como referencia la funcin seno) siguiente:

que conduce a un valor complejo de 1m2:

72

MQUINAS ELCTRICAS

donde qJ = arctg ~, por lo que la correspondiente corriente instantnea o solucin Rz de la ecuacin (h) ser: iz = y como quiera que W2 = ser:
sWI'

wL

sEmZ

R~+ w;L;

sen (Wzt - c5 -

qJ)

si se denomina Xz = wl Lz' la expresin de la corriente

donde ImZ=
e)

J R; + sZX;
T=
lalZ

El par desarrollado por la mquina responde a la ecuacin: .. dLaZ de +


. lblz

dLbz de

(e)

que teniendo en cuenta que: dLaZ d de = de (Lm cos e)

= - Lm sen e = - Lm sen (wmt + c5) = +Lm cos e = +Lm cos (wn/


+ c5)

dLbz de

=-

d (Lm sen e) de

y recordando que wm = wl (1 - s); w2 =

SWI'

al sustituir en (e) resulta:

T = Lmlml iz sen (wzt - c5)


y al sustituir el valor de iz calculado en el epgrafe anterior se obtiene:

T = Lmlml/mZ sen (wzt - c5 es decir

qJ)

sen (wzt - c5)

que corresponde a un valor medio:

pero teniendo en cuenta que al ser tg cos


tp

qJ

= wzLzlRz, el valor del cos

qJ

es:

= --r===

JI + tg

qJ

CAPTULO l.

CIRCUITOS MAGNTICOS Y CONVERSI6N DE ENERGA

73

y como quiera que

resulta un par medio:

y como adems se tiene que:

el valor del par medio en funcin de 1m2 es:


T _

R/;'2
2sw,

med -

Como quiera adems que para una onda sinusoidal, la corriente eficaz 12 es igual al valor mximo /,,12 dividido por el par medio resultante anterior se puede expresar tambin como:

J2,

o en funcin de la f.e.m. eficaz E2 del rotor: T R s2E2 2 2 w, (R; + S2 X;)

med -

de este modo el motor asncrono bifsico produce un par medio distinto de cero, cuya magnitud depende de la velocidad. En particular en el arranque se cumple s = 1, por lo que el par correspondiente ser:

R2E;
Ta =
W,

(R2 + X2)

Obsrvese que si wm fuera igual a w, se cumplir s = O, Yen estas condiciones el par medio sera cero. El motor nunca podr girar a la velocidad de sincronismo y es por ello que estos motores se denominan asncronos. Generalmente el motor suele trabajar para deslizamientos comprendidos entre el 2 y el 8 por 100. La potencia mecnica que desarrollar el motor cuando gire a la velocidad angular wm ser:

pero teniendo en cuenta que co; =

W, (1 -

s), resultar:

Rl; Pmec = -SW,

W,

(1 - s) = Rl22

(1s 1)
-

expresin que se demostrar de un modo ms tradicional en el Captulo 4, dedicado a motores asncronos.

74

MQUINAS ELCTRICAS

EJEMPLO DE APLICACIN 1.15. MOTORES PASO A PASO DE RELUCTANCIA VARIABLE Un motor paso a paso (step-step o stepper) es un dispositivo electromecnico que convierte una serie de impulsos elctricos en desplazamientos angulares discretos. En la Figura 1.48 se representa un motor de este tipo cuyo esttor consta de tres secciones: a, by c. El rotor tiene ocho salientes magnticos. Cada devanado del esttor est distribuido alrededor de la periferia formando ocho salientes magnticos, de tal modo que los salientes o dientes correspondientes a cadafase estn desplazados 15 entre s. Las inductancias de cada devanado o fase son mximas cuando estn enfrentados los dientes del rotor con los correspondientes del esttor de cada fase. Si se denomina e al ngulo que forma el centro de un diente del rotor con el correspondiente ms cercano del esttor de la fase a (medido, por ejemplo, en el sentido de las agujas del reloj), las inductancias propias de los devanados de cada fase, en funcin de la posicin del rotor, vendrn expresadas por las ecuaciones:

Calcular la expresin del par resultante en el rotor en los casos siguientes: 1) cuando se excita nicamente el devanado a; 2) cuando se excita solamente el devanado b; 3) cuando se excitan a la vez los devanados a y b.
Devanado del estator

Seccin por fase a polos del estator

Seccin por fase b

Seccin por fase e

fase a fase b fase e Secciones o fases del estator

Figura 1.48. Motor paso a paso.

SOLUCIN
1.

La expresin del par, de acuerdo con (1.107), ser:

lo que significa que el rotor girar en sentido contrario a las agujas del reloj (signo menos de la expresin anterior) hasta que el par sea nulo, es decir, para e = o, lo que

CAPTULO J.

CIRCUITOS MAGNTICOS Y CONVERSIN DE ENERGA

75

corresponde a una posicin en la que se enfrentan los dientes del rotor con los correspondientes del esttor de la fase a. 2. En este caso resultar: 1 .2 dLh'2 T" = 2" 1" d8 = -41"Lm sen 8(8 - 15 ) que significa que el rotor girar un ngulo de 15 en sentido antihorario, lo que corresponde al enfrentamiento de los dientes del rotor con los del esttor de la fase b. 3. En este caso se""t:umplir:

donde se ha supuesto que las corrientes de ambos devanados son iguales, es decir:

y el par anterior se puede escribir tambin:

8(e- 7,5) El par anterior se anula para 0= 7,5 Ytiene un valor mximo que es fi veces del
T"b

= -8i"

cos 60 Lm sen 8(8 - 7,5)

= -4fi

i2Lm sen

que se obtiene excitando una sola bobina. Si se hubiera partido de la excitacin de la bobina a, se observa que al excitar a la vez dos devanados a y b se producir un giro antihorario de 7,5. De este modo el motor paso a paso aqu descrito permite realizar desplazamientos angulares de 7,5. En la prctica existen motores paso a paso con escalones de 2; 2,5; 5; 7,5 y 15 por cada impulso aplicado a los devanados del esttor. Este tipo de motores se utiliza en sistemas de control digital, en los que el motor recibe un tren de impulsos para situar la posicin del rotor en el ngulo deseado. Cada impulso aplicado a las bobinas del esttor hace girar el rotor el ngulo Og de paso. El sentido puede ser directo o inverso segn sea el orden de sucesin de impulsos en las bobinas. Las ventajas de estos motores son: 1) respuesta rpida (menos de 1 milisegundo), 2) insensibilidad a las vibraciones y choques, 3) larga duracin (del orden de millones de ciclos), 4) posicionamiento preciso, 5) insensibilidad a las variaciones de tensin y a las amplitudes de los impulsos. La versatilidad de estos motores es tambin muy importante y se pueden utilizar como: a) motores de frecuencia variable, es decir, de velocidad variable; b) motores de c.c. sin escobillas; e) servomotores en circuito abierto que eliminan la realimentacin, y d) motores sncronos. Las aplicaciones ms importantes incluyen el posicionamiento de piezas en mquinas-herramientas, impresoras de ordenador, accionamientos para plumillas de sistemas registradores, plotters x-y, relojes elctricos, etc. Por ejemplo, en las impresoras de ordenadores, los motores paso-paso se utilizan para efectuar el avance del papel; existe un sistema de engranajes o caja de velocidad de relacin 4,5: 1 entre el motor y el papel, de tal modo que al aplicar impulsos al devanado del esttor, se pueden conseguir avances con pasos en el papel de 6 a 8 lneas por pulgada.

PROBLEMAS
1.1. Calcular la intensidad que debe aplicarse a la bobina del circuito magntico de la Figura P.I.I. para establecer en la columna derecha un flujo de 10-3 Wb. La perrneabi-

76

MQUINAS ELCTRICAS

lidad relativa se supone que es constante en todos los puntos y de valor u; = 400, Yla seccin S = 10 cnr' es la misma en toda la estructura, excepto en la columna izquierda, que vale 20 cnr'. La longitud l es igual a 10 cm. Calcular tambin el flujo en el brazo central.

s' = 2s

Figura P.I.I.

[Resp.: 1 = 9,95 A; <I> = 2,2 mWb.] 1.2. Un circuito magntico tiene una seccin uniforme de 8 cnr' y una longitud magntica media igual a 0,3 metros. Si la curva de magnetizacin del material viene expresada aproximadamente por la ecuacin:
1,55 H B = -77+H B: Teslas; H: A.v/m

Calcular la c.c. en amperios que debe introducirse en la bobina de excitacin, que tiene 100 espiras, para producir un flujo en el ncleo de 8 . 10-4 Wb. [Resp.: 0,42 A.] 1.3. Calcular la corriente necesaria en la bobina de la Figura P.l.2 para producir una densidad de flujo en el entrehierro igual a 0,8 Teslas. El ncleo est hecho de un material cuya curva de imanacin viene dada por:
1,6H B = -75 +H B: Teslas; H: A.v/m

Figura P.I.2.

[Resp.: 6,83 A.]

CAPTULO l.

C1RCUITOS MAGNTICOS y CONVERSIN DE ENERGA

77

1.4. En la estructura magntica mostrada en la Figura P. 1.3, la densidad de flujo en el entrehierro de la derecha es de 1 Wb/m2 El ncleo est hecho de un material cuya curva de imanacin viene dada por:
B=

1,5 H 1.000 + H
2mm

B: Teslas; H: Av/m

Imm

Figura P.1.3.

la longitud 1 = 10 cm y la seccin transversal es uniforme y vale 5 cm", Calcular las corrientes /1 e /2 que deben circular por las bobinas para que el flujo en el entrehierro izquierdo sea nulo. [Resp.: 11 ~ 28 A; 12 ~ 8 A] 1.5. La estructura magntica mostrada en la Figura P. 1.4 est construida con un material cuya curva de imanacin se expresa por:
B=

1,5H lOO+H

B: Teslas; H: Av/m

2mm

T
+
Figura P.l.4.

La longitud de la trayectoria magntica media en el ncleo es igual a 0,75 m. Las medidas de la seccin transversal son de 6 x 8 cm". La longitud del entrehierro es de 2 mm y el flujo en el mismo es igual a 4 mWb (en el sentido indicado en la Fig. P.1.4). Determinar el nmero de espiras de la bobina B. [Resp.: NB ~ 1.237 espiras.]

78

MQUINAS ELCTRICAS

1.6. El ncleo magntico mostrado en la Figura P.l.5 tiene una seccin transversal uniforme igual a 100 cm". La bobina A tiene 1.000 espiras, circulando una c.c. de 0,5 A. en la direccin indicada. Determinar la corriente lB' para conseguir un flujo nulo en el brazo central. La permeabilidad relativa es f.1r = 200. [Resp.: IR = 1,25 A.]

Figura P.1.5.

1.7. El circuito magntico de la Figura P.1.6 est construido con un material, cuya curva de magnetizacin viene dada por:
1,5 H B = -50+H B: Teslas; H: A.v/m

Figura P.l.6.

La seccin de la columna central vale 50 crrr' y en el resto es uniforme y de valor 25 crrr', Si NI = N2 = 360 espiras, calcular el valor de II = 12 para producir un flujo de 5 . 10-3 Wb en el entrehierro. [Resp.: 11,32 A.] 1.8. La estructura magntica de la Figura P.1.7 est fabricada con dos tipos de materiales, cuyas curvas de magnetizacin vienen expresadas por las ecuaciones:
1,1 HI BI =---5.000 +HI B2 = 2,1 H2 B: Teslas; H: A.v/m

2.000 + H2

CAPTULO /.

CIRCUITOS MAGNTICOS Y CONVERSIN DE ENERGA

79

Figura P.l.7.

Calcular la intensidad 1 que debe circular por la bobina para producir un flujo de 1,5 . 10-4 Wb, si la seccin es uniforme y vale 15 crrr', [Resp.: 1 A] 1.9. Una estructura magntica homognea tiene una longitud magntica media igual a 50 cm; y tiene una seccin uniforme de 10 cnr'. Si la bobina tiene 100 espiras y la curva de magnetizacin viene expresada por:
B =

15 H 100+H

B: Teslas; H: Av/m

Cuando circula por la bobina una intensidad de 0,1 A se pide el valor del coeficiente de autoinduccin calculado por los tres procedimientos siguientes: a) Empleando la frmula: L = N d<Dldi.b) Utilizando la expresin: L = N <D/i.c) Calculando la energa magntica almacenada por medio de la expresin (1.44) e igualando a 1/2 Li2 [Resp.: a) 2,08 H; b) 2,5 H; c) 2,34 H.] 1.10. Una bobina con ncleo de hierro, tiene 500 espiras, siendo su resistencia despreciable. La seccin del ncleo es uniforme y vale 25 crrr', siendo la longitud magntica media igual a 80 cm. La curva de imanacin del material es:
B = 2H

ISO +H

B: Teslas; H: Av/m

Si la tensin aplicada es alterna y de 220 V eficaces y la frecuencia es de 50 Hz, calcular: a) Circuito equivalente de la bobina. b) Corriente de excitacin.
NOTA: Se conoce, por la informacin proporcionada por el constructor, que a la tensin nominal de 220 V las prdidas en el ncleo son de 5 Wlkg. El peso especfico del material es igual a 7,8 kg/dm', [Resp.: a) RFe = 620,5 n; Xp = 1.972 n; b) 0,372 A.]

1.11. Una bobina con ncleo de hierro absorbe una corriente de 0,5 A cuando se aplica una tensin senoidal de 220 V eficaces a sus bornes. Si la potencia absorbida fue de 30 W, deducir el circuito equivalente de la bobina. [Resp.: RFe = 1.617,64 n; Xp = 457,38 n.] 1.12. Un cerrojo elctrico consiste en una armadura fija cilndrica hueca y un vstago cilndrico, dispuestos como se indica en la Figura P.l.8. Supuesto que la reluctancia del hierro es despreciable frente a la del entrehierro, y que la unin vstago-armadura pres~ta un entrehierro despreciable frente al entrehierro principal e, calcular: a) La

80

MQUINAS ELCTRICAS

energa almacenada en el entrehierro en julios si e = 1cm; la superficie del entrehierro es de 0,8 cm? y laintensidad de excitacin es de 1 A de c.c. b) Fuerza magntica en el caso anterior.

Figura P.I.S.

[Resp.: a) 5 . 10-3 julios; b) 0,497 Newton.] 1.13. El ncleo magntico de la Figura P.1.9 tiene una seccin transversal cuadrada de 3 x 3 cm. El entrehierro x = 5 mm. La bobina tiene 250 espiras y una resistencia de 11 n. La f.m.m. que necesita el hierro es despreciable. Calcular la energa almacenada en el entrehierro y la fuerza total que acta sobre la armadura cuando se aplican a la bobina 220 V de c.c.
x=5mm

Armadura mvil

Armadura fija

Figura P.I.9.

[Resp.: 2,82 julios; 565,8 N.] 1.14. La estructura magntica de la Figura P.l.lO tiene una permeabilidad relativa /1r = 100; la longitud de la trayectoria magntica media es igual a 1 m en el hierro. El valor de la seccin transversal es de 100 cm". La longitud total del entrehierro (dos partes) es de 0,2 cm. El flujo en el entrehierro es de 4 . 10-3 Wb y su sentido es el indicado en la figura. La bobina A tiene 1.000 espiras y la B tiene N espiras, circulando por ambas bobinas una c.c. de 6 A. Se pide: a) Determinar el nmero de espiras de la bobina B. b) Calcular la fuerza con que es atrada la armadura mvil. e) Si se coloca una espira como se indica en la Figura P.l.IO, cul ser la lectura del voltmetro? 1) Si la corriente de alimentacin

CAPTULO

J.

CIRCUITOS MAGNTICOS Y CONVERSI6N DE ENERGA

81

es de c.c. 2) Si la corriente de alimentacin es senoidal y de tal magnitud que produzca el mismo valor eficaz de flujo en el entrehierro. La frecuencia es de 50 Hz. NOTA: Se supone que para resolver el apartado. e) el entrehierro est abierto. [Resp.: a) 363 espiras. b) 130 kg. e) O volt., 1,25 V.]

6A

Figura P.l.l0 1.15. Hallar una expresin de la fuerza en el bloque deslizante A de la Figura P.l.ll. Despreciar la reluctancia del hierro. Las bobinas estn alimentadas con c.c. y los parmetros son: NI

= 200

; N2

= 100

; i = 10 A ; i2

= 15 A

= 3 mm

e = 10 mm

= 10 cm"

[Resp.: 250,4 N hacia la izquierda.]

Figura P.1.11. 1.16. Las inductancias de un dispositivo electromagntico son: mostrado en la Figura P.l. 12

La"

= LI

+ L2 cos 28 ; Lbh = L. - L2 cos 28 ; La2 = Lm cos 8 Lb2 = Lm sen 8 ; Lah = L2 sen 28 ; L22 = constante

Calcular la expresin del par producido si las corrientes son de la forma:

82

MQUINAS ELCTRICAS

Eje b del estator

Eje magntico del rotor Posicin del rotor en t= O

Figura P.1.12.

1.17. Para el sistema de la Figura 1.43 del captulo, los valores de las inductancias son:
LII

=5+2cos2e

; Ln=3+cos2e

; L12= 10cosO

si los devanados se alimentan con corrientes continuas de valores: il = 1 A; i2= 0,5 A. Calcular: a) Energa magntica almacenada en funcin de O; b) Par mecnico desarrollado en funcin de O. [Resp.: a) Wm = 2,875 + 1,25 cos 2e + 5 cos e; b) T = - 2,25 sen 2e - 5 sen e.] 1.18. El dispositivo electromagntico mostrado en la Figura 1.43 del captulo tiene una inductancia mxima y mnima en el devanado del rotor de 0,6 H y 0,3 H, respectivamente, y los valores mximos y mnimos de la inductancia correspondiente del esttor son de 1 H Y 0,5 H, respectivamente. La inductancia mutua mxima es de 0,7 H. Ambos devanados llevan una corriente constante de valor amperios. a) Calcular el par cuando e = 45. b) Si el rotor se mueve lentamente desde e = 90 hasta 0=0, calcular: 1) trabajo mecnico realizado, 2) cambio en la energa magntica almacenada, 3) entrada elctrica. c) Si el rotor gira a una velocidad de 100 rad/s, calcular las f.e.m.s. el Y e2 producidas en las bobinas cuando 0=45. [Resp.: a) -1,79 N.m.; b) 1)2,2 Julios, 2) 2,2 Julios, 3) 4,4 Julios; c) -141 V, -112 V.]

Ji

1.19. Una mquina elctrica tiene forma cilndrica tanto en la estructura del esttor como en la del rotor. Los valores de las inductancias son:
LII

(esttor) = 0,1 [H] ; L22 (rotor) = 0,04 [H]

; LI2 = 0,05 cos

e [H]

donde e expresa el ngulo que forman los ejes de ambos devanados. a) Si la mquina gira a una velocidad (1)m = 200 rad/s y por uno de los devanados circula una corriente 10 sen 200t, cul ser la f.e.m. mxima (de pico) inducida en la otra bobina? b) Supngase que los devanados se conectan en serie y circula por ellos una corriente 10 sen 2oot. Para qu velocidades del rotor desarrollar la mquina un par medio? c) Cul es el valor mximo del par medio que puede obtenerse en el caso b)?
NOTA: Despreciar las resistencias elctricas de los devanados. [Resp.: a) 100 V; b) O; 2w = 2 . 200 = 400 rad/s; c) 1,25 N.m.]

CAPTULO

l.

CIRCUITOS

MAGNTICOS

y CONVERSIN

DE ENERGA

83

1.20. Una mquina elctrica con salientes magnticos tanto en el esttor como en el rotor tiene las siguientes inductancias: L (esttor) = 0,75 + 0,35 cos 2e Henrios ; LZ2 (rotor) = 0,5 + 0,2 cos L2 (esttor-rotor) = 0,8 cos Henrios

2e Henrios

Las resistencias de los devanados son despreciables. Si por el devanado del esttor circula una corriente i (t) = sen 314t, y el rotor est en cortocircuito, calcular la corriente it) que circular por el rotor y el par resultante, cuando e = 135. [Resp.: i2(t) = 1,6 sen 314t; T= 0,034 (cos 628t - 1).]

fi

BIOGRAFAS
1. BARKHAUSEN, Heinrich (1881-1956). Fsico alemn. Catedrtico de Comunicaciones en la Universidad de Dresden (1911). Su mayor contribucin a la fsica se refiere a la magnetizacin del hierro. Al someter al hierro a un campo magntico continuamente creciente, su magnetizacin se incrementa a saltos, y no continuamente. Esos saltos van acompaados de sonidos que, previamente amplificados, se pueden escuchar en un altavoz, como una serie de chasquidos. Este efecto Barkhausen se explic finalmente, al llegarse a conocer que el hierro est constituido por unas zonas o dominios, en los que los espines estn alineados; la alineacin de las zonas ocasiona rozamientos entre ellas, provocando los chasquidos que se pueden escuchar en el altavoz. 2. BELL, Alexander Graham (1847-1922). Fsico ingls-americano. Estudi en Londres y ms tarde, en Canad, estuvo perfeccionando el sistema de enseanza para sordomudos, inventado por su padre.Fue nombrado profesor de fisiologa vocal en Bastan (1872). Se interes en la produccin mecnicadel sonido, basando su trabajo en las teoras de Helmholtz. Invent ms tarde, en 1876, el telfono, solicitando el mismo da que Elisha Gray la patente correspondiente. Mejor tambin el fongrafo de Edison, invent la balanza de induccin, se interes por la aeronutica e hizo experimentosen relacin con el aire acondicionado. No descuid en ningn momento sus estudios pedaggicos,sobre todo en lo relacionado con la enseanza de los sordomudos. 3. BOHR, Niels Henrik David (1885-1962). Fsico dans. Estudi en la Universidad de Copenhague y ampliestudios en Cambridge, donde trabaj con Rutherford. En 1916 fue nombrado catedrtico de Fsicaen Copenhague. Basndose en las teoras de Rutherford y en la teora cuntica de Planck, demostr un modelo del tomo de hidrgeno que explicaba las lneas espectrales descubiertas porFraunhofer y las regularidades descritas por Balmer. El modelo creado por Bohr fue completadopor Sornmerfeld considerando las rbitas de los electrones de forma elptica. Premio Nobel de Fsica en 1922. En 1943, durante la segunda Guerra Mundial, se traslad a los EE. UU. para evitar ser encarcelado por los alemanes; trabaj en el proyecto de la bomba atmica en los lamoshasta 1945. Bohr trabaj incansablemente en favor del desarrollo de la energa atmica para usos pacficos. 4. FOUCAULT, Jean Bernard Lean (1819-1868). Fsico francs. Estudi Medicina hasta doctorarse, peroabandonesta carrera para dedicarse exclusivamente a las ciencias fsicas y, sobre todo, a la ptica(trabajandocon Arrnand Fizau). Son importantes sus experimentos para determinar la velocidadde la luz, demostrando que la velocidad en el aire es mucho mayor que en el agua. El nombre deFoucaultse asocia ms a menudo con una serie de experimentos espectaculares que empezaron en 1851para estudiar el movimiento de rotacin de la Tierra con ayuda del pndulo. 5. FROELlCH, Oskar (1843-1909). Ingeniero elctrico suizo. Estudi en Berna y Konigsberg, Trabaj como ingeniero en la Casa de Siemens y Halske (1873). En 1902 fue nombrado profesor de metalurgiaprctica y electroqumica de la Escuela Superior Industrial de Charlotenburgo. Public varioslibrosde electricidad y magnetismo y otro de mquinas dinamoelctricas. Fue el primero en dar una aproximacin matemtica a la curva de imanacin de un material ferromagntico, para simplificarel modelo del circuito magntico de una mquina elctrica.

84

MQUINAS ELCTRICAS

GILBERT,William (1540-1603). Fsico y mdico ingls. Ejerci la medicina en Londres; ms tarde fue mdico de la reina Isabel y del rey Jacobo 1, de los cuales recibi una pensin anual para sus investigaciones cientficas. Su obra maestra fue De magnete magneticisque corporibus, en la que resuma todos los conocimientos que en esa poca se conocan sobre el magnetismo terrestre. La gran contribucin de Gilbert consisti en considerar a la Tierra como un inmenso imn esfrico, por lo que las agujas imantadas apuntaban hacia los polos magnticos terrestres. Tambin elabor teoras sobre la estructura del universo que fueron avanzadas y atrevidas para aquella poca. 7. HOPKINSON, John (1849-1898). Ingeniero y fsico ingls. Estudi en el Trinity College de Cambridge y ms tarde en Londres (1871). Entre 1872 y 1878 trabaj en una fbrica de faros en Birminghan, introduciendo grandes perfeccionamientos en estos aparatos, y luego se estableci como ingeniero consultor en Londres. En 1890 fue nombrado catedrtico del recin fundado Laboratorio Siemens en el King' s College de Londres. Hopkinson tena una gran formacin terica y mostraba una gran habilidad prctica, por lo que son notables sus investigaciones tericas y la solucin concreta de problemas de ingeniera. Escribi ms de sesenta artculos, con una gran maestra y conocimiento de la materia, y varios libros sobre la corriente alterna y mquinas elctricas. Sus trabajos fundamentales se refieren al estudio de la magnetizacin del hierro, asentando las bases cientficas para la construccin y el clculo de las mquinas elctricas (junto con su hermano Edward); fue el primero que estudi con rigor la estabilidad esttica del acoplamiento de alternadores y las condiciones que deban cumplirse para poder efectuar dicho acoplamiento. Fue ingeniero consultor de la English Edison Company, donde su misin era redisear las dinamos ineficientes construidas por la Compaa Edison americana. Desgraciadamente, muri prematuramente en un accidente de montaa en una ascensin a los Alpes junto con tres de sus hijos. 8. MORSE, Samuel Finley Breese (1791-1872). Inventor americano. Graduado en Yale, estudi arte en Inglaterra. Se aficion a los experimentos elctricos y fue ayudado por Henry en todas sus dudas cuando estuvo construyendo su clebre telgrafo (1844). El primer mensaje de Morse fue Qu ha creado Dios?, que envi en una clave de puntos y rayas invencin suya, y que por ello an se denomina cdigo Morse. Morse nunca reconoci la ayuda de Henry en sus experimentos. Realmente la idea del telgrafo se debe a varios cientficos: Henry en EE. UU., y Wheatstone, Gauss y Weber en Europa. El mrito de Morse fue crear su celbre cdigo. 9. Prcou, Romuald Victor (1855-1942). Ingeniero francs. Estudi en la Escuela Central de Artes y Oficios (1877). En ese mismo ao realiz una instalacin de alumbrado por arco elctrico alimentada por una dinamo Gramme. En la Exposicin Internacional de Pars de 1881 fue invitado por un grupo de financieros franceses para que hiciera un informe sobre las primeras lmparas Edison expuestas en la Feria y los sistemas de generacin y distribucin de energa elctrica para instalaciones de alumbrado. Su informe favorable hizo que se estableciera la Compaa Edison francesa en 1882 y Picou se encarg de la supervisin de la construccin de la fbrica de lmparas en Ivry. Unos meses ms tarde se traslad a los EE. UU., donde conoci a Edison y aprendi a disear y proyectar centrales elctricas. A su vuelta a Francia en 1883 se le nombr director de la planta de Ivry. Como ingeniero proyect gran nmero de centrales elctricas para la Sociedad Lebon y otras compaas. Fue ingeniero jefe responsable de los servicios elctricos en las Exposiciones de Pars de 1889 y 1900. Autor de varios libros de texto sobre mquinas elctricas, distribucin y transporte de la electricidad, etc. Fue profesor de Electrotecnia durante diez aos en la Ecole National des Ponts et Chausses. Miembro fundador de la sociedad francesa de Electricistas, siendo Presidente de la misma en 1898. Presidente del Comit Electrotcnico Francs, perteneci a la Oficina de Control de las Instalaciones Elctricas y obtuvo grandes premios y condecoraciones: oficial de la Legin de Honor, Premio Plant de la Academia de Ciencias, Medalla Mascart. 10. ROTERS,Herbert C. (1903-1973). Ingeniero elctrico americano. Se gradu en el Instituto de Tecnologa Stevens y ms tarde, en 1930, obtuvo el ttulo de Master en Ciencias en el MIT. Fue profesor adjunto de Ingeniera Elctrica en Stevens durante ms de quince aos. Escribi en 1941 el libro Electromagnetic Devices, que fue y sigue siendo un texto de referencia sobre circuitos magnticos para muchos ingenieros, habendose traducido a varios idiomas, entre ellos el ruso. Gan fama mundial por sus estudios sobre el motor de histresis. En 1947 fue premiado por el 6

CAPTULO l.

CIRCUITOS MAGNTICOS Y CONVERSIN DE ENERGA

85

11.

AlEE por sus investigaciones sobre mquinas elctricas. Fue presidente de la Empresa Hysteresis Motor Research, una firma especializada en el diseo y desarrollo del motor de histresis. WEBER, Wilhelm Eduard (1804-1891). Fsico alemn. Estudi en la Universidad de Halle, donde obtuvo el grado de Doctor en 1826. En 1831 fue nombrado catedrtico de Fsica en Gotinga, donde permaneci hasta 1837, fecha en que fue expulsado de la Universidad por protestar contra el rey de Hannover (duque de Cumberland), que haba derogado la constitucin. En 1833 invent un primitivo telgrafo electromagntico. En 1843 acept la Ctedra de Fsica en Leipzig y seis aos ms tarde volvi a Gotinga para hacerse cargo de su antiguo puesto, en el que permaneci el resto de su vida. Colabor con el gran matemtico Gauss en teoras del magnetismo. Introdujo en 1846 un sistema lgico de unidades elctricas, de un modo anlogo al que unos aos antes haba desarrollado Gauss con las unidades magnticas.

REFERENCIAS
1. 2. CHATELAIN,1.: Trait d'lectricit, vol. X, Machines lectriques. ditions Georgi, Lausanne, 1983. DANIELS,A. R.: lntroduction to Electrical Machines. MacMillan Press, London, 1976. DELTORO, V.: Electric Machines and Power Systems. Prentice-Hall Inc., Englewood Cliffs, N. J., 1985. EL-HAWARY,M. E.: Principies of Electric Machines with Power Electronic Applications. Reston Book, Prentice-Hall, Englewood Cliffs, N. J., 1986. ENGELMANN, R. H.: Static and Rotating Electromagnetic Devices. Marcel Sekker, Inc., New York, 1982. FITZGERALD,A. E.; KINGSLEY,CH. Jr.; UMANS, S. D.: Electric Machinery, fourth ed. (SI). McGraw-Hill Book Co., New York, 1985. GEHMLICH,D. K.; HAMMOND,S. B.: Electromechanical Systems. McGraw-Hill Book Co., New York, 1967. GbNEN, T.: Electrical Machines. Power International Press, California, 1998. GOURISHANKAR, V.: Conversin de Energa Electromecnica. Representaciones y Servicios de Ingeniera S. A., International Textbook Co., Mxico, 1969. GURU,B. S., HIZIROGLU, H. R.: Electrical Machinery and Transformers, 2." ed. Oxford University Press, New York, 1995. MATSCH,L. W.: Mquinas electromagnticas y electromecnicas. Representaciones y Servicios de Ingeniera S.A., International Textbook Co., Mxico, 1974. MORGAN,A. T.: General theory of Electrical Machines. Heyden, London, 1979. NAGRATH,l. J.; KOTHARI,D. P.: Electric Machines. Tata McGraw-HiIl Pub., New Delhi, 1985. ROTERS,H. C.; Electromagnetic devices. John Wiley Sons, New York, 1941. SARMA,M. S.: Electric Machines, 2." ed. PWS Publishing Co. International Thomsom Pub., Boston, 1996. SAY, M. G.: lntroduction to the Unified Theory of Electromagnetic Machines. Pitman, London, 1971. SCHMITZ,N. L.; NOVOTNY,D. W.: lntroductory Electromechanics. The Ronald Press Co., New York, 1965. SLEMON, G. R.; STRAUGHEN, A.: Electric Machines. Addison-Wesley Pub., Reading, Massachusetts, 1980. STEIN,R.; HUNT, W. T.: Electric Power System Components: Transformers and Rotating Machines. Van Nostrand Reinhold Co., New York, 1979. STEVEN, R. E.: Electrical Machines and Power Electronics. Van Nostrand Reinhold Co., England, 1983.

3. 4. 5. 6. 7. 8. 9. 10.
11. 12.

13.
14. 15. 16. 17. 18. 19. 20.

CAPTULO

Principios generales de las mquinas elctricas

2.1. INTRODUCCIN
Las mquinas elctricas son el resultado de una aplicacin inteligente de los principios del electromagnetismoyen particular de la ley de induccin de Faraday. Las mquinas elctricas se caracterizan por tener circuitos elctricos y magnticos entrelazados. Durante todo el proceso histrico de su desarrollo desempearon un papel rector, que determinaba el movimiento de toda la ingeniera elctrica, merced a su aplicacin en los campos de la generacin, transporte, distribuciny utilizacin de la energa elctrica. Las mquinas elctricas realizan una conversin de energa de una forma a otra, una de las cuales, al menos, es elctrica. En base a este punto de vista, estrictamente energtico, es posible clasificarlas en tres tipos fundamentales: 1. GENERADOR: que transforma la energa mecnica en elctrica. La accin se desarrolla por el movimiento de una bobina en un campo magntico, resultando una f.e.m. inducida que al aplicarla a un circuito externo produce una corriente que interacciona con el campo y desarrolla una fuerza mecnica que se opone al movimiento. En consecuencia, el generador necesita una energa mecnica de entrada para producir la energa elctrica correspondiente. 2. MOTOR: que transforma la energa elctrica en mecnica. La accin se desarrolla introduciendo una corriente en la mquina por medio de una fuente externa, que interacciona con el campo produciendo un movimiento de la mquina; aparece entonces una f.e.m. inducida que se opone a la corriente y que por ello se denomina fuerza contraelectromotriz. En consecuencia, el motor necesita una energa elctrica de entrada para producir la energa mecnica correspondiente. 3. TRANSFORMADOR: que transforma una energa elctrica de entrada (de c.a.) con determinadas magnitudes de tensin y corriente en otra energa elctrica de salida (de c.a.) con magnitudes diferentes. Los generadores y motores tienen un acceso mecnico y por ello son mquinas dotadas de movimiento, que normalmente es de rotacin; por el contrario, los transformadores son mquinas elctricas que tienen nicamente accesos elctricos y son mquinas estticas. Cada mquina en particular cumple el principio de reciprocidad electromagntica, lo cual quiere decir que son reversibles, pudiendo funcionar como generador o como motor (en la prctica, existe en realidad alguna diferencia en su construccin, que caracteriza uno u otro modo de funcionamiento). El estudio de las mquinas elctricas ha experimentado grandes cambiosen su exposicin a lo largo de la historia. El anlisis tradicional consista en estudiar 87

88

MQUINAS ELC[~RJCAS

independientemente cada una de las mquinas por separado, destacando las diferencias entre ellas desde el punto de vista de funcionamiento, diseo, construccin, etc.; actualmente, y a partir de los estudios de Park (17) YKron (13), se han expuesto teoras generalizadas y unificadas, dando lugar a una serie de textos ya clsicos en esta materia debidos a White-Woodson (24), Adkins (1), Jones (12), Gibbs (8), etc., donde se destacaban ms las analogas que las diferencias, permitiendo tambin un anlisis transitorio de estas mquinas, a base de ecuaciones de circuito del tipo matricial e incluso tensorial; este enfoque tiene un gran atractivo en cursos de doctorado pero resulta poco/fsico para aquellos ingenieros cuya formacin requiere un conocimiento prctico de las mquinas en cuanto a sus aplicaciones tecnolgicas se refiere. Por ello, en este libro sobre mquinas elctricas se ha preferido seguir un mtodo que se acerca ms a la enseanza tradicional, pero destacando en este captulo aquellos aspectos que son comunes a todas las mquinas elctricas, de tal forma que en lecciones posteriores pueda hacerse un anlisis particular de cada tipo de mquina, dejando a un lado aquellos aspectos de diseo y construccin que pertenecen a reas ms especficas de la ingeniera. El captulo comienza describiendo los elementos bsicos que incorporan las mquinas elctricas rotativas, en sus aspectos elctricos magntico y mecnico. Se muestran detalles constructivos de la forma del estampado de las chapas magnticas, ranuras, etc. Se analiza el comportamiento de los colectores de anillos y del colector de delgas o conmutador, haciendo especial hincapi en la forma de ondas que se obtienen en los mismos. A continuacin se incluye una descripcin sucinta de la forma de los devanados de las mquinas elctricas, pero sin llegar a estudiar las leyes de los bobinados, que pertenecen a reas ms especializadas de la ingeniera elctrica. Se analizan los conceptos generales de prdidas y calentamiento, describiendo los tipos de aislamientos empleados de acuerdo con la Norma UNE EN 600341:1995 y CEI-85 (1984). Se indica el concepto de potencia asignada de una mquina, las clases de proteccin IP y los tipos de servicio a los que se destinan. Tambin se hace un anlisis del rendimiento de las mquinas elctricas, destacando la variacin del rendimiento en funcin de la potencia suministrada. Comienza luego el anlisis de las f.m.m.s. y campo magntico en el entrehierro producido por diferentes configuraciones de los devanados: concentrado y distribuido. Se realiza con gran detalle la exposicin del teorema de Ferraris y su importancia en la produccin de campos magnticos giratorios. Se expone el teorema de Leblanc, en el que se identifica un campo alternativo con un doble campo giratorio. A continuacin se hace un anlisis general de la f.e.m. inducida en una mquina elctrica y los factores que intervienen en la mejora de la calidad de la onda producida; en especial se estudian los armnicos de f.e.m. y su eliminacin utilizando el acortamiento y distribucin de las bobinas. Tambin se estudia la creacin del par electromagntico en una mquina elctrica a partir de los fasores espaciales de f.m.m. Posteriormente, y de acuerdo con la relacin entre las frecuencias de inductor e inducido y la velocidad del rotor, se hace una clasificacin general de las mquinas y seguidamente se describen de un modo cualitativo los principales tipos de mquinas. En captulos posteriores se hacen los estudios detallados de cada una de las mquinas elctricas.

2.2.

ELEMENTOS BSICOS DE LAS MQUINAS ELCTRICAS

En trminos generales, se puede decir que una mquina elctrica rotativa se compone de dos partes, como se indica esquemticamente en la Figura 2.1. Hay una parte fija, que se denomi-

CAPTULO 2.

PRINCIPIOS GENERALES DE LAS MQUINAS ELCTRICAS

89

Figura 2.1. La mquina elctrica como convertidor de energa.

na esttor y que tiene forma cilndrica; en el caso de mquinas de gran velocidad, dicho cilindroes largo en comparacin con su dimetro, mientras que para las de pequea velocidad es relativamente corto. En la cavidad del esttor se coloca el rotor, que, 'como su nombre indica, es la parte giratoria de la mquina. El rotor se monta en un eje que descansa en dos rodamientoso cojinetes; stos pueden estar montados en sendos pedestales que se apoyan en la bancada, o formar parte de las culatas o tapas que estn sujetas a la carcasa del esttor. El espaciode aire que separa el esttor del rotor, necesario para que pueda girar la mquina, se denominaentrehierro, siendo el campo magntico existente en el mismo el que constituye el mediode acoplamiento entre los sistemas elctrico y mecnico. Normalmente tanto en el esttor como en el rotor existen devanados hechos con conductores de cobre por los que circulan corrientes suministradas o cedidas a un circuito exterior que constituye el sistema elctrico. Uno de los devanados tiene por misin crear un flujo en el entrehierro y por ello se denomina inductor, y tambin excitacin o campo. El otro devanado recibe el flujo del primero y se inducen en l corrientes que se cierran por el circuitoexterior y se denomina inducido. Lo mismo puede situarse el inductor en el esttor y el inducido en el rotor o viceversa; lo que realmente cuenta es el movimiento relativo entreambos devanados y tericamente puede elegirse cualquiera de ambas soluciones, aunque en la prctica su situacin la determinan las condiciones tecnolgicas de facilidad de construccin,aislamiento, refrigeracin, etc. Desde el punto de vista de la construccin, el esttortiene su parte exterior recubierta por la carcasa o culata, estando constituida por un cilindrohueco al que se unen los pies y los dispositivos de fijacin de la mquina (Fig. 2.2). En las mquinas pequeas se construye en forma de un monobloque de fundicin de hierro coladoy en las mquinas grandes se compone de dos o cuatro partes ensambladas. La forma de la carcasa vara de un constructor a otro, estrndo condicionada por el sistema de refrigeracin y por la proteccin exigible en el lugar de trabajo (contra objetos slidos, polvo, agua,antideflagrantes, etc.) En los lados de la carcasa van colocadas las tapas que cerrarn el motor y que tienen un hueco central en su interior para alojar los cojinetes del rotor. El esttor y el rotor se construyen con material ferromagntico, de tal forma que para evitarprdidasen el hierro suelen realizarse con chapas magnticas de acero al silicio, convenientementeranuradas para alojar en su interior los devanados correspondientes. En la Figura 2.3 se muestran diversas formas de chapas magnticas empleadas en la construccinde mquinaselctricas; se observa que las que configuran el rotor disponen en el centro de un agujerocircular, para introducir por el mismo el eje, disponiendo tambin de su corres-

90

MQUINAS ELCTRICAS

Anillo de elevacin

Chavetero Eje Rodamiento

Figura 2.2. Aspectos constructivos de una mquina elctrica.

pondiente chavetero para que la sujecin sea perfecta. En las mquinas grandes existen tambin unos agujeros en las chapas distribuidos en su superficie que hacen de conductos de ventilacin, para que penetre por ellos el aire producido por un ventilador acoplado al mismo eje, evitndose en gran parte el aumento de temperatura que se obtendra debido a las prdidas en el hierro. Las ranuras para alojar los conductores de los devanados pueden ser del tipo abierto, semicerrado y cerrado, como se indica en la Figura 2.4. Exceptuando el caso de las ranuras cerradas, que se emplean casi exclusivamente en la construccin del devanado del rotor de los motores de induccin (asncronos) enjaula de ardilla, las ranuras suelen presentar una seccin del tipo rectangular; las abiertas se utilizan en las grandes mquinas, donde el devanado se prepara con sus bobinas totalmente acabadas en una bobinadora o tomo de baja velocidad, de tal forma que permite la colocacin de la bobina entera en la ranura, cerrando sta por medio de un calzo aislante; las ranuras semicerradas se emplean en las mquinas pequeas, donde el devanado se coloca, haciendo deslizar los hilos conductores de la bobina, uno por uno por la garganta de entrada.

Chapa de estator Chapas de estator y rotor

Pieza polar del rotor

Chapas de transformadores

Pieza polar del estator

Figura 2.3. Tipos de chapas magnticas.

CAPTULO 2.

PRINCIPIOS GENERALES DE LAS MQUINAS ELCTRICAS

91

Diente

Conductores
a) Ranura abierta b) Ranura semicerrada e) Ranura cerrada

Figura 2.4. Tipos de ranura.

Desde el punto de vista de la configuracin fsica, las mquinas elctricas adoptan tres formas bsicas, como se indica en la Figura 2.5. En el caso a) se tienen dos superficies totalmentecilndricas, con un entrehierro uniforme; esta disposicin se encuentra en las mquinas asncronas, estando situado el inductor en el primario, y tambin se emplea en los turboaltemadores de las centrales trmicas, situando el inductor en el rotor. En los casos b) y e) la superficie del esttor o del rotor presenta unos salientes magnticos denominados polosque estn provistos a su vez de unas expansiones o cuernos polares (Fig. 2.5c); en estos polos se sita siempre el devanado inductor, recorrido normalmente por c.c. y creando un campo magntico que puede asimilarse al que produce un imn permanente. La disposicin indicadaen la Figura 2.5b es la empleada en las mquinas sncronas y la de la Figura 2.5c se utiliza en las mquinas de c.c. Todos los esquemas de la Figura 2.5 presentan un circuito magntico que forma dos polos;se dice entonces que la mquina es bipolar. La mquina mostrada en la Figura 2.6a es tambinbipolar, pero existen mquinas con un nmero superior de polos, denominadas multiLnea neutra
I

a) Esttor y rotor cilndricos

b) Esttor cilndrico y rotor

con polos salientes

e) Esttor con polos salientes y rotor cilndrico

Figura 2.5. Configuraciones bsicas de esttor-rotor.

92

MQUINAS ELCTRICAS

polares, donde los polos N y S se suceden de una forma alternativa; por ejemplo, en la Figura 2.6b se muestra una mquina tetrapolar. La lnea media entre un polo y el siguiente se denomina lnea neutra, y la distancia entre dos polos consecutivos se denomina paso polar. En una mquina bipolar, por ejemplo la de la Figura 2.6a, se produce un ciclo completo magntico en una vuelta completa del rotor; sin embargo, para una mquina multipolar con p pares de polos, en una revolucin completa del rotor se recorren p ciclos magnticos completos; por ej., para la mquina representada en la Figura 2.6b, se tiene p = 2 (es decir, 4 polos), y una revolucin del rotor corresponde a dos ciclos magnticos. Como quiera que una revolucin del rotor corresponde a un ngulo geomtrico de.360, y un ciclo magntico corresponde a un recorrido de 360 magnticos, se concluye que para una mquina de p pares de polos un ngulo geomtrico (X corresponde a un ngulo magntico {} dado por la siguiente igualdad:
{}=p.(X

(2.1)

La expresin anterior es muy importante en el estudio de las mquinas elctricas ya que permite relacionar los ngulos geomtricos medidos por un observador con los grados magnticos que efectivamente v la mquina. Por ejemplo, para la mquina bipolar (p = 1) de la Figura 2.6a, el ngulo geomtrico entre los polos N YS es de 180 que corresponde a medio ciclo magntico, es decir a 180 magnticos, sin embargo para la mquina tetrapolar (p = 2) de la Figura 2.6b, el ngulo geomtrico entre dos polos N Y S consecutivos es de 90, que corresponde a medio ciclo magntico, es decir, a 180 magnticos. En lo sucesivo, si no existe una indicacin especial, expresaremos los ngulos slo en grados magnticos, tambin llamados grados elctricos, ya que las f.e.m.s. inducidas en las bobinas dependen de la variacin entre posiciones magnticas.

2.3.

COLECTOR DE DELGAS Y COLECTOR DE ANILLOS

Se ha indicado en el apartado anterior que una mquina elctrica rotativa est formada por dos partes denominadas esttor y rotor, que llevan unos devanados que reciben los nombres de
Lnea neutra (interpolar)

Paso polar = 1800 geomtricos = 1800 magnticos = 1800 elctricos a) Mquina bipolar

Paso polar = 900 geomtricos = 1800 magnticos = 1800 elctricos b) Mquina tetrapolar

Figura 2.6. Mquinas bipolares y tetrapolares. ngulos geomtricos y elctricos.

CAPTULO 2.

PRINCIPIOS GENERALES DE LAS MQUINAS ELCTRICAS

93

inductore inducido. Para introducir o sacar corrientes de los bobinados situados en el esttor de la mquina basta con hacer unas conexiones fijas directas desde el sistema exterior a estos devanados;sin embargo, para realizar esta operacin con las bobinas del rotor es preciso recurrir a sistemas colectores, que difieren entre s, segn sea la mquina de c.a. o de c.c. Para ver el funcionamientode estos colectores se va a considerar el estudio de un generador elemental,constituido por un imn en el esttor, que hace de inductor, y una espira en el rotor, que hace de inducido. Consideremos el esquema de la Figura 2.7, donde una espira gira a una velocidadwm rad/s dentro del campo magntico B de un imn permanente. Los extremos de la espiravan a parar a dos anillos de bronce sobre los que rozan unas escobillas de grafito, a las cualesse conecta el circuito exterior, compuesto por un receptor de energa, simulado por una resistenciade carga R. Los vectores B, induccin magntica, y S, superficie de la espira, forman en un momento determinado un ngulo elctrico po; siendo p el nmero de pares de polos de la mquina y a el ngulo geomtrico correspondiente. De acuerdo con la ley de Faraday, la f.e.m. inducida en la espira al girar dentro del campo magntico del imn ser:
e = --

dI!> d =dt dt

(B

S cos prx)

(2.2)

teniendo en cuenta que se cumple:


W
m

d n = - = 2n-

dt

60

(2.3)

donde n indica el nmero de r.p.m. de la espira, y tomando como referencia p = Oen t = O, se obtendr la siguiente f.e.m. en el inducido:
(2.4)

queal comparar con la expresin general de una f.e.m. alterna de pulsacin co= 2nf, siendo! la frecuencia en Hz, expresada por: e = Em sen tot
(2.5)

~ \ Escobillas

Polos inductores f.e.m. generada en la espira

R (carga externa)

Figura 2.7.

Colector de anillos.

94

MQUINAS ELCTRICAS

se deduce que la relacin entre las r.p.m. de la mquina y la frecuencia de la f.e.m. alterna obtenida es: w es decir: np
f= 60 (2.7)

= 2nf = pw = p2n 60
ni

(2.6)

En consecuencia, en la espira se obtiene una f.e.m. alterna, cuya frecuencia es proporcional a la velocidad de giro y al nmero de pares de polos de la mquina. Por otra parte, al estar las escobillas rozando los anillos colectores, se consigue que la corriente que circula por el circuito exterior sea de la misma forma que la que se obtiene en la espira del inducido. Para evitar falsos contactos, existe un muelle que presiona ligeramente la escobilla a la superficie cilndrica del anillo, como se observa en la Figura 2.8. Las escobillas se hacen modernamente con grafito electroltico, aunque a veces son metalografticas; la eleccin de este elemento se debe a su ventaja de poseer buena conductividad elctrica a la par de no desgastar excesivamente los anillos. Existe otra forma de enviar la f.e.m. inducida en la espira a un circuito exterior, y es emplear el llamado colector de delgas, cuyo funcionamiento, en el caso ms elemental, se puede explicar con ayuda del esquema de la Figura 2.9. Se observa que ahora los extremos de la espira van a parar a un anillo formado por dos segmentos de cobre, denominados delgas, aislados entre s y del eje de la mquina por medio de un cilindro de mica. Sobre las delgas van colocadas unas escobillas, fijas en el espacio, a las cuales se conecta el circuito exterior. La misin del colector de delgas es obligar a que la corriente que atraviesa el circuito exterior circule siempre en el mismo sentido, de tal forma que aunque la f.e.m. inducida en la espira sea senoidal (c.a.), y como ya se ha demostrado en el caso anterior, la corriente que atraviesa el circuito sea unidireccional; esta operacin se denomina rectificacin, y se realiza de una forma automtica con el colector de delgas. La posicin de las escobillas no es indiferente, ya que para rectificar totalmente la c.a. del inducido es necesario colocar las escobillas tal como muestra la Figura 2.9, es decir, de tal modo que la f.e.m. inducida en la espira sea igual a cero en el momento en el que la escobilla pasa de una delga a la otra. Para ver ms claramente el proceso de rectificacin, en la Figura 2.10 se ha representado una revolucin completa de la espira, habindose sustituido la resistencia R del circuito de la Figura 2.9 por un milivoltmetro con el cero en el centro, para hacer ms patente la polaridad de la tensin resultante entre las escobillas de la mquina.

Eje

Conductor de salida

Figura 2.8. Anillo deslizante y escobilla.

CAPTULO

2.

PRINCIPIOS GENERALES DE LAS MQUINAS ELCTRICAS

95

Lneas de induccin b

Escobillas

+
Figura 2.9. Colector de dos delgas. En la parte inferior de la Figura 2.10 se ha representado la tensin correspondienteen cada instante de tiempo. Para comprender la forma de esta onda generada, debe suponerse que la induccinproducida por el imn se distribuye senoidalmenteen el entrehierro de la mquina, es decir, la induccin es mxima debajo de cada polo y nula en la zona de la lnea neutra. La f.e.m.inducidaen la espiravieneexpresadapor la leyde Faradayy es una f.e.m.de movimiento: d<l> e=--= dt

(vxB)dl

(2.8)

;.

En el caso de la Figura 2.10, en el instante ti' el plano de la espira es vertical, por lo que el flujo magntico es mximo y en consecuencia la f.e.m. inducida es nula, lo que est de acuerdocon (2.2), que es en definitiva la parte izquierda de (2.8). Al mismo resultado se llega analizando el trmino integral de (2.8); en este caso en los tramos ab y cd de la espira se inducen unas f.e.m.s. nulas, ya que en esta situacin las inducciones en ambas ramas de la espira son nulas por estar situadas en la lnea neutra, y en consecuencia el voltmetro de la Figura 2.10a sealar cero voltios. Al pasar la espira desde el instante ti al instante t2, la f.e.m. inducidair aumentando progresivamente, ya que va elevndose la induccin en las ramas de la espira, al acercarse las mismas al centro de los polos. De hecho, en el instante t2 la f.e.m. inducidaser mxima ya que corresponde a la mxima induccin en las ramas de la espira, la f.e.m.en la parte oscura de la espira se dirigir de b a a, en el sentido atrs-adelante, que es el sentidodel producto vectorial v x B que aparece en (2.8), mientras que en la parte clara de la espira la f.e.m. inducida ir de d a e, sentido adelante-atrs, y de ah el sentido de la desviacin que aparece en el voltmetro de la Figura 2.10 en el instante [2' que se dirige de la escobilla negra a la escobilla blanca. Como quiera que al moverse la espira, la escobilla negra/blanca siempre hace contacto con el conductor que se encuentra bajo el polo norte/sur, la polaridadde la escobilla negra/blanca ser siempre positiva/negativa, por lo que la corriente se dirigir siempre en el circuito exterior desde la escobilla negra a la escobilla blanca, lo que significa que la tensin de salida tiene carcter unidireccional. De este modo con un colectorde delgas se consigue que la forma de onda que se obtiene en el circuito exterior sea diferente a la forma de onda que existe en el inducido. En particular, una seal de c.a. en el inducido se transforma en unidireccional (c.c.) en el circuito exterior.

96

MQUINAS ELCTRICAS

b) Instante 12

e) Instante 13

d) Instante 14

e = Ovoltios

e = +E voltios

Figura 2.10. Funcionamiento del colector de delgas y tensin obtenida entre escobillas.

En la prctica, para obtener una c.c. que tenga menos rizado (menor oscilacin) se aumenta el nmero de delgas del colector,con ms bobinas en el inducido.En la Figura 2.11a se muestra un colector con cuatro delgas y dos bobinas, observndose en la Figura 2.11b que la forma de onda que se obtiene posee menos variacin, y se acerca ms a una c.c. constante. En las mquinas reales, con el fin de que las ondulaciones de la f.e.m. se reduzcan an ms y de que la d.d.p. entre las delgas no llegue a ser tan alta que produzca chispas en el colector, se procura que el nmero de delgas sea relativamente grande. En la Figura 2.12 se muestra un colector de delgas, tambin llamado conmutador. Las delgas son de cobre endurecido a las que se da una forma adecuada para ser montadas en una estructura circular. Cada delga va soldada y remachada con un conductor vertical o taln para que sirva de pieza de contacto con la bobina correspondiente. El remache asegura la resistencia mecnica suficien-

Figura 2.11. Inducido con dos bobinas y colector con cuatro delgas.

CAPTULO 2.

PRINCIPIOS GENERALES DE LAS MQUINAS ELCTRICAS

97

Aislamiento de mica

Tomillo de apriete

Figura 2.12. Perspectivay seccinde un colectorde delgas. te para la conexin y la soldadura mantiene un buen contacto elctrico. Para evitar que las delgas puedan salirse del colector por la fuerza centrfuga, se construyen en forma de cola de milano. Las delgas van aisladas entre s por separadores de mica y tambin estn aisladas del cilindro soporte por lminas del mismo dielctrico.

2.4. DEVANADOS
Se denominan devanados de una mquina elctrica a los arrollamientos del inductor y del inducido. El material para la realizacin de las bobinas suele ser el cobre en forma de hilo esmaltado (la misin del aislante es ofrecer una separacin elctrica entre las espiras) en las mquinaspequeas y en forma de pletina para las mquinas de gran potencia, cuyo aislamiento se realiza recubrindolas con cinta de algodn. Tambin se emplea el aluminio, pero su aplicacin es casi exclusiva de los rotores en jaula de ardilla de los motores asncronos. Los inductores de las mquinas sncronas y de las mquinas de c.c. se ejecutan en forma de arrollamientoconcentrado, devanando una bobina alrededor de los polos tal como se seala en la Figura 2.13. Este sistema se emplea tambin en los transformadores, pero con detalles ms especficos que se explicarn en el Captulo 3. Los inducidos de las mquinas de c.a. y c.c. se ejecutan en forma de arrollamientos distribuidos para cubrir toda la periferia de la mquina, situando las bobinas en las ranuras practicadasal efecto. En los albores de la electricidad estos arrollamientos se ejecutaban en forma de devanado en aniDo, ejemplo de lo cual fue la clebre dinamo de Grarnme (Fig. 2.14a), donde se arrollabael hilo sobre un anillo cilndrico recorriendo la periferia del inducido. El defecto de este tipo de bobinado es que slo se aprovechan los conductores de la periferia exterior para producir f.e.m., ya que los conductores internos no son atravesados por ninguna induccin, debido a que el campo magntico despus de saltar el entrehierro se encauza por dentro del anilloferromagntico,antes de seguir recto, cruzando el cilindro interior del anillo, que presenta mayor reluctancia magntica. Hoy da solamente se emplean los devanados en tam-

98

MQUINAS ELCTRICAS

Devanado concentrado Conductores del devanado

Figura 2.13. Perspectiva y seccin del devanado concentrado de un polo.

bor, donde la totalidad de los conductores estn colocados en la superficie exterior del cilindro ferromagntico que forma el inducido. En la Figura 2.14b se muestra un ejemplo de este tipo de bobinado aplicado a un inducido de una mquina de C.C.; se observa en este devanado que, exceptuando las dos partes frontales, todo el cobre del inducido es activo, es decir, corta o es atravesadopor la induccin y acta, por lo tanto, como generador de f.e.m. Normalmente las bobinas que forman el devanado suelen construirse fuera de la mquina, y posteriormente son encintadas antes de colocarlas en las ranuras correspondientes,con lo que se consigue una gran rapidez en la formacin del inducido. Dependiendo del nmero de ramas de bobina existente en una ranura, los devanados se clasifican en arrollamientos de una capa y de dos capas. En el arrollamiento de una capa, en cada ranura se sita un solo lado de bobina, mientras que en el de dos capas se sitan dos lados de bobina por ranura; en este ltimo caso, un lado de la bobina est colocado en la parte superior de una ranura y el otro lado se sita en la parte inferior de la otra. Los devanados pueden ser abiertos o cerrados; los devanados abiertos tienen un principio y un final y se emplean en las mquinas de c.a.; los devanados cerrados no tienen ni principio ni fin, y para sacar la corriente al exterior deben hacerse tomas intermedias (por medio de delgas), y este tipo de devanado es el que se emplea en las mquinas de c.c. Desde el punto de vista de la forma de las bobinas, los devanados pueden clasificarse en concntricos y excntricos (empleados en c.a.), y tambin en ondulados e imbricados (empleados en c.c.). Los devanados concntricos estn formados por bobinas de diferente anchura o paso, que tienen un eje comn; un ejemplo de ellos se muestra en la Figura 2.15a. Los devanados excntricos estn constituidos por bobinas de igual paso pero desfasadas entre s en el espacio, como se indica en la Figura 2.15b.

Colector de delgas
a) Devanado en anillo b) Devanado en tambor'

Figura 2.14. Tipos de inducidos.

CAPTULO

2.

PRINCIPIOS GENERALES DE LAS MQUINAS ELCTRICAS

99

.
___.
Desarrolloi

i
!
a) Devanado concntrico

Figura 2.15. Devanados concntricos y excntricos.

Los devanados imbricados son anlogos en la forma a los excntricos y se emplean en las mquinasde c.c., hacindose derivaciones al colector de delgas como indica la Figura 2.16a. Losdevanados ondulados se caracterizan porque al bobinar se va recorriendo el inducido y en consecuencia el colector, formando las ramas de las bobinas una figura de onda como se indica en la Figura 2.16b. El estudio de la teora de los bobinados no se dar en este libro, porquepertenece ms a una asignatura de Clculo y Construccin de Mquinas Elctricas; sin

)'/

LEY: )'

= YI -

Y2

e
G Desarrollo Devanado imbricado

LEY: y = YI + )'2 F Desarrollo

Figura 2.16. Devanado imbricado (arriba). Devanado ondulado (abajo).

100

MQUINAS ELCTRICAS

embargo, como normas bsicas se pueden citar los dos principios generales siguientes: a) la anchura (paso) de cada bobina debe hacerse lo ms cercana a un paso polar con objeto de que los dos lados de la bobina quedenfrente a dos polos adyacentes N y S, para obtener as la mximafe.m. inducida en cada una de las bobinas y aprovechar de este modo lo ms posible el cobre del arrollamiento; b) en los devanados de los generadores de c.a. es conveniente que el arrollamiento se prevea para que genere una fe.m. senoidal exenta lo ms posible de armnicos, y para ello es preciso distribuir el devanado en las ranuras y emplear acortamientos de paso en las bobinas.

2.5. PRDIDAS y CALENTAMIENTO


En la transformacin electromecnica de la energa que tiene lugar en una mquina elctrica, una fraccin de la potencia transformada se convierte en calor y prcticamente no se utiliza, constituyendo el conjunto las llamadas prdidas de la mquina. Desde el punto de vista fsico, una mquina tiene un circuito elctrico, constituido por los devanados del inductor e inducido, donde se producen unas prdidas por efecto Joule, llamadas vulgarmente prdidas en el cobre, por ser ste el material que suele utilizarse para la fabricacin de los bobinados (aunque tambin puede ser el aluminio); por otra parte, los arrollamientos se colocan en una estructura magntica, realizada con chapas de acero al silicio que ofrecen, debido a los campos magnticos variables, unas prdidas denominadas prdidas en el hierro; como quiera que adems la mquina ofrece un acceso mecnico, aparecen en l unas prdidas mecnicas debidas a los rozamientos o fricciones de las diversas partes del sistema. El estudio de estas prdidas es de gran inters, porque influyen sobre dos magnitudes muy importantes en la explotacin: el rendimiento y el calentamiento de una mquina elctrica. Vamos a analizar en consecuencia y ms detalladamente cada una de estas prdidas:

a) Prdidas en el cobre
Son consecuencia de la inevitable resistencia que presentan los conductores elctricos, dando lugar a una prdida en forma de calor por efecto de Joule. En trminos generales responden a la ecuacin: (2.9) en donde Rj e ij representan, respectivamente, la resistencia y corriente que corresponden al devanado i. ya sea del inductor o del inducido. Denominando Pj a la resistividad de los conductores, lj a su longitud y Sj a su seccin transversal; sustituyendo en (2.9) la resistencia por su valor p)/ Sj resulta:

r: = Ipj(~}} = Ip)j(~)s} P . Jf . (volumen)

(2.10)

llamando Jj a la densidad de la corriente, es decir, al cociente: i/sj y al ser el volumen del cobre igual a ljsj se obtiene:
Peu =

(2.11)

que expresa las prdidas en el cobre en funcin de la resistividad, densidad de ,corriente y volumen del bobinado empleado. Se observa que las prdidas son proporcionales al volumen del material y a la densidad de corriente que circula por los conductores, y esta densidad suele

CAPTULO 2.

PRINCIPIOS GENERALES DE LAS MQUINAS ELCTRICAS

101

tomar un valor que vara entre 5 Azrnm"para las mquinas pequeas a 3 Azmm!en las mquinas grandes. b)

Prdidas en el hierro

Se producenen todos los rganos de las mquinas que son recorridos por flujos variables. Como ya se ha estudiado en el captulo anterior, se componen a su vez de las prdidas por histresis y las prdidas por corrientes de FoucauIt (parsitas), y de acuerdo con (1.61) se tiene: (2.12) siendo kH Y kF constantes; f, la frecuencia; Bm' la induccin mxima; a, el espesor de las chapas magnticas; (J, la conductividad de las mismas; a, un parmetro, y vol, el volumen de hierro. Como ya se ha indicado en el captulo anterior, la forma de reducir estas prdidas es emplearncleos magnticos de acero al silicio en forma de chapas; esto disminuye el valor de las prdidas por histresis (ya que el ciclo se hace ms estrecho) y reduce las prdidas por corriente de Foucault debido a la adicin de silicio y a aislar las chapas entre s. La laminacin puede hacerse en caliente o en fro (granos orientados), resultando unas prdidas del ordende 0,8 a 1,3 W/kg a 1Tesla para las chapas ordinarias (laminadas en caliente) y de 0,4 a 0,5 W/kg a 1 Tesla para las de grano orientado. Estas prdidas se transforman en calor en la masa de hierro. e)

Prdidas mecnicas

Sondebidas a los rozamientos de los cojinetes, a la friccin de las escobillas y a la ventilacin (rozamiento con el aire). Como es lgico estas prdidas slo existen en las mquinas que disponen de un rgano giratorio. Las prdidas por rozamiento y friccin son directamente proporcionalesa la velocidad, mientras que las prdidas por ventilacin se consideran proporcionales a la 3: potencia de la velocidad. En consecuencia: (2.13) La clasificacin anterior se ha realizado teniendo en cuenta el lugar donde se producen las prdidas,pero tambin puede realizarse teniendo en cuenta la forma en que varan las mismas y as se tienen: a) Prdidas fijas: lf; b) Prdidas variables: Pv. Las prdidas fijas son aquellas que no varan aunque cambie la potencia absorbida o cedida por la mquina. A este respecto se observa que las prdidas mecnicas, segn se ha comentado en los prrafos anteriores, slo dependen de la velocidad, y como quiera que la mayorparte de las mquinas tienen velocidad constante o sensiblemente constante, se podrn considerarestas prdidas como constantes. Tambin pueden suponerse fijas las prdidas en el hierro,ya que las mquinas suelen trabajar con unos valores de B y f constantes y de acuerdo con (2.12) implicarn unas prdidas constantes. Las prdidas variables son aquellas que varan segn sea la potencia que absorbe o cede la mquina; como quiera que una variacin de potencia supone un cambio correspondiente en la corriente (la tensin de una mquina es sensiblementeconstante), aparecer una variacin tambin en las prdidas por efecto Joule en los devanados. De los prrafos anteriores se deducen las siguientes igualdades: (2.14 ) es decir, las prdidas fijas estn constituidas por las prdidas en el hierro y mecnicas, mientras que las prdidas variables son debidas al efecto Joule en el cobre.

102

MQUINAS ELCTRICAS

Todas las prdidas anteriores son transformadas en calor. Este calor se transmite en parte al ambiente, por radiacin y conveccin, y es absorbido en parte por la estructura de la mquina, elevando su temperatura, elevacin que est en relacin con su calor especfico. Cuando esta temperatura alcanza un determinado valor para el cual la cantidad de calor producida, correspondiente a las prdidas, se iguala con la cantidad de calor cedida al ambiente, la temperatura permanece constante. Se dice entonces que la mquina ha alcanzado la temperatura de rgimen. La temperatura de rgimen es una magnitud de gran importancia, ya que mediante la misma se puede determinar si los materiales aislantes con los que est construida la mquina van a trabajar correctamente. En efecto, los aislantes, constituidos en su gran parte por materiales orgnicos, se carbonizan a una determinada temperatura, perdiendo su cualidad de tales; tambin a temperaturas inferiores sufren un deterioro, siendo ste tanto ms rpido cuanto ms elevada sea la temperatura a la que estn sometidos. Es evidente que al degenerar la funcin aislante de estos materiales, la mquina no trabaje en buenas condiciones y acabe averindose al producirse cortocircuitos entre espiras o defectos a tierra. Existe una regla aproximada pero muy ilustrativa, llamada de Montsinger, debida a V. M. Montsinger, ingeniero de la General Electric, quien en 1930, tras un estudio exhaustivo de nueve aos sobre aislantes de transformadores, proporcion una frmula emprica conocida como la ley de los 10 "C, lo que significa que para cada incremento de JO oC de temperatura sobre el mximo recomendado, la vida del aislante se reduca a la mitad, e inversamente, una disminucin de JO "C doblara su vida. Debido a esto, se han establecido normas que especifican aquellas temperaturas mximas que no se deben sobrepasar en los devanados y en las diferentes partes de una mquina; ms concretamente, en Espaa las Normas UNE 21-305, UNE EN 60034-1:1995 y CEI-85 (1984) fijan una temperatura ambiente convencional y determinan la mxima temperatura que pueden soportar los aislantes, y en funcin de sta se clasifican en: l. AISLAMIENTO CLASE A. Son aislamientos constituidos por materiales fibrosos a base de celulosa ()seda, impregnados con lquidos aislantes; la temperatura mxima que en ningn caso debe sobrepasarse en este tipo de aislantes es de 105 oc. AISLAMIENTO CLASE E. Son aislamientos a base defibras orgnicas sintticas; la temperatura mxima que en ningn caso debe sobrepasarse en este tipo de aislantes es de 120 oc. AISLAMIENTO CLASE B. Son aislamientos formados por materiales a base de polister y poliimidos aglutinados con materiales orgnicos o impregnados con stos; la temperatura mxima que en ningn caso debe sobrepasarse en este tipo de aislantes es de 130 oc. AISLAMIENTO CLASE F. Son aislamientos formados por materiales a base de fibra de mica, amianto y fibra de vidrio aglutinados con materiales sintticos, como siliconas, polisteres o epxidos; la temperatura mxima que en ningn caso debe sobrepasarse en este tipo de aislantes es de 155 oc. AISLAMIENTO CLASE H. Son aislamientos formados por materiales a base de mica, amianto ofibra de vidrio aglutinados con siliconas de alta estabilidad trmica; la temperatura mxima que en ningn caso debe sobrepasarse en este tipo de aislantes es de 180 oc. AISLAMIENTO CLASE 200. Son aislamientosformados por materiales a base de mica, vidrio, cermica, etc.; la temperatura mxima que en ningn caso debe sobrepasarse en este tipo de aislantes es de 200 oc.

2.

3.

4.

5.

6.

CAPTULO

2.

PRINCIPIOS GENERALES DE LAS MQUINAS ELCTRICAS

103

7. AISLAMIENTO CLASE 220. Son aislamientos formados por materiales a base de mica, vidrio, cermica, etc., poliimidas tipo Kapton; la temperatura mxima que en ningn caso debe sobrepasarse en este tipo de aislantes es de 220 oc. 8. AISLAMIENTO CLASE 250. Son aislamientos formados por materiales a base de mica, vidrio, cermica, etc., poliimidas tipo Kapton; la temperatura mxima que en ningn caso debe sobrepasarse en este tipo de aislantes es de 250 oc. La temperatura ambiente del fluido refrigerante se toma, de acuerdo con la misma Norma, como40 "C (para menos de 1.000m de altitud) *. En consecuencia, los calentamientos (elevacin de temperatura respecto de la ambiente) admisibles para los aislantes anteriores sern: Clase A: 65 -c. Clase E: 80 -c. Clase B: 90 -c, Clase F: 115 -c, Clase H: 140 oc. Clase 200: 160 -c, Clase 220: 180 -c. Clase 250: 210 oc.

2.6. POTENCIA ASIGNADA O NOMINAL. TIPOS DE SERVICIO


De acuerdo con la Norma UNE EN 60034-1:1995 referente a mquinas elctricas rotativas. Parte 1: Caractersticas asignadas y caractersticas de funcionamiento, en su seccin 2 de definiciones se seala lo siguiente: Valor asignado: valor de una magnitud, asignado generalmente por un fabricante, para una condicin de funcionamiento especificada de una mquina. Potencia asignada: valor numrico de la potencia incluido en las caractersticas asignadas. Se incluyen tambin en esta seccin de la Norma UNE otras magnitudes asignadas para mquinas rotativas, indicndose ms adelante que estas magnitudes son atribuidas por el fabricante,el cual debe elegir una de las clases asignadas segn el tipo de servicio, y que en muchoscasos suele ser el correspondiente al tipo continuo mximo basado en el servicio SI (serviciocontinuo). De este modo la potencia asignada es un valor que seala libremente el fabricante enfuncin de la clase de servicio a que se destine la mquina sin que se produzcan calentamientos inadmisibles para la vida de los aislantes; por ejemplo, para un motor que vayaa funcionar en servicio continuo el fabricante le puede asignar una potencia de 11 kW, pero si va a funcionar en rgimen temporal con grandes perodos de reposo puede asignarle una potencia superior como por ejemplo 15 kW, teniendo en cuenta que los calentamientos previstosvan a ser menores. La Norma UNE indicada seala tambin que las caractersticas asignadasde una mquina elctrica se deben marcar en la placa de caractersticas. En definitiva, la potencia asignada es un concepto ms verstil que el antiguo de potencia nominal (aunque en alguna parte de este libro se tomen como sinnimos debido a que la potencia asignadase marca o nomina en la placa de caractersticas) y representa la potencia que puede desarrollar una mquina, cuando las restantes condiciones son las asignadas, sin que aparezcan calentamientos en sus diversos rganos que alcancen o sobrepasen las correspondientes temperaturaslmites expuestas en el apartado anterior y que dependen de la clase de servicio.

* A grandes alturas la evacuacin del calor de la mquina disminuye por ser menor la densidad del aire. Los datos expuestos se refieren a altitudes inferiores a 1.000 m. Para altitudes superiores se deben consultar las Normas (se establecen reducciones en los calentamientos que oscilan entre un 2 y un 5 por 100 por cada 500 m que sobrepasen los 1.000 m).

104

MQUINAS ELCTRICAS

Se observa de esta definicin que la potencia asignada est estrechamente vinculada al problema del calentamiento de sus diversas partes constitutivas y en consecuencia a los aislantes que intervienen en su elaboracin. En el epgrafe 8 de la Norma UNE EN 60034-1 se definen adems las potencias asignadas de la forma siguiente:
a) b)

c) d)

En generadores de C.C.: la potencia asignada es la potencia elctrica en los bornes, y debe expresarse en vatios (W). En generadores de c.a.: la potencia asignada es la potencia elctrica aparente en los bornes, y debe expresarse en voltamperios (VA) junto con la indicacin del factor de potencia. El f.d.p. asignado para los generadores sncronos debe ser 0,8 sobreexcitado, salvo especificacin contraria. En motores: la potencia asignada es la potencia mecnica disponible en el eje, y debe expresarse en vatios (W). En transformadores: la potencia asignada es la potencia elctrica aparente en bornes del primario o del secundario y debe expresarse en voltamperios (VA). (Norma UNE 20-10 1.)

De acuerdo con la Seccin 10 de la Norma UNE EN 60034-1:1995, todas las mquinas elctricas deben estar provistas de una o varias placas de caractersticas en las que deben marcarse de forma indeleble los puntos aplicables de la lista siguiente: 1)Nombre o marca del constructor. 2) Nmero de serie del fabricante o marca de fabricacin. 3) Informacin para identificar el ao de fabricacin. 4) Cdigo del fabricante para la mquina. 5) Para las mquinas de c.a., el nmero de fases. 6) Nmero(s) de las normas aplicables para las caractersticas asignadas y las caractersticas de funcionamiento. Si se indica 34, quiere decir que se cumplen todas las normas de la serie 34 de la CEI que sean pertinentes. 7) Grado de proteccin proporcionada por las envolventes (Cdigo IP) de acuerdo con CEI 34-5. 8) Clasificacin trmica o el calentamiento admisible. 9) Clase(s) de caractersticas asignadas y el(los) servicio(s) tipo(s) si la mquina est diseada para caractersticas asignadas diferentes de las del tipo continuo mximo, servicio tipo S1. 10) Potencia(s) asignada(s). 11) Tensin(es) asignadas(s) o gama de tensiones asignadas. 12) Para las mquinas de c.a., la frecuencia asignada o la gama de frecuencias asignadas. 13) Intensidad(es) asignada(s). 14) Velocidad(es) asignada(s). 15) Para las mquinas trifsicas de c.a. con ms de tres puntos de conexin, instrucciones de conexin mediante un diagrama en forma de texto... 26) Para mquinas previstas para funcionar en un solo sentido de giro, el sentido de giro indicado por una flecha. En la Figura 2.17 se muestra a modo de ejemplo una placa de caractersticas tpica de un motor asncrono en el que se sealan las magnitudes asignadas ms importantes. El grado de proteccin de envolventes sealado en el punto 7 de la relacin anterior viene definido en las Normas UNE 20.324-93 y CEI 34-5:1991 y se expresa con las letras IP (ndice de Proteccin) seguidas de tres nmeros: el primero indica la proteccin contra objetos slidos, el segundo la proteccin contra el agua y el tercero la proteccin contra choques mecnicos, de acuerdo con lo indicado en la Tabla 2.1. La clase de proteccin representa en definitiva las caractersticas de la envolvente de la mquina con vistas a: 1. Proteger a las personas contra el peligro de tocar directamente partes mviles o que estn en contacto con partes bajo tensin. Es lo que se denomina proteccin contra contactos directos. 2. Proteger la mquina contra la entrada de cuerpos slidos y lquidos desde el medio ambiente exterior. 3. Proteger la mquina contra choques mecnicos.

CAPTULO 2.

PRINCIPIOS GENERALES DE LAS MQUINAS ELCTRICAS

105

Figura 2.17. Placade caractersticas de un motorelctrico. Se pueden obtener tantas clases de proteccin como combinaciones posibles nos da la tabla, aunque no todas esas posibilidades se realizan en la prctica (por ejemplo, la proteccin de las mquinas elctricas contra choque mecnicos tiene en general poca jrnportancia). El tipo de proteccin que se utiliza can mayor frecuencia en ingeniera civil es el IP447, pero dependiendo de las condiciones en las que se prevea el trabajo de la mquina, el usuario deber definir el grado IP que ms se adapte a sus necesidades. Por ejemplo, los grupos motobomba sumergidos en pozos para suministro de agua a poblaciones tienen una proteccin IP687. En el caso de cuadros generales y acometidas elctricas de obras que son de tipo intemperie y de montaje provisional es aconsejable que sean de al menos IP557. Otro aspecto a considerar en las mquinas elctricas es el concepto de servicio, y que se define como los perodos de funcionamiento en vaco y reposo a los que est sometida una mquinateniendo en cuenta su duracin y secuencia en el tiempo. Las Normas UNE 20-113-7 Y UNE-EN 60034-1:1995 definen diez clases de servicios (sealados con la letra S seguida de un dgito, del 1 al 10) y que significan:
1.

2.

3.

4.

5.

Servicio continuo - Servicio tipo SI. Funcionamiento con carga constante y de Una duracin suficiente para que se establezca el equilibrio trmico. Servicio temporal - Servicio tipo S2. Funcionamiento can carga constante durante un perodo de tiempo determinado, menor que el requerido para alcanzar el equilibrio trmico, seguido de Unperodo de reposo suficiente para que la temperatura descienda hasta igualarse a la del fluido de refrigeracin dentro de un margen de 2 K. Servicio intermitente peridico - Servicio tipo S3. Sucesin de ciclos de servicios idnticos, comprendiendo cada uno un perodo de funcionamiento con carga constante y un perodo de reposo. En este servicio el ciclo es tal que la intensidad de arranque no influye de forma apreciable en el calentamiento. Servicio intermitente peridico con arranque - Servicio tipo S4. Sucesin de ciclos de servicios idnticos, comprendiendo cada uno un perodo de tiempo de arranque, un perodo de funcionamiento con carga constante y un perodo de reposo. Servicio intermitente peridico con frenado elctrico - Servicio tipo S5. Sucesin de ciclos de servicios idnticos, comprendiendo cada uno un perodo de tiempo de arranque, un perodo de funcionamiento con carga constante, un perodo de frenado elctrico rpido y un perodo de reposo.

106

MQUINAS ELCTRICAS

Tabla 2.1. Dgitos para indicar la clase de proteccin

Sin proteccin Proteccin contra contactos directos casuales de grandes superficies, por ejemplo con la mano. Proteccin contra la penetracin de cuerpos slidos extraos de dimetro superior a 50 mm.

Sin proteccin Protccin contra la cada vertical de gotas de agua.

Sin proteccin Energa de choque: 0,225 julios.

Proteccin contra contactos directos con los dedos. Proteccin contra la penetracin de cuerpos slidos extraos de dimetro superior a 12 mm. Proteccin contra contactos directos de herramientas, hilos, etc., mayores de 2,5 mm de dimetro. Proteccin contra la penetracin de cuerpos slidos extraos de dimetro superior a 2,5 mm. Proteccin contra contactos directos con herramientas, hilos, etc., mayores de l mm de dimetro. Proteccin contra la penetracin de cuerpos slidos extraos de dimetro superior a 1 mm. Proteccin total contra contactos directos. Proteccin contra depsitos de polvo perjudiciales. Proteccin total contra contactos directos. Proteccin total contra la penetracin de polvo.

Proteccin contra la cada de gotas de agua inclinadas en cualquier ngulo hasta l SO con la vertical. Proteccin contra el rociado de agua en un ngulo de hasta 60 con la vertical. Energa de choque: 0,5 julios.

Proteccin contra la proyeccin de agua en todas las direcciones.

Proteccin contra chorros de agua en todas las direcciones. Proteccin contra inundaciones pasajeras. Proteccin contra los efectos de inmersin. Proteccin contra inmersin prolongada.

Energa de choque: 2 julios.

7 8 9

Energa de choque: 6 julios.

Energa de choque: 20 julios

CAPTULO 2.

PRINCIPIOS GENERALES DE LAS MQUINAS ELCTRICAS

107

6. Servicio ininterrumpido peridico con carga intermitente - Servicio tipo S6. Sucesin de ciclos de serviciosidnticos, comprendiendo cada uno un perodo de funcionamiento con carga constante y un perodo de vaco. No existe perodo de reposo. 7. Servicio ininterrumpido peridico con frenado elctrico - Servicio tipo S7. Sucesin de ciclos de servicios idnticos, comprendiendo cada uno un perodo de arranque, un perodo de funcionamiento con carga constante y un perodo de frenado elctrico. No existe perodo de reposo. 8. Servicio ininterrumpido peridico con cambios de carga y de velocidad relacionados - Servicio tipo S8. Sucesin de ciclos de servicios idnticos, comprendiendo cada uno un perodo de funcionamiento con carga constante correspondiente a una velocidad de giro determinada, seguido de uno o varios perodos de funcionamiento con otras cargas constantes correspondientes a velocidades de giro diferentes (realizados, por ejemplo, por cambio del nmero de polos en el caso de motores de induccin). No existe perodo de reposo. 9. Servicio con variaciones no peridicas de carga y de velocidad - Servicio tipo S9. Servicio en el cual la carga y la velocidad tienen generalmente una variacin no peridica en el margen de funcionamiento admisible. Este servicio incluye frecuentemente sobrecargas aplicadas que pueden ser ampliamente superiores'a la plena carga. 10. Servicio con cargas constantes diferentes - Servicio tipo S10. Servicio que consiste en un mximo de cuatro valores diferentes de carga, cada uno de los cuales se mantiene un tiempo suficientepara permitir que la mquina alcance el equilibrio trmico. La carga mxima en un ciclo de servicio puede tener un valor cero (vaco o reposo). Finalmente, otro detalle a destacar en las mquinas son las formas constructivas extemas. En Espaa vienen definidas por la Norma UNE-EN 60034-7. Existen dos cdigos, el ms simpleutiliza una letra mayscula B o V seguida de una o dos cifras, que sirven para definir la posicin del eje: horizontal o vertical, si existe brida de sujecin o plato soporte, cojinetes,etc. Por otra parte, para reducir costes y permitir el intercambio de motores procedentes de distintos fabricantes, se ha llevado a cabo una normalizacin de las principales dimensiones mecnicas relativas al montaje de las mquinas: dimetro y longitud del eje, distancia entre patas, dimetro de la brida de anclaje, alturas de ejes, etc.

2.7.

RENDIMIENTO

Comoya se ha indicado en el apartado 2.5, en las mquinas elctricas, como en cualquier otro dispositivo de transformacinde la energa, existen unas prdidas,de tal forma que solamente se entregaa la salidauna parte de la energa absorbidaen la entrada.El rendimientose define como el cocienteentre la potencia til y la potencia absorbida o total, de acuerdo con la expresin:
IJ =

Potencia til P, =Potencia total PT

(2.15)

llamando Pp a la potencia perdida se verifica: PT en consecuencia (2.15), toma la forma:


11=--

=r. + P
P.,

(2.16)

r. + p"

(2.17)

108

MQUINAS ELCTRICAS

En el caso de un generador. la potencia til es la potencia elctrica entregada a la carga y en el caso de un motor es la potencia mecnica en el rbol de la mquina. La potencia perdida. teniendo en cuenta el apartado 2.5. es igual a: (2.18) que teniendo en cuenta (2.14) se puede poner: Pp = p+

r;

(2.19)

donde P, indica las prdidas constantes de la mquiria y Py las prdidas variables con la carga. Las prdidas variables son debidas a las prdidas en el cobre. y en consecuencia son proporcionales a 12 Como quiera adems que la corriente es proporcional a la potencia aparente (S = VI). se podr escribir: (2.20) Teniendo en cuenta que P; = VI cos S cos ({J
I'f=
({J

= S cos

({J.

la expresin (2.17) se podr escribir: S (2.21)


({J

S cos ({J + lf + bS2

S + (lf + bS2)/cos

lo que indica que para una misma potencia aparente S. el rendimiento es tanto mejor cuanto mayor es el f.d.p. Si el f.d.p. es constante, (2.21) se podr escribir denominando a al cos ({J, y resulta:
aS I'f = -------:: aS + lf+ bS2

(2.22)

Se puede dibujar ahora la curva I'f = feS), expresada por la ecuacin anterior. Se observa que I'f = Opara S = O,Yadems la funcin tiene un mximo, definido por la condicin:
dl'f

- =O

dS

= lf=

bS2

(2.23)

es decir, el rendimiento es mximo para una potencia aparente S tal, que coinciden para ese rgimen las prdidas fijas con las variables, esto es, las prdidas en el cobre (prdidas variables) son iguales a la suma de las prdidas mecnicas ms las del hierro (prdidas fijas). Se denomina ndice de carga o factor de utilizacin: C, al cociente entre la potencia aparente til S y la asignada SN' de tal forma que:
C=SN

(2.24)

El ndice de carga es el ptimo cuando la potencia til es la de mximo rendimiento, es decir: (2.25) pero teniendo en cuenta (2.23):
S~mx =

Pt -JI;

(2.26)

CAPTULO 2.

PRINCIPIOS GENERALES DE LAS MQUINAS ELCTRICAS

109

i S~mx

Figura 2.18.

Curvas de rendimiento en funcin de la potencia para diversos f.d.p.

que al sustituir en (2.25) resulta: Prdidas fijas Prdidas variables a la potencia nominal (2.27)

En la Figura 2.18 se indica la funcin r = feS), para un f.d.p. constante; se seala tambin a trazos la misma curva para un f.d.p. menor. Observandola variacin del rendimiento con la potencia, se pueden deducir las siguientes consecuenciasprcticas:
a) Se debe evitar el funcionamiento con cargas reducidas, pequeo. b) Se debe procurar que la mquina funcione con un ndice para obtener un mejor rendimiento. c) Se debe rechazar toda mquina cuya potencia asignada pecto al servicio a que se destina, pues trabajara con miento sera pequeo. En consecuencia, para un mismo da y pagada sera mayor. ya que el rendimiento sera de carga prximo al ptimo sea demasiado elevada rescarga reducida y su renditrabajo, la energa absorbi-

EJEMPLO DE APLICACIN

2.1

Una mquina elctrica de 40 kVA tiene unas prdidas en el hierro de 750 W, las prdidas en el cobre a plena carga o asignada son de 2.000 Wy las prdidas mecnicas son de 1.000 W. Calcular: a) Potencia aparente de mximo rendimiento. b) Prdidas en el cobre en el caso anterior. c) Rendimiento mximo para un f.d.p. unidad. d) Rendimiento a plena carga con f.d.p. 0,8. e) Rendimiento a media carga con f.d.p. 0,6.

SOLUCIN
a)

El ndice de carga ptimo de acuerdo con (2.27) es: C= 750 + 1.000 = 0,9354 2.000

110

MQUINAS ELCTRICAS

lo que indica una potencia aparente de mximo rendimiento, segn (2.25): S~mx = 0,9354'
b)

40 = 37,41 kV A

Cuando trabaja la mquina con mximo rendimiento, de acuerdo con (2.23) las prdidas fijas y variables coinciden, es decir:

Pv = P, = PFe +
e) El rendimiento mximo ser: S"mx COS tp r = _-=:.:__ __ Srymx COS (/J + 2lj
d)

P'II = 750

+ 1.000 = 1.750 W

37,41 . 1 ------=91,44% 37,41 . 1 + 2 1,75

A plena carga el rendimiento tendr por expresin: r = --"----SNCOS (/J + lf+


Sr.cos f{J

P;

40.000' 0,8 --------= 89,51 % 40.000 . 0,8 + 1.750 + 2.000

e)

A media carga, es decir, mitad de potencia, las prdidas del cobre, de acuerdo con (2.20) se reducirn a la 4." parte, esto es, sern de 500 W y el rendimiento valdr:

n=

(SN/2) cos (SN/2) cos


(/J

(/J

+ lf+ P;

20.000' 0,6 20.000' 0,6 + 1.750 + 500

= 84,21 %

2.8. F.M.M. Y CAMPO MAGNTICO EN EL ENTREHIERRO DE UNA MQUINA ELCTRICA


El campo magntico en el entrehierro de una mquina elctrica es el resultado de las f.m.m.s. combinadas de los devanados inductor e inducido que actan en esa regin. En principio, es el devanado inductor el que produce el campo en el entrehierro, creando f.e.m.s. en el devanado del inducido, que dan lugar a corrientes cuando se cierra el circuito por un sistema exterior (generador). Al circular una intensidad por el devanado del inducido, se crea una f.m.m. de reaccin de inducido, que al combinarse con la f.m.m. del inductor origina, de acuerdo con la ley de Ampre, el campo magntico resultante en el entrehierro de la mquina. Teniendo en cuenta adems que, de acuerdo con la ley de Faraday, la f.e.m. inducida es funcin de la induccin, se podr comprender la importancia de la distribucin del campo magntico en la forma de onda de la f.e.m. Se van a analizar en este apartado las formas de las f.m.m.s, y campos producidos por diferentes tipos de devanados para poder estudiar posteriormente las f.e.m.s. que se obtienen en el inducido y los pares electromagnticos a que dan lugar. Con objeto de hacer ms sencillo el clculo, para destacar ms claramente los principios fsicos involucrados, se supondr una mquina rotativa cilndrica, es decir, sin polos salientes tanto en el esttor como en el rotor, lo que representa la existencia de un entrehierro de espesor uniforme. Se admitir asimismo que la permeabilidad del hierro es infinita, lo que da lugar a considerar que la reluctancia del hierro es despreciable, por lo que no se requiere ninguna f.m.m, para producir la induccin en esta parte del circuito magntico. Para simplificar las figuras se supondr tambin que la mquina es bipolar, coincidiendo en este caso, de acuerdo con (2.1), el nmero de grados geomtricos con los magnticos.

CAPTULO 2.

PRINCIPiOS GENERALES DE LAS MQUiNAS ELCTRICAS

111

2.8.1. Campo magntico y f.m.m. producida por un devanado concentrado de paso diametral
Considerarnosen primer lugar una bobina de N espiras representada por el esquema simplificadode la Figura 2.19a. (Por motivos didcticos, se han alargado las cabezas de bobina para facilitarla visin de la misma. En la prctica constructiva real, estas cabezas se doblan adaptando su recorrido a la periferia del esttor para no impedir as el movimiento del rotor). Se tratade determinar la forma de la distribucin tanto del campo magntico como de la f.m.m. a lo largo del entrehierro. La bobina est recorrida por una corriente de i amperios, que en principio supondremos que es de c.c. En la Figura 2.19a se han representado las lneas de campo magntico que produce la bobina;estas lneas atraviesan radialmente el entrehierro y se cierran por los ncleos ferromagnticosde esttor y rotor (campo solenoidal). El sentido de las lneas de induccin viene determinadopor la regla de Ampre de la mano derecha y van de la cara izquierda a la cara derecha de la bobina.Debe destacarse que desde el punto de vista del entrehierro de la mquina, la parte izquierdadel esttor se comporta como un polo norte, ya que las lneas de induccin magntica sedirigendel material ferromagntico del esttor al entrehierro pasando despus al rotor, mientrasque la parte derecha del esttor es un polo sur porque las lneas de induccin magntica pasandel entrehierro a la parte derecha del esttor. El retomo de las lneas de campo se realiza por el ncleo ferromagntico del esttor. En la Figura 2.l9b se muestra el modelo magntico equivalentede la Figura 2. 19a, en el que se representa un esttor con unos imanes imaginarios nortey sur que simulan el campo magntico equivalente de la bobina. En el esquema de la Figura2.19 se ha considerado que la bobina tiene una anchura de 180 magnticos, lo cual indica,para el caso de que la mquina tenga dos polos, que el paso de bobina es diametral *. En la Figura 2.20 se representa en la parte superior izquierda la seccin transversal de la mquinade la Figura 2.l9a, en la que se muestran las lneas de induccin B producidas por la
0

Lneas de induccin magntica

a) Campo magntico producido por una bobina de paso diametral

b) Polos equivalentes producidos por una bobina de paso diametral

Figura 2.19. Bobina de paso diametral y sentido del campo magntico que produce.

* La denominacin diametral se emplea tambin para definir bobinas cuya anchura sea de un pa~o polar (1800 magnticos) aunque la mquina tenga cualquier nmero de polos. Tambin se utiliza la expresin de paso completo o polar.

112

MQUINAS ELCTRICAS

Eje de la bobina

Eje de la bobina

a)

+180
e)

f.m.m.

:
:+180
: f) d)

e)

-1

Distribucin de la f.m.m. senoidal

Tercer armnico Quinto armnico Desarrollo en serie de Fourier de la f.m.m.

Figura 2.20. Induccin y f.m.m. de una bobina y su desarrollo en serie de Fourier.

bobina. A su derecha se ha dibujado un esquema desarrollado de la mquina al cortar el conjunto por la seccin MM'. El eje de la bobina se toma como referencia de posiciones angulares (e = O).En la Figura 2.20b se han sealado el sentido de las lneas de induccin en el entrehierro (para dar claridad a la figura se ha evitado dibujar el cierre de las lneas de B en el esttor y el rotor). Se han asignado los sentidos de las lneas de induccin en el entrehierro de la Figura 2.20b teniendo en cuenta la regla de la mano derecha; entre A y A' aparece un polo sur, mientras que entre A' y A se obtiene un polo norte, lo que est de acuerdo con el sentido dibujado en la Figura 2.20a. Para poder determinar la magnitud de la induccin en cada punto del entrehierro ser necesario aplicar al circuito magntico de la Figura 2.20b la ley de Ampre en forma integral:

H di = Ni

(2.28)

CAPTULO 2.

PRINCIPIOS GENERALES DE LAS MQUINAS ELCTRICAS

113

Para utilizar la ley anterior es preciso recordar que si se considera infinita la permeabilidad del hierro tanto en el esttor como en el rotor, la diferencia de potencial magntico en estas zonas, definida por la ecuacin (1.21) del captulo anterior, ser igual a cero, o de otro modo: la f.m.m. necesaria en la bobina para producir una induccin en el hierro es despreciable. Por consiguiente, toda la f.m.m. aplicada a la bobina se requiere nicamente para crear el campo magntico en el entrehierro. El recinto'}' indicado en la ley de Ampre (2.28) puede ser cualquiera siempre que sea cerrado. Considrese, por ejemplo, que se ha elegido el circuito 1 (que coincide con una lnea de induccin) de la Figura 2.20b; al aplicar (2.28) resultar:

J,

i H . di ~ fb
a

H . di +

r
b(rotor)

H . di +

fd
e

H . di +

fa
d(esttor)

H . di = Ni

(2.29)

y teniendo en cuenta que la f.m.m. necesaria, tanto en el rotor como en el esttor, es igual a cero, se obtiene:

H . dl +

H . dl = Ni

(2.30)

Para deducir las ecuaciones (2.29) y (2.30) debe comprobar el lector que el circuito 1 de la Figura 2.20b se ha recorrido a izquierdas, por lo que son positivas las corrientes que atraviesan este circuito en sentido saliente al plano del papel. En nuestro caso, como existen N conductores llevando una corriente i cada uno en la ranura A, se obtiene un valor +Ni (ya que las corrientes i son salientes). Otro aspecto a considerar en la ecuacin (2.30) es que en general, si no se elige el recinto de integracin'}' aprovechando algn tipo de simetra del circuito magntico, no resulta inmediato el clculo de los campos H a lo largo del entrehierro. Por ejemplo, en el circuito 1 elegido, los campos H de a a b y de c a d pueden no ser iguales, por lo que en la ecuacin (2.30) se tienen dos campos incgnitas, lo que hara necesario aplicar la ley de Ampre a un nuevo circuito para poder resolver el problema. Se hace necesario, por consiguiente, saber elegir el recinto de integracin'}' de un modo ms inteligente. Debe destacarse entonces el hecho de que cualquier mquina elctrica rotativa tiene simetra circular con un nmero par de polos, y es por ello que sea cual sea la distribucin del devanado, el campo magntico en el entre hierro para un ngulo fi tiene siempre la misma magnitud que el campo en fi + 1800 (magnticos), pero de sentido opuesto; es decir, se cumple: H(fi)=-H(fi+n) (2.31)

Es evidente, por lo tanto, que si se elige un recinto de integracin con una anchura de 1800, se simplificar enormemente el problema. En la Figura 2.20b el recinto 2 se ha elegido cumpliendo esta condicin. Si se aplica a este circuito el teorema de Ampre resultar:

H . di

H . dl = Ni

(2.32)

pero teniendo en cuenta (2.31) dar lugar a: H g + H g = Ni

Ni H=-

2g

(2.33)

donde g representa el espesor del entrehierro. Es indudable la ventaja obtenida, ya que nos ha permitido obtener de un modo inmediato el campo en las zonas ef y gh. Si se desea ahora

114

MQUINAS ELCTRICAS

determinar el campo en cualquier punto del entrehierro, lo ms conveniente ser tomar el circuito 2 e irlo trasladando hacia la izquierda o derecha para ir barriendo todos los puntos del entrehierro. En nuestro caso, en el que se dispone de una nica bobina, no se obtiene informacin adicional alguna, ya que para cualquier posicin del circuito 2 siempre se obtiene la misma ecuacin (2.32). El campo es en consecuencia uniforme y su valor es el expresado en (2.33). El valor de la induccin en el entrehierro se obtiene de una forma inmediata, resultando ser: (2.34) En la Figura 2.20c se ha representado la distribucin de la induccin B en el entrehierro en funcin de O. Es una onda rectangular cuya amplitud viene definida por (2.34) y que invierte su signo en los puntos donde se localizan los conductores. La onda es positiva en aquellas zonas en las que las lneas de campo se dirigen del rotor al esttor (polo sur), mientras que es negativa en la regin en que las lneas de campo van del esttor al rotor (polo norte). En el estudio de las mquinas elctricas resulta ms interesante representar la distribucin de la f.m.m. en el entrehierro que la onda de induccin, y esto es debido a que la onda de f.m.m, (realmente la onda de tensin magntica en el entrehierro) es independiente del espesor de entrehierro. Otra ventaja adicional es que la f.m.m. es una funcin lineal de la corriente, por lo que puede aplicarse el principio de superposicin a una combinacin de f.m.m.s.; de hecho es la f.m.m. total la que origina la induccin resultante en el entrehierro. Sin embargo, debido a que la curva de imanacin de un material magntico es no lineal, no puede aplicarse el principio de superposicin a las inducciones. Es por ello que a partir de ahora en los ejemplos que se tratarn posteriormente nicamente se estudiar la distribucin de la f.m.m, en el entrehierro. Para el caso que nos ocupa, se define como r.m.m. o tensin magntica en un punto del entrehierro de referencia angular O a:
.'7(0) =

H di

(2.35)

El clculo de la f.m.m. anterior se realiza de un modo similar al del campo magntico ya estudiado, ya que se cumple una relacin similar a (2.31), es decir: (2.36) y eligiendo el circuito 2 de la Figura 2.20b, que tiene una anchura de 180, y aplicando al mismo la ley de Ampre se obtiene la relacin (2.32), y teniendo en cuenta (2.35), resulta:
.'7(0)

= -,'7(0
=

+ n)

= Ni

(2.37)

pero de acuerdo con (2.36) se transforma en:


2.'7(0) = Ni Ni .'7(0) = -

(2.38)

El resultado (2.37) se obtiene teniendo en cuenta que al igual que las inducciones, se consideran como f.m.m.s. positivas las que van de rotor al esttor y negativas las que se dirigen de esttor al rotor. Moviendo el recinto de integracin 2 se obtendr el valor de ,'7(0) en cualquier punto del entrehierro. En la Figura 2.20d se ha dibujado la distribucin de f.m.m., que es una onda rectangular de valor mximo Ni/2 y que es positiva entre -90 y +90 Ynegativa entre 90 y-90. La onda de f.m.m. (y tambin la de induccin) es una funcin peridica que

CAPTULO 2.

PRINCIPIOS GENERALES DE LAS MQUINAS ELCTRICAS

115

se puede descomponer en serie de Fourier. El lector puede demostrar que el desarrollo es de la forma: :7(8) = ,:7;" cos

e+~
2

cos 38 + ... + ,7;, cos he + ...

(2.39)

donde los valores SZ;; vienen definidos por:

,c;;;, = -

frr/2
-nr;

;7(8) cos h8 d8

(2.40)

11:

y teniendo en cuenta el valor de g(8) expresado en (2.38), la integral anterior da lugar a: 4 1 Ni h11: :7: = - - - sen _ h 11: h 2 2 que al llevar a (2.39) nos da: 4Ni [ cos 8 - 1 cos 38 + -1 cos 58 + ... ] Y(8) = 11:

(2.41)

(2.42)

En definitiva, la onda rectangular de f.m.m. es la suma de una onda fundamental senoidal que responde a la expresin: 4 Ni ,7(8)1 = - - cos 8 11: 2 (2.43)

y de armnicos impares cuya amplitud es lIh veces el fundamental. En la Figura 2.20e se han representadolas componentes: fundamental, tercero y quinto armnico de la onda de f.m.m. El dibujo est hecho a escala para que se aprecien claramente las relaciones entre las amplitudes de las tres ondas. El lector puede comprobar adems que la distribucin espacial del armnicoh tiene h mximos en un ciclo completo. Si se consideran despreciables los armnicos, entonces la distribucin de f.m.m. se puede suponer que obedece a la expresin: ,7(8) = ](8)1

=- 11:

4 Ni 2

cos 8

(2.44)

que es una onda distribuida senoidalmente en el entrehierro de la mquina y cuyo valor de picoest alineado con el eje magntico de la bobina. Esta f.m.m. producir una induccin en el entrehierro de la misma forma, ya que de acuerdo con (2.35)resulta:
R(8) = _' g

7(8)

; B(8) = floR(8)

(2.45)

es decir:
B( 8) = flo --

,7(8)
g

= - flo 11:

Ni

2g

cos 8

(2.46)

quecorresponde a la componente fundamental de induccin de la onda de la Figura 2.20c. En laFigura 2.20f se ha representado esta distribucin senoidal de campo magntico en el entrehierropor unas componentes de lneas de fuerza que estn ms concentradas en el eje de la bobinay se van separando a medida que llegan a las posiciones de 90 respecto del eje. Es unaforma grfica de hacer patente que el campo es mayor (est ms concentrado) en la zona correspondienteal eje de la bobina. Para evitar la incomodidad de tener que dibujar la distribucinde induccin o en general de f.m.m. de la forma mostrada en la Figura 2.20fse utiliza

116

MQUINAS ELCTRICAS

en Ingeniera Elctrica el concepto de fasor espacial *. Es un concepto anlogo en cierto modo al de los faso res temporales que se emplea con acierto en el estudio de los circuitos de c.a. Para mostrarlo de un modo grfico, considrese la distribucin de f.m.m. senoidal (2.44), que se puede escribir: .71(8) = F cos 8 donde F = - m m

4 Ni 11: 2

(2.47)

En la Figura 2.21 se ha vuelto a dibujar en la parte izquierda el esquema de la Figura 2.20f que representaba la onda de distribucin espacial de f.m.m. definida en (2.47). Se observa que esta distribucin queda definida completamente si se conoce su amplitud Fm Yla posicin espacial del mximo positivo de la onda (eje de la onda). Para esa informacin basta dibujar un segmento orientado apuntando hacia la regin del espacio donde la onda presenta su mximo positivo y cuyo mdulo sea igual a la amplitud de la onda. Este segmento orientado se ha dibujado en la parte derecha de la Figura 2.21 y representa elfasor espacial de la f.m.m. sealada a la izquierda. En definitiva el fasor espacial corresponde en este caso a una onda estacionaria de f.m.m. cuya distribucin espacial por la periferia del entrehierro la describe la funcin cos 8; su mximo coincide con el eje magntico de la bobina. Este modo de representar distribuciones senoidales de f.m.m.s. en el entrehierro es muy til en la descripcin de los fenmenos magnticos en las mquinas elctricas. Si, partiendo de la ecuacin (2.47), se supone que se alimenta la bobina con una corriente senoidal: i = 1m cos on, entonces la f.m.m. producida ser:
.'3'(8, t)

=- 11:

4 NIm

cos wt cos 8 = [Fm cos wt] cos 8

(2.48)

donde se ha denominado Fm a:
4 NIm F =-m tt. 2

(2.49)

Para ver el significado de la expresin (2.48), en la Figura 2.22 se ha dibujado el fasor espacial .'3'(8, t) en diferentes instantes de tiempo. En la parte superior de la figura se ha vuelto a representar la bobina en seccin transversal. En la parte central se muestra la forma de la corriente alterna i = 1m cos wt y en la parte inferior se observa el fasor espacial de f.m.m.

Distribucin de la f.m.m. senoidal

Fasor espacial equivalente a la distribucin senoidal

Figura 2.21. Distribucin senoidal de f.m.m. y fasor de f.m.m equivalente.

* El lector interesado en el tema de fasores espaciales puede consultar el libro Fundamentos de mquinas elctricas rotativas, de Luis Serrano Iribarnegaray (Ed. Marcombo, Barcelona, 1989).

CAPTULO 2.

PRINCIPIOS GENERALES DE LAS MQUINAS ELCTRICAS

117

i(t)

Eje bobina

A'

t;
1500 1800 00 30
-1m

t.;
wt

3600

Figura 2.22.

F.m.m. producida por un devanado concentrado alimentado con c.a.

en el entrehierro que est orientado con el eje de la bobina y cuya amplitud es proporcional al valor de la corriente en cada instante. Por ejemplo, si se considera el instante inicial (t = 0, wt = O),la corriente es 1m Yel valor de la f.m.m. de acuerdo con (2.48), ser igual al valor sealado en (2.49), que es una onda de f.m.m. de amplitud F m y que est distribuida senoidalmente por el entrehierro y cuyo fasor espacial correspondiente se muestra en el primer esquema de la parte inferior de la Figura 2.22. A medida que evoluciona el tiempo, la corriente que circula por la bobina sigue una distribucin senoidal, lo que hace modificar la amplitud de f.m.rn., por lo que el fasor correspondiente va cambiando su amplitud tal como se seala en la parte inferior de la Figura 2.22. En definitiva, la onda de f.m.m. y su fasor espacial permanecen fijos en el espacio pero su amplitud vara senoidalmente con el tiempo. Se dice entonces que la onda estacionaria de f.m.m. es alternativa o pulsante.

2.8.2. F .m.m. producida por un devanado distribuido


En la prctica constructiva habitual de las mquinas elctricas, con objeto de aprovechar toda la periferia tanto del esttor como del rotor, las bobinas se distribuyen en ranuras, lo que permite no solamente una utilizacin ms ptima de la mquina sino tambin una mejora en la calidad de la onda de f.m.m. e induccin, que se traducir en una f.e.m. inducida en las bobinasde carcter ms senoidal. Para comprobar de un modo ms fehaciente este hecho se va a considerar el esquema de la Figura 2.23, que representa un devanado constituido por tres bobinasde N espiras cada una llevando una corriente de i amperios (las bobinas estn conectadas en serie). La determinacin de la f.m.m. resultante se muestra en la Figura 2.24. En la Figura 2.24a se ha representado el esquema desarrollado de la mquina de la Figura 2.23. En la parte inferior derecha de la Figura 2.24 se han dibujado las ondas de f.m.m. de cada una de las bobinas,cuyos ejes no estn alineados y que responden a lo estudiado en el epgrafe anterior. En la Figura 2.24b se ha obtenido la f.m.m. resultante aplicando simplemente el principio de superposicin.Es una onda escalonada que obviamente se parece ms a una senoide. Cada

118

MQUINAS ELCTRICAS

Figura 2.23.

Devanado distribuido y formado por tres bobinas.

vez que se atraviesa una ranura se tiene un salto Ni/2 en la f.m.m. La obtencin de la onda de f.m.m. puede hacerse tambin aplicando la ley de Ampre. En la Figura 2.24a se ilustra el procedimiento. Se ha elegido un recinto de integracin abcd de 180 magnticos de anchura. Si denominamos F] a la tensin magntica existente en el entrehierro en la zona correspondiente a la lnea ad, el valor correspondiente en eb ser el mismo pero de sentido contrario
0

Eje del ~evanado

Figura 2.24.

Distribucin de f.m.m. producida por un devanado distribuido de tres bobinas.

CAPTULO 2.

PRINCIPIOS GENERALES DE LAS MQUINAS ELCTRICAS

119

debido a la simetra del sistema y que se expres mediante la ecuacin (2.36). Si se aplica la ley de Ampre a este recinto resultar:
2Ft = Ni + Ni + Ni = 3Ni

(2.50)

es decir:

F, =3-

Ni

(2.51)

Como ya se ha indicado en el epgrafe anterior, si se desea calcular las f.m.m.s. o tensiones magnticas en el entrehierro en otros puntos, se deber hacer un barrido con el recinto de integracin, y as resulta: Contorno a'b'c'd': Contorno a"b"c"d": Contorno a"'b"'c'''d''':
2F2

= 2Ni -

Ni

= Ni

Ni F2=+-

2F3 = Ni - 2Ni 2F4

= - Ni =
=

F3 ---F4

Ni

2.
Ni

(2.52)

= -3Ni

= -3-

En las expresiones anteriores se han considerado las tensiones magnticas positivas en aquellas zonas en las que las lneas de campo magntico se dirigen del rotor al esttor. Como era de esperar, los resultados obtenidos son acordes con los logrados aplicando el principio de superposicin. Un anlisis de Fourier de la onda escalonada de la Figura 2.24b da como resultado para el fundamental de f.m.m., y que puede considerarse como aproximacin de la onda real escalonada, siendo: 4 4 Ni .<:7(8) = - Kd 3F,1I cos 8 = - Kd 3 - cos 8 n n 2 (2.53)

El factor Kd tiene en cuenta el efecto de la distribucin del devanado (en el epgrafe 2.9.2 se demostrar su valor aplicado al clculo de f.e.rn.s. de devanados distribuidos). La onda de f.m.m. sigue siendo una onda pulsatoria o estacionaria de amplitud constante y que se distribuye senoidalmente por el entrehierro. Si se alimenta el devanado con una c.a. i = 1m cos on el valor de la f.m.m. ser de la forma: .9'(8, t) = Fm cos on cos 8
4 3Nlm F =-K-m n d 2

(2.54)

que es anloga a la expresada en (2.47). En general los devanados de las mquinas elctricas estn distribuidos en diferentes fases y polos. Si se considera una mquina de 2p polos y NI espiras por fase distribuidas en varias bobinas, la f.rn.m. resultante equivalente a (2.53) ser: :7(8) = - Kd
n

Ni
_L COS

2p

(2.55)

120

MQUINAS ELCTRICAS

Tngase en cuenta, para obtener este resultado, que si en cada fase hay b bobinas en serie por cada par de polos y es N el nmero de espiras que integran cada bobina, se podr escribir: NI = b . N . p

= bN = _[
p

(2.56)

En (2.53) b era igual a 3, por lo que se puede pasar de (2.53) a (2.55) simplemente sustituyendo 3N por N/p.

2.8.3. F.m.m. producida por un devanado trifsico. Campos giratorios. Teorema de Ferraris
Vamos a estudiar ahora un caso que tiene una gran utilidad prctica en el funcionamiento de las mquinas elctricas. Consideremos un sistema formado por tres devanados, colocados bien sea en el esttor o en el rotor, de tal forma que estn desfasados entre s 120 elctricos en el espacio, como se indica de una forma esquemtica en la Figura 2.25. Se sealan tres grupos de bobinas cuyos principios son A, B, C (corrientes salientes del plano de la pgina) y sus finales son A', B', C' (corrientes entrantes). Interesa calcular la f.m.m. que existe en un punto del entrehierro, determinado por el ngulo respecto al eje del devanado AA' (fase a), debido a la contribucin de los tres arrollamientos, al circular por ellos un sistema de corrientes trifsicas equilibradas, a saber:

e,

(2.57)

Suponiendo, como ya se ha indicado en los casos anteriores, que la distribucin de la f.m.m. de cada devanado sea senoidal en el espacio [vanse expresiones (2.47) y (2.54)], cada devanado producir una f.m.m. pulsatoria o alternativa orientada en su eje respectivo. Como quiera que los ejes magnticos estn desfasados 120 elctricos en el espacio, las f.m.m.s. que producen cada devanado en el punto P del entrehierro sern:

.C;;: = Fm cos tt cos e ; ~c:;,= Fm cos

(wt - 120) (e

cos

(e -

120) (2.58)

g:; = Fm cos (wt

+ 120) cos

+ 120)

Estator trifsico

Eje bobina AA'

Eje bobina CC'

Figura 2.25. Tres devanados desfasados en el espacio 120 elctricos.

CAPTULO 2.

PRINCIPIOS GENERALES DE LAS MQUINAS ELCTRICAS

121

Debe recalcarse que los devanados llevan corrientes desfasadas 120 en el tiempo y que los bobinados estn desfasados 120 elctricos en el espacio. En consecuencia la onda de f.m.m. resultante en el punto P ser igual a la suma de las tres ondas pulsatorias anteriores: J(8, ,7(8,
t) = t) =

.7;; +

.q- + .~
120) cos (8 - 120) +

(2.59)

Fm[cos

wt

cos 8 + cos

(wt -

+ cos (wt + 120) cos (8 + 120)] Si se hace uso de fa igualdad trigonomtrica: 1 cos A cos B = -[cos (A - B) + cos (A + B)]
2

(2.60a)

(2.60b)

la ecuacin (2.59) se convierte en: 3 .7(8, t) = - Fm cos (on - 8) + -[cos

r.
2

(cot + 8) +

+ cos (wt + 8 - 120) + cos (wt + 8 + 120)]

(2.61)

Los tres sumandos contenidos entre corchetes representan tres fasores simtricos desfasados 120, por lo que su resultante es nula. De este modo el resultado final es: J(8,
t)

=F cos (wt 2 m

8) =

Fmcos (wt 2

pa)

(2.62)

que representa la f.m.m. resultante en el entrehierro. Obsrvese que en un mismo punto del espacio (a = constante) la f.m.m. vara en funcin del tiempo segn una sinusoide de amplitud (3/2)Fm y en el mismo instante de tiempo (t = constante) est distribuida sinusoidalmente en el entrehierro. En consecuencia (2.62) tiene el carcter de una onda que se mueve alrededor del entrehierro, es una f.m.m. giratoria. Para comprender el significado fsico de esta f.m.m., en la parte superior de la Figura 2.26 se ha representado la evolucin con el tiempo de las tres corrientes. En la parte inferior se ha efectuado la suma haciendo uso de los fasores espaciales, 10 que representa una gran ventaja didctica, ya que se hacen ms visibles las componentes de las f.m.m.s. individuales. Para comprender el mensaje de la Figura 2.26 vamos a considerar dos tiempos de estudio de los seisrepresentados. Para t = O, es decir, wt = O, los valores de las corrientes en las bobinas, de acuerdo con (2.57), son: (2.63) Estosvalores se pueden comprobar en las curvas de corrientes instantneas de la parte superior de la Figura 2.26. De acuerdo con estos valores, se observa que en este tiempo t = O, la f.m.m.~ vale Fm Yest orientada hacia el eje positivo de la fase a, mientras que las f.m.m.s. ~ y .'7; valen -Fn,l2 Ypor ser negativas estn orientadas hacia los ejes negativos de las fases b y c. Se comprueba que el mdulo de la suma de estos tres fasores es: 3 Fm F,1I + 2( Fm) 2 cos 60 = 2" (2.64)

122

MQUINAS ELCTRICAS

(t)

330~.j

.... A

~
..,

I%~
...
B

e
-F",
2 3 B A' A A'

A'

1 0=0 1
.., A

10= 1201 3 -E
2
m

lo = 3601
e
B

B'

A'

A'

10= 1801
Figura 2.26. F.m.m. de un devanado trifsico alimentado con corrientes trifsicas

El primer sumando de la ecuacin representa el valor de la amplitud de Fa' mientras que el segundo evidencia la ley del paralelogramo para sumar Fb y Fe' que tienen una amplitud Fm/2 y forman 60 con la direccin de Fa' Evidentemente, la suma se orienta hacia el ngulo e = O. Si se pasa al instante t = T/6, es decir, cot = 60, los valores de las corrientes son ahora:
.
la

= lb = '2 ;

1m

.
le

= - 1m

(2.65)

Es por ello que las ondas de f.m.m. de las fases a y b son positivas (se orientan en el sentido positivo del eje de sus bobinas) y tienen una amplitud Fn,/2, mientras que la f.m.m. de la fase e, de acuerdo con (2.65), ser negativa y de magnitud Fm' Tal situacin se ha plasmado en el segundo esquema de la parte inferior de la Figura 2.26. Se observa que la suma sigue siendo 3F,) 2 pero que el mximo de la onda se produce ahora en = 60, valor que coincide con el lapso de tiempo transcurrido de T/6 segundos (es decir, cot = 60). En la Figura 2.26 se ha representado un ciclo completo de las corrientes en eltiempo y se observa grficamente que corresponde a un ciclo completo de rotacin del fasor de f.m.m. resultante. Es interesante destacar en esta figura que el valor mximo de laf.m.m. resultante

CAPTULO 2.

PRINCIPIOS GENERALES DE LAS MQUINAS ELCTRICAS

123

coincide con el eje del devanado que est llevando en ese momento la corriente mxima o de pico (sea sta positiva o negativa). Se dice entonces que se ha producido un campo magnti-

co giratorio que presenta dos caractersticas fundamentales: 1) tiene una amplitud constante, y 2) gira a velocidad constante. Si la mquina es bipolar, que es el caso representado en la Figura 2.26, se observa que una variacin de 360 elctricos en el tiempo corresponde a un giro de 360 magnticos en el espacio.Como quiera que para una mquina bipolar coinciden los grados magnticos con los geomtricos o mecnicos, cada ciclo de variacin de la corriente provoca una revolucin completa de la f.m.m, Si se realiza el devanado para cuatro polos, entonces sern necesarios dosciclos de variacin de la corriente para obtener una revolucin en la f.m.m, En general, si la mquina tiene 2p polos la velocidad de giro del fasor espacial de f.m.m, ser:
0 0

ro wm =p

(2.66)

y como quiera que si se denomina n a la velocidad de giro de la f.m.m, en r.p.m. y f a la

frecuencia de las corrientes se cumple:


n ro",= 2n 60 ro = 2nf

(2.67)

al sustituir en (2.66) resultar:


n=-

60f p

(2.68)

que se denomina velocidad de sincronismo del campo giratorio y que es funcin directa de la frecuencia y funcin inversa del nmero de pares de polos de la mquina. Existe una forma alternativa (ms analtica) para demostrar que la onda de f.m.m. expresada en (2.62) es giratoria. Como quiera que la ecuacin de la f.rn.m. producida por un devanado trifsico es:
.7(e, t) =

2 F", cos (rol

- e)

(2.69)

al considerar un observador que viaje con la onda en un punto de fase constante eo' que para ser ms concretos puede ser incluso la cresta de la onda, en la que se cumple o = O, la posicin de tal observador viene definida por la condicin:

rol - e

= eo = O =

.7(e, t)

= (3/2)Fm

(cresta de la onda) ; e =

rol

(2.70)

lo que significa que el punto lgido de la onda en la que se encuentra el observador se del entrehierro, ya que su posicin e depende del tiempo*. La velocidad de la onda se puede obtener derivando la ltima expresin (2.70) respecto del tiempo, y as resulta:
desplaza alrededor de la circunferencia

de -=ro
dt

(2.71)

* Para mayor claridad y a modo de comparacin: es como cuando un windsurfista se monta con su tabla en la crestade una ola del mar: la ola (onda) se mueve hacia la playa, sin embargo el windsurfista (si es lo suficientemente hbil)vara su posicin absoluta respecto de la playa, pero conservando su situacin encima de la cresta de la ola, es decir, sobre el mismo punto de ella.

124

MQUINAS ELCTRICAS

Como quiera adems que segn (2.1) el ngulo magntico () = p, siendo a el ngulo geomtrico o mecnico, resultar: w-p-=O

da dt

(2.72)

pero como quiera que la derivada de la ecuacin anterior representa la velocidad angular mecnica de movimiento de la onda, se tendr: (2.73) y teniendo en cuenta (2.67) se obtiene una velocidad en r.p.m.: n=p

60f

(2.74)

que es el mismo valor que se calcul en (2.68). Si se considera el caso de Espaa, donde la frecuencia es de 50 Hz, las velocidades de sincronismo que se obtienen segn sea el nmero de polos (2, 4, 6, 8, etc.) son 3.000, 1.500, 1.000, 750, etc., respectivamente. El estudio anterior constituye la demostracin del teorema de Ferraris, e indica la posibilidad de producir un campo magntico giratorio, a partir de un sistema de tres devanados fijos desfasados 120 elctricos en el espacio, por los que se introducen corrientes desfasadas 120 en el tiempo. (Obsrvese la coincidencia de los grados de desfase tanto en el espacio como en el tiempo.) El teorema es vlido y se puede generalizar para un sistema de m arrollamientos desfasados en el espacio 2'1T/m radianes elctricos, por los que circulan corrientes desfasadas 2'1T/m radianes en el tiempo. El resultado que se obtiene, equivalente a (2.62), es ahora:

,7(tJ, t) = - Fm cos (wt - tJ) 2

(2.75)

Para el caso particular de sistemas bifsicos, se dispone de dos devanados a 90 elctricos en el espacio con corrientes desfasadas 90 en el tiempo. El resultado (2.75) es vlido haciendo m = 2. Es muy importante que el lector asimile con profundidad el teorema de Ferraris, ya que es la base del funcionamiento de las mquinas elctricas de c.a. Es indudable que el resultado es sorprendente: con tres devanados que cada uno de ellos produce un campo alternativo se ha

logrado, al combinarlos adecuadamente en el espacio y con corrientes apropiadas en el tiempo, un campo magntico de amplitud constante (circular) y que es giratorio, o de otro modo, elfenmeno es equivalente en cierta manera a un imn permanente que se moviera a la velocidad de sincronismo.
Otro aspecto a destacar en este teorema es que el sentido del campo giratorio puede invertirse, si se permutan entre s las corrientes de dos cualesquiera de las fases que constituyen el sistema trifsico. Por ejemplo, si en el caso de la Figura 2.24 se intercambian las corrientes de los devanados b y e expresadas en (2.57) se tendr ahora: (2.76)

CAPTuLO 2.

PRINCIPIOS GENERALES DE LAS MQUINAS ELCTRICAS

125

y operando de un modo similar al demostrado anteriormente se obtiene una f.m.m. total anloga a (2.62) y que obedece a la ecuacin:

3 3 iY(O, t) =2" F.n cos (cot + O) =2" Fm cos (wt + prx)

(2.77)

que representa un campo magntico giratorio de amplitud constante 3Fm/2 y que gira a una velocidad:
da w =-=-m dt co p 60f , n=-p

(2.78)

es decir, de sentido contrario al original y de la misma magnitud. Experiencia prctica. En un laboratorio de Electrotecnia se puede comprobar fcilmente la existencia del campo giratorio de una forma muy simple si se dispone de un motor asncrono trifsico. Para ello debe desmontarse el motor y separar el rotor del esttor. Se introduce a continuacin una bola de acero (por ejemplo, de un viejo cojinete) dentro del esttor y se aplica a ste una tensin trifsica alterna regulable mediante un autotransformador trifsico (latensin aplicada debe ser del orden de 1/10a 1/5de la tensin asignada). Se observar que la bola empieza a rodar dentro de la periferia del esttor siguiendo al campo magntico giratorio quese forma. Al invertir una fase de la red trifsica, se comprobar que la bola rueda en sentido inverso.Este fenmeno, que suele causar un fuerte impacto a los estudiantes, se emplea en los talleresde reparacin de devanados elctricos para comprobar que el rebobinado se ha hecho correctamente.La bola de cojinete se puede sustituir por un bote metlico al que previamente se le ha incorporado un eje, y se comprobar que el bote gira arrastrado por el campo magntico giratorio.ste es el principio de funcionamiento del motor asncrono trifsico, que se estudia condetalle en el Captulo 4. Nota adicional: Si se realiza la experiencia de la bola y a continuacinla del bote, se observa que ste se mueve en sentido inverso al de la bola; esto se debe a que la bola rueda sobre la periferia del esttor, mientras que el bote gira sobre su eje.

2.8.4. Relacin entre un campo alternativo y un campo giratorio. Teorema de Lebanc


Unavez conocido el funcionamiento de un campo giratorio podemos dar una nueva interpretacinal campo alternativo producido por una bobina. Si se parte de la expresin (2.47) de la f.m.m. producida por un devanado concentrado alimentado con corriente alterna: ET(O,
t)

= Fm cos

on cos 0= Fm cos tt cos p

(2.79)

teniendo en cuenta la igualdad trigonomtrica (2.60), la ecuacin anterior se transforma en: ,'Y(O,t) = - cos ton - prx) + - cos (en + prx) 2 2

r.

r:

(2.80)

El primer sumando anterior, que es similar a (2.62), corresponde a una onda de f.m.m. rotativa de amplitud F",/2 que gira en sentido directo (es decir, para el ngulo rx positivo de la Figura 2.25 y que corresponde al contrario a las agujas del reloj) a velocidad angular )m = + colp (es decir, n = + 60f/p). El segundo sumando de (2.80) es una onda de f.m.m. rotativa de amplitud Fm/2 que gira en sentido inverso a velocidad angular wm = - colp (es decir, n = - 60f/p).

126

MQUINAS ELCTRICAS

En definitiva, laf.m.m. pulsatoria (2.79) producida por una bobina recorridapor c.a.puede representarsepos dosf.m.m.s. rotativas que giran en sentidos contrarios a velocidades:
wm=p
W =>

n=p

60J

(2.81)

La definicin anterior constituye el teorema de Leblanc. El hecho de que la f.m.m. de un devanado monofsico excitado por c.a. se pueda descomponer en dos f.m.m.s. rotativas de sentidos contrarios es un paso conceptual importante para comprender el funcionamiento del motor de induccin monofsico, como el lector comprobar al estudiar el Captulo 4.

2.9. F.E.M. INDUCIDA EN UN DEVANADO DE UNA MQUINA ELCTRICA 2.9.1. Generalidades


En los devanados de las mquinas elctricas se inducen f.e.m.s. debidas a las variaciones del flujo enlazado por los arrollamientos. Estos cambios son el resultado de:
a)

La variacin con el tiempo de la magnitud del flujo, lo que da lugar a la llamada f.e.m. de pulsacin o de accin transformadora: ep' b) Del movimiento del circuito inducido, respecto del flujo, resultando una f.e.m. de rotacin, velocidad o movimiento: e.. e) De la combinacin de los dos casos anteriores, apareciendo las f.e.m.s, ep y ero

El clculo de la f.e.m. se realiza en cada caso, aplicando la ley de Faraday, y para analizar este problema de generacin se va a considerar el prototipo de mquina elctrica que se indica en la Figura 2.27, constituido por un devanado inductor l, Yun arrollamiento inducido 2, que consiste en un bobinado de N2 espiras concentradas de paso diametral. Ambos devanados estn situados en el esttor y en el rotor, respectivamente, girando ste a una velocidad wm rad/s. Se van a considerar adems las siguientes hiptesis: 1) El flujo inductor $ vara senoidalmente con el tiempo; para ello se introduce en el esttor una corriente alterna de frecuencia J y pulsacin w.

c.a.

i--+

Figura 2.27. Mquina elctrica elemental con dos devanados.

CAPTULO 2.

PRINCIPIOS GENERALES DE LAS MQUINAS ELCTRICAS

127

2) El flujo inductor se distribuye senoidalmente por la periferia del entrehierro. 3) El eje del devanado del rotor tiene una posicin respecto del eje de flujo del esttor, definido por la expresin a = wmt, es decir, en t = Ose tiene a = O(a es el nmero de grados geomtricos). 4) El bobinado del inducido est en circuito abierto, para considerar nicamente el efecto de generacin de f.e.m.; la frecuencia de la seal obtenida se denominar jj, que corresponde a una pulsacin w2 De acuerdo con las hiptesis anteriores, y teniendo en cuenta la expresin (2.47) de la ondade f.m.m. producida por un devanado concentrado, si para simplificar suponemos que la reluctancia del circuito magntico es constante, al aplicar la ley de Hopkinson se obtendr una expresin para el flujo distribuido en el entrehierro similar a la de la f.m.m., es decir:
c]:>1 = c]:>m

cos cot cos p

(2.82)

donde <l>", expresa el valor mximo de flujo. En consecuencia, la f.e.m. inducida ser: (2.83) El primer trmino del segundo miembro corresponde a la f.e.m. debida a la pulsacin de flujo,mientras que el segundo trmino corresponde a la f.e.m. debida a la rotacin del inducido. La expresin (2.83) se puede escribir de la siguiente manera: e2 =
N2wl<l>m

[sen (w1 + pwm)t + sen (w1

pwm)t] +

(2.84) expresinque responde a la forma general: e2 = -N2c]:>m

[(w1 + pwm) sen (w1 + pwm)t + (w1

pwm) sen (w1

pwm)t]

(2.85)

La ecuacin anterior indica que la f.e.m. inducida en el rotor, e2, contiene pulsaciones de valorw2 que responden a la expresin general: (2.86)
y teniendoen cuenta que:
W1

= 2nfl

W = 2nm 60

(2.87)

laexpresin(2.86) se transforma en: f2=flnp 60 (2.88)

ecuacin muy importante que relaciona las frecuencias de los circuitos inductor e inducido conla velocidad del rotor y el nmero de polos. Esta expresin permitir en el apartado 2.10 realizaruna clasificacin general de las mquinas elctricas.

128

MQUINAS ELCTRICAS

La ecuacin general de la f.e.m. definida por (2.83) permite analizar los dos casos particulares siguientes:

a) Inducido fijo. Flujo variable


En este caso O)m= 0, ya que el devanado del rotor es estacionario, y de acuerdo con (2.83) la f.e.m. resultante, debida a la pulsacin del flujo, ser: (2.89) Esta situacin corresponde a una mquina denominada regulador de induccin monofsico; existe un rotor cuya posicin se puede variar respecto al esttor, y se observa que cuando p = n/2la f.e.m. resultante es nula, lo cual puede comprobarse en la Figura 2.27, ya que el rotor no abraza ningn flujo. Tambin corresponde al caso del transformador para el cual p = O; es decir, todo el flujo producido por el inductor, que ahora se denomina primario, es abrazado por el inducido, que recibe el nombre secundario. En el caso real, no es necesario recurrir a la disposicin de la Figura 2.27, y ambos devanados estn arrollados sobre el mismo ncleo magntico sin necesidad de entrehierros. La f.e.m. inducida tendr una expresin instantnea definida por: (2.90) que corresponde a la (2.89) cuando p = O. El valor eficaz de la f.e.m. ser: N20)/Pm E2= que teniendo en cuenta (2.87) da lugar a:

j2

(2.91)

E2

= }2 Nd-/J>m = 4,44

2n

NJ$",

(2.92)

expresin importante y que se emplear en el estudio de los transformadores. De acuerdo con (2.88) se observa que las frecuencias de las corrientes del inducido h coinciden con las del inductor JI' es decir,J2 = ~.

b)

Inducido mvil. Flujo constante

En este caso WI = 0, lo que indica que el devanado inductor est alimentado por una c.c.; de acuerdo con la expresin general (2.83), la f.e.m. resultante, debida al movimiento del inducido, ser: e2 = N2PO)",$m sen p y teniendo en cuenta que
CI.

(2.93)

= wmt es: (2.94)

que expresa una f.e.m. de valor eficaz: (2.95) y cuya pulsacin vale: w2

= pO)", = J2 = 60

np

(2.96)

CAPTULO 2.

PRINCIPIOS GENE.RALES DE LAS MQUINAS ELCTRICAS

129

que est de acuerdo con la ecuacin general (2.88), que relaciona las frecuencias del inductor e inducido. Las mquinas que responden a estas consideraciones se denominan sncronas, y deben su nombre a que segn (2.96) la frecuencia del inducido es proporcional a la velocidad del rotor (como ya se ver ms tarde, las mquinas de c.c. pertenecen tambin a esta clasificacin, pero debido a la rectificacin mecnica del colector de delgas, la expresin de la f.e.m. difiere de la expresada aqu). La expresin (2.95) de la f.e.m. para las mquinas sncronas puede tomar otra forma si se tiene en cuenta que wm = 2nn/60, resultando:

E2

y260

2n pn pn :::, - N/1>m = 4,44 - N/Dm


60

(2.97)

y teniendo en cuenta la identidad (2.96), resulta:

E2 = 4,44 N2f/Dm

(2.98)

En realidad las expresiones anteriores vienen afectadas por unos coeficientes que tienen en cuenta la forma real del flujo y la distribucin del devanado, como se demostrar en el epgrafe 2.9.2.

EJEMPLO DE APLICACIN 2.2 Se tiene un transformador monofsico, constituido por dos devanados, primario y secundario, colocados en un ncleo magntico de seccin uniforme S = 10 cm', como indica la Figura 2.28. Los devanados primario y secundario tienen 400 y 639 espiras, respectivamente.Elprimario se conecta a una red de 127 V, 50 Hz. En el supuesto de despreciar la cada de tensindel primario, calcular: 1) Densidad deflujo mxima existente en el ncleo. 2) F.e.m. inducidaen el secundario.

rv
NI = 400 espiras N2 = 693 espiras

Figura 2.28.

SOLUCIN

1. Al despreciar la cada de tensin en el devanado primario se cumple VI = E)' y


de acuerdo con la expresin (2.92) aplicada al primario, se cumplir: = 4,44 Nzf/Dm' de donde se deduce:
<1>
m

VI = El =

= 4,44

127 . 50 . 400

1,43 . 10-3 Wb

130

MQUINAS ELCTRICAS

y como quiera que <1>m= Bm. S, se tendr: 1,43 10-3 B = 10. 10-4 = 1,43 Teslas 2. Al aplicar (2.92) al secundario se obtiene: E2 = 4,44 Nzil<1>m' resulta: que al sustituir valores,

E2 = 4,44 . 693 . 50 . 1,43 . 10-3 = 220 V

2.9.2. Factores que afectan a la f.e.m. inducida en un devanado


En el epgrafe anterior se ha demostrado la expresin general de la f.e.m. que aparece en un devanado; el anlisis ha partido de unas hiptesis que facilitaban el clculo, pero que en la realidad no son correctas. Las mquinas elctricas rotativas reales, a diferencia del prototipo de mquina indicado en la Figura 2.27, muestran las siguientes diferencias:
a)
b) c)

El flujo inductor no se reparte siempre de una forma senoidal por el entrehierro. El devanado no se encuentra concentrado, sino que est distribuido en ranuras a lo largo de la periferia de la mquina. Los arrollamientos no son siempre de paso diametral, sino que presentan acortamientos de paso, con objeto de mejorar la onda de f.e.m. inducida.

Cada uno de estos inconvenientes que aparecen en las mquinas reales introduce un factor, por el cual la f.e.m. inducida, en la prctica, es menor que la calculada anteriormente. De acuerdo con las diferencias apuntadas aparecen los factores de reduccin correspondientes, denominados: factor de forma, factor de distribucin y factor de paso o acortamiento. Veamos el significado y clculo de cada uno de ellos:

1. Factor de forma
Este factoraparecedebido a que el flujo no tiene una distribucinsenoidalen el entrehierro.Si se considerael prototipode mquina de la Figura 2.27, en el supuestode que el flujo inductor sea constantey de valor mximo <1>m' aparecer una f.e.m. cuyo valor medio vendr expresado por: E
= N2
-

d<1> <1>m- (-<1>m) = N2 = 4 N2fz<1> dt TI2

(2.99)

donde T indica el tiempo que tarda en recorrer un ciclo magntico completo (en una mquina con 2p = 2 coincide con el tiempo de una revolucin completa del rotor). El clculo expresado en (2.99) se ha realizado observando el flujo que barre el inducido en un semiperiodo. Se observa que la expresin anterior no tiene en cuenta la forma de este flujo para calcular el valor medio de la f.e.m.; sin embargo, para calcular el valor eficaz de la misma deber multiplicarse (2.99) por un coeficiente que s tiene en cuenta esta forma de onda, que se denomina factor de forma y que se define como:
K = --'-----f

valor eficaz valor medio

(2.100)

De este modo, teniendoen cuenta (2.99)y (2.100), la expresinde la f.e.m. eficaz inducida ser: (2.101)

CAPTULO 2.

PRINCIPIOS GENERALES DE LAS MQUINAS ELCTRICAS

131

y en el caso de que la onda de flujo se reparta senoidalmente por el entrehierro, se cumplir:

-=

2j2

1,11

(2.102)

donde se ha tenido en cuenta las expresiones del valor medio y eficaz de una onda alterna y que conoce el lector de un Curso de Teora de Circuitos. En la prctica se consigue que la distribucin sea senoidal aumentando la curvatura de los polos inductores frente a la superficie del inducido. En las mquinas con inductor cilndrico se emplean tcnicas de distribucin del devanado, como ya se ha indicado en el epgrafe 2.8.2.

2. Factor de distribucin
En el epgrafe anterior se ha calculado la f.e.m. producida por un devanado concentrado y de paso diametral. En la prctica el arrollamiento est distribuido en ranuras a lo largo de toda la periferia de tal forma que las f.e.m.s. del bobinado van desfasadas y su suma no es aritmtica sino vectorial. Si denominamos q al nmero de ranuras por polo y fase de la mquina, m al nmero de fases y 2p al nmero de polos, el nmero de ranuras de la mquina designado por K ser:

K=qm2p
El ngulo geomtrico entre dos ranuras consecutivas 3600 y=K ser:

(2.103)

(2.104)

que corresponde a un ngulo elctrico py. Supongamos que se trata de calcular la f.e.m. debida a las tres bobinas de la misma fase que se indican en la Figura 2.29, donde cada bobina tiene N espiras.

Figura 2.29. Devanado distribuidode tres bobinas por fase.

132

MQUINAS ELCTRICAS

Las f.e.m.s. de cada bobina sern iguales pero irn desfasadas en el tiempo, lo mismo que van en el espacio, resultando el diagrama fasorial de la Figura 2.30; el nmero de bobinas existentes es en general q y las f.e.m.s. parciales estn representadas por vectores iguales
Ebob= IABI

= IBC = leDI

Todos los extremos de los vectores que representan las f.e.m.s. se encuentran situados sobre una circunferencia de radio genrico R. La f.e.m. resultante Edebida a todo el devanado est representada por el vector AD y su magnitud es:
qpy E = IADI = 21QDI =.2R sen -

(2.105)

Si se llega a considerar que el devanado est concentrado, la f.e.m. terica hubiera sido:
E,

= qEbobina= qlABI = q21API = 2qR

sen 2

py

(2.106)

El coeficiente o factor de distribucin se define como cociente de la f.e.m. geomtrica E y la terica E" y se designa con el smbolo Kd, resultando:
E _lKd E,

sen-

qpy 2_ py

(2.107)

sen-

En consecuencia, y de acuerdo con la frmula anterior, la f.e.m. producida por un devanado distribuido se podr calcular como si estuviera concentrado, como ya se hizo en el epgrafe anterior, y el resultado habr que multiplicarlo por Kd para obtener la f.e.m. real, que tiene en cuenta la diferencia de fase entre las f.e.m.s. parciales de cada bobina. Si el nmero de ranuras es muy elevado puede considerarse que forman un arco continuo; si se denomina y, = qy el ngulo geomtrico total que abarca todo el bobinado, se tendr: PY, sen2
q' sen2q

sen-

py,

Kd = lm
q~
00

PY,

Py,

(2.108)

Figura 2.30. Composicin geomtrica o fasorial de f.e.m.s.

CAPTULO 2.

PRINCIPIOS GENERALES DE LAS MQUINAS ELCTRICAS

133

que puede obtenerse tambin como cociente de la cuerda al arco que subtiende (devanado uniformemente distribuido). Para una mquina trifsica se cumple: py, = n/3, y en consecuencia el coeficiente de distribucin valdr: sen 6 n
6
n

Kd = --

= 0,955

y para una mquina de c.c. se tiene: py, = n, y en consecuencia: py, senKd = -tt

= - = 0,637

3. Factor de paso o acortamiento


Losdevanadosreales tienen un paso acortado en vez de un paso diametral, ya que de esta forma se eliminan armnicos. Para una bobina de paso diametral le corresponde una anchura de 180 elctricos,lo cual quiere decir que si una rama est situada frente a un polo norte, (vase rama A dela Fig. 2.31a), la otra parte de la bobina est situada frente al polo sur (rama 1\'). En el caso de la Figura 2.31a, si una rama de la bobina est situada en A y la otra en 1\ se ha acortado el pasoen un ngulo pa elctrico. Si se denomina E, la f.e.m. de cada rama, la f.e.m. real de la bobinavendr expresada por la suma vectorial indicada en la Figura 2.31b, cuyo valor es:
Ebob = IPRI
0

= 21pSI

pCl. = 2Er cos 2

(2.109)

Si las f.e.m.s. llegan a sumarse aritmticamente, que es lo que sucede con las bobinas de paso diametral, se obtendr una f.e.m. terica E, dada por:
E, = 2Er (2.11 O)

El coeficiente de acortamiento Ka define el cociente:


K =-=cosa Ebob

E,

(2.111)

b)

Figura 2.31. Bobinacon paso acortado.

134

MQUINAS ELCTRICAS

En consecuencia, y teniendo en cuenta todos los coeficientes de devanado: distribucin y paso y el de forma, la f.e.m. de un devanado en su forma ms general posible, teniendo en cuenta (2.101), (2.107) y (2.111), ser: (2.112) En el caso particular de distribucin senoidal de flujo en el entrehierro, K = 1,11, resultando: (2.113) En consecuencia, las expresiones demostradas en el epgrafe 2.9.1 debern aplicarse multiplicadas por el factor de devanado K = Kd Ka' para tener en cuenta la distribucin y acortamiento del mismo. Las expresiones (2.107) Y (2.111) pueden utilizarse para calcular las f.e.m.s. debidas a los armnicos de f.m.m., de tal forma que si h es el orden de un armnico los coeficientes Kd y Ka vienen expresados por: qhp sen-2
hpy pha 2

Kd=----

K =cosa

(2.114)

q'senT

ya que para un armnico de orden h, el ngulo es h veces mayor. Esta afirmacin es fcil de comprender si se observa el grfico de la Figura 2.20e, en el que se mostraban las f.m.m.s. fundamental, de tercero y quinto armnico que aparecan en el entrehierro para el caso de un devanado concentrado. Se advierte que mientras la onda fundamental cubre un ciclo en 360 elctricos, en el mismo espacio el tercer armnico da tres ciclos exactos. O de otro modo, un ngulo elctrico de 360 para la onda fundamental es observado como un ngulo de 3.360 = 1.080 para el tercer armnico y de 5.360 = 1.800 para el quinto armnico. De este modo un ngulo de 8 grados para elfundamental es interpretado como un ngulo h 8 para el armnico de orden h. EJEMPLO DE APLICACIN 2.3
Una mquina elctrica tiene un devanado trifsico distribuido en 36 ranuras. Cada bobina est acortada un ngulo de 30 elctricos y est formada por 40 espiras, devanadas en una sola capa. La mquina tiene 4 polos y gira a una velocidad de 1.500 r.p.m. El flujo por polo es de 0,2 Wb y est distribuido senoidalmente por el entrehierro. Calcular lafe.m. inducida por fase.

SOLUCIN La f.e.m. vendr expresada por: E = 4,44 Kd Kah N2 cuyo valor es:
Cf> m;N2

es el nmero de espiras por fase,

3640 N2 = -= 240 espiras/fase 23 La frecuencia f2 vale:


np 1.5002 f2 = - = = 50 Hz

60

60

CAPTULO 2.

PRINCIPIOS GENERALES DE LAS MQUINAS ELCTRICAS

135

El coeficiente de distribucin es: sen-

qpy py 2

2 Kd=---q. senEl ngulo geomtrico entre dos ranuras es:

= --36 = 10

360

y el nmero de ranuras por polo y fase es:


K 36 q=-=-=3 2pm 43 en consecuencia: 3210 sen--2 210 3 sen-2

Kd

= ----

= 0,960

el coeficiente de acortamiento ser:

Ka = cos - = cos 15 = 0,966 2 y la f.e.m. por fase tendr una magnitud:


E

pa

= 4,44

. 0,960 . 0,966 . 50 . 240 . 0,2

= 9.882

voltios

2.9.3. Armnicos de r.e.m.: origen y eliminacin


Al estudiar en los epgrafes 2.8.1. y 2.8.2. la distribucin de f.m.m. producida por un devanado, bien sea concentrado o distribuido, se demostr (vase Figs. 2.20 y 2.23) que la f.m.m. no era senoidal y que por lo tanto contena armnicos. Se demostr asimismo (vaser ecuacin 2.41) que debido a la simetra del circuito magntico solamente se producan armnicos impares. Las f.m.m.s. combinadas de inductor e inducido dan lugar a la induccin en el entrehierro y que a su vez es el origen de las f.e.m.s inducidas en la mquina. En definitiva, lo que sucede es que los diferentes armnicos presentes en la onda de f.m.m. estarn presentes (en principio) en la f.e.m. inducida de la mquina. Si como es evidente se desea conseguir un tipo de onda de forma senoidal, ser necesario proceder a la eliminacin de los armnicos. Desgraciadamente, una anulacin total es imposible, pero dado que de acuerdo con (2.40) y (2.41) las amplitudes de los armnicos son inversamente proporcionales a su orden, es decir, el armnico tercero tiene una amplitud 1/3 del fundamental, el armnico quinto tienen una amplitud 1/5 del fundamental, etc., con llegar a anular hasta el quinto o sptimo armnico se habr conseguido una onda prcticamente senoidal. Para comprender el proceso de eliminacin de armnicos se va a considerar que se dispone de una mquina con inducido trifsico (por ejemplo, una mquina sncrona o alternador),

136

MQUINAS ELCTRICAS

vamos a prestar atencin primero a las fe. m.s. de tercer armnico. Si las f.e.m.s. de primer armnico o fundamental de cada una de las fases son de la forma:
eal = Elm
COS

wt ; ehl = Elm

COS

(wt - 120) ; ecl = Elm

COS

(wt + 120) (2.115)

y las f.e.m.s. de tercer armnico son: ea3 = E3m


COS

3wt ; eb3 = E)m cos (3wt - 3 . 120) =E3m


E3m COS

COS

3wt (2.116)

ec3 =

(3wt + 3 . 120) =

E3m COS

3wt

para comprender las expresiones anteriores hay que'tener en cuenta que un ngulo elctrico 8 para el primer armnico corresponde a un ngulo elctrico he para el armnico de orden h. Se observa en (2.116) que las fe. m.s. de tercer armnico inducidas en las tres fases son iguales en mdulo y fase. Si los devanados se conectan en estrella (vase Fig. 2.32a), y se denomina E3 al fasor de f.e.m. de tercer armnico, que teniendo en cuenta (2.116) es la misma para las tres fases, las tensiones compuestas de tercer armnico sern:
Eab3

= Ea3

Eb3

= E3 - E3 = O ;

Ebc3

= Eb3

Ec3

= E3 - E3 = O (2.117)

que nos indica que aunque las tensiones de tercer armnico estn presentes en cada una de las fases, no aparecern en los terminales de la mquina. En la Figura 2.32, Z) representa la impedancia de cada fase a las corrientes de tercer armnico. Si se considera ahora que los devanados se conectan en tringulo (Fig. 2.32b), aun funcionando la mquina en vaco, es decir, sin conectar una carga entre los terminales, se obtendr una corriente de circulacin de tercer armnico en los bobinados de la mquina, de valor: 13=-=3Z3 ZJ
3E3 EJ (2.118)

y las tensiones de tercer armnico que aparecern entre los terminales externos sern:
Eab3

= E3 - Z313 = O ;

Ebc3

= E3 - Z31J = O ;

Eca3

= E3 - ZJIJ = O

(2.119)

En consecuencia, independientemente de que los devanados se conecten en estrella o tringulo, no aparecern tensiones de tercer armnico en los terminales de lnea. El mismo resultado se obtiene para todos los armnicos mltiplos de 3 tales como el 9, 15, 21, etc. (no se han incluido en esta relacin los armnicos pares mltiplos de 3, que como ya sabemos, por razones de simetra no existen). Generalmente los alternadores de las centrales elctricas se
a

e a)

b b)

Figura 2.32. Tercer armnico en conexiones estrella y tringulo.

CAPTULO 2.

PRINCIPIOS GENERALES DE LAS MQUINAS ELCTRICAS

137

conectan en estrella para evitar la circulacin de corriente de tercer armnico que estaran presentes si los devanados se conectaran en tringulo. En resumen, los armnicos presentes en la f.e.m. de una mquina trifsica son:
h = 5, 7, 11, 13, 17, 19,23,25,29,

etc.

(2.120)

Si se desea eliminar ms armnicos, ser preciso recurrir, como se ha explicado en el epgrafe 2.9.2, al acortamiento y distribucin de las bobinas. En las ecuaciones (2.114) se dan las expresiones de .los coeficientes de acortamiento y distribucin en funcin del orden del armnico. Si se desea eliminar el quinto armnico actuando sobre el acortamiento de las bobinas ser necesario proceder a acortar las bobinas 1/5 del paso polar; es decir, segn la Figura 2.31, se tomar: po:

=-

1 180 = 36 5

(2.121)

En esta situacin, de acuerdo con (2.114), resultar un coeficiente de acortamiento para el 5 armnico:
K

a.'

e;

= cos - 2 = cos --- 2

hp

5 36

= cos 90 = O

(2.122)

Engeneral, para eliminar por acortamiento de bobina una f.e.m. del armnico de orden h, ser necesarioelegir un ngulo p, de valor lIh del paso polar. As, para el 7. armnico el ngulo elctrico de acortamiento deber ser: po:

=-

1 180 = 25,71 7

(2.123)

Es evidente que si se desean eliminar simultneamente los armnicos 5. y 7., deber elegirse un ngulo de acortamiento comprendido entre los valores calculados en (2.121) y (2.123); por ejemplo, si se toma p = 30 resultarn los siguientes coeficientes de acortamiento:
0,

KaS

5 30 .= cos -- 2 = 0,259

Ka7

= cos --

7 30 2

= - 0,259

(2.124)

que no logra una anulacin completa de ninguno de los dos armnicos pero donde se ha conseguidouna reduccin ostensible de ambos. (El signo menos en Ka7 indica la inversin de fase en el 7. armnico). Si se aprovecha simultneamente la distribucin de las bobinas en diferentes ranuras, se podrlograr una reduccin adicional. Supngase, por ejemplo, un devanado trifsico con tres ranuraspor polo y fase (devanado de 1 capa). El ngulo elctrico entre ranuras ser: py

paso polar 180 = -- = 20 ranuras/polo 3 3

(2.125)

138

MQUINAS ELCTRICAS

y al ser q = 3, aplicando (2.114) resultar:

3520 sen--2 K ----dS5.20 3 sen-2

---=0218'

sen 150 3 sen 50

"

K/=----

2 7 . 20 3 'sen-2

sen

3720
---

sen 210

3 sen 70

= -0,177

(2.126)

De este modo los factores de devanado resultantes para los armnicos 5. y 7. sern:
KS = KdSKaS= 0,259' 0,218 K7 = Kd7Ka7=

= 0,0565 (- 0,259)(- O,177 ) = 0,0458

(2.127)

que nos indica una reduccin bastante efectiva de estos armnicos. Deben destacarse dos aspectos adicionales: 1) Al efectuar un acortamiento y distribucin de las bobinas se reducirn los armnicos no slo de tensin sino tambin de f.m.m., lo que se traducir en una mayor pureza de la onda de f.e.m. de salida. 2) La realizacin de tales medidas tambin reduce el valor de la f.e.m. fundamental, pero esta disminucin se ve compensada con creces por la perfeccin en la senoide de salida. As, por ejemplo, en la situacin aqu estudiada con:
p

= 30

; q

=3

; py

= 20

(2.128)

los coeficientes de devanado para la onda fundamental, de acuerdo con (2.107) y (2.111), sern: 30 K cos - = 0,966
a

KJ =

sen 30 3 sen 10

0,960

(2.129)

lo que representa un factor de reduccin:


K

= KaKd = 0,966 . 0,96 = 0,927

(2.130)

que prcticamente no modifica la magnitud de la f.e.m. fundamental (representa una reduccin inferior al 8 por 100).

2.10. PAR ELECTROMAGNTICO ELCTRICAS

EN LAS MQUINAS

En el epgrafe 1.8 del Captulo 1 se estudi la creacin de un par electromagntico en un sistema de rotacin desde el punto de vista del acoplamiento magntico entre bobinas, es decir, basado en las variaciones que sufren las inductancias de los devanados al cambiar la posicin del rotor. Este enfoque tiene la gran ventaja didctica de que al considerar las bobinas como arrollamientos concentrados no es necesario profundizar sobre cmo estn constituidos los devanados: distribucin en ranuras, acortamiento de bobinas, etc., y solamente se presta atencin a los fenmenos implicados en la produccin del par: posicin de reluctancia mnima, alineamiento entre ejes, etc. Existe un procedimiento alternativo para demostrar cmo se genera el par electromagntico en una mquina elctrica que se basa en la interaccin de las f.m.m.s. producidas por los

CAPTULO

2.

PRINCIPIOS GENERALES DE LAS MQUINAS ELCTRICAS

139

devanados situados en el esttor y en el rotor. Se ha estudiado en el epgrafe 2.8 de este captulo la produccin de f.m.m.s. por diversos devanados; por ejemplo, cuando el esttor tienepolos salientes y se sita un devanado concentrado alimentado con c.c., la f.m.m. correspondiente es de amplitud constante y fija en el espacio; en el caso de que los polos estn colocadosen el rotor y si ste se mueve a velocidad angular mecnica wm' la f.m.m. aunque es constante en amplitud se mueve con una velocidad angular elctrica pWm' En el caso de que se disponga en el esttor de un devanado trifsico (o en general polifsico) alimentado por corrientes trifsicas (en general polifsicas) de pulsacin w) = 2nfi' la f.m.m. resultante es, segn (2.62), de amplitud constante, se distribuye senoidalmente por el entrehierro y adems es de naturaleza rotativa, por lo que para su representacin se debe utilizar el concepto de [asor espacial. El fasor espacial es una generalizacin del fasor temporal utilizado en el estudio de los circuitos de c.a. y es un vector cuyo mdulo es igual a la magnitud que representa: campo magntico,j.m.m., etc., y que apunta hacia el lugar del entrehierro en el que su valores mximo positivo; en el caso de que se trate de magnitudes fijas en el espacio, el fasor espacial tambin tendr una posicin fija en el espacio (lo que significa que no se trata de un fasor sino de un vector espacial), pero si representa un campo magntico o f.m.m. rotativa, el correspondiente fasor espacial tambin girar a la velocidad correspondiente a la magnitud querepresenta. Cuando el devanado trifsico se sita en el rotor, el fasor espacial de f.m.m. se mover respecto al propio rotor a una velocidad angular correspondiente a su frecuencia; si sta esf2' la velocidad angular ser w2 = 2nf2' pero a la que habr que sumar o restar en su caso la velocidad angular elctrica del rotor pwm; si se considera que ambos sentidos son coincidentes, la velocidad angular elctrica total de la f.m.m. del rotor ser igual a w2 + pWm Y que de acuerdo con (2.86) es igual a OJ). Esto significa que si, por ejemplo, se dispone de devanados trifsicos tanto en el esttor como en el rotor (lo que se cumple si se trata de una mquina asncrona o de induccin), las f.m.m.s. se mueven a la misma velocidad angular elctrica correspondiente a la pulsacin de las corrientes estatricas w)' Esta coincidencia entre ambas velocidades es un principio fundamental en todas las mquinas elctricas; por ejemplo,en las mquinas sncronas el rotor o inductor se alimenta con c.c. w) = O,y al girar a una velocidad angular mecnica (0"" la velocidad angular elctrica correspondiente de su f.m.m.ser igual a pw",; si se considera que en el esttor se ha colocado un devanado trifsico, la pulsacin de sus corrientes w) debe ser igual al valor anterior, es decir, se cumple w2 = pw"" lo que est de acuerdo con la expresin general (2.86). Para poder comprender cmo se combinan las f.m.m.s, de una mquina elctrica para engendrar un par electromagntico se va a considerar una mquina con simetra cilndrica tal como se muestra en la Figura 2.33a. Se va a suponer que tanto en el esttor como en el rotor se sitan sendos devanados trifsicos (polifsicos) por los que circulan corrientes trifsicas (polifsicas) de tal manera que las f.m.rn.s. producidas se distribuyen senoidalmente por el entrehierro y girando, como ya se ha indicado en el prrafo anterior, a la misma velocidad angular elctrica (1)) (esta configuracin corresponde realmente a una mquina asncrona, pero el anlisis es similar para cualquier tipo de mquina elctrica). Debe destacarse que para simplificar el dibujo, en la Figura 2.33a no se han representado los devanados reales de la mquina sino unos arrollamientos equivalentes que producen las f.m.m.s.de los devanados del esttor F) y del rotor F2 (que de un modo simplificado equivalen a la aparicin de sendos polos norte y sur, tanto en el esttor como en el rotor). En la Figura 2.33b se muestran los fasores espaciales correspondientes, que giran a la misma velocidad angular elctrica (1)) y que estn desfasados entre s un ngulo elctrico J, que se denomina ngulo de par o ngulo de potencia porque su valor depende de la carga aplicada a la mquina.

140

MQUINAS ELCTRICAS

1(+)
70(-) Motor

Eje rotor
WI

/
b) Composicin de las f.m.m.s.

a) Situacin de los polos en esttor y rotor

Figura 2.33. Mquinaelctrica de polos lisos El par electromagntico que aparece entre el esttor y el rotor es en definitiva consecuencia de la accin que tiene lugar entre ambas ondas de f.m.m. para intentar alinear sus ejes magnticos (en su versin con los polos o imanes equivalentes, lo que sucede es que el polo norte/sur del rotor intentar seguir al polo sur/norte mvil del esttor). En rgimen permanente el ngulo b es constante; en otras palabras, los fasores espaciales de f.m.m. se mueven a la misma velocidad angular wl Y estn desfasados UlI ngulo elctrico b, dando lugar a una f.m.m. resultante F en el entrehierro que al igual que sus componentes tiene una evolucin senoidal en el espacio. La relacin entre F, F2 Y F se obtiene de la diagonal del paralelogramo y se puede escribir: (2.131) donde Fp F2 y F son los valores mximos o de pico de las ondas de f.m.m. Suponiendo la permeabilidad del hierro infinita y el campo magntico radial, la ley de Ampre permite calcular el valor de la intensidad de campo magntico H en el entrehierro y que viene expresada por:

H=-

(2.132)

donde g es el espesor del entrehierro y H se expresa en valores mximos (al igual que F). Si el circuito magntico es lineal, coinciden los valores de la energa y coenerga magntica almacenadas en el entrehierro por unidad de volumen y cuyos valores, de acuerdo con (1.44), son:

wm =

W'"

'H2

2 /-lo

med

2 /-lo 2 =

H2

11

"'0

F2

4g2

(2.133)

En la ecuacin anterior se ha tenido en cuenta que al ser el campo H senoidal, el valor medio del cuadrado de una onda de este tipo es igual a la mitad del cuadrado de su valor mximo. Si se considera que el dimetro medio del entrehierro es D y que la longitud axial de la mquina es la coenerga magntica almacenada en el entrehierro ser entonces:

e,

W~ = w~ (volumen del entrehierro)

= /-lo

F2
2

4g

(n De g)

(2.134)

CAPTULO 2.

PRINCIPIOS GENERALES DE LAS MQUINAS ELCTRICAS

141

y al sustituir el valor de F calculado en (2.130) en la ecuacin anterior (2.134) resulta:

Wm =

110nDe 4g

(FI + F2 + 2FIF2 cos 15)

(2.135)

De acuerdo con (1.129), el par electromagntico es la derivada de la coenerga magntica respectoa la posicin geomtrica del rotor, cuyo valor en grados elctricos es = 15 y que equivalea b/p en grados mecnicos o geomtricos, siendo p el nmero de pares de polos de la mquina.De este modo se puede escribir:

T= -dondeK = P 110 n D
2g

oW~

o(b/p)

= -p

110 tt De
2g

FIF2 sen b = -KFFZ sen 15

(2.135)

e es una constante que depende de las dimensiones fsicas de la mquina.

La ecuacin anterior indica que el par electromagntico es proporcional a los valores mximoso de pico de las f.m.m.s. del esttor F y del rotor F2 y tambin al seno del ngulo b que forman ambos fasores espaciales. El signo menos significa que las f.m.m.s. tienden a alinearse reduciendo el ngulo 15 que forman ambas ondas. Debe destacarse que-el par anterior aparecetanto en el esttor como en el rotor, siendo contrarios entre s. Si la mquina funciona comomotor, el par anterior provocar el movimiento del rotor (y si funciona como generador tendera frenarlo). En ambos casos el par en el esttor se transmitir a la cimentacin de la mquina. Se pueden escribir frmulas alternativas a (2.135) teniendo en cuenta que, segn la Figura 2.33b, se cumplen las siguientes igualdades: F sen b = F sen b2 ; Fz sen b = F sen b porlo que (2.135) admite tambin las siguientes expresiones equivalentes: T = - KFF sen b ; T = -KFF2 sen b2 (2.137) (2.136)

Encada una de las ecuaciones anteriores interviene la f.m.m. resultante F e incluyen en cada casosolamente una de las f.m.m.s. parciales, bien sea F o F2 y el seno del ngulo que forman cadauna de ellas con la f.m.m. resultante F. Si se desprecia la saturacin magntica, para que seconsidereque hay proporcionalidad entre f.m.m. y flujo se puede obtener otra frmula del par en funcin del flujo magntico por polo. Tngase en cuenta para ello que la induccin magnticamxima en el entrehierro B producida por la f.m.m. resultante F, teniendo en cuenta(2.132), es igual a: (2.138)
y por consiguiente el flujo medio por polo tendr un valor:

2 2 110F cI>=-B(readelpolo)=-_ ti t: g

(n-e
2p

D )

(2.139)

El factor 2/n de la expresin anterior procede de expresar el valor medio de la senoide de induccinen funcin de su valor mximo; se ha tenido tambin en cuenta que un polo cubre l/2p de la circunferencia total del entrehierro, que es de espesor g, y que la mquina tiene una

142

MQUINAS ELCTRICAS

profundidad en sentido axial igual a . De este modo la expresin (2.135) se puede escribir en funcin del flujo del siguiente modo: T

= -"2 /

ti

<l> F sen b

= "2 p

ti

<l> F2 sen b2

(2.140)

donde el flujo magntico <l> es consecuencia de la f.m.m. resultante F producida por ambos devanados.

2.11. CLASIFICACIN GENERAL DE LAS MQUINAS ELCTRICAS


En la expresin (2.88) se han relacionado las frecuencias del inductor e inducido de una mquina elctrica por medio del nmero de polos y las r.p.m. del rotor, de acuerdo a la ecuacin general: (2.141) Los diferentes tipos de mquinas se pueden clasificar (vase Fig. 2.34) atendiendo a una serie de criterios selectivos y ordenados que tienen en cuenta la existencia de rganos mviles, el tipo de corriente aplicada al inductor y la forma de las conexiones externas, de acuerdo con el siguiente proceso: PRIMER CRITERIO: Movimiento del inductor o inducido.
a)

Si no existen rganos mviles, n = O, lo que indica, de acuerdo con (2.141), que: (2.142) es decir, coinciden las frecuencias del inductor e inducido, resultando las llamadas mquinas estticas. En el caso en que n i= O, se sigue conservando la expresin general:

b)

!2=!dando origen a las mquinas rotativas.

np 60

(2.143)

SEGUNDO CRITERIO: Caractersticas del flujo inductor.


a)

Si el flujo inductor es constante, es decir, independiente del tiempo, indica que la frecuencia de la corriente que alimenta este devanado es], = O. En el caso de mquinas estticas, teniendo en cuenta (2.142) se cumplira: (2.144) no existiendo ninguna mquina que cumpla esta condicin, ya que al no aparecer variaciones de flujo en el inducido no se tiene conversin de energa.

CAPTULO 2.

PRINCIPIOS GENERALES DE LAS MQUINAS ELCTRICAS

143

MqUINAS ESTATICAS

Transformador SI M. sncronas

NO

s
MQUINAS ROTATIVAS

Ql el

bO

M. corriente continua

M. asncronas

np -;!.L 7'

60

Jo-

Motores c.a. universales

Figura 2.34. Cuadro de clasificacin general de las mquinas elctricas.

Para las mquinas dotadas de movimiento, la condicin f = O,teniendo en cuenta (T143), da lugar a la expresin general:

J;=-

np 60

(2.145)

es decir, la frecuencia del inducido es funcin directa de la velocidad del rotor. Normalmente cuando el inductor est recorrido por una c.c. (JI = O),la mquina tiene una estructura fsica en forma de polos salientes, tal como se observa en las Figuras 2.5b y e, sobre los cuales van arrolladas las bobinas inductoras. b) Si el flujo inductor es variable, eS!1 =1= O, lo cual indica, en el caso de mquinas estticas, y de acuerdo con (2.142), que: (2.146)
y para las mquinas rotativas se sigue cumpliendo la expresin general (2.141).

144

MQUINAS ELCTRICAS

TERCER CRITERIO: Dispositivo de conexin al circuito exterior.


a)

Si la mquina tiene un inducido cuya unin con el circuito exterior se realiza por medio de conexiones fijas (caso en que el inducido est situado en el esttor) o por anillos (inducido en el rotor), de acuerdo con lo explicado en el epgrafe 2.3, la frecuencia en el circuito exterior definida por JL es igual que la frecuencia del inducido J2' es decir: (2.147) en el caso de mquinas estticas se obtiene el transformador y el regulador de induccin monofsico. En el caso de mquinas mviles con JI = O, teniendo en cuenta (2.141), se cumplir: (2.148) que dan origen a las mquinas sncronas constituidas por el generador sncrono o alternador y el motor sncrono. En el caso de mquinas mviles cou ], t= O, teniendo en cuenta que responden a la expresin general (2.141), se cumplir: (2.149) que dan lugar a las mquinas asncronas o de induccin, constituidas por el generador asncrono, el motor asncrono y los convertidores asncronos. Si la mquina tiene un inducido cuya unin con el circuito exterior se realiza por medio de colector de delgas, de acuerdo con lo explicado en el epgrafe 2.3, la frecuencia en el circuito exterior es de diferente valor que la del inducido, es decir: (2.150) el colector de delgas se coloca siempre en el rotor, y por ello en las mquinas estticas esta combinacin no existe. En las mquinas mviles en que JI = O, de acuerdo con (2.145) se cumplir: np

b)

JLt=J260 JL

(2.151)

que dan origen a las mquinas de c.c. (denominadas as porque en ellas se cumple: = O); estas mquinas incluyen: el generador de c.c. o dinamo, el motor de c.c., la conmutatriz y otras mquinas especiales como la amplidina, la metadina, el rototrol, etc. En las mquinas mviles en las quej', t= O, teniendo en cuenta que responden a la expresin general (2.141), se cumplir: (2.152) que dan lugar a los motores de e.a, con conmutador o con colector de delgas.

En el cuadro de clasificacin general de las mquinas elctricas de la Figura 2.33, los cuadros en forma de rombos indican los interrogantes a cada uno de los criterios comentados. Con este diagrama se puede estudiar de forma cualitativa y general el funcionamiento y

CAPTULO

2.

PRINCIPIOS GENERALES DE LAS MQUINAS ELCTRICAS

145

propiedades de cada grupo de mquinas, tal como se va a realizar en el epgrafe 2.12. En posteriores captulos se har un anlisis ms profundo de cada una de las mquinas elctricas.

2.12. ANLISIS CUALITATIVO DE LAS PRINCIPALES MQUINAS ELCTRICAS


Vamos a realizar en este apartado un anlisis bsico de funcionamiento de las principales mquinas elctricas' a partir del cuadro general de clasificacin de la Figura 2.34.

2.12.1. Transformadores
Son mquinas estticas, n = O, constituidas por dos devanados, inductor e inducido. El devanado inductor se conecta a una fuente de c.a. de frecuencia II y se denomina primario. El devanado inducido tiene una frecuencia j, = f. y entrega energa elctrica a un circuito exterior por medio de conexiones fijas (fL = 12); este arrollamiento inducido recibe el nombre de secundario. Para aumentar la induccin magntica del sistema y mejorar el acoplamiento entre ambos devanados, stos van arrollados sobre un ncleo cerrado comn, constituido por un apilamiento de chapas magnticas de acero al silicio, tal como se muestra en la Figura 2.35. Si la tensin entre los terminales del arrollamiento primario, VI' es menor que la tensin secundaria, V2, es decir, VI < V2, el transformador se denomina elevador; en el caso contrario se llama transformador reductor (VI > V). En el supuesto de considerar el transformador ideal, sin prdidas ni cadas de tensin, de acuerdo a la expresin (2.92) se cumplir en primario y secundario: (2.153) siendo <1>", el flujo mximo que atraviesa ambos arrollamientos y NI y N2 los nmeros de espiras respectivas. De las ecuaciones anteriores (2.153) se deduce:
-=-=/n

VI

NI

V2

N2

(2.154)

Red primaria de alta tensin

Red secundaria de baja tensin

Centralelctrica

TRANSFORMADOR

Motores y otras cargas

Figura 2.35. Transformador.

146

MQUINAS ELCTRICAS

que se denomina relacin de transformacin. Si designamos con /1 e /2 las corrientes primaria y secundaria, en el caso ideal se cumplir: (2.155) que expresa la igualdad entre las potencias de entrada y salida. Teniendo en cuenta adems (2.154) se puede poner: (2.156) que indica que para una determinada potencia a transmitir, si se eleva la tensin V2 > V]' se obtiene /2 < 11, lo que indica la conveniencia de instalar los transformadores en las grandes redes elctricas, pues manejando altas tensiones las corrientes se reducen y como consecuencia de ello se requiere una menor seccin en los conductores de cobre de la lnea, con el consiguiente ahorro econmico. Los transformadores permiten, en consecuencia, adaptar de un modo sencillo las tensiones de las redes a los valores ms adecuados y econmicos. Si el devanado secundario se coloca en un rotor de tal forma que se pueda controlar la posicin entre el inductor y el inducido, se obtiene el llamado regulador de induccin monofsico, que permite obtener una tensin V2 funcin de la posicin. En la prctica los reguladores de induccin son trifsicos y se basan en un principio de funcionamiento diferente, ms cercano a los de las mquinas asncronas o de induccin.

2.12.2. Mquinas sncronas


Son mquinas rotativas, n i= O,y de acuerdo con la Figura 2.34 se caracterizan por ser: (2.157) Es decir, consisten en un inductor alimentado por c.c. (JI = O),que se denomina tambin devanado de excitacin o campo, que suele colocarse en el rotor (Fig. 2.36), alimentado por medio de dos anillos. El inducido normalmente es trifsico y suele colocarse en el esttor (en las mquinas de pequea potencia se utiliza con frecuencia la posicin inversa, es decir, se sita el inductor en el esttor y el inducido en el rotor, existiendo entonces tres anillos en el rotor). Cuando funciona como generador (alternador), se introduce energa mecnica por el eje, y al aplicar c.c. al inductor, se obtiene en el inducido una f.e.m. de frecuenciaj, = np/60, que se aplica a la carga. La c.c. necesaria para alimentar el inductor se obtiene de una pequea dinamo excitatriz, que est situada en el mismo eje de la mquina (vase epgrafe 2.12.3)*. El alternador es con mucho la mquina generadora ms importante que existe, y proporciona la mayor parte de la energa elctrica que hoy se consume; estn situados en las centrales hidrulicas, trmicas y nucleares con potencias de hasta 1.000 MW; tambin se encuentran en los grupos electrgenos acoplados a motores de combustin interna.

* Modernamente la c.c. se extrae de un alternador piloto colocado en el mismo eje. previa rectificacin (conversin de la c.a. en c.c.).

CAPTULO

2.

PRINCIPIOS GENERALES DE LAS MQUINAS ELCTRICAS

147

Inductor de polos salientes (> 4 polos) (Centrales hidrulicas)

Anillos para introducir la corriente continua para los polos

Entrada elctrica (motores sncronos)

Inductor de polos lsos (2 o 4 polos) (centrales trmicas)

Figura 2.36. Tipos de mquinas sncronas. Se observa que la frecuencia de la cargaj; que coincide con la del inducido lfL = f2 = = np/60) es directamente proporcional a la velocidad; como quiera que la frecuencia es una magnitud que debe mantenerse esencialmente constante, para que sea posible un enlace entre las diversas centrales de un pas es preciso que los motores primarios que mueven los alternadores: turbinas hidrulicas del tipo Pelton, Francis y Kaplan, turbinas trmicas, etc., giren a velocidad constante; para ello se dota a estas ltimas mquinas de reguladores tacomtricos, que actan sobre la entrada de agua o vapor en formas muy diversas. La mquina sncrona puede funcionar tambin como motor introduciendo una c.a. de frecuenciaj', por el inducido (teniendo el inductor f, = O), apareciendo un par en el rotor que lo hace girar a velocidad:
11

60f) =--P

(2.158)

cuya magnitud se observa que es funcin directa de la frecuencia (velocidad de sincronismo). Este motor tiene el inconveniente de que gira a una velocidad fija, con el consiguiente problema de arranque y prdida de sincronismo cuando se producen pares de frenado bruscos. Los motores sncronos se utilizan cuando interesa una gran constancia en la velocidad, como en relojes elctricos y en algunos tipos de servomecanismos. Como quiera que tambin tienen la propiedad de poder regular su f.d.p. actuando sobre la c.c. de excitacin, se emplean tambin para regular el f.d.p. de las instalaciones (se dice entonces que funcionan como condensadores sncronos). El mismo alternador de una central elctrica puede funcionar como motor sncrono, operacin que se realiza en las modernas centrales de bombeo tomando energa elctrica de la red y acumulando energa hidrulica aguas arriba de la presa.

2.12.3. Mquinas de c.c.


Son mquinas rotativas, n
=1=

O, y de acuerdo con la Figura 2.34, se caracterizan por ser: (2.159)

148

MQUINAS ELCTRICAS

El nombre de mquinas de c.c, se debe a que el valor de la frecuencia de la carga es f = O, lo cual se consigue por la accin rectificadora del colector. Estas mquinas disponen de un inductor alimentado por c.c. (h = O), que al igual que las mquinas sncronas se denomina tambin devanado de excitacin o campo y se sita en el esttor (Fig. 2.37). Cuando funciona en rgimen generador, se suministra una energa de rotacin al eje y se aplica una c.c. a la excitacin, obtenindose en el inducido una corriente alterna de frecuencia:
f2 =-

np

60

(2.160)

Debido a la accin rectificadora del colector se obtiene una c.c. entre las escobillas de salida
(fL = O),energa que es aplicada a la carga. El generador de c.c. se conoce con el nombre de

dinamo, y tiene la importancia histrica de ser el primer tipo de generador empleado para la obtencin de energa elctrica a gran escala. Hoy en da se han sustituido para esta misin por rectificadores que permiten obtener una c.c. por conversin de la c.a. de la red. Cuando funcionan como motores de C.C., es preciso introducir c.c. por el inductor y por las escobillas del inducido, apareciendo un par que hace girar el rotor de la mquina. La velocidad de giro puede regularse fcilmente, controlando la corriente del inductor o del inducido, o de ambas a la vez. Esta facilidad de regulacin de velocidad de los motores de C.C., unida a los altos pares de arranque que se pueden obtener, ha hecho que este tipo de motor fuera insustituible en aquellas aplicaciones que necesitaban velocidad variable, tales como trenes de laminacin y traccin elctrica. (En Espaa, la traccin elctrica se realiza a 3.000 V de c.c. en los ferrocarrilesy de 600 V a 1.500V de c.c. en los trenes metropolitanos Metro*. El tren de alta velocidad espaol AVE funciona con corriente alterna monofsica a 25 kV). Si los devanados inductor e inducido llevan alimentaciones separadas, se tiene la llamada mquina con excitacin independiente. Normalmente, ambos circuitos se conectan elctricamente entre s; cuando se conectan en paralelo, se obtiene la mquina shunt o derivacin, en
Terminales de alimentacin al inductor (excitacin) Estator con polos salientes (inductor)

Figura 2.37. Mquina de c.c.

* Desde el ao 2002, el tren metropolitano de Madrid, debido a su gran extensin, ha comenzado a construir redes de 1.500 V de c.c. para alimentar algunas de sus lneas. Por ejemplo, la nueva lnea 8, que enlaza Nuevos Ministerios con el aeropuerto de Barajas, se inaugur en mayo de 2002 con las unidades 5/8.000, que disponen de equipo de traccin bitensin (1.500/600 V de c.c.) y con motores de c.a. asncronos alimentados mediante onduladores o inversores electrnicos equipados con IGBT.

CAPTULO 2.

PRINCIPIOS GENERALES DE LAS MQUINAS ELCTRICAS

149

la que el inductor est formado por un devanado de muchas espiras de hilo delgado por el que se deriva una corriente pequea. Cuando se conectan en serie, el inductor tiene pocas espiras de hilo grueso, ya que por l circula toda la corriente del inducido (de gran valor). Si la mquina dispone de ambos tipos de excitacin, serie y paralelo, se denomina compound o compuesta. Estos sistemas de conexin se emplean para producir la autoexcitacin de la mquina, a partir del magnetismo remanente de los polos inductores. Es frecuente (aunque no son normas generales) emplear la conexin serie en motores y la shunt y compound en generadores. Si a la mquina de c.c. bsica de la Figura 2.37 se la hacen conexiones adecuadas en el devanado del inducido y son llevadas al exterior por medio de anillos colectores como indica la Figura 2.38, se obtiene una mquina denominada conmutatriz o convertidor sncrono. Tal mquina puede funcionar: a) entregando al exterior c.a. y C.C.; b) trabajando como motor sncrono y generador de c.c. simultneamente; e) trabajando como motor de c.c. y generador de c.a. simultneamente. Este tipo de mquina se ha utilizado sobre todo funcionando como en el caso b) y se empleaba para convertir, en un solo grupo, la c.a. de la red en c.c. (por ejemplo, para suministrar la energa de c.c. de los tranvas); hoy en da ya han desaparecido, siendo sustituidas por convertidores estticos electrnicos.

2.12.4. Mquinas asncronas o de induccin


Son mquinas rotativas, n i= O, Y de acuerdo con el cuadro de clasificacin Figura 2.34 se caracterizan por: general de la

(2.161) Estn constituidas por un devanado inductor situado en el esttor por el que se introduce unac.a. de frecuencaj.. En el caso de mquinas de potencia superior a 1/2 CV, el devanado anterior es trifsico, al igual que la corriente de alimentacin, apareciendo, de acuerdo con el apartado 2.8.3, un campo magntico giratorio cuya velocidad, teniendo en cuenta la expresin (2.68), es: nl=-p
Terminales de alimentacin Devanado inducido

60!1

(2.162)

\
,,\\\\\\\\\\\\\\\\\\\~~\\\~
,"" ,. ...#

,~Delgas

Rotor con anillos y con colector de delgas

Figura 2.38.

Conmutatriz.

150

MQUINAS ELCTRICAS

El devanado inducido est en el rotor y puede ser trifsico o no; sin embargo, y como se comprender ms adelante al estudiar el Captulo 4, debe estar bobinado para el mismo nmero de polos que el devanado del esttor. En la Figura 2.39 se indica el esquema bsico, donde se ha supuesto un arrollamiento trifsico en el rotor, cuyas salidas van a tres anillos, donde se conecta no una carga, sino un restato de arranque. En la mayora de los casos el rotor est formado por una serie de conductores puestos en cortocircuito por dos anillos extremos, formando un devanado que se conoce con el nombre de jaula de ardilla. La mquina puede funcionar como:
a) Motor. Es el caso ms normal. En esta situacin el campo giratorio del esttor induce f.e.m.s. en el devanado del rotor y al estar ste en cortocircuito (jaula de ardilla) o cerrado por medio de un restato de arranque (rotor devanado o con anillos) aparecen corrientes en el rotor que, al reaccionar con el campo giratorio del esttor, mueven la mquina a una velocidad n muy cercana y por debajo de la de sincronismo ni' de tal forma que la identidad segunda de (2.161) queda expresada:

(2.163) Se denomina deslizamiento v al cociente: (2.164) que teniendo en cuenta (2.162) se expresa: ---n p s=____:_-60JI 60JI

J2
JI

(2.165)

P
Barras de la jaula

Figura 2.39. Tipos de mquinas asncronas o de induccin.

CAPTULO 2.

PRINCIPIOS GENERALES DE lAS MQUINAS ELCTRICAS

151

Los deslizamientos a plena carga de estos motores que giran a una velocidad asncrona respecto al campo giratorio del esttor varan entre el 3 y 8 por 100 y es difcil de regular; sin embargo, la simplicidad y robustez de estos motores (sobre todo en el caso de rotor en jaula de ardilla) los hacen aptos para todo tipo de trabajo en el que no sea necesario un control preciso de la velocidad, como en gras, ascensores, mquinas herramientas, hormigoneras, etc., y por ello es la mquina electromagntica de mayor aplicacin en la ingeniera, cubriendo ms del 80 por 100 de los motores elctricos empleados en la industria. En potencias pequeas 1/2 CV) el esttor es monofsico y de acuerdo con el apartado 2.8.4 se obtienen dos campos giratorios de sentido contrario, que no producen par de arranque en el rotor, teniendo que recurrir a procedimientos especiales de arranque, como ya se explicar en el Captulo 4.
b) Generador. Si girando la mquina asncrona como motor, a una velocidad n < n1, se obliga a mover el rotor, por un medio exterior, a una velocidad superior a la de sincronismo y en su mismo sentido, de acuerdo con (2.164) el deslizamiento se hace negativo y la mquina absorbe entonces energa mecnica que se convierte en elctrica, devuelta a la red por el esttor a frecuenciaj.. La mquina trabaja entonces como generador, pero este tipo de funcionamiento no se utiliza casi nunca en la prctica porque no es autnomo, siendo necesaria la red elctrica de alimentacin para suministrar la corriente de magnetizacin de la mquina. No obstante, existen procedimientos de autoexcitacin de generadores asncronos a base de condensadores, como comprobar el lector en el Captulo 4. e) Convertidor de frecuencia. Si se alimenta el esttor de una mquina elctrica por medio de una red de c.a. de frecuenciaj y se mueve el rotor por un medio mecnico exterior a velocidad n, se obtiene una frecuencia en el rotor, cuyo valor est definido en (2.161):

np !,=J;I 60

(2.166)

el sumando es positivo o negativo segn que el sentido de giro del rotor tenga diferente o igual sentido que el campo giratorio del esttor. La mquina recibe energa elctrica por el inductor y energa mecnica por el eje, de tal forma que por los anillos del inducido se puede alimentar una carga elctrica a frecuencia ; #- !I'

2.12.5. Motores de c.a. de colector. Motores universales


Son mquinas rotativas, n #- O, y de acuerdo con la Figura 2.34 se caracterizan por ser: (2.167) es decir, consisten en un inductor, situado en el esttor, alimentado generalmente por c.a. monofsica. El inducido est en el rotor y dispone de colector de delgas con una apariencia fsica anloga a las mquinas de c.c. (Figura 2.37). Normalmente los devanados del esttor y rotor van en serie, resultando una mquina con caractersticas similares al motor serie de c.c. En su versin de pequea potencia (fraccin de CV) son muy empleadas en aparatos electrodomsticos: batidoras, mquinas de afeitar, taladros elctricos de mano, secadores, etc. Con potencias ms elevadas se utilizan en traccin elctrica a frecuencias que oscilan entre 50 Hz y 50/3 = 16,66 Hz. Pueden adaptarse tambin a un funcionamiento con c.a. o C.C., recibiendo entonces el nombre de motores universales.

152

MQUINAS EL'CTRICAS

PROBLEMAS
2.1. Una mquina elctrica de 100kVA tiene unas prdidas fijas de 1.000 W y unas prdidas variables de 1.500 W a plena carga (asignada). Calcular: a) Rendimiento de la mquina, cuando trabaja a l/2 de la plena carga con f.d.p. 0,8. b) Potencia de mximo rendimiento. e) ndice de carga ptimo. [Resp.: a) 96,68%; b) 81,65 kVA; e) 0,8165.] 2.2. Una mquina de 20 kVA tiene unas prdidas en el cobre a plena carga de 400 W, unas prdidas en el hierro de 150 W y unas prdidas mecnicas de 200 W. Determinar: a) Rendimiento a 3/4 de la plena carga con f.d.p. 0,8. b) Rendimiento a plena carga con f.d.p. 0,9. e) Potencia aparente de mximo rendimiento. d) Rendimiento en el caso anterior para f.d.p. unidad. [Resp.: a) 95,43 %; b) 96%; e) 18,71 kVA; d) 96,39 %.] 2.3. Se tiene la estructura magntica de la Figura P.2.1. El entrehierro tiene un espesor g. Se coloca en el rotor una bobina de paso diametral AA' formada por N espiras por las que circula una corriente i. Dibujar la curva de induccin B producida y dar la expresin del desarrollo en serie de Fourier de la misma. Se desprecia la reluctancia del hierro.
8(1.)

Figura P.2.l.

Figura P.2.2.

[Resp.: La curva de distribucin de la induccin se indica en la Figura P.2.2. El desarrollo en serie de Fourier es: [B(a) = ~ /Jo Ni (sen o: + ~ sen 3a + ... + ~ sen ha + ... ).] t: 2g 3 h 2.4. Se tiene la estructura magntica de la Figura P.2.3. Los parmetros son idnticos a los del problema 1, pero ahora las bobinas A_A'y B-B' estn acortadas un ngulo elctrico y. Dibujar la curva de induccin B producida y obtener el desarrollo en serie de Fourier de la misma. Deducir conclusiones prcticas.
B(1.)

Figura P.2.3.

Figura P.2.4.

CAPTULO 2.

PRINCIPIOS GENERALES DE LAS MQUINAS ELCTRICAS

153

[Resp.: La curva de distribucin de 8 se indica en la Figura P.2.4. El desarrollo en serie de Fourier es:
8(a) = - 110 cos - sen a + - cos - sen 3a + ... + - cos - sen ha + ... ti 2g 2 3 2 h 2

Ni (Y

3y

hy

Se observa que el resultado es el mismo que se obtendra considerando las bobinas de paso diametral, pero introduciendo un factor de acortamiento expresado por: Ka = cos

1,

donde h indica el orden del armnico.]

2.5. Considerar la estructura magntica de la Figura P.2.5, que contiene 8 ranuras donde se colocan 4 bobinas de N espiras cada una, recorridas por una corriente i. En el supuesto de despreciar la reluctancia del hierro, dibujar la curva de induccin magntica producida por el conjunto. Deducir conclusiones prcticas.

Figura P.2.S

Figura P.2.6

[Resp.: La distribucin de la induccin se muestra en la Figura P.2.6, cuyo desarrollo en serie de Fourier es: sen sen sen 4 Ni 2 1 2 1 2 8(a) = q. - 11 --sen a + sen 3a + ... + sen ha + ... n o 2g 8 3 38 h h8 q sen q sen q sen 2 2 2

q8

q38

qh8)

donde q indica el nmero de ranuras por polo, que en este caso vale 4. Se observa que el resultado es el mismo que el que se obtendra considerando las cuatro bobinas concentradas, pero introduciendo un factor de distribucin expresado por:

q"B
senKd = h
qsen

2
he

, donde 8 indica el ngulo elctrico entre ranuras consecutivas.]

2.6. Considerar la estructura magntica de la Figura P.2.7, donde se muestra un rotor bobinado con un arrollamiento uniformemente distribuido, con un total de N espiras. El devanado puede considerarse como una generalizacin del problema anterior cuando

154

MQUINAS ELCTRICAS

el nmero de ranuras se hace muy elevado. Dibujar la forma de B y obtener su desarrollo en serie de Fourier.
71/2

71

Figura P.2.7.

Figura P.2.S.

[Resp.: El esquema de B se indica en la Figura P.2.8, cuyo desarrollo es:


4 Ni B(r:x) = - 110 -_

sen n 2
-

2g

n 222

sen sen 3n hn) 1 2 1 2 sen r:x + - --sen 3r:x + .. , + - --sen ho: + .. , 3 3n h hn


-

El resultado es un caso particular del problema 2.5 cuando la extensin del devanado es l = n, que se denomina extensin de fase. La distribucin anterior es tpica de los arrollamientos del inducido de las mquinas de c.c.] 2.7. Considerar la estructura magntica de la Figura P.2.9, donde se muestra un rotor bobinado con un arrollamiento uniformemente distribuido, con un total de N espiras que ocupan una extensin de fase de a radianes. Dibujar la forma de la induccin B y obtener su desarrollo en serie de Fourier.
71/2
8(a)

Figura P.2.9.

Figura P.2.10.

[Resp.: El esquema de B es el indicado en la Figura P.2.1O,cuyo desarrollo es:


4 Ni B(r:x) = - 110 --

sen sen sen ha) a 3a 2 1 2 1 2 sen r:x + - -sen 3r:x + ... + - -sen ha + ... 2g a 3 3a h ha ( 2 2 2

CAPTULO

2.

PRINCIPIOS GENERALES DE LAS MQUINAS ELCTRICAS

155

que coincide con la solucin del probo 2.6 cuando la extensin de fase (J = n. Esta forma de B se obtiene en los devanados de excitacin de los turboaltemadores de las centrales trmicas.] 2.8. El diagrama de la Figura P.2.ll representa la forma del campo de un polo saliente de un alternador. Calcular: a) la amplitud del fundamental d la onda de induccin; b) amplitud de la onda fundamental cuando () = n/6; e) factor de forma de la onda de induccin cuando () = n/6.
B(cx)

2n

Figura P.2.11. 4Bm cos () 2fiBm ; b) --;


t:

[Resp.: a)

e)

- = 1,255.] 4

2.9. Se tiene el circuito magntico de la Figura P.2.l2 formado por un devanado inductor situado en el esttor, alimentado por una c.a. de 50 Hz, que produce un flujo senoidal en el entrehierro de valor mximo 8 mWb. En el rotor se sita un arrollamiento de 10 espiras concentradas de paso diametral. Calcular la f.e.m. inducida en la bobina del rotor cuando gira a 1.500 r.p.m. Si se considera como referencia que en t = O, () = O.

Eje magntico del estator

Figura P.2.12.

[Resp.: e = -25,12 cos 314 t cos 157 t + 12,56 sen 314 t sen 157 t.] _.10. Calcular el valor del factor de distribucin de una mquina trifsica que tiene 12 ranuras por polo. [Resp.: 0,958.]

156

MQUINAS ELCTRICAS

2.11.

El inducido de una mquina bipolar est completamente bobinado con N espiras uniformemente distribuidas de paso diametral. La f.e.m. inducida en cada espira tiene un valor eficaz de 10 V. Cul ser la f.e.m. inducida en todo el devanado con todas las espiras en serie? 20 [Resp.: - N voltios.]
1t

2.12.

El inducido de una mquina bipolar tiene dos bobinas de paso diametral, una de 100 espiras y otra de 50 espiras, que forman entre s un ngulo de 30. El inducido est en el rotor y se hace girar ste a una velocidad de 1.000 r.p.m. dentro del campo uniforme de los polos, cuya induccin vale 100 gauss. El rea de cada bobina es de 400 crrr'. Si las bobinas se conectan en serie, hallar la lectura que se obtendr en un voltmetro conectado entre las escobillas de la mquina. [Resp.: 4,31 Vo 1,83 V.] Un alternador trifsico de 20 polos tiene un devanado conectado en estrella de 180 ranuras y 10 conductores por ranura. El flujo por polo tiene un valor mximo de 0,04 Wb, y est distribuido senoidalmente en el entrehierro. La velocidad es de 300 r.p.m. Hallar las f.e.m.s. de fase y lnea, en el supuesto de que las bobinas sean de paso diametral. [Resp.: 2.557,45 V; 4.429,6 V.] Un alternador trifsico conectado en estrella tiene 6 polos y debe dar una f.e.m. de 380 V a 50 Hz. El esttor tiene 3 ranuras por polo y fase y 4 conductores por ranura. Calcular: a) las r.p.m. a las que debe girar el rotor; b) el flujo mximo por polo si tiene una distribucin senoidal. NOTA: Las bobinas son de paso diametral. [Resp.: a) 1.000 r.p.m; b) 0,0287 Wb.] El inducido de una alternador de 20 polos, 50 Hz, tiene un total de 180 ranuras. Calcular las f.e.m.s. que se obtienen en los siguientes casos: a) cuando se bobina un devanado monofsico que cubre 5 ranuras por polo; b) dem cuando se cubren todas las ranuras; e) cuando se bobina un arrollamiento trifsico que cubre todas las ranuras. Las bobinas son de paso diametral, y en cada ranura se colocan 6 conductores. El flujo est distribuido senoidalmente en el entrehierro y tiene un valor mximo de 0,025 Wb. [Resp.: a) 1.470 V; b) 1.920 V; e) 960 V/fase. Si se comparan los resultados a) y b) se observa que en el 2. caso se obtiene un 30 por 100 ms de f.e.m. pero empleando un 80 por 100 ms de material (cobre); de aqu se deduce la conveniencia de no devanar enteramente un inducido cuando el arrollamiento es monofsico.] Una mquina elctrica tiene un inducido con 9 ranuras por polo, estando las bobinas acortadas en 2 ranuras. Calcular el factor de acortamiento del devanado. [Resp.: 0,940.] Un rotor bipolar, excitado para dar un flujo mximo por polo de 0,02 Wb, gira a 3.000 r.p.m. dentro de un esttor que contiene 18 ranuras. Se colocan dos bobinas de 50 espiras A y B en el esttor del modo siguiente: Bobina A: Lados de bobina en las ranuras 1 y 11; Bobina B: Lados de bobina en las ranuras 2 y 10. Calcular la f.e.m. resultante cuando las bobinas se conectan en serie. La distribucin de flujo es senoidal. [Resp.: 437,25 V o O V; dependiendo de si las f.e.m.s. son aditivas o substractivas.] Resolver el problema anterior si la bobina B est situada entre las ranuras 2 y 12. [Resp.: 430,61 V o 75,93 V.]

2.13.

2.14.

2.15.

2.16.

2.17.

2.18.

CAPTULO 2.

PRINCIPIOS GENERALES DE LAS MQUINAS ELCTRICAS

157

2.19. Un alternador trifsico de 20 polos, conectado en estrella, gira a 300 r.p.m. El inducido tiene 360 ranuras y 6 conductores por ranura. La anchura de las bobinas es de 5/6 del paso polar. Si el flujo mximo por polo, de distribucin senoidal, es de 0,086 Wb, cules sern los valores de las f.e.m.s. de fase y de lnea? [Resp.: 6.347,3 V; 10.993,8 V.] 2.20. Se dispone de una mquina elctrica bipolar experimental de entrehierro uniforme, que tiene una distribucin de induccin en el entrehierro expresada por la ecuacin:

B(e~ =

1,05 cos

e + 0,35 cos 3e + 0,21 cos 5e + 0,15 cos 7e

Teslas

El paso polar es de 50 cm y la longitud del ncleo de 40 cm. Si el inducido consta de una bobina de 5 espiras con una anchura de 4/5 del paso polar y la frecuencia de la f.e.m. generada es de 50 Hz, calcular la f.e.m. inducida en la bobina y su valor eficaz. [Resp.: e = 199,72 cos e + 41,14 cos 3e - 17,63 cos 7e; e = oit; 144,72 v.]

BIOGRAFAS
DAVENPORT, Thomas (1802-1851). Inventor norteamericano. Comenz haciendo experiencias con electroimanes. En 1831 construy un motor elctrico, probablemente el primero que se conoce en la historia de la ingeniera elctrica. Consista en dos electroimanes fijos y otros dos mviles, situando estos ltimos en los radios de una rueda y estando unidos a un dispositivo conmutador; el motor se alimentaba mediante una batera de Volta. Este motor fue utilizado por Davenport para mover un cochecito alrededor de una va circular, lo que representaba en cierto modo el primer prototipo de ferrocarril con traccin elctrica. 2. FERRARIS, Galileo (1847-1897). Fsico e ingeniero italiano. Obtuvo su ttulo de ingeniero en 1869, y ley su tesis doctoral en 1872, que versaba sobre la teora matemtica de la propagacin de la electricidad en slidos homogneos. Fue profesor de la Escuela Militar de Turn. Presidente de la Exposicin Internacional de Electricidad de Turn en 1883, en la que se expuso por primera vez el transformador de Gaulard y Gibbs. Basndose en sus conocimientos sobre la polarizacin circular de la luz, se le ocurri hacer un experimento similar con electroimanes, colocando dos de ellos en ngulo recto y alimentndolos con corrientes en cuadratura y de este modo consigui un campo magntico giratorio (1885); ms tarde aplic estas ideas en la construccin de un motor basado en este principio (motor asncrono o de induccin), que present a la Real Academia de Turn en 1888. Se atribuye, sin embargo, a Tesla la paternidad de este motor y a Ferraris el invento del campo magntico giratorio. Ferraris particip con la AEG-Oerlikon en el diseo de la red de transporte de c.a. de 175 km de longitud entre Lauffen y Frankfurt para la Exposicin Internacional en esta ltima ciudad en 189l. 3. FrrZGERALD, Arthur Eugene (1909-1978). Ingeniero americano. Estudi en el Politcnico de Brooklyn (1929) y ms tarde se gradu en el MIT (1931), doctorndose en este centro en 1937. En 1931 ingres en el MIT en calidad de ayudante de investigacin, en 1940 era profesor ayudante, asociado en 1945 y catedrtico de Ingeniera Elctrica en 1952. Fue el responsable de la organizacin de las enseanzas de ingeniera elctrica en el MIT en electricidad; escribi para los primeros un libro de texto de ingeniera elctrica que fue adoptado por ms de lOOuniversidades de todo el mundo y para los segundos escribi un texto moderno sobre mquinas elctricas que tambin se ense en ms de 50 universidades (este ltimo libro fue escrito en colaboracin con Charles Kingsley). Dirigi el analizador de redes del MIT y supervis los estudios de sistemas elctricos de potencia. Trabaj tambin como ingeniero consultor de la empresa Jackson y Moreland. Fellow del IEEE en 1956. 4. KRON, Gabriel (1901-1967). Ingeniero elctrico y matemtico hngaro-americano. Se gradu en la Universidad de Michigan (1924). En 1926 ampli su formacin en Europa, dedicndose a estudiar la teora del anlisis tensorial. En 1934 se incorpor a la General Electric de EE. UU., para trabajar en el anlisis de sistemas elctricos de potencia. En 1935 gan el Premio Montefiore l.

158

MQUINAS ELCTRICAS

5.

6.

7.

8.

por su artculo: Non-Riemanian Dynamics uf Rotating Electrical Machinery. Escribi varios libros y una gran cantidad de artculos sobre las aplicaciones de la geometra no euclidiana y el clculo tensorial a la ingeniera elctrica. Sus aportaciones han sido a menudo comparadas con las que hizo Steinmetz en la teora de circuitos a principio del siglo xx. La mquina generalizada de Kron permiti profundizar en el anlisis dinmico y transitorio de las mquinas elctricas, lo que transform enormemente la enseanza de esta materia en todas las universidades del mundo. En 1966 se jubil de la General Electric cuando estaba trabajando en el anlisis de redes dimensionales. LEBLANC, Maurice (1857-1923). Ingeniero francs. Se gradu en la Escuela Politcnica (1878). Trabaj en la Compaa de ferrocarriles del Este, haciendo mejoras en las locomotoras elctricas orientado por Marcel Deprez. A partir de 1888 se dedic a sus investigaciones sobre aparatos elctricos: alternadores, transformadores, conmutatrices, cambiadores de frecuencia, etc. Invent, en unin con Hutin, el devanado amortiguador en los alternadores para mejorar su comportamiento transitorio. En 1897 la General Electric le ofreci el puesto de Ingeniero Jefe pero no acept. Ms tarde, en 1901, George Westinghouse obtuvo permiso para explotar sus patentes en EE. UU. A raz de sus contactos con esta empresa fue nombrado Ingeniero Jefe del Consejo de Administracin de la Compaa Westinghouse en Francia. En sus ltimos aos se dedic a la mecnica, para hacer un frigorfico domstico, estudiando compresores y condensadores; tambin dise motores para aviones y propuso utilizar corrientes de alta frecuencia para la propulsin de trenes elctricos. Fue Presidente del Comit Electrotcnico Internacional entre 1912 y 1914. MONTSINGER, Vincent Melanchton (1884-?). Ingeniero americano. Se gradu en la Universidad de Carolina del Norte (1909). Ingres a continuacin en la General Electric, donde fue asignado al departamento de ensayos elctricos. Entre 1912 y 1919 realiz investigaciones en el Laboratorio de Desarrollo. A partir de 1922 se dedic al diseo de transformadores y a los problemas de calentamiento y aislamiento de los devanados. A l se debe la clebre ley de que la vida de un aislante se reduce a la mitad por cada 10 "C de elevacin de temperatura respecto a la mxima admitida. Perteneci a diversos Comits de patrones y normas. Fue uno de los representantes americanos de la Comisin Electrotcnica Internacional entre 1937 y 1944. PARK, Robert H. (1902-1994). Ingeniero elctrico americano. Naci en Estrasburgo (Alemania) el 15de marzo de 1902 (sus padres eran americanos). Muri el 18 de febrero de 1994en Providence, Estados Unidos. Se gradu en el Massachusetts lnstitute ofTechnology en 1923. Ampli estudios en Estocolmo. Su publicacin que le dio a conocer mundialmente fue el artculo publicado en dos partes: Two-Reaction Theory of Synchronous Machines. (Part 1,Generalized Method of Analysis. AlEE Trans., Vol. 48, pgs. 716-730, July, 1929); Part 11,AlEE Trans., Vol. 52, pgs. 352-355, June, 1933). En este trabajo Park, basndose en los ejes d y q de Blondel, desarroll las ecuaciones de las mquinas sncronas que serviran ms tarde a Gabriel Kron para realizar un estudio generalizado y unificado de todas las mquinas elctricas. Irnicamente, recibi un Premio del AlEE en 1931 por otro artculo: Circuit Breaker Recovery Voltages-Magnitudes and Rates of Rise (AlEE Trans., Vol. 50, pgs. 204-238, March, 1931). De cualquier modo, prcticamente estos dos artculos le consagraron mundialmente al principio de su carrera. Fund en 1950 su propia empresa para fabricar botellas de plstico y contribuy mucho a su automatizacin. Premiado en 1972 con la medalla Lamme del IEEE. Estaba en posesin de 17 patentes industriales. ROWLAND, Henry Augustus (1848-1901). Fsico americano. Se gradu en Ingeniera en el Politcnico de Rensselaer, N. Y., en 1870. Estudi en Berln bajo las enseanzas de Helmholz. En 1876 se hizo cargo de la Ctedra de Fsica en la Universidad John Hopkins en Baltimore, puesto que ejerci hasta su muerte. Estudi el campo magntico producido por cuerpos cargados en movimiento. Redetermin el valor del ohmio y el equivalente mecnico del calor. Dise redes de difraccin que tenan grandes ventajas en espectroscopia astronmica. No tuvo gran notoriedad en su pas, pero en palabras de Maxwell, se le puede considerar como uno de los mejores fsicos :::'''''' del siglo xrx. E, ltimos ';;0' estuvo desarrollando un sistema rnltiplex de tele-

'o,

l.
t

9.

SAXTON, Joseph (1799-1873). Inventor americano. Desde pequeo mostr grandes cualidades manuales y un gran talento en la construccin de aparatos de precisin. Fue aprendiz de relojero y en 1824 recibi un premio del Instituto Franklin por la construccin de un reloj de pndulo con

f.

CAPTULO 2.

PRINCIPIOSGENERALESDE LAS MQUINASELCTRICAS

159

compensacin de temperatura. Invent tambin una rueda dentada para relojes de forma epicicloidal y un pirmetro. En 1829 se traslad a Londres, donde expuso instrumental cientfico construido por l mismo; entre los visitantes de la exposicin se encontraban Faraday, Wheatstone y los ingenieros Cubitt y Telford. Saxton construy un contador elctrico para Cubitt y algunos motores elctricos basados en el principio de induccin que acababa de descubrir Faraday. En 1835 volvi a EE. UU., encargndose de la construccin de balanzas de precisin para laboratorios. Desde 1844 fue el Director de la Oficina de Pesos y Medidas americana. 10. WILDE,Henry (1833-1919). Inventor ingls. Mostr desde muy joven grandes dotes manuales. Dise a los veinticinco aos un cdigo telegrfico con su adaptador al transmisor y al receptor. Sus principales.trabajos se refieren a la construccin de mquinas elctricas. En 1863 obtuvo una patente (nm. 3.006 del mes de diciembre) por un generador electromagntico con excitatriz magnetoelctrica. En 1866 present un artculo a la Royal Society titulado A new powerful generator of Dynamic Electricity, en el que demostraba su principio dinamoelctrico. Cre la Compaa Wilde en Manchester, dedicada al desarrollo de sus patentes. Construy en sus talleres, alternadores y dinamos que exportaba a toda Europa. Se dio cuenta de la importancia del acoplamiento en paralelo de alternadores, aunque la teora matemtica fue desarrollada algo ms tarde por J. Hopkinson. Dise lmparas de arco para alumbrado y contribuy con sus mtodos constructivos al desarrollo de las mquinas elctricas. En sus ltimos aos realiz estudios sobre magnetismo terrestre y locomocin area. 11. WOLLASTON, William Hyde (1766-1828). Qumico y fsico ingls. Curs la carrera de Medicina en la Universidad de Cambridge y ejerci de mdico durante siete aos; despus se dedic a la investigacin en Fsica y Qumica. Ide mtodos para trabajar el platino (el metal precioso del siglo XVIII),con lo que hizo una gran fortuna. Descubri el paladio y el rodio. Hizo grandes contribuciones a la qumica y a la mineraloga. Se interes por el experimento de Oersted e intent conseguir el efecto contrario, es decir, obtener una corriente elctrica a partir de un campo magntico. No consigui su objetivo, pero fue su ayudante Faraday quien logr el descubrimiento. Estudi el espectro de la luz y fue uno de los primeros que observ la luz ultravioleta.

REFERENCIAS
1. ADKJNS, B.: General Theory of Electrical Machines. Chapman Hall, London, 1957. 2. CORTES, M.: Curso Moderno de Mquinas Elctricas Rotativas. Editores Tcnicos Asociados, Barcelona, 1973. 3. CHATELAIN, 1.: Trait d'lectricit. Vol. X: Machines elctriques. Editions Georgi Lausanne, 1983. 4. DANIELS, A. R.: Introduction to Electrical Machines. MacMillan Press, London, 1976. 5. DELTORO, V.: Electric Machines and Power Systems. Prentice Hall Inc., Englewood Cliffs, N. J., 1985. 6. EL-HA W ARY,M. E.: Principies of Electric Machines wit Power Electronic Applications. Reston Book, Prentice Hall, Englewood Cliffs, N. r., 1986. 7. FITZGERALD, A. E.; KINGSLEY, CH. Jr.; UMANS,S. D.: Electric Machinery, fourth ed. (S. 1.). McGraw-Hill Book Co., New York, 1985. 8. GIBBS,W. J.: Tensors in Electrical Machine Theory. Chapman Hall, London, 1973. 9. GINGRICH, H. W.: Mquinas Elctricas, Transformadores y Controles. Prentice Hall, Inc., Englewood Cliffs, N. l Edicin en castellano de Ed. Dossat S. A., Madrid, 1980. 10. HERRANZ, G.: Convertidores Electromecnicos de Energa. Marcombo, Boixareu Editores, Barcelona, 1980. 11. IVANOV-SMOLENSKY.: Electrical Machines. Mir Pub., Moscow, 1982. 12. JONES, C. V.: Unified Theory of Electrical Machines. Butterworth, London, 1967. 13. KRON,G.: Generalized Theory of Electrical Machinery. Transactions AlEE, 49, 666 (1930). 14. MCPHERSON, G.: An lntroduction to Electrical Machines and Transformers. John Wiley & Sons, New York, 1981. 15. NAGRATH, I. J.; KOTHARJ, D. P.: Electric Machines. Tata McGraw-Hill Pub, New Delhi, 1985.

160

MQUINASELCTRICAS
PARK,R. H.: Two-Reaction Theory ofSynchronous Machines. Transactions AlEE, 48, 716 (1929). PARKERSMITH,S.: Problemas de Ingeniera Elctrica. Ed. Selecciones Cientficas, Madrid, 1961. SANJURJO, R.: Mquinas Elctricas. McGraw-HilllInteramericana de Espaa S. A., Madrid, 1989. SANZFEITO, J.: Mquinas Elctricas. Prentice Hall, Pearson Education, Madrid, 2002. SEGUIER, G.; NOTELET,F.: Electrotechnique Industrielle. Technique et Documentation, Pars, 1980. SERRANOIRIBARNEGARAY, L.: Fundamentos de Mquinas Elctricas Rotativas. Marcombo, Boixareu Ed., Barcelona, 1989. SISKIND,CH.: Electrical Machines: Direct and Alternating Current. McGraw-Hill Book Co., New York, 1950. STEIN,R.; HUNT, W. T.: Electric Power System Components: Transformers and Rotating Machines. Van Nostrand Reinhold Co., New York, 1979. WHITE, D. c.; WOODSON,H. H.: Electromechanical Energy Conversion. J. Wiley & Sons, New York, 1959.

16. 17. 18. 19. 20. 21. 22. 23. 24.

CAPTULO 3

Transformadores .

3.1. INTRODUCCIN
Como ya se ha indicado brevemente en el captulo anterior, el transformador* es una mquina elctrica esttica, destinada a funcionar con corriente alterna, constituida por dos arrollamientos, primario y secundario, que permite transformar la energa elctrica, con unas magnitudes V-I determinadas, a otras con valores en general diferentes. La importancia de los transformadores se debe a que gracias a ellos ha sido posible el enorme desarrollo en la utilizacin de la energa elctrica, haciendo posible la realizacin prctica y econmica del transporte de la energa a grandes distancias. Tngase en cuenta que la transmisin de la energa elctrica desde los lugares donde se produce (centrales) hasta los numerosos centros de consumo es tanto ms econmica cuanto ms alta es la tensin de las lneas, pues con ello se hace menor la corriente y en consecuencia se reduce la seccin de los conductores. Sin embargo, las tensiones generadas en los alternadores de las centrales elctricas estn limitadas, por consideraciones tecnolgicas, a valores que oscilan entre los 15 y 30 kV, que son insuficientes para alcanzar tal objetivo; por otro lado, salvo casos sumamente raros, la corriente a alta tensin no puede enviarse directamente a los aparatos de utilizacin, porque stos requieren normalmente tensiones ms bajas. Los transformadores permiten conciliar de una forma idnea estas necesidades opuestas, de tal forma que para reducir las prdidas en la lnea se realiza una primera transformacin que eleva la tensin de salida de los alternadores a valoresdel orden de 380-400 kV, a los cuales se realiza el transporte de energa; existiendo en los centros receptores otros transformadores que realizan el proceso inverso, reduciendo la tensinhasta los niveles que se consideren convenientes para la distribucin y consumo de esta energa. El arrollamiento de mayor tensin recibe el nombre de devanado de alta tensin (A.T.) y el de menor tensin se denomina devanado de baja tensin (B.T.). El proceso de transformacintiene un gran rendimiento al no disponer la mquina de rganos mviles, pudindose llegar en los grandes transformadores a valores del orden del 99,7 por 100. Este captulo comienza describiendo los aspectos constructivos de los transformadores, mostrandolas formas bsicas de los ncleos, indicando el tipo de chapa magntica y la forma de su apilamiento; se observa despus la disposicin de los devanados en el ncleo y su aislamiento,analizando luego los sistemas de refrigeracin ms comunes. Se indican los tipos de aceite y sus propiedades dielctricas y refrigerantes, y se deduce la necesidad de un depsito conservador o de expansin en los transformadores, finalizando este epgrafe con la descripcin de los pasatapas empleados, haciendo una breve introduccin a la misin del rel

En el argot elctrico, al transformador se le denomina trafo.

161

162

MQUINAS ELCTRICAS

Buchholz y explicando el significado de los valores asignados que contiene la placa de caractersticas del transformador. Contina el captulo estudiando el principio de funcionamiento del transformador ideal, donde se observan las relaciones bsicas existentes entre las diferentes magnitudes que intervienen, relacionando la corriente de vaco de la mquina con la corriente de excitacin absorbida por una bobina con ncleo de hierro, que se estudi en el Captulo l. Se completa ms tarde el anlisis introduciendo los efectos de resistencia y dispersin de los arrollamientos, indicndose el diagrama vectorial en carga. Se deduce luego el circuito equivalente del transformador y se comentan los ensayos necesarios para su determinacin completa. A partir de este esquema se deduce la cada de tensin mediante el diagrama de Kapp, haciendo especial mencin al efecto Ferranti, y se completa el estudio con el anlisis de las prdidas y el rendimiento de estas mquinas. Ms tarde se analizan los armnicos de la corriente de vaco de un transformador y los efectos que producen cuando circulan estas corrientes por las lneas areas. Se explica la corriente de conexin de los transformadores y los valores transitorios a que dan lugar, que pueden confundirse a veces con corrientes de cortocircuito de la instalacin. Se estudian luego los transformadores trifsicos, haciendo especial hincapi en la determinacin del ngulo horario. Se analizan los armnicos en las corrientes de excitacin de los transformadores trifsicos y la forma de evitarlos. A continuacin se hace una descripcin de las principales conexiones de estos transformadores y sus propiedades. Se incluye despus el acoplamiento en paralelo de transformadores explicando las condiciones que deben cumplir para distribuir la carga de un modo adecuado. Tambin se estudia el autotransformador, indicando sus ventajas e inconvenientes respecto al transformador clsico. Se explican los transformadores con tomas, dando detalles prcticos de la colocacin de los elementos de conmutacin. Se explican despus los transformadores de medida, que son necesarios para poder detectar altas tensiones y altas corrientes, adaptndolas a los calibres normalizados de los instrumentos de medicin como voltmetros, ampermetros, etc., y tambin para aplicar estos transformadores en los sistemas de proteccin de mquinas: rels de mxima, diferenciales, etc. El captulo finaliza con un apndice dedicado a transformaciones especiales; en particular explica la transformacin de trifsica a bifsica o conexin Scott y la transformacin de trifsica a hexafsica utilizada en los equipos rectificadores.

3.2. PRINCIPALES ASPECTOS CONSTRUCTIVOS


El transformador consta de las siguientes partes principales: a) ncleo, b) devanado, e) sistema de refrigeracin y d) aisladores pasantes de salida.

a)

Ncleo

Se denomina ncleo del transformador el sistema que forma su circuito magntico, que est constituido por chapas de acero al silicio, modernamente laminadas en fro (grano orientado), que han sido sometidas a un tratamiento qumico especial denominado comercialmente carlte, que las recubre de una capa aislante muy delgada (0,01 mm), lo que reduce considerablemente las prdidas en el hierro. El circuito magntico est compuesto por las columnas, que son las partes donde se montan los devanados, y las culatas, que son las partes que realizan la unin entre las columnas. Los espacios entre las columnas y las culatas, por los cuales pasan los devanados, se llaman ventanas del ncleo. Segn sea la posicin relativa entre el ncleo y los' devanados, los transformadores se clasifican en acorazados, en los que, como muestra la Figura 3.la, los

CAPTULO

3.

TRANSFORMADORES

163

Devanado de B.T. (interior)

Devanado de A.T. Ncleo magntico .. . (exterior) Devanado de A.T. (exterior) Culatas

Ventana
a)

Devanado de B.T. (interior)

Acorazado

h)

De columnas

Figura 3.1. Circuitos magnticos de transformadores monofsicos.

devanados estn en su mayor parte abrazados o acorazados por el ncleo magntico, y de columnas (Fig. 3.1b), en los que son los devanados los que rodean casi por completo el ncleo magntico. En el tipo acorazado las espiras quedan ms sujetas, pero el tipo de columnas es de construccin ms sencilla y se adapta mejor a las altas tensiones, porque la superficie que ha de aislarse es ms reducida; por ello es el que se utiliza ms generalmente en la prctica (excepto en transformadores monofsicos de baja potencia y tensin). Los circuitos magnticos de la Figura 3.1 corresponden a transformadores monofsicos, y las secciones de las columnas y culatas son iguales para hacer que la induccin sea la misma en todo el circuito magntico; en el caso de la Figura 3.la, la columna central tiene doble superficie que las laterales ya que por ella circula doble flujo que en estas ltimas. Cuando se trata de transformadores trifsicos, el circuito magntico consta de tres columnas idnticas, tal como se muestra en la Figura 3.2. Las uniones de las columnas con las culatas se denominan juntas, y deben tener un espesor lo ms pequeo posible con objeto de reducir al mximo la reluctancia del circuito magntico. La culata superior se tiene que poder abrir para poder colocar las bobinas y los aislantes. Las uniones o juntas pueden realizarse a tope (o plana) o bien al solape (entrelazada). En la construccin a tope (Fig. 3.3a) las columnas y las culatas se montan separadamente
Tres columnas idnticas

Primario y secundario de la l.' fase

2.' fase

3.' fase

Figura 3.2. Circuito magntico y devanados de un transformador trifsico.

164

MQUINAS ELCTRICAS

a) Unionesa tope

b)Uniones al solape

Figura 3.3. Uniones de chapas de transformadores.

y luego se unen con ayuda de piezas de sujecin. En la construccin al solape todo el ncleo magntico se construye de una vez, de tal forma que, como indica la Figura 3.3b, se van ensamblando las chapas con un desfase de posicin entre capas sucesivas (pares e impares) igual a la anchura de las chapas de la culata; este montaje, aunque es ms complicado que el anterior, permite un aumento de la estabilidad mecnica del conjunto. En cualquiera de los dos casos, existe una zona al lado de la junta en la que el flujo no sigue la direccin de laminacin y esto origina, en el caso de chapas de grano orientado, un calentamiento local debido al aumento de prdidas en el hierro; para evitar esto, las uniones, bien sean a tope o al solape, no se realizan a 90 como indica la Figura 3.3, sino a 45. Otro aspecto caracterstico de los ncleos lo muestran las secciones transversales de las columnas, que en los transformadorespequeos se construyende forma cuadrada (Fig. 3.1). Sin embargo, en la mayora de los casos, para obtener un mejor aprovechamiento del rea interior de los devanados (de seccin circular), la seccin transversal de cada rama tiene forma de un polgono escalonado, con un nmero de escalones que es tanto mayor cuanto ms elevada sea la potencia del transformador. Se dice entonces que la seccin es del tipo cruciforme. En la Figura 3.4 se muestran algunos ejemplos tpicos indicando tambin la potencia mxima de utilizacin correspondiente a cada configuracin. En los transformadores de gran
l kVA 10kVA lOOkVA 1.000 kVA

d=
10

Figura 3.4. Ncleos de transformadortipo cruciforme.

CAPTULO 3.

TRANSFORMADORES

165

potencia, para mejorar la evacuacin de calor se intercalan canales de ventilacin entre los paquetes de chapas. El conjunto de las chapas debe ser finalmente apretado por medio de bridas de madera o de perfiles de hierro con la ayuda de bulones aislados; de esta forma se consigue dar rigidez mecnica al conjunto y se evitan vibraciones (vase Fig. 3.2). Nota de diseo: Las dimensiones mostradas en la Figura 3.4 proceden de calcular el rea mxima para un determinado tipo de seccin cruciforme. Por ejemplo, si se considera una seccin con tres escalones, el rea real de hierro vale: S = b2 + 2ae - 2be; las condiciones geomtricas que se deducen de la figura son: d2 = a2 + e2; d' = 2b2, despejando de estas dos ltimas ecuaciones e en funcin de d y a; y b en funcin de d, y sustituyendo en la expresin de la seccin del hierro, resulta:

Para un determinadodimetro d, el valorde la anchura a para conseguirla mxima seccin se obtienederivandoS respectode a e igualandoesta derivadaa cero, lo que da lugar a: a = O,906d; como consecuencia de ello, los valores de b y e se obtienen de las condiciones geomtricas, resultandoser: b = O,707d; e = 0,423d, que son los valores mostradosen la Figura 3.4. El lector puedeextendereste resultadopara cualquierotro nmero de escalonesde la seccincruciformey demostrarde este modo los valores de las dimensiones mostradas en la figura.

b) Devanados
Constituyen el circuito elctrico del transformador; se realizan por medio de conductores de cobre, en forma de hilos redondos (para dimetros inferiores a 4 mm) o de seccin rectangular (pletinas de cobre) cuando se requieren secciones mayores. Los conductores estn recubiertos por una capa aislante, que suele ser de barniz en los pequeos transformadores y que en el caso de pletinas est formada por una o varias capas de fibra de algodn o cinta de papel. Segn sea la disposicin relativa entre los arrollamientos de AT. y B.T., los devanados pueden ser concntricos o alternados. En los devanados concntricos los bobinados tienen forma de cilindros coaxiales (Fig. 3.5a); generalmente se coloca ms cerca de la columna el arrollamiento de B.T., ya que es ms fcil de aislar que el devanado de AT., y entre ambos bobinados se intercala un cilindro aislante de cartn o papel baquelizado. En los devanados alternados (Fig. 3.5b) los arrollamientos se subdividen en secciones o galletas, de tal forma que las partes de los devanados de AT. y B.T. se suceden alternativamentea lo largo de

B.T. A.T. B.T. A.T. B.T.

t\\\\\$i
b) Devanado alternado (en galletas)

;R
;;;

a) Devanado concntrico

Figura 3.5. Devanadosconcntricos y alternados.

166

MQUINAS ELCTRICAS

la columna. Para disminuir el flujo de dispersin, es frecuente que en cada extremo se coloque media bobina, que por razones obvias de aislamiento pertenecen al arrollamiento de B.T.

e) Sistemas de refrigeracin
En un transformador, como en cualquier otro tipo de mquina elctrica, existen una serie de prdidas que se transforman en calor y que contribuyen al calentamiento de la mquina. Para evitar que se consigan altas temperaturas que puedan afectar la vida de los aislamientos de los devanados es preciso dotar al transformador de un sistema de refrigeracin adecuado. Para potencias pequeas, la superficie externa de la mquina es suficiente para lograr la evacuacin de calor necesaria, lo que da lugar a los llamados transformadores en seco. Para potencias elevadas se emplea como medio refrigerante el aceite, resultando los transformadores en bao de aceite. El aceite tiene una doble misin de refrigerante y aislante, ya que posee una capacidad trmica y una rigidez dielctrica superior a la del aire. En estos transformadores, la parte activa se introduce en una cuba de aceite mineral, cuyo aspecto externo puede tener forma plana, ondulada, con tubos o con radiadores adosados, realizndose la eliminacin del calor por radiacin y conveccin natural. El aceite mineral empleado procede de un subproducto de la destilacin fraccionada del petrleo y con el tiempo puede experimentar un proceso de envejecimiento, lo que indica que se oxida y polimeriza formando lodos, proceso que es activado por la temperatura, la humedad y el contacto con el oxgeno del aire; con ello, el aceite presenta una disminucin de sus propiedades refrigerantes y aislantes. Para atenuar este efecto suelen aadirse al aceite productos qumicos inhibidores, y tambin se dota a la cuba de un depsito de expansin o conservador colocado en la parte alta del transformador (Fig. 3.6). La misin de este depsito es doble: por una parte se logra que la cuba principal est totalmente llena de aceite, de tal forma que slo existe una pequea superficie de contacto con el aire en el conservador (la capacidad de este depsito es del orden del 8 por 100 del total); por otra parte, este depsito es el que absorbe las dilataciones del aceite al calentarse. Cuando el transformador se enfra, el aire penetra por l (se dice entonces que el transformador respira), y como el aire arrastra humedad, que es absorbida por el aceite, para evitarlo se coloca a la entrada un desecador de cloruro clcico o un gel de slice. Desde un punto de vista histrico, la utilizacin del aceite mineral en su doble vertiente de aislante y refrigerante hizo posible el desarrollo de transformadores de gran potencia. El aceite mineral tiene, sin embargo, dos inconvenientes graves, a saber: 1) es inflamable, y 2) sus vapores, en ciertas condiciones, forman con el aire mezclas explosivas. Por estos motivos la utilizacin del aceite mineral est prohibida en ciertos locales y ambientes. Hasta 1932 no se haba logrado un sustituto del aceite mineral que fuera til para lus transformadores. En este ao se logr desarrollar un lquido aislante sinttic. (aceite sinttico) conocido con el nombre genrico de askarel, que era en realidad un hidrocarburo aromtico clorado que ofreca grandes ventajas frente a los aceites clsicos de transformadores (hidrocarburos puros), ya que no era ni inflamable ni explosivo. Estos aceites sintticos se han conocido en el mercado con los nombres comerciales de pyranol, pyraleno, inerteen, etc. Desgraciadamente, debido a las dificultades de eliminacin y reduccin del pyraleno, con el consiguiente impacto ecolgico que representa, a partir de la dcada de los ochenta, se ha prohibido su utilizacin en la construccin de nuevos transformadores. Modernamente se ha impulsado el uso de los denominados aceites de siliconas, que representan un nuevo avance tecnolgico para intentar aunar las misiones aislantes y refrigerantes con un reducido impacto ambiental. En la Tabla 3.1 se muestran las caractersticas fundamentales de los principales aceites utilizados en la construccin de transformadores y su comparacin con las del aire.

CAPTULO 3.

TRANSFORMADORES

167

Depsito conservador del aceite

Argolla de elevacin Tapa de cierre de la cuba Conexin de B.T. Bridas de apriete (madera) Conexin de A.T.

Ncleo magntico (cruciforrne) _[)eVilna,:!o de B.T. Bobina aislante (galletas)

Bridas de apriete (madera)

tl::!:j;~--Ruedas de transporte
Cuba principal con aletas de refrigeracin

Figura 3.6.

Aspectos constructivos de un transformador.

Tabla 3.1.

Caractersticas de aceites de transformador

Hidrocarburos puros (aceite de trafo) Pyralenos Aceite de silicio Aire

900

0,16

2,2

200

1.820 960 1.293

0,01 0,15 0,024

4,5 2,56

290 200 a 300 32

168

MQUINAS ELCTRICAS

Los transformadores de distribucin de menos de 200 kVA estn normalmente sumergidos en aceite dentro de la cuba principal de acero. El aceite transmite el calor a la cuba, desde donde se dispersa por conveccin y por radiacin al aire exterior. A medida que la potencia asignada va siendo mayor, se van aadiendo radiadores externos para aumentar la superficie de enfrentamiento de la cuba llena de aceite. El aceite circula alrededor de los devanados hacia los radiadores, en donde el calor es cedido al aire exterior. En el caso de potencias ms elevadas, se insufla aire sobre los radiadores mediante ventiladores adecuados. En transformadores del orden de los MVA se puede refrigerar mediante un intercambiador de calor aceite-agua. El aceite caliente se bombea a travs de un serpentn en contacto con agua fra. Este sistema es muy eficaz pero tambin muy costoso, ya que a su vez debe enfriarse el agua para ponerla otra vez en circulacin. El tipo de refrigeracin de un transformador se designa segn las Normas CEI (Comisin Electrotcnica Internacional) por cuatro letras. Las dos primeras se refieren al tipo de refrigerante en contacto con los arrollamientos y a la naturaleza de su circulacin y las otras dos letras se refieren al refrigerante en contacto con el sistema de refrigeracin exterior y a su modo de circulacin. Los smbolos empleados son los indicados en la Tabla 3.2. Por ejemplo, un transformador en bao de aceite, con circulacin natural por conveccin, que a su vez est refrigerado por aire con movimiento natural, se designar por las letras ONAN. Si el movimiento del aire llega a hacerse con la ayuda de ventiladores se hubiera designado por ONAF. Al inicio de la dcada de 1980 se inici un nuevo sistema de construccin de transformadores secos encapsulados en resina epoxi. Este tipo de transformador es el ms idneo para instalaciones que requieren gran seguridad, fundamentalmente en interiores, locales de pblica concurrencia, hospitales, centros comerciales, ferrocarriles metropolitanos, fbricas de productos combustibles, minas, etc. No propagan el fuego, son autoextinguibles, no se derrama material infamable ni contaminante en caso de avera, como ocurre con el aceite y la silicona. No requieren mantenimiento, no tienen niveles que controlar ni foso colector de aceites y no requieren equipos contra incendios. Todo ello hace que sea el transformador ms seguro y fiable del mercado en la actualidad. Los arrollamientos de alta tensin estn completamente encapsulados en una masa de resina epoxi cargada con silicato de fluor, tratada convenientemente para mejorar la adherencia y la resistencia a la humedad; el conductor es en forma de hilos esmaltados o pletinas recubiertas con papel aislante. Los devanados de baja tensin emplean conductores en forma de pletinas de cobre aisladas con papel; a partir de los 400 kVA se utiliza la tcnica de bobinados en bandas, que consiste en enrollar, sobre un modelo cilndrico, una banda de conductor junto con otra de un aislamiento flexible. La aplicacin de esta tcnica, junto con el empleo de aislamientos preimpregnados, permite obtener unos
Tabla 3.2. Smbolos empleados para sealar la naturaleza del refrigerante y su modo de circulacin

Aceite mineral Pyraleno Gas Agua Aire Aislante slido

L G
W

Natural Forzada

N F

A
S

CAPTULO 3.

TRANSFORMADORES

169

arrollamientos compactos, resistentes a la humedad, de fcil disipacin de calor y muy buen comportamiento a los esfuerzos dinmicos que se producen en caso de cortocircuitos.

d)

Aisladores pasantes y otros elementos

Los bornes de los transformadores de media tensin se llevan al exterior de la cuba mediante unos aisladores pasantes (pasatapas) de porcelana, rellenos de aire o aceite. Cuando se utilizan altas tensiones aparece un fuerte campo elctrico entre el conductor terminal y el borde del orificio en la tapa superior de la cuba, y para evitar la perforacin del aislador, ste se realiza con una serie de cilindros que rodean la borna metlica dentro del espacio cerrado que contiene el aceite. Los pasatapas de A.T. y B.T. en un transformador se distinguen por su altura, siendo tanto ms altos cuanto mayor es la tensin, como se puede observar en la Figura 3.6. Otro elemento que suelen llevar los transformadores de gran potencia es el llamado rel de gas o rel Buchholz (vase Fig. 3.7), que protege a la mquina de sobrecargas peligrosas, fallos de aislamiento, etc. Este rel se coloca en el tubo que une la cuba principal con el depsito de expansin, y funciona por el movimiento del vapor de aceite producido por un calentamiento anmalo del transformador que hace bascular un sistema de dos flotadores: el primero (nm. 1 de la Fig. 3.7) es sensible a las sobrecargas ligeras, y al descender de la posicin mostrada en la figura provoca la activacin de una alarma acstica; el segundo (nm. 2 de la Fig. 3.7) es sensible a las sobrecargas elevadas, que dan lugar a una formacin tumultuosa de gas en la cuba principal, que al empujar al flotador provoca el cierre del circuito de unos rels que controlan el disparo de unos disyuntores de entrada y salida del transformador. En la Figura 3.8 se muestra un esquema detallado de un transformador con sus tres proyecciones principales, donde pueden apreciarse cada uno de los elementos mencionados en este apartado. Se observa que mirando el transformador por la parte de A.T. aparecen las

+
Depsito conservador Alarma acstic Rel Red de A.T.

TRANSFORMADOR

Flotador 2

O O
(J

.... A
----v

\)

Burbujas de gas ._----c>olr"?

o \:/'

CUBA PRINCIPAL

Red de B.T.

Figura 3.7.

Rel Buchholz y esquema elctrico de proteccin.

170

MQUINAS ELCTRICAS

Figura 3.8. Alzado, perfil y planta de un transformador.

letras ABC (de izquierda a derecha) para designar los terminales de A.T. e igualmente para el lado de B.T., pero en este caso las letras van con minscula. El neutro se seala con n o N (B.T. o A.T., respectivamente) y va colocado a la izquierda del terminal a o A. Las potencias comerciales empleadas en los transformadores de distribucin (en kVA) estn relacionadas por el factor 21/3 ~ 1,26 y responden a los valores asignados siguientes:
5

50 500

6,3 63 630

8
80

800

10 100 1.000

12,5 125

16 160

20 200

25 250

31,5 315

40
400

En la Tabla 3.3 se muestra un cuadro de caractersticas tcnicas de transformadores trifsicos de la serie de 24 kV, que incluyen la potencia, grupo de conexin (se ver ms adelante su significado), prdidas, etc. Tambin se sealan las dimensiones principales y el peso total con aceite. Los principales smbolos empleados para representar los transformadores se indican en la Figura 3.9. El smbolo a) suele ser el ms utilizado, yen este caso representa un transformador trifsico de 100 kVA, conexin tringulo-estrella, y relacin 15.000 V/380-220 V (el doble valor secundario indica que la estrella tiene neutro). Cuando el transformador es monofsico las lneas de entrada y salida van cruzadas por dos barras (en vez de las tres que se indican en la Figura 3.9a). Los smbolos b) y e) representan un transformador monofsico de 10 kVA, 50 Hz, relacin 3.000 V/220 V. e)

Placa de caractersticas del transformador

La placa de caractersticas de un transformador es una cartulina metlica serigrafiada que incluye los datos de potencia asignada, tensiones asignadas, frecuencia e impedancia equiva-

CAPITULO

3.

TRANSFORMADORES

171

Tabla 3.3.

Caractersticas tcnicas de transformadores trifsicos

Po (kW) P" (kW)


"" (%)

lo en % t.: Long. (m)


Anch. (m) Altura (m) Peso (kg)

0,'24 0,33 0,40 1,39 1,87 2,20 4 4 4 4,1 4,7 3,3 0,8 0,9 0,9 0,7 0,7 0,8 1,2 1,3 1,4 385 481 570

0,48 2,53 4 3 1,0 0,8 1,4 655

0,58 2,97 4 2,7 1,2 0,8 1,4 731

0,69 3,49 4 2,4 1,3 0,8 1,5 834

0,82 0,98 1,17 1,38 1,64 1,96 2,15 4,10 4,86 5,80 6,89 8,22 10,24 13,3 4 4 4 4 4 6 6 1,8 1,6 2,2 2,1 2 1,9 2 1,4 1,4 1,5 1,5 1,7 1,9 2,0 0,9 0,9 1,0 1,0 1,2 1,3 1,3 1,7 1,5 1,6 1,5 1,5 1,7 1,9 976 1.100 1.422 1.640 \.930 2.267 2.645

lente en tanto por ciento, o cada de tensin relativa de cortocircuito (vase epgrafe 3.6.2). Si el transformador tiene tomas variadoras de tensin, se incluyen asimismo las tensiones de las diferentes derivaciones (vase epgrafe 3.14). Tambin se indica el esquema de conexiones internas, la especificacin del tipo de transformador, clase de refrigeracin, nombre del fabricante, serie, cdigo y en algunos casos referencias sobre las instrucciones de funcionamiento. Para el lector interesado en normativa tcnica es interesante conocer la Instruccin CEI-76, dedicada a transformadores de potencia, o en su caso la UNE 20-101-75. Las tensiones asignadas o nominales son aquellas para las cuales se ha proyectado el transformador y sern los valores base empleados en los ensayos y en la utilizacin del transformador. La potencia asignada siempre se refiere a la potencia aparente y se aplica tanto al devanado primario como al secundario. Para transformadores monofsicos sera igual a:

(a)

15kV

100 kVA

10 kVA

0 W o o
3~~0~~ 50 Hz
10kVA b) e)

000 V

a)

Figura 3.9.

Smbolos empleados para designar un transformador.

172

MQUINAS ELCTRICAS

donde el subndice n indica asignada o nominal y los subndices 1 y 2 se aplican a magnitudes de primario y secundario, respectivamente. Para transformadores trifsicos se escribira en funcin de los valores de lnea:
(b)

La potencia asignada junto con las tensiones asignadas fijan la capacidad de corriente de los devanados del transformador. De la magnitud de la corriente dependen las prdidas en el cobre, las cuales a su vez inciden en el calentamiento de los arrollamientos; el que ello suceda es crtico, ya que un sobrecalentamiento acorta drsticamente la vida de los aislantes. Los transformadores pueden llegar a tener ms de una potencia asignada, segn se utilice o no refrigeracin forzada o dependiendo de la altitud de la zona en la que vaya a trabajar la mquina. Los trminos asignada y plena carga son sinnimos. Conocidas las tensiones asignadas de primario y secundario, y la potencia aparente asignada, las ecuaciones (a) y (b) permiten calcular las corrientes asignadas o de plena carga del transformador para cada uno de los devanados.

3.3. PRINCIPIO DE FUNCIONAMIENTO DE UN TRANSFORMADOR IDEAL


Consideremos el transformador monofsico de la Figura 3.10, constituido por un ncleo magntico real de permeabilidad finita, que presenta unas prdidas en el hierro PFe Yunos arrollamientos primario y secundario con un nmero de espiras NI y N2, respectivamente. Supondremos que el transformador se alimenta por el devanado de tensin ms elevada, es decir, se considera que la mquina va a trabajar como transformador reductor. Los convenios de signos adoptados para las corrientes y tensiones en la Figura 3.10 corresponden al sentido normal de transferencia de la energa, es decir: 1) el primario constituye un receptor respecto a la fuente de alimentacin (la red), lo que significa que este devanado absorbe una corriente y una potencia y desarrolla una f.c.e.m. (fuerza contraelectromotriz); 2) el secundario se comporta como un generador respecto a la carga conectada en sus bornes, suministrando una corriente y una potencia, siendo a su vez el asiento de una f.e.m. inducida.

CARGA ZL L '-P2

Figura 3.10.

Transformador monofsico con ncleo real.

CAPTULO 3.

TRANSFORMADORES

173

Para comprender mejor el funcionamiento del transformador, sin que las imperfecciones reales que tiene la mquina enmascaren los fenmenos fsicos que tienen lugar, vamos a suponer que en un principio se cumplen las condiciones ideales siguientes: a) Los devanados primario y secundario tienen resistencias hmicas despreciables, lo que significa que no hay prdidas por efecto Joule y no existen cadas de tensiones resistivas en el transformador. En el sistema real estas resistencias son de pequeo valor pero no nulas. b) No existen flujos de dispersin, lo que significa que todo el flujo magntico est confinado al ncleo y enlaza ambos devanados primario y secundario. En el transformador real existen pequeas partes delflujo que solamente atraviesan a cada uno de los arrollamientos y que son losflujos de dispersin que completan su circuito a travs del aire. Al aplicar una tensin alterna VI al primario, circular por l una corriente alterna, que producir a su vez un flujo alterno en el ncleo cuyo sentido vendr determinado por la ley de Ampre aplicada a este arrollamiento. En la Figura 3.10 se muestran los sentidos positivos de la corriente y el flujo para el instante definido por la polaridad de la tensin aplicada. Debido a la variacin peridica de este flujo se crearn f.e.m.s. inducidas en los arrollamientos, que de acuerdo con la ley de Faraday respondern a las ecuaciones: (3.1) Estas f.e.m.s. tienen las polaridades sealadas en la Figura 3.10 para que estn de acuerdo con la ley de Lenz, de oposicin al cambio de flujo. Realmente el representa una f.c.e.m. porque se opone a la tensin aplicada VI y limita de hecho la corriente de primario. La polaridad asignada a e2 en la Figura 3.10 tiene en cuenta que al cerrar el interruptor S del secundario se tendera a producir una corriente i2 en el sentido mostrado en la figura, de tal modo que al circular por el devanado secundario dara lugar (aplicar la ley de Ampre a este arrollamiento) a una accin antagonista sobre el flujo primario como as lo requiere la ley de Lenz. De ah la no inclusin del signo menos en las expresiones (3.1), porque ya se ha tenido en cuenta al sealar las polaridades de las f.e.m.s. en la Figura 3.10. En definitiva, de acuerdo con este convenio de signos laf.m.m. del secundario acta en contra de laf.m.m. primaria produciendo un efecto desmagnetizante sobre sta. Se observa en la Figura 3.10 que los terminales superiores de los devanados primario y secundario tienen en el instante indicado una polaridad positiva respecto de los otros. Para destacar este hecho, en la teora de circuitos con acoplamientos magnticos se suelen sealar con un punto aquellos terminales de las bobinas que tienen simultneamente la misma polaridad instantnea, de ah la justificacin de haber dibujado un punto en los terminales superiores de los devanados del transformador de la Figura 3.10. Existe un modo ms inmediato de identificar estos bornes homlogos, y es considerar primeramente un sentido de flujo positivo en el ncleo y sealar a continuacin con un punto aquellos extremos de los arrollamientos por los que hay que introducir corriente para obtener flujos de sentido positivo. Obsrvese en el caso de la Figura 3.10 que si se considera un flujo positivo con sentido dextrgiro, habr que introducir corriente por los terminales superiores para que, teniendo en cuenta la ley de Ampre, se originen flujos de sentido positivo, y de ah una nueva justificacin de que los terminales superiores son homlogos y que por ello se han sealado con un punto. Existe una Recomendacin de las Normas CEI (Comit Electrotcnico Internacional), y que tambin se

174

MQUINAS ELCTRICAS

ha aplicado al esquema de la Figura 3.10, por la que se deben designar los terminales de la misma polaridad con la misma letra, mayscula para el lado de A.T. Yminscula para el lado de B.T., los extremos positivos en la forma A-a y los negativos en la forma N-a' ( si el transformador es trifsico se emplean las letras B y C para las otras dos fases); en la Figura 3.10 puede comprobar el lector esta doble simbologa de letras y puntos. La ventaja de estos convenios es la de conocer las polaridades de los devanados sin necesidad de tener en cuenta los sentidos de los arrollamientos en el ncleo del transformador. Una vez designados los sentidos de las f.e.m.s. y de las corrientes en el transformador, interesa conocer las relaciones existentes entre las tensiones, los flujos y las f.e.m.s, Como quiera que los devanados son ideales, la aplicacin del 2. lema de Kirchhoff a los circuitos primario y secundario de la Figura 3.10 nos da:
e., = v, =

No-

dcD - dt

(3.2)

Si se parte de un flujo senoidal de la forma: (3.3) teniendo en cuenta (3.1) y (3.2) se cumplir:
(3.4)

lo que indica que las tensiones y f.e.m.s. van adelantadas 90 respecto al flujo, siendo sus valores eficaces:

(3.5)

Dividiendo entre s las ecuaciones anteriores resulta:


-=-=-=m

VI

El

NI

V2

E2

Ne

(3.6)

donde el factor m se denomina relacin de transformacin. De este modo, en un transformador ideal la relacin de tensiones coincide con la relacin de espiras, que es en definitiva la relacin de transformacin. Si el interruptor S de la Figura 3.10 est abierto, el transformador funciona sin carga o en rgimen de vaco. El primario se comportar como una bobina con ncleo de hierro, cuyo estudio ya se realiz en el epgrafe 1.6.2.En este caso el transformador absorber una corriente de vaco il) anloga a la corriente iere analizada en aquel apartado. La corriente io forma un ngulo ({JI) con la tensin aplicada VI' de tal forma que la potencia absorbida en vaco, denominada Po' ser igual a las prdidas en el hierro Pp,. en el ncleo del transformador, cumplindose una identidad anloga a (1.74):

(3.7)
donde VI e lo representan los valores eficaces de la tensin y la corriente, respectivamente. La corriente lo tiene dos componente, una activa Ir" Yotra reactiva 1". En la FIgura 3.11 se muestra el diagrama fasorial de un transformador ideal en vaco, donde se muestran las mag-

CAPTULO

3.

TRANSFORMADORES

175

Figura 3.11.

Diagrama fasorial de tensiones y corrientes en vaco.

nitudes anteriores con sus fases correspondientes, habindose tomado como referencia de fases la tensin aplicada VI' Cuando se cierra el interruptor S (Figura 3.10), el transformador funciona en carga y aparece una corriente iz que circula por el circuito secundario, que responde.a un valor complejo o fasorial:
(3.8)

es decir, Iz se retrasa ({Jz de la f.e.m. Ez' La corriente iz al circular por el devanado secundario produce una f.m.m. desmagnetizante Nz iz que se opone a la f.m.m. primaria existente NI io' Es por ello que si esta f.m.m. de secundario no queda neutralizada por una corriente adicional que circule por el primario, el flujo en el ncleo se ver reducido profundamente, con las consiguientes reducciones en las fe. m.s. el Y ez que son proporcionales a l y se romper el equilibrio entre VI y el en el primario (ecuacin 3.2). Para que pueda restablecerse el equilibrio es preciso neutralizar la f.m.m. Nz i2 del secundario, mediante una corriente adicional primaria i~ equivalente a una f.m.m. NI i~ de valor:
(3.9)

de donde se deduce el valor de la corriente i~ adicional primaria: (3.10) De este modo, y como se indica en la Figura 3.10, la corriente total necesaria en el primario il ser igual a:
.
II

.
lo

., .
12

lo

iz +m

(3.11)

que corresponde en forma fasorial a: (3.12)

176

MQUINAS ELCTRICAS

donde se ha denominado I~a l/m. La ecuacin anterior expresa la relacin entre la corriente primaria 11, de vaco lo y secundaria 12 Esta ecuacin (3.12) o su equivalente en forma instantnea (3.11) nos indica que la corriente primaria tiene dos componentes. 1. Una corriente de excitacin o de vaco lo cuya misin es producir el flujo en el ncleo magntico y vencer las prdidas en el hierro a travs de sus componentes Ifl e 1 Fe' respectivamente. 2. Una componente de carga I~que equilibra o contrarresta la accin desmagnetizante de la f.m.m. secundaria para que el flujo en el ncleo permanezca constante e independiente de la carga, como as lo requiere la ecuacin (3.5). Un modo ms simple de demostrar la ecuacin (3.12) es proceder en sentido inverso. As, la primera ecuacin (3.5) nos indica que si la tensin primaria VI es constante deber permanecer constante el flujo <Dm en el ncleo magntico, para cualquier rgimen de carga. Si se denomina of a la reluctancia del circuito magntico del transformador, la ley de Hopkinson (1.17) nos indica que si el flujo es constante, tambin deber ser constante la f.m.m. necesaria para producirlo en cualquier rgimen de carga. En particular debern ser iguales las f.m.m.s. en vaco y en carga. En vaco, como las corrientes que circulan por los devanados son II = lo e 12 = O,resultar una f.m.m. total:
(3.13)

mientras que en carga, cuando las corrientes de circulacin son II e 12, se tiene una f.m.m. resultante: (3.14) El signo menos de la expresin anterior est de acuerdo con la accin desmagnetizante del secundario y que puede comprobar el lector aplicando la teora de los circuitos magnticos al esquema de la Figura 3.10. Al igualar (3.13) y (3.14) resulta: (3.15) de donde se deduce: (3.16) que coincide con la expresin ya demostrada (3.12). A plena carga la corriente I~ es de ordinario veinte veces por lo menos mayor que lo, por lo que puede despreciarse en la expresin (3.16) la corriente de vaco frente a I~,que se denomina corriente secundaria reducida, yen consecuencia la ecuacin (3.16) se transforma en la expresin aproximada: (3.17) cuya cantidad ya se estableci en la ecuacin (2.146) del captulo anterior, considerando un transformador ideal sin prdidas en el ncleo.

CAPTuLO 3.

TRANSFORMADORES

177

EJEMPLO

DE APLICACIN

3.1

El transformador ideal de la Figura 3.10 tiene dos devanados con NI = 300 espiras y N2 = 100 espiras. La longitud de la trayectoria magntica media es de 50 cm y la seccin transversal del ncleo magntico es de JO cm', La curva de imanacin del material responde a la ecuacin: B =

1,8.10-2 H B: Teslas H: Av/m 1+ 10-2 H '

Al aplicar al primario una tensin VI = 150 cos 314 t voltios se comprueba que las prdidas en el ncleo son de 20 W. Determinar: a) Corriente de vaco lo absorbida por el transformador. b) Tensin secundaria V2. c) Si el secundario alimenta una impedancia de carga ZL = 0,5 L 60 .Q, determinar la corriente secundaria 12 y la corriente primaria que absorber el transformador de la red. Sugerencia: En el ejemplo de aplicacin 1.7 est resuelta la primera parte del problema.

SOLUCIN
a)

La tensin aplicada tiene una tensin eficaz y una frecuencia de valores:


VI

= r:. =
v2
VI

150

106,06 V ;

f = - = 50 Hz
2n

De este modo, teniendo en cuenta (3.5) resulta un flujo mximo en el ncleo:


m. w
m

4,44 fNI

106,06 4,44' 50 300

1,59' 10- Wb

que corresponde a una induccin mxima: B

= -S = m

<l> m

1,59 . 10-3 10 10-4

1,59 Teslas

lo que requiere una intensidad del campo magntico, teniendo en cuenta la curva de imanacin del material: B

= m

159
'

= 1+ '

18 10-2 H m 10-2 Hm

=>

Hm

= 757 Av/m

Suponiendo que la corriente de imanacin I~sea de forma senoidal, tambin lo ser la intensidad del campo magntico Hm que ella produce, por lo que se tendr un valor eficaz de campo:
H=

j2

e. = 757 j2 = 535,28 Av/m


535,28' 0,5
300

y como H = NI/e, dara lugar a una corriente de imanacin: 1=


J1

::::;09A
'

178

MQUINAS ELCTRICAS

La componente [Fe de la corriente de vaco puede determinarse de acuerdo con (1.74) por la ecuacin:

y por consiguiente:
[Fe

= --

20 = O,19 A 106,06

Si se toma VI como referencia de fases, la corriente de vaco, de acuerdo con el diagrama fasorial de la Figura 3.11, tendr la siguiente expresin compleja:
lo = 0,19 - jO,9 = 0,92 L -78,08 A b)

que corresponde a un mdulo de 0,92 A. De acuerdo con (3.6) se tiene:

=
e)

106,06
V2

300
100

de donde se deduce un mdulo de V2 igual a 35,35 V. La tensin secundaria calculada en el apartado anterior est en fase con la tensin primaria (vase Figura 3.11), por lo que la expresin compleja ser:
V2 = 35,35 LO V

De este modo la corriente 12 valdr, segn (3.8):


12 =

35,35 LO 0,5 L 60

= 70,7 L -60 A

que corresponde a un mdulo de la corriente secundaria de 70,7 A. La corriente primaria correspondiente se obtendr aplicando (3.12), donde m = 3001100 = 3, resultando:
I1 = lo + ~ = 0,92 L -78,08 +

70,7 L -60 3

= 24,44 L -60,68 A

Obsrvese que la expresin (3.17) nos da un valor aproximado muy cercano al real:
I1 I
~ ~

= 23,57 L -60 A

3.4. FUNCIONAMIENTO DE UN TRANSFORMADOR REAL


En el epgrafe anterior se ha realizado el estudio de un transformador ideal en el que los arrollamientos no tenan resistencia ni flujos de dispersin. En los transformadores reales hay que tener en cuenta ambas cualidades. La aparicin de resistencia es inherente a la constitucin de los devanados con hilo conductor. En la Figura 3.12 se muestra el circuito del transformadorde la Figura 3.10, donde para mayor claridad se han considerado las resistencias RI y R2 de los arrollamientos, fuera de las propias bobinas. Se observa tambin en el transformador real que de todo el flujo producido por los devanados slo existe una parte comn a ambos y

CAPTULO 3.

TRANSFORMADORES

179

Figura 3.12.

Transformador real con resistencias elctricas y flujos de dispersin.

representada por 11> en la Figura 3.12; lo anterior es consecuencia de los flujos de dispersin que aparecen en los arrollamientos y que se distribuyen por caminos no magnticos, en particular por los conductores y el aire que rodea las bobinas. Si se denominan 11>1 y 11>2 a los flujos totales que atraviesan los devanados primario y secundario y I1>di' I1>d2 a los flujos de dispersin respectivos se cumplir: (3.18) A primera vista, la introduccin de los flujos de dispersin complica nuestro estudio, ya que desaparece la idea del flujo comn nico que exista en el transformador ideal. Sin embargo, se puede conservar tal forma de proceder si se aaden en serie a cada arrollamiento unas bobinas con el mismo nmero de espiras que los devanados correspondientes, de tal modo que al circular por ellas las intensidades respectivas den lugar a los mismos flujos de dispersin I1>dl y I1>d2 que en los bobinados reales. En la Figura 3.13 se ha representado esta idea, donde se han indicado con Ldl y Ld2 los coeficientes de autoinduccin respectivos de estas bobinas adicionales (con ncleo de aire), cuyos valores de acuerdo con su definicin sern: (3.19) y que dan lugar a las reactancias de dispersin XI y X2 de ambos devanados: (3.20)

RI

CARGA NI
ZL L (/)2

Figura 3.13. Transformador real con bobinas ideales en el ncleo.

180

MQUINAS ELCTRICAS

La aplicacin del 2. lema de Kirchhoff a los circuitos primario y secundario de la Figura 3.13 nos da: (3.21) donde los valores de e, y e2 vienen expresados por las ecuaciones (3.1): (3.22) que corresponden a los valores eficaces segn (3.5): (3.23) donde <l> es el flujo comn mximo que circula por el circuito magntico de la Figura 3.13. Las ecuaciones (3.21) se expresan en forma compleja:
ni

(3.24) Si se tiene en cuenta (3.23), la relacin entre los valores eficaces de las f.e.m.s. inducidas ser:
-=-=m

E,

N,

E2

N2

(3.25)

ecuacin que se cumple siempre tanto en el transformador ideal (vase ecuacin 3.6) como en el transformador real que aqu se estudia. Ahora bien, si se tiene en cuenta (3.21) o (3.24), en el transformador real dejan de cumplirse las igualdades entre f.e.m.s. y tensiones que aparecan en el transformador ideal (vase ecuaciones 3.2), por lo que en el caso real el cociente entre las tensiones primaria y secundaria deja de ser igual a la relacin de transformacin (vase ecuacin 3.6). En los transformadores que usa la industria, las cadas de tensin a plena carga son del orden del 1 al 1 por 100 de las tensiones asignadas, por lo que las relaciones (3.24) se convierten en las ecuaciones aproximadas:

(3.26) y de ah que la relacin entre las tensiones primaria y secundaria ser aproximadamente igual a:
-:::::;m

V,

V2

(3.27)

Si el transformador trabaja en vaco, las relaciones reales (3.24) se transforman en: V, =E, +R,Io+}X,Io ; V2=E2 (3.28)

ya que 12 es igual a cero. Como quiera que en la prctica la corriente de vaco lo es del orden de 0,6 a 8 por 100 de I'n (corriente asignada o de plena carga del primario), las cadas de tensin en vaco definidas por RJo YX,/o en la primera ecuacin (3.28) son muy pequeas (del orden de 0,004 a 0,06 por 100 de V,), y de ah que en vaco se puedan considerar como suficientemente exactas las igualdades: (3.29)

CAPTULO 3.

TRANSFORMADORES

181

donde V20 representa la magnitud de la tensin secundaria en vaco. Por consiguiente, y teniendo en cuenta (3.25) y (3.29), se podr escribir:
El VI NI -=-=-=m E2 V20 N2

(3.30)

que nos define la relacin de transformacin como el cociente entre la tensin primaria aplicada al transformador y la tensin secundaria en vaco. Este cociente es el que incluye el fabricante en la placa de caractersticas de la mquina. En el funcionamiento en carga la primera relacin aproximada (3.26) y la primera ecuacin (3.23) nos indica que los flujos magnticos en vaco y en carga son prcticamente iguales, lo que significa que las f.m.m.s. en ambos estados de carga coinciden, por lo que la ecuacin (3.16), que relaciona las corrientes del transformador, se puede considerar vlida a todos los efectos y que se vuelve a escribir a continuacin: (3.31)

Las ecuaciones (3.24) y (3.31) definen el comportamiento elctrico del transformador en carga. Las ecuaciones (3.24) relacionan las tensiones con las f.e.m.s. y cadas de tensin dentro de los devanados del transformador, mientras que (3.31) nos indica la relacin entre las corrientes primaria, secundaria y de vaco. EJEMPLO DE APLICACIN 3.2
Un transformador monofsico de 10 kVA, relacin 500/100 V, tiene las siguientes impedancias de los devanados: Z, = R, + j X = 0,2 + j 0,4 Q; Z2 = R2 + j X2 = 0,008 + j 0,016 Q. Al alimentar el transformador por una tensin de 500 V que se toma como referencia de fases, la corriente de vaco absorbida responde a la forma compleja: lo = 0,2 L _700 A. Calcular: a) Valores de E; E2 y V2 cuando el transformador trabaja en vaco. b) Si el secundario lleva una corriente de la forma 12 = 100 L - 300, calcular los nuevos valores de El' E2 y V2

SOLUCIN
a)

En vaco se cumplen las relaciones (3.28). Teniendo en cuenta adems que: VI = 500 L 0 resultar: EJ = 500 LO - (0,2 + jO,4) 0,2 L -70 que corresponde a una f.e.m. E2 segn (3.30): E2
0 0 ;

m = 500/100 = 5

= 499,91 L 0,0011

= -EJ = 99,98 L
m

0,0011

y teniendo en cuenta la 2: ecuacin (3.28) se tiene: V2 = V20 = E2 = 99,98 L 0,0011

182

MQUINAS ELCTRICAS

b) Cuando el transformador funciona con carga, la corriente primaria vendr expresada por la ecuacin (3.31): 12 100 L -30 1I = lo + - = 0,2 L -700 + m 5 y teniendo en cuenta (3.24) resulta:
El = 500 L 0 - (0,2 + JOA) . 20;15 L -30,37 = 492,47 L 0,57

20,15 L -30,37

lo que corresponde, segn (3.25), a una f.e.m. E2:


El = 98,49 L 0,57 E2 = m

y a una tensin secundaria V2: V2 = 98,49 L 0,57 - (0,008 + jO,OI6) . 100 L -30 ~ 97 LO

3.5.

CIRCUITO EQUIVALENTE DE UN TRANSFORMADOR

Los problemas relacionados con las tensiones y las corrientes en los transformadores se pueden resolver con gran precisin por medio del clculo complejo, que es un mtodo analtico que ha sustituido hoy en da (gracias a las excelentes calculadoras electrnicas de bolsillo de que se dispone actualmente en el mercado) a los complejos diagramas vectoriales que se fueron utilizando hasta la dcada de los setenta. Aun as, el clculo resulta ser algo laborioso, por lo que suele recurrirse a la sustitucin del transformador por un circuito equivalente simplificado que incorpore todos los fenmenos fsicos que se producen en la mquina real. El desarrollo de circuitos equivalentes de las mquinas elctricas no es una invencin moderna, estando ligado a la propia evolucin y expansin de la ingeniera elctrica. Hay que reconocer, sin embargo, la excelente labor realizada por Steinmetz, quien a principios del siglo xx asent las bases cientficas de la tecnologa elctrica. La ventaja de desarrollar circuitos equivalentes de mquinas elctricas es poder aplicar todo el potencial de la teora de redes elctricas para conocer con antelacin la respuesta de una mquina en unas determinadas condiciones de funcionamiento. En el caso del transformador el desarrollo de un circuito equivalente se inicia reduciendo ambos devanados al mismo nmero de espiras. Generalmente se reduce el secundario al primario, lo que quiere decir que se sustituye el transformador original por otro que tiene el mismo primario con NI espiras y un nuevo secundario con un nmero de espiras N~ igual a NI' Para que este nuevo transformador sea equivalente al original, deben conservarse las condiciones energticas de la mquina, es decir, las potencias activa y reactiva y su distribucin entre los diversos elementos del circuito secundario. Todas las magnitudes relativas a este nuevo devanado se indican con los mismos smbolos del transformador real pero afectados con una tilde, como indica la Figura 3.14, donde los valores de tensiones y corrientes se expresan en forma compleja.

CAPTULO 3.

TRANSFORMADORES

183

Figura 3.14.

Circuito equivalente de un transformador real reducido al primario.

De acuerdo con el principio de igualdad de potencias, prdidas, etc., se obtienen las siguientes relaciones entre las magnitudes secundarias de los transformadores real y equivalente:

a) F.e.m.s.y tensiones
En el transformador real se cumple:
El NI -=-=m E2 N2 y en el transformador equivalente, al ser N~ = NI' se tiene: El NI - =- = 1 E~ N~
=>

(3.32)

E~ = El = mE2

(3.33)

Es decir, la f.e.m. E; del nuevo secundario es m veces mayor que laf.e.m. E2 que exista en el transformador real. De forma anloga se tendr para la tensin V;: (3.34) b)

Corrientes
S2 = V2/2 = V;/;

La conservacin de la potencia aparente de ambos secundarios indica que: (3.35)

y teniendo en cuenta la relacin (3.34) se obtiene: / ' -~ /2-

(3.36)

Es decir, la corriente 1; del nuevo secundario es m veces menor que la corriente 1} que exista en el transformador real. e)

Impedancias
RJi = R~/~2 (3.37)

Al igualar las potencias activas que se disipan en las resistencias, se obtiene:

de donde se deduce, teniendo en cuenta (3.3Q): R~ = m2R2 (3.38)

184

MQUINAS ELCTRICAS

Es decir, la resistencia R; del nuevo secundario es m2 veces la resistencia R2 que existia en el transformador real. De forma similar, planteando la conservacin de la potencia reactiva en las reactancias, resulta: (3.39) y por consiguiente:

x~= m2 X2

(3.40)

Es decir, la reactancia X; del nuevo secundario es ';'2 veces la reactancia Xl que exista en el transformador real. En general, cualquier impedancia conectada en el secundario del transformador real, por ejemplo la impedancia de carga ZL L (/J2 en la red de la Figura 3.13, se reducir al primario siguiendo las relaciones (3.38) o (3.40), por lo que se convertir en una impedancia Z de valor (vase Fig. 3.14): (3.41) lo que indica que cualquier impedancia Z L conectada en el secundario del transformador se convierte en un valor m2ZL en el transformador equivalente. Tngase en cuenta, para demostrar la igualdad anterior de un modo general, que en la Figura 3.13 del transformador real se cumple:
ZL=-

V2
12

(3.42)

siendo la impedancia reducida o transferida al primario en el circuito equivalente de la Figura 3.14:


Z'-~
L -

V'

l'
2

(3.43)

y sustituyendo (3.34), (3.36) y (3.42) en (3.43) resultar:


ZL=-=--=--=m

V~ mV2 I~ 12 m

m2V2 12

ZL

(3.44)

que coincide con la relacin (3.41), obtenida all por analoga con las equivalencias de (3.38)
y (3.40).

Es innegable que observando el circuito equivalente inicial obtenido en la Figura 3.14, el lectorpuede imaginarmultitudde redes equivalentescon tal de imponerdiferentescondicionesal nmero de espiras elegido N~del nuevo transformador.La importanciafundamentalde la reduccin de los devanadosal haber elegido la igualdadespecial N~= NI estriba en que se puede llegar a obtener una representacindel transformadoren la que no exista la funcin transformacin,o dicho en otros trminos, se va a sustituirel transformadorreal, cuyos devanadosestn acoplados magnticamente,por un circuito cuyos elementos estn acoplados slo elctricamente. En efecto, si se observa el circuito de la Figura 3.14 y se tiene en cuenta la igualdad (3.33), existe una identidad entre las f.e.m.s. primaria y secundaria, lo cual permite reunir los extremos de igual polaridad instantnea, sustituyendo ambos devanados por uno' solo como muestra la Figura 3.15. Por este arrollamiento nico circular una corriente diferencia: 11- I~,

CAPTULO 3.

TRANSFORMADORES

185

+
Vi

=:

VI

Figura 3.15.

Circuito equivalente de un transformador real reducido al primario.

que teniendo en cuenta las identidades (3.16) Y(3.31) es igual a la corriente de vaco lo. sta a su vez, tiene dos componentes, una activa [Fe y otra reactiva Il' y como ya se demostr en el epgrafe 1.6.2 del Captulo 1, representan un circuito paralelo formado por una resistencia RFe, cuyas prdidas por efecto Joule indican las prdidas en el hierro del transformador y por una reactancia XIl por la que se deriva la corriente de magnetizacin de l mquina (vase Fig. 1.25 del Captulo 1). De acuerdo con estos razonamientos, el circuito de la Figura 3.15 se transforma en el de la Figura 3.16, lo que representa el denominado circuito equivalente exacto del transformador reducido al primario. El mismo proceso seguido hasta aqu para obtener el circuito equivalente del transformador reducido al primario se puede emplear en sentido inverso, es decir, tomando un primario con un nmero de espiras N; = Nz Ydejando inalterado el secundario; se obtiene as el llamado circuito equivalente reducido al secundario cuyo esquema se indica en la Figura 3.17, y cuya comprobacin de magnitudes se deja como ejercicio al lector. En este texto se utilizar normalmente el tipo de circuito obtenido en primer lugar y mostrado en la Figura 3.16. Este circuito responde fielmente al comportamiento del transformador real y por ello se denomina circuito equivalente exacto. En la prctica, y debido al reducido valor de lo frente a las corrientes II e 12, se suele trabajar con un circuito equivalente aproximado que se obtiene trasladando la rama en paralelo por la que se deriva la corriente de vaco a los bornes de entrada del primario, resultando el esquema de la Figura 3.18a. Con este circuito no se introducen errores apreciables en el clculo y sin embargo se simplifica enormemente el estudio de la mquina. El esquema puede simplificarse an

RI

XI

/1

/2

Figura 3.16.

Circuito equivalente exacto de un transformador real reducido al primario.

186

MQUINAS ELCTRICAS

R,-! I _ m2
+
, VI

x,-!:L I _ m2

X2

R2 12

+
V2

=m

VI

=--;;?

XI'

Figura 3.17.

Circuito equivalente exacto de un transformador real reducido al secundario.

ms observando la conexin en serie constituida por las ramas primaria y secundaria (reducida). Si se denomina:
Rec = RI

+ R~ : Resistencia de cortocircuito
(3.45)

Xce = XI + X~ : Reactancia de cortocircuito

el circuito de la Figura 3.18a se convierte en el de la Figura 3.18b. Con ayuda de este ltimo circuito equivalente simplificado pueden resolverse una serie de problemas prcticos que afectan a la utilizacin del transformador; en particular para el clculo de la cada de tensin y el rendimiento. Inclusive, si en un problema real se requiere nicamente la determinacin de la cada de tensin del transformador, se puede prescindir de la rama paralelo, ya que no afecta esencialmente al clculo de aqulla; de este modo el circuito resultante ser la impedancia serie: Rcc + j Xcc' Como quiera, adems, que en los grandes transformadores se cumple que Xcc es varias veces Ree, se puede utilizar solamente la reactancia serie Xcc para representar el circuito equivalente del transformador. Este esquema final es el que se utiliza cuando se realizan estudios de grandes redes en sistemas elctricos de potencia: anlisis de estabilidad, cortocircuitos, etc.

II

X +
VI

Rcc

x.;
-----7 12

o------__ --~vvv---roaIO~ -O

+
VI

lo

+
v

XI'

o------e-----------------------------o
a)

o------e-----------------~
b)

Figura 3.18. Circuito equivalente aproximado de un transformador reducido al primario.

CAPTULO 3.

TRANSFORMADORES

187

EJEMPLO DE APLICACIN 3.3 A partir de las ecuaciones (3.24), (3.25) y (3.3 ), que definen el comportamiento de un transformador real, deducir de un modo analtico el circuito equivalente exacto de la Figura 3.16. SOLUCiN Las ecuaciones de partida son:

Si en la ecuacin 2) se multiplica por la relacin de transformacin m resulta: mV2 = mE2 o en forma equivalente:
-

mR212 - j mXzl2

y denominando:
E~ = mE2
;

V~ = m V 2

la ecuacin 2/) se convierte en:

lo que da lugar a las ecuaciones transformadas siguientes:


a) VI

= El

+ RI I1 + j XI I1

b) V~ = E' - R21~ - j X~I~ ; c) El

= E~

; d) I1

= lo + I~

que el lector puede comprobar fcilmente que son las ecuaciones que rigen el comportamiento elctrico del circuito de la Figura 3.16.

3.6. ENSAVOS DEL TRANSFORMADOR


Los ensayos de un transformador representan las diversas pruebas que deben prepararse para verificar el comportamiento de la mquina. En la prctica resulta difcil la realizacin de ensayos reales directos por dos motivos esenciales: 1) la gran cantidad de energa que ha de disiparse en tales pruebas, 2) es prcticamente imposible disponer de cargas lo suficientemente elevadas (sobre todo cuando la potencia del transformador es grande) para hacer un ensayo en situaciones reales. Ahora bien, el comportamiento de un transformador, bajo cualquier condicin de trabajo, puede predecirse con suficiente exactitud si se conocen los parmetros del circuito equivalente. Tanto el fabricante como el usuario del transformador necesitan esta informacin. Sin embargo, no es sencillo ni fiable obtener estos parmetros de los datos de diseo o proyecto. Afortunadamente, los elementos que intervienen en el circuito equivalente aproximado se pueden obtener con unos ensayos muy simples que tienen adems la ventaja de requerir muy poco consumo de energa (la suficiente para suministrar nicamente las prdidas de la mquina), de ah que sean pruebas sin carga real. Los dos ensayos fundamentales que se utilizan en

188

MQUINAS ELCTRICAS

la prctica para la determinacin de los parmetros del circuito equivalente de un transformador son: a) Ensayo de vaco y b) Ensayo de cortocircuito. Antes de proceder a la descripcin de estos ensayos vamos a indicar aqu un mtodo para determinar la polaridad de los terminales de un transformador, que le ayudar al lector a comprender el convenio de punto utilizado en la explicacin del epgrafe 3.3 y le servir ms adelante para entender los ndices horarios de los transformadores trifsicos y la conexin en paralelo de estas mquinas. Consideremos el esquema simplificado del transformador mostrado en la Figura 3.19, en el que se desconoce el sentido del arrollamiento de los devanados. Se trata de determinar la polaridad relativa de los terminales secundarios respecto del primario. El proceso a seguir es el siguiente: se sealan previamente los terminales del primario con las letras A y A'. Se unen a continuacin dos terminales cualesquiera de primario y secundario, en el caso que nos ocupa se han unido A con x. Se conectan tres voltmetros de c.a. en la forma que se seala. Al alimentar el primario con una tensin alterna se leen los valores sealados por los tres voltmetros VI' V2 y V)" Si la tensin V3 es igual a VI - V2 significa entonces que el terminal x es el homlogo a A, por lo que deber identificarse finalmente este borne con la letra a de acuerdo con lo indicado en el epgrafe 3.3; el otro terminal y corresponder entonces a a'. De un modo equivalente con el convenio de punto: se deber sealar con un punto el terminal a que es el homlogo de <<A. Si la tensin V3 es igual a VI + V2, entonces el terminal y es ahora el homlogo de A y, por tanto, el r es el homlogo de A'. Con este simple procedimiento se puede realizar el ensayo de polaridad; en la prctica el devanado de alta tensin puede conectarse a una red alterna de 220 V, 50 Hz, aun cuando la tensin asignada de este devanado sea de centenares de voltios.

3.6.1. Ensayo de vaco


Consiste esta prueba en aplicar al primario del transformador la tensin asignada, estando el secundario en circuito abierto. Al mismo tiempo debe medirse la potencia absorbida Po' la corriente de vaco lo Y la tensin secundaria, de acuerdo con el esquema de conexiones de la Figura 3.20*.

+
Red

Figura 3.19. Disposicin de los aparatosde medidaparadeterminarla polaridad. * El ensayo de vaco se indica por didctica que se realiza alimentando el devanado primario, ya que se pretende obtener el circuito equivalente reducido al primario. En la prctica real este ensayo se realiza alimentando el devanado de B.T. porque normalmente su tensin de rgimen est comprendida en las escalas de los aparatos de medida empleados. Adems existe menos peligro para el operador al trabajar con B.T.

CAPTULO 3.

TRANSFORMADORES

189

TRANSFORMADOR

Figura 3.20. Esquema elctrico del ensayo de vaco.

Como quiera que las prdidas RIg en vaco son despreciables (debido al pequeo valor de 10)' la potencia absorbida en vaco coincide prcticamente con las prdidas en el hierro, lo que est de acuerdo con el circuito equivalente aproximado de la Figura 3.21a, que se deduce del esquema general de la Figura 3.18b, al ser [2 = O.De las medidas efectuadas puede obtenerse el factor de potencia en vaco, de acuerdo con la ecuacin: (3.46) Por otra parte, debido al pequeo valor de la cada de tensin primaria, se puede considerar que la magnitud Vln coincide prcticamente con E" resultando el diagrama vectorial de vaco de la Figura 3.21b, en el que se ha tomado la tensin primaria como referencia de fases. En este esquema las dos componentes de 10 valen: (3.47) De donde pueden obtenerse ya los valores de los parmetros RFe Y XI':
VI RFe=[Fe

; XI'

"T.
l'

VI

(3.48)

Es decir, el ensayo de vaco permite determinar las prdidas en el hierro del transformador y tambin los parmetros de la rama paralelo del circuito equivalente del mismo. Del ensayo de vaco puede obtenerse tambin la relacin de transformacin, merced a que la tensin Vln aplicada coincide prcticamente con E" adems la f.e.m. E2 es igual a la tensin
lo
Fe

----+

VI =EI

+
Fe

v.,

RFe

XI'

11' lO

cD a)
b)

Figura 3.21. Circuito equivalente en vaco y diagrama fasorial correspondiente.

190

MQUINAS ELCTRICAS

medida en el secundario en vaco y que se denomina V20. En consecuencia, se cumplir de acuerdo con (3.30):
NI El VIII m=-=-=N2 E2 V20 (3.49)

3.6.2. Ensayo de cortocircuito


En este ensayo se cortocircuita el devanado secundario y se aplica al primario una tensin que se va elevando gradualmente desde cero hasta que circula la corriente asignada de plena carga por los devanados *. El esquema y tipos de aparatos necesarios para la realizacin de este ensayo se indican en la Figura 3.22. La tensin aplicada necesaria en esta prueba representa un pequeo porcentaje respecto a la asignada (3-10 por 100 de VIII)' por lo que el flujo en el ncleo es pequeo, siendo en consecuencia despreciables las prdidas en el hierro. La potencia absorbida en cortocircuito coincide con las prdidas en el cobre, lo que est de acuerdo con el circuito equivalente aproximado de la Figura 3.23a, que se deduce del esquema general de la Figura 3.18b, al despreciar la rama en paralelo, como consecuencia del pequeo valor de la corriente lo frente a t.; De las medidas efectuadas se puede obtener el f.d.p. de cortocircuito, de acuerdo con la ecuacin:
(3.50)

Si en el circuito de la Figura 3.23a se toma la corriente como referencia, se obtiene el diagrama vectorial de la Figura 3.23b, del cual se deduce:
VR,." = Reellll

Vke

COS

<PeL'

VX'T

= X,.,1

11l

= Vice sen

v;

(3.51)

y en consecuencia:
R"e = Vice

i;

COS

<Pee

(3.52)

Tensin variable de c.a.

Cortocircuito TRANSFORMADOR

Figura 3.22. Circuito elctrico del ensayo de cortocircuito.

* Este ensayo se realiza en la prctica alimentando el transformador por el lado de A.T., de esta forma la corriente a medir en el primario ser de un valor razonable. Al mismo tiempo, la tensin de alimentacin slo ser una pequea parte de la nominal, estando comprendida dentro de las escalas de los instrumentos de medida usuales.

CAPTULO 3.

TRANSFORMADORES

191

ol------~Ar--FO()O

flce

fll1~

o------------------~
Figura 3.23. Circuito equivalente de cortocircuito y esquema fasorial de tensiones.

Es decir, el ensayo de cortocircuito permite determinar los parmetros de la rama serie del circuito equivalente del transformador, y de ah que se designen con los smbolos Ree Y Xcc' Debe destacarse que el ensayo de cortocircuito determina la impedancia total del transformador pero no da informacin de cmo estn distribuidos estos valores totales entre el primario y el secundario. Es decir, se obtiene, segn (3.45): (3.53) Para poder determinar los valores individuales de las resistencias R 1 Y R~ es preciso aplicar c.c. a cada uno de los devanados y obtener las resistencias R, y R2 (no R~) aplicando la ley de Ohm y utilizando un factor corrector para tener en cuenta el efecto pelicular que se produce con c.a. (la resistencia hmica es funcin de la frecuencia, debido a la distribucin no uniforme de la corriente por la seccin transversal del conductor). No existen procedimientos para separar en la 2: ecuacin (3.53) XI de X~. En la prctica de la ingeniera elctrica, cuando se desea conocer la distribucin de RLe Y Xec entre ambos devanados es frecuente recurrir a la solucin aproximada siguiente:
R 1- R'2 - _5!. 2

(3.54)

Otro aspecto a tener en cuenta en el ensayo en cortocircuito es que la potencia absorbida coincide con la prdida en el cobre de los devanados correspondiente a la corriente que fluye en esa situacin. Si como exigen las Normas de Ensayos (CEI, UNE, VDE, etc.) esta corriente es la asignada, las prdidas correspondientes representarn las prdidas en el cobre a plena carga. Pero qu sucede si el ensayo de cortocircuito no est hecho con corriente asignada? Esta situacin suele ser conflictiva para los estudiantes. En principio el ensayo no estara realizado conforme a Normas y podra no ser vlido. Si se considera que los parmetros Ree Y Xcc son lineales, lo que significa que sus valores no dependen de la corriente que circula por ellos*, el proceso a seguir sera el mismo que el que as se ha empleado para calcular (3.52) y se obtendran esos mismos valores. El conflicto est en la interpretacin de: 1) las prdidas en cortocircuito, que ya no sern las prdidas en el cobre asignadas nominales o de plena carga sino las prdidas en el cobre al rgimen de carga impuesto por la corriente de cortocircuito a la que se haya realizado el ensayo, 2) la tensin de cortocircuito, que ser proporcional a la corriente a la que se haya efectuado el ensayo. Estimamos que la confusin procede de una

* La realidad es que los parmetros mencionados exhiben una pequea caracterstica no lineal, y de ah la exigencia de las diferentes Normas de que la corriente de ~ortocircuito se haga coincidir con la nominal o de plena carga.

192

MQUINAS ELCTRICAS

indefinicin de las magnitudes que entran en juego. Para aclarar este problema denominaremos: (3.55) a la tensin de cortocircuito con corriente asignada, corriente de cortocircuito igual a la asignada, y potencia de cortocircuito con corriente asignada, respectivamente. Si el ensayo no est hecho con la corriente asignada (nominal), las magnitudes correspondientes se designarn as: (3.56) con ambos juegos de valores (3.55) y (3.56) se obtendrn las mismas soluciones (si el sistema es lineal) expresadas en (3.52). Definidas las corrientes Iln' = Iln e IICOrlO' las relaciones entre las otras magnitudes, teniendo en cuenta el circuito equivalente de la Figura 3.23a, sern:
Vcorto

(3.57)

I,corto

de donde se deduce: Vice = VleorlO -1Iln

; r: = Peortop
1 corto

I~n

(3.58)

Icorto

Las igualdades (3.58) representan de este modo las relaciones de cambio para transformar las magnitudes de ambos ensayos. Nuestro consejo al lector es que si el ensayo de cortocircuito no se ha realizado con corriente asignada [es decir, se han ledo las magnitudes expresadas en (3.56)], se apliquen las ecuaciones de transformacin (3.58) para determinar las magnitudes (3.55) definidas para corriente asignada y determinar los parmetros del circuito equivalente a partir de las ecuaciones (3.50) y siguientes. Normalmente las cadas de tensin indicadas en (3.51) suelen expresarse en tanto por ciento respecto a la tensin asignada, resultando:
Cee

= Vice. 100

v,

(3.59)

El ensayo de cortocircuito debe distinguirse de la falta o fallo de cortocircuito que puede suceder en un transformador alimentado por su tensin asignada primaria cuando por accidente se unen entre s los bornes del devanado secundario. El circuito equivalente en esta situacin es tambin el indicado en la Figura 3.23 (ensayo de cortocircuito); sin embargo, ahora el transformador est alimentado por una tensin VI" (en vez de Vice>,apareciendo una fuerte corriente de circulacin lilaila (o 121alloen el secundario), muy peligrosa para la vida de la mquina debido a los fuertes efectos trmicos y electrodinmicos que produce. Desde el punto de vista de circuito equivalente, el valor de lilaila vendr expresado por:
1110110

VI" Z
ce

(3.60)

y teniendo en cuenta que del diagrama vectorial de la Figura 3.23b se deduce: (3.61)

CAPTULO 3.

TRANSFORMADORES

193

se podr poner: (3.62) que al hacer uso de la La identidad de (3.59) resultar: (3.63) lo que indica que l corriente de cortocircuito de falta est en relacin inversa con cec' Cuanto mayor sea el valor de cce tanto menor ser el valor de la corriente de cortocircuito. Como quiera que un alto valor de cec implica, como se demostrar en el epgrafe 3.7, una fuerte cada de tensin en el transformador, deber adoptarse una solucin de compromiso entre ambos aspectos contradictorios. En la prctica, los transformadores industriales menores de 1.000 kV A, tienen un valor de Cee comprendido entre el 1 y 6 por 100 (transformadores de distribucin); sin embargo, para potencias mayores se aumenta hasta un margen del 6 al 13 por 100. Como se demostrar ms adelante, el valor de Cee tiene tambin gran importancia en el acoplamiento en paralelo de transformadores. Generalmente la componente ": es superior a ":

EJEMPW DE APLICACIN 3.4 Un transformador monofsico de 250 kVA, 15.000/250 V, 50 Hz, ha dado los siguientes resultados en unos ensayos: Vaco: 250 V, 80 A, 4.000 W (datos medidos en el lado de B.T.). Cortocircuito: 600 V, corriente asignada, 5.000 W (datos medidos en el lado de A.T.). Calcular: a) Parmetros del circuito equivalente del transformador reducido al primario. b] Corriente de cortocircuito de falta.
SOLUCIN a) Antes de comenzar el problema se ha de observar que los ensayos no han sido determinados en el primario (vase que la prueba de vaco se ha realizado en el lado de B.T., que en este caso es el lado de 250 V, es decir, el secundario). Es preciso reducir todas las medidas alIado donde se desea obtener el circuito equivalente (primario); para ello se emplear la tcnica expuesta en el epgrafe 3.5. Teniendo en cuenta que la relacin de transformacin es:

m=--=60
250 el ensayo de vaco reducido al primario corresponder VI a los valores: 1,33 A ; 4.000 W

15.000

= 250

. 60

15.000 V

; lo

= 80/60 =

es decir, la tensin se multiplica por la relacin de transformacin, mientras que la corriente debe dividirse por esa cantidad, permaneciendo inalterada la potencia. El f.d.p. en vaco ser entonces: cos
ip

4.000 15.000' 1,33

02 '

194

MQUINAS ELCTRICAS

que corresponde a un sen obtiene:


R
Fe

(/Jo

= 0,98.En consecuencia, y de acuerdo con (3.48),se

15.000 15.000 = 564 ka . x = 1,33.0,2 ' , l' 1,33.0,98

1l,5 ka

Se observa que esta rama paralelo es de gran impedancia, lo que est de acuerdo con la realidad, ya que la corriente de vaco suele estar comprendida en los transformadores industriales entre el 1 y el 8 por 100. En nuestro caso, teniendo en cuenta que la corriente asignada del primario vale:"
1, = -

s,

V'n

250.000 15.000

= 16,67 A

la corriente de vaco lo = 1,33A representa un valor relativo:


lo 1,33 -=--=8% i; 16.67

del ensayo se deduce tambin que las prdidas en el hierro son de 4.000 W. Para calcular la rama serie del circuito equivalente se ha de emplear el ensayo de cortocircuito, cuyos datos estn ya medidos en el lado primario (A.T.); por tanto. estas medidas son de utilizacin directa. El f.d.p. de cortocircuito vale:
COS(/J
ce

=--= V'ee/'n

r:

5.000 =05 600. 16,67 '

que corresponde a sen (/Jee = 0,866.De acuerdo con las expresiones (3.51)y (3.59)se obtiene:
R = -ce

600 . 05 = 18a 16,67 '

ce

600 = -. O 866= 31 17 a 16,67' ,

que son de pequeo valor en comparacin con la rama paralelo. El valor relativo de la tensin de cortocircuito, de acuerdo con (3.59),es:

b)

Al ocurrir una falta de cortocircuito en el transformador, la corriente correspondiente, que aparece en el primario de acuerdo con (3.60),ser:
I'falta

= 4 .16,67= 416,75A

100

que corresponde en el secundario a una intensidad:

CAPTULO 3.

TRANSFORMADORES

195

y como quiera que /211 es igual a: Sil /211 se tendr:


/2/alta

V2n

250.000 250

100 A

100 =4

.1.000 = 25 kA

valores muy superiores a los asignados de la mquina y que habrn de eliminarse por medio de protecciones adecuadas: rels de sobreintensidad, Buchholz, etc., en el menor tiempo posible para no daar al transformador.

3.7.

CADA DE TENSIN EN UN TRANSFORMADOR

Considrese un transformador alimentado por su tensin asignada primaria \(In' En vaco, el secundario proporciona una tensin V20; cuando se conecta una carga a la mquina, debido a la impedancia interna del transformador la tensin medida en el secundario ya no ser la anterior sino otro valor que denominaremos V2 La diferencia aritmtica o escalar entre ambas tensiones: (3.64) representa la cada de tensin interna del transformador. Se denomina cada de tensin relativa o simplemente regulacin a la cada de tensin interna, respecto a la tensin secundaria en vaco (asignada), expresada en tanto por ciento y que se designa por el smbolo Ce (no confundir con Cee definido en (3.59):
C
e

v20 -

V20

2'100%*

(3.65)

Al trabajar con el circuito equivalente reducido al primario es ms conveniente expresar el cociente anterior en funcin de las magnitudes primarias; si se multiplica por la relacin de transformacin m cada trmino de la ecuacin (3.65) y se tiene en cuenta (3.34) y (3.49) resulta:
C
e

v In -

V'

V1n

2.

100%

(3.66)

Para calcular esta relacin se va a considerar un transformador que lleva una corriente secundaria J, con un f.d.p. inductivo (o en retraso) como indica la Figura 3.24. Al aplicar el 2. lema de Kirchhoff al circuito equivalente aproximado del transformador reducido al primario se obtiene: (3.67)

* En Europa este cociente define la cada de tensin relativa. En EE. UU. se emplea el trmino regulacin, yen este caso en el denominador se pone V2 en vez de V,o' Aqu no se har distincin entre ambas definiciones.

196

MQUINAS ELCTRICAS

11

O------~---Jvvv---J
+

z; L ({>2

O------ __ ----------------~--~

Figura 3.24.

Circuito elctrico equivalente para determinar la cada de tensin de un transformador.

que permite calcular la tensin secundaria reducida en funcin de la tensin aplicada al transformador y de la corriente secundaria reducida al primario. Obteniendo en la ecuacin anterior la magnitud de V~,la expresin (3.66) permitir calcular la cada de tensin relativa del transformador. En la prctica, debido a que la cada de tensin del transformador representa un valor reducido 10 por 100) respecto a las tensiones puestas en juego, no suele emplearse la ecuacin fasorial (3.67) para calcular sino que se recurre a un mtodo aproximado propuesto a finales del siglo pasado por el profesor Gisbert Kapp. En la Figura 3.25 se muestra el diagrama fasorial correspondiente al circuito equivalente de la Figura 3.24 y que representa en definitiva la ecuacin fasorial (3.67), donde se ha tomado la tensin como referencia y se ha considerado un f.d.p. inductivo (/; se retrasa ({J2 respecto de V;). Se observa en este grfico que el numerador de (3.66) viene expresado por:

V;,

V;

Vln

V;

= IOSI -

IOPI

= IPSI

(3.68)

siendo S el punto de interseccin de la recta prolongacin de V; con la circunferencia trazada con centro en O y radio Vln. Como quiera que en los transformadores industriales las cadas de tensin son pequeas frente a las magnitudes de Vln y V;, se puede admitir que:
Vln
-

V; = IPSI ~ IPRI

(3.69)

siendo R la proyeccin del afijo del vector VI sobre la recta OS.

Figura 3.25.

Diagrama fasorial de un transformador en carga.

CAPTULO 3.

TRANSFORMADORES

197

El tringulo de cada de tensin PTM se denomina tringulo de Kapp y sus dimensiones son mucho menores que Vln y V~, pero en la Figura 3.25 se ha exagerado su tamao para mayor claridad del diagrama. Teniendo en cuenta que se cumple:

IPRI
resulta:

= IPQI

+ IQRI

= IPQI

+ IMNI

(3.70)

IPR I = RcJ~ cos


Vln V~ =

({J2

+ XcJ~ sen

({J2

(3.71)

por lo que la cada absoluta de tensin tendr un valor:


-

RcJ~ cos

({J2

+ XcJ~ sen

({J2

(3.72)

Si se denomina ndice de carga C al cociente entre la corriente secundaria del transformador y la asignada correspondiente, es decir:
/2 /~ /1 C=-=- ~i: /~n i:

(3.73)

la expresin (3.72) se puede poner: (3.74) o en valores relati vos: (3.75) donde se ha tenido en cuenta, de acuerdo con (3.51) y (3.59), que:

(3.76) Si el f.d.p. de la carga hubiera sido capacitivo, el lector puede demostrar con una simple composicin vectorial que el trmino C Excc sen ({J2 en la expresin (3.75) sera negativo. Este trmino puede en ocasiones ser superior al otro, resultando cadas de tensin negativas, lo que indica que V~ > Vln, o de otra forma, que V2 > V20; es decir, aparecen tensiones en carga superiores a las de vaco. Este fenmeno se conoce con el nombre de efecto Ferranti, por ser el nombre del ingeniero ingls que observ y explic este resultado por primera vez (vase biografa al final de este captulo). Nota prctica: Como ya se ha indicado en el epgrafe 3.6.2, el valor de Ecc en transformadores de menos de 1.000 kVA est comprendido entre el1 y 6 por 100 y para potencias mayores es del 6 al 13 por 100. Esto significa, de acuerdo con (3.75), que las cadas de tensin en los transformadores de gran potencia son mayores que en los de pequea potencia. Sin embargo, segn seala (3.63), cuanto mayor es Eccmenor es la corriente (relativa) de cortocircuito de falta y por consiguiente menor la potencia de cortocircuito necesaria en el disyuntor de maniobra, lo que reduce extraordinariamente el precio de ste. En la soldadura elctrica se construyen transformadores con grandes entrehierros (shunt magntico) para aumentar deliberadamente las reactancias de dispersin de los devanados (en definitiva, para aumentar Ecc)' Es por ello que estos transformadores tienen cadas de tensin

198

MQUINAS ELCTRICAS

elevadas. La tensin secundaria en vaco es del orden de 70 a 100 Y para que pueda producirse el arco inicial, pero esta tensin cae en carga (cuando se suelda con el electrodo) hasta 15 a 25 Y, que es la tensin de mantenimiento del arco. En estos transformadores tambin se preparan derivaciones en las espiras para modificar la relacin de transformacin y conseguir diversas corrientes de soldadura. EJEMPLO DE APLICACIN 3.5 Se dispone de un transformador monofsico de 250 kVA, 15.000/250 V, 50 Hz, que tiene unos parmetros Ree= 18 n; X" = 31,17 n (vase ejemplo de aplicacin 3.4). Calcular: a) Cadas de tensin relativas cR Y Cx . b) Regulacin a plena carga conf.d.p. 0,8 inductivo. c) Tensin secundaria en el caso 'nterTor. d) Regulacin a media carga y tensin secundaria correspondiente con f.d.p. 0,6 capacitivo. e) Regulacin a 3/4 de la plena carga con f.d.p. unidad y tensin secundaria correspondiente. NOTA: La tensin primaria se mantiene constante en todos los casos en 15.000 V. SOLUCIN
a)

La corriente asignada primaria del transformador vale: 1 - 111 -

s,
In

250.000

15.000

16,67 A

yen consecuencia, teniendo en cuenta (3.76), se tiene: ':


b)

18 16,67 15.000

. 100 = 2 %

; ':=

31,17' 16,67 15.000

. 100 = 3,46 %

A plena carga

e=

1, Y la regulacin de acuerdo con (3.75) ser: c,

1 . 2 . 0,8 + 1 . 3,46 . 0,6

= 3,68 %
=>

c) Teniendo en cuenta que:


Ce

VI' - V~

v,

15.000 => Y~ = 14.448 Y => Vz = 240,8 Y

100 % =

15.000 - V;

. 100 = 3,68 %

d)

A media carga (e = 1/2) y para f.d.p. 0,6 capacitivo se cumplir:


8,

= 2' 2

0,6 -

2' 3,46'

0,8

= -0,784%

e)

que correspondea Vz = 251,96 Y, que es superiorincluso a la de vaco (efecto Ferranti). Para e = 3/4 y f.d.p. unidad resulta:
8,

= 4 2 . 1 + 4 3,46 . O =

1,5%

=>

Vz

= 246,25 Y

EJEMPLO DE APLICACIN 3.6 Un transformador monofsico de la kVA, relacin 1.000/100 V, tiene los siguientes parmetros de tensiones relativas de cortocircuito: CRee = 6 por 100; ': = 8 por 100. En el secunda-

CAPTULO 3.

TRANSFORMADORES

199

rio del transformador se conecta una impedancia de 2 L 30 Q. 1) Si la tensin secundaria se considera igual a 100 V, cul ser el valor de la tensin primaria necesaria para que la tensin secundaria se mantenga constante al alimentar la carga mencionada? 2) Si la tensin primaria se mantiene constante e igual a 1.000 V, cul ser la tensin que se obtendr en el secundario al alimentar la carga? SOLUCIN
La corriente primaria asignada del transformador ser: lOA

11 =-=--=
n

Sn

10.000 1.000

Vln

y teniendo en cuenta (3.76) resultar: ': =~'100 v, "; =~'100

R 1

= =

6 =

R 10
1.000

_cc_.

100

= =

R" = 6 Q; Xcc = 8 Q

X 1

v,

Xc) O 8=--100 1.000

que determina la impedancia de cortocircuito del transformador reducida al primario. 1) En la Figura 3.26 se muestra el circuito equivalente que define el problema y que es copia del esquema de la Figura 3.24 adaptado a nuestro caso. Al conocerse la magnitud de la tensin secundaria, parece conveniente elegir esta tensin como referencia y as se tiene:

V2 = 100 LO
Ahora bien, como quiera que la relacin de transformacin es igual a 1.000/100 = 10, se podr escribir: V~ = mV2 = 1.000 LO V ; Z = m2ZL = 200 L 30 Q

lo que da lugar a una corriente secundaria reducida en el circuito de la Figura 3.26: 1.000 LO l~= 200 L 300 5 L -30 A

y teniendo en cuenta la ecuacin (3.67) aplicada a nuestro circuito se tendr:


V = 1.000 L 0 + (6 + j8) 5 L -30 = 1.046,16 L 1,080 es decir, se requieren 1.046,16 V en el primario para que se mantengan 100 V en el secundario

sn +
v

jsn

z; L. (/J2

Figura 3.26.

200

MQUINAS ELCTRICAS

con la carga mencionada. El procedimiento seguido constituye una aplicacin directa de la teora de circuitos al esquema de la Figura 3.24. Si se aplica el mtodo aproximado representado por una adaptacin de la ecuacin (3.72) resulta:
Vln = V~ + RC(I~ CoS(/)2 + XcJ~ sen
(/)2

donde

(/)2

= 30 inductivo. Al sustituir los valores correspondientes se obtiene: VI = l.000 + 6 . 5 . 0,866 + 8 . ? . 0,5 = 1.045,98 V

que es un valor muy cercano al real obtenido resolviendo directamente la ecuacin fasorial. 2) En esta situacin se conoce ahora la tensin primaria del esquema de la Figura 3.24, que al tomarla como referencia de fases: VI = l.ooo LO, Yal aplicar el 2.lema de Kirchhoff al circuito de la Figura 3.24, resulta: 1'=
2

l.000 LO =478L-318A 6 + j 8 + 200 L 30' ,

Por tanto, la tensin V~que se obtendr en este circuito ser: V~ = Z{ . I~ = 200 L 30 4,78 L -31,08

= 955,85 L

-1,08

que corresponde a un mdulo de la tensin secundaria reducida V~= 955,85 V Ya una tensin secundaria de:
V2

955,85 = -V~ = -= 95,585 V m 10

que es la solucin pedida. Observar el lector que en este caso resulta ms conveniente aplicar la teora clsica de circuitos para resolver el problema que utilizar mtodos aproximados, por no conocer a priori la corriente secundaria.

3.8. PRDIDAS Y RENDIMIENTO DE UN TRANSFORMADOR


Como ya se ha indicado en el captulo anterior, una mquina elctrica presenta unas prdidas fijas y unas prdidas variables. Las prdidas fijas se componen de las prdidas mecnicas, que no existen en el transformador (al no poseer esta mquina rganos mviles), y las prdidas en el hierro. Las prdidas variables, que cambian segn sea el rgimen de carga, son debidas a las prdidas en el cobre. De acuerdo con lo expresado en el epgrafe 3.6, ambas prdidas pueden obtenerse de los ensayos del transformador. Se debe recordar que se cumpla:
(3.77)

La segunda identidad representa las prdidas en el cobre a plena carga, puesto que el ensayo de cortocircuito se realiza con corriente asignada. En general, para una corriente secundaria 12 (o reducida 1~)se cumplir: (3.78) Teniendo en cuenta la definicin (3.73) de ndice de carga y la expresin (3.71), la potencia perdida en el cobre en cualquier rgimen de carga se podr expresar como:
I

(3.79)

CAPTULO 3.

TRANSFORMADORES

201

Como en cualquier mquina elctrica, el rendimiento es el cociente entre la potencia til o potencia secundaria y la potencia total o de entrada en el primario, es decir:
r=-=-PI P2+Pp

P2

P2

(3.80)

donde Pp representa la potencia perdida. Si el secundario suministra una corriente /2 a la tensin V2 con f.d.p. cos ({J2' se tendr: (3.81) donde se ha hecho uso de las relaciones (3.73), (3.77) y (3.79); por consiguiente, el rendimiento del transformador resulta ser:

donde el producto V)2" representa la potencia asignada del transformador en.kVA. Se observa que el rendimiento empeora cuanto menor es el f.d.p. de la carga y cuanto menor es el ndice de carga. Como ya se ha demostrado en el captulo anterior, el rendimiento es mximo, para una determinada carga para la cual coinciden las prdidas fijas y variables, es decir, cuando se cumple: (3.83) resultando un ndice de carga ptimo al cual se obtiene el rendimiento mximo dado por: (3.84) Si el transformador trabajara siempre a plena carga convendra que el ndice anterior fuera igual a la unidad, de este modo la mquina trabajara con mximo rendimiento; sin embargo, lo normal es que un transformador trabaje con cargas variables, y esto hace que en la prctica se diseen estas mquinas con un ndice de carga comprendido entre 0,5 y 0,7 para los grandes transformadores de las centrales elctricas y entre 0,3 y 0,5 para los transformadores de distribucin de pequea potencia. EJEMPLO DE APLICACIN 3.7 Se dispone de un transformador monofsico de 250 kVA, 15.000/250 V, 50 Hz, que tiene unas prdidas en el hierro de 4.000 W y unas prdidas en el cobre a plena carga de 5.000 W. Calcular: a) Rendimiento a plena carga con f.d.p. 0,8. b) Rendimiento a media carga con fd.p. unidad. c) Potencia de mximo rendimiento. d) Rendimiento mximo para f.d.p. 0.9. SOLUCIN
a)

De los datos anteriores se deduce:

202

MQUINAS ELCTRICAS

A plena carga, el ndice C es igual al, y el rendimiento del transformador, teniendo en cuenta (3.82), vale:
17 b)

1.250' 0,8 = 95,7 % 1.250 . 0,8 + 4 + 5

A media carga (C = 1/2) se tendr:


17

(112). 250 1 (1/2)250' 1 + 4 +' (1/2)2. 5

= 96%

e)

Se observa que el rendimiento en este caso es mayor que en el caso anterior debido a que ahora trabaja la mquina con mejor f.d.p., pese a que el transformador funciona a media carga. El ndice de carga para el que se obtiene mximo rendimiento es, de acuerdo con (3.84):
Capt = ~

= 0,894

=>

S,/mx = 0,894 . 250 = 223,6 kV A

d)

El rendimiento mximo ser entonces igual a:

S .=
qrnax

0,894 . 250 . 0,9 =962% 0,894 . 250 . 0,9 + 4 + 0,8942. 5 '

El rendimiento maximun maximorum se cumplir para el ndice de carga anterior, con f.d.p. unidad; el lector puede demostrar que es igual a 96,55 %.

3.9.

CORRIENTE DE EXCITACIN O DE VACO DE UN TRANSFORMADOR. ARMNICOS DE LA CORRIENTE DE VACO

Cuando un transformador trabaja en vaco (i2 = O), la corriente que absorbe del primario io se denomina corriente de vaco y es anloga a la corriente de excitacin de una bobina con ncleo de hierro, que ya se estudi en el epgrafe 1.6.3 del Captulo l. En la Figura 3.27 se muestra la forma de onda de io' que tiene una forma de onda acampanada (en el supuesto de que el flujo magntico en el ncleo est por encima del codo de la curva de imanacin). Un anlisis de Fourier de esta corriente demuestra que contiene armnicos de magnitud apreciable. En la Figura 3.27 se han representado: la onda fundamental o de primer armnico y de la de tercer armnico. En un caso real el reparto de armnicos es aproximadamente el siguiente: 45 por 100 del tercero, 15 por 100 del quinto, 3 por 100 del sptimo y porcentajes menores para armnicos de mayor orden. Los valores anteriores se refieren a porcentajes respecto a la onda senoidal equivalente a la corriente de excitacin total. En un anlisis simplificado se puede considerar la existencia de un fundamental de corriente a 50 Hz y un tercer armnico a 150 Hz. La circulacin en lneas areas de un tercer armnico de corriente tiene el inconveniente de que siendo audible la frecuencia de 150 Hz, puede interferir por induccin mutua con lneas telefnicas paralelas cercanas. Otro inconveniente de la circulacin de terceros armnicos de corriente por las lneas areas y sistemas elctricos en general es que deforman las tensiones hacindolas perder su carcter senoidal. Tngase en cuenta que aunque el gene-

CAPTULO 3.

TRANSFORMADORES

203

Primer armnico de la corriente de vaco


Corriente de vaco de un transformador

Tercer armnico de la corriente de vaco

Figura 3.27. Corriente de vaco de un transformador y armnicos l ." y 3.

rador de principio de lnea suministre una tensin senoidal, las corrientes producen cadas de tensin y las componentes fundamentales (a 50 Hz) seguirn produciendo al final de lnea un sistema simtrico de tensiones; sin embargo, los terceros armnicos provocarn cadas que deformarn la forma de las tensiones a finales de lnea. En el epgrafe 3.11.2 se analizarn los efectos de los armnicos en la transformacin de sistemas trifsicos.

3.10.

CORRIENTE DE CONEXIN DE UN TRANSFORMADOR

La corriente de vaco io analizada en el epgrafe anterior representa en rgimen permanente un valor del orden de 5 a 8 por 100 de la corriente asignada del transformador, con una induccin mxima que se sita en el codo de la curva de imanacin del material magntico con el que est construido el ncleo. y que hoy da para chapas de grano orientado se sita en un valor cercano a 1,4 Teslas (1..j.000gauss). Hay que tener en cuenta, sin embargo, que la aplicacin brusca de la tensin asignada Vln a un transformador da lugar a un periodo transitorio en el que la corriente (supngase por sencillez que el transformador est en vaco) puede alcanzar varias veces la asignada y que por ello en ocasiones se ha confundido incluso con una corriente de cortocircuito. Vamos a analizar este fenmeno, en el que influye tanto el instante de la conexin como el flujo remanente existente en el ncleo. Vamos a considerar una tensin aplicada al transformador de la forma:
\ 1(1)

j2

VI cos (wt + x)

(3.85)

La magnitud mxima a la que llegar el flujo en el primer semiciclo de la tensin aplicada depender del ngulo de fase de la misma en el instante de la conexin. Si la tensin inicial es, por ejemplo, de la forma: (3.86) y se considera el transformador ideal, se cumplir: (3.87)

204

MQUINAS ELCTRICAS

que al integrar nos da: <I>(t) = y

r:. V
L, 1

sen tot Nw

+ e

(3.88)

donde e indica la constante de integracin. Si se supone en un principio que el flujo inicial es cero, la constante anterior ser igual a cero, por lo que segn (3.88) el flujo correspondiente ser ya el de rgimen permanente y no causar problemas. Sin embargo, si el instante de la conexin corresponde al momento en el que la tensin aplicada (3.85) tenga una fase !Y. = -900, es decir: v(t)

= j2

V cos (wt - 900)

= j2

V sen cot

(3.89)

el flujo resultante ser ahora igual a: <I>(t)

= <;i;

VI N,w

L, --

cos tt +

e2 = -<I>m cos

cot + C,

(3.90) por lo que el (3.91)

Si en t = O, se tiene un flujo nulo, y la ecuacin anterior nos indica que flujo instantneo valdr: <I>(t) = <1>m (1 - cos wt) lo cual indica que para

e2 = <1>m'

on = 180

(final del primer semiciclo) se obtiene un flujo:

<1> = <1> m (l - cos 1800)

= 2<1> m

(3.92)

que es doble que el flujo mximo de rgimen permanente. En la prctica, si se considera un flujo remanente en el ncleo que puede llegar a 0,5<1>m se puede alcanzar flujos instantneos:

2<1>m + <1> r = 2,5<1> m

(3.93)

lo que se traduce, si se tiene en cuenta la curva de imanacin del material, en una corriente de excitacin que puede llegar a valer 100 veces la corriente de vaco de rgimen permanente. Si se considera que esta ltima corriente es del orden del 5 al 8 por 100 de la corriente asignada, se alcanzarn corrientes instantneas de 5 a 8 veces la asignada, lo que provocar fuerzas electromagnticas en los devanados del orden de 52 a 82, es decir, de 25 a 64 veces las del rgimen de plena carga. Debido al valor no nulo de la resistencia del devanado primario, la corriente transitoria de conexin se amortigua rpidamente al cabo de unos pocos ciclos *, y es por ello que al tratarse de corrientes de poca duracin se tengan que utilizar protecciones que no acten rpidamente, evitando la desconexin del disyuntor o interruptor automtico principal.

3.11. TRANSFORMADORES TRIFSICOS 3.11.1. Generalidades

La transformacin de tensiones y corrientes en los sistemas trifsicos puede realizarse de dos maneras distintas. La primera de ellas consiste en emplear un transformador monofsico en

El ritmo de amortiguamiento es funcin del cociente R/Ldl y es ms rpido en los primeros ciclos que en los ltimos, debido a que la saturacin del ncleo hace que la inductancia no sea constante en el perodo transitorio.

CAPTULO 3.

TRANSFORMADORES

205

cada una de las tres fases, de tal manera que se formen circuitos magnticos independientes. Este sistema es poco econmico, ya que emplea mucho volumen de hierro, a la par que es poco prctico por ser necesario incorporar tres unidades idnticas para realizar la transformacin total. La segunda manera consiste en emplear un solo ncleo magntico en el que se incorporan todos los devanados necesarios. En este caso, el sistema est formado por tres columnas iguales sobre las que se arrollan las espiras que constituyen los bobinados primario y secundario de cada fase, como indica la Figura 3.28a. Se puede considerar que este ncleo trifsicoes el resultado de la unin de tres sistemas monofsicos de acuerdo con el proceso de simplificacin que se observa en las Figuras 3.28b, e y d, donde para dar sencillez a la figura se ha prescindido de los devanados correspondientes. Si el sistema de alimentacin es trifsico equilibrado, los tres flujos <11a' <IIb Y<IIe son iguales en magnitud, estando desfasados 120 en el tiempo, resultando un flujo total <liTen la columna central cuyo valor es cero, y se puede suprimir por tanto esta columna de retomo, como muestra la Figura 3.28d. El sistema resultante a) se obtiene como proyeccin sobre un mismo plano del ncleo d); tal disposicin es la que se utiliza en la prctica pese a que presenta una asimetra en la columna central, merced al menor recorrido del flujo. Este hecho produce un desequilibrio en las corrientes de vaco de las tres fases, pero su efecto en carga es prcticamente despreciable. En el estudio del transformador trifsico hay que considerar cada columna como un transformador monofsico, de tal forma que los ensayos, esquemas equivalentes, etc., deben expresarse en valores simples, pudindose aplicar entonces las mismas tcnicas de anlisis empleadas en el estudio de los transformadores monofsicos. Por ejemplo, la relacin de transformacin ser el cociente entre el nmero de espiras por fase del primario y el nmero de espiras por fase del secundario, que coincidir con la relacin entre las f.e.m.s. por fase entre primario y secundario. La designacin de los terminales de los devanados se har siguiendo las recomendaciones expresadasen los epgrafes 3.2 y 3.3, empleando las letras A, B, e para denominar los principiosde las bobinas del devanado de A.T., que se supondr situado en el primario; las mismas letras en minscula, a, b, e, indicarn los terminales de la misma polaridad instantnea en el devanado secundario. Los dems terminales se expresarn con las mismas letras con tilde, como se muestra en la Figura 3.28a.

Figura 3.28.

Circuito magntico de un trafo trifsico y su gnesis a partir de tres trafos monofsicos.

206

MQUINAS ELCTRICAS

Las formas que ms frecuentemente se emplean para realizar las conexiones de los arrollamientos son: en estrella (con o sin hilo neutro), en tringulo y en zig-zag. En la conexin en estrella se unen en un mismo punto los tres extremos de los arrollamientos que poseen la misma polaridad, existiendo dos formas bsicas segn se unan los terminales A, B, e o A', B', e' (a, b, e o a', b', e' para el secundario). En la conexin en tringulo se unen sucesivamente los extremos de polaridad opuesta de cada dos devanados hasta cerrar el circuito; segn sea el orden de sucesin en que se realiza esta operacin, puede existir dos formas diferentes. La conexin en zig-zag, que en la prctica slo se emplea en el lado de menor tensin, consiste en subdividir en dos partes iguales los devanados secundarios, una parte se conecta en estrella y luego cada rama se une en serie con las bobinas invertidas de las fases adyacentes, siguiendo un determinado orden cclico. Estas tres formas bsicas se muestran en la Figura 3.29 (slo para el devanado de B.T.). Las diferentes conexiones se designan con letras, de acuerdo a la siguiente nomenclatura: Estrella: Tringulo: Zig-zag:
y (en el lado de A.T.), D( ),

y (en el lado de B.T.).


d ( ).

Z (

),

z (

).

Dependiendode los tipos de conexin de los devanados de un transformador,pueden aparecer unas diferencias de fase entre las tensiones compuestas de primario y secundario. Para que esta fase quede unvocamente determinada se supondr que el transformador se alimenta por medio de un sistema equilibrado de tensiones de sentido directo RST (es decir, ABC), de tal forma que se considerarnngulos positivos los de retraso del lado de menor tensin respecto al devanado de tensin ms elevada. Estos ngulos no se miden en grados, sino en mltiplos de 30 = 2n/12 radianes, lo que permite identificarloscon los que forman entre s las agujas de un reloj cuando marca una hora exacta, y esto da lugar al llamado ndice horario del transformador; por ejemplo, un ndice igual a 5 significa un retraso de 5 . 30 = 150 de la tensin menorrespectoa la superior.El procesoa seguirpara determinarel ndicehorarioes el siguiente:
0

Se representan las Le.m.s. simples del devanado primario, de tal forma que el terminal A se sita en la parte superior del diagrama (coincidiendo con el nmero 12 de un reloj imaginario que se colocar superpuesto al esquema vectorial). 2. Se representan las f.e.m.s. simples secundarias, y para ello debe tenerse en cuenta que los devanados primario y secundario situados en la misma columna producen f.e.m.s en fase (para los bornes homlogos).
a

l.

a) Estrella b) Tringulo
e)

Zig-zag

Figura 3.29. Tipos de conexiones de los transformadores trifsicos.

CAPTULO 3.

TRANSFORMADORES

207

3. Se superponen ambos diagramas. El ngulo horario es el que forman dos vectores, uno que pasa por el punto A y el centro del diagrama y el otro el que pasa por a y ese mismo centro *. De aqu se deduce el ndice horario. Para ver de una forma prctica el desarrollo anterior, se va a considerar el clculo del ndice horario del transformador tringulo-estrella indicado en la parte izquierda de la Figura 3.30 donde se muestra la tapa de bornes junto con una representacin esquemtica de las bobinas abatidas sobre el plano de aqullas (mirando el transformador por la parte de A.T., las bornas se suceden de izquierda a derecha en el sentido ABC). Primeramente se dibujan las f.e.m.s. simples del primario, colocando el terminal A en la parte superior (Fig. 3.30a). Se tendr un tringulo ABC, donde se seala en cada lado los terminales de la bobina a que corresponden y el orden en que se suceden. Se representan luego las f.e.m.s. secundarias (Fig. 3.30b), teniendo en cuenta que los arrollamientos de la misma columna estn en fase, y as se observa que a-a' es paralela aA-A', b-b' a R-R' y e-e' a C-C'. Al superponer ambos diagramas se obtiene la Figura 3.30e. El ngulo horario es el que forman los vectores Oa y OA (siendo Oel centro del diagrama), y se observa que el ngulo es de 330 de retraso de la B.T. frente a la A.T., es decir, 330/30 = 11, ndice horario que se puede leer directamente en el reloj imaginario que se ha dibujado en la misma figura. La forma simblica que se empleara para definir este transformador sera Dyl i, que significa: a) primario conectado en tringulo (A.T.), b) secundario conectado en estrella (B.T.), e) el ndice horario es igual a 11. Dependiendo de la forma en la que se efectan las conexiones en primario y secundario y del sentido de sucesin de fases de la red, se pueden obtener en el transformador anterior los ndices horarios: 1,5, 7, 11. Si se introducen adems arrollamientos en zig-zag, se encuentran un gran nmero de combinaciones posibles. En la prctica se utiliza un nmero limitado de acoplamientos que se dan en la Tabla 3.3, estando agrupados segnsea el valor del ndice horario. La importancia de este cuadro se justificar en el epgra0

= C'
Q

a)
b' A
Q

b b)

C=B'

B'----B

B=A' 12

B b

C e _ ... _-_ ... ------

e)

Figura 3.30. Determinacin del ndice horario de un transformador Dy.

* El lector puede demostrar fcilmente que este ngul~ coincide con el que forman las tensiones compuestas. Esteprocedimiento evita complicaciones grficas en el diagrama.

208

MQUINAS ELCTRICAS

Tabla 3.3.

Tipos de acoplamientos de transformadores trifsicos

DdO

CDBCDb
A a

Dd6

O (0)

YyO

CAB~b

6 (180)

Yy6

DzO

Dy5

CBrt. C~B~d
c~:<j
C

Dz6

Dyll

c:V' .i..: c:~c CB~


a' a' a'

5 (150)

Yd5

11 (330)
a

Ydll

cB1~
c

Yz5

~b~' B

Yz11
a'

C~B),~b

fe 3.12. En la tabla se han recuadrado los tipos de conexin que se prefieren en el diseo de nuevas instalaciones. Experiencia prctica de laboratorio: Si se dispone de un transformador trifsico con todas las bobinas de primario y secundario independientes se puede preparar una prctica curiosa que suele sorprender a los estudiantes. Supngase que para mayor sencillez se utiliza un transformador trifsico de relacin unidad como el mostrado en la Figura 3.31. A continuacin debe prepararse el montaje que se seala en la figura; es decir, conectar a cada bobina secundaria una lmpara incandescente de la potencia adecuada al transformador (por ejemplo, supngase que las lmparas son de 220 V, 60 W). Dejar abierto el devanado primario AA', conectar un interruptor en paralelo con el devanado primario CC' y alimentar el transformador con una red monofsica de 220 V que se aplica al devanado central primario BB'. Primera experiencia: Qu sucede con las lmparas si el interruptor S est abierto? Solucin: Se observa que la lmpara 2 brilla con su flujo luminoso normal (plena luz), mientras que las lmparas 1 y 3 estn a media luz. El resultado se explica debido a que los flujos

CAPTULO 3.

TRANSFORMADORES

209

220V

'\J

A'
a

a'

Trafo trifsico (relacin m = 1)

Lmpara 1

Lmpara 2

Lmpara 3

Figura 3.31.

Experiencia prctica.

magnticos en las ramas laterales prcticamente son la mitad del que existe en la rama central, y por consiguiente en los devanados secundarios aa' y ce' se inducen unas tensiones de 110 V mientras que en el bb' la tensin inducida es de 220 V. De ah que las lmparas 1 y 3 estn sometidas a una tensin mitad de su valor asignado mientras que la lmpara 2 trabaja a su tensin asignada. Segunda experiencia: Qu sucede con las lmparas al cerrar el interruptor S? Solucin: Se observar que la lmpara 1que estaba antes a media luz se enciende con su flujo luminoso normal (plena luz). La lmpara 2 sigue igual que antes, es decir, a plena luz, mientras que la lmpara 3 se apaga. La explicacin de esta experiencia es la siguiente: al cortocircuitar el devanado ce (no preocuparse, no aparecen corrientes elevadas al realizar esta operacin!) obligamos a que la tensin en este devanado sea igual a cero voltios, por lo que se anula el flujo en esta columna [vase simplemente las relaciones (3.5) que relacionan flujo con la f.e.m. inducida] y como consecuencia de ello el flujo central, que antes se reparta por igual entre las columnas laterales, est obligado a pasar solamente por la columna de la izquierda y de este modo las tensiones inducidas en los devanados aa' y bb' sern iguales entre s e iguales a 220 V, mientras que la tensin inducida en el devanado ce' ser de cero voltios. En definitiva, al cortocircuitar un devanado se produce la expulsin del flujo que pasa por l, por lo que el flujo magntico debe buscar otro camino de cierre. Este tipo de experimento se puede completar practicando entrehierros en las columnas. Por ejemplo, si en el caso de la Figura 3.31 se tiene un entrehierro en la columna de la izquierda, estando el interruptor S abierto, las lmparas 2 y 3 funcionarn prcticamente con tensin asignada (por consiguiente, a plena luz), mientras que la 2 estar casi apagada, ya que por esta columna no pasar casi flujo magntico debido a que su reluctancia magntica es elevada por existir un entrehierro en aqulla. Sin embargo, al cerrar el interruptor S, la lmpara 3 se apagar porque se anula el flujo de su columna; es decir, en esta situacin el flujo de la columna central se cerrar ahora por la columna de la izquierda y la lmpara 1 que estaba apagada se encender.

210

MQUINAS ELCTRICAS

3.11.2. Armnicos en las corrientes de excitacin de transformadores trifsicos


a) Bancos trifsicos a base de transformadores monofsicos
En el epgrafe 3.9 se demostr que si se aplica una tensin senoidal a un transformador monofsico y el ncleo trabaja en el codo de la curva de imanacin, se obtiene una corriente de excitacin de forma acampanada con un contenido de armnicos impares. Aparte del armnico fundamental, el ms importante es el tercero. De este modo la corriente de vaco del transformador responde a la forma simplificada siguiente:
io

= io.1

+ io.3

= J2

10.1 cos (wt + JI) +

J2

lO,)

cos (3wt + J3)

(3.94)

Si se considera un banco trifsico a base de tres transformadores monofsicos, como se muestra en la Figura 3.32, donde se han conectado los devanados en estrella tanto en el lado primario como en el secundario, las corrientes de vaco de cada uno de los transformadores, de acuerdo con (3.94), sern:
io.l iO,l1 iO.l/l

= = =

J2 1
}2 1

0,1,1

cos (wt + JI) + cos (wt + cos (wt +


<(JI <(JI

0,1./1

J2 1

J2 1 ,3.1 cos (3wt + J3) 120) + fi 1 cos (3wt +


0 0.3,11

<(J3 <(J3

3 . 120) (3.95) + 3 . 120)

0,1,111

+ 120) +

}2 1

0,3.111

cos (3wt +

Se observa en las expresiones anteriores que las componentes fundamentales de las corrientes de vaco de cada transformador forman un sistema trifsico equilibrado, mientras que las corrientes de tercer armnico estn en fase. En la Figura 3.33 se muestra de un modo grfico este hecho. Si el primario del grupo transformador dispone de hilo neutro, la aplicacin del primer lema de Kirchhoff al nudo N' de la Figura 3.32 nos indica que por el hilo neutro retorna una corriente:
in,N = 3

[J2 1

0.3 COS

(wt + J3) 1 = 3 in.3

(3.96)

es decir, circula una corriente de tercer armnico que es el triple del valor correspondiente a cada uno de los transformadores (por simetra, los componentes fundamentales dan una suma nula).
R

~
'0,1

f-ob

'o, II

II

-o

T ()..
N

_____.iO1I1
+-:I0, N

e
III

e :.....o-

N'

Figura 3.32. Banco trifsico a base de tres transformadoresmonofsicos.

CAPTULO 3.

TRANSFORMADORES

211

Como ya se indic en el epgrafe 3.9, la circulacin de corrientes armnicas en las lneas areas producen interferencias en lneas telefnicas paralelas cercanas, debido a que su frecuencia de 150 Hz est en el rango audible del odo humano. Por otra parte, la circulacin de los terceros armnicos de corriente por las lneas areas deforman las tensiones que dejan de ser sinusoidales; tngase en cuenta que aunque al principio de una lnea trifsica se tenga un sistema equilibrado y senoidal de tensiones, las componentes fundamentales (primer armnico de 50 Hz) de las corrientes producirn al final de la lnea un sistema de tensiones simtrico. Sin embargo, los terceros armnicos de corriente producen cadas de tensin que van en fase entre s, dando lugar a una deformacin y desequilibrio de las tensiones que resultarn al final de la lnea. Es por este motivo por lo que en las lneas de A. T. no se suele disponer de neutro de retorno. Ahora bien, si se considera anulado el conductor neutro en el esquema de la Figura 3.32, la aplicacin del primer lema de Kirchhoff al nudo N' de esta figura exigir el que la suma de las tres corrientes mostradas en las ecuaciones (3.95) sea igual a cero. En forma fasorial esta suma es: (3.97) es decir: (3.98) El primer parntesis da un valor nulo, puesto que representa la suma de las componentes fundamentales de las corrientes de vaco, que de acuerdo con (3.95) constituyen un sistema simtrico equilibrado. Para que se anule el segundo parntesis de (3.98) ser preciso que se anulen las componentes de corriente de tercer armnico, es decir, se cumplir:
1 0.3.1 =1(U.l1 =10.3./11 =0

(3.99)

La no existencia de terceros armnicos en la corriente de excitacin indica que la corriente se hace senoidal, lo que obligar a que el flujo magntico pierda su carcter senoidal. La Figura 3.34 justifica grficamente este hecho. La composicin grfica es inversa a la realizaiO. 1.1: Primer armnico de la corriente de vaco del trafo I

O.N

io. 1. 11: dem del trafo "


io.1. 111: Im del trafo III

1 '

o
...

f \:~/ \t/ x/ \., ..\


j

"r

>.~..J.'

1IfIi,' ,_,
10.3.111'

-. ......

#'

io.3." dem del trafo II


io.3. 1: Tercer armnico de la corriente de vaco del trafo 1

. Idem del trafo 111

Figura 3.33.

Armnicos principal y tercero de la bancada trifsica de la Figura 3.32.

212

MQUINAS ELCTRICAS

Onda de flujo $ (r)

Tercer armnico de flujo

"O
OJ

.... o

,~
Q.

Figura 3.34. Deformacin del flujo magntico para corrientes de circulacin senoidales.

da en el epgrafe 1.6.3,en la que se demostraba la deformacin en la corriente de excitacin si se parta de un flujo senoidal. En el caso de la Figura 3.34, al partir de una io senoidal (es decir, una f.m.m. de excitacin senoidal), y teniendo en cuenta el carcter no lineal de la curva de imanacin, dar lugar a un flujo no senoidal en cada uno de los ncleos del banco transformador de la Figura 3.32. Como se muestra en la Figura 3.34, la nueva onda de flujo puede descomponerse en una fundamental ms un tercer armnico (se desprecian los armnicos de orden superior) y como consecuencia de este hecho, al no ser senoidales los flujos, tampoco lo sern las f.e.m.s. inducidas en los secundarios, lo que equivale a decir que el banco trifsico se ha convertido en un generador de armnicos de tensin. Estas f.e.m.s. sern, de un modo anlogo a (3.95), de la siguiente forma:
el ell elJl

= el./

+ eJ.! = j2 + e3.I1
e3,lIJ

EI./ EI.I! El,lU

cos (wt +

1X1)

+
-

j2

EJ.I

cos (3wt +
E3.I!

IXJ) 1X3)

= el.I1

= j2
=

cos (wt + IXI cos (wt + IXI

= e I./IJ +

j2

j2 + 120) + j2
120) +

cos (3wt +

(3.100a)

EJ,lIJ

cos (3wt + iX3)

Las f.e.m.s. fundamentales son simtricas y equilibradas y las de tercer armnico tienen la misma magnitud y fase. En la Figura 3.35 se han representado los fasores correspondientes a las f.e.m.s. (3.100). Este diagrama fasorial requiere una explicacin, ya que muestra vectores giratorios que giran a diferente velocidad por tener diferente frecuencia. As, las componentes

CAPTULO 3.
_.,...- ...-.....

TRANSFORMADORES

213

()~)H'
-v

"'
. .. ..._ e'

b)

(~~:::::>,~,
EI,m "',., ..

---------------------
(/ lOO
J-I:'"

z(

b Fenmeno de oscilacin del neutro (neutro inquieto)

\'" E3.11

-,..... - ..----:::.::_-------------------

..-----..------------.----------------------------------------------------

velocidad de giro: 100 Hz

Figura 3.35.

F.e.m.s. de primer y tercer armnico y movimiento del.neutro.

fundamentales de f.e.m. girarn en sentido contrario a las agujas del reloj a una velocidad correspondiente a 50 Hz, mientras que los terceros armnicos de f.e.m. deben girar a 150 Hz. Si se suponen quietos los fasores fundamentales de f.e.m., por ejemplo porque los observadores giramos con ellos, se vern los fasores correspondientes a los terceros armnicos de f.e.m. girando a razn de 100 Hz. Si imaginamos en la Figura 3.35a que estn fijos el neutro n y los fasores fundamentales, entonces los fasores de tercer armnico estarn girando a razn de 100 Hz, con centros en a, by e. Los segmentos na', nb' y nc' representan las f.e.m.s de cada una de las fases del secundario del transformador y que irn variando con el tiempo en virtud del movimiento de a', b' y e'; sin embargo, los segmentos a'b', b'c' y c'a', que representan las f.e.m.s compuestas, forman un tringulo indeformable cuyos lados tienen longitudes constantes pero dotados de un movimiento de vaivn en virtud del movimiento circular de los vrtices a', b' y e'. Se puede dar otra visin ms cmoda de este fenmeno, y es considerar quieto el tringulo final a'b'c' (Fig. 3.35b) Yadmitir que se mueve el neutro segn una circunferencia de radio E3 (magnitud del tercer armnico). De ah la denominacin que hacen algunos autores del punto n, que lo definen como neutro inquieto (oscilacin del neutro). La interpretacin matemtica de esta construccin grfica es inmediata si el lector analiza las ecuaciones (3.100). Se observa que las f.e.m.s. de cada fase secundaria varan con el tiempo en virtud de la presencia del tercer armnico. Sin embargo, esto no sucede con las f.e.m.s. o tensiones compuestas, que como sabemos se obtienen como diferencia entre las tensiones simples correspondientes, ya que con la resta desaparecen las componentes de tercer armnico. Es decir, en las tensiones compuestas no existen componentes de tercer armnico.

En definitiva, y como conclusin a este estudio: en un banco trifsico estrella-estrella a tres hilos aparecen sobretensiones en las tensiones simples debido a la presencia de componentes de tercer armnico, lo que puede resultar perjudicial para la vida de los aislantes y puede provocar a veces grandes tensiones resonantes entre la inductancia del transformador y la capacidad de la lnea; por esta razn los transformadores monofsicos del banco trifsico se deben proyectar con inducciones bajas (por debajo del codo de la curva de imanacin), lo que supone un encarecimiento de los mismos.

214 b)

MQUINAS ELCTRICAS

Transformadores con ncleo magntico trifsico

Consideremos un transformador Y-y con ncleo magntico trifsico como el mostrado en la Figura 3.36. Por las razones expuestas en el apartado anterior, no se dispone de neutro en la lnea de alimentacin del primario. Por lo tanto, no circularn terceros armnicos de corrientes de excitacin y, por consiguiente, las corrientes de vaco sern sinusoidales, lo que obliga, segn se ha sealado en el epgrafe anterior, a que los flujos no puedan ser senoidales y contengan por ello una componente de tercer armnico. Al ser el ncleo trifsico (Fig. 3.36) la suma de los flujos de las tres columnas, deben anularse, ya que no existe cuarta columna de retomo (en un nudo magntico la suma de los flujos debe ser igual acero). Evidentemente, la suma de los flujos fundamentaleses cero, ya que estn desfasados en el tiempo 120. Sin embargo, no ocurre 10 mismo con los flujos de terceros armnicos, que estn en fase (homopolares), es decir, pulsan a un tiempo <1>3.1 = <1>3.11 = <1>3,1/1' por lo que su suma <1>31 + <1>3/1 + <1>3/11 =1- O. Este hecho paradjico se resuelve, de acuerdo con el circuito magntico de'la Figura 3.36, haciendo que la suma de los flujos anteriores, no nulas, pueda saltar por algn sitio. Efectivamente, los terceros armnicos de flujo tienden a cerrarse por una cuarta columna de retomo que es el aire, 10 que representa un camino de gran reluctancia, y por tanto apenas existirn terceros armnicos de flujo. La consecuencia de orden prctico es que las tensiones simples secundarias no presentan terceros armnicos de gran consideracin, aun cuando se proyecte el transformador para funcionamiento en la zona de saturacin. Una forma prctica de eliminar los flujos de terceros armnicos evitando que salten por el aire es conectar el secundario en tringulo. En la Figura 3.37 se observa tal disposicin. Supngase por el momento que el tringulo est abierto por medio del interruptor D. Adems de los fundamentales de flujo, existirn componentes de tercer armnico que saltarn por el aire, que inducirn f.e.m.s. en el devanado secundario, con expresiones similares a las indicadas (3.100a). Entre los extremos m y n del interruptor D aparecer una tensin que ser la suma de las f.e.m.s. inducidas en cada una de las bobinas secundarias:
(3.100b)

expresin en la que se ha tenido en cuenta que el primer parntesis se anula porque las f.e.m.s. de primer armnico son iguales y desfasadas 120 mientras que las de tercer armnico al ser las tres idnticas se obtiene el triple de una de ellas.
0,

Figura 3.36. Transformador con ncleotrifsicoy conexinYy. Flujosprincipales y de tercerarmnico.

CAPTULO 3.

TRANSFORMADORES

215

Figura 3.37.

Borrado de los flujos de tercer armnico por medio de la conexin en tringulo.

En el momento en que se cierre el tringulo por medio del interruptor D, la f.e.m. resultante anterior 3e, producir una corriente de tercer armnico, que al circular por los tres devanados secundarios crear, en cada uno de ellos, flujos en oposicin a los flujos armnicos preexistentes producidos por las corrientes primarias de excitacin, por lo que finalmente se cumplir: <D} 1 = <D} 11 = <D3111 = O.En definitiva, se ha producido un borrado o anulacin de los flujos de tercer .armnico. Es como si el devanado secundario en tringulo permitiera llevar o conducir la componente perdida de tercer armnico de la corriente de excitacin primaria, haciendo que tanto los flujos como las tensiones inducidas tengan la forma sinusoidal. Se consigue el mismo efecto dotando al transformador Y-y de arrollamientos de compensacin o devanado terciario (Fig. 3.38). El devanado terciario es un arrollamiento conectado en tringulo sin conexin con el exterior, cuyo principio de funcionamiento es anlogo al de la conexin del secundario en tringulo (Fig. 3.37). El devanado terciario constituye un camino de baja impedancia para que puedan circular las corrientes de tercer armnico. Este

PRIMARIO

TERCIARIO

SECUNDARIO

Figura 3.38. Transformador trifsico con devanado terciario.

216

MQUINAS ELCTRICAS

montaje se utiliza cuando se desea eliminar los flujos armnicos pero pudiendo disponer de un neutro secundario.

3.11.3.
e)

Conexiones de los transformadores trifsicos

Las conexiones bsicas de los transformadores trifsicos son: a) Y- y; b) Y-d; c) D-y; d) D-d; Y-z. Vamos a analizar las ventajas e inconvenientes de cada tipo de conexin.

a)

Conexin Y-y

Para una tensin dada entre fases Vu la tensin en bornes de una fase de un transformador conectado en estrella es VL mientras que cuando se trata de transformadores conectados en tringulo la tensin en cada bobina es VL' o sea, un 73 por 100 mayor; la corriente en cada bobina de un transformador conectado en estrella es igual a la corriente de lnea 1u mientras que la de un transformador conectado en tringulo es ILIJ3, o sea, el 58 por 100 de la corriente de lnea. As pues, siempre que las restantes condiciones sean las mismas, el devanado de un transformador conectado en estrella tendr menos espiras, necesitar un conductor de mayor seccin transversal que un transformador equivalente conectado en tringulo y su construccin ser algo menos costosa. Las construcciones con hilos ms gruesos (mayor seccin transversal) proporcionan devanados mecnicamente fuertes y, por tanto, mejor dispuestos para resistir los esfuerzos debidos a grandes corrientes de cortocircuito, mientras que las tensiones ms bajas reducen el esfuerzo dielctrico sobre el aislamiento. Por tales razones, la conexin Y-y se considera ventajosa cuando han de enlazarse dos sistemas de tensiones relativamente altas, e incluso existe otra razn que puede resultar conveniente en determinados casos, la de que no existe desplazamiento de fase entre las tensiones de primario y secundario. La conexin Y-y presenta dos inconvenientes graves:

/J3,

1.

2.

Si la carga del transformador es desequilibrada, se producen flujos alternos en las columnas que van en fase, es decir, pulsan al unsono (homopolares), lo que da lugar a tensiones homopolares que provocan un desequilibrio en las tensiones simples (traslado del neutro) tanto del primario como del secundario. Existen terceros armnicos de tensin, como ya se ha indicado en el epgrafe anterior.

Los problemas anteriores se pueden resolver mediante una de las siguientes tcnicas:
a)

b)

Conectando rgidamente a tierra los neutros de los transformadores, especialmente el del primario. Esta solucin no es del todo conveniente, ya que entonces se permite que circulen los armnicos de la corriente de vaco. Aadir un tercer devanado (terciario) conectado en tringulo (vase Fig. 3.38) que anula los flujos homopolares debidos bien sea a los armnicos de tensin como a los desequilibrios de las cargas. A pesar de no ser necesario, los devanados terciarios tienen terminales hacia el exterior del transformador que se utilizan en la alimentacin de los servicios auxiliares del centro de transformacin en el que estn situados. Como el devanado terciario debe ser capaz de soportar corrientes circulantes, normalmente su capacidad es del orden de la tercera parte de la potencia asignada de los otros dos devanados. En la prctica, la conexin Y-y se usa muy poco, ya que pueden lograrse iguales prestaciones con otro tipo de transformador trifsico.

CAPTULO 3.

TRANSFORMADORES

217

b)

Conexin Y-d

Esta conexin no tiene problema con los componentes de tercer armnico de tensin (vase epgrafe 3.11.2), puesto que stos dan lugar a una corriente circulante en el lado conectado en tringulo. La conexin se comporta razonablemente bien bajo cargas desequilibradas, ya que el tringulo redistribuye parcialmente cualquier desequilibrio que se presente. Sin embargo, esta disposicin tiene el problema de que, debido a la conexin en tringulo, las tensiones secundarias sufren un desplazamiento de 30 con respecto a las tensiones del primario, lo cual puede causar inconvenientes al conectar en paralelo los secundarios de dos grupos transformadores, ya que para hacer esta maniobra, y como se demostrar ms adelante, es preciso que los diferentes grupos tengan el mismo ndice horario. La conexin y-d se adapta particularmente bien a transformadores en sistemas de alta tensin en el extremo reductor de tensin de la lnea.

e) Conexin D-y
Esta conexin presenta las mismas ventajas y el mismo desplazamiento de fase que la conexin Y-d. Se utiliza como transformador elevador en las redes de A.T. El empleo de la conexin en estrella en la parte de alta tensin permite poner a tierra el punto neutro, con lo que queda limitado el potencial sobre cualquiera de las fases a la tensin simple del sistema, reducindose al mismo tiempo el coste de los devanados de A. T., por las razones expuestas en la explicacin de la conexin Y-y. Esta conexin es tambin muy utilizada en los transformadores de distribucin, correspondiendo la estrella al lado de baja tensin, que permite de este modo alimentar cargas trifsicas y cargas monofsicas (en este caso entre fase y neutro). El primario en tringulo tiende a compensar los desequilibrios producidos por las cargas monofsicas.

d)

Conexin

n-e

Esta conexin se utiliza en transformadores de B.T., ya que se necesitan ms espiras/fase de menor seccin. Se comporta bien frente a cargas desequilibradas. La ausencia de neutro puede ser a veces una desventaja. Este montaje puede trabajar al 58 por 100 (es decir, 11)3) de la potencia asignada trabajando como tringulo abierto en el caso de un banco trifsico con transformadores monofsicos en el que hay una unidad averiada y hay que proceder a su reparacin.

e) Conexin Y-z
La conexin zig-zag se emplea nicamente en el lado de B.T. Este montaje se utiliza en redes de distribucin, ya que permite el uso de un neutro en el secundario. Se comporta bien frente a desequilibrios de cargas. Debido a la composicin de tensiones del lado secundario, se requiere un 15 por 100 ms de espiras que en una conexin en estrella convencional. EJEMPLO DE APLICACIN 3.8 Se conecta un transformador trifsico reductor a una lnea de 20 kV y absorbe 20 A. Si la relacin de espiras por fase es igual a 100, calcular la tensin compuesta y la corriente de lnea en el secundario del transformador,para las siguientes conexiones: a) Estrella-estrella.

218

MQUINAS ELCTRICAS

b) Tringulo-tringulo. e) Estrella-tringulo. prdidas del transformador.

d) Tringulo-estrella. NOTA: Se desprecian las

SOLUCIN La potencia del transformador es en todos los casos igual a:


S=

13

V / =

13 . 20.000 . 20 :::::693 kV A
20.000
VI =

En las diferentes conexiones, se tiene:


a)

Estrella-estrella: La tensin simple de primario ser:

13

y la corriente de cada fase coincide con la de la lnea, es decir, /1 = 20 A. Los valores simples secundarios vendrn ligados por la relacin de transformacin, de acuerdo con la siguiente ecuacin:

de donde se deduce:
V2

= 13

200

/2

= 2.000 A

b)

e)

que en valores compuestos corresponden a 200 V Y2.000 A. Tringulo-tringulo: De forma anloga al caso anterior, se deducen unos valores de lnea igual a 200 V Y2.000 A en el secundario. Estrella-tringulo: Los valores simples de primario, teniendo en cuenta que est conectado en estrella, sern:
VI

20.000

13 ; ;

/1

= 20 A

Los valores simples de tensin y corriente secundaria, de acuerdo con la relacin de transformacin, dan:
V2

= 13

200

/2

= 2.000 A

d)

que teniendo en cuenta que este devanado est conectado en tringulo, corresponde a unos valores de lnea de voltios y amperios. Tringulo-estrella: La tensin y corriente de cada fase del primario al estar conectado este devanado en tringulo son:

200/13

2.00013

VI = 20.000 V

CAPTULO 3.

TRANSFORMADORES

219

y los valores correspondientes

en secundario seran: ; 12

Vz = 200 V
lnea de

2.000

J3

quie teniendo en cuenta que este devanado est en estrella, resultan unos valores de

200J3 y 2.000/.)3

A.

EJEMPLO DE APLICACIN 3.9 Determinar el ndice horario del transformador estrella zig-zag representado en la Figura 3.39. Si el primario tiene NI espiras por fase y el secundario N, (total, suma de las dos mitades), calcular la relacin entre las fe.m.s. de lnea de primario y secundario.

A'

'----x--~----

R'

C'

Figura 3.39.

SOLUCIN En la Figura 3.40a se representa el diagrama vectorial de las f.e.m.s. primarias, formando una estrella de sucesin ABC. En la Figura 3.40b se muestra la composicin vectorial de las f.e.m.s. secundarias, donde aparece la estrella invertida b;, a la cual se suman las f.e.m.s. de las fases adyacentes. Al superponer ambos diagramas se obtiene la Figura 3.40c, donde se observa que el ndice horario es igual a 11. Para calcular la relacin de las f.e.m.s. compuestas, se observa que en el primario se cumple:

a;,

c;,

En el secundario, si se denomina E2 la f.e.m. inducida en cada semidevanado (es decir, producida por N2/2 espiras), se observa en la composicin vectorial de la Figura 3.40b que se cumple:

y en consecuencia, resulta:

220

MQUINAS ELCTRICAS

A b'I

al

B 12

e'

el

Figura 3.40.

EJEMPLO DE APLICACIN 3.10


La Figura 3.41 muestra tres transformadores monofsicos de relacin 1.000 V/2OO V cada

uno y con unas impedancias de primario y secundario, respectivamente, de valores: Z = 0,75 + j 1 O; Z2 = 0,03 + j 0,04 O; se consideran despreciables las ramas paralelo de cada uno de los transformadores. Los tres transformadores se unen entre s formando sus primarios una conexin en tringulo y sus secundarios una conexin en estrella. El conjunto alimenta una carga equilibrada conectada en estrella de 2 L 45 O/fase. Si la tensin simple secundaria es igual a 200 V, la sucesin de fases es RST y se toma como referencia de fase secundaria la tensin Van' Determinar las expresionesfasoriales de: 1) las corrientes la' lb e le; 2) las corrientes 11,12 e 13; 3) las corrientes IR' t, e IT; 4) tensin VRS' VSTY VTW
II

M R

__.A A'
12

~
S

__.
N

11
B'

---.

IT

13

__. e
p C'

III

Figura 3.41.

CAPTULO 3.

TRANSFORMADORES

221

SOLUCIN 1) Resolveremos el problema para el transformador superior, dando las respuestas de los otros transformadores a base de hacer un corrimiento cclico de sus fases respecto del primario. As, se tiene: Van

= V = 200 L 0 V
00'

lo que da lugar a una corriente en la 1." fase secundaria: I

200 L 0 2 L45

100 L -45 A

y para las otras dos fases se tendr: lb = 100 L -165 A ; le = 100 L 75 A 2) La f.e.m. secundaria del primer transformador, de acuerdo con (3.24), ser:

es decir: Eoa'= 200 L 0 + (0,03 + jO,04) . 100 L -45

= 204,95 L

0,2 V

que corresponde a una f.e.m. primaria, de acuerdo con (3.25), de:


EAA, = m . Eoo' = 5 . 204,95 L 0,20 = 1.024,76 L 0,20 V

ya que m = 1.000/200 = 5. Adems, si se tiene en cuenta que se desprecian las corrientes de vaco (rama paralelo), la corriente primaria por fase del primer transformador, de acuerdo con (3.31), ser:
----

100 L -45 5

= 20 L -450 A

y para los otros dos (teniendo en cuenta el sentido de sucesin de fases):

3) Las corrientes primarias de lnea se obtendrn aplicando el primer lema de Krchhoff a los nudos M, N Y P. Para la fase R se tiene: IR = IJ - 13 = 20 L _45
0 -

20 L 75 = 20y13 L -75 = 34,64 L -75 A


0 0 0

que corresponden a las otras dos fases:

r, = 34,64 L -195

; Ir = 34,64 L 45

4) La tensin primaria del primer transformador se obtiene aplicando la l ." ecuacin (3.24), resultando: VAA' = EAA, + ZJ IJ = 1.024,76 L 0,2 + (0,75 + ji)
0

20 L -45 V
0

que conducen a un valor: VAA' = 1.049,5 LO

222

MQUINAS ELCTRICAS

que corresponde, de acuerdo con la Figura 3.41, a la tensin primaria V RS' y para las otras fases resultar: VST

1.049,5 L -120

; VTR

1.049,5 L +120

El lector puede comprobar que la tensin compuesta secundaria Vab = Vrs es igual a: Vrs = Vab = Vaa,
-

VW

=J3 . 200 L 30 = 346,4 L 30

que al comparar con la primaria correspondiente: VRS = 1.049,5 L 0 nos indica que la tensin secundaria de lnea (lado de B.T.) se adelanta 30 respecto a la tensin primaria de lnea. O de otro modo: la tensin secundaria de lnea se retrasa 330 respecto a la tensin primaria, lo que corresponde a un ndice horario 330/30 = 11, que el lector puede demostrar ms fcilmente por el mtodo grfico explicado en el epgrafe 3.11.1. EJEMPLO DE APLICACIN 3.11. CIRCUITO EQUIVALENTE MONOFSICO DE UN SISTEMA TRIFSICO La Figura 3.42 muestra el esquema de una instalacin trifsica equilibrada. Se dispone de un transformador de 50 kVA, conexin Dy I, relacin compuesta 15.000/380 V, con las siguientes tensiones relativas de cortocircuito: Cee = lO por lOO;cxec = 8 por 100, que alimenta por su secundario una carga equilibrada en estrella de 5 L 0 n/fase a travs de una lnea de impedancia 0,1 + j 0,2 n/hilo. Calcular: a) Parmetros Rc<" Xcc y Z del circuito equivalente aproximado del transformador reducido al primario. b) Si se aplica al primario una tensin trifsica equilibrada de 15 kV de lnea (lectura del voltmetro VI)' determinar las lecturas de los voltmetros VA y VB'
Transformador Dy 1 0,1 + )0,2 n/hilo

S o-

__;;.B..J

Figura 3.42.

SOLUCIN Los problemas de transformadorestrifsicosse pueden resolverpor dos procedimientos distintos: a) desde el punto de mquina elctrica, teniendo en cuenta la configuracin interna real fsica con la que se unen los devanados. Este procedimiento suele ser ms til cuando solamente se analiza el transformador; b) desde el punto de sistema elctrico de potencia completo.

CAPTULO 3.

TRANSFORMADORES

223

Este procedimiento tiene inters cuando el transformador forma parte de una red elctrica ms completa, que puede incluir lneas de alimentacin del transformador y de ste a las cargas. En el primer caso, el problema se resuelve de un modo similar al indicado en el ejemplo de aplicacin anterior 3. JO, es decir, se calculan las distintas tensiones y corrientes que tienen lugar en uno de los transformadores monofsicos que constituyen el conjunto trifsico. A partir de los resultados obtenidos para este transformador monofsico, se calculan las tensiones y corrientes de lnea teniendo en cuenta las conexiones elctricas que existen entre los tres transformadores monofsicos. Un procedimiento similar se utiliza en el caso de tratarse de transformadores trifsicos de tres columnas, calculando las tensiones y corrientes que tienen lugar en una sola columna del transformador y determinando a posteriori las tensiones y corrientes de lnea en funcin de las conexiones internas entre los arrollamientos. Este pomtodo es muy til para el ingeniero constructor del transformador, ya que tiene en cuenta la realidad fsica de cmo estn unidos entre s los devanados. Cuando el transformador es un componente ms de un sistema elctrico de potencia, el ingeniero no est tan interesado en lo que sucede dentro de cada columna del transformador sino en las modificaciones que ste produce en los niveles de corrientes y tensiones de la red. En esta situacin es ms prctico desarrollar un circuito equivalente monofsico de todo el conjunto: red de alimentacin, transformador, lneas e impedancias de carga: lo que se resuelve ms fcilmente si todas las fuentes (red), devanados del transformador y cargas se suponen conectados en estrella. En el caso de que algn componente de la red est realmente conectado en tringulo, deber obtenerse su equivalente en estrella de acuerdo con las reglas de transformacin tringulo-estrella que se utiliza en la teora de los circuitos trifsicos. Como aplicacin de estas ideas, se va a resolver este ejercicio de dos maneras distintas, para que el lector utilice el que ms le interese en funcin del tipo de problema que est resolviendo.

1. Primer procedimiento.
a)

Desde el punto de vista de mquina elctrica

Las tensiones simples de primario y secundario del transformador (valores asignados) son:
VI"

15.000 V ; VZIl

= J3 = 219,39 V

380

ya que el primario est en tringulo y el secundario en estrella. De este modo la relacin de transformacin entre tensiones simples ser:
15.000 m = -=--= VZIl 219,39

v,

68,37

La corriente asignada por fase primaria es: 50.000


SN =3
VI/l . fIn =>

fl" =3 . 15.000 = 1,11 A

Yteniendo en cuenta las definiciones (3.59) y (3.76) aplicadas a una fase del primario, resultar 0,1 = 15.000

z.; 1,11
=>

Zcc = 1.350 n

0,08 = 15.000

x.;

1,11
=>

Xcc = 1.080 n

224

MQUINAS ELCTRICAS

de donde se deduce:

Ree = JZ:c - X:, = 810 n

Figura 3.43. b)

El circuito equivalente por fase reducido al primario de la instalacin es el mostrado en la Figura 3.43, en el que se ha tomado como referencia de fases la tensin primaria (realmente quiere decir la tensin primaria de la 1: fase del transformador). En este circuito se tiene: Impedancia reducida de la lnea: m2 (0,1 + jO,2) = 467,45 + j934,9 n Impedancia reducida de la lnea de la carga: m2 5 LO = 23.372,6 + jO por lo que la corriente I~ valdr: l' =
2

15.000 LO = 0,606 L -4,67 A (810 + j 1.080) + (467,45 + j934,9) + 23.372,6

y en consecuencia las tensiones reducidas V~/ y V~/ sern:


v~/= [(467,45 + j934,9) + 23.372,6]' 0,606 L -4,67 = 14.458,2 L -2,42

V~/ = 23.372,6 . 0,606 L -4,67 = 14.163,8 L -4,67 que corresponden a unos mdulos:
V~/ = 14.458,2 V ; V~/ = 14.163,8 V

y en consecuencia las tensiones secundarias por fase sern:


, V~/ 14.458,2 VA/= - = ~ 211,5 V m 68,37 , V~/ 14.163,8 VB/= - = ~ 207,2 V m 68,37

que al estar conectado el secundario en estrella corresponde a unos valores de lnea: VA =

J3 . 211,5 ~

366,28 V ; VB

=J3 . 207,2 ~

358,9 V

que corresponden a las lecturas que sealarn los voltmetros de la Figura 3.42. Comprobar el lector que la resolucin de un problema de transformadores trifsicos por este procedimiento es anloga a la de los transformadores monofsicos. La clave consiste en visualizar una sola de las fases o columna del transformadorreal (parte del primario y secundario correspondiente).Este transformadorindividual se comporta exactamente igual que el transformadormonofsicoestudiadoen la primeraparte de este captulo.Para transformadorestrifsicos los clculos de impedancia, cada de tensin, rendimiento, etc., se efectan por fase, empleandoexactamentelas mismas tcnicas ya desarrolladaspara transformadoresmonofsicos.

CAPTULO

3.

TRANSFORMADORES

225

Para evitar errores en el planteamiento de las ecuaciones, conviene que se siga este procedimiento: 1) Obtener la relacin de transformacin como cociente de las tensiones asignadas simples de primario y secundario. 2) Determinar los parmetros por fase del transformador. 3) Dibujar el circuito equivalente de la instalacin reducida al primario, en la que se indiquen los valores de las potencias, tensiones y corrientes por fase. 4) Resolver el circuito equivalente, indicando los valores de las soluciones por fase. 5) Expresar las soluciones en valores de lnea, teniendo en cuenta para ello las conexiones especficas del primario y secundario del transformador. 2. Segundo procedimiento. Desde el punto de vista de sistema elctrico En esta situacin, y tal como ya se ha indicado en la introduccin al problema, conviene analizar la red en todo su conjunto, para lo cual hay que conseguir un equivalente monofsico de todo el sistema, lo que requiere transformar todos los elementos que estn conectados en tringulo en una estrella equivalente. En este problema solamente es el primario del transformador el que est conectado en tringulo, por lo que vamos a preparar un circuito equivalente al de la Figura 3.42 pero con el primario en estrella. En la Figura 3.44a se m~estra el esquema correspondiente y en la Figura 3.44b se representa el circuito equivalente por fase de todo el sistema: red de alimentacin, transformador y carga.
Transformador Yy
R 0--.----;;:> A

0,1 + jO,2 Q/hilo

equivalente

a)

So-------~

m'- 15.0001.[3 - 3801.[3

15.000 380

= 39,47
0,1 + jO,2 Q

b)

Cf ~ Cf
Figura 3.44.

~~n

En la figura anterior debe destacarse que la relacin de transformacin del transformador equivalente es diferente a la real calculada con el primer mtodo y que por ello denominamos mi y que es igual al cociente de la tensin asignada simple del primario en estrella VI,,). dividido por la tensin asignada simple del secundario en estrella V2m,, es decir:
mi =

V;n = 15.000/.)3 = 3947


V2n,

380/.)3

226

MQUINAS ELCTRICAS

Como observa el lector, con este procedimiento coinciden las relaciones de tensin tanto como cociente de tensiones simples como de tensiones compuestas. Por otro lado, la impedancia equivalente del transformador reducida al primario calculada en el caso anterior, cuando se consideraba el primario conectado en tringulo, era igual a:
Zccl,

= 810 + j 1.080

Sin embargo, la impedancia por fase reducida al primario del transformador equivalente estrella-estrella, de acuerdo con las reglas de conversin tringulo-estrella, ser: 810 + j 1.080 ----'3'---= 270 + j360

El circuito equivalente monofsico de la Figura 3.44b, al reducir el secundario al primario da lugar al indicado en la Figura 3.45, teniendo en cuenta que m' = 39,47. En este circuito se puede calcular la corriente 1 = I~, que vale:
1

15.000 --LO 1
1

= I: = (270 + j360)
2

+ 39,472

J3(0,1 + jO,2) + 39,47

5 LOo'

= 1 05 L -4 67 A

ZCCA = 270 + j360n

m'2(0,l + jO,2)

Figura 3.45.

yen consecuencia las tensiones reducidas V~fy V~f sern:


V~f= [39,472 (0,1 + jO,2) + 39,472 . 5 L OOJ 1,05 L -4,67 ~ 8.350,4 L -2,42 V~f = [39,472

5 L OOJ 1,05 L -4,67 ~ 8.180,3 L -4,67 V~f = 8.350,4 V ; V~f = 8.180,3 V

que corresponden a unos mdulos:

por lo que las tensiones secundarias por fase sern:


~f= -

V~f 8.350,4 = -= 211,5 V mi 39,47

VBf

V~f =-=
mi

--=

8.180,3 39,47

207,2 V

que corresponden a unos valores de lnea: VA =

J3 . 211,5 ~

366,3 V ; VB

=J3. 207,2 ~

358,9 V

que coinciden con los valores calculados con el primer procedimiento. El lector puede comprobar que este procedimiento tiene grandes ventajas analticas en la resolucin de problemas complejos en los que pueden existir lneas de alimentacin al trans-

CAPTULO 3.

TRANSFORMADORES

227

formador, cargas en estrella o tringulo, varios transformadores implicados en los clculos, lneas o feeders de alimentacin a las cargas, etc.

3.12. ACOPLAMIENTO EN PARALELO DE TRANSFORMADORES


Cuando aumenta la demanda en un sistema elctrico es necesario con frecuencia elevar la potencia de los transformadores de alimentacin. Antes que retirar una unidad antigua y sustituirla por otra nueva y de mayor potencia, es ms econmico disponer de un transformador adicional conectado en paralelo con el ya existente y cubrir as la demanda. Por otro lado, el consumo de un sistema elctrico puede variar fuertemente de unas pocas a otras del ao; si se proyecta la instalacin con un nico transformador, ste deber tener la potencia suficiente para poder suministrar la carga solicitada en los momentos de mayor demanda, y en esta situacin en las pocas de menor consumo el transformador trabajar con poca carga, por lo que su rendimiento ser bajo. Una forma de mejorar la instalacin elctrica ser colocar dos o tres transformadores ms pequeos en paralelo de tal forma que en los perodos de menor demanda funcione solamente uno de ellos y en pocas de mayor consumo se incorporan nuevas unidades en paralelo. Este procedimiento mejora notablemente el rendimiento de la instalacin, con la ventaja adicional de que si se produjera la avera de un transformador siempre cabe la posibilidad de alimentar la carga ms esencial con los otros transformadores. De ah la importancia del estudio del acoplamiento de transformadores en paralelo. Se dice que dos o ms transformadores funcionan en paralelo cuando sus bornes homlogos estn unidos entre s, tanto del lado primario como del secundario, por conexiones de resistencia y reactancia despreciables. Para poder realizar un perfecto servicio en paralelo se han de cumplir las condiciones siguientes: Los transformadores deben pertenecer al mismo grupo de conexiones (mismo ngulo horario). 2. Los transformadores deben tener las mismas tensiones primarias y secundarias (lo que indica la misma relacin de transformacin entre tensiones compuestas). 3. Los transformadores deben tener idnticas tensiones relativas de cortocircuito (Be). Las dos primeras condiciones son necesarias incluso en el funcionamiento en vaco, y de esta forma se evitan corrientes de circulacin entre ambos transformadores en ausencia de carga, debido a la no coincidencia de las fases o a la desigualdad de sus f.e.m.s. secundarias. La tercera condicin es necesaria adems para el correcto funcionamiento en carga, de forma que esta ltima se distribuya entre los transformadores proporcionalmente a sus potencias asignadas aparentes. En la prctica se admiten unas desviaciones mximas del 10 por 100 y se suele exigir que la relacin de las potencias asignadas de los transformadores conectados en paralelo no supere la relacin 3:l. En la Figura 3.46a se muestra el esquema convencional que representa dos transformadores en paralelo. En la Figura 3.46b se indica el circuito equivalente por fase, reducido al primario del sistema que se forma, donde por sencillez se ha omitido la rama de vaco. Analizando este circuito se observa que al ser comunes las tensiones primaria y secundaria, las cadas de tensin han de ser idnticas, es decir: (3.101) l.

228

MQUINAS ELCTRICAS

Al
VI

AlJ

A'I

TRAFO
1

TRAFO
11
jll

Zrc/ 1lJl

ZrclJ

alJ

\t

1
~ CARGA ~
Figura 3.46. Conexinde transformadoresen paralelo.

donde 11e Iu representan las corrientes que lleva cada transformador reducidas al primario. Si se denominan [In e IUn las intensidades asignadas de cada mquina, la expresin (3.10 1) se puede escribir tambin en forma modular:
11 ZCC/ln ---,100=---,100
[In Vln IlJ ZCC1!Un

i;

v;

(3.102)

11

que teniendo en cuenta las definiciones de ndice de carga (3.73) y tensin de cortocircuito (3.76), se puede poner tambin: (3.103) es decir:

eu

(3.104)
ecC!

lo que indica que los ndices de carga de cada transformador son inversamente proporcionales a las tensiones de cortocircuito. La 3.a condicin exigible para realizar el acoplamiento en paralelo indica que ecC! debe ser igual a eCCl/O lo que significa en la ecuacin (3.104)que los ndices de carga deben ser iguales, es decir, el reparto de potencias se realiza en forma proporcional a las potencias asignadas respectivas. Para acoplar en paralelo transformadores trifsicos con el mismo ndice horario se conectarn los bornes marcados con la misma letra a la misma lnea de la red, tanto en el lado de A.T. como en el de B.T., como se indica en la Figura 3.47. En el caso de que se desconozcan los ndices horarios (teniendo los transformadores la misma relacin de tensiones), se conectarn al azar los bornes de A.T. a la red de entrada, uniendo en la parte de la B.T. dos bornes cualesquiera entre s (por ejemplo, el cI con el cu); con un voltmetro se medirn las d.p.p. entre los terminales de B.T. restantes, y los bornes cuya d.d.p. sea cero sern bornes homlogos y se podrn conectar a la misma fas'ede la lnea; en el caso de que no se lograra este resultado, se unirn otros bornes de B.T. (por ejemplo, el

CAPTULO 3.

TRANSFORMADORES

229

R S T

T RAFO
1

C B A e b a ;>

e BA e b a

TRAFO

11

Figura 3.47. Conexin en paralelo de transformadores trifsicos del mismo ndice horario. c con b,,) y se seguir el procedimiento anterior; si no se llegara nuevamente a encontrar bornes homlogos se har otra combinacin en B.T., etc. Agotadas todas las posibilidades en el lado de B.T., se proceder a intercambiar entre s dos bornes de la parte de A.T. de uno de los transformadores y se proceder como antes; si an no se llegara a encontrar bornes homlogos, querr decir que los dos transformadores no pueden conectarse en paralelo (a no ser que se cambien las conexiones internas).

EJEMPLO DE APLICACIN 3.12 Dos transformadores monofsicos tienen las siguientes caractersticas:
TRANSFORMADOR TRANSFORMADOR 1: 1.000/100 V 100 kVA 200kVA
f.RCC = f.RCC

3 %,

f.RCC =

4%

f.CC

= 5%
5%

11: 1.000/100 V

= 3 %, f.xcc = 4 %

f.cc=

Se conectan en paralelo para alimentar una carga de 150 kVA, conf.d.p. 0,8 inductivo. Calcular: a) impedancias internas de ambos transformadores; b) reparto de corrientes, potencias activas y aparentes. NOTA: Se supone la tensin secundaria constante e igual a 100 V.
SOLUCiN a) Transformador I

La intensidad asignada del primario es: 11 =


n

100.000

1.000

= 100 A

y teniendo en cuenta las expresiones (3.76) se obtiene:


RCCf 100 0,03 = --=-=1. ooo::.__-

XCC 100 004=--, 1.000

de donde se deduce RcCf = 0,3 n; XCC = 0,4 n, que expresan los valores de la impedancia de cortocircuito (interna) del transformador 1 reducida al primario.

230

MQUINAS ELCTRICAS

Transformador 11 Operando de una forma similar, se obtiene:


RCCI/

== 0,15

n ;

XCCI/

= 0,2

b)

El circuito equivalente del acoplamiento en paralelo, reducido al primario, ser el indicado en la Figura 3.48. La corriente suministrada por el conjunto (reducida al primario), vale: IT
O,3Q

==

150.000 1.000

== 150 A

JOAQ

O,ISQ

jO,2Q

v; = mV2=

1.000 voltios

o----------------------------~
Figura 3.48.

Si se toma V~como referencia de fase, la intensidad anterior expresada en forma compleja ser:
IT == 150 L -370

La igualdad en las cadas de tensin de ambos transformadores se expresar: (0,3 + )0,4) I1 == (0,15 + )0,2) 11/ Como adems el primer lema de Kirchhoff en el nudo A indica que: II + 11/ == 150 L -37 se obtienen como soluciones:
1/==50L-3r ; 11/= 100L -370

En consecuencia, las potencias aparentes de cada transformado; ern:


SI = V~ll = 1.00050 == 50 kVA ; SI/

== 1.000100

==

l(j;

kVA

y las potencias activas:


PI == 50 cos 37

== 50 . 0,8 == 40 kW

PI/

== 100 cos 37

== 100 0,8 = 80 kW

siendo los ndices de carga:

e ==..?'~=~==~
I

51/

100

2'

. e _~_100_~
1/ -

S1111 - 200 - 2

este resultado est de acuerdo con lo previsto, ya que las cadas relativas de tensin ecc coinciden en ambos transformadores.

CAPTULO 3.

TRANSFORMADORES

231

3.13. AUTOTRANSFORMADORES
El autotransformador es un transformador especial formado por un devanado continuo, que se utiliza a la vez como primario y secundario, por lo que las tensiones de alimentacin y salida no van aisladas entre s. A diferencia del transformador de dos devanados, un autotransformador transfiere energa entre los dos circuitos, en parte por acoplamiento magntico y en parte por conexin elctrica directa. Para ver la comparacin con el transformador se va a considerar el esquema de la Figura 3.49; en a) se muestra un transformador de dos devanados separados y en b) un autotransformador, ambos con las mismas tensiones de entrada y salida. Se supondr que las dos mquinas son ideales y que no existen cadas de tensin, corriente de vaco, etc. Igualmente, se supone que son iguales los flujos en el ncleo y la densidad de corriente en los devanados. Se observa que el transformador necesita N espiras en el primario y N, espiras en el secundario; en cambio, el autotransformador slo emplea N espiras; en una parte de ellas, N-NZ' circula la corriente /1' mientras que por la otra, que es una derivacin que recoge Nz espiras, circula una corriente /z-/ en el sentido indicado en la Figura 3.49b. El peso en cobre en cada uno de los casos es proporcional al nmero de espiras y a la corriente que las recorre. (Se debe tener en cuenta que, fijando una densidad de corriente J, la seccin de los conductores es proporcional a la intensidad que circula por ellos, ya que J = l/S.) Teniendo en cuenta adems que se verifica: (3.105) la relacin entre el peso de cobre como autotransformador G" y como transformador G{ser:
-= G{

G"

I(N - Nz) + (/z -/) Nz Vz =1-IN + IzNz V

(3.106)

lo que indica un ahorro en material para el autotransformador frente al transformador. Por ejemplo, si V/V2 = 2, se obtiene una economa en el cobre de un 50 por 100. La reduccin en el nmero de espiras total permite tambin emplear circuitos magnticos con menos ventana, lo que supone un menor peso en hierro. El autotransformador tiene en consecuencia menos prdidas en el cobre y en el hierro, lo que mejora el rendimiento y la cada de tensin frente al transformador.
1

--+
N-Nz 12

-+

i
N a) Transformador

12-1

+vz

b) Autotransformador

Figura 3.49. Comparacin entre un transformador y un autotransformador.

232

MQUINAS ELCTRICAS

El autotransformador, sin embargo, presenta dos inconvenientes fundamentales: por una parte, debido a su menor resistencia y reactancia (al existir menos espiras), resulta un bajo valor de la cada eco lo que supone en caso de una falta de cortocircuito, de acuerdo con (3.63), corrientes ms elevadas; por otra parte, si la relacin de transformacin es muy elevada, por ejemplo, 10.000/220 Y, tiene el inconveniente (Fig. 3.50) de presentar un borne comn a los arrollamientos de A.T. y B.T. (A' = a'). Supngase, por ejemplo, que el terminal A por un accidente queda conectado a tierra; en consecuencia, los terminales de B.T. presentan en a' y a unas d.d.p. respecto de tierra de 10.000V Y 9.780 Y, respectivamente. Este inconveniente se puede evitar conectando a tierra el borne comn A' = a'; sin embargo, el problema puede volver a reproducirse si por un fallo se queda el circuito abierto entre a y a' (ya que el terminal a quedara al mismo potencial que A respecto de tierra, es decir, a 10.000 Y). Debido a estos graves inconvenientes, se suele limitar el uso de los autotransformadores a los casos en que no exista mucha diferencia entre las tensiones VI y V2. En cualquier caso, se est obligado conectar a tierra el borne comn de ambos devanados. En los laboratorios es muy frecuente tener un tipo de autotransformador de relacin variable, donde el terminal a se mueve mediante un cursor, lo que permite obtener diversas tensiones de salida a partir de una tensin primaria constante. El aparato se conoce vulgarmente por el nombre de variac (variable alternating current, es decir, corriente alterna variable).Tambin se construyen autotransformadores de tipo trifsico con conexin Yy.

3.14. TRANSFORMADORES CON TOMAS


En las redes de energa elctrica es necesario un control de la tensin por varios motivos: 1) para ajustar o regular la tensin de alimentacin o de suministro a los usuarios dentro de los lmites legales prescritos; 2) para controlar el flujo de potencia activa y reactiva en la red; 3) para ajuste de la tensin, de acuerdo con los correspondientes cambios en la carga. Estos cambios pueden ser de tipo estacional (5-10 por 100), diario (3-5 por 100) Y de corta duracin (1-2 por 100) y se realizan modificando la relacin de transformacin de los transformadores del sistema elctrico a base de cambiar el nmero de espiras por medio de tomas en los devanados. Los ajustes o modificaciones ocasionales se realizan con el transformador sin tensin. Los cambios diarios o de corta duracin se realizan con el transformador en carga.

Figura 3.50. Falta a tierra en un autotransforrnador.

CAPTULO 3.

TRANSFORMADORES

233

3.14.1. Tomas de regulacin


Para regular la tensin de un transformador monofsico se establecen diversas derivaciones, llamadas tomas, en uno de los arrollamientos (Fig. 3.51) Yal propio tiempo se dispone de un medio para conmutar. La recomendacin UNESA prescribe cinco posiciones de regulacin con tensiones por escaln del 2,5 por 100 y un lmite mnimo de 5 por 100. Es decir, +5 por 100, +2,5 por 100, O,-2,5 por 100, -5 por 100. Las tomas pueden preverse en uno u otro de los arrollamientos. En caso de trapsformadores trifsicos (Fig. 3.52) se pueden disponer las tomas en las entradas de las fases, o se puede variar NI por corrimiento del punto neutro. Desde el punto de vista econmico, no es indiferente el empleo de cualquiera de las posibles soluciones tcnicas por varios motivos: No es econmico disponer las tomas en baja tensin o, en general, en arrollamientos con tensiones relativamente bajas. Las derivaciones de los devanados y los contactos de los conmutadores deberan preverse para elevadas intensidades. Es ms econmico conmutar en alta. 2. Si ambos devanados son para alta tensin, desde el punto de vista del conmutador, pueden ser aceptables cualquiera de las dos soluciones. En tal caso, es preferible disponer la conmutacin en el arrollamiento correspondiente a la tensin variable. El objeto es poder proyectar el transformador con la mayor economa posible, es decir, trabajando en todos los casos con la misma induccin B en el ncleo.
1.

Un aspecto tcnico adicional que decide la colocacin de las tomas en el devanado de A.T. es que cuando se trata de transformadores con una tensin reducida en el lado de B.T., al tener entre las tomas un nmero entero de espiras, el escaln mnimo que puede conseguirse en B.T. puede ser superior al exigido. Por ejemplo, considrese un transformador con una tensin de 260 V/fase y 15 V/espira: si se ponen tomas entre dos espiras consecutivas se obtiene un cambio relativo de: 15 -= 5,77% 260 que representa el escaln mnimo posible y supone un cambio sustancial en la variacin de tensin del transformador. Es preciso destacar que, si un arrollamiento con tomas tiene una seccin constante (todas sus tomas estn previstas para la mxima intensidad asignada [1)' la potencia del transforma-

Figura 3.51. Transformador con tomas en el primario o en el secundario.

234

MQUINAS ELCTRICAS

Figura 3.52. Transformadores trifsicos con tomas.

dor ser diversa para cada toma. Por ejemplo, si se tienen tres tomas: +5 por 100, O,-5 por 100, en la toma superior la potencia que puede dar el transformador es 1,05 V"/n para la toma normal se tendr Vn/Il y para la toma inferior 0,95 Vn/n' (Puede proyectarse el transformador con secciones escalonadas, de forma que las tomas estn preparadas para intensidades inversamente proporcionales a las tensiones. As se tienen los transformadores de potencia constante, frente a los anteriores de intensidad constante). Se entender por potencia asignada de un transformador la correspondiente a la toma normal. sta determinar la relacin de transformacin asignada m. En principio, puede establecerse cualquier nmero de tomas, y as: 30.000 V 2,5 % 5% - 7,5 %/380 - 220 V que representa un campo de regulacin desde +5 por 100 a -7,5 por 100. Desde el punto de vista econmico, conviene no excederse innecesariamente en el margen de regulacin, ya que encarece el precio del transformador. Adems existe otra razn de seguridad en relacin a los cortocircuitos. Un aspecto importante a tener en cuenta es la situacin de las tomas frente a la posicin geomtrica de los devanados. Conviene que las tomas se realicen (Fig. 3.53a) en la parte central del devanado de A.T. del transformador (a base de modificar la distancia entre los dedos de las tomas), pues con esto se logra reducir los esfuerzos axiales, que pueden ser muy fuertes en casos de cortocircuitos. En la Figura 3.53b se muestra un esquema de conexin incorrecto, ya que se producen por asimetra fuertes esfuerzos axiales que en el caso a) quedan contrarrestados.

3.14.2. Elementos de conmutacin


El paso de una tensin a otra puede realizarse en forma simple (econmica) o en forma ms prctica. Las modalidades de conmutacin son: l. Con transformador sin tensin. 2. Con transformador en carga.

CAPTULO 3.

TRANSFORMADORES

235

a)

CORRECTO

b) INCORRECTO

Figura 3.53.

Situacin geomtrica de las tomas.

Lo ms econmico es disponer los extremos de las tomas hasta un tablero en el interior de la caja o cuba del transformador. accesible mediante un registro. Una mejora sin gran encarecimiento consiste en hacer accesibles las diversas tomas desde el exterior (tomas sobre tapas). Puede lograrse disponiendo tantos juegos de aisladores sobre tapa como derivaciones existan, o bien dando salida por un mismo pasatapa a todas las derivacionesde una misma columna, dadas las escasas tensiones que suelen existir entre tales tomas. Cabe una solucin mixta entre las anteriores, es decir, de las diversas tomas sobre tablero conectar, con salida al exterior, aquellas que, en un sector de trabajo determinado, se cree que van a tener ms utilizacin. Otra mejora consiste en disponer conmutadores que permitan la maniobra desde el exterior, sin tensin. Su cuerpo de conmutacin suele estar sumergido en el aceite de la caja, ya que no realiza cortes de corriente. En ocasiones no es tolerable la interrupcin del servicio. En otras, dada la frecuencia con que se prev realizar los cambios de tomas, es conveniente tener que proceder a previas interrupciones. Existen casos en que las conmutaciones de tensiones se realizan, normalmente, en el proceso de trabajo. Interesa, por consiguiente, conmutadores o reguladores de tensin bajo carga. La necesidad de no interrumpir el paso de corriente (carga) crea el problema que se muestra en la Figura 3.54. Al estar el contacto mvil pasajeramente puenteando unas espiras del arrollamiento, se crea localmente en ellas un cortocircuito.La forma de evitarlo se ve en b). Se ha representado la operacin de conmutacin en tres tiempos. Los dos contactos mvi-

Figura 3.54. Detalle del funcionamiento del cambiador de tomas en carga.

236

MQUINAS ELCTRICAS

les estn unidos a travs de una impedancia (puede ser una bobina o incluso una resistencia, dado el breve tiempo de conmutacin) de valor reducido pero suficiente para moderar la corriente de cortocircuito. El proceso completo de la conmutacin est mecnicamente establecido, de forma que sea brusco y completo,es decir, independientede la voluntad del operador. Al existir cortes de arco, se producirn alteraciones del aceite. Esto hace necesario disponer la conmutacin en un departamento con aceite separado del resto. Es preciso evitar la difusin de los productos del arco.

3.15. TRANSFORMADORES DE MEDIDA


En la prctica de la Electrotecnia es frecuente el empleo de tensiones y corrientes elevadas que conviene medir. Sin embargo, no es posible proceder a una conexin directa de los aparatos de medida a los circuitos de A.T., debido al peligro que presentara el personal que se acercara a los instrumentos. Los transformadoresde medida permiten separar ambos circuitos, adaptando al mismo tiempo las magnitudes de la red a la de los instrumentos, que generalmente estn normalizados en unos valores de 5 A para los ampermetros y 110 V para los voltmetros (a plena escala). Para la conexin de los ampermetros y bobinas amperimtricas de los instrumentos se emplean los denominados transfonnadores de corriente, mientras que para la conexin de los voltmetros y bobinas voltimtricas se emplean los transformadores de tensin. Unos y otros deben garantizar el aislamiento necesariopara la seguridaddel personal, y adems deben reproducir con la mayor precisin las magnitudes primarias (de acuerdo con la relacin de transformacin correspondiente),para no dar lugar a errores en las medidas efectuadas con los aparatos. Las caractersticas que definen un transformadorde medida vienen recogidas en la Norma UNE 21088.

3.15.1. Transformadores de tensin


Su forma de funcionamientoy conexin es anloga a la de los transformadoresde potencia En la Figura 3.55 se muestra su conexin en paralelo con la red alimentando un voltmetro (en general, a su secundario se conectan en paralelo todas las bobinas voltimtricasde los aparatos de medida). A diferencia con los transformadoresde potencia, debido a la alta impedancia de la carga conectada, el transformadorde tensin funciona casi en vaco, y de esta forma la cada de tensin interna es muy pequea. Adems, y como muestra la Figura 3.55, un borne de su secundario debe conectarse a tierra para prevenir el peligro de un contacto accidental entre primario y secundario. El transformador de tensin debe proporcionar una tensin secundaria proporcionala la tensin primaria. Si se tiene en cuenta la relacin existente entre ambas tensiones y la corriente de carga, de acuerdo a la expresin (3.72) resulta:
VI - V~ = RcJ~ cos cp

+ XJ~ sen cp

(3.107)

Idealmente conviene que se cumpla: (3.108)*

* Al hablar de transformadores de medida de tensin se prefiere denominar a la relacin de transformacin K, en lugar de m, que se utiliza en transformadores de potencia.

CAPTULO 3.

TRANSFORMADORES

237

RED

f'v

CARGA

VOLTMETRO

Figura 3.55. Transformador de tensin alimentando un voltmetro. lo que indica, de acuerdo con (3.107), que debe procurarse que las cadas de tensin sean lo ms mnimas posibles. Por ello estos transformadores deben presentar una reactancia pequea y una comente de carga reducida, lo cual obliga a disear el sistema con poco flujo de dispersin y un circuito magntico que necesite una pequea corriente de vaco. Las tensiones primarias de los transformadores de tensin estn normalizadas en los valores asignados siguientes: 110, 220, 385, 440, 2.200, 3.300, 5.500, 6.600, 11.000, 13.200, 16.500, 22.000, 27.500, 33.000, 44.000, 55.000, 66.000, 110.000, 132.000, 220.000, 396.000. La tensin secundaria asignada est normalizada en 110 voltios. Los transformadores de tensin se definen segn su carga asignada en VA, denominada tambin potencia de precisin, que es la mxima carga que puede conectarse a su secundario (teniendo en cuenta tambin la potencia disipada en los hilos de conexin), para que el error de la medida est comprendido dentro de los mrgenes indicados por el constructor. Los valores normales de la potencia de precisin para un f.d.p. de 0,8 son: 10, 15,25,30,50,75, 100, 150, 200, 300, 400, 500 VA. Al hablar de transformadores de medida de tensin tambin se definen los siguientes tipos de errores:
a)

Error de relacin o de tensin: que indica la desviacin porcentual de la tensin realmente existente en el secundario, V2, con respecto a la que debera existir si el transformador fuera ideal, V/Kv, es decir:

ev =

V2 - V/Kv VI

. 100

(3.109)

b)

Error de fase o de ngulo: que es la diferencia de fase existente entre los vectores VI y V2, y se mide en minutos. Este error tiene importancia cuando se trata de medir la energa (por ejemplo, en el caso de que el secundario del transformador de tensin alimenta la bobina voltimtrica de un vatmetro o un contador, ya que entonces altera el valor real del ngulo formado por VI e 11),

De acuerdo con estos errores, se dispone en la prctica de las siguientes clases de precisin: 0,1; 0,2; 0,5; 1;3, que indican el valor mximo de s, permitido cuando el transformador se carga con su potencia asignada y un f.d.p. de 0,8 inductivo (en la prctica se indica tambin el valor del error de fase a que corresponde, en minutos). En la Tabla 3.4 se indican los valores de los consumos en VA de los aparatos de medida ms frecuentes.

238

MQUINAS ELCTRICAS

Tabla 3.4.

Consumosen VA de los aparatos de medida que se pueden conectar a un trafo de tensin

Voltmetros: Indicadores Registradores Vatmetros: Indicadores Registradores Fasmetros: Indicadores Registradores Contadores Frecuencmetros: Indicadores Registradores Rels: de mxima tensin temporizados de mxima tensin direccionales de mnima tensin de distancia

2-6 10-25

1-4 3-15

4-5
15-20 3-5

1-5 10-15

10-15 25-35 25-40 5-15 10-30

3.15.2. Transformadores de corriente


Tienen por misin reducir la corriente de la red a valores ms apropiados a las escalas de los instrumentos(5 A en general ya veces tambin l A a plena escala). Se conectan en serie con la lnea. En la Figura 3.56 se muestran los dos tipos de smbolos ms empleados. En el secundario se conectan en serie los ampermetros y bobinas amperimtricas de los aparatos de medida. Debido a la baja impedancia de estos aparatos, los transformadores de intensidad trabajan prcticamente en cortocircuito, por ello se emplean bajas inducciones en

RED

Av
TRAFODE INTENSIDAD

CARGA T.!.

CARGA

AMPERMETRO AMPERMETRO Figura 3.56. Transformador de intensidad. Forma de conexin y representacin.

CAPTULO 3.

TRANSFORMADORES

239

el ncleo 3.000 gauss, es decir, 0,3 Teslas). Segn indica la frmula (3.12), la relacin entre las corrientes secundaria, primaria y de vaco es: , -~ I1 = I~ + lo ; 1 2Idealmente se debe cumplir: (3.111) donde se ha denominado K a la relacin entre las corrientes asignadas de primario y secundario. Para que se cumpla (3.111) es preciso que el circuito magntico est muy bien diseado para hacer despreciable lo frente a I1 o I~. Sin embargo, en este tipo de transformador es interesante observar que no influyen en la medida los posibles flujos de dispersin. De ah que se empleen transformadores como indica la Figura 3.57a, donde se muestra un primario que se reduce a un solo conductor que pasa por el interior del ncleo magntico. En B.T. se utiliza tambin, por su comodidad, un tipo de transformador de intensidad denominado de tenaza o de pinzas (Fig. 3.57b), que permite medir la corriente de una lnea sin necesidad de interrumpir su funcionamiento, a base de abrir las tenazas e introducir dentro el cable conductor, que actuar como devanado primario (1 espira); el arrollamiento secundario alimenta un ampermetro incorporado generalmente en el mismo aparato, existiendo la posibilidad de obtener varias escalas conmutando el juego de espiras de este devanado. En los transformadores de intensidad, la f.m.m. primaria est contrarrestada por la f.m.m. secundaria; si por cualquier circunstancia se abriera este ltimo circuito, de acuerdo con (3.110) se tendra I1 = lo; es decir, toda la f.m.m. del primario se convertira en f.m.m. de excitacin, la cual no ha variado porque I1 es una corriente que depende de la red, y esto hace crecer peligrosamente el flujo, lo cual lleva consigo un aumento en las prdidas en el hierro y en las tensiones del secundario (peligrando la vida de los aislantes y la seguridad del personal). En consecuencia, debe evitarse terminantemente dejar en circuito abierto un transformador de intensidad. Si se desea cambiar una carga (por ejemplo, el ampermetro), se tienen dos soluciones: o se interrumpe el servicio de la lnea para proceder al cambio necesario, o se puede realizar la operacin sin desconectar la red, si previamente se cortocircuita el secundario del transformador de intensidad. Si los transformadores de intensidad se conectan a redes de A.T. es preciso unir uno de los bornes secundarios a tierra para prevenir el peligro de contacto accidental entre los devanados primario y secundario. Las corrientes primarias de los transformadores de intensidad estn normalizadas en los valores asignados siguientes: 5, (3.110)

In

RED

"v
TRAFODE INTENSIDAD

CARGA

RED

"v

CARGA

TENAZAS AMPERMETRAS

AMPERMETRO
a) Trafo de intensidad con N

=J

b) Tenazas amperimtricas

Figura 3.57. Tipos de transformadoresde intensidad.

240

MQUINAS ELCTRICAS

10, 15,20,25,30,50, 75, 100, 150,200,300,400,500,600, 750, 1.000,2.000,3.000,4.000, 6.000, 10.000 amperios. La corriente secundaria asignada est normalizada en 5 A y tambin se utiliza la escala de 1 A. Los transformadores de intensidad, de forma anloga a los de tensin, se definen segn sea el valor de su potencia asignada en VA (existen de 10, 15, 30, 50, 75, 100, 150, 200, 300 y 400 VA), que es la mxima carga que puede conectarse a su secundario para que el error de la medida est comprendido en los valores indicados por el fabricante. y as se definen:
a)

Error de intensidad o de relacin: que indica la desviacin porcentual de la corriente realmente existente en el secundario; 12, con respecto a la que debera existir si el transformador fuera ideal, l/K;, es decir:
8;

12 - LfK

1, K;

100

(3.112)

b)

Error de fase: que es la diferencia de fase existente entre los vectores 1, e 12 y se expresa en minutos.

Las clases de precisin son idnticas a las indicadas para los transformadores de tensin, y las potencias consumidas por los aparatos de medida ms frecuentes se indica en la Tabla 3.5. En la prctica los transformadores de corriente y tensin se encuentran combinados en las instalaciones para medir: intensidad, tensin, potencia, energa, etc. En la Figura 3.58 se muestra una disposicin tpica (se ha elegido una red monofsica para dar mayor sencillez al esquema). Se observa que las bobinas amperimtricas estn conectadas en serie, mientras que
Tabla 3.5. Consumos en VA de aparatos de medida que se conectan en serie

Ampermetros: Indicadores Registradores Vatmetros: Indicadores Registradores Fasmetros: Indicadores Registradores Contadores Rels: de sobreintensidad, de sobreintensidad, direccionales diferenciales de distancia

0,25-2 1,5-9

1-4 1,5-8

2-6 6-12 0,5-3

instantneos temporizados

1-10 1-5 1,5-10 3-12 6-20

CAPTULO 3.

TRANSFORMADORES

241

RED

"v

CARGA

TRAFODE TENSIN

CONTADOR DE

Figura 3.58.

Transformadores de intensidad y tensin en un equipo de medida.

las voltimtricas van dispuestas en paralelo. Para mayor claridad se ha dibujado con trazo ms grueso el circuito amperimtrico. En las instalaciones elctricas existen tambin transformadores de intensidad y tensin que se emplean para alimentar aparatos de proteccin automtica (rels) de los diversos equipos del sistema: generadores, transformadores, etc.

3.16. APNDICE: TRANSFORMACIONES ESPECIALES


Como complemento a este captulo se exponen a continuacin dos tipos de transformaciones que se han utilizado o se utilizan actualmente en la prctica para modificar el nmero de fases de una instalacin empleando transformadores.

3.16.1. Transformacin trifsica a bifsica y viceversa. Conexin Scott


Es una conexin especfica de dos transformadores monofsicos para convertir un sistema de tensiones trifsicas en bifsicas y viceversa. Esta disposicin se debe al ingeniero americano Charles F. Scott, quien la invent en 1894 cuando trabajaba en la Compaa Westinghouse y se basa en el hecho bien conocido de que en un sistema trifsico en estrella la tensin compuesta entre dos fases est en cuadratura con la tensin simple de la tercera fase (por ejemplo, VST forma 90 con VRN). En la Figura 3.59 se muestra el esquema de conexiones y el diagrama fasorial de tensiones. Se dispone de dos transformadores monofsicos con el mismo nmero de espiras secundarias N2, mientras que el nmero de espiras primarias estn en la misma relacin que la altura y la base de un tringulo equiltero; es decir, de acuerdo con la Figu-

242

MQUINAS ELCTRICAS

TRANSFORMADOR A

N2
TRANSFORMADOR B

Figura 3.59. Transformacinde trifsica a bifsica. Conexin Scott.

ra 3.59, el transformador A tiene

J3 NI espiras, mientras que el transformador B tiene NI 2


m =--a N2

espiras en su primario. De este modo las respectivas relaciones de transformacin son:

fiN 2

. m -_
' b -

NI

N2

(3.113)

Si se denomina VL a la tensin compuesta del sistema trifsico, y se toma como referencia de fases la tensin compuesta VSTo el valor complejo de la tensin primaria aplicada al transformador A es: (3.114) y de este modo el valor complejo de la tensin secundaria del transformador A ser:

~ V L 90

NI

(3.115)

De un modo similar, el valor complejo de la tensin primaria aplicada al transformador B es la tensin compuesta trifsica VSTo cuya expresin fasorial, de acuerdo con la Figura 3.59, es: (3.116)

CAPTUW

3.

TRANSFORMADORES

243

y de este modo el valor complejo de la tensin secundaria del transformador B ser:

VI/ VL L 0 V2b=-=-=--mb NI

NI

V LO
L

(3.117)

N}
De acuerdo con los resultados (3.115) y (3.117), se obtienen dos tensiones secundarias de igual mdulo y desfasadas entre s 90; es decir, se ha transformado el sistema trifsico simtrico primario en un sistema bifsico secundario simtrico. En el caso de que interese la inclusin de un hilo neutro en el primario, se puede realizar mediante una toma en el punto N del primario del transformador A, y que de acuerdo con la construccin fasorial deber situarse a l/3 de la altura (baricentro del tringulo de tensiones), lo que significa que el tramo MN tiene 1/3 de las espiras totales del tramo RM. La conexin Scott se emple inicialmente para transformar las corrientes bifsicas de los alternadores de la central del Nigara (instalacin que fue la primera en EE.UU. en la que se utiliz la c.a. y construida por la Compaa Westinghouse, que se inaugur en 1896) en un sistema trifsico. Se ha utilizado tambin la conexin Scott en traccin elctrica.,para alimentar dos tramos monofsicos de una red ferroviaria, de tal modo que si los consumos monofsicos de los trenes son iguales, sus corrientes se distribuyen por igual en las tres fases del sistema trifsico de entrada (primario). En la actualidad este tipo de montaje ha cado en desuso; de hecho, sirva como ejemplo actual el sistema que se utiliza para la alimentacin de los ferrocarriles espaoles de alta velocidad (AVE), que tienen una potencia por tren de 8.800 kW y utilizan centros de transformacin trifsicos con una entrada de 132 kV (e incluso de 220 kV y 380 kV) y que por medio de dos transformadores monofsicos convencionales de 20 MVA alimentan a 25 kV dos catenarias distintas, para reducir al mximo los desequilibrios en el primario; la alimentacin con estas elevadas tensiones hace que el impacto de los desequilibrios en la red primaria trifsica sea muy reducido. Los centros de transformacin de estas lneas del AVE estn separados entre s de 40 a 50 km.

3.16.2. Transformacin trifsica a hexafsica


Un sistema hexafsico consiste en seis tensiones desfasadas entre s 60. Existen varios procedimientos para obtener un sistema hexafsico a partir de un esquema trifsico. En la Figura 3.60a se muestra uno de los ms utilizados. Se parte de un transformador de ncleo trifsico; el devanado primario est conectado en tringulo y el devanado secundario es una doble estrella. En la Figura 3.60b se muestran los fasores de las tensiones de cada una de las fases para primario y secundario. Tngase en cuenta para realizar esta construccin que todos los devanados de la misma columna tienen sus tensiones en fase, con las polaridades que prescribe el criterio de punto, es decir, los terminales que llevan punto tienen la misma polaridad instantnea. As, en la primera columna la tensin primaria es VRS' por lo que la tensin secundaria Vrn estar en fase con VRS' mientras que la tensin secundaria Vr 'n estar en oposicin de fase respecto a VRS' Yas se construyen los fasores de las dems tensiones del diagrama mostrado en la Figura 3.60b. Este tipo de transformadoresse utiliza en instalaciones rectificadoras para convertir la c.a. de una red de distribucin de energa elctrica en C.C., como por ejemplo: instalaciones de electrlisis, traccin elctrica, instalacionesde c.c. en alta tensin (HVOC: high voltage direct current), ya que tienen la ventaja de que al aumentar el nmero de fases se reduce el nmero de armnicos. Para instalaciones de mucha potencia se utilizan tambin sistemas dodecafsi

244

MQUINAS ELCTRICAS

a) Devanado del transformador

b) Diagramas fasoriales de tensiones

Figura 3.60. Transformacin de trifsica en hexafsica. La conversin trifsica a dodecafsica se puede conseguir fcilmente mediante dos transformadores trifsicos, uno idntico al de la Figura 3.60a, es decir, con conexin tringulo-doble estrella, y el otro con conexin estrella-doble estrella. De este modo, debido a que uno de los primarios est en estrella y el otro en tringulo, se consigue un desfase de 30 entre el juego de los dos sistemas hexafsicos secundarios, por lo que se obtienen doce tensiones secundarias separadas entre s 30, lo que reduce enormemente el rizado de la C.C. obtenida a la salida de los rectificadores conectados a los secundarios. En el Captulo 7, dedicado a los accionamientos elctricos, se dan ejemplos del uso de estos transformadores en circuitos rectificadores.
COSo

PROBLEMAS
3.1. Un transformador monofsico de 100 kVA, 3.000/220 V, 50 Hz, tiene 100 espiras en el devanado secundario. Supuesto que el transformador es ideal, calcular: a) corrientes primaria y secundaria a plena carga, b) flujo mximo, c) nmero de espiras del arrollamiento primario. [Resp.: a) 33,33 A, 454,55 A; b) 9,91 . 10-3 Wb; c) 1.364 espiras aprox.] Un transformador monofsico de 10 kV A, 220/380 V, 50 Hz, ha dado los siguientes resultados en unos ensayos: Vaco: 220 V, 2 A, 150 W (medidos en el lado de B.T.). Cortocircuito: 10 V, 26,32 A, 75 W (medidos en el lado de A.T. ). Calcular: a) parmetros del circuito equivalente del transformador reducido al primario. b) Si el primario se alimenta a 220 V, calcular la tensin secundaria cuando el transformador funciona a plena carga con f.d.p. 0,8 inductivo. [Resp.: a) RFe = 323,5 n, XII = 117,02 n; Ree = 0,036 n; Xee = 0,122 n; b) 372

3.2.

v:

3.3.

Un transformador monofsico de 125 kV A, 3.000/380 V, 50 Hz, ha dado los siguientes resultados en unos ensayos: Vaco: 3.000 V, 0,8A, 1.000 W (medidos en el primario). Cortocircuito: 10 V, 300 A, 750 W (medidos en el secundario). Calcular: a) Componentes de la corriente de vaco, b) Potencia de prdidas en el hierro y de prdidas en el cobre a plena carga. c) Rendimiento a plena carga con f.d.p. unidad, 0,8 inductivo y 0,8 capacitivo. d) Tensin secundaria a plena carga, con f.d.p. anteriores (se supone que al primario se le aplica la tensin asignada de 3.000 V). I [Resp.: a) IFe = 0,334 A, IJ1 = 0,727 A; b) PFe = 1.000 W, r; = 901,8 W; c) 98,5 %, 98,13 %,98,13 %; d) 377,26 V, 371,47 V, 384,14 v.i

CAPTULO 3.

TRANSFORMADORES

245

3.4.

Un transformador monofsico de 75 kVA, 3.000/220 V, 50 Hz, necesita 200 V aplicados al primario para que circule la corriente asignada en cortocircuito, siendo la potencia absorbida en el ensayo de 2 kW. Determinar: a) Cada de tensin relativa y tensin secundaria correspondiente cuando trabaja a plena carga con f.d.p. unidad, 0,8 inductivo y 0,8 capacitivo. b) Si la potencia absorbida en vaco es de 1,5 kW, calcular el rendimiento a plena y media carga con f.d.p. 0,8. [Resp.: a) 2,67 %, 214,13 V; 5,81 %, 207,21 V; -1,55 %, 223,4 V; b) 94,49 %, 93,75 %.]

3.5. Un transformador monofsico de 20 kVA, 460/200 V, 50 Hz, tiene unas prdidas en el hierro a la tensin asignada de 360 W y unas prdidas en el cobre a plena carga de 500 W. Calcular: a) Rendimiento a media carga con f.d.p. 0,8. b) Potencia aparente de mximo rendimiento. e) rendimiento mximo cuando el f.d.p. es la unidad. [Resp.: a) 94,28 %; b) 16,97 kVA; e) 95,93 %.] 3.6. El rendimiento para un factor de potencia unidad de un transformador monofsico de 200 kVA, 3.000/380 V, es de 0,98 tanto para la plena carga como para media carga. El f.d.p. en vaco es de 0,2 y la cada de tensin relativa a plena carga, con un f.d.p. 0,8 inductivo, es del 4 por 100. Determinar los parmetros del circuito equivalente del transformador reducido al primario. [Resp.: RFe = 6,61 kn; XI' = 1,36 kn; Ree = 0,61 n; Xcc = 2,19 n.] 3.7. El rendimiento mximo de un transformador monofsico de 500 kVA, 3.300/500 V, 50 Hz, es del 97 por 100 y ocurre para los 3/4 de la plena carga con f.d.p. unidad. Se observa en un ensayo de cortocircuito que son necesarios 330 V aplicados al primario para que circule en ese estado la corriente asignada por el transformador. Calcular la cada relativa de tensin a plena carga con f.d.p. 0,8 inductivo. [Resp.: 7,51 %.] 3.8. Calcular los rendimientos de un transformador de 100 kVA para media carga, plena carga y uno y un cuarto de la plena carga, con f.d.p.: a) unidad, b) 0,8. Las prdidas en el cobre a plena carga son de 1.000 W y las prdidas en el hierro son de 1.000 W. [Resp.: a) 97,56%, 98,04%, 97,98%; b) 96,96%, 97,56%, 97,50%.] 3.9. El rendimiento de un transformador monofsico de 100 kVA es de 93,02 por 100 cuando suministra la plena carga con un f.d.p. de 0,8 y de 94,34 por 100 a media carga con f.d.p. unidad. Calcular: a) Prdida en el hierro. b) Prdida en el cobre a plena carga. [Resp. a) 2 kW; b) 4 kW.] 3.10. Dos transformadores monofsicos de lOOkVA, 1.000/100 V, 50 Hz, funcionan en paralelo. Las impedancias de cortocircuito reducidas al primario de cada uno son Zcel = 0,3 + j 0,4 n y Zccll = 0,4 + j 0,3 n, respectivamente. Se desea alimentar a 100 V una carga de 150 kVA con f.d.p. 0,8 inductivo. Calcular las corrientes, potencias aparentes y activas suministradas por cada transformador. [Resp.: II = 111 = 757,6 A; Sil = SIl = 75,76 kVA ; PI = 53,57 kW; PII = 66,43 kW.] 3.11. Dos transformadores de 100 kVA, 1.000/100 V, 50 Hz, funcionan en paralelo. Los ensayos de cortocircuito de estos transformadores cuando funcionan con corriente asignada con los devanados de B.T. en cortocircuito dan los siguientes resultados:

246

MQUINAS ELCTRICAS

11

30 voltios 90 voltios

1.200 vatios J .800 vatios

a) Si se desea alimentar a 100 V una carga de lOOkW con f.d.p. 0,8 inductivo, cul ser el reparto de potencias aparentes y activas en cada transformador? b) Cul es la mayor potencia, con f.d.p. unidad, que pueden llevar los dos transformadores en paralelo sin sobrecargar ninguno de ellos? . [Resp.: a) 75,31 kVA, 67,5 kW, 25,1 kVA, 17,5 kW; b) 132,78 kVA.]

3.12. Un transformador de 40 kVA, 1.000/100 V, ha dado los siguientes resultados en un ensayo de cortocircuito: 51 V, 40 A, 400 W (medidas en el lado de A.T.). Se desea conectar en paralelo con otro transformador de 20 kVA, 1.000/100 V, que en un ensayo de cortocircuito ha dado: 42 V, 20 A, 245 W (medidas en el lado de A.T.). Indicar cmo se repartirn una potencia de 60 kVA con f.d.p. 0,8 inductivo. [Resp.: Transformador 1: 37,46 kVA, 29,15 kW; Transformador II: 22,72 kVA, 18,85 kW. Este transformador ir sobrecargado.] 3.13. Un transformador monofsico de 500 kVA, 15.000/3.000 V, 50 Hz, ha dado los siguientes resultados en unos ensayos: Vaco: 15.000 V, 1,67 A, 4.000 W (medidos en el lado de A.T.). Cortocircuito: 750 V, 33,33 A, 10.000 W (medidos en el lado de A.T.). a) Calcular los parmetros del circuito equivalente del transformador reducido al primario. b) Calcular las cadas relativas de tensin BR. ' Bx ' Bee. c) Hallar el rendimiento del transformador cuando funciona a plena carga cn f.d.p. 0,8 inductivo. d) Calcular la potencia aparente de mximo rendimiento del transformador y el rendimiento mximo para un f.d.p. unidad. e) Si se aplican 15.000 V al primario y se conecta una carga en el secundario que absorbe 100 A con f.d.p. 0,8 inductivo, cul ser el valor de la tensin secundaria?, y la cada relativa de tensin? f) Contestar a la pregunta anterior en el caso de que la carga absorba los 100 A con f.d.p. 0,8 capacitivo. g) Si se produce un cortocircuito en el secundario del transformador, cul sera el valor de la corriente primaria en rgimen permanente? h) Se acopla en paralelo este transformador con otro de potencia doble, con la misma relacin de la transformacin 15.000/3.000 V Y la misma cada relativa Bee pero su componente resistiva BR es mitad que la del transformador original. Si se desea alimentar una potencia totl de 1.000kVA con f.d.p. 0,6 inductivo, cules sern los valores de las potencias activas y aparentes suministradas por cada transformador? Cules seran los rendimientos de ambos transformadores en este caso si el segundo de ellos tiene una potencia de prdidas en el hierro de 6 kW? [Resp.: a) RFe = 56,18 kn, XII = 9,1 kQ, Ree = 9 n, x; = 20,63 n, b) BRee = 2%; ex = 4,58%; Bee = 5%. c) 96,5%; d) 316,22 kVA; 97,53%; e) 2.921,74 V; f)'i020,66 V; g) 666 A; h) Transformador 1:335,1 kVA, 236,5 kW, 96,5%; Transformador II: 670,2 kVA, 363,5 kW, 97,2%.] 3.14. Los ensayos de vaco y cortocircuito de un transformador monofsico de 10 kVA, relacin 2.000/200 V, han dado los siguientes resultados: Vaco: Medidas realizadas en el lado de B.T. (secundario). Potencia consumida en vaco = 300 'W; Tensin aplicada = 200 V; Corriente absorbida = 5 amperios. Cortocircuito: Medidas realiza-

CAPTULO 3.

TRANSFORMADORES

247

das en el lado de A.T. (primario). Potencia absorbida = 500 W; Tensin aplicada = 140 V; Corriente absorbida = asignada. Determinar: a) Parmetros del circuito equivalente aproximado por fase del transformador reducido al primario. b) Si se conecta el primario del transformador a una red monofsica de 2.000 V y el secundario alimenta una carga que consume 45 amperios con f.d.p. 0,8 capacitivo, determinar la tensin secundaria que se obtiene a la salida del transformador y el rendimiento de la mquina en estas condiciones. c) Calcular la potencia aparente de mximo rendimiento del transformador y el rendimiento mximo con f.d.p. unidad. [Resp.: a) RFe = 13,33 ka, XI' = 4,19 ill, Ree ~ 20 n Xcc = 19,6 a; b) 198,1 V, 91%; c) 7,75 kVA, 92,81%.] 3.15. Un transformador monofsico de 200 kVA, relacin 2.000/200 V, ha dado los siguientes resultados en unos ensayos de vaco y cortocircuito: Vaco: 200 V, 100 A, 5.000 W (medidos en el lado de B.T.). Cortocircuito: 200 V, 100 A, 7.000 W (medidos en el lado de A.T.). Calcular: 1) Parmetros del circuito equivalente aproximado del transformador reducido al primario. 2) Si se alimenta el primario con una red de 2.000 V Y el secundario lleva una corriente de 800 A, con f.d.p. 0,8 inductivo, cul ser la tensin en bornes del secundario y el rendimiento del transformador en estas condiciones? 3) Si estando alimentado el primario con una red de 2.000 V se conecta en el secundario una carga constituida por una resistencia de 0,3 ohmios en serie con una reactancia capacitiva de 0,4 ohmios (es decir, una impedancia compleja Z = 0,3 - jO,4 ohmios), cul ser el valor de la tensin en bornes del secundario? (efecto Ferranti), cul ser el valor de la corriente primaria? (tngase en cuenta la rama en paralelo del circuito equivalente del transformador, para contestar a esta ltima pregunta). [Resp.: 1) RFe = 800 a, XI' = 206,6 a, Rce = 0,7 a, x; = 1,873 a; 2) 186,53 V, 92,64 %, 3) 204,3 V, 35,8 A.] Un transformador monofsico de 160 kV A, relacin 2.000/200 V, 50 Hz, ha dado los siguientes resultados en unos ensayos: Vaco: 2.000 V, 1 A, 1.000 W (medidos en el lado de alta tensin). Cortocircuito: 8 V, ICOrlO = Iasignada' 2.560 W (medidos en el lado de B.T.). Calcular: a) Circuito equivalente aproximado del transformador reducido al primario (indicando los valores de los parmetros de este circuito que se obtienen de los ensayos). b) Si se aplica la tensin asignada al primario, cul ser la tensin secundaria cuando el transformador suministre una corriente secundaria de 400 A con f.d.p. 0,8 inductivo? Cunto vale el rendimiento del transformador en este caso? c) Cunto vale la potencia aparente de mximo rendimiento y el rendimiento mximo con f.d.p. 0,8 capacitivo? d) Cul ser la regulacin o cada de tensin relativa del transformador en el caso anterior? [Resp.: a) RFe = 4.000 a, XI' = 2.309,5 a, Ree = 0,4 a, x; = 0,916 a; b) 196,52 V, 97,5 %; c) 100 kV A, 97,56 %; d) -0,5725 %.] Un transformador monofsico de io kV A, relacin 1.0001100 V, 50 Hz, tiene los siguientes parmetros suministrados por el fabricante: PFe = 200 W, Bce = 10 %, Bx = 8 %. Calcular: a) Tensin secundaria y rendimiento del transformador cuando alimenta una carga que absorbe una corriente de 50 amperios con f.d.p. 0,707 inductivo (la tensin primaria se supone que es de 1.000 V). b) Si la tensin secundaria es de 100 voltios y el transformador alimenta una carga de 5 kW con f.d.p. 0,8 capacitivo, cul sera la tensin primaria correspondiente para que la tensin secundaria no vare? c) Si se aplica al transformador una tensin primaria de 1.000 voltios y se conecta en el

3.16.

3.17.

248

MQUINAS ELCTRICAS

secundario una impedancia: Z = 0,5 L 36,87 n, cul ser la tensin secundaria que tendr el transformador y el rendimiento en estas condiciones? d) Se acopla el transformador anterior con otro de 25 kV A, relacin 1.0oo!l 00 V, 50 Hz, cuya impedancia de cortocircuito reducida a primario vale: Zccll = 3,2 + j 2,4 ohmios. Ambos transformadores alimentan una carga a tensin constante de 100 V Y que absorbe una potencia de 20 kW con f.d.p. unidad. Cules sern las potencias aparentes, activas y reactivas suministradas por cada uno de los transformadores? [Resp.: a) 95,05 V, 90,56 %; b) 1.000 V; c) 83,8 V, 85,63 %; d) SI = 5.760 V A, PI = 5.643 W, QI = 1.161 VAR, Sil = 14.400 v; PI/ = 14.354 W, QII = -1.161 VAR] 3.18. Un transformador trifsico de 2.000 kV A, 6.600/33.000 V, tiene un primario conectado en tringulo y un secundario en estrella. La impedancia de cada fase del primario es 0,5 + j 2,6 n y la correspondiente del secundario es de 4,3 + j 21,7 n. Calcular la tensin en bornes del secundario a plena carga con f.d.p. 0,8 inductivo cuando el primario se conecta a la tensin asignada. [Resp.: 31.014 V.] Se han realizado unos ensayos en un transformador trifsico de 100 kVA, 400/ 6.600 V, 50 Hz, conexin estrella-tringulo, dando los siguientes resultados: Vaco: 400 V, 1.250 W (datos medidos en el lado de B.T.). Cortocircuito: 314 V, corriente de plena carga, 1.600 W (datos medidos en el lado de A.T.). Calcular: a) Rendimiento a plena carga con f.d.p. 0,8 en retraso. b) Rendimiento a media carga con f.d.p. unidad. c) Potencia aparente de mximo rendimiento y rendimiento mximo con f.d.p. unidad. d) Tensin necesaria en el primario para el caso a) si la tensin secundaria se mantiene 6.600 V. [Resp.: a) 96,56 %; b) 96,81 %; c) 88 kV A, 97,25 %; d) 416 V.] En la Figura P.3.1 se muestra el esquema de conexiones y las lecturas de los aparatos de medida para la realizacin de los ensayos de vaco y cortocircuito de un transformador trifsico Yd de 10 kVA, relacin de tensiones compuestas 1.000 V/lOO V. Determinar: 1) Parmetros del circuito equivalente por fase del transformador reducido al primario. 2) Angulo horario del transformador (alimentacin con sistema de secuencia directa).

3.19.

3.20.

Lecturas en vaco: PI
IO(lnea)

= 265,2
PI

W;

P2

= -65,2
W;
P2

W; VI(lInell)

1.000 V; V2(lnea)

100 V;

= 0,35 A.

Lecturas en cortocircuito:
57,7 A.

= 360,2

= -210,2

W; V2cc(lnea)

10 V; 12cc(lnea) =

ENSAYO DE VACO
}---QA

b'

Figura P.3.1.

CAPTULO

3.

TRANSFORMADORES

249

ENSAYO DE CORTOCIRCUITO r-----~A


a

.. ~ Cortocircuuo

t-----.:"f

a'
B

Figura P.3.1.

Continuacin.

[Resp.: 1) RFe

50 kn, X~= 1.747 n, Ree = 1,5 n Xc, = 9,89 n; 2) Yd 11.]

3.21. La Figura P.3.2 muestra el esquema simplificado de la instalacin elctrica de un grupo de bombeo utilizado para un sistema de riego por aspersin. Se dispone de una red de distribucin de 15 kV, 50 Hz, que por medio de un transformador Dy 11, 100 kVA, relacin compuesta: 15 kV/380 V, suministra energa elctrica al grupo motobomba a travs de una lnea resistiva de 0,2 ohmios por hilo. El grupo motobomba est representado por una impedancia de 6 L 36,87 ohmios por fase.
0,2 Q/hilo

lOO kVA, Dy 1I 15 kV/380 V

955 JiF/fase

Motobomba Z = 6L36,87ohmlfase

Figura P.3.2.

Las caractersticas del transformador que se leen en su placa de caractersticas son las siguientes: 100kVA,Dy11;15kV/380V;
8cc=1O%; 8x,,=8%

Calcular: 1) Parmetros Ree' X y Zee del circuito equivalente del transformador reducido al primario (se desprecia la rama paralelo del circuito equivalente). 2) Tensin secundaria de lnea en bornes del secundario del transformador y tensin en bornes del grupo motobomba, si la red de distribucin en A.T. tiene una tensin constante de lnea de 15 kV. 3) Rendimiento del transformador en estas condiciones. 4) Si para corregir el f.d.p. del grupo motobomba se conecta una batera de condensadores

250

MQUINAS ELCTRICAS

en estrella de 955 .uF/fase (como se indica en la figura), cul ser la nueva tensin de lnea en bornes del grupo motobomba? [Resp.: 1) Rcc = 404,05 n = 540,14 n, = 675 n. 2) 357,4 V, 330,4 V; 3) 95,39 %; 4) 340,5 V.]

x;

z;

3.22.

Se tiene un transformador trifsico de 250 kV A, conexin YyO, con una relacin de tensiones compuestas de 15.000/380 V. De los datos del fabricante se conocen los siguientes parmetros: Cee = 10 %, cx. = 8 %, Y se considera despreciable el efecto de la rama paralelo del circuito equivalente deltransformador, a) Calcular: parmetros Ree Y Xce del circuito equivalente por fase del transformador reducido al primario y corriente que circulara por el secundario si por una falta se produce un cortocircuito franco en los bornes del secundario. (Se supone para resolver esta ltima cuestin que la tensin de alimentacin del primario es la asignada de 15.000 V). b) Si la tensin compuesta de lnea en el secundario es de 380 V Y se conecta al transformador una carga en estrella de 15 L 60 ohmios por fase, cul ser la tensin compuesta que debe aplicarse al primario para que la tensin secundaria siga permaneciendo constante en 380 V de lnea. Cul ser el rendimiento del transformador en estas condiciones? c) Si se conecta este transformador en paralelo con otro de 350 kV A, conexin YyO, con la misma relacin de tensiones y de valores Cee 10 % Y cx .. 9 %, cmo se repartirn una potencia de 400 kW con f.d.p. 0,8 inductivo? (es decir, calcular las potencias activas y aparentes suministradas por cada transformador). [Resp.: a) Rec = 54 n = 72 n, t-: = 3.798 A; b) 15.052 V, 99,54 %; c) S = 209,15 kVA, = 180,3 kW, Q = 106,00 kVAR, Sil = 292,8 kVA, p = 219,5 kW, Qll = 194 kVAR.]

r,

x;

3.23.

Un transformador trifsico tiene las siguientes caractersticas asignadas: Conexin, YyO; potencia aparente asignada, 100 kVA; relacin de tensiones compuestas, 3.000 V/380 V. Los resultados de unos ensayos de vaco y cortocircuito han dado los siguientes valores: Vaco: 3.000 V, Po = 5 kW, medidos en el lado de A.T. (primario). Cortocircuito: 300 V, llcc = corriente asignada, Pec = 6 kW (medidos en el primario). (NOT A: Las potencias anteriores son totales trifsicas y las tensiones son compuestas o de lnea). Si la tensin secundaria de lnea se mantiene constante en 380 V, se pide: 1) La tensin compuesta necesaria en el primario cuando el transformador alimenta una carga trifsica equilibrada de 50 kW con f.d.p. 0,6 capacitivo. 2) Potencia aparente de mximo rendimiento y rendimiento mximo del transformador para un f.d.p. unidad. 3) Se desea ampliar la instalacin para alimentar una carga trifsica de 120 kW con f.d.p. 0,8 inductivo, por lo que se acopla en paralelo este transformador con otro cuyas caractersticas asignadas son las siguientes: Conexin, YyO; potencia asignada, 50 kVA; relacin, 3.000 V/380 V; cR = 8%, ;x.= 6%; Po = 2 kW. Calcular los valores de las potencias aparentes, activas y reactivas suministradas por cada transformador y los rendimientos correspondientes. [Resp.: 1) 2.930,1 V; 2) 91,3 kV A, 90,13 %; 3) 100 kVA, 75 kW, 68 kVAr, 87,2 %, 50 kVA, 45 kW, 22 kVAr, 88,24 %.] Un transformador monofsico de 50 kV A, 6.600/220 resultados en unos ensayos: Vaco: 1.000 W, 220 V, B.T.). Cortocircuito: 1.200 W, 300 V, 7,575 A (datos lar: parmetros del circuito equivalente aproximado V, 50 Hz, ha dado los siguientes 25 A (datos ledos en el lado de ledos en el lado A.T.) 1) Calcudel transformador reducido al

3.24.

CAPTULO 3.

TRANSFORMADORES

251

primario. 2) Se dispone de tres transformadores monofsicos idnticos al citado, que se conectan en tringulo en la parte de A.T. (primario) y en estrella la parte de B.T. (secundario). Si se alimenta el primario de esta combinacin a una red trifsica a 6.600 Y, determinar la tensin secundaria de lnea, si la corriente secundaria de lnea es de 200 A con un f.d.p. 0,8 inductivo. 3) La combinacin sealada en el apartado anterior (conexin Dy) se conecta nuevamente a una red trifsica de 6.600 y y en el secundario se coloca una carga trifsica equilibrada en tringulo de impedancia: 2,16 + j 1,62 ohmios/fase; calcular la tensin secundaria de lnea y el rendimiento del transformador en estas condiciones. [Resp.: 1) 'R" = 20,91 n, X" = 33,62 n. 2) 366,85 Y; 3) 364,25 Y, 94,55 %.] 3.25. En la Figura P.3.3 se muestra el esquema interno de las bobinas de un transformador trifsico de 50 kYA, relacin de tensiones compuestas 1.0001100Y, 50 Hz, conexin Dy, que ha dado los siguientes valores en unos ensayos: Vaco: 1.000 Y (lnea), 3 A (lnea), 1 kW (total, las tres fases). Cortocircuito: 100 Y (lnea), IeorlO = Iarignada' 1,5 kW (totales). Ambos ensayos se han medido en el lado de A.T.

Figura P.3.3.

a) Determinar los parmetros del circuito equivalente aproximado del transformador reducido al primario. b) ngulo horario del transformador, cuando se alimenta mediante un sistema directo de tensiones. c) Tensin primaria de lnea y rendimiento del transformador cuando alimenta una carga en el secundario que absorbe una potencia de 25 kW con f.d.p. 0,8 inductivo. (La tensin secundaria de lnea se mantiene siempre constante en 100 Y.) d) Cul es la potencia aparente de mximo rendimiento del transformador y el rendimiento mximo con f.d.p. unidad? e) Se desea ampliar la instalacin para alimentar una carga trifsica de 80 kW con f.d.p. 0,8 inductivo, por lo que se hace necesario acoplar en paralelo este transformador con otro Dy de igual ndice horario y cuyas caractersticas asignadas son: 75 kYA, 1.000/100 Y; GR . = 6,5 %; Gx . = 5 %. Calcular los valores de las potencias aparentes, activas y rectivas suministradas por cada transformador. Es adecuado el reparto de las potencias aparentes? Por qu? NOTA: La tensin secundaria es constante en todos los casos e igual a 100 Y de lnea. [Resp, a) RPe = 3.000 n, XII = 588,3 n, Re(' = 1,8 n, x; = 5,72 n; b) Dyll; c) 1.050,77 Y, 94,04 %; d) 40,825 kYA, 95,33 %; e) S, = 36,9 kYA, P, = 18,66 kW, Q, = 31,83 kYAr, Sil = 67,5 kYA, PII = 61,34 kW, QII = 28,16 kVAr, e, = 0,738, ell = 0,9. No es adecuado el reparto: ya que e, =1 ell. El transformador II resulta siempre ms sobrecargado.]

252

MQUINAS ELCTRICAS

3.26. Determinar el ngulo horario del transformador indicado en la Figura P.3.4.

Figura P.3.4.

[Resp.: DzO.] 3.27. La Figura P.3.5 muestra un transformador trifsico tringulo-estrella de relacin de tensiones compuestas 10.000/380 V, que alimenta en el lado de baja tensin una carga trifsica equilibrada conectada en estrella. Para medir la potencia absorbida por la carga se utilizaron dos vatmetros PI y P2 cuyas lecturas fueron: PI :: 1.000 kW, P2 :: 500 kW; suponiendo que en estas condiciones el transformador trabaja a plena carga y que la tensin secundaria compuesta es de 380 V, calcular: 1) Potencia aparente o de plena carga del transformador (kVA). 2) Si las impedancias de los devanados primario y secundario son: ZI:: 2,9 + j5,1 ohmios; Z2:: 1,5' 10-3 + j2,36' 10-3 ohmios, calcular la tensin primaria VI necesaria en el primario para alimentar la carga a 380 V de lnea. 3) Calcular el rendimiento del transformador a plena carga si el ndice de carga ptimo o de mximo rendimiento del transformador es igual a 0,75. 4) Cul es el valor del ngulo horario del transformador si la sucesin de fases es RST? NOTA: Utilizar el circuito equivalente aproximado del transformador reducido al primario. [Resp.: 1) 1.732 kVA; 2) 10.588,6 V; 3) 94,12%; 4) Dyll.]

50---------'

Figura P.3.5.

3.28. La Figura P.3.6 muestra un transformador trifsico estrella-tringulo de relacin de tensiones compuestas 15.000/380V, que alimenta en el lado de baja tensin una carga trifsica equilibrada conectada en tringulo de 0,3 L + 36,87 ohmios/fase. Suponiendo que en estas condiciones el transformador trabaja a plena carga y que la tensin secundaria es de 380 V, calcular: 1) Potencia aparente o de plena carga del transformador en kVA. 2) Si las impedancias de los devanados primario y secundario por fase

CAPTULO

3.

TRANSFORMADORES

253

son, respectivamente, Z1 = 2 + j 4 ohmios, Z2 = 1 . 10-3 + j 2 . 10-3 ohmios, calcular la tensin V1 de lnea para alimentar la carga a 380 V. 3) Calcular el rendimiento del transformador si el ndice de carga ptimo o de mximo rendimiento es igual a 0,8. 4) Cul es el valor del ngulo horario del transformador si la sucesin de fases es RST? NOTA: Utilizar el circuito equivalente aproximado del transformador reducido al primario. [Resp.: 1) 1.444 kVA; 2) 15.483,5 V; 3) 96,8%; 4) Ydl1.]
R O--..------QA
Q

so---------~
Figura P.3.6.

= 0,3L 36,87ohmlfase

3.29. Se dispone de un transformador trifsico de 50 MVA, 380 kV/60 kV (tensiones compuestas), conexin YyO,que ha sido sometido a un ensayo de cortocircuito, alimentado por el lado de B.T. Las medidas fueron las siguientes: V2ee = 4,2 kV de lnea; [2ce = 420,5 A; Pec (total) = 184 kW. El transformador est conectado a una red de 370 kV de tensin compuesta, 50 Hz, a travs de una lnea trifsica de 50 km de longitud, de resistencia despreciable y reactancia 0,4 ohmios por kilmetro. El transformador alimenta por su secundario una carga trifsica equilibrada conectada en tringulo constituida por una resistencia de 220 ohmios en serie con un condensador de 15 microfaradios. Calcular: a) tensin secundaria de lnea V2; b) corriente secundaria /2; e) rendimiento del transformador en esas condiciones si las prdidas en el hierro son de 150 kW. NOTA: Para realizar el problema se considera despreciable la impedancia en paralelo del circuito equivalente.

Red de A.T. 370 kV j20 n/hilo


50 MVA; YyO

380 kV/60 kV (lnea)

15.tF CARGA SECUNDARIA

Figura P.3.7. [Resp. a) 60.789 V; b) 345,42 A; e) 98,97 %.] 3.30. La Figura P.3.8 muestra el esquema simplificado de la instalacin elctrica para la alimentacin de un grupo de bombeo y su aplicacin en regados. Se dispone de una red de distribucin trifsica alimentada por una pequea minicentral hidrulica situa-

254

MQUINAS ELCTRICAS

da en un rea cercana a la estacin de bombeo. La minicentral consiste en un grupo turbina-alternador que genera una tensin asignada de 3.000 y Y que a travs de una red de media tensin de impedancia despreciable alimenta un transformador reductor de tensin para la alimentacin de la estacin de bombeo.
Alternador 0,05 + jO,l Q/hilo

Transformador

Red de distribucin

s 2L36,87 ohm/fase Grupo motobomba

Figura P.3.8. Las caractersticas asignadas del transformador son: SN = 100 kYA; conexion Dyll; relacin de tensiones compuestas, 3.000/380 Y; cadas relativas de tensin: Gee = 10 %, Gx . = 8 %. La red de distribucin que une el transformador con el grupo de bombeo tiene '{lOaimpedancia Z = 0,05 + jO, 1 ohmios por hilo. La estacin de bombeo se puede representar por una carga trifsica equilibrada conectada en estrella de impedancia 2 L 36,87 ohmios por fase. Calcular: 1) Parmetros Rec' Xec del circuito equivalente por fase del transformador reducido al primario. 2) Tensin compuesta en el primario del transformador (y que debe generar el alternador) para que la tensin en la carga (grupo de bombeo) sea de 380 Y de lnea. Determinar en esta situacin el rendimiento del transformador si las prdidas en el hierro son de 2 kW. 3) Si la tensin en el primario del transformador es de 3.000 Y (tensin compuesta), cul ser el valor de la tensin de lnea que se tendr en el grupo de bombeo? Determinar en este caso la cada relativa de tensin (regulacin) del transformador. [Resp.: 1) Rec = 16,2 n, x; = 21,6 n; 2) 3.360,6 Y, 92%; 3) 339,3 V, 6,24%.]

BIOGRAFAS
Frederick (1882-1946). Ingeniero americano. Estudi en la Universidad de Lehigh (1905) y en Comell (1909). Fue profesor de Cornell varios aos. En 1907 ingres en la General Eleetrie en Pittsfield, donde trabaj en el departamento de transformadores, llegando en 1926a ser subdirector. En 1924 obtuvo el premio Coffin por sus trabajos sobre regulacin de tensin en transformadores y por el desarrollo de transformadores no resonantes. Fue autor de muchos artculos tcnicos y editor y coautor del famoso libro Transformer Engineering, que fue traducido a varios idiomas. Estaba en posesin de ms de 20 patentes en el campo de la ingeniera elctrica. 2. Bov AGIAN, Aran (1888-?). Ingeniero armenio-americano. Emigr a los EE.UU. en 1910. Estudi en el Swarthmore College de Philadelphia (1915). En ese mismo ao ingres en la General Eleetrie en el departamento de transformadores. Dise un transformador de impulsos para ensayos de A.T. de 50 MW, 240 kV. Profesor visitante de ingeniera elctrica en el MIT. Estaba en posesin de ms de 18 patentes en el campo de la electricidad.
1.
BLUME, Louis

CAPTULO 3.

TRANSFORMADORES

255

3. FERRANTl, Sebastian Ziani (1864-1930). Ingeniero britnico de origen italiano. Estudi en el University College de Londres, donde tuvo como profesores a Carey Foster y a Sir Oliver Lodge. Trabaj en todos los campos de la electrotecnia. A los catorce aos ya haba construido una dinamo. Invent un alternador (1882), un modelo de contador elctrico de mercurio por c.c. (1883) y un transformador (1885). En 1886, cuando contaba solamente veintidos aos, fue nombrado ingeniero jefe para ocuparse del sistema de distribucin de la Grosvernor Gallery; para la ejecucin de este proyecto, Ferranti, que era un acrrimo defensor de la c.a. (en Inglaterra defendan la c.a.: Ferranti, Gordon. W. M. Mordey y Silvanus Thomson, mientras que estaban a favor de la C.C.: Lord Kelvin, Crompton, A. W. Kennedy y John Hopkinson; en EE.UU. estaban a favor de la c.a.: Tesla, Sprague y C. P. Steinmetz, y a favor de la C.C.: Edison), ide una red de transporte de c.a. a 10.000 V, lo que representaba en aquella poca una utopa, por una parte porque se prescinda de la c.c. y por otra porque la tensin de transporte se consideraba inalcanzable. El propio Ferranti dise dos alternadores monofsicos de 400 kW cada uno a 10.000 V, sustituy el esquema serie de Gaulard y Gibbs por un acoplamiento en paralelo y con una reduccin a 2.400 V de tensin secundaria, y tuvo que ocuparse tambin del diseo de cables de A.T., inventando nuevos procedimientos de aislamiento. Cuando se puso en funcionamiento esta instalacin se comprob que se requeran nicamente 8.500 V en la central generadora situada en Deptford para que llegaran 10.000 Va la Grosvernor Gallery; este fenmeno, conocido hoy da como efecto Ferranti, se atribuy a la interaccin de los devanados del transformador con la capacidad del cable. Ferranti, a partir de 1900se dedic al desarrollo de turbinas de vapor para centrales trmicas. Fue presidente del lEE en 1910 y 1911 Y miembro de la Royal Society en 1927. 4. GAULARD, Lucien (1850-1888). Fsico francs. Se dedic inicialmente a trabajos qumicos en relacin con la fabricacin de explosivos y luego se ocup en experimentos elctricos. En 1881 present una pila termoqumica en la Exposicin Internacional de Electricidad de Pars. Se dedic despus a estudiar el problema del transporte y distribucin de energa elctrica, habiendo acertado (en unin con Gibbs) a desarrollar sus generadores secundarios, que hoy denominamos transformadores. En 1884, con motivo de la Exposicin Internacional de Turn, realiz una red de transporte de 80 km de longitud entre la estacin de Lanzo y Turn (la red formaba un bucle que pasaba tambin por la ciudad de Venaria), en la que utilizaba sus generadores secundarios para optimizar el rendimiento del transporte; por esta demostracin Gaulard recibi un premio de 10.000 liras del Gobierno italiano. El Presidente del Jurado era el profesor Galileo Ferraris. Desgraciadamente, el primario de este transformador se colocaba en serie y tena por ello grandes defectos; un ao ms tarde, los ingenieros de la casa Ganz de Budapest: Deri, Blathi y Zypernowski; mejoraron el diseo, proponiendo una conexin paralelo, a tensin constante, para el primario, que es el diseo actual. La patente del transformador se adjudic a los hngaros, y Gaulard, preso de una crisis nerviosa, muri prematuramente en el hospital de Santa Ana de Pars antes de cumplir los treinta y ocho aos. 5. KAPP, Gisbert (1852-1922). Ingeniero austro-britnico. Naci en Viena y obtuvo el ttulo de ingeniero en el Politcnico de Zurich, donde fue alumno de Kohlrausch. En 1875 se traslad a Inglaterra y pas la mayor parte de su vida en este pas. Sus trabajos en electricidad comenzaron en 1882, cuando ingres como ingeniero en la empresa del coronel Crompton, una de las industrias pioneras en la construccin de maquinaria elctrica en Inglaterra. Ms tarde, en 1885, se estableci por su cuenta como ingeniero consultor. Se le deben grandes inventos en el campo de la ingeniera elctrica: fue el primero que ide el devanado compound en las mquinas de C.C.; introdujo el inductor multipolar en las dinamos; patent una dinamo autorregulada para iluminacin por arco elctrico. Realiz numerosos proyectos de centrales elctricas, diseando l mismo los alternadores y transformadores. Fue un defensor de la c.a. Escribi muchos artculos y libros sobre ingeniera elctrica: transporte de energa elctrica, corrientes alternas, dinamos, alternadores y transformadores, transformadores para corrientes monofsicas y polifsicas, diseos electromecnicos, principios de ingeniera elctrica y sus aplicaciones, etc. Entre 1894y 1905 actu como Secretario de la Verband Deutscher Elektrotechniker en Berln. (El objetivo de esta sociedad fue la creacin de normas de estandarizacin, conocidas hoy da por sus iniciales VDE, y que han sido adaptadas a todos los pases.) Mientras estuvo en Alemania prepar a estudiantes postgraduados en Charlo-

256

MQUINAS ELCTRICAS

6.

7.

8.

9.

tenburgo. Fue nombrado Doctor Honoris Causa por las Universidades de Dresden y Karlsruhe. En 1905 volvi a Inglaterra y fue nombrado catedrtico de ingeniera elctrica de la Universidad de Birmingham, a propuesta de Sylvanus Thompson y Sir Oliver Lodge, puesto en el que permaneci el resto de su vida. KARAPETOFF, Vladimir Nikitich (1876-1948). Ingeniero ruso-americano. Estudi Ingeniera Civil en San Petersburgo, donde se gradu en 1877. Ampli estudios de Ingeniera Elctrica en Alemania (1899-1900). Profesor de Electrotecnia e Hidrulica en su Universidad natal (1900-1902). En 1902 se traslad a los Estados Unidos, donde trabaj en la Compaa Westinghouse. Catedrtico de Ingeniera Elctrica en la Universidad de Cornell desde 1908. Escribi numerosos libros y artculos sobre ingeniera, ciencia, educacin, msica y poesa. En particular, su libro Ingeniera Elctrica Experimental, traducido avarios idiomas, fue tomado como referencia obligada en muchas Universidades y Centros de Investigacin del mundo. Su aficin por la msica hizo que se graduara en piano en 1934 en la Universidad de New York. RHMKORFF, Heinsrich Daniel (1803-1877). Inventor alemn. Trabaj como aprendiz de mecnico en Hannover; en 1824 trabaj en Inglaterra en el taller de Joseph Brahmah, inventor de la prensa hidrulica, en 1827 se fue a Pars, trabajando con Charles Chevalier, conocido por su taller de instrumentos pticos. En 1855 fund su propia empresa, en la que se dedic a la construccin de aparatos elctricos de precisin. Su principal invencin es la bobina de induccin capaz de producir chispas de ms de 30 cm de longitud. El carrete de Rhmkorff, que as se conoce esta bobina, se us ampliamente para el funcionamiento de tubos de Geissler y Crookes y para otras necesidades de A.T. en laboratorios de fsica de todas las universidades del mundo. Scorr, Charles Feldon (1864-1944). Ingeniero americano. Estudi en la Universidad de Ohio, en Columbus, donde se gradu como ingeniero en 1885.Realiz estudios de postgrado en la Universidad Johns Hopkins. Al acabar su carrera trabaj como instalador elctrico en la Baldwin Locomotive Works. En 1888 ingres en la Westinghouse Electric and Manufacturing Co., trabajando en el laboratorio de ensayos. Ms tarde se asoci con Nikola Tesla para desarrollar el motor asncrono de corriente alterna. En la Westinghouse lleg en 1897a director del departamento de ingeniera elctrica y en 1904 pas a consultor de ingeniera. Durante su trabajo en la Westinghouse fue pionero en el transporte de energa elctrica en alta tensin y en el diseo y construccin de mquinas elctricas. Intervino en el diseo de mquinas elctricas para la central del Nigara, inaugurada en 1896, que dispona de alternadores bifsicos. Fue en 1894, y trabajando en el proyecto de la central anterior, cuando invent la denominada conexin en T o conexin Scott de transformadores para convertir la c.a. bifsica en trifsica y viceversa. Durante la observacin de los ensayos de aisladores en 1895, en el laboratorio de la Westinghouse, predijo las prdidas por efecto corona en las lneas de A.T. En el Congreso de San Luis de 1904 fue el director del captulo de redes de transporte. En 1911 acept una ctedra de ingeniera elctrica en la Sheffield Scientific School de la Universidad de Yale, cargo en el que estara el resto de su vida y donde realiz grandes esfuerzos para elevar el nivel cientfico de las enseanzas tcnicas. Public muchos artculos sobre ingeniera elctrica. En 1930 recibi la medalla Edison por sus contribuciones a la ciencia del transporte de energa elctrica. Presidente n." 15 del AlEE en el bienio 1902-1903 (American Institute of Electrical Engineers, que al unirse en 1962 con el IRE, Institute of Radio Engineers, dio lugar al actual IEEE, Institute of Electrical and Electronic Engineers). STANLEY, William (1858-1916). Inventor americano. Al acabar sus estudios de bachillerato, trabaj en una compaa que fabricaba instrumentos telegrficos. En 1880se incorpor a la empresa del Dr. Edward Weston, que se dedicaba a la construccin de aparatos de medida. Dos aos ms tarde mont su propio laboratorio, donde realiz experimentos con acumuladores y transporte de energa en A.T. En 1884 se incorpor a la recin creada empresa Westinghouse, en la que dise la mayor parte de las mquinas elctricas de las centrales que construa esta compaa. Invent el autotransformador, varios tipos de generadores y motores de c.a., un vatmetro de induccin y aparatos para neutralizar la induccin de las lneas telefnicas. En 1890fund la Compaa Elctrica Stanley, que ms tarde fue absorbida por la General Electric. Stanley sigui trabajando como ingeniero consultor de la nueva empresa el resto de su vida.

CAPTULO 3.

TRANSFORMADORES

257

10. STlGANT, S. Austen (1890-1973). Ingeniero britnico. Estudi en el Aston Technical College. A los veintidos aos ingres como delineante en la Compaa Westinghouse britnica, donde ms tarde llegara a ingeniero encargado del proyecto de transformadores. En 1930 pas al puesto de asesor tcnico y encargado del departamento de formacin, donde fue director durante casi treinta aos. En 1950 fund la Tensor Society en el Reino Unido. Stigant escribi ms de 150 artculos tcnicos y ocho libros sobre ingeniera elctrica. El que le dio mayor renombre fue el que llevaba por ttulo: The J&P Transformer Book, figurando como coautor A. C. Franklin, ingeniero jefe de la empresa inglesa Johnson & Phillips. Este texto, de obligada consulta para los especialistas, comenz a editarse en 1925 bajo la direccin de Morgan Lacey. En 1973 se public la undcima edicin. 11. THOMPSON, Silvanus Phillips (1851-1916). Fsico e ingeniero ingls. Estudi en la Royal School of Mines, en Londres. En 1876 se fue a Heidelberg, recibiendo las enseanzas de Bunsen. A su vuelta a Inglaterra fue nombrado catedrtico de Fsica en el University College de Bristol (1876); ms tarde, en 1885, fue nombrado catedrtico de Fsica e Ingeniera Elctrica en el City and Guilds Technical College de Londres, donde ense el resto de su vida. Como profesor era un maestro incomparable, pues tena una facilidad innata para exponer en su ctedra los ms arduos problemas cientficos en forma tan clara y amena que provocaba la admiracin de sus alumnos. Escribi libros de divulgacin sobre electricidad y magnetismo, clculo diferencial e integral, pero su obra maestra fue un magnfico tratado de mquinas dinamoelctricas, que alcanz seis ediciones y que fue traducido a varios idiomas. Este libro fue un texto imprescindible en muchas Universidades del mundo en la primera mitad del siglo xx. Conferenciante infatigable, ayud a divulgar los progresos de la electricidad, hacindola asequible al gran pblico. Aficionado a la msica y dotado de un excelente odo, en 1910ley una conferencia en la Physical Society sobre lazos de histresis y figuras de Lissajous, que era una mezcla exquisita de magnetismo, sonido y teoremas matemticos. Goz de grandes honores y distinciones. En 1891 fue uno de los vicepresidentes honorarios de la Exposicin Elctrica de Frankfurt y en el mismo ao fue elegido Fellow de la Royal Society. Doctor Honoris Causa por la Universidad de Konigsberg (1894). Escribi las biografas de Faraday (1898) y de Lord Kelvin (1910). Presidente de la Institucin de Ingenieros Elctricos ingleses en 1899.

REFERENCIAS
1. AEG: Manual AEG 2. Tratado de Electrotecnia Prctica. Ed. Elitera-Verlag, Berln,1972. 2. BAILEY, B. F.; GAULT, 1. S.: Alternating-Current Machinery. McGraw-HiIl, New York, 1951. 3. BERROSTEGUIETA,1.: Introduccin a los transformadores de medida. Electrotcnica Arteche Hermanos S. L., Mungua (Vizcaya), 1992. 4. BLUME, L. F.; BOYAGIAN, A.; MONTSINGER, V. M.: Transformer Engineering, General Electric Co., 2." edition, John Wiley & Sons, Inc., New York, 1951. 5. CHAPMAN, S. 1.: Mquinas Elctricas. McGraw-HiII Latinoamericana, S. A., Bogot, Colombia, 1987. 6. CHATELAIN, J.: Trait d'lectricit, Vol. X: Machines lectriques. Editions Georgi, Lausanne, 1983. 7. DELTORO, V.: Electric Machines and Power Systems. Prentice Hall Inc., Englewood Cliffs, N. J., 1985. 8. DORMONT, M. J.: Transformadores. Servicio de Publicaciones de la E.T.S. de Ingenieros Industriales, Madrid, 1971. 9. FrTZGERALD, A. E.; KINGSLEY, CH.; UMANS, S. D.: Electric Machinery, Fourth Edition (S.I.). McGraw-HiII Book C., New York, 1985. 10. FOUILLE, A.: Electrotecnia para Ingenieros, tomo n. Ed. Aguilar, Madrid, 1969. 11. FRANKLIN, A. c.; FRANKLIN, D. P.: J & P Transformer Book. Johnson and Phillips, l l ." edition, Butterworths, London, 1983. 12. Grsas, J. B.: Transformers: Principies and Practice. McGraw-Hill Book Co., New York, 1950. 13. IVANOV-SMOLENSKY, A.: Electrical Machines, Vol. 1. Mir Pub., Moscow, 1982.

258
14. 15. 16. 17. 18. 19. 20. 21. 22. 23. 24. 25. 26. 27.

MQUINASELCTRICAS
Kosow,I. L.: Electric Machinery and Transformers. Prentice Hall, Inc., Englewood Cliffs, New Jersey, 1972. KOSTENKO,M. P.; PIOTROYSKI, L. M.: Mquinas elctricas. Ed. Mir, Mosc, 1975. LANGSDORF, A. S.: Teora de las mquinas de c.a. Ed. de) Castillo, Madrid, 1967. MCPHERSON,G.: An Introduction to Electrical Machines and Transformers. John Wiley & Sons, New York, 1981. M.I.T. Staff: Magnetic Circuit and Transformers. Massachusetts Institute ofTechnology, Boston, 1962. MOELLER, F.; KUBLER, E.; WERR, TH.: Electrotecnia General y Aplicada, Tomo I1, Parte l. Ed. Labor, Barcelona, 1962. NAGRATH,1. J.; KOTHARI,D. P.: Electric Machines. Tata McGraw-HiII Pub., New Delhi, 1985. RAs, E.: Transformadores de potencia, de medida y de proteccin. Ed. Tcnicas Marcombo, Ed. Boixerau, Barcelona, 1975. SAY, M. G.: Alternating Current Machines. Pitman Pub. Co., London, 1976. SISKIND,CH.: Electrical Machines: Direct and Alternating Current. McGraw-Hill Book Co., New York,1950. SLEMON,G. R.; STRAUGEN, A.: Electric Machines: Direct and Alternating Current. McGraw-Hill Book, New York, 1950. STEIN, R.; HUNT,W. T.: Electric Power System Components: Transformers and Rotating Machines. Van Nostrand Reinhold Co., New York, 1979. WESTlNGHOUSE, Co.: Electrical Transmission and Distribution Reference Book. East Pittsburg, 1950. WILDI,TH.: Tecnologa de los Sistemas Elctricos de Potencia. Ed. Hispano Europea S. A., Barcelona, 1983.

CAPTULO 4

Mquinas asncronas

4.1. INTRODUCCIN
El principio de funcionamiento de las mquinas asncronas se basa en el concepto de campo magntico giratorio, ya explicado en el Captulo 2. El descubrimiento original fue publicado en 1888 por el profesor Galileo Ferraris en Italia y por Nikola Tesla en los EE.UU. Ambos diseos de motores asncronos se basaban en la produccin de campos magnticos giratorios con sistemas bifsicos, es decir, utilizando dos bobinas a 90 alimentadas con corrientes en cuadratura. Desgraciadamente, el motor bifsico de Ferraris tena un circuito magntico abierto y un rotor en forma de disco de cobre, por lo que desarrollaba una potencia muy baja y no tena inters comercial. Sin embargo, Tesla, que dio a conocer su motor dos meses ms tarde que Ferraris, utiliz devanados concentrados tanto en el esttor como en el rotor, logrando con ello un motor ms prctico, y de ah que se considere a Tesla el inventor de este tipo de mquinas. Las patentes de Tesla fueron adquiridas por G. Westinghouse, quien construy en sus fbricas motores bifsicos que puso en el mercado alrededor de 1890. En este mismo ao el ingeniero de la AEG Dolivo Dobrowolsky invent el motor asncrono trifsico, empleando un rotor en forma de jaula de ardilla y utilizando un devanado distribuido en el esttor. En el ao 1891 Dobrowolsky present en la Exposicin de Electricidad de Frankfurt un motor asncrono con rotor devanado que dispona de un restato de arranque a base de resistencias lquidas. En el ao 1893 Dobrowolsky haba desarrollado tambin motores asncronos con doble jaula de ardilla, que posean mejores cualidades de arranque que el motor en cortocircuito convencional (sin embargo, fue el francs P. Boucherot quien ms investig con este tipo de rotor). A principios del siglo xx se impuso el sistema trifsico europeo frente al bifsico americano, por lo que las mquinas asncronas empezaron a ser (y son) trifsicas. La diferencia de la mquina asncrona con los dems tipos de mquinas se debe a que no existe corriente conducida a uno de los arrollamientos. La corriente que circula por uno de los devanados (generalmente el situado en el rotor) se debe a la f.e.m. inducida por la accin del flujo del otro, y por esta razn se denominan mquinas de induccin. Tambin reciben el nombre de mquinas asncronas debido a que la velocidad de giro del rotor no es la de sincronismo impuesta por la frecuencia de la red. La importancia de los motores asncronos se debe a su construccin simple y robusta, sobre todo en el caso del rotor en forma de jaula, que les hace trabajar en las circunstancias ms adversas, dando un excelente servicio con pequeo mantenimiento. Hoy en da se puede decir que ms del 80 por 100 de los motores elctricos industriales emplean este tipo de mquina, trabajando con una frecuencia de alimentacin constante. Sin embargo, histricamente su inconveniente ms grave ha sido la limitacin para regular su velocidad, y de ah que cuando esto era necesario, en diversas

259

260

MQUINAS ELCTRICAS

aplicaciones como la traccin elctrica, trenes de larninacin, etc., eran sustituidos por motores de C.C., que eran ms idneos para este servicio. Desde finales del siglo xx y con el desarrollo tan espectacular de la electrnica industrial, con accionamientos electrnicos como inversores u onduladores y cicloconvertidores, que permiten obtener una frecuencia variable a partir de la frecuencia constante de la red, y con la introduccin del microprocesador en la electrnica de potencia, se han realizado grandes cambios, y los motores asncronos se estn imponiendo poco a poco en los accionamientos elctricos de velocidad variable. En el Captulo 7 se estudian estos convertidores electrnicos y su aplicacin en la regulacin de velocidad de motores elctricos tanto de c.c. como de c.a.. En este captulo comenzamos analizando los aspectos constructivos de los motores asncronos. Observando la forma del esttor y los tipos de rotor existentes, se explica con detalle la caja de bornes del motor y sus formas de conexin. Se estudia luego el principio de funcionamiento de los motores asncronos trifsicos, detallando que la accin de las fuerzas en el rotor se produce en las ranuras, no en los conductores. Se calculan las relaciones de f.e.in.s. y corrientes en los devanados del esttor y del rotor y se define el concepto de deslizamiento. A partir de estos clculos, se establece el circuito equivalente de la mquina, en la que se aprovecha su gran analoga con el transformador. Se hace especial hincapi en el significado de la resistencia de carga, responsable de la energa mecnica que se produce en el rbol de la mquina. Se advierte del riesgo de utilizar circuitos equivalentes aproximados para estudiar estas mquinas debido a que la corriente de vaco es comparativamente ms alta que en el caso de transformadores. Se establece con ello un circuito equivalente aproximado con ajuste de tensin, que suele dar buenos resultados en la prctica. A continuacin se sealan los ensayos necesarios para determinar los parmetros del circuito equivalente, lo que permite ms tarde hacer un estudio analtico del balance de potencias en el motor y su distribucin en las diversas partes que componen la mquina, obteniendo una serie de relaciones tiles entre ellos. Se define entonces el concepto de par electromagntico de rotacin y se llega a una frmula analtica que muestra la dependencia con los principales parmetros de la mquina. Se dibuja la curva par-velocidad de la mquina, distinguiendo la caracterstica natural de las artificiales, obteniendo una serie de expresiones que permiten dibujar las caractersticas artificiales a partir de la natural, lo que simplifica el trabajo grfico. Se explican los modos de funcionamiento de la mquina asncrona, analizando con detalle el trabajo en rgimen motor, generador y freno. En cada caso se utilizan figuras que explican con sencillez las transferencias de energa que se producen entre los terminales elctrico y mecnico. Se pasa luego a explicar el diagrama del crculo como procedimiento grfico que permite determinar el comportamiento de la mquina, observando el lugar geomtrico de las corrientes del motor, donde puede medirse a escala las diferentes variables que son de inters en el estudio de la mquina. En la actualidad el diagrama del crculo, gracias a los ordenadores y calculadoras existentes en el mercado, ha perdido importancia, pero se ha incluido aqu por su importancia histrica y didctica. Se estudian ms tarde los mtodos de arranque de los motores trifsicos segn sean en jaula de ardilla o con rotor devanado. Se explican los motores de doble jaula de ardilla, deduciendo de un modo intuitivo el circuito equivalente correspondiente. A continuacin se trata el tema de regulacin de velocidad, analizando con detalle el motor de dos velocidades, que corresponde a la conexin Dahlander. Se incorpora una pregunta en la que se estudia la dinmica del motor asncrono y se determinan los tiempos de aceleracin y las prdidas de energa que se producen en el proceso de arranque y frenado. Contina el captulo con el estudio de los motores asncronos monofsicos, en los que se deduce el circuito equivalente de un modo heurstico y en el que se explican los procedimientos de arranque

CAPTULO 4.

MQUINAS ASNCRONAS

261

de estas mquinas. La leccin finaliza con un estudio del funcionamiento del motor asncrono trifsico cuando est alimentado por tensiones trifsicas desequilibradas y la descripcin de algunas mquinas asncronas especiales, tales como el regulador de induccin, los selsyn o ejes elctricos y el motor de induccin lineal. Se incluye un apndice dedicado al anlisis del par de rotacin de un motor asncrono desde el punto de vista fsico.

4.2. ASPECTOS CONSTRUCTIVOS


La mquina asncrona o de induccin al igual que cualquier otro dispositivo de conversin electromecnica de la energa de tipo rotativo, est formada por un esttor y un rotor. En el esttor se coloca normalmente el inductor, alimentado por una red mono o trifsica. El desarrollo de este captulo se dedica a la mquina trifsica, exceptuando el epgrafe 4.13, donde se estudiar el funcionamiento y arranque de los motores monofsicos. El rotor es el inducido, y las corrientes que circulan por l aparecen como consecuencia de la interaccin con el flujo del esttor. Dependiendo del tipo de rotor, estas mquinas se clasifican en: ti) rotor en jaula de ardilla o en cortocircuito, y b) rotor devanado o con anillos. El esttor est formado por un apilamiento de chapas de acero al silicio que disponen de unas ranuras en su periferia interior en las que se sita un devanado trifsico distribuido, alimentado por una corriente del mismo tipo, de tal forma que se obtiene un flujo giratorio de amplitud constante (vase epgrafe 2.8.3) distribuido senoidalmente por el entrehierro. El esttor est rodeado por la carcasa, tal como indica la Figura 4.1, disponindose en sta las correspondientes patas de fijacin y los anillos o cncamos de elevacin y transporte. El rotor est constituido por un conjunto de chapas apiladas, formando un cilindro, que tienen unas ranuras en la circunferencia exterior, donde se coloca el devanado. En el tipo en forma de jaula de ardilla se tienen una serie de conductores de cobre o aluminio puestos en cortocircuito por dos anillos laterales (el nombre de jaula proviene del aspecto que tomara este devanado si se omitiera el apilamiento de hierro); en la actualidad, en las mquinas pequeas, se aplica un mtodo de fundicin de aluminio, con el que se producen al mismo tiempo las barras del rotor y los anillos laterales, resultando un conjunto como el que se muestra en la Figura 4.2. En el caso de rotor devanado o con anillos, se tiene un arrollamiento trifsico similar al situado en el esttor, en el que las tres fases se conectan por un lado en

Figura 4.1. Esttor de un motor asncrono.

262

MQUINAS ELCTRICAS

Figura 4.2. Rotor en jaula de ardilla.

estrella y por el otro se envan a unos anillos aislados entre s, como muestra la Figura 4.3. Esta disposicin hace posible la introduccin de resistencias externas por los anillos para limitar las corrientes de arranque, mejorar las caractersticas del par y controlar la velocidad. La mquina asncrona, adems de disponer de un esttor y un rotor, est dotada de otros elementos mecnicos necesarios para su funcionamiento: tapas o cubos, rodamientos, carcasa, etc. En los motores de mediana y gran potencia existe un ventilador en el eje, cuya misin es producir una refrigeracin forzada de la mquina. A veces la carcasa tiene forma ondulada para mejorar la evacuacin del calor que se produce como consecuencia de las prdidas que aparecen en el motor. Un detalle importante a considerar en los motores asncronos trifsicos es la disposicin de los terminales del devanado del esttor en la llamada caja de bornes de la mquina. A esta caja o placa se llevan los extremos de los bobinados, en la forma que se muestra en la Figura 4.4a. De acuerdo con la Norma UNE-EN 60034-8, los principios de los arrollamientos se designan con las letras VI' VI Y WI (antiguamente V, Vy W) Ylos extremos finales con V2, V2 y W2 (antiguamente X, Y, Z), respectivamente. Debe destacarse que los terminales de la misma fase no estn enfrentados en la regleta de bornes, y esto se debe a que esta disposicin facilita el conexionado de la mquina, haciendo uso de unas lminas de latn adecuadas, tal como muestran las Figuras 4.4b y c. Se observa que en la conexin en tringulo se unen los terminales VI con W2, VI con V2 y WI con V2, mientras que en la conexin en estrella se unen entre s los terminales W 2 con U2 y U2 con V2 (en este caso, solamente seran necesarias dos lminas de latn para hacer los puentes correspondientes, pero se utilizan los tres, dos de ellos superpuestos, para poder disponer de las tres lminas cuando se necesite hacer la conexin en tringulo del motor). Los esquemas desarrollados de ambas conexiones se muestran en la Figura 4.5. La conexin en estrella se emplea cuando la mquina ha de conectarse a la tensin ms elevada

Figura 4.3. Rotor devanado o con anillos.

CAPTULO

4.

MQUINAS ASNCRONAS

263

REDC.A.

RED C.A.

a) Caja de bornes

b) Conexin tringulo

e) Conexin estrella

Figura 4.4.

Placa de bornes. Conexiones estrella y tringulo.

indicada en su placa de caractersticas, utilizando la conexin en tringulo para la tensin ms baja. Por ejemplo, si se tiene un motor asncrono en cuya placa aparecen los valores 30 CV, 220/380 V, 69,2/40 A, 1.450 r.p.m., quiere decir que la potencia asignada es de 30 CV, girando a plena carga a 1.450r.p.m., se puede conectar a una red de 220 V, disponiendo sus terminales en tringulo (tensin ms baja), y entonces absorbe a plena carga una corriente de lnea de 69,2 A, tambin se puede alimentar por una red de 380 V, pero para ello han de conectarse los devanados en estrella (tensin ms alta), de tal forma que entonces la mquina consume una corriente (a plena carga) de 40 A. Los cambios en las conexiones anteriores se justifican por la necesidad de que la mquina trabaje en ambos casos con las mismas tensiones en los arrollamientos. Para invertir el giro del motor es preciso cambiar el sentido de movimiento del campo giratorio, lo cual se logra intercambiando entre s dos cualesquiera de los cables que se unen a la red de alimentacin. En la Figura 4.6 se muestran una serie de conexiones tpicas, indicando los sentidos de giro correspondientes. La posicin de trabajo de estos motores puede ser con eje horizontal o vertical. La Norma DIN 42950 da una relacin de posibles versiones en funcin de ello y segn que la mquina posea patas o bridas y que la fijacin se haga sobre el suelo, pared o techo. Las Normas americanas NEMA (Publicacin MGI-1963) especifican varias clases de diseo, definidas por las letras A, B, C, O YF, dependiendo de las relaciones: par arranque/par asignado, corriente de arranque/corrienteasignada.Las potencias asignadas de fabricacin varan en un amplio rango. En la Tabla 4.1 se muestra un cuadro de caractersticas tcnicas de motores asncronos trifsicos con rotor de jaula de ardilla para una velocidad de sincronismo de 1.500 r.p.m. La potencia asignada mecnica se expresa tanto en CV como en kW (1 CV = 736 W). Las tensiones de trabajo de los motores de la serie anterior oscilan entre 220,380 Y500 V. Cuando las potencias necesarias son ms elevadas es conveniente emplear mquinas alimen-

RED C.A.
T

RED C.A.
T

Figura 4.5. Conexiones de la placa de bornes y sentidos de rotacin que se obtienen.

264

MQUINAS ELCTRICAS

R ~
S ~

.-

__

+--.

~ __.-

+-~~

-+__~

__

Conexin Sentido de giro

Figura 4.6. Esquemas desarrollados de las conexiones estrella y tringulo. tadas por redes de A.T., y es frecuente en grandes motores utilizar tensiones del orden de 3 kV a 6 kV, cubriendo una gama de potencias entre 200 y 20.000 kW.

4.3. PRINCIPIO DE FUNCIONAMIENTO


Generalmente la mquina asncrona suele funcionar como motor, y a este rgimen de funcionamiento nos referimos en lo sucesivo, mientras no se diga lo contrario. El devanado del esttor est constituido por tres arrollamientos desfasados 1200 en el espacio y de 2p polos; al introducir por ellos corrientes de una red trifsica de frecuencia JI' se produce una onda rotativa de f.m.m. distribuida senoidalmente por la periferia del entrehierro, que produce un flujo giratorio cuya velocidad viene expresada, de acuerdo con (2.68), por: (r.p.m) (4.1)

que recibe el nombre de velocidad de sincronismo. Este flujo giratorio inducir f.e.m.s. en los conductores del rotor, y si est su circuito elctrico cerrado, aparecern corrientes que reaccionarn con el flujo del esttor. En la Figura 4.8a se muestra en un determinado instante el sentido de la induccin B en el entrehierro producida por el devanado del esttor, cuya distribucin es senoidal, lo que se representa por medio de una diferencia en la concentracin de lneas de B. De acuerdo con la ley de Faraday, la f.e.m. inducida en un conductor de longitud L que se mueve a la velocidad v dentro de un campo B tiene un valor:

e=

(v x B) di = (v x B) . L

(4.2)

Para determinar su sentido debe considerarse que el rotor gira en sentido contrario al campo para tener en cuenta el movimiento relativo mutuo entre ambos sistemas; en la Figura 4.7 se ha sealado, de este modo, el sentido saliente de las corrientes en los conductores del rotor.

CAPTULO 4.

MQUINAS ASNCRONAS

265

Tabla 4.1.

Caractersticas tcnicas de motores asncronos trifsicos con rotor en jaula de ardilla

1/12 1/8 1/6 1/4 1/3 1/2 3/4 1 1,5 2 3 4 5,5 7,5 10 15 20 25 30 40 50 60 75 100 125 150 180 220 270 340 430 545

0,06 0,09 0,12 0,18 0,25 0,37 0,55 0,75 1,1 1,5 2,2 3 4 5,5 7,5 11 15 18,5 22 30 37 45 55 75 90 110 132 160 200 250 315 400

2,5 2,8 3,5 4,1 4,8 6,0 8,0 9,4 12 15,6 22 24 42 50 66 92 110 170 195 265 320 350 470 630 700 850 960 1.060 1.410 1.630 2.020 2.380

0,00016 0,00019 0,0003 0,0004 0,0006 0,0008 0,0015 0,0018 0,0028 0,0035 0,0048 0,0058 0,011 0,023 0,028 0,05 0,07 0,16 0,16 0,3 0,5 0,55 0,67 1,5 1,75 2,75 3 3,2 6,4 7,5 13 16

1.300 1.315 1.315 1.320 1.345 1.375 1.400 1.400 1.400 1.410 1.410 1.410 1.435 1.450 1.450 1.460 1.460 1.460 1.460 1.470 1.470 1.470 1.475 1.480 1.480 1.485 1.485 1.485 1.485 1.485 1.490 1.490

54 57 56 60 64 66 71 74 74 76 78 79 84 84 86 88 89 89,5 90,5 92 92,5 93 93 94 94 94,5 95 95 95 95,5 95,S 96

0,73 0,71 0,75 0,75 0,76 0,76 0,80 0,79 0,81 0,81 0,83 0,83 0,82 0,85 0,85 0,86 0,88 0,84 0,85 0,87 0,87 0,87 0,87 0,87 0,87 0,87 0.87 0.87 0,89 0,89 0,89 0,89

0,23 0,34 0,44 0,61 0.78 1,12 1,47 1,95 2,8 3,7 5,2 7,0 8,8 11.7 15,6 22 29 37 43 57 70 85 103 139 165 205 245 295 360 445 560 715

0,44 0,66 0,88 1,3 1,8 2,5 3,7 5,1 7,5 10 15 20 27 36 49 72 98 124 147 199 245 298 363 494 593 722 867 1.050 1.300 1.640 2.060 2.610

2,0 2,0 1,9 1,9 1,9 2,0 2,3 2,5 2,1 2,4 2,3 2,6 2,8 2,2 2,4 2,4 2,2 2,6 2,6 2,6 2,6 2,6 2,6 2,6 2,6 2,6 2,6 2,4 2.4 2,4 2,2 2,2

2,8 3,0 3,0 3,0 3,4 3,7 4,7 5,0 4,9 5,3 5,9 6,0 7,0 7,0 7,9 8,0 8,0 6,0 6,0 6,0 6,0 6,0 6,0 6,0 6,0 6,0 6,0 6,5 6,8 6,8 7,0 7,0

2,3 2,6 2,6 2,7 3,0 2,8 3,3 3,0 2,9 2,3 2,3 2,2 2,2 2,2 2,1 2,1 2,1 2,1 2,0 2,4 2,4 2,4 2.4 2,4

Al circular corriente por los conductores del rotor, aparecer en los mismos una fuerza cuyo sentido se obtiene aplicando la conocida ley vectorial (ley de Laplace):
F = (L x B) (4.3)

En la Figura 4.8 se muestra el sentido de la fuerza obtenida mediante la aplicacin de la ecuacin anterior. Obsrvese que fsicamente la fuerza se produce como consecuencia de una

266

MQUINAS ELCTRICAS

Figura 4.7.

Sentido de la corriente inducida en los conductores del rotor.

deformacin del campo inductor debido a la corriente que circula por el conductor del rotor. Si se tiene en cuenta este resultado y se lleva a la Figura 4.7, se deduce que el sentido de la fuerza es el de seguir al campo magntico giratorio del esttor. Multiplicando la fuerza anterior por el radio del rotor e integrando esta accin sobre el nmero total de conductores del rotor se obtendr el par total de la mquina, que tender a mover el rotor siguiendo al campo giratorio del esttor. El razonamiento anterior, aunque da los resultados correctos, no es del todo cierto, debido a que en la realidad, y como muestra la Figura 4.9a, los conductores del rotor estn situados dentro de unas ranuras, de tal forma que el campo B no atraviesa al conductor y en consecuencia, de acuerdocon la expresin(4.3) la fuerza resultantees nula. La explicacinde esta paradoja debe buscarse en la deformacinde las lneas de B al circular corriente por los conductores. En la Figura 4.9a se muestra el reparto de la induccin en la ranura y el diente cuando la intensidad en el conductor es cero; se observa que debido a la menor reluctancia de los dientes, las lneas de B tienden a concentrarse en ellos sin atravesar apenas al conductor. En la Figura 4.9b se muestra la forma de las lneas de induccin producidas nicamente por el conductor llevando corriente. En la Figura 4.9c se representa la resultante de ambos campos; se observa que la deformacin de las lneas de induccin es similar a la que se obtena para el caso de un conductor aislado (Fig. 4.8), apareciendo una fuerza resultante en el sentido indicado, pero con la diferencia fundamental de que esta fuerza acta realmente en los dientes y no en los conductores (lo que constituye un hecho afortunado, ya que si la fuerza actuara sobre los conductores comprimira los aislamientos de stos sobre los dientes, lo que sera perjudicial para la vida de los aislantes). El momento total de estas fuerzas origina el par de rotacin de la mquina, que obliga a girar al rotor siguiendo el movimiento del campo giratorio, de tal forma que cuanto ms se
B i

+(W
a) h)

Figura 4.8.

Sentido de la fuerza que se produce en un conductor del rotor.

CAPTULO

4.

MQUINAS ASNCRONAS

267

ESTTOR

ESTTOR

ESTTOR

deB
F

ROTOR

ROTOR

ROTOR

(1)

b)

e)

Figura 4.9. Determinacin del sentidode la fuerzaen un conductorsituadodentrode una ranura. aproxima a la velocidad ni del campo, tanto menor resulta la f.e.m. inducida en los conductores del rotor y, en consecuencia, resultan tambin reducidas las corrientes en el mismo, provocando esto una disminucin del par interno o par electromagntico del motor. Si como caso lmite, el rotor girase a la velocidad de sincronismo ni' no habra entonces movimiento del campo giratorio respecto del rotor, desapareciendo con ello la f.e.m. inducida (4.2) y como consecuencia de esto se anulara la corriente y el par. De este modo la velocidad de sincronismo ni constituye el lmite terico al que puede girar el rotor. El motor debe girar a una velocidad inferior a la de sincronismo (n < ni)' es decir, su velocidad de rgimen es asncrona. Como se ha indicado en el epgrafe 2.11.4, se conoce con el nombre de deslizamiento al cociente:
(4.4)

cuyo valor est comprendido en los motores industriales entre el 3 y el 8 por 100 a plena carga. Al aumentar la carga mecnica del motor, el par resistente se hace mayor que el par interno y el deslizamiento aumenta; esto provoca un aumento en las corrientes del rotor, gracias a lo cual aumenta el par motor y se establece el equilibrio dinmico de los momentos resistente y motor. Nota prctica de laboratorio: Para medir la velocidad de giro de un motor asncrono se utilizan tacmetros de tipo mecnico o en la actualidad de tipo ptico; sin embargo, esta medida no es lo suficientemente precisa para la determinacin del deslizamiento, ya que al ser la diferencia ni - n muy pequea cualquier error de lectura tiene como consecuencia un error importante en el deslizamiento. Es por este motivo que en los ensayos de laboratorio se mide la velocidad del rotor por mtodos estroboscpicos o modernamente con encders acopiados al eje del motor. De acuerdo con la expresin (2.155), las frecuencias de las corrientes del rotor estn relacionadas con la frecuencia del esttor por medio de la expresin:
(4.5)

En el caso de que el rotor est parado, se cumple n = O,es decir, s = 1, lo que indica que en estas circunstancias las frecuencias del esttor y del rotor coinciden, esto es:
(4.6)

268

MQUINAS ELCTRICAS

Si se denomina E2 el valor eficaz de la f.e.m. por fase del rotor, N2 al nmero de espiras por fase, <1\, al flujo mximo que lo atraviesa y K2 = Kd2Ka2 al coeficiente del devanado, se cumplir, de acuerdo con (2.113): (4.7) y de una forma similar, si se denomina El al valor eficaz de la f.e.m. inducida por fase en el esttor, se tendr: (4.8) donde NI es el nmero de espiras por fase y KI el factor de devanado correspondiente. Las expresiones (4.7) y (4.8) recuerdan las que se obtienen en un transformador donde el primario es el esttor y el secundario es el rotor. En efecto, el lector puede comparar la analoga existente entre las ecuaciones (4.7) y (4.8) con las (3.5) que se obtenan en el estudio de los transformadores. La diferencia estriba en que en los motores aparecen unos coeficientes de devanado KI y K2 que representan factores reductores (cuyos valores son menores, pero muy cercanos a la unidad) para tener en cuenta que las f.e.m.s. de las diversas espiras del devanado, al estar distribuido en ranuras por las periferias del esttor y del rotor, llevan un desfase entre s, lo que obliga a realizar una suma geomtrica (fasorial) de las f.e.m.s. inducidas en las diferentes bobinas, cosa que no ocurre en el caso de los transformadores, donde las f.e.m.s. de todas las espiras van en fase, por tratarse de un devanado concentrado, y la f.e.m. total se obtiene evidentemente como suma aritmtica de las f.e.m.s. individuales. Cuando el rotor gira a la velocidad n, en el sentido del campo giratorio, el deslizamiento ya no es la unidad y las frecuencias de las corrientes del rotor son iguales a!2. Denominando E2, a la nueva f.e.m. inducida en este devanado, se cumplir:
E2s = 4,44 K2!2N/I>m

(4.9)

y comparando (4.7) y (4.9) se obtiene: (4.10) expresin que relaciona las f.e.m.s. inducidas en el rotor, segn se considere que est en movimiento, E2" o parado, E; La f.e.m. anterior E2s producir unas corrientes en el rotor de frecuencia !2' de tal forma que stas a su vez crearn un campo giratorio, cuya velocidad respecto a su propio movimiento ser:
n2=-p

60!2

(4.11)

ya que el rotor est devanado con el mismo nmero de polos que el esttor. Como la mquina gira a n r.p.m., la velocidad del campo giratorio del rotor respecto a un referencial en reposo ser n2 + n. Si se tienen en cuenta las expresiones (4.1) y (4.5) resulta:
nI - n pnI pin, - n) !2=S!I =---=--'--nI

60

60

(4.12)

y al comparar con (4.11) se deduce:


n2 = nI - n

(4.13)

CAPTULO 4.

MQUINAS ASNCRONAS

269

En consecuencia, la velocidad absoluta del campo del rotor ser:


n2

+n

= (ni

- n) + n

= ni

(4.14)

lo que indica que el campo del rotor gira en sincronismo con el campo del esttor. Realmente, son las f.m.m.s. de ambos devanados las que interaccionan para producir el flujo resultante en el entrehierro. Debe hacerse notar que esta interaccin slo es posible si las f.m.m.s. estn enclavadas sincrnicamente, es decir, si las ondas de f.m.m. de esttor y rotor giran a la misma velocidad nI' lo que requiere segn (4.1), (4.11) Y(4.14) que el nmero de polos con el qne se confeccionan ambos arrollamientos sean ignales, lo que representa una exigencia constructiva de estas mquinas. No es necesario, sin embargo, que el nmero de fases del esttor y del rotor deban ser iguales, ya que el campo giratorio dentro del cual se mueve el rotor es independiente del nmero de fases del esttor. Los motores con rotor devanado o con anillos se construyen normalmente para tres fases, es decir, igual que las del esttor; sin embargo, el motor enjaula de ardilla est formado por un gran nmero de barras puestas en cortocircuito, dando lugar a un devanado polifsico, en general de m2 fases. Lo anterior es fcil de comprender: si se considera, por ejemplo, un rotor trifsico de dos polos y 6 barras o conductores en total, se habr formado un devanado trifsico en el que cada fase consiste en una sola espira (dos barras opuestas formaran la espira). Si considerando una mquina bipolar, el rotor tienen 10 barras, podemos decir que se ha logrado un devanado pentafsico con una espira por fase. En general se podr decir que si el rotor tiene B barras y 2p polos, se tendrn m2 fases:
m2=2p

(4.15)

donde cada fase est formada por una nica espira. Debe destacarse que cuando el rotor es de jaula de ardilla, las leyes del bobinado del esttor son las que determinan el nmero de polos del motor. En el rotor se obtienen corrientes por induccin, por lo que las diferencias de fase que aparecen entre las corrientes de las diversas barras del rotor coinciden con el ngulo elctrico que forman las mismas. As, si el rotor tiene 36 barras y el esttor tiene 2 polos, se habrn formado 18 fases, pero la misma jaula de ardilla en el interior de un esttor de 4 polos dara lugar a 9 fases, etc. En resumen, una jaula de ardilla es equivalente a un devanado rotrico de m2 fases de 1 espira/fase, donde mz viene expresado por la relacin (4.15). Cuando el rotor est bobinado (o con anillos) se dispone entonces de m2 fases (normalmente m2 = 3) con N2 espiras por fase. En ambas situaciones, el esttor siempre est formado por mi fases (generalmente mi = 3) con NI espiras por fase. Como quiera que el sentido de transferencia de la energa en un motor asncrono se produce de esttor a rotor por induccin electromagntica de un modo similar al que se obtena entre el primario y el secundario de un transformador, esto hace que la analoga se traslade no solamente a la simbologa de las magnitudes implicadas sino incluso tambin, en algunos autores, a las propias denominaciones. De ah que al estudiar motores asncronos se consideren homnimas las expresiones: esttor y primario, rotor y secundario. sta es tambin la causa de que todos los parmetros que aparecen en el esttor lleven el subndice 1 y los que aparecen en el rotor tengan el subndice 2. De hecho, y como se comprobar en el epgrafe 4.4, el circuito equivalente desarrollado para el transformador ser la gua para deducir el circuito equivalente del motor. Si se desea establecer las ecuaciones de comportamiento elctrico del esttor y del rotor, ser preciso tener en cuenta que los arrollamientos tienen unas

270

MQUINAS ELCTRICAS

Figura 4.10.

Circuito equivalente por fase del motor asncrono trifsico.

resistencias RI y R; ohmios/fase y que adems existen flujos de dispersin en los devanados del esttor y rotor que dan lugar a las autoinducciones Ldl y Ld2 En consecuencia, las reactancias de los arrollamientos en reposo, cuando la pulsacin de la red es wl = 21ffi' sern: (4.16) Sin embargo, al girar el rotor la frecuencia secundaria cambia al valor h, dando lugar a la reactancia XZs' que en funcin de X2 vale: (4.17) En la Figura 4.10 se muestra un esquema simplificado por fase del motor en el que se muestran los parmetros anteriores. Se observa que el primario est alimentado por la red de tensin VI y debe vencer las cadas de tensin en la impedancia de este devanado y el flujo comn a esttor y rotor induce en los arrollamientos f.e.m.s. El y Ezs' cuyas magnitudes se indican en las expresiones (4.8) y (4.9). Es importante que el lector examine en la Figura 4.10 las polaridades de las f.e.m.s. con la correspondencia de los terminales homlogos de esttor y rotor, sealados con un punto, y tambin los sentidos de las corrientes primaria y secundaria, que se han asignado de un modo similar al explicado en los transformadores, actuando la f.m.m. secundaria en contra (efecto desmagnetirante) respecto de laf.m.m. primaria (ya que la corriente II entra por punto y la corriente 12 sale por punto). La impedancia del rotor est formada por la resistencia Rz Yla reactancia X2s' estando este devanado cenado en cortocircuito. Las ecuaciones elctricas correspondientes se obtendrn aplicando el 2. lema de Kirchhoff a las mallas de primario y secundario, resultando: (4.18) Debe tenerse en cuenta adems que las frecuencias de ambos circuitos son diferentes y de valores y f2' respectivamente.

t.

4.4.

CIRCUITO EQUIVALENTE DEL MOTOR ASNCRONO

El circuito equivalente de un motor asncrono tiene como objetivo, al igual que en el caso de transformadores, el obtener una red que explique el comportamiento de la mquina, pero en la que no aparezca la accin transformadora entre los circuitos de primario y secundario, lo cual

CAPTULO 4.

MQUINAS ASNCRONAS

271

trae consigo el reducir las magnitudes de un devanado al otro, generalmente del rotor al esttor. En el transformador la operacin se haca directamente debido a que las frecuencias de los arrollamientos eran idnticas, pero en el motor aparentemente se tiene una dificultad, ya que las frecuencias de las corrientes del esttor y del rotor son diferentes, como se observa claramente en la Figura 4.10. En este circuito, la corriente que circula por el rotor, de acuerdo con la 2: ecuacin (4.18), ser: (4.19) que teniendo en cuenta las igualdades (4.10) Y (4.17) se convierte en: (4.20)

La transformacin de la ecuacin (4.19) en la (4.2Q) requiere una meditacin profunda, que es preciso que comprenda el lector, ya que nos va a permitir obtener ms'tarde el circuito equivalente del motor asncrono. Obsrvese que en la ecuacin (4.19) los parmetros de f.e.m. (Ez,) y reactancia (X2) estn referidos, de acuerdo con (4.9) y (4.17), a la frecuencia j, del rotor en movimiento. Sin embargo, la ltima ecuacin (4.20), que define la misma intensidad 12 (mdulo y fase) que (4.19), tiene unos parmetros de f.e.m. (E2) y reactancia (X2) que estn referidos, segn (4.7) y (4.16), a la frecuenciaj, del esttor. Ahora bien, de acuerdo con (4.5) Y(4.6), la frecuencia del rotor coincide con la del esttor cuando la mquina est parada. Por este motivo la ecuacin (4.20) describe en definitiva el comportamiento de un rotor pseudoestacionario con unos parmetros E2 y X2 referidos a rotor parado (independientes del deslizamiento), pero en el que la nueva resistencia del rotor para tener en cuenta estos cambios es ahora R/s en vez de R2 Para visualizar mejor estos cambios se han preparado los circuitos de la Figura 4.11. El caso a) es una repeticin del esquema de la Figura 4.10, en el que para mayor claridad se ha omitido la figura de la mquina. La ecuacin de la corriente en el rotor responde a la ecuacin (4.19). En la Figura 4.11b se ha modificado el circuito del rotor para adaptarlo a la ecuacin (4.20); ntese en este nuevo secundario que E2 y X; son, respectivamente, la f.e.m. y la reactancia del rotor en reposo, independientes del movimiento; el efecto de ste se incluye en R/s, de tal modo que la frecuencia de este rotor estacionario ficticio esf r- Para ver el cambio que se ha producido en la resistencia del rotor se puede hacer una transformacin de la ecuacin (4.20): (4.21)

En la Figura 4.11c se muestra el circuito correspondiente a la expresin anterior, que consta de la resistencia propia del rotor R2 ms otra resistencia Re de valor: (4.22)

272

MQUINAS ELCTRICAS

rotor fijo RI
Xl X2s

RZ

-rrr-

X2

R2

+
V,

, =r:
1,=1,
N, R,
X, X2
a)

R!s

b)

+
V,

Figura 4.11.

Desarrollo del circuito equivalente de un motor asncrono.

que depende del movimiento (del valor del deslizamiento). La resistencia Re se denomina resistencia de carga y representar el efecto equivalente a la carga mecnica que lleve el motor, o de otro modo la potencia elctrica disipada en Re (multiplicada por el nmero de fases) representar la potencia desarrollada por el motor en su movimiento de rotacin, es decir, la potencia mecnica en el eje. El circuito final obtenido de la Figura 4.11c no rene todava las ventajas analticas de un circuito elctrico, ya que existen acoplamientos magnticos. Es preciso, al igual que se haca en transformadores, reducir el secundario al primario (en nuestro, caso reducir o trasladar el rotor al esttor). Recurdese que en el caso de transformadores para hacer este cambio se requera considerar un nuevo secundario en el que se elega un nmero de espiras N~ = NI Y de este modo se modificaban las magnitudes secundarias a los nuevos valores:

E~ = mE2

t: z-- !3_

(4.23)

En la situacin del motor el proceso es ms complejo debido a la influencia de los factores de devanado y a que en general los nmeros de fases de esttor y rotor no coinciden. En la Figura 4.12a se ha repetido el esquema de la Figura 4.11 e y en la Figura 4.12b se ha utilizado un secundario equivalente en el que las magnitudes correspondientes se han sealado con tilde. En cada caso se han indicado en los circuitos el nmero de fases y factores de devanado tanto del esttor como del rotor.

CAPTULO 4.

MQUINAS ASNCRONAS

273

RI

XI

x2

R2

+
VI

It
;,

R~ =R;(}-I)
a)

RI

XI

b) NI

N=NI

Figura 4.12. Circuito equivalente reducido al esttor.

Para centrar el tema y a modo de recapitulacin: sabemos que disponemos de un motor asncrono cuyo circuito inicial equivalente por fase es el mostrado en la Figura 4.12a, con los siguientes parmetros:

(4.24) En la Figura 4.12b se ha mostrado un circuito equivalente, en el que se conserva intacto el primario pero en el que las magnitudes del secundario son: ROTOR EQUIVALENTE: m', K' N' } 2 2' 2 { E~,V~,I~,R~,X~ (4.25)

Si este nuevo rotor se quiere reducir al esttor, para conseguir una simplificacin posterior, se tendrn que adaptar sus parmetros a los del primario, lo que significa proceder a las igualdades siguientes: (4.26) Como consecuencia de ello, los nuevos parmetros del rotor sern: a)

Fuerza electromotriz E~

De acuerdo con (4.7), (4.8) Y(4.26) se podr escribir: E~

= 4,44 K~fIN~<1>m = 4,44 KJINI<1>m

= El

(4.27)

es decir, el nuevo rotor tiene una f.e.m. E~ igual a El' lo que permitir luego unir el primario con el secundario, que es lo que se trata de conseguir.

274

MQUINAS ELCTRICAS

Por otro lado, si se divide (4.8) entre (4.7) resulta:


El KINI -=--=m

E2

K2N2

(4.28)
v

que se denomina relacin de transformacin de tensiones. Por consiguiente, de acuerdo con (4.27) Y(4.28) se tiene: (4.29) que determina la f.e.m. del nuevo rotor E~ frente a la real E2 b) Corriente

I~

Si los dos secundarios de la Figura 4.12 son equivalentes, debern suministrar la misma potencia rotrica, es decir: (4.30) y teniendo en cuenta (4.29) da lugar a:
, m2 1 /2 = - mi m;

/2 =

mzK2N2
mlKINI

/2 = -

/2

(4.31)

donde m es igual a: mi
mlKINI

m2K2N2

mi =-m

m2

(4.32)
v

que se denomina relacin de transformacin de corrientes. e) Impedancias R~, X~, R;

Para ver la regla de transformacin de impedancias deber aplicarse el principio de igualdad energtica. Si se consideran, por ejemplo, las prdidas en el cobre en los circuitos de la Figura 4.12 se podr escribir: (4.33) y teniendo en cuenta (4.31) Y (4.32) resultar: (4.34) y de un modo anlogo: (4.35) donde el producto mvm constituye la denominada relacin de transformacin de impedancias. El lector notar que las reglas de transformacin aqu demostradas representan una generalizacin de las empleadas en el estudio de transformadores. Puede comprobar que si coinciden las fases del esttor y del rotor, coinciden entonces las relaciones de transformacin de tensiones y corrientes (m" = m), dando lugar a unas magnitudes transformadas similares a las (4.23) que se obtenan en transformadores. Teniendo en cuenta los valores transformados del nuevo rotor, y de acuerdo con la igualdad (4.27), se podrn unir los terminales A-A' del primario con los correspondientes a-a' del

CAPTULO

4.

MQUINAS ASNCRONAS

275

secundario (rotor) en la Figura 4.12b. El esquema correspondiente se muestra en la Figura 4.13a, donde se ha dibujado la rama paralelo por la que se derivar la corriente de vaco del motor de un modo anlogo a lo que ocurra en el caso de un transformador. La ecuacin que relaciona las corrientes de esttor y rotor se obtienen del esquema de la Figura 4.13a, aplicando el primer lema de Kirchhoff en el nudo A, lo que da lugar a: (4.36)* y las ecuaciones elctricas de primario y secundario correspondiente VI = El sern:

+ RII) + jXII
(4.37)

E~= R~I~ + R:I~ + jX~I~

Al igual que suceda con los transformadores, se obtiene una gran ventaja analtica si se traslada la rama de vaco a los terminales de entrada, lo que da lugar al circuito equivalente aproximado de la Figura 4.13b. Los errores que ahora se obtienen con esta aproximacin son superiores a los que resultaban en el transformador; esto se debe a la presencia del entrehierro en los motores, que hace que la corriente de vaco sea ahora del 35 al 40 por lOO de la asignada, mientras que en el caso del transformador era del orden del 3 al 8 por 100 de la asignada. Con el circuito equivalente aproximado se obtienen corrientes en el rotor que son apreciablemente ms altas que los valores reales. De todos modos, la aproximacin realizada es normalmente aceptable (al menos en clculos preliminares) para motores de ms de 10 kW, y es el que se seguir en este texto.
RI XI A X2

R2

+
V,

/2

R; =R;(}-l)
a)

A' R) XI

Xl

+
V,

V
Iv

!
Figura 4,13, Circuitos equivalentes:
a)

R;.

=R;(}-I)
b)

exacto:

b)

aproximado.

* En el ejemplo de aplicacin 4.1 se demuestra esta igualdad de un modo ms riguroso. Aqu se ha preferido seguir un mtodo ms didctico e intuitivo utilizando la analoga con el transformador.

276

MQUINAS ELCTRICAS

Circuito equivalente aproximado con ajuste de tensin *


Se puede conseguir un circuito equivalente aproximado ms preciso que el de la Figura 4.13b reduciendo el valor de la tensin de alimentacin, tal como se demuestra a continuacin. Si se parte del circuito equivalente exacto de la Figura 4.13a y se tiene en cuenta la La ecuacin (4.37), se puede escribir: (4.38) y dado que se cumple (4.36): (4.39) donde el valor de

lo segn el circuito equivalente exacto es igual a:


(4.40)

lo que representa que se desprecia la corriente /Fe frente a la corriente de imanacin /1" Sustituyendo (4.39) y (4.40) en (4.38) resulta:
VI = El + (RI + jXI)[~

}X"

I~J
0

(4.41)

y de acuerdo con la 2: ecuacin (4.37), que es simplemente la aplicacin del 2. lema de Kirchhoff al circuito de la Figura 4.13a, se tiene: (4.42) que al llevar a (4.41) nos da: (4.43) y dado que normalmente XI' RI y admitiendo que El es un poco menor pero con un valor cercano a V I en el rango de funcionamiento normal, la ecuacin anterior se transforma en:
V;

= VI

(1- ~J =[

R I + j XI + ~~+ j X~ ]

I~

(4.44)

que responde al circuito de la Figura 4.14, que es anlogo al de la Figura 4.13b pero en el que se ha corregido la tensin primaria para obtener valores ms reales de las corrientes /1 e l~. Hay que hacer constar que este circuito aproximado sirve para determinar magnitudes como: corrientes, potencias perdidas, potencias mecnicas, etc., pero si se desea calcular la potencia absorbida por el motor de la red se deber utilizar la tensin real VI aplicada al motor y no V;.

* Algunos autores utilizan un circuito equivalente simplificado a base de obtener el circuito equivalente de Thvenin entre A y A' en la Figura 4.13a.

CAPTULO 4.

MQUINAS ASNCRONAS

277

R +

!(
IFe

+
RFe X~

f~

Figura 4.14. Circuito equivalente aproximado corregido. EJEMPLO DE APLICACIN 4.1 Demostrar la ecuacin (4.36) partiendo de la igualdad defm.m.s. en vaco y en carga de un motor asncrono polifsico. SOLUCIN En el Captulo 2 se demostr que el valor mximo de la f.m.m. resultante producida por un devanado de m fases (expresin 2.75) era:
m i7=-F

(4.45)

donde Fm representa la f.m.m. mxima producida por el devanado de una fase, que segn (2.55) vale:
4 N/ji I F = - K --"--'----m

t:

2p

(4.46)

ya que i(t) = ji I cos on. Por consiguiente, la f.m.m. mxima resultante producida por un devanado de m fases es:
m 4 N/ji ,7=--K

2n

2p

=-m--

ji n

N/KI p

(4.47)

Si se considera que el motor trabaja en vaco, la corriente que absorber el motor de la red ser lo' Ypor el rotor circular una corriente despreciable. En consecuencia, la f.m.m. (giratoria) que producir el flujo del motor ser: ji ,7= n
m --

NKIO
p

(4.48)

Cuando el motor trabaja en carga, las corrientes que circulan por ambos devanados son II e 12, respectivamente, y teniendo en cuenta la diferencia entre el nmero de fases, espiras y factores de devanado del rotor y el esttor y adems el carcter desmagnetizante del rotor, se tendr una f.m.m. total: (4.49)

278

MQUINAS ELCTRICAS

que al igualar con (4.48) Y simplificando nos da:

m,N,K,1o = m.N, Kili de donde se deduce:

11l2N2K212

(4.50)

, mi=
que es la expresin (4.36) que se deseaba demostrar.

mJK,N, m2K2N2

(4.51)

4.5.
a)

ENSAVOS DEL MOTOR ASNCRONO*


b)

Los ensayos del motor asncrono permiten determinar los parmetros del circuito equivalente de la mquina. De forma similar al transformador, existen dos tipos de pruebas, denominadas:

ensayo de vaco y

ensayo de cortocircuito.

4.5.1. Ensayo de vaco o de rotor libre


Consiste en hacer funcionar el motor sin ninguna carga mecnica en el eje, es decir, la mquina trabaja a rotor libre. Se debe aplicar la tensin asignada al primario Vln, midiendo la potencia absorbida Po Y la corriente de vaco lo' Existe una gran diferencia entre este ensayo en el motor asncrono y el correspondiente del transformador. Obsrvese que si la mquina pudiera funcionar en esta prueba a la velocidad de sincronismo n = ni' el deslizamiento sera igual a cero, lo que indicara, en el circuito equivalente exacto de la Figura 4.13a, que la resistencia de carga R;' se hace infinita y, en consecuencia, l~ sera nula, y resultara un esquema similar al indicado en la Figura 3.21 a para el transformador (despreciando las prdidas en el cobre del primario). Sin embargo, y como ya se ha explicado anteriormente, el motor asncrono no puede girar a la velocidad de sincronismo, ya que l~ sera nula y no existira ningn par electromagntico en el eje. En estas circunstancias lo que sucede realmente es que el motor en vaco gira a una velocidad muy cercana a la del campo giratorio, lo que indica, desde el punto de vista del circuito de la Figura 4.l3a, que la resistencia de carga R; tiene un valor muy elevado pero no infinito; como no se ejerce ningn par de carga en el eje, la potencia disipada en esta resistencia representa la perdida en rozamiento y ventilacin del motor. Como quiera adems que l~ es de pequeo valor debido a la alta impedancia de R:, se podrn despreciar las prdidas en el cobre del devanado del rotor. Denominando Peul las prdidas en el cobre del esttor en este ensayo, PFe a las prdidas en el hierro y Pm a las prdidas mecnicas se cumplir: (4.52) Para determinar cada una de las prdidas anteriores es preciso completar el ensayo de vaco con medidas adicionales; las prdidas en el cobre ~ul pueden calcularse si se mide previamente la resistencia R I de cada una de las fases del esttor (esta operacin se realiza introduciendo c.c. en una fase del primario, midiendo tensin y corriente, la ley de Ohm da el valor de RI' En la prctica, para tener en cuenta el efecto pelicular o skin en los conductores, se suele aumentar el valor anterior entre un 10 y un 20 por 100). Para determinar PFe Y Pm es

* Para una discusin ms completa de estos ensayos consultar: IEEE Standard Test Procedure for Polyphase Induction Motors and Generators n." 112 (1996).

CAPTULO 4.

MQUINAS ASNCRONAS

279

a)

b)

Figura 4.15. Reparto de las prdidas en vaco en funcin de la tensin.

preciso alimentar el motor por una tensin variable, comenzando con el valor nominal o asignado Vln o algo superior y reduciendo hasta un valor que puede tomarse entre el 30 y el 50 por 100 de VI,,; en cada escaln de tensin se deben medir los valores de Po' lo YVi' Ya partir de ellos se deducen las prdidas PFe + Pm en cada etapa, de acuerdo con la ecuacin: (4.53) Al representar Pm + PFe en funcin de VI se obtiene una curva de tipo parablico, como se indica en la Figura 4.15a. Extrapolando esta curva, hasta que corte el eje de ordenadas, se obtiene para VI = O,el valor de las prdidas mecnicas p,,, ' ya que entonces las prdidas en el hierro son nulas al no existir flujo. El valor de las prdidas en el hierro a la tensin asignada vendr dado por la ordenada existente entre la curva Pm + PFe Yla horizontal Pm a esa tensin, como muestra la figura anterior. Para reducir errores en la extrapolacin de la curva parablica de la Figura 4.15a, es ms prctico representar P,n + PFe en funcin del cuadrado de la tensin Vi' tal como se muestra en la Figura 4.15b. En este caso la curva se transforma en una lnea recta, cuya ordenada en el origen representa la prdida mecnica del motor P Conocidas las prdidas PFe (separada de P",) se podr calcular la rama paralelo del circuito equivalente, como muestra la Figura 4.16, que absorber la potencia PFe, de acuerdo con las siguientes expresiones aproximadas:
In

(4.54a)

lo (por fase)

+
Vln

-R Fe

(por fase)

Figura 4.16.

Circuito equivalente en vaco.

280

MQUINAS ELCTRICAS

de donde se deduce:
(4.54b)

4.5.2. Ensayo de cortocircuito o de rotor bloqueado


Este ensayo se realiza bloqueando el rotor impidindole que gire, es decir, n = O,por lo que se tendr: s = 1, = O,lo que indica que el motor se comporta como un transformador con el secundario en cortocircuito. Al esttor se le aplica una tensin creciente, partiendo de cero, hasta que la corriente absorbida sea la asignada, 'Icc = 'In (por fase), midiendo a la vez la tensin aplicada VIce (jase) y la potencia absorbida P (total). La corriente de vaco es entonces despreciable frente a Tln debido a la pequea tensin necesaria, resultando el circuito equivalente de la Figura 4.17, que se obtiene al despreciar la rama paralelo del esquema de la Figura 4.13 y hacer = O,debido a que en estas condiciones el deslizamiento es la unidad. De las medidas efectuadas puede obtenerse el f.d.p. en cortocircuito:

R;

'o

R;

(4.55) y de aqu resultan los valores:

x; = XI + X = 1 sen qJcc
In

, 2

VIce

(4.56)

En consecuencia, el ensayo de cortocircuito permite obtener los parmetros de la rama serie del motor. Como quiera adems que RI se ha determinado con un ensayo en C.C., la primera ecuacin (4.56) permitir calcular R~, valor que es necesario conocer, ya que est incluido en R;. Si se desea utilizar en el estudio del motor el circuito equivalente exacto (Fig. 4.l3a), habr que repartir la cantidad Xcc entre XI y X~. A falta de informacin se elegir XI = X~. EJEMPLO DE APLICACIN 4.2
Un motor trifsico conectado en estrella, de 15 CV, 380 V, 50 Hz, 4 polos, ha dado los siguientes resultados en unos ensayos: VACO: 380 V, 3 A, 700 W. CORTOCIRCUITO: 100 V, 20 A, 1.200 W. Si la resistencia de cada fase del devanado primario es igual a 0,5 n y las prdidas mecnicas son de 250 W, calcular los parmetros del circuito equivalente del motor.
R'2

+
VIce

-----+
Ilcc= Iln

(por fase)

(por fase)

Figura 4.17. Circuito equivalente en cortocircuito.

CAPTULO 4.

MQUINAS ASNCRONAS

281

SOLUCIN
a)

Las prdidas en el cobre del primario en vaco son:


PCIlI = 3 . 0,5 . 32 = 13,5 W

Como quiera que las prdidas mecnicas son de 250 W, el valor de PFe, de acuerdo con (4.52), ser: PFe = Po - Peul
-

Pm = 700 - 13,5 - 250 = 436,50 W

El f.d.p. de la rama paralelo del circuito equivalente del motor, de acuerdo con (4.54), vale: cos
({Jo

436,5 ~. 2203

= 0,22 ; sen

({Jo

= 0,98

ya que la tensin simple aplicada en el ensayo ha sido: 380

j3 ~ 220 V
Los valores de JFe e JI" teniendo en cuenta (4.54), sern:
JFe = 3 0,22 = 0,66 A ; JI' = 3 . 0,98 = 2,94 A

y en consecuencia de las ecuaciones (4.55) se deduce:


RFe = -

220 = 333,33 0,66

220 X =-=7483 l' 2,94

'

b)

100 La tensin de cortocircuito por fase es igual a: Vice = (:; = 57,74 V, Yel f.d.p. en ese estado ser: v3 cos
({Jee

l.200 = 0,35 ; 3 57,54' 20

({Jec

= 0,94

Por tanto, los parmetros de la rama serie del circuito equivalente del motor, de acuerdo con (4.56), sern: 57,74 R I + R~ = -. 0,35 = 1,01 20

57,74 X I + X' = --'20 094 2 ' = 271 ,

4.6. BALANCE DE POTENCIAS


En un motor asncrono existe una transformacin de energa elctrica en mecnica, que se transmite desde el esttor al rotor, a travs del entrehierro, y el proceso de conversin est inevitablemente ligado con las prdidas en los' diferentes rganos de la mquina. Vamos a analizar el balance de energa que se produce en el funcionamiento del motor.

282

MQUINAS ELCTRICAS

La potencia que la mquina absorbe de la red, si VI es la tensin aplicada por fase, I la corriente por fase y <p 1 el desfase entre ambas magnitudes, ser:
1

(4.57) Esta potencia llega al esttor, y una parte se transforma en calor por efecto Joule en sus devanados, cuyo valor es: (4.58) y la otra parte se pierde en el hierro: PFel. La suma de ambas prdidas representa la disipacin total en el esttor Ppl: (4.59) Como quiera que las frecuencias de las corrientes en el rotor son muy reducidas, debido a que los deslizamientos en la mquina suelen ser pequeos (por ejemplo, para s = 5 por 100 con JI = 50 Hz, resulta unaj, = 2,5 Hz JI)' se considera entonces que prcticamente es el hierro del esttor el nico origen de las prdidas ferromagnticas. De acuerdo con el circuito equivalente del motor de la Figura 4.13a, se podr escribir: (4.60) La potencia electromagntica que llegar al rotor a travs del entrehierro, y que denominaremos Pa (potencia en el entrehierro), tendr una magnitud: (4.61) En el rotor aparecen unas prdidas adicionales debidas al efecto Joule, Pcu2' Yde valor: (4.62) Las prdidas en el hierro del rotor son despreciables debido al pequeo valor deJ2. La potencia que llegar al rbol de la mquina, denominada potencia mecnica interna, Pm, ser:
P =P-P mI a
cu2

(4.63)

que teniendo en cuenta el significado de la resistencia de carga podr poner:

R; del circuito equivalente, se


(4.64)

La potencia til en el eje ser algo menor, debido a las prdidas mecnicas por rozamiento y ventilacin; denominando p,,, a estas prdidas y P, a la potencia til, resultar:

P, = P; - P;

(4.65)

En la Figura 4.18 se muestra, en la parte superior, el circuito equivalente exacto del motor y en la parte inferior un dibujo simplificado de la mquina. En cada caso se muestran, con flechas, las prdidas que se producen en las diversas partes del motor. Es instructivo que el lector determine las potencias con el circuito equivalente y verifique claramente la situacin de las mismas en laJigura real. Obsrvese en ambos casos que se obtiene una potencia til de

CAPTULO

4.

MQUINAS ASNCRONAS

283

S
R

+
~v

pp: Prdidas en el esttor

frB
~

Prdidas mecnicas

Potencia elctrica ROTOR (jaula de ardilla) Potencia de entrehierro (potencia que llega al rotor) ESTATOR

Potencia til

Figura 4.18.

Circuito equivalente exacto y distribucin de las potencias en el motor.

salida a partir de una potencia de entrada PI' El rendimiento del motor vendr expresado por el cociente: (4.66) Existen algunas relaciones tiles entre las potencias anteriores; por ejemplo, el cociente entre Pcu2 Y Pmi, teniendo en cuenta (4.62) y (4.64), resulta ser:
~'u2 S

Pmi

1- s

(4.67)

Por otra parte, la potencia que atraviesa el entrehierro Pu se puede poner: (4.68) que relaciona Pa con Pcu2' Estas relaciones facilitan al estudio analtico de la mquina y la realizacin de ejercicios prcticos.

284

MQUINAS ELCTRICAS

EJEMPLO DE APliCACIN 4.3


Un motor de induccin trifsico de 6 polos, 50 Hz, absorbe una potencia de 20 kW, cuando gira a 960 r.p.m. Las prdidas totales del esttor son 0,5 kWy las de rozamiento y ventilacin son de 1kW. Calcular: a) El deslizamiento. b) Prdidas en el cobre del rotor. c) Rendimiento.

SOLUCIN
a)

La velocidad de sincronismo del motor es:


ni

= --

60fl p

= --

60 50 = 1.000 r.p.m. 3

En consecuencia, el deslizamiento valdr:


s

= ni - n =
ni

1.000 - 960 = 4 % 1.000

b)

La potencia que llega al rotor ser:


Pa = PI - Ppl = 20 - 0,5 = 19,5 kW

resultando un valor de Peu2' de acuerdo con (4.68):


Peu2 = sPa = 0,04 . 19,5 = 0,78 kW.

c) La potencia til en el rbol de la mquina tendr una magnitud:


Pu = Pa - Peu2
-

P; = 19,5 - 0,78 - 1 = 17,72 kW

En consecuencia, el rendimiento de la mquina ser:

r; 17,72 ti = P = 20 = 88,6 %
I

EJEMPLO DE APliCACIN 4.4


Un motor asncrono trifsico de rotor devanado, 2p = 4 polos, se conecta a una red trifsica de 380 V de tensin compuesta. El esttor y el rotor estn conectados en estrella. La relacin de transformacin de tensiones coincide con la de corrientes y es igual a 2,5. Los parmetros del circuito equivalente del motor por fase son: R = 0,5 O; X = 1,5 O; Rz = 0,1 O; X2 = 0,2 O; RFe = 360 O; X~ = 40 O. Las prdidas mecnicas son de 250 W. Si el deslizamiento a plena carga es del 5 por 100: a) Calcular, utilizando el circuito equivalente exacto del motor: 1) Corriente del esttor; 2) Corriente del rotor; 3) Corriente lo; 4) Prdidas en el hierro; 5) Potencia activa y reactiva absorbida por el motor de la red; 6) Potencia mecnica interna; 7) Potencia mecnica til; 8) Rendimiento del motor; 9) Corriente de arranque y su f.d.p. Repetir el problema utilizando el circuito equivalente aproximado del motor.

b)

CAPTULO 4.

MQUINAS ASNCRONAS

285

SOLUCIN a) Circuito equivalente exacto:


1.

El circuito equivalente exacto del motor por fase es el mostrado en la Figura 4.19, en el que la rama paralelo del circuito se sita entre el esttor y el rotor. Los clculos elctricos en esta situacin son ms laboriosos que con el circuito equivalente aproximado, como se va a apreciar en el desarrollo del problema. La impedanciadel rotorreducida al esttor,teniendoen cuenta que m; = mi = 2,5, es: R~ = m2R2

= 2,52 0,1 = 0,625 n ;

X~= 2,52 0,2

1,25 n

Figura 4.19.

El valor de la impedancia de carga, teniendo en cuenta que el deslizamiento es igual a 0,05, ser: R;

= R~ (~s

1)

= 0,625 (_1_ 0,05

1)

11,875 n

Conviene comenzar el problema calculando el valor de la impedancia total del rotor Zr mostrada en la Figura 4.19: Zr

= (R~ + j X~) + R; = (0,625 + j 1,25) + 11,875 = 12,5 + j 1,25 = 12,56 L


Y, =

5,71 n

que corresponde a un admitancia Yr:

- =

r;

12,56 L 5,71

= 0,0796 L

5,71 = 0,0792 - jO,0079 siemens

La admitancia total del paralelo Yp mostrada en la Figura 4.19 vale: 1 1 1 1 Yp = - + - + Yr = - + - + (0,0792 - jO,0079) = RFe jx, 360 j40
= 0,082 - jO,033 = 0,088 L -21,9 siemens

que corresponde a una impedancia Zp = l/Yp = 11,36 L 21,9 n. De este modo la impedancia total del motor que se observa desde los terminales de entrada es:
Ztotal

= (R

+ j XI) + Zp = (0,5 + /1,5) + 11,36 L 21,9 = (0,5 + j 1,5) + + (10,54 + j4,24) = 11,04 + j5,74 = 12,44 L 27,47 n

286

MQUINAS ELCTRICAS

Si se toma como referencia la tensin de alimentacin: 380 VI= j3LOO=219,39LOoV la corriente del esttor ser:
II =

V
_1 ZtoTUI

='

21939 L 0 12,44 L 27,47

= 17,63 L -27,47 A

2. Para determinar la corriente I~ del rotor debe calcularse previamente la d.d.p. entre los nudos A y B del circuito equivalente de la Figura 4.19, y cuyo valor es: VAB

= Zpll =
12 =
I

11,36 L 21,9 . 17,63 L -27,47

= 200,28 L -5,57

De este modo la corriente del rotor reducida al esttor vale:

VAB

r;

200,28 L -5,57 12,56 L 5,71

15,95 L -11,28 A

por lo que la corriente real del rotor ser: 12 = I~m = 15,95 L -11,28 . 2,5 3. Las corrientes por las ramas en paralelo son: VAB IF = e

= 39,875 L -11,28

RFe

200,28 L -5,57 360 200,28 L -5,5r j40

0,556 L -5,57 A
::::; 5 L -95,57 A;

I =- = !1 jX/1

VAB

por lo que la corriente lo valdr: lo

= IFe

+ I, = 0,556 L -5,57 + 5 L -95,57

= 5,03 L -89,23

4. De acuerdo con el apartado anterior, las prdidas en el hierro sern:


PFe

= 3

VAB

. Fe

= 3200,280,556 = 334,1 W
::::;

que tambin se puede obtener del siguiente modo:


PFe = 3RFJie

= 3 . 360

0,5562

334,9 W ::::; 334,1 W (errores de redondeo)

5. La potencia compleja absorbida por el motor de la red es: SI = 3VII~

= 3219,39

LO . 17,63 L +27,47 = 10.295,4 + j 5.352,6

11.603,7 L 27,47

lo que significaque el motor absorbeuna potenciaactiva PI = 10.295,4W y una potencia reactiva QI = 5.352,6 Var, siendo el f.d.p. del motor igual a cos 27,47 = 0,887. 6. La potencia mecnica interna del motor se calcula a partir de la expresin:
Pm; = 3R;~2 = 3 . 11,875 . 15,952
::::;

9.063,1 W

CAPTULO 4.

MQUINAS AsNCRONAS

287

7. Como quiera que las prdidas mecnicas son de 250 W, la potencia til en el eje del motor es: Pu = Pm
-

P'1I = 9.063,1 - 250 = 8.813,1 W

8. El rendimiento del motor es el cociente entre la potencia til en el eje y la potencia activa absorbida de la red, que de acuerdo con los resultados anteriores ser igual a:
1J

= Pu = 8.813,1 r, 10.295,4

85,6%

9. La corriente de arranque se obtendr del circuito de la Figura 4.19 haciendo s = 1. La impedancia del rotor vale en este caso: Z = R~ +l X~= 0,625

+l 1,25 = 1,398 L 63,43 Q

que corresponde a una admitancia: Y, = IIZr = 0,716 L -63,43 siemens. La admitancia del paralelo Yp es igual a: 1 1 . Yp = - + - + 0,716 L -63,43 = 0,739 L -64,09 siemens 360 }40 que representa una impedancia de Zp = l/Yp = 1,353 L 64,09 = 0,591 +l 1,217 Q. Por consiguiente, la impedancia total del motor ser:
Ztotol = (R + }X) + Zp = (0,5 +) 1,5) + 0,591 +} 1,217 = 2,93 L 68,12 Q

y por tanto la corriente absorbida por el motor en el arranque ser: 219,39 LO Iarranque= 2,93 L 68,12 = 74,93 L -68,12 A Es decir, la corriente de arranque es de 74,93 A Ytiene un f.d.p.: cos ({J = cos 68,12 = = 0,373. Obsrvese que la corriente de arranque es varias veces la asignada. En nuestro caso, teniendo en cuenta los resultados de los apartados 1) y 9), resulta ser: Iarranque 74,93 --=--=4,25 I 17,63 b) Circuito equivalente aproximado:

En este caso la rama paralelo del circuito equivalente de la Figura 4.19 se debe trasladar a la entrada del motor, lo que simplificar enormemente los clculos correspondientes. Debe hacerse constar que el uso de este circuito equivalente aproximado da resultados algo superiores a los reales. Conviene que el lector compare los resultados obtenidos con ambos procedimientos. 1. Corriente del esttor: El procedimiento de clculo es directo y requiere determinar las corrientes I~, IFe e Iw cuyos valores son: 1=
I

21939 LO ' (0,5+}1,5)+(0,625+}1,25)+11,875'

= 16 5 L -11 94 A ,

288

MQUINAS ELCTRICAS

I Fe

219,39 L 0

360

= O61 ' L 0
[Ji

. I Ji ,

219,39 LO j 40

= 5'48 L -90

lo que da lugar a una corriente primaria (esttor): I = [Fe + 2. La corriente del rotor ser: 12 = I~m = 16,5 L -11,94 . 2,5 + I~ = 18,96 L -27,96

= 41,25 L -11,94
-90 A

3. De acuerdo con el resultado de las corrientes del apartado 1) se tena:


IFe

= 0,61 LO

[Ji

= 5,48 L

por lo que la corriente de vaco ser: lo = 4.


[Fe

+ [/' = 0,61 LO + 5,48 L -90

= 5,51 L

-83,65 A

Las prdidas en el hierro sern: PFe = 3VJFe

= 3 . 219,39'

0,61

= 401,5 W =

5. La potencia compleja absorbida por el motor de la red ser:


SI

= 3VI~ = 3

219,39 LO '18,96 L +27,96 = 12.478,9 L 27,96 ~ 11.022 + j5.851

lo que significa que el motor absorbe una potencia activa PI = 11.022 W, una potencia reactiva QI = 5.851 VAr, siendo el f.d.p. del motor igual a cos 27,96 = 0,883. 6. La potencia mecnica interna es, segn (4.64): Pmi = 3 . 11,875 . 16,52 7. La potencia mecnica til valdr:
~

9.700 W

P"

= 9.700 -

250 = 9.450 W

Tngase en cuenta que las prdidas mecnicas eran de 250 W. 8. El rendimiento del motor ser:
r

= - = -PI

r;

9.450

11.022

= 85,76 %

9. La corriente de arranque se obtiene del circuito equivalente aproximado haciendo s = 1, resultando ser: I1 = IFe + lJi + I~ I1 = (0,61 - j 5,48) + 219,39 LO 0,5 + j 1,5 + 0,625 + j 1,25

= 79,16 L -68,84

que corresponde a un f.d.p.: cos q> = cos 68,84 = 0,36. Si el lector compara los resultados obtenidos por ambos procedimientos, observar que son bastante parecidos. Con el circuito equivalente aproximado ~e obtienen valores algo ms elevados que los reales. El mtodo aproximado es mucho ms rpido de resolver y de ah sus ventajas prcticas en la resolucin de problemas. Sin

CAPTULO

4.

MQUINAS AsNCRONAS

289

embargo, si la corriente de la rama paralelo lo fuera ms elevada, se observaran mayores diferencias, por lo que en ese caso debera utilizarse forzosamente el circuito equivalente exacto del motor asncrono para no cometer graves errores.

4.7.

PAR DE ROTACIN

4.7.1. Generalidades
Si es P" la potencia mecnica til desarrollada por el motor y n la velocidad en r.p.m. a la que gira el rotor, el par til T (tarque en ingls) en N.m en el rbol de la mquina ser el cociente entre Pu y la velocidad angular de giro w = 2nn/60, expresando n en r.p.m.:
P T= __ u_

n 2n:-

(4.69)

60

Si se desprecian las prdidas mecnicas del motor, la potencia til coincide con la mecnica interna y el par anterior se puede poner tambin:
T=_"_lI_ n 2n-

(4.70)

60

De la definicin de deslizamiento se deduce:


s nI = --

n
=>

nI

= nI (l - s)

(4.71)

y la expresin del par se convierte en:


T = ----''''---

r:

(4.72)

2n:- (l -s)

nI

60

y teniendo en cuenta (4.68) resulta:


T=--

Po
nI

(4.73)

2n:-

60

Las dos ecuaciones (4.70) y (4.73) expresan el par de rotacin del motor, en funcin de la potencia mecnica interna Pmi o de la potencia que atraviesa el entrehierro Pa' La ltima expresin es muy til, ya que en el denominador figura la velocidad del campo giratorio, que para un determinado motor es constante si permanece invariable la frecuencia de alimentacin, y en consecuencia puede tomarse como medida del par el producto T 2n:nl60, es decir, la potencia que desarrollara el par si la mquina girase a la velocidad de sincronismo. Por este motivo, se dice entonces que el par est expresado en vatios sncronos, y segn (4.73), el par as evaluado es precisamente igual a la potencia en vatios que el campo giratorio transmi-

290

MQUINAS ELCTRICAS

te al rotor. Este concepto es importante en el estudio del diagrama circular de la mquina asncrona, como se ver en el epgrafe 4.8 de este captulo. Teniendo en cuenta (4.62) y (4.68), la expresin del par (4.73) se puede poner:

T=---

(4.74)

pero de acuerdo con el circuito equivalente aproximado de la Figura 4.l3b, el mdulo de la corriente I~vale: (4.75)

donde se ha llamado Xcc a la reactancia XI + X~. Sustituyendo (4.75) en (4.74) se obtiene:

(4.76)

que expresa el valor del par electromagntico producido por la mquina en funcin de los parmetros del motor. Se observa en la expresin (4.76) que el par se hace cero cuando s = O y s = oo; por otra parte, la caracterstica T = fes) presenta unos mximos en los que debe cumplirse:

-dT = O
ds

=;.

s
'"

R~ = +----===

JR2 I

+x2 ce

(4.77)

El signo negativo significa un funcionamiento como generador, ya que entonces la velocidad de rotacin es superior a la del campo giratorio. Sustituyendo (4.77) en (4.76) se obtiene el valor del par mximo de la mquina: Tmx =
mlV~ + ------'--'-----I

27! ~ 2[ R + J R~ + X(2cJ

(4.78)

El + expresa el valor del par mximo como motor y el signo - indica la magnitud del par mximo como generador. Observamos en la expresin anterior que el par mximo no vara cuando cambia la resistencia del rotor; sin embargo, y de acuerdo con (4.77), el deslizamiento al cual se obtiene el par mximo es proporcional a la resistencia del rotor, y de aqu se deriva una cuestin tcnica de gran importancia prctica, ya que variando la resistencia del rotor por introduccin de resistencias adicionales (caso aplicable nicamente a mquinas con rotor devanado o de anillos) se puede conseguir que el par mximo se obtenga a una velocidad deseada; en particular, si se logra hacer s", = 1, se obtiene el par mximo en el arranque. En la Figura 4.20 se ha dibujado el diagrama T = fes), denominado par-deslizamiento ~ par-velocidad de la mquina asncrona. Se han representado dos curvas diferentes a y b segn sea el

CAPTULO

4.

MQUINAS ASNCRONAS

291

111

(3
GENERADOR

-1

-2 deslizamiento (s) velocidad (11)

Figura 4.20.

Curvas par-velocidad de una mquina asncrona. Zonas de funcionamiento.

valor de la resistencia del rotor. Se observa, como se acaba de indicar, que el par mximo no se altera por la variacin de la resistencia del rotor; sin embargo, s que cambian los valores del deslizamiento a los cuales se obtienen los pares mximos. La curva par-velocidad que se obtiene con la resistencia propia del rotor constituye la caracterstica natural del par, mientras que las curvas par-velocidad que resultan con la introduccin de resistencias adicionales se denominan caractersticas artificiales. Si para la caracterstica natural se tiene, segn (4.77), un deslizamiento para par mximo: s
=-===
m 2

J RI +X

R~

(4.79)
2 cc

para las caractersticas artificiales, si se denomina R~2 la resistencia total reducida del rotor, que incluye la propia del rotor ms la adicional por fase que se introduce por los anillos, se tendr: (4.80) Dividiendo (4.80) por (4.79) resulta: (4.81) Es decir, el cociente entre el deslizamiento para par mximo (denominado tambin crtico) de la caracterstica artificial y el de la caracterstica natural es igual a la razn de las resistencia totales del rotor para cada caso.

292

MQUINAS ELCTRICAS

A veces es interesante referir el par de un motor al par mximo y al deslizamiento crtico sm' As, si dividimos (4.76) entre (4.78), despus de algunas simplificaciones se obtiene la
frmula de Kloss: T -+2as sm
S

(4.82) +m

sm
S

donde s es el deslizamiento del motor para el cual se obtiene el par T; el parmetro a es igual a:
a=-

RJ R' 2

(4.83)

Si se ha construido la curva de par natural (con la resistencia interna del rotor R~), se puede construir, con ms sencillez, una artificial para cualquier resistencia total del rotor R~2 sin tener que realizar nuevamente los clculos mediante la frmula general del par (4.76). Tngase en cuenta que si se denomina T' al par con la curva artificial, s; el deslizamiento mximo correspondiente y s' el deslizamiento que se tiene para el par T', resultar una ecuacin similar a (4.82):
T'

2(1 + a's~)
-+2a's' , sm

i.:

s'

s'. m,+..!!'.
S

RJ a' =R' 72

(4.84)

ya que los pares mximos de la curva natural y artificial son idnticos porque segn (4.78) no dependen de la resistencia del rotor. Los pares del motor coincidirn para las caractersticas natural y artificial cuando (4.82) sea igual a (4.84), es decir: 2(1 + a's~)
-+2a's'+..!!'. , m, sm

s'

s'.
S

(4.85)

y teniendo en cuenta que segn (4.81), (4.83) Y(4.84) se cumplir: (4.86) la ecuacin (4.85) se transforma de este modo en: (4.87) y despus de algunas simplificaciones resulta: s s' (4.88)

y teniendo en cuenta (4.81) y (4.88) se cumplir la igualdad: s (4.89)

R'2

CAPTULO 4.

MQUINAS ASNCRONAS

293

Es decir, a igualdad de pares en las curvas caractersticas artificiales y natural de un motor asncrono, el deslizamiento en la caracterstica artificial respecto a la natural coincide con el cociente de resistencias totales en el rotor correspondientes a cada curva. A veces es interesante, para realizar clculos previos, tener expresiones aproximadas del par. Si en la frmula de Kloss (4.82) se desprecia la resistencia del esttor *, se transforma en:
T 2

(4.90)

Para deslizamientos pequeos (s sm) la expresin anterior se convierte en: --=T 2s

(4.91)

que es la ecuacin de una recta (vase Fig. 4.20). Para deslizamientos grandes (s sm)' la frmula (4.90) se transforma en: (4.92) que es la ecuacin de una hiprbola. En la prctica, la adaptacin a esta curva slo se cumple en la zona de arranque y de frenado del motor.

4.7.2. Tipos de funcionamiento de la mquina asncrona


Las ecuaciones que rigen el comportamiento de la mquina asncrona ya han sido expuestas en los epgrafes 4.6 y 4.7.1; vamos a destacar aquf tres de ellas: (4.64), (4.68) Y(4.73), que nos sern de utilidad para comprender los sentidos de transferencia de energa entre la red (terminal elctrico) y la carga (terminal mecnico) y tambin el signo del par electromagntico generado. Estas ecuaciones son: Potencia mecnica interna: Potencia de entrehierro:
p = Pmi 1- s
Q

(4.93)

Par electromagntico:

T=~

2n-

ni

60

En la Figura 4.20 se han sealado la forma de las curvas par-velocidad de una mquina asncrona o de induccin. Dependiendo del valor del deslizamiento, se distinguen tres zonas distintas que caracterizan tres modos de funcionamiento, a saber: motor, generador y freno.

* En los motores de potencias medias y pequeas se cumple de forma aproximada: R, motores de gran potencia se pueden despreciar R I frente a R~.

= R~,mientras que para

294

MQUINAS ELCTRICAS

a)

Rgimen motor

Representa el modo de funcionamiento ms caracterstico y corresponde al rango de deslizamientos comprendidos entre O y 1. De acuerdo con la expresin (4.71), que nos da la velocidad del rotor en funcin de s:
n=n,(I-s)

(4.94)

el rango anterior corresponde a velocidades comprendidas entre n, (velocidad de sincronismo) y O (parada). Si se tiene en cuenta el campo de variacin de s en el rgimen motor, las ecuaciones (4.93) nos indican que cuando la mquina trabaja como motor se tiene:
a) b) c) La potencia mecnica interna es positiva. Es decir, se transmite energia mecnica al eje. La potencia en el entrehierro es positiva, lo que indica un par electromagntico positivo. Si la potencia de entrehierro es positiva, quiere decir que se transfiere energa en el sentido esttor-rotor. Como quiera adems que la energa en el esttor procede de la red, la potencia elctrica absorbida tendr el sentido de p',. En definitiva, la potencia elctrica que absorbe la mquina de la red es positiva.

De acuerdo con las conclusiones anteriores, en la Figura 4.21 se ha representado simblicamente una mquina asncrona, en la que para mayor claridad se ha separado el esttor del rotor. Se han sealado con flechas los sentidos de las potencias puestas en juego. Se observa que en este comportamiento de la mquina de induccin como motor, la transferencia de energa se produce en el sentido terminal elctrico-terminal mecnico. La mquina convierte la energa elctrica en mecnica. En la Figura 4.22 se muestra la curva par-velocidad correspondiente a este rgimen de trabajo, que es una ampliacin de la curva de la Figura 4.20 correspondiente a la zona de deslizamientos comprendida entre O y 1. Obsrvese la doble escala utilizada en el eje de abscisas calibrada en deslizamientos o en velocidades del rotor. Los puntos ms caractersticos son: l. Punto O. Funcionamiento en sincronismo: s = O; T = O, en este caso la velocidad de rotacin del motor es la de sincronismo, lo que como se demostr en el epgrafe 4.2 constituye una imposibilidad fsica. De hecho, el que el par electromagntico producido resulte igual a cero est corroborando tal afirmacin, porque la mquina a esta velocidad no podra ni tan siquiera vencer los pares resistentes de rozamiento.

ROTOR

Figura 4.21.

Reparto de potencias en el funcionamiento como motor.

CAPTULO

4.

MQUINAS ASNCRONAS

295

velocidad (n)

Figura 4.22. Curvaspar-velocidad de un motorasncrono. 2. Punto A. Rgimen asignado o nominal: s = s,,; T = T", que corresponde a la velocidad asignada y al par nominal o asignado o de plena carga, se produce generalmente para deslizamientos comprendidos entre el 3 y el 8 por 100,que representan velocidades cercanas a la de sincronismo que se sitan en la parte derecha de la curva de la Figura 4.22. 3. Punto C. Funcionamiento con par mximo: s = s",; T = Tm, representa el par mximo o crtico del motor y se produce para deslizamientos comprendidos entre el 15 y el 30 por 100. 4. Punto D. Rgimen de arranque: s = 1; T = Ta, en este caso la velocidad es cero y corresponde al par de arranque. Se observa en la Figura 4.22 que el par mximo divide a la curva en dos partes, una estable O< s < sm y otra inestable sm < s < 1; la zona estable corresponde a la parte de la curva en la que se obtiene un aumento del par conforme el motor disminuye su velocidad, lo que implica una dT/ds positiva (debe tenerse en cuenta, para evitar confusiones, que en la curva de la Figura 4.22, la escala de deslizamientos es decreciente y por ello la zona estable corresponde a la parte derecha de la curva). El motor en su zona estable presenta una caracterstica dura o rgida, lo que significa que la velocidad disminuye muy poco con el par y es por ello que se puede decir que estas mquinas giran a una velocidad asncrona prcticamente constante. (De ah que no se hayan empleado este tipo de motores en accionamientos que requeran regulacin de velocidad. Modernamente, debido a los avances de la electrnica de potencia, y tal como se explica en el Captulo 7, se ha logrado regular la velocidad de estos motores variando la frecuencia de alimentacin con convertidores electrnicos.) Para examinar el comportamiento de un motor asncrono cuando pone en movimiento una carga mecnica en el eje, es preciso conocer la dependencia de velocidad de rotacin con el par resistente ofrecido por la carga. A estos efectos los diferentes mecanismos de produccin se pueden clasificar a grandes rasgos en los dos tipos siguientes:
a)

Cargas con par resistente constante, independiente de la velocidad. La curva parvelocidad de estos mecanismos est representada por la horizontal b en la Figura 4.22. Este tipo de par resistente lo poseen las gras, ascensores, montacargas, mecanismos de avance de mquinas, cintas transportadoras en las que permanezca constante el material que se desplaza y otros tipos de mecanismos en los que el par resistente principal sea el de rozamiento.

296

MQUINAS ELCTRICAS

b)

Cargas con par resistente creciente con la velocidad. Generalmente son mecanismos en los que el par resistente es funcin del cuadrado de la velocidad y por ello presentan una curva de tipo parablico (curva e en la Fig. 4.22). Este tipo de par se presenta en las bombas centrfugas, ventiladores, hlices, etc., es decir, en el movimiento de fluidos. Se conocen tambin como cargas con par resistente tipo ventilador.

Bajo la existencia de los pares motor o electromagntico T y resistente o de carga T, se producir el comportamiento dinmico del motor, que responder a la ecuacin clsica de la mecnica:
T- T = Jr

dw dt

(4.95)

donde J es el momento de inercia de las partes giratorias, que incluye la del rotor del motor ms el mecanismo de accionamiento, y w la velocidad angular de giro del rotor (en rad/s). En el epgrafe 4.12 se har un estudio detallado de la dinmica del motor asncrono. Ahora es suficiente darse cuenta, segn la ecuacin (4.95), que si el par motor T es superior al par resistente Tr, se producir un momento resultante T-Tr > Oque har que el motor se acelere (dwldt> O);la aceleracin continuar hasta que ambos pares se hagan iguales y en este momento la mquina habr adquirido su velocidad de rgimen. En la Figura 4.22 se observa que si el motor mueve un par resistente constante Tr, el estado de equilibrio se conseguir cuando se alcance el punto A, punto de interseccin de las curvas del par motor con la del par resistente, donde ambos pares se hacen iguales. Si la carga tiene un par resistente tipo ventilador (curva e de la Fig. 4.22), el estado de equilibrio final se consigue en el punto B. Es evidente, en cualquier caso, que para que la mquina inicie su marcha el par de arranque desarrollado por el motor debe ser superior al par resistente que en este momento ofrece la carga. Supngase ahora que el sistema motor-carga ha alcanzado un equilibrio (por ejemplo, estamos situados en el punto B de la Fig. 4.22). Si en esta situacin aumenta el par resistente, el trmino de la izquierda de (4.95) se har negativo, lo que se traduce en una deceleracin del rotor. El nuevo equilibrio se restablecer para una velocidad menor a la cual se vuelva a cumplir la igualdad de pares motor y resistente. Ahora bien, si el par resistente aumentara hasta alcanzar el punto e, correspondiente al par mximo que puede desarrollar el motor, un posterior aumento de la carga tendera a reducir la velocidad, pero al no estar compensado por un aumento del par electromagntico desarrollado por el motor (zona inestable de la Fig. 4.22), la mquina acabar parndose. El cociente entre el par mximo y nominal o asignado TmJTn define la capacidad de sobrecarga del motor y es un dato suministrado por el fabricantejunto con el cociente del par de arranque y nominal T) Tn En motores normales la capacidad de sobrecarga oscila entre 1,8 y 2,7; en motores sometidos a fuertes sobrecargas instantneas, por ejemplo, en las gras, se suelen exigir valores superiores a 3. La relacin TjTn oscila entre 1,2 y 2.

b)

Rgimen generador

Como se observa en la Figura4.20, correspondea velocidadessuperioresa la de sincronismo,lo que comporta deslizamientosnegativos.En este caso, el sentido de rotacin del flujo respecto al secundario de la mquina se invierte con relacin al rgimen motor, lo que lleva consigo una inversinen el sentido de la f.e.m. del rotor, que provoca a su vez una inversin en la corriente y en el par. En consecuencia,el par desarrollado por la mquina asncrona se convierte en par de frenado respecto al momento de rotacin del motor primario. La mquina asncrona funciona

CAPTULO 4.

MQUINAS ASNCRONAS

297

como generador, recibiendo energa mecnica de un motor externo que gira a una velocidad superior a la de sincronismo, y entregando energa elctrica a la red por el esttor. En efecto, si observamos (4.93), en el rgimen generador, al ser el deslizamiento negativo, se tiene: a) La potencia mecnica interna se hace negativa. La mquina absorbe potencia mecnica por el eje, que es suministrada por el motor primario que la mueve a una velocidad superior a la de sincronismo. b) La potencia en el entrehierro se hace negativa, por lo que el par electromagntico cambia de 'signo respecto al comportamiento como motor. c) Si la potencia en el entrehierro es negativa, la transferencia de energa se hace de rotor a esttor. La mquina entrega energa a la red por el esttor. En la Figura 4.23 se resumen las conclusiones anteriores, sealando con flechas los sentidos de transferencia real de la energa. La mquina convierte energa mecnica en elctrica. Se puede disponer de un generador asncrono conectando un motor normal de jaula de ardilla a una red trifsica y acoplndolo, por ejemplo, a un motor de combustin interna (motor de gasolina o diesel), tal como se muestra en la Figura 4.24a. Inicialmente la mquina arrancar como motor asncrono arrastrando el motor primario de combustin interna, llegando a una velocidad de rgimen como motor cercana a la de sincronismo. A continuacin se arrancar el motor primario, que debe girar en el sentido establecido por el motor asncrono. Tan pronto como el motor primario supere la velocidad de sincronismo, la mquina asncrona se convertir en generador, cediendo potencia activa al sistema elctrico al que est conectada. Ahora bien, si se tiene en cuenta el circuito equivalente aproximado de la mquina asncrona sealado en la Figura 4.18a, al ser el deslizamiento negativo, la impedancia de la rama secundaria tendr la parte real negativa (debido a que se hace negativa la resistencia de carga R;, cuyo valor es muy superior a la suma de R; + R~); como consecuencia de ello, la corriente secundaria reducida 1; absorbida se retrasar ms de 90 de la tensin aplicada, tal como se muestra en la Figura 4.24b, dando lugar a una corriente total absorbida de la red 11m, cuya fase ({Jm es superior a 90, y es por ello por lo que la potencia absorbida de la red: mi Vl11m cos ({Jm es negativa. O de otro modo, si se invierte el sentido de la corriente absorbida 11m para tener en cuenta el convenio generador, se observa que la potencia activa producida o suministrada a la red es: (4.96) Se observa en la Figura 4.24b que la corriente Ilg se adelanta a la tensin de la red, lo que indica que la mquina asncrona suministra a la red una potencia reactiva capacitiva, o de

gB
~
ROTOR ESTTOR Figura 4.23. Reparto de potencias en el funcionamiento como generador.

298

MQUINAS ELCTRICAS

MOTOR DrESEL

a)

b)

Figura 4.24.

Grupo diesel-generador asncrono. Diagrama fasorial del generador.

otro modo, que la mquina asncrona funcionando como generador necesita recibir de la red a la cual se conecta una potencia reactiva inductiva, como as se seala en la Figura 4.24a. Fsicamente esta potencia reactiva que necesita recibir el generador asncrono se requiere para mantener el campo magntico de su esttor, ya que esta mquina no posee un circuito independiente de excitacin, como es el caso de los alternadores. sta es la principal limitacin del generador asncrono, ya que al necesitar una red que le suministre la potencia reactiva que requiere, no puede funcionar (en principio) como un generador aislado como sucede con los alternadores. La nica ventaja, sin embargo, del generador asncrono es su sencillez: no necesita un circuito independiente para su excitacin y no tiene que girar continuamente a una velocidad fija. Siempre que su velocidad sea ligeramente mayor que la de sincronismo, funcionar como un generador con respecto al sistema de potencia al que se encuentre conectado. En la medida en la que el par aplicado a su eje sea mayor, tanto mayor ser la potencia de salida resultante. Generalmente, la salida asignada se alcanza con poco deslizamiento, normalmente inferior al 3 por 100. El hecho de que no necesite regulacin de tensin, porque sta viene impuesta por la red externa, hace que este generador sea una buena alternativa en centrales elicas, donde las velocidades del viento son muy dispares. Es posible hacer que una mquina asncrona trabaje como generador aislado, independientemente de una red externa, siempre que haya condensadores disponibles para suministrar la potencia reactiva que necesita (o de otro modo, condensadoresque absorban la energa reactiva que produce el generador). Para ello se conecta una batera de condensadores a los bornes del motor, que tambin se unen a la carga elctrica receptora externa. Se dice entonces que el generador trabaja en rgimen de autoexcitacin. La frecuencia generada es algo menor de la que correspondea la velocidad de rotacin.La tensin en bornes aumenta con la capacidad, que est limitada por la saturacin del circuito magntico de hierro. Si la capacidad es insuficiente no aparecer tensin en el generador. De ah que la eleccin de la capacidad necesaria sea un problema difcil y ms an si se tiene en cuenta que los condensadores deben tambin suministrar la potencia reactiva que requieren las cargas conectadas a la mquina.

e) Rgimen de freno
De acuerdo con la Figura 4.20, el rgimen de frenado de una mquina asncrona se produce para deslizamientos superiores a la unidad, lo que corresponde a velocidades negativas. En

CAPTULO

4.

MQUINAS ASNCRONAS

299

esta situacin el rotor gira en sentido contrario al campo giratorio, de tal forma que la mquina recibe energa de la red y energa mecnica por el eje. En efecto, si consideramos las ecuaciones (4.93) para s > 1 (normalmente s est comprendido entre 1 y 2), resulta: a) La resistencia de carga R: se hace negativa y por tanto tambin es negativa la potencia mecnica interna. La mquina recibe energa mecnica por el eje. b) La potencia de entrehierro es, segn la 2." ecuacin (4.93), el cociente de dos cantidades negativas; por consiguiente, Pa es positiva y el par electromagntico es positivo. c) Si la potencia de entrehierro es positiva, quiere decir que se transfiere la energa en el sentido esttor-rotor. Por ello la potencia que se absorbe de la red es positiva. En la Figura 4.25 se muestran estas conclusiones. Durante el perodo de frenado la mquina recibe energa mecnica por el eje y tambin energa elctrica de la red; esto origina grandes corrientes rotricas, con las consiguientes prdidas por efecto Joule tanto en el esttor como en el rotor, que es donde se disipan las potencias que recibe la mquina en esta situacin. Este rgimen de frenado se utiliza en la prctica cuando se desea parar rpidamente un motor. La maniobra se realiza invirtiendo dos fases de la alimentacin, de forma que el campo giratorio pase sbitamente a girar en sentido contrario al del rotor. Por ejemplo, si se considera que un motor estaba girando en sentido directo a la velocidad n, el deslizamiento en esta fase sera:
s=--=I-111

ni - n

n
111

(4.97)

Al invertir dos fases, el deslizamiento respecto al nuevo campo giratorio sera:


s'=
11 I

(-n)

= l + - = 2- s

11

(4.98)

ya que la velocidad a la que giraba el motor se hace negativa respecto al nuevo campo rotativo. Como s suele tener un valor reducido (3 a 8 por 100) en el momento de la inversin, el nuevo deslizamiento s' tiene un valor cercano a 2. Este mtodo de frenado en el que se procede a invertir el campo giratorio del motor recibe el nombre de frenado a contracorriente. El rotor, al estar girando en sentido opuesto al campo, va gradualmente disminuyendo su velocidad, y cuando sta llega a cero, el motor debe ser desconectado de la red, ya que en caso de no hacerlo la mquina pasara de nuevo a rgimen motor pero girando ahora en sentido contrario al original.

ROTOR ESTTOR

Figura 4.25.

Reparto de potencias en el funcionamiento como freno.

300

MQUINAS ELCTRICAS

En el frenado a contracorriente se pueden producir corrientes incluso muy superiores a las de arranque, y por ello no debe emplearse este sistema de frenado con demasiada frecuencia porque la elevacin de temperatura puede llegar a fundir las barras del rotor y sobrecalentar el devanado del esttor. Los motores preparados para realizar este frenado son generalmente de rotor devanado, de tal modo que al realizar esta maniobra se introducen resistencias adicionales en el rotor para limitar las corrientes a magnitudes admisibles y deseables. Para que el lector tome conciencia de lo delicado de esta operacin, sirvan estos datos, que se demuestran ms tarde en el epgrafe 4.12.3: 1. El calor disipado en el rotor durante el perodo de puesta en marcha (arranque), desde cero a la velocidad asignada, es igual a la energa cintica final almacenada en todas las partes rotativas. 2. Sin embargo, el calor disipado en el rotor durante el frenado a contracorriente, desde la velocidad asignada hasta cero, es igual a tres veces la energa cintica original de todas las partes rotativas. En la tcnica de los accionamientos elctricos existen tambin otros tipos de frenado. As, se tiene el frenado por recuperacin de energa (que algunos autores denominan frenado regenerativo) y que aparece en la mquina asncrona cuando trabaja como generador, por tanto, a una velocidad superior a la de sincronismo. Se puede producir este frenado en las mquinas de elevacin y transporte cuando se bajan cargas pesadas. Este rgimen de frenado aparece con frecuencia en los motores de c.c. (Captulo 6) utilizados en la traccin elctrica, al bajar un tren por pendientes elevadas (si el tren tiende a embalarse, aparecer un par de frenado que tiende a evitar un posible descarrilamiento). Existe tambin otro tipo de frenado denominado dinmico, que consiste en desconectar el esttor de la red y aplicar una c.c. al mismo por medio de una fuente auxiliar; de esta forma se produce un campo de amplitud constante que es fijo en el espacio y que al reaccionar con el campo giratorio del rotor provoca un frenado de la mquina. Este tipo de frenado se utiliza en los trenes de laminacin de plantas siderrgicas y se emplea para conseguir una parada rpida y exacta de muchos mecanismos, reduciendo el tiempo de paro de los accionamientos principales. EJEMPLO DE APLICACIN 4.5 Un motor asncrono trifsico con rotor en jaula de ardilla de 220/380 V, 50 Hz, 10 polos, tiene los siguientes parmetros del circuito equivalente: R = 0,5 Q; X = 3 Q; R; = 0,8 Q; X; = 3,5 n. Se desprecia la ramaparalelo del circuito equivalente y las prdidas mecnicas. Si la mquina se conecta a una red trifsica de 380 V de lnea, 50 Hz, veamos: 1) Cmo se conectar el esttor de la mquina? 2) Calcular la corriente de arranque del motor. 3) Si el deslizamiento a plena carga es del 4 por 100, calcular la corriente absorbida, potencia mecnica desarrollada, par electromagntico, potencia activa absorbida de la red y rendimiento en estas condiciones. 4) Velocidad (en rgimen motor) a la cual se obtiene el par mximo y valor del par mximo correspondiente. SOLUCIN
1.

Como el motor es de 220/380 V, se deber conectar en estrella a una red de 380 V (vase epgrafe 4.2).

CAPTULO 4.

MQUINAS ASNCRONAS

301

2. En el arranque el deslizamiento es igual al, por tanto la impedancia del circuito equivalente aproximado (Fig. 4.13b) ser: ZT=(RI+R~)+j(XI+X~)
=>

ZT=I,3+j6,5n

y tomando la tensin como referencia, se obtiene una corriente de arranque:


-LO II
a

380

J3

1,3 + j6,5

33,1 L -78,65 A

3. a) Cuando el deslizamiento es del 4 por 100, la impedancia equivalente total ser: ZT = (RI + :~) + jcc, + X~) que al sustituir valores nos da: ZT= ( 0,5 + -0,8) + j6,5 = 20,5 + j6,5 0,04

que corresponde a una corriente de plena carga o asignada:


-LO II
b)
~

380

I~ =

20,5 + j6,5

J3

10,2 L -17,6 A

La potencia mecnica desarrollada ser, segn (4.64):


p
mt

= pu = 3 . 08 '

(_1_ 0,04 - 1) . 10' 22

= 5.9927 ,

e)

La velocidad de sincronismo vale, segn (4.1)


ni

= --

6050
5

= 600 r.p.m.

ya que el nmero de pares de polos es igual a 5. De donde se deduce, de acuerdo con (4.71), una velocidad del rotor:
n

= ni (1 -

s)

= 600 (1 5.992,7 576 2n60

0,04)

= 576 r.p.m.

y teniendo en cuenta (4.72) resulta un par:


T

= --

= 9935 '

N.m

d)

La potencia elctrica absorbida de la red ser, segn (4.57):


P, = 3 .

380 J3 . 10,2' cos 17,6 = 6.399,2 W

302

MQUINAS ELCTRICAS

e)

Y teniendo en cuenta (4.56) se obtiene un rendimiento:


IJ

= -= 93,65 % 6.399,2

5.992,7

4.

a)

El deslizamiento para par mximo viene expresado en (4.77), que al sustituir valores nos da: 0,8 0,1227

que corresponde a una velocidad, segn (4.71):


n = 600 (1 - 0,1227) = 526,37 r.p.m.
b)

y de acuerdo con (4.78) corresponde a un par mximo:

T = -----..:....___----m

3(fiY

2n - . 2[0,5 + JO,52 + 6,52J 60

600

163,7 N.m

EJEMPLO DE APLICACIN 4.6 La mquina del problema anterior, conectada a la misma red, se hace girar por un motor primario auxiliar a una velocidad de 615 r.p.m. Tomando la tensin simple de la red como referencia de fases (por ejemplo, para la fase R), calcular: 1) expresin fasoria! de la corriente absorbida por la mquina y corriente entregada por la misma; 2) potencia mecnica absorbida del motor primario; 3) potencias activas y reactivas suministradas a la red; 4) cul es el rendimiento del generador? SOLUCIN
1.

El deslizamiento de la mquina, segn (4.7l), vale s

600 - 615 600

= -0,025

Por consiguiente, la impedancia equivalente de la mquina ser:

es decir: ZT

= ( 0,5 -

--0,8 ) + j6,5 0,025

= -3,15

+ j6,5

CAPTULO 4.

MQUINAS ASNCRONAS

303

que da lugar a una corriente absorbida:

11m -

_ j3

380 -LO

-31,5 + )6,5

_ 6,82 L -168,34 A
-

que corresponde a una corriente generada igual y de sentido contrario:


118

= 6,82 L 11,66 A

2. La potencia mecnica desarrollada por la mquina es, segn (4.64):

r; = r; = 3 . 0,8 (

1 ) . 6,822 -0,025 - 1

= -4.576,8

es decir, la mquina asncrona recibe 4.576,8 W de potencia mecnica del motor primario. 3. La potencia elctrica activa entregada a la red se obtendr aplicando (4.57) pero utilizando la corriente generada, es decir:
PI = 3

380 J3

6,82 . cos 11,66 = 4.396,1 W

y de modo anlogo la potencia reactiva suministrada a la red ser:

QI = -3

380

J3

6,82 sen] 1,66 = -907,2 VAR

que es de tipo capacitivo. O recprocamente, la red deber entregar una potencia reactiva inductiva al generador asncrono de 907,2 VAR. Ambas potencias se podran haber obtenido simultneamente empleando el concepto de potencia compleja utilizado en los circuitos elctricos, es decir: SI = 3VIII~ = 3

j3
P

380

LO 6,82 L -11,66

= 4.396,1 - j907,2

= 4.396,1 W

; Q = -907,2 VAR

que coincide con los valores anteriores. 4. El rendimiento del generador se entender ahora como cociente entre la potencia elctrica que representa la salida y potencia mecnica que representa la entrada. Teniendo en cuenta los resultados del problema resulta ser:
IJ

= - = -P"

PI

4.396,1

4.576,8

= 96,05 %

EJEMPLO DE APLICACIN 4.7 El motor del problema 4.5 est girando a plena carga con un deslizamiento del 4 por lOO,es decir, a 576 r.p.m. De repente se cambian dos fases de la red. Calcular en ese instante: 1) expresinfasorial de la corriente absorbida por la mquina, tomando como referencia la tensin simple de la red; 2) potencia mecnica absorbida del motorprimario; 3) par desarrollado como freno; 4) potencia activa absorbida de la red.

304

MQUINAS ELCTRICAS

SOLUCIN
1.

En el momento de la conmutacin, el deslizamiento de la mquina es, segn (4.71): 600 - (-576) s=----600 1,96

lo que da lugar a una impedancia compleja equivalente:

ZT

= (0,5

+ 0,8) + j 6,5 1,96

= 0,908 + j 6,5 = 6,56 L 82,05 Q

y por consiguiente la corriente absorbida ser:


-LO

380

I =
I

J3

6,56 L -82,05

33,43 L -82,05 A

que si se compara con el resultado del primer apartado del problema 4.5 se deduce que es un valor similar a la corriente de arranque. 2. La potencia mecnica desarrollada ser, segn (4.64):

r: = r, = 30,8

C,~6 - 1).33,432

= -1.313,6

lo que significa que absorbe una potencia mecnica de 1.313,6 W. 3. El par desarrollado ser, segn (4.72):
T=

-1.313,6 576 2n60

= -21,78

N.m

4.

Es decir, el motor produce un par electromagntico de 21,78 N.m en el sentido del nuevo campo giratorio, esto es, en oposicin al giro que llevaba la mquina trabajando como motor. Como quiera que el par resistente que llevaba la mquina es, de acuerdo con el resultado 3c del problema 4.5, de 99,38 N.m, que es superior al par motor producido en el momento de la inversin (21,78 N.m), la mquina, por consiguiente, acabar parndose. La potencia elctrica absorbida de la red ser, segn (4.57):
PI

= 3 . J3 . 33,43 . cos 82,05 = 3.043,2 W

380

Observar el lector que la mquina trabajando como freno recibe potencia tanto por la red como por el eje, es decir, absorbe un potencia total:
PI + ~,= 3.043,2 + 1.313,6 = 4.356,8 W

Toda la potencia anterior se disipar en calor en los devanados de la mquina. Comprobmoslo:

CAPTULO 4.

MQUTNAS ASNCRONAS

305

que al sustituir valores nos da:


Pp

= 3(0,5 + 0,8) . 33,43 = 4.358,5 W


2

;:::: 4.356,8 W

que coincide con la anteriormente calculada, salvo errores de redondeo.


EJEMPLO DE APLICACIN 4.8

Un motor de induccin trifsico de anillos rorantes tiene un esttor conectado en estrella, de 4 polos. El motor funciona con una alimentacin de 50 Hz y 380 V de tensin compuesta. Los parmetros del circuito equivalente son: R = 0,5 Q; R; = 0,51 Q; Xcc = 2,7 Q. En el supuesto de despreciar la rama paralelo del circuito equivalente y suponiendo despreciables las prdidas mecnicas, calcular: a) Par motor desarrollado para un deslizamiento del 4 por 100. b) Par de arranque. c) Velocidad para par mximo. d) Par mximo. e) Resistencia que debe conectarse por fase, en serie con el rotor para obtener el par mximo en el arranque. NOTA: La relacin de espiras de primario a secundario es igual a 2 y los factores del devanado se consideran iguales a la unidad.

SOLUCIN
a)

De acuerdo con la expresin general del par (4.76) se tiene:

T = -------,---

2n

;[(
= --

RI +

:~y

+ X;c

380 La tensin aplicada por fase es igual a h;:::: 220 V, y la velocidad de sincronismo vale: 3

nI

= --

601. p

6050

1.500 r.p.m.

En consecuencia, el par resultante para un deslizamiento del 4 por 100 resulta ser: 051 3 _'_ 2202 0,04

- ---------,--mx -

1.500 [( 0,51)2 Zt: -0,5 + -+ 2,72 60 0,04

64,45 N.m

b)

El par de arranque se obtiene haciendo s = 1, resultando: 051 3 _'_ 2202 1


.

T
mx

1.500 [( 0,51)2 2n -0,5 + + 2,72 60 1

J '

= 5673 N.m

306

MQUINAS ELCTRICAS

c)

El deslizamiento para par mximo, de acuerdo con 4.77, es:


s =
m

R~ JR2+X2 1
ce

0,51 J052+272 '


,

= 0186
'

que teniendo en cuenta (4.76) corresponde a una velocidad:


n d)

ni

(1 - s) = l.500 (l - 0,186) = l.221 r.p.m.

Para encontrar la magnitud de par mximo, se llevar el valor de sm = 0,186 a la expresin general del par, resultando: 051 3-' -2202 0,186

Tmx

= ----------2---

l.500 [( 0,5 + -0,51 ) + 2,7 2rr -60 0,186


e)

2J

142,4 N.m

Para obtener el par mximo en el arranque es preciso que se cumpla: s=s


m

=1=

R'+R'
2 a2 ce

JR2+X2 1

donde se ha denominado R~2la resistencia adicional conectada por fase al circuito del rotor, y al operar se obtiene:

Para calcular la resistencia adicional real debe tenerse en cuenta que:

Al ser los factores de devanado iguales y tener la mquina igual nmero de fases en primario y secundario, se cumplir:

y en consecuencia: Ra2 = - = 0,56 4 EJEMPLO DE APLICACIN 4.9 En un motor asncrono trifsico el par de arranque es igual al nominal o de plena carga, y se sabe tambin que el par mximo es doble que el nominal. Calcular: 1) deslizamiento para par mximo; 2) deslizamiento a plena carga; 3) cociente corriente de arranque/corriente de plena carga. NOTA: Despreciar la impedancia del esttor y la rama paralelo del circuito equivalente.
R~2
Q

CAPTULO 4.

MQUINAS AsNCRONAS

307

SOLUCIN
1.

Si se desprecia la impedancia del esttor, la ecuacin (4.90) nos relaciona cualquier par producido a un determinado deslizamiento s, respecto al par mximo, es decir:
-::---

y as se tiene para el par nominal: 2

(a)

y para el par de arranque (s

:: l)

resultar:

T 1 2 2sm -=-=---::-t; 2 l s", l + s~ -+s; 1

(b)

ya que Ta es, segn el enunciado, igual a Tn De esta ltima ecuacin se obtiene:


S/~ -

4s", + l = O

que conduce a los valores s;

= 3,73

; s;

= 0,268

La primera solucin, aparte de representar un deslizamiento en la zona de trabajo como freno, dara lugar a una resistencia del rotor excesivamente elevada, lo que conducira a un rendimiento bajo (motor mal proyectado). Tiene sentido la otra solucin, sm = 0,268. 2. Sustituyendo el resultado anterior en la ecuacin a) resulta:
4=--+--

0,268 s

0,268

que da lugar a los siguientes resultados: s= 1 s = 0,072

Ser vlida la 2." solucin, que da un deslizamiento ms reducido. El otro valor correspondera a la otra situacin anulada anteriormente, con una resistencia del rotor anormalmente alta. 3. La ecuacin (4.75) nos da el mdulo de la corriente absorbida por el motor (JI = J~), y teniendo en cuenta que se desprecia la impedancia del esttor se puede escribir:
VI JI = ----;=~==

, (RI)2 2 S

+ X/2 2

308

MQUINAS ELCTRICAS

Ahora bien, como quiera adems que segn (4.77) el deslizamiento para par mximo, si se desprecia la impedancia del esttor, es igual a: sm

= X'

R'
2

=>

R'2

= sm X'

que llevando a la expresin de la corriente, da lugar a:

1,

x;Jt

+ (';)'

que corresponde a unas corrientes de arranque y de plena carga:

1,"

x)t (~)2
+
2

0,072

y conduce a un cociente de intensidades:


[lo

-=
[In

JI + 192,9s~ =--=372 3,854


~

1,035

'

donde se ha tenido en cuenta que segn el apartado l el valor de sm era igual a 0,268. EJf.MPLO DE APLICACIN 4.10
Un motor asncrono trifsico con rotor en jaula de ardilla de 220/380 V, 6 polos, 50 Hz, se conecta correctamente a una red trifsica de 220 V, 50 Hz y est protegido con un rel trmico calibrado a [[ OA. La corriente de lnea a plena carga es de [00 A y se observa que en el arranque la corriente absorbida es 6 veces la de plena carga, desarrollando un par que es 1,8 veces el nominal. La resistencia por fase del esttor R es igual a la reducida del rotor R;. Se suponen despreciables las prdidas mecnicas y la corriente de vaco (quiere decir que no hace falta considerar la rama paralelo del circuito equivalente del motor). Se pide: 1) Conexin del esttor del motor. 2) Velocidad del rotor a plena carga y valores de los parmetros: RJ>R; Y Xcc del circuito equivalente del motor. 3) Potencia, par y rendimiento del motor a plena carga. 4) Estando el motor girando en condiciones nominales o asignadas, se produce de repente una cada de tensin en la red de un 15 por 100. Cul ser la nueva velocidad que adquirir el motor y la corriente absorbida de la red, si el par de carga es constante? Disparar el rel trmico?

SOLUCIN
1.

De acuerdo con la placa de caractersticas, el motor es de 220/380 Y; como la tensin de la red es de 220 Y, el motor se deber conectar en tringulo (vase epgrafe 4.2).

CAPTULO 4.

MQUINAS ASNCRONAS

309

2. De acuerdo con el circuito equivalente aproximado del motor, los mdulos de las corrientes de fase del motor a plena carga y en el arranque cumplirn las ecuaciones: 100 )3= 220 600 220

(a)

Ahora bien, segn (4.74), el valor del par es:


A = -mI nI

= constante

2rc60 De este modo, y de acuerdo con el enunciado, el cociente del par de arranque asignado o nominal ser:

r, 6002 S - = 1 8 = -T" ' 1002


nI

= s

= 0,05

Como quiera que la velocidad de sincronismo es segn (4.1): 6050 = -3= 1.000 r.p.m.

la velocidad del motor a plena carga, de acuerdo con (4.71), ser:


n = 1.000 (1 - 0,05) = 950 r.p.m.

Llevando el valor de s = 0,05 a las ecuaciones (a) y teniendo en cuenta que segn el enunciado se cumple R] = R~, se obtienen dos ecuaciones con dos incgnitas, que dan lugar a los siguientes valores: RI=R~=0,18n;
Xee=0,523n

3. La potencia desarrollada por el motor a plena carga (I~= 1001)3) es, segn (4.64): Pmi = P" = 3R~ ~ - 1 /2 = 3 . 0,18 0,05 - 1

( 1 )'2

(1

)(100)2 )3 = 34.200 W

y el par asignado de acuerdo con (4.70), teniendo en cuenta que n = 950 r.p.m., ser:
T=--=

34.200 950 2rc60

343 8 N.m '

Las prdidas en la mquina son nicamente las del cobre:


Pp = Peul

+ Peu2 = 3 . 2 . 0,18 .

(fiY

= 3.600 W

y por tanto, el rendimiento del motor ser:

'1 =

34.200 = 9048 % 34.200 + 3.600 '

310

MQUINAS ELCTRICAS

4.

La nueva tensin de la red ser: 220 - 15 % 220 = 187 V y teniendo en cuenta la ecuacin general del par (4.76) se podr escribir:

= 343,8= 2.

L:s[(0,,8 0,:8)'
3 0,18' 1872 +

0,523']

que da lugar a la ecuacin de 2. grado: 0,306~ - 0,46s + 0,0324 = O cuyas soluciones son: SI = 0,0735; S2 = 1,43. La solucin vlida es la primera, porque la otra corresponde a la zona de trabajo como freno. De acuerdo con (4.71), la velocidad a la que girar el rotor ser:
n = 1.000 (1 - 0,0735) = 926,5 r.p.m.

Con el deslizamiento SI = 0,0735, calcularemos el mdulo de la corriente por fase que absorbe el motor, teniendo en cuenta el circuito equivalente o bien directamente aplicando (4.75):
/1 = /~ = -----r=====:==== = 69,76 A O 18 0,18 + _'_ + 0,5232 ( 0,0735

187

)2

que corresponde a una corriente de lnea, por estar el motor en tringulo:


/llflea

= 69,76

j3

= 120,83 A

que al ser superior a 110 A, har disparar el rel trmico.

4.8. DIAGRAMA DEL CRCULO 4.8.1. Introduccin


Las caractersticas de funcionamiento y otras propiedades del motor asncrono se pueden obtener construyendo el lugar geomtrico de las corrientes del esttor. Como se demostrar en el epgrafe 4.8.2, la figura resultante es un crculo y para su determinacin completa es necesario realizar los dos ensayos clsicos: vaco o rotor libre y cortocircuito o rotor bloqueado. El diagrama circular permite de una forma grfica analizar el comportamiento de las mquinas asncronas de una manera muy simple, ya que no se necesita realizar ningn clculo analtico. Han sido varios los investigadores que han tratado este tema: Behrend, Heyland, Blondel, Ossanna, Sartori, etc. *

* A. Heyland: Elektrotechnische Zeitschrift, 11 octubre 1894, tomo XV, pg. 563; 3 octubre 1895, tomo XVI, pg. 649. L'clairage lectrique, 7 y 14dejulio de 1900,tomo XXIV, 14-26Y49-59. G. Ossanna: Elektrotechnische Zeitschrift, 23 agosto 1900, tomo XXI, pgs. 712-713.

CAPTULO 4.

MQUINAS ASNCRONAS

311

En el estudio que se seguir aqu se partir del circuito equivalente aproximado de la mquina asncrona, al cual se aplicar la teora de la inversin matemtica. Debe recordarse que dos figuras son inversas respecto de un punto, denominado centro de inversin o polo, cuando sus puntos homlogos estn alineados con ese centro, y el producto de las distancias del polo a cada par de puntos homlogos es una constante denominada potencia de inversin y cuyo valor se acostumbra tomar igual a 1. Se puede demostrar fcilmente que la figura inversa de una recta que no pase por el centro de inversin es una circunferencia que pasa por el polo, y cuyo centro se halla sobre la perpendicular a la recta trazada por el centro de la inversin.

4.8.2. Deduccin del diagrama circular


Considrese el circuito equivalente aproximado de la mquina asncrona indicada en la Figura 4.26. La impedancia de la rama serie es igual a: Zs =

(RI

+ ~~) + j(X

+ X~)

(4.99)

y denominando Y, = l/Z, a la admitancia correspondiente, se cumplir: I~ = VI . Y, (4.100)

Si es Yo la admitancia de la rama de vaco, resultar una corriente primaria I1 igual a: (4.101) Para construir el lugar geomtrico de I1 debe conocerse previamente el de lo e I~.El vector que representa lo tiene una orientacin completamente definida debido a la constancia de VI e Yo; sin embargo, el vector 1; es variable en magnitud y sentido, ya que depende de Ys' que a su vez vara con el deslizamiento de la mquina. Para deducir el diagrama circular se debe comenzar representando el lugar geomtrico de Z, para los diferentes valores de s, tomando seguidamente su figura inversa para determinar Y, y con ello I~;al aplicar entonces la igualdad (4.101) podr obtenerse la forma de variacin de 11, Para representar Zs se han tomado en la Figura 4.27 las resistencias en ordenadas y las reactancias en abscisas, de tal forma que al ir variando s, el afijo del vector Zs va recorriendo la recta MN paralela al eje de ordenadas, debido a que la reactancia XI + X; es una magnitud constante. Se han sealado en esta recta diversos puntos importantes que se obtienen al dar valores especficos al deslizamiento s, y as resulta:

R'e = R'2

(~-l)
S

Figura 4.26.

Circuito equivalente aproximado de la mquina asncrona.

312

MQUINAS ELCTRICAS

ugar geomtrico de Z,

.\
\

I/(X + X2)

'.'.j. ,.....,.
...
P;(s=O) N

GENERADOR ..../(. ..-._._._._.-._._... Lugar geomtrico de Y,

Figura 4.27. Lugares geomtricos de impedancias y admitancias.

PUNTO P;: Se obtiene al hacer s = O, resultando, de acuerdo con (4.99), una componente resistiva infinita; por ello este punto se encuentra en el infinito de la recta MN. b) PUNTO P;c: Se obtiene al hacer s = 1, resultando un valor de la resistencia igual a R +R~. e) PUNTO P'oc,: Se obtiene al hacer s = 00, lo que da lugar a una resistencia total R.
a)

En el punto geomtrico P' (O < s < 1) la resistencia tiene una magnitud R + R'[s. Para el punto particular T', la resistencia total se ha hecho cero, lo que indica, de acuerdo con (4.99), que: R~ R +-=0
s
=>

s=--2
R

R'

(4.102)

Para obtener la figura inversa que corresponde a la admitancia Y, habr que trazar una circunferencia cuyo centro se encuentra sobre una recta perpendicular a MN que pase por el polo O. Para determinar el dimetro de la circunferencia se observa que si T es el homlogo de T', se debe cumplir:

IOTI . IOT'1 =

=>

IOTI - IOn -

1
X + X~

(4.103)

Como la circunferencia debe pasar adems por el polo, se podr ya construir el lugar geomtrico de Ys En la Figura 4.27 se han representado los puntos homlogos de P;, P;c y P'oc" que estarn situados sobre la circunferencia y alineados con el polo. Se observa que al punto situado en el infinito, le corresponde el punto Ps' situado en el centro de la inversin. Se han destacado las zonas que comportan un rgimen de funcionamiento especfico de la mquina como: motor, generador y freno, que corresponde a deslizamientos:O < s < 1;s < O; 1 < s < 00, respectivamente.

P;,

CAPTULO

4.

MQUINAS ASNCRONAS

313

Para determinar el lugar geomtrico de la corriente I~es preciso, de acuerdo con (4.100), multiplicar por VI todas las magnitudes del crculo, de tal forma que si se desea que las direcciones de Y, sean las de las intensidades, es preciso, a efectos de diagrama fasorial, que el eje de las tensiones coincida con el de las resistencias (conductancias), ya que las intensidades en stas van en fase con la tensin aplicada. Para determinar posteriormente 11 deber aplicarse la intensidad (4.101), lo que se consigue, como muestra la Figura 4.28, aadiendo lo a I~;tomando como afijo de la corriente de vaco el origen O == ~, resultando un nuevo origen 01, El dimetro del crculo es ahora igual a VJ(X1 + X~), lo que permite trazar la circunferencia correspondiente. En la prctica la construccin de este diagrama se realiza por medio de los ensayos de vaco y cortocircuito. Recurdese que el ensayo de vaco daba las lecturas: VI'lo' Po' lo que permite conocer lo Y (/Jo y, en consecuencia, se podr determinar la situacin del punto Ps; en la horizontal que pasa por l se encontrar el centro de la circunferencia. En el ensayo de cortocircuito se daban las lecturas: Vlcc' Ilcc = 11", Pcc' lo que permite conocer el cos (/Jcc; ahora bien, para situar el punto Pec en el diagrama es preciso conocer la corriente de cortocircuito que se obtendra en la mquina a la tensin asignada (obsrvese que la construccin del diagrama se realiza tomando como parmetro la tensin asignada), y en consecuencia la corriente de cortocircuito real a tener en cuenta ser: (4.104) que corresponde al vector 01 Pee (no dibujado en la Fig. 4.28) y cuyo ngulo con VI es el (/Jec' Conociendo los puntos P, y Pec Y la recta donde debe situarse el centro de la circunferencia, puede ya construirse sta fcilmente. En la Figura 4.28 se indica el mtodo, que consiste en trazar la mediatriz del vector I~cc hasta cortar en Wa la horizontal que pasa por P,; el punto W determina el centro del crculo.

M ~:c (s= 1)
'. ~.I\

"',''&.
,

',,''O

, ,

P",(s = oo)

...
.i

..

i ; ; ;

.\,~
, ..... ,:l'"

,'

'

,.,.,.,..

..... _.

GENERADOR
............ _ -

,.,.,.,., .

.-.,.,.

Figura 4.28.

Diagrama del crculo de la mquina asncrona.

314

MQUINAS ELCTRICAS

Se observa en el diagrama de la Figura 4.28 que para un punto genrico de funcionamiento, tal como el P, la ordenada correspondiente a PD = 11 COS (/JI representa la componente activa de la corriente absorbida por la mquina; si se multiplican las ordenadas por mi Vi' siendo VI la tensin simple de la red, se obtiene la potencia activa absorbida de la red: (4.105) En consecuencia, la distancia entre los puntos del crculo y el eje horizontal que pasa por el origen 01 dan a una cierta escala la potencia absorbida por la mquina. Los puntos situados por encima del eje horizontal corresponden a potencias positivas, es decir, absorbidas por la mquina (motor y freno), mientras que los puntos situados en la parte inferior corresponden a potencias cedidas a la red (generador):
AB BCR~ PccB' RI = B'e'

(4.106)

lo que indica que el punto B' divide al segmento Pece' en partes proporcionales a R, y R~, o que el punto B divide al segmento AC en idntica forma. En el caso del punto Pc," se tiene una corriente de cortocircuito 1~cc; como en esta situacin la mquina no produce energa mecnica, toda la energa absorbida se disipa en forma de calor en los arrollamientos. Las prdidas en el cobre vendrn expresadas como: (4.107)
y de acuerdo con (4.106) se podr poner:

= m, VIIB'C'I (Pcu2)cc = mi R~1~;c= mi VIIP"B'I


P,ltl

(Pcu')cc= mi R/~;c

(4.108) (4.109)

y de forma anloga, para el punto de trabajo P, que corresponde a una corriente 1~,resultar:

= mIR/~2 = mlVIBC P,u2 = mIR~I;2 = mIV,AB

(4.110) (4.111)

Las prdidas en el hierro se obtienen fcilmente, teniendo en cuenta en la Figura 4.28 que: (4.112) Para calcular la potencia mecnica interna desarrollada por el motor habr que deducir de la potencia absorbida, representada por el segmento PD (expresin 4.105), la potencia perdida en el hierro: segmento CD, y las disipadas por efecto Joule, segmentos AB y Be. En consecuencia, la potencia mecnica interna desarrollada por la mquina vendr expresada por el segmento:
PA = PC - CD - (AB

+ BC)

(4.113)

es decir: (4.114) Por 10 tanto, las distancias de los puntos del crculo a la lnea P\~, indican la potencia mecnica interna desarrollada por la mquina. Por ello a la recta P, Pec se la conoce con el nombre de lnea de potencias.

CAPTULO 4.

MQUINAS ASNCRONAS

315

El rendimiento de la mquina, trabajando como motor, si se prescinde de las prdidas mecnicas, viene expresado por el cociente:
IJ =-=

r:
PI

m,V,PA m,V,PD

PA PD

(4.115)

La potencia que llega al rotor P es la suma de la potencia mecnica interna ms las prdidas en el cobre de ese devanado, resultando, de acuerdo con (4.111) Y (4.114):
ll

(4.116) En consecuencia, la distancia vertical entre los puntos de la circunferencia y la recta P, Px expresa la potencia que llega al rotor. Como quiera que el par, de acuerdo con la expresin (4.73), es de la forma: P m,V,_ T=-"-=--PB n, n, 2n- 2n60 60 (4.117)

la magnitud PB indicar a una cierta escala el par electromagntico producido por la mquina. Como el denominador de (4.117) es la velocidad de sincronismo, indica que Pa es una medida del par, la que da origen a la unidad vatio sncrono. Como las distancias a la lnea P, P oc determinan el valor del par, se conoce a esta recta con el nombre de lnea de pares. Por ejemplo,el par de arranque (s =1)vendr expresado por la distancia PecB " medido a la escala adecuada. El diagrama circular permite determinar tambin cmodamente el deslizamiento de la mquina; debe recordarse, de acuerdo con (4.68), que se cumple: s

= -r.: = m,V,AB
r.
---'.-'-c=

m,V,PB

AB PB

(4.118)

que permite conocer el deslizamiento para el punto de trabajo P. Para calcular el valor de la potencia mxima desarrollada en el rbol de la mquina y el par mximo debern obtenerse los puntos de la circunferencia que distan ms de las lneas de potencias y de pares, respectivamente; para ello se trazarn las perpendiculares a las rectas anterioresdesde el punto W, centro del crculo. En la Figura 4.28 se muestra esta construccin geomtrica, que da lugar a los puntos de trabajo L y M.

4.8.3. Eleccin de escalas en el diagrama del crculo


Comose ha demostrado en el apartado anterior, el diagrama del crculo permite determinar diferentes magnitudes de la mquina: corrientes, potencias, pares, etc.; es preciso para ello asignar diferentes escalas de medida para poder expresar cualquier segmento, en la unidad que le corresponda. Eligiendo una determinada escala para las intensidades, se podrn obtener las de las dems magnitudes de acuerdo con la siguiente distribucin: a) Intensidades: l amperio =

eA milmetros; es decir:

1 1 mm = - amperios

eA

(4.119)

316

MQUINAS ELCTRICAS

b) Potencias:
1 mm = -~
mi VI

103 eA vatios, es decir: 1kW = -mm m1V1

(4.120)

e) Pares:
1 mm = _1 e -1 vatios asncronos (medidos hasta la recta . P,P~). .- Es decir:
A

mV

1 mm=-----N.m

mlVI

eA

(4.121)

ni 2n-

60

que expresado en m . kg (1m . kg = 9,8 N.m) resulta: 1 mm = -ml~

e,

ni 2n-9,8

= --

mlVI

eA

--= m . kg 1,02nl

(4.122)

60

y por tanto: (4.123)


A partir de los ensayos de vaco y cortocircuito y de acuerdo con estas escalas se podr construir el diagrama circular, del cual se deducirn las diferentes propiedades caractersticas de las mquinas asncronas.

4.9. ARRANQUE
Se denomina arranque el proceso de puesta en marcha de una mquina elctrica. En un motor asncrono, para que esta operacin pueda llevarse a cabo es preciso, como se ha demostrado en el epgrafe 4.7, que el par de arranque sea superior al par resistente de la carga; de esta forma se obtiene un momento de aceleracin que obliga a girar el rotor a una velocidad cada vez ms elevada, obteniendo el rgimen permanente cuando se igualan pares motor y resistente. El proceso de arranque va acompaado de un consumo elevado de corriente, lo que est justificado desde el punto de vista de circuito equivalente, ya que la resistencia de carga es nula en el instante inicial, debido a que el deslizamiento es la unidad, por lo que el motor ofrece una baja impedancia, estando prcticamente en cortocircuito. Las normas de los diferentes pases establecen las mximas corrientes de arranque permitidas. En Espaa, el Reglamento Electrotcnico para Baja Tensin, RBT (aprobado por Real Decreto 842/2002, de 2 de agosto), en su Instruccin ITe-BTA7, apartado 6, sobreintensidad de arranque, fija los lmites de la relacin corriente de arranque/corriente de plena carga para motores de c.a., segn se indica en la Tabla 4.2.

R:

CAPTULO 4.

MQUINAS ASNCRONAS

317

Tabla 4.2.

De 0,75 kW a 1,5 kW De 1,5 kW a 5,0 kW De 5,0 kW a 15,0 kW De ms de 15,0 kW

4,5 3,0 2,0

1,5

Para reducir las corrientes en el momento de la puesta en marcha de un motor se emplean mtodos especiales de arranque, segn que la mquina tenga su rotor en forma de jaula de ardilla o con anillos.

4.9.1. Arranque de los motores en jaula de ardilla


a)

Arranque directo

Se emplea nicamente en los motores de pequea potencia. El esquema de conexiones se indica en la Figura 4.29a, donde se ha supuesto que el esttor est conectado en estrella. En la Figura 4.29b se indica la disposicin de la caja de bornes del motor. Este mtodo se aplica a mquinas de una potencia inferior a 5 kW, cuando se trata de instalaciones conectadas a la red urbana (de esta forma no se sobrepasan los valores mximos admitidos por el Reglamento). En las grandes fbricas que tienen una gran potencia instalada, recibiendo energa en A.T. y disponiendo de subestacin transformadora, puede llegarse a arranques directos con motores de hasta 100 ev.

i:: ....

---+--..... ---Interruptor general

UI

al Circuito elctrico Figura 4.29.

b l Caja de bornes

Esquema elctrico del arranque directo.

318

MQUINAS ELCTRICAS

b) Arranque por autotransformador


Consiste en intercalar un autotransformador entre la red y el motor, de tal forma que la tensin aplicada en el arranque sea slo una fraccin de la asignada. El proceso puede realizarse en dos o tres escalones y con tensiones no inferiores al 40, 60 y 75 por 100 de la tensin de la lnea. En la Figura 4.30 se muestra un arranque por autotransformador, con dos etapas de tensin. En la posicin l del conmutador se alimenta el autotransformador con la tensin de la red, aplicando al motor solamente una fraccin de esta tensin de la red (etapa de arranque). Cuando la mquina ha aumentado su velocidad hasta un valor adecuado, cercano al asignado, el conmutador se pasa a la posicin 2, lo que eleva la tensin que llega al motor y ste sigue aumentando de velocidad. Finalmente se pasa el conmutador a la posicin 3, de tal forma que la tensin de la red queda aplicada directamente al esttor del motor. Si se tiene en cuenta, de acuerdo con (4.76), que el par vara con el cuadrado de la tensin aplicada en el momento de la puesta en marcha del motor, la relacin entre el par de arranque con autotr'ansformador Ta,aur Yel que se obtiene en conexin directa Ta, ser: (4.124) donde x indica la fraccin de tensin, respecto a la asignada, que se aplica con autotransformador. Por ello, si se aplica una tensin del 70 por 100 de la asignada el par de arranque con autotransformador es el 49 por 100 del par con arranque directo. De aqu se desprende que este mtodo de puesta en marcha solamente es posible en los casos en que el par resistente ofrecido por la carga no sea muy elevado. Para observar la reduccin de la corriente de arranque con este procedimiento, en la Figura 4.31a se muestra el circuito equivalente del motor en esas condiciones. La corriente de arranque con la tensin asignada aplicada V'red tendr una magnitud: (4.125)

i:o:------~~------------------------Interruptor general

trifsico

Figura 4.30. Esquema elctrico del arranque con autotransformador.

CAPTULO

4.

MQUINAS ASNCRONAS

319

R
+
llcc(por fase)

+
VI red

la motor

...........

VI motor (por fase)

+
V motor= XVI red

a)

b)

Figura 4.31. Circuito equivalenteen el arranque.

y la corriente de arranque en el motor, con una tensin


amOlor

V motor = xVlred,

ser: (4.126)

= -Zee

xVred

= xl

ee

que corresponde a una corriente en la red, primario del autotransformador (vase Fig. 4.31b): (4.127) Por ejemplo, si la tensin aplicada es el 70 por 100 de la asignada la corriente en la red es del 49 por 100 de la que se obtendra con arranque directo.

e) Conmutacin estrella tringulo


Este mtodo solamente se puede utilizar en aquellos motores que estn preparados para funcionaren tringulo con la tensin de la red. La mquina se conecta en estrella en el momento del arranque y se pasa despus a tringulo cuando est en funcionamiento. La operacin se realiza en la actualidad con automatismo de contactores, con un circuito de fuerza y otro de mando o control; se requieren tres contactores: uno, denominadoprincipal, para la alimentacin de los principios de bobina de los devanados del motor; otro contactor se encarga de realizar la conexin del devanado en estrella, y el tercero ejecuta la conexin tringulo; adems se necesita un rel de tiempo para ajustar el momento en que se pasa de la conexin estrella a la conexin tringulo. Para facilitar la comprensin de este sistema de arranque estrella-tringulo, en la Figura 4.32 se muestra el esquema de un circuito que utiliza un conmutador manual especial (hoy da est en desuso, pero tiene una gran ventaja didctica por la simplicidadde su esquema). La posicin l del conmutador de la Figura 4.32 se emplea para el arranque y conecta los devanadosen estrella; el esttor recibe la alimentacin por Vi' VI Y W y el conmutadorpuentea los terminales V2, V2 y Wz (Compararel esquema resultante con el de la Fig. 4.6b). Una vez que la mquina alcanza una velocidadestable el conmutador se pasa a la posicin 2, puenteando los terminales VI con Wz, VI con Vz y WI con V2' y la mquina queda en tringulo (Comparar el esquema resultante con el de la Fig. 4.6a). Se observa que con este procedimiento de arranque se aplica al motor, en el momento de la puesta en marcha, una tensin por fase de una magnitud 11J) de la asignada. Desde el punto de vista analtico, equivale este mtodo a un caso particular del arranque por autotransformador cuando x = l/J), y las relaciones (4.124) y (4.127) se convierten en:
T.= ur. (

v:; 3

)Z T =-T 1
a

(4.128)

320

MQUINAS ELCTRICAS

i::---------------Interruptor general
U

ESTATOR

r1> (tringulo)
W1

MARCHA CONMUTADOR Estrella-tringulo

MOTOR

Figura 4.32. Esquemaelctricodel arranqueestrella-tringulo con conmutadormanual. (4.129) Es decir, la corriente de arranque en estrella es la tercera parte de la corriente de arranque que absorbera el motor si se conectara en tringulo. Se observa que el par de arranque se reduce tambin a la tercera parte. En los motores industriales la relacin entre el par de arranque y nominal, TJT vara entre 1,2 y 2; en consecuencia, el par de arranque resultante oscila entre 0,4 y 0,67 del par nominal y por ello este procedimiento solamente se aplica en aquellos casos en los que el par resistente de la carga en el momento de la puesta en marcha no excede, como media, del 50 por 100 del par asignado o nominal, como sucede en determinadas aplicaciones: bombas centrfugas, ventiladores, etc.
II,

4.9.2. Arranque de los motores de rotor bobinado


En los motores de rotor devanado o con anillos se puede reducir la corriente de arranque introduciendo una resistencia adicional en cada una de las fases del rotor. La operacin se realiza con la ayuda de un restato trifsico, como se indica en la Figura 4.33, donde se ha supuesto que los devanados de la mquina estn conectados en estrella. En el arranque se introduce toda la resistencia adicional (posicin 1), de esta forma aumenta la impedancia de la mquina y se reduce la corriente inicial; conforme el motor inicia su marcha, se va eliminando resistencia del restato pasando el mando mvil a las posiciones 2, 3 y 4, que conforman una serie de contactos o plots, en la ltima posicin queda cortocircuitado el rotor y finaliza la operacin de arranque. En esta situacin, para reducir las prdidas mecnicas del motor y tambin el desgaste de anillos y escobillas, estas mquinas llevan a menudo dispositivos para levantar las escobillas y poner en cortocircuito los anillos. Hoy da la operacin de arranque se realiza automticamente por mediacin de contactores y rels de tiempo que van eliminando secuencialmente las resistencia adicionales. Como se ha indicado en el epgrafe 4.7, la introduccin de resistencia en el rotor modifica las curvas T = fes), desplazando el par mximo a valores de mayor deslizamiento, pero sin modificar el par mximo, como se muestra en la Figura 4.34. Se puede incluso lograr,

CAPTULO 4.

MQUINAS ASNCRONAS

321

u
el J.L1

<t:

::

Reostato de arranque

Interruptor general

Figura 4.33.

Motor asncrono de rotor devanado y restato de arranque correspondiente.

para un determinado valor de la resistencia adicional, que se obtenga el par mximo en el arranque, y de acuerdo con (4.79), es preciso para ello que se cumpla: (4.130) de donde se obtiene: (4.131) En la Figura 4.34 se ha representado esta situacin por una resistencia total reducida del rotor de valor R~.En el supuesto de que las cuatro curvas mostradas en la Figura 4.34 corres-

Figura 4.34.

Curvas par-velocidad al variar las resistencias del rotor.

322

MQUINAS ELCTRICAS

pondan a las posiciones de los cuatro plots de la Figura 4.33, se puede analizar de una forma combinada el proceso de arranque de la mquina. Supngase para ello que el par resistente es constante e igual a T, (curva a) de la Figura 4.34. En el instante de la puesta en marcha el par de la carga, definido por la ordenada SR, es inferior al par de arranque de la mquina, definido por TR en la situacin de plot 1; la diferencia entre estos pares provoca el despegue del rotor y la mquina se va acelerando siguiendo la curva del par que corresponde a la resistencia R~, segn el camino sealado con trazo ms fuerte en la Figura 4.34; conforme se va conmutando el restato de arranque, el par del motor pasa de una curva a la siguiente, y se han indicado por medio de los puntos A, B Y e los momentos en que se producen estos cambios, hasta llegar al rgimen estable de funcionamiento en el punto D. Las conmutaciones se han producido en los instantes en que el par pasaba por un valor Tmn, pero podra haberse elegido un criterio de corriente mnima, o simplemente un sistema de relojera que conmutara cada resistencia en un tiempo predeterminado.

EJEMPLO DE APLICACIN 4.11 Un motor asncrono trifsico tiene los siguientes parmetros: R = R; = 0,5 Q; Xee = 5 Q. Si la capacidad de sobrecarga Tmx/Tn es igual a 2,2, calcular la relacin entre el par de arranque y el par nominal en los siguientes casos: a) arranque directo, b) arranque por autotransformador con una tensin inicial del 75 por 100 de la asignada, c) arranque estrella-tringulo.
SOLUCIN De acuerdo con (4.78) el par mximo viene expresado por:

donde se ha denominado K a: m/2n__J_.Al sustituir valores resulta: 60

t. = 2" V]

2 [

1 ] KV~ 0,5 + JO,52 + 52 = 11,05

El par de plena carga o nominal es igual a:

T=n

r.:
2,2

KV2] 24,31

puesto que la capacidad de sobrecarga es igual a 2,2.


a)

El par de arranque se puede obtener haciendo s = 1, en la expresin general del par (4.76), resultando:

T=
a

m]R2'V]2
n 2n __J_ [(R + R')2 + X2]
60 ]
2 ce

=K
(R

R'V2
]

+ R')2 + X2
2

2]

ee

CAPTULO 4.

MQUlNAS ASNCRONAS

323

que al aplicar valores da:

En consecuencia, el cociente T)Tn resulta ser:

T
_!1_

t;

24,31 = -KV~ . -= O 468


52

KV~

b)

El par de arranque con autotransformador y toma de 75 por 100 es igual a 0,752 veces el par de arranque con conexin directa (vase expresin 4.124), y el cociente anterior vale: ~TII = 075 2 _!1_ = 0752 ' Tn'

0468 = O , 263 ,

c)

En el arranque estrella-tringulo, consecuencia: -

de acuerdo con (4.128), se cumple: Ta. = T)3, y en

TaA T"

= - - = - 0,468 = 0,156

r,

3Tn

EJEMPLO DE APLICACIN 4.12 Un motor de induccin trifsico de 4 polos y rotor devanado tiene los arrollamientos conectados en estrella. Cuando se conecta a su tensin asignada de 380 V, 50 Hz, desarrolla el par de plena carga a 1.425 r.p.m. Los parmetros del circuito equivalente son: R, = R; = 1 Q; Xcc = 6 Q; mv = mi = 2. Calcular: a) Corriente y par de plena carga. b) Corriente y par de arranque con los anillos cortocircuitados. c) Par mximo y velocidad correspondiente. d) Resistencia que debe introducirse en los anillos del rotor por fase, para que la mquina desarrolle el par de plena carga en el arranque con la menor corriente posible en el esttor e intensidad correspondiente.
SOLUCIN a) La velocidad de sincronismo es:
ni

= --

60; p

6050 = --

1.500 r.p.m.

En consecuencia, el deslizamiento
s = ni ni

a plena carga es:

1.500 - 1.425 1.500

= 5%

El circuito equivalente a plena carga es el indicado en la Figura 4.35, resultando una intensidad absorbida en el primario de valor:

11 =

220

J(2 + 19)2 + 6

= 10,07 A

324

MQUINAS ELCTRICAS

20

60

+
R~= 380

Ji ~

. 220 voltios

1(_1__ 1) = 19O 0,05

Figura 4.35.

La potencia mecnica interna vale: Pmi = 3RJ = 3 . 19 . 10,072 = 5.783,65 W y el par de plena carga ser: T =--= n n 2n60
b)

r;

5.783,65

1.425 2n-60

=3876N.m '

En el arranque R; = O, y la intensidad correspondiente, teniendo en cuenta el circuito de la Figura 4.35, ser: 34,79 A Para calcular el par de arranque se utilizar la expresin general (4.76), en la que se har s = 1, resultando:
T = -----=----'--n 2n _I_ [(R + R')2 + X2]

3R~ v
I 2

60

ce

3 1 .2202 ------= 1.500 2n-[22 + 62] 60

23,11 N.m

e) El deslizamiento para par mximo, de acuerdo con (4.77), resulta ser: 0,164 que corresponde a una velocidad:
n

= ni

(1 - s) = 1.500 (1 - 0,164)

1.253,4 r.p.m.

d)

Igualando el par de arranque, con una resistencia adicional conectada, al par de plena carga se obtiene:
3(R~ + R~2)v 38,76 = -----=----=::.........:'------

2n 6~ [(R + R~ + R~J2 + X~]

CAPTULO 4.

MQUINAS ASNCRONAS

325

que puede ponerse: (2 + Ra2) + 6


,2 2

3 . 2002 1.500 2n--'3876 60 '

, , = (1 + Ra2) = 23,85 (1 + Ra2)

cuya ecuacin de 2. grado conduce a los valores: R~2 = 19 que en valores sin reducir son: Ra2 = 4,75

n ;
n ;

R~2 = 0,85

n
n

Ra2 = 0,21

La primera de estas dos soluciones es la que producir menor corriente de arranque debido a su mayor magnitud, siendo la intensidad correspondiente igual a: 1 =
a

220 J(2

19)2 + 62

= 10 07 A
'

que coincide con la corriente de plena carga, lo que era de esperar, ya que la resistencia adicional R~2 coincide con la resistencia de carga R;, lo que est de acuerdo con el significado fsico de la misma.

4.10. MOTORES DE DOBLE JAULA DE ARDILLA


El motor de doble jaula de ardilla fue inventado por Dolivo Dobrowolsky el ao 1893, siendo desarrollado ms tarde por Boucherot. Esta configuracin representa una de las variedades del motor asncrono en cortocircuito que utiliza el fenmeno de desplazamiento de la corriente en el devanado del rotor para mejorar las propiedades de arranque del motor. El esttor de estos motores es trifsico convencional, mientras que el rotor est constituido por dos jaulas que generalmente disponen del mismo nmero de barras situadas en las mismas ranuras (Fig. 4.36a). La jaula externa, situada ms cerca del entrehierro, tiene menor seccin que la otra y se construye con un material de alta resistividad (latn). La jaula interior se hace con cobre electroltico. En el caso en que se construyan ambas jaulas con aluminio, se aumentan ms las diferencias entre las secciones de las mismas. Ambas jaulas estn separadas entre s en cada ranura por medio de una rendija delgada. La misin de esta rendija es aumentar los flujos de dispersin de la jaula interior, y de este modo se consigue una jaula

a)

b)

Figura 4.36. Rotor de doble jaula y de ranura profunda.

326

MQUINAS ELCTRICAS

externa con alta resistencia y baja reactancia, mientras que la interna presenta baja resistencia y gran reactancia. Los anillos de cortocircuito pueden estar separados y unidos despus elctricamente, o tener uno nico para ambas jaulas. De este modo la corriente rotrica se distribuir entre cada jaula dependiendo de las impedancias de las mismas. La teora que rige el funcionamiento de este tipo de rotores es la siguiente: en el arranque la frecuencia del rotor f2 coincide con la del esttor, de tal modo que las reactancias son muy superiores a las resistencias; como consecuencia de ello la corriente fluye en su mayor parte por la jaula que presenta menor reactancia (la exterior). A velocidades normales las frecuencias rotricas, como ya sabemos, son despreciables, por lo que predominan las resistencias frente a las reactancias, y de este modo la mayor parte de la corriente circula por la jaula interior, que tiene menos resistencia. Con todo ello se consigue que en el arranque la resistencia sea alta, lo que conduce a un alto par de arranque y una baja corriente. A velocidad asignada la resistencia es baja, con lo que se consigue un buen rendimiento. Se han desarrollado diversas teoras para obtener el circuito equivalente de estas mquinas. (El lector interesado en este tema puede consultar el texto de P. L. Alger citado en la bibliografa). Si se desprecian los efectos de inductancia mutua entre ambas jaulas y se considera lineal el cambio de las reactancias de las jaulas con las frecuencias (que significa despreciar el efecto pelicular), se puede considerar aceptable el circuito que se muestra en la Figura 4.37, que es copia en definitiva del circuito equivalente convencional (Fig. 4.13a) pero en el que se han considerado las dos jaulas en paralelo en el circuito secundario. El clculo de potencias y pares se realiza entonces teniendo en cuenta ambos circuitos rotricos. Otra variedad constructiva para conseguir motores con alto par de arranque es dotar al rotor de una simple jaula con ranuras profundas (vase Fig. 4.36b) en las que se introducen conductores rectangulares o trapezoidales, con mayor profundidad que anchura. El cambio de la resistencia con la frecuencia se debe a las corrientes parsitas o de Foucault que se inducen en las barras, que son muy altas en el arranque merced a que entonces la frecuencia en el rotor es alta y prcticamente despreciable a velocidades ordinarias. El comportamiento de este tipo de rotores es de este modo, similar al de doble jaula. EJEMPLO DE APLICACIN 4.13 Se dispone de un motor de doble jaula que tiene una resistencia del esttor despreciable, as como la ramaparalelo del circuito equivalente. Las impedancias a rotorparado de lasjaulas interior y exterior son, respectivamente: Z2 = 0,05 + jO,4 O/fase; Z3 = 0,5 + jO,] O/fase. Calcular la relacin de pares debidos a las dos jaulas: 1) en el arranque; 2) cuando la mquina gira con un deslizamiento del 5 por JOO.
Jaula interior Jaula exterior

t: 3

Figura 4.37.

Circuito elctrico equivalente de un motor asncrono de doble jaula.

CAPTULO 4.

MQUINAS ASNCRONAS

327

SOLUCIN 1. De acuerdo con (4.76), teniendo en cuenta que se desprecia la impedancia del esttor, resulta:

donde en la constante A se han incluido todos lo parmetros que se conservan constantes y que incluyen un factor m' en el denominador (relacin de transformacin de impedancias) para poder resolver el problema con parmetros directamente en el secundario. Teniendo en cuenta la ecuacin anterior resultar un cociente entre pares de arranque:
a

= -T = o.s: + 0,12 .
a2

Ta3

0,5

0,052 + 0,42 0,05

= 6,25

2. Procediendo de un modo anlogo, para un deslizamiento del 5 por 100 resulta: 0,5 0,05[

(~'~~y +
'

0,42]
= 0,116

0,05

Si analizamos los resultados obtenidos se deduce que la jaula exterior contribuye 6,25 veces ms que lajaula interior en el momento del arranque, mientras que con un deslizamiento del 5 por 100 slo contribuye 0,116 veces, o de otro modo, a velocidades de rgimen, la jaula interior da un par 1/0,116 = 8,62 veces el de la jaula exterior.

4.11. REGULACIN DE VELOCIDAD


Como se ha indicado anteriormente, el motor de induccin tiene una caracterstica par-velocidad, que en la zona de trabajo estable es muy rgida, lo que significa que funciona a una velocidad asncrona prcticamente constante. Si se une a esta prioridad la sencillez de su construccin, esta mquina es idnea en aquellas condiciones de servicio donde se requiere una velocidad constante. Cuando es necesario disponer de una regulacin de velocidad, se prefieren los motores de c.c. (vase Cap. 6), y de ah sus aplicaciones en traccin elctrica, trenes de laminacin, etc. Debido al mayor precio de los motores de c.c. y a sus gastos de mantenimiento, se ha intentado desarrollar diversos mtodos de regulacin de los motores de induccin, que hoy da gracias a la electrnica de potencia se van incorporando cada vez ms a este mercado. Se van a indicar en este epgrafe los principios generales que pueden aplicarse para controlar la velocidad de los motores asncronos. De acuerdo con la expresin (4.4), que define el deslizamiento, la velocidad del rotor viene dada por la expresin:
n

6011 = ni (l - s) = -p

(l - s)

(4.132)

328

MQUINAS ELCTRICAS

que indica que la velocidad depende del deslizamiento, de la frecuencia y del nmero de pares de polos.

4.11.1. Regulacin por variacin del nmero de polos


Variando el nmero de polos del esttor de la mquina, cambia la velocidad del campo giratorio y en consecuencia vara la velocidad de rotacin del motor. El procedimiento utiliza diversos devanados en el esttor dependiendo de las velocidades que se quieran obtener; generalmente, por limitacin de espacio en las ranuras suelen emplearse dos combinaciones diferentes, dando lugar a dos r.p.m. asncronas cercanas a 3.000, 1.500, 1.000, 750, 500, etc., r.p.m. (a 50 Hz). Para que el funcionamiento sea posible es preciso que el rotor sea de jaula de ardilla, ya que este tipo de rotor adapta automticamente (por induccin) su nmero de polos al existente en el esttor. Con frecuencia se emplean dos escalones de velocidad en la relacin 2: 1, Ycon un solo devanado que se conmuta adecuadamente, 10 que se denomina conexin Dahlander. En este caso, cada fase del devanado consta de dos partes iguales, que pueden ponerse en serie o en paralelo, dando lugar a una reduccin del nmero de polos a la mitad del original, aumentando en consecuencia la velocidad del rotor a prcticamente el doble. En la Figura 4.38 se muestra el procedimiento utilizado. En a) se muestra el devanado formando 4 polos, estando las bobinas conectadas en serie. En b) se muestra el devanado formando 2 polos, con las bobinas conectadas en paralelo; se observa en este caso que la corriente en la bobina cd ha sufrido una inversin. En el caso de devanados trifsicos, la conmutacin se realiza de acuerdo con los circuitos mostrados en la Figura 4.39. El cambio de estrella a doble estrella se emplea en los accionamientos en los que el cambio de velocidad se hace con cargas que trabajan a par constante. Para demostrar esta afirmacin deben compararse los dos esquemas en el supuesto de que la mquina trabaje en

N e

s
d

i
J:ffimumnrn
llllIlllillill
a

i
mumnrn
llllIlllillill
b

fJ

lllIIIIIIIill
a

lllIIIIIIIill
b

::!J
+d

---+

---+ ---+
d b) 2 polos

---+
e

a) 4 polos

Figura 4.38. Conexin Dahlander. Formacin de los polos.

CAPTULO 4.

MQUINAS ASNCRONAS

329

Par constante

Estrella R

2p polos

Doble estrella
T T

p polos

Potencia constante Tringulo


2p polos

Doble estrella

P polos

Figura 4.39.

Conexin Dahlander. Cambios a par constante y a potencia constante.

ambos casos con idntico f.d.p. y las mismas corrientes en las bobinas. Para la estrella se cumple:

P=3

)3 1"
j3

cos

qJ

= j3Vln

cos

tp

(4.133)

En la doble estrella resulta: Pi). = 3


V

2/" cos

qJ

= 2P)

(4.134)

Como en este caso la velocidad es doble, debido a que aparecen p pares de polos, el par se mantendr constante. El cambio de tringulo a doble estrella se emplea en aquellas aplicaciones en que se conserva la potencia. En este sistema se cumple: P/1 = y en doble estrella: Pu

3Vl cos qJ
n

(4.135)

= 3 j3

21"cos

qJ

= j3

P/1 = 1,155P/1

(4.136)

es decir, prcticamente se conserva la potencia en la conmutacin. La regulacin de velocidad que se obtiene en la conexin Dahlander no resulta suave, sino escalonada, pero el procedimiento es muy econmico, resultando unas caractersticas mecnicas duras o rgidas. Gracias a sus ventajas este procedimiento se aplica en ciertos tipos de mquinas herramientas con objeto de disminuir sus transmisiones mecnicas y tambin se emplea en ventiladores, extractores de polvo, ascensores, etc.

330

MQUINAS ELCTRICAS

4.11.2. Regulacin por variacin del deslizamiento


La variacin en el deslizamiento puede realizarse controlando la tensin aplicada al motor, pero el procedimiento no es aconsejable porque de acuerdo con (4.70) resulta afectado el par, ya que ste vara con el cuadrado de la tensin. Se puede regular el deslizamiento tambin, variando la resistencia del rotor, como se ha indicado en el epgrafe anterior; el procedimiento, sin embargo, tiene grandes prdidas debido al efecto Joule en el restato de regulacin, que es similar al de arranque pero diseado para trabajar en rgimen continuo. Existen otros sistemas especiales como la conexin en cascada, Kramer, Scherbius, etc., que se basan en la regulacin de la potencia de deslizamiento sPa de la mquina. Hoy en da se han sustituido por sistemas electrnicos ms o menos sofisticados, como se explica en el Captulo 7.

4.11.3. Regulacin por variacin en la frecuencia


La variacin de la frecuencia de alimentacin puede realizarse por medio de convertidores de frecuencia rotativos (p. ej., un alternador movido por un mecanismo regulable, cuya tensin generada se aplica al esttor del motor de induccin). Sin embargo, hoy da la conversin se realiza estticamente por medio de SRC (rectificadores controlados de silicio o tiristores). Durante la regulacin de la velocidad por medio de la frecuencia se debe mantener el flujo constante para que el par se conserve y la mquina disponga de una capacidad de sobrecarga suficiente; si se desprecian las cadas de tensin en el esttor, la condicin anterior se satisface, de acuerdo con (4.8),si se mantiene constante la relacin V/fl' dando lugar a unas curvas T = fes) bastante rgidas en la zona de trabajo (vase Cap. 7). EJEMPLO DE APLICACIN 4.14 Un motor asncrono trifsico de anillos rozantes, de 8 polos, 50 Hz, tiene los siguientes parmetros: R J = R; = 1 O; X" = 5 O. El devanado del esttor est conectado en tringulo y el rotor en estrella. Al aplicar una tensin de 380 Val esttor, dejando los anillos en circuito abierto, se ha medido en el rotor una tensin de 219,4 V entrefases. El motor desarrolla el par de plena carga con los anillos en cortocircuito para un deslizamiento de 4 por 100. Calcular la velocidad a que girar la mquina si se introduce en el rotor una resistencia de 0,5 O/fase y se aplica el par de plena carga al rbol del motor. NOTA: La tensin aplicada al esttor es la misma en ambos casos. SOLUCIN Del ensayo realizado con el rotor en circuito abierto puede obtenerse la relacin de transformacin de tensiones. Tngase en cuenta que el esttor, al estar conectado en tringulo, mide una tensin por fase igual a 380 V; en cambio en el rotor, al estar en estrella, se medirn: 219,4/.)i En consecuencia, de acuerdo con (4.28), se cumplir:

que coincide con

mi

al tener el rotor el mismo nmero de fases que el esttor.

'APTULO 4.

MQUINAS ASNCRONAS

331

De acuerdo con (4.76), el par de plena carga del motor ser:

s T" = 2n

R')2 +X,: J ~ R +-;


[(

donde se ha llamado K a: J

Al sustituir valores resulta:

1'" = K

0,04[(1

O,~)' + 5'

K -28-,0-4

Cuando se introduce en el rotor una resistencia de 0,5 n/fase, es decir, una resistencia reducida igual a:
m,.miRa1

= 32 0,5 = 4,5 n

La resistencia reducida total del circuito del rotor vale l + 4,5 = 5,5 n, y como quiera que en estas circunstancias la mquina desarrolla el mismo par, si se denomina 52 al nuevo deslizamiento se cumplir:
K T =--=K 5,5/50 s

"28,04 de donde se obtiene:


52

[( 1+5,5)1 +5

2J

K =--

28,04

= 0,22 o

52

= 5,28

El primer valor es el que interesa, ya que corresponde al funcionamiento como motor. La segunda solucin se refiere al trabajo como freno. 6050 La velocidad correspondiente es: n = n (1 - 5) = -(1 - 0,22) = 585 r.p.m.
4

4.12. DINMICA DEL MOTOR ASNCRONO 4.12.1. Generalidades


Cuando se examinan los regmenes transitorios en los accionamientos con motores asncronos, se desprecia en primera aproximacin los procesos electromagnticos, ya que stos se desarrollan ms rpidamente que los procesos mecnicos.

332

MQUINAS ELCTRICAS

Si se denomina T el par electromagntico producido por el motor, T, al par resistente que ofrece la carga, J al momento polar de inercia de las masas giratorias (motor + carga) y w la velocidad angular de rotacin, la dinmica del cuerpo en rotacin nos ofrece la ecuacin de equilibrio de momentos: dw T-T.=JI dt

[N.m]

(4.137)

que nos indica que el par de rotacin desarrollado por el motor se equilibra con el par resistente y por el momento dinmico. El momento de inercia del accionamiento se puede expresar as: GD2 J=mp2 =-4g (4.138)

siendo p y D, respectivamente, el radio y el dimetro de inercia en metros, G el peso de las masas giratorias en newton, g = 9,81 mis 2 la aceleracin de la gravedad y m la masa giratoria en kg. Sustituyendo en (4.137) J y w por GD2 y n, respectivamente, se obtiene la ecuacin de la dinmica de rotacin en una forma ms conocida por el ingeniero: GD2 2n dn T-T=---=-r 4g 60 dt GD2 dn 375 dt (4.139)

La magnitud GD2 recibe el nombre de momento giratorio. Esta magnitud se incluye en los catlogos de los fabricantes de motores elctricos con la denominacin PD2 (que la miden en kg . rrr'), El valor de catlogo de GD2 se deber dividir por cuatro, para obtener J en unidades SI (kg m"), En lo que sigue trabajaremos con la ecuacin (4.137), que es la equivalente a (4.139) pero en unidades internacionales SI. Si se parte de (4.137) se observa que:

1. Cuando T> T" 2.


3.

> O; es decir, tiene lugar la aceleracin del motor. dt dw Cuando T < T , - < O; lo que indica que el motor sufre una deceleracin. , dt

dw

Cuando T

dt estado de equilibrio.

= T"

dw

= O, y en este caso

el motor trabaja en rgimen permanente en un

Habitualmente un motor asncrono pone en accin un mecanismo a travs de un sistema de transmisin (caja de engranajes, convertidor de par). En este caso deber reducirse el par resistente de un eje de rotacin a otro, a base de aplicar el balance energtico del sistema. En este caso, las prdidas de la potencia en las transmisiones intermedias se toman en consideracin, introduciendo en los clculos el rendimiento respectivo (r). Si designamos con wI la velocidad del rbol del motor y con w2 la velocidad del rbol donde se coloca el par resistente, se tendr una igualdad de potencias: (4.140)

CAPTULO 4.

MQUINAS ASNCRONAS

333

donde T2 es el par resistente que ofrece la carga en el mecanismo de produccin y T = T,es el mismo par resistente reducido al rbol del motor. De la ecuacin anterior se deduce: w2 1 T2i T = T2--=(4.141)

w '1

'1

donde i es la relacin de velocidad de los engranajes w/w. Observar el lector una analoga de la ecuacin anterior con el cambio de tensiones de un secundario a un primario en un transformador elctrico (la relacin de transformacin es ahora la relacin de velocidades). De un modo anlogo, la reduccin de momentos de inercia (m.d.i.) a un eje de rotacin est basada en el balance de energa cintica de los elementos que se mueven. Si se denomina, por ejemplo, 1m el m.d.i. del rotor del motor que se mueve a velocidad angular w y 12 el m.d.i, de la carga que se mueve a velocidad w2' se cumplir:

1 2 1 2 1 2 - 1w = - 1w + - 1,w, 22m 2 - de donde se deduce que el m.d.i, reducido al rbol del motor ser:

(4.142)

1= i; + 12(

W2)2 = i; + .21
W 1

(4.143)

ecuacin que es anloga a la reduccin de impedancias de un transformador de secundario a primario:

4.12.2. Tiempo de arranque de un motor asncrono


Si se desea calcular el tiempo que tarda un motor desde que se inicia el arranque hasta que alcanza la velocidad de rgimen, la ecuacin (4.137) nos dar: T-T=1r

dw

dt

(4.144)

Si se denomina w la velocidad de sincronismo del campo giratorio, de una forma similar a (4.71) se podr escribir:
t =

w (1

- s)

(4.145)

que llevando a (4.144) nos da: T- T =-1w r

ds dt

(4.146)

que al integrar nos da: (4.147) Como quiera que el trmino T - T, es no lineal, la integracin anterior debe realizarse por mtodos numricos; lA indica el tiempo de aceleracin, s sera el deslizamiento en el momento del arranque (s = O) y S2 el deslizamiento a plena carga (S2 = sn).

334

MQUINAS ELCTRICAS

Si se considera que el motor arranca en vaco, es decir, sin mover ninguna carga tT, = O),y se prescinde de la impedancia del esttor del motor, el par desarrollado por ste en estas condiciones es, segn (4.90): (4.148)

que llevando a (4.147) nos da: (4.149) es decir: 1wI t = - -A

2Tmx

[S~ --- -

2sm

si + s
ni

SI] InS2

(4.150)

y tomando

SI

= I Y S2 == 0,05, resulta:
fA

==

'mee

[_1_ + 1,5S] 4s",


=-1(1)1

(4.151)

donde 'mee vale: rmee

r.:

(4.152)

que se denomina constante de tiempo electromecnica y representa en s misma el tiempo necesario para que el rotor de inercia 1 se acelere hasta la velocidad de sincronismo wI bajo la accin del par mximo Tmx' Esta constante de tiempo vara entre 0,4 y 0,6 segundos para los motores de c.c. y motores sncronos de 6 polos. El tiempo de aceleracin o de arranque (4.151) es mnimo cuando: (4.153) que nos determina el valor del deslizamiento sm para conseguir un tiempo de arranque mnimo. Como quiera que segn (4.79) el valor de sn es proporcional a la resistencia del rotor, modificando sta (solamente se puede hacer en los motores con rotor devanado) se podrn minimizar los tiempos de arranque de estas mquinas.

4.12.3. Prdidas de energa en rgimen dinmico


Si se desprecian las prdidas en el hierro y rozamientos en el motor asncrono y se considera nicamente la rama serie del circuito equivalente, las prdidas de energa al poner en marcha el motor sern nicamente las prdidas en el cobre y de este modo se podr escribir: (4.154)

CAPTULO 4.

MQUINAS ASNCRONAS

335

donde L1 Wa expresa la energa disipada en el cobre en el proceso de arranque. Al despreciar la rama paralelo del circuito equivalente se cumplir IJ = I~ Ypor tanto (4.154) se transformaen: L1Wa =

LA 3(RJ + R;)I?
dt=--ds JW
J

dt

(4.155)

Si el motor arranca en vaco (T, = O) se tiene, de acuerdo con (4.146): (4.156)

Como quiera adems que segn (4.68) y (4.73) se cumple: (4.157) resulta: (4.158) que llevando a (4.155) nos da: L1 Wa

= LA

3R; I?

(1 + ~Ddt = LA TwJs (1 + ~J dt

(4.159)

y teniendo en cuenta (4.156):

L1Wa = si se considera
Si

L
(

Jwi (1

~Ds

dt

(4.160)

1 Y SI

= O,resultar:
L1W
a
=_J

Jw2

+___.!.

R;

R)

[Julios]

(4.161)

es decir: Jw~ Jw~ R J L1W =-+-" 2 2 R~ (4.162)

El primer sumando de la ecuacin anterior representa las prdidas en el cobre del rotor, que coinciden con la energa cintica final almacenada en todas las partes rotativas. El segundo sumando representa las prdidas en el cobre del esttor al poner en marcha el motor. De un modo anlogo se puede calcular la prdida de energa durante el frenado a contracorriente. Si se parte de la ecuacin (4.160) Yse toman como lmites de integracin Si = 2 y SI = 1, resulta:
Jw2 L1W =3_ I 2
J

1+___.!. R'2

RJ

(4.163)

que nos demuestra que el calor disipado en el rotor durante un proceso de frenado a contracorriente (desde la velocidad asignada acero) es igual a tres veces la energa cintica original

336

MQUINAS ELCTRICAS

de todas las partes giratorias (supuesto R, ~ O, lo que representa una aproximacin aceptable para motores de tamao medio-grande). Observando las ecuaciones (4.162) y (4.163) se deduce que se pueden reducir las prdidas de energa durante los regmenes transitorios de un motor asncrono disminuyendo el momento de inercia del accionamiento elctrico. Para ello es frecuente en la prctica colocar dos motores (en vez de uno solo) que tengan cada uno la mitad de la potencia total necesaria. Al hacer esta sustitucin se disminuye el dimetro de los rotores de los motores y, pese al aumento del peso global de stos, se reduce el momento de inercia total. Tambin pueden emplearse motores especiales con rotor alargado, que es una tcnica muy utilizada en metalurgia (trenes de laminacin). Los rotores de estos motores se hacen de menor dimetro que los ordinarios, lo que permite reducir su momento de inercia. Un procedimiento moderno que reduce las prdidas en el rgimen de arranque de un motor asncrono es alimentar la mquina con una tensin que se va elevando gradualmente. Con objeto de que no se produzcan reducciones en el par electromagntico desarrollado, la variacin de tensin va acompaada de una variacin de frecuencia proporcional (vase epgrafe 4.1l.3). En los motores de dos velocidades o conexin Dahlander (vase epgrafe 4.11.1) se pueden reducir las prdidas de energa a la velocidad superior realizando un arranque por etapas. Por ejemplo, si es un motor de 1.500/3.000 r.p.m., se hace un arranque que siga el proceso 0-1.500-3.000 r.p.m. ms que el directo 0-3.000 r.p.m. EJEMPLO DE APLICACIN 4.15 Se dispone de un motor asncrono trifsica de 2,5 kW, 50 Hz, con conexin Dahlander para 2 polos y 4 polos. El momento de inercia del rotor es J = 0,25 kg . m2 y la resistencia el esttor R es 1,5 veces la resistencia reducida del rotor R;. Calcular: 1) Prdidas de energa producidas en el proceso de arranque segn se efecte ste en una o dos etapas. 2) Prdidas de energa producidas en el frenado a contracorriente en una o dos etapas. SOLUCIN 1. Las velocidades de sincronismo son:
ni = --

60f,
p

= --

60' 50

= 1.500 r.p.m.

n2

= ---

60 50 1

= 3.000 r.p.m.

que corresponden a velocidades angulares: wl = 157 rad/s; w2 = 314 rad/s. Las prdidas de energa en el arranque directo hasta la velocidad final w2 es, segn (4.161): dW =
a

0,25' 314

(1 + 1,5) = 30,81 . 103 Julios = 8,56 . 10-3 kWh

Si el arranque se realiza en dos etapas, se tendr:


a)

Arranque hasta la velocidad wl: dW


I

0,25' 1572 . 2,5 = 7.703 Julios = 2,14' 10-3 kWh 2

CAPTULO 4.

MQUINAS ASNCRONAS

337

b)

Paso de la velocidad
~Wa2

w2:

0,25(314 - 157)2 2

. 2,5 = 7.703 Julios = 2,14 . 10-3 kWh

que da lugar a un valor total: ~ Wa + ~ Wa2 = 4,28 . 10-3 kWh

que es la mitad de la energa perdida en el arranque realizado en una sola etapa. 2. De un modo anlogo al anterior y aplicando (4.163) se obtiene: Frenado en una sola etapa: Frenado en dos etapas:
~ Wf = 25,68 . 10-3 kWh ~ W = 12,84 . 10-3 kWh

Ambas prdidas son el triple que las producidas en el arranque. Se sigue cumpliendo que las prdidas de energa en el frenado a contracorriente en dos etapas es la mitad que la producida en una sola etapa.

4.13. MOTOR DE INDUCCIN MONOFSICO 4.13.1. Principio de funcionamiento


Bsicamente, un motor de induccin monofsico est formado por un rotor en jaula de ardilla anlogo al de los motores trifsicos y un esttor en el que se dispone un devanado alimentado con c.a. monofsica. Normalmente se construyen con potencias inferiores a l CV y por ello reciben tambin el nombre de motores fraccionarios. El campo de aplicacin est muy restringido y se limita en su mayora a las instalaciones domsticas: lavadoras, ventiladores, tocadiscos, etc. En la Figura 4.40 se muestra el esquema de conexiones bsico. Al introducir una corriente alterna en el devanado del esttor se produce una fuerza magnetomotriz en el entrehierro, distribuida senoidalmente en el espacio y de carcter pulsatorio, cuya expresin, de acuerdo con el epgrafe 2.8 del Captulo 2 (expresin 2.47), es:
,7(a, t) = Fm cos wt cos p

(4.164)

La f.m.m. anterior produce un campo magntico proporcional en el entrehierro, el cual induce a su vez corrientes en el rotor, como si fuera el circuito secundario de un transformador, de tal

RED

'V

ESTTOR

~... G
.

J/~~x

"'Ej~'~agntico del estator

ROTOR

Figura 4.40. Motor asncrono monofsico.

338

MQUINAS ELCTRICAS

forma que los pares de rotacin originados por la accin recproca de las intensidades de las dos mitades del arrollamiento del rotor con el campo inductor del esttor son opuestas entre s, y en consecuencia, el par resultante que acta sobre el rotor en reposo es nulo. La ausencia de par inicial de arranque representa la particularidad caracterstica del motor monofsico y por ello esta mquina no puede arrancar por s misma. De acuerdo con el teorema de Leblanc (epgrafe 2.8.4), la f.m.m. alternativa (4.164) puede expresarse tambin como: ,Y(ex,
t) =

"2 Fm [cos

(wf +

pex) + cos

(wlt -

pex)]

(4.166)

Cada campo giratorio dar lugar a una caracterstica par-deslizamiento anloga a la mostrada en la Figura 4.20 para los motores trifsicos. Si el motor gira en el sentido del campo positivo directo (uno de ellos elegido arbitrariamente) a velocidad n, el deslizamiento de este campo Sd tendr un valor:
_
Sd-S

_ nI - n_
----

(4.167)

nI

mientras que el deslizamiento correspondiente del campo inverso Si ser:


s=
I

nl-(-n) nI

1 +-=2-05
nI

(4.168)

expresin que relaciona los deslizamientos de ambos campos. Las caractersticas T = f(s) para cada uno de los sistemas giratorios y su resultante se muestran en la Figura 4.4l. Debe observarse que el campo directo produce un par positivo, mientras que el campo inverso crea un par negativo. En el arranque, Sd = Si = 1, se tiene un par nulo y en consecuencia, como ya se ha indicado, el motor no puede despegar; sin embargo, y como muestra la Figura 4.41, si por cualquier procedimiento se ayudara a girar el rotor en un sentido, aparecer inmediatamente un par de arranque que si es superior al par resistente pondr en funcionamiento la mquina. Por ejemplo, si el motor llevara conectada una carga cuyo par resistente es del tipo ventilador, como muestra la Figura 4.41, y se moviera el rotor por un medio mecnico exterior de tal forma que se sobrepasara el punto A, en este momento, al ser el par motor superior al par

Par

resistente

Si

=2-

Figura 4.4l.

Curvas par-velocidad de un motor asncrono monofsico.

CAPTULO

4.

MQUINAS ASNCRONAS

339

resistente, la mquina se acelerar hasta obtener el rgimen estable en el punto B. Se observa que la velocidad final que se obtiene es inferior a la que resultara con un motor trifsico con idntica curva (punto C), y de ah que estos motores trabajen con un deslizamiento relativamente elevado. Si en un motor trifsico se desconecta una fase, se tendra un funcionamiento anlogo al descrito aqu, ya que la mquina trabajara como motor monofsico. En estas condiciones no hay par de arranque; la potencia y el par mximo se reducen considerablemente, y si la carga actuara en el eje con el par nominal, se producira un calentamiento excesivo en los devanados.

4.13.2. Circuito equivalente


Podemos obtener el circuito equivalente de un motor asncrono monofsico teniendo en cuenta el ya conocido de un motor trifsico (Fig. 4.13a) y la teora del doble campo giratorio desarrollada en el epgrafe anterior. Por razones didcticas se va a proceder a un desarrollo heurstico de tipo semicuantitativo evitando el empleo de las componentes simtricas trifsicas que en rigor se necesitan. De acuerdo con la teora del doble campo giratorio, se puede considerar que el motor monofsico de la Figura 4.40 es equivalente a dos motores trifsicos iguales, cuyos rotores estn situados en un mismo rbol (Fig. 4.42) Y cuyos esttores estn conectados en serie de tal modo que los campos magnticos creados por ellos giran en el espacio en sentidos contrarios (vase en la Fig. 4.42 la inversin de una fase en el esttor del motor 2 respecto del 1). Si se considera el rotor parado, el circuito equivalente del motor monofsico ser el mostrado en la Figura 4.43a (se han considerado despreciables las prdidas en el hierro). Este

i::~----~~~----

';;(

~H--'

= 0=)
_F,n/~2(
MOTOR 2

MOTOR 1

Figura 4.42. Equivalencia de un motormonofsicocon dos trifsicosque giren en sentidoscontrarios.

340

MQUINAS ELCTRICAS

J)

R)

X) + E)

v)

!
a)

ro Xi
12

- e.,
...........
1)

Zd-'

lo

XI'/2

>
>

V)

x,

+ ~
Z-.!

R/2

E)

~ XI'/2

lu

ROTOR ROTOR PARADO MVIL > R/2~


R'l2

r2

R/2~

R'2/2 2-s

'- ~
X/2 b)

XY2

Figura 4.43. Desarrollo del circuito equivalente del motor asncrono monofsico.

circuito no es ms que el correspondiente al motor trifsico (Fig. 4.13a) para un deslizamiento nulo; E) representa la f.e.m. inducida en el devanado del esttor debida al campo alternativo existente en el entrehierro. De acuerdo con el esquema de la Figura 4.42, el circuito equivalente de la Figura 4.43a se puede poner como se muestra en la Figura 4.43b, que consiste en dos mitades idnticas conectadas en serie, cada una correspondiente a cada campo giratorio. Se han denominado E)d y E)las f.e.m.s. inducidas en el esttor debidas a cada campo; comparando ambas figuras resulta: (4.169) Si el motor est parado (n = O), los deslizamientos de la mquina respecto a cada campo giratorio sern iguales a la unidad, como as lo confirman las ecuaciones (4.167) Y(4.168). En esta situacin los circuitos equivalentes de los motores componentes que definen el motor monofsico sern idnticos. En definitiva, lo que sucede en la Figura 4.43 es que se cumple:
E)d=

E)=2

E)

(4.170)

Cuando el rotor gira a la velocidad n respecto al campo directo, los deslizamientos correspondientes sern, segn (4.167) y (4.168):
Sd

=S

s = 2 - s

(4.171)

y de ah que las resistencias rotricas se modifiquen de un modo diferente para cada campo. En la Figura 4.43b se muestran a la derecha los nuevos valores de estas resistencias. En esta situacin las impedancias Zd y Z son desiguales, de hecho se tiene Zd Z, por lo que Eld E1 y el efecto sobre el motor es que el campo directo predomina sobre el campo inverso, lo que provoca un par electromagntico neto. En definitiva, lo que ocurre cuando gira el motor es que aumenta la amplitud del campo giratorio directo mientras que disminuye la amplitud del campo inverso. Para que el motor monofsico pueda arrancar se necesita que los dos campos giratorios tengan diferentes amplitudes en condiciones estacionarias. Esto requiere, como se explicar en el epgrafe siguiente, que se incorpore un bobinado adicional al motor monofsico y que se

CAPTULO 4.

MQUINAS ASNCRONAS

341

denomina devanado auxiliar, que se sita a 90 elctricos respecto al otro bobinado, denominado devanado principal (se trata, en definitiva, de preparar un pseudomotor bifsico). El devanado auxiliar tiene menos espiras que el devanado principal y se construye con hilo ms delgado. Este devanado auxiliar se desconecta de la red una vez que el motor ha comenzado a girar. En el epgrafe 4.13.3 se explicarn otros procedimientos adicionales para dotar al motor monofsico de una capacidad de autoarranque. Con el circuito equivalente a rotor mvil del motor monofsico mostrado en la Figura 4.43b se puede hacer un anlisis cuantitativo de esta mquina de un modo similar al que se utiliz en el anlisis del motor trifsico. En el caso del motor monofsico se dispone de dos resistencias de carga de campo directo e inverso: (4.172) que dan lugar a las potencias mecnicas internas: (4.173) lo que corresponde a una potencia mecnica total: (4.174) ya un par neto en el eje, segn (4.72) y (4.166): \
T=wl(l-s)=wl

mi

'2"

R~[I~ /2 ] ---;-2-s
COS !.pI VI

(4.175)

La potencia activa absorbida por el motor de la red ser, segn el circuito de la Figura 4.43b:
PI = VJI

(4.176) y la corriente absorbida /1' Por consi-

donde !.pI indica el desfase entre la tensin aplicada guiente, el rendimiento del motor ser:
1]

=PI

r:

(4.177)

El lector puede obtener cualquier otra potencia analizando simplemente el circuito equivalente de la Figura 4.43b.

4.13.3. Arranque de los motores de induccin monofsicos


Hemos visto en la Figura 4.41 que el motor monofsico no tiene par de arranque y que por tanto no puede iniciar la marcha por s mismo. Por otro lado, se advierte en la misma figura que si se acciona el rotor y se le impulsa en cualquier sentido, la mquina desarrolla entonces un par interno que acelera el rotor hasta conseguir la velocidad de rgimen. Un procedimiento simple sera, si el motor trabajara en vaco, moverlo en cualquier sentido con un par motor exterior; esto se puede realizar en la prctica mediante una polea a la que se arrolla una cuerda, y al tirar de sta se conseguir el impulso inicial para que arranque la mquina; el sentido de giro depender del sentido de arrollamiento de la cuerda sobre la polea. En motores

342

MQUINAS ELCTRICAS

pequeos trabajando en vaco es suficiente, para arrancar el motor, mover el eje con los dedos de una mano (mucho cuidado con esta operacin). De cualquier modo, estos procedimientos mecnicos para conseguir el despegue de un motor monofsico no son prcticos y por consiguiente estn desechados. Afortunadamente, se puede conseguir el arranque de un motor monofsico por procedimientos elctricos. La clave de la solucin est en conseguir un campo giratorio nico en el momento del arranque. Para ello, y de acuerdo con el teorema de Ferraris (vase epgrafe 2.8.3), el mtodo ms simple sera preparar un devanado bifsico. Un sistema bifsico consiste, como se muestra en la Figura 4.44, en dos devanados estatricos desfasados 90 elctricos en el espacio que llevan corrientes desfasadas 90 elctricos en el tiempo. Si en el caso de la Figura 4.44 las corrientes son de la forma: i,

= j21

cos co.t

; iz =

J21 cos (co.z -

90)

= j21 sen

wJt

(4.178)

se producirn en los ejes de las bobinas unas f.m.m.s.: (4.179) donde por simplicidad se han considerado ambos devanados con el mismo nmero de espiras. La composicin vectorial de ambas f.m.m.s. que se muestran en la Figura 4.44, nos indica que:

.7; = J.7z + .7/ = Fm


tg (pa) = -:-

.7;

.7;"

= tg wt

=> pa

= wt

=>

da dt

(4.180)

lo que corresponde a una f.m.m. resultante de amplitud constante Fm Y que gira a la velocidad de sincronismo w/p, lo que est de acuerdo con la ecuacin (2.75), que expresaba el teorema de Ferraris con un nmero de fases m = 2. Evidentemente, si se intercambian las alimentaciones de ambos devanados, se obtendr un campo giratorio de sentido contrario al expresado antes. Si las corrientes que alimentan los dos devanados del esttor (el rotor es siempre en jaula de ardilla) tienen amplitudes diferentes o si los bobinados tienen diferente nmero de espiras, se sigue obteniendo una f.m.m. giratoria, pero su amplitud va variando con la posicin, ofreciendo un valor mximo en el eje de un devanado y un valor mnimo en el eje del otro. El lugar geomtrico del afijo del vector (fasor espacial) de f.rn.m. es una elipse con los semiejes correspondientes a F mx Y F mn que resultan en los ejes de los devanados.

n'.'
N F,

N'Cr
//

Figura 4.44. F.m.m.s. eh un motor bifsico.

CAPTULO 4.

MQUINAS ASNCRONAS

343

Si adems las corrientes de los devanados tienen un desfase distinto a 90 elctricos, el lugar geomtrico de f.m.m. sigue siendo una elipse en la que los ejes principales no se corresponden con los ejes de las bobinas. Esta ltima combinacin es la que generalmente se emplea en el arranque de los motores reales, debido a la dificultad en conseguir con una nica red monofsica dos corrientes desfasadas 90 elctricos, como as lo requiere el teorema de Ferraris. Los procedimientos de arranque de los motores asncronos monofsicos que constituyen los diversos tipos existentes en el mercado son: a) motores de fase partida; b) motores con condensador; e) motores con espira de sombra. Veamos una descripcin de estas versiones:

a) Motor de fase partida


En este motor se sitan en el esttor dos devanados desfasados 90 elctricos en el espacio (Fig. 4.45). El primer devanado, denominado principal, cubre los 2/3 de las ranuras y tiene gran reactancia y baja resistencia, mientras que el otro, denominado auxiliar, cubre el resto del esttor y tiene alta resistencia y baja reactancia (se realiza con hilo ms delgado), de tal forma que est en serie con un interruptor centrfugo situado en el eje del motor. El diagrama fasorial de las corrientes que atraviesan los arrollamientos' se muestra en la Figura 4.46a, donde se ha tomado la tensin de la red como referencia de fase. Se observa que el ngulo formado por V e lp es superior al que existe entre Vela, en virtud de la mayor reactancia del devanado principal. En la prctica, el ngulo cp que forman las corrientes es prximo a los 30, y como los arrollamientos estn defasados 90 en el espacio, resulta un campo giratorio de naturaleza elptica debido a que los mdulos de la e lp no son iguales y no forman entre s 90; este campo giratorio produce el despegue del motor, y cuando la velocidad del rotor alcanza un valor del orden de 70 por 100 del nominal, el interruptor centrfugo desconecta el devanado auxiliar, que al estar realizado con un hilo delgado no es capaz de soportar un funcionamiento continuo. De esta forma el motor queda funcionando como monofsico. En la Figura 4.46b se muestra la curva por velocidad de este motor, en la que se indica el instante (correspondiente a la velocidad n) en el que acta el interruptor centrfugo y se desconecta el devanado auxiliar. Obsrvese que la curva par-velocidad cuando actan los dos devanados es similar a la de un motor trifsico y produce un par de arranque, mientras que la curva correspondiente cuando funciona slo el devanado principal tiene un par de arranque

+ v rv
Interruptor centrfugo Devanado principal

o
ROTOR

Devanado auxiliar

Figura 4.45. Motor de fase partida.

344

MQUINAS ELCTRICAS

Devanado principal + auxiliar

v
Par resistente

o
al
Figura 4.46.

~
Diagrama fasorial y curva par-velocidad de un motor de fase partida.

nulo. Este tipo de motores se utiliza en ventiladores, bombas centrfugas, lavadoras, etc. Se construyen generalmente desde potencia 50 W hasta 500 W. b)

Motor con arranque por condensador

En este tipo de motor, el devanado auxiliar lleva en serie un condensador (Fig. 4.47). El empleo de este condensador tiene grandes ventajas, ya que pueden conseguirse desfases en las corrientes de las bobinas que se acercan casi a los 90 En la Figura 4.48a se muestra el diagrama fasorial de las corrientes, en el que se ha tomado la tensin de la red como referencia; se observa que la corriente del devanado principal se retrasa respecto a la tensin debido a su carcter inductivo, mientras que la corriente en el devanado auxiliar se adelanta respecto a la tensin debido a su carcter capacitivo. En la Figura 4.48b se muestra la curva par-velocidad de este motor, que si se compara con la Figura 4.46b se desprende que el motor con condensador ofrece un par de arranque considerablemente mayor que en el caso del montaje con fase partida. Los condensadores em0

Devanado principal

o
ROTOR

r ~ ______ T
I
I

Interruptor centrfugo

earranque

--,.. vvv

r=;

Devanado auxiliar

Figura 4.47.

Motor monofsico de arranque por condensador.

CAPTULO 4.

MQUINAS ASNCRONAS

345

pleados son del tipo electroltico y permanecen conectados nicamente durante el perodo de arranque *. Una vez desconectado el condensador el motor queda funcionando como monofsico empleando el devanado principal. A veces, con objeto de mejorar las caractersticas del par y el f.d.p. de la mquina, se emplean condensadores de papel impregnados con aceite que trabajan de forma continua, como se indica en la Figura 4.47. Tambin se puede emplear una solucin mixta, con un condensador electroltico para el arranque y otro de papel para rgimen continuo. Por ejemplo, un motor de 1/2 CV lleva un condensador electroltico de 250 J.1F para el arranque y un condensador de papel impregnado de aceite de 115 J.1F para el rgimen normal. Este tipo de motores se emplean en compresores, bombas, algunos tipos de mquinas herramienta, equipos de refrigeracin y aire acondicionado, etc. Se construyen hasta potencias que llegan a 6 kW. Un motor trifsico convencional se puede adaptar para trabajar en redes monofsicas empleando condensadores, es decir, funcionando como motor monofsico con condensador. En la Figura 4.49a se muestra el esquema correspondiente; una bobina se conecta directamente a la red, mientras que las otras dos bobinas se unen en serie a la red, pero previamente se conecta en paralelo un condensador con una de estas ltimas; dependiendo con qu bobina est conectado el condensador, el motor girar en un sentido o en el contrario tal como se seala en la Figura 4.49a. El motor trifsico en estas condiciones arranca por 's mismo con las mismas caractersticas que un motor monofsico normal. Se consigue con la disposicin anterior que los flujos de ambos juegos de bobinas estn a 90 separados en el espacio, simulando la bobina WI W2 el devanado principal y la unin serie de las bobinas VI V2 con VI V2 el devanado auxiliar, en el que un condensador se ha colocado en paralelo con una de estas ltimas bobinas. Utilizando un condensador adecuado se puede conseguir que la potencia del motor como monofsico pueda llegar al 80-90 por 100 de su valor asignado como trifsico. Para una red de 220 V se necesitan unos 70 J.1F por kilovatio de potencia til del motor (Norma UNE 48501). El condensador debe preverse para una tensin alrededor de 1,25 veces la de red, debido a los efectos de sobretensiones a que suele estar sometido como consecuencia de los fenmenos de resonancia.
Devanado principal + auxiliar

Par resistente

ni a) b)

111

Figura 4.48, Diagrama fasorial y curva par-velocidad de un motor monofsico con condensador.

La tcnica del condensador electroltico se utiliza para conseguir altas capacidades, pero los condensadores de este tipo tienen una polaridad definida y por ello su uso est limitado a la corriente continua. En el caso que nos ocupa, al trabajar con c.a. deben funcionar durante el breve tiempo del perodo de arranque.

346

MQUINAS ELCTRICAS

~ o----.~-------------U
220V

12

~o----f-~~----

~C)
~1 r- __

_I_ L.

b)

a)

Figura 4.49.

Adaptacin de un motor trifsico para trabajar como monofsico.

Explicacin tcnica: En la Figura 4.49b se muestra el esquema elctrico equivalente de la Figura 4.49a, donde se ha denominado Z L q>a la impedancia de cada una de las bobinas del motor. La rama 1tiene una impedancia ZI = Z L tp, mientras que la impedancia de la rama 2

vale: Z L q>(-jX) Z? = Z L q> + -----=-Z L q>- j X;


ZLq>l[ 'X lc ZLq>-jXc ] =

(a)
De este modo las corrientes de las ramas son:

y cuyo diagrama fasorial se muestra en la Figura 4.4ge. Se observa que las corrientes estn desfasadas entre s e grados. El objetivo a conseguir es encontrar el valor de la reactancia X; (en definitiva, de la capacidad e del condensador) que haga mximo el ngulo e. De la ecuacin (a) se deduce:
q>- X, e = arctg Z sen Z cos q>

arctg

Z sen q>- 2Xc Z cos q>

~
(b)

~ e = f3 e

rx ~

tg

f3 - tg rx e = 1tg = + tg f3 . tg a Z2 -

ZXc cos q> 3ZXc sen q>+ 2Xc2

El ngulo ser mximo si su tangente tambin es mxima; por consiguiente, derivando la ltima ecuacin anterior respecto de Xc e igualando esta derivada a cero se obtiene el valor de Xc que cumple este objetivo, y que resulta ser:
(e)

CAPTULO 4.

MQUINAS AsNCRONAS

347

Si se considera un motor trifsico de 1 kW, Vjase


cp = 45,57), la corriente que consume es: P 1]3Vcoscp 1.000 0,71'3'220'0,7

= 220 V, 1] = 0,71; cos cp = 0,7 (es decir


1 = 3,05 L -45,57

1=

= 3,05 A =

Por consiguiente, la impedancia del motor ser:


Z - - -

V 1

220 3,05 L -45,57

72,16 L 45,57

y la reactancia capacitiva que se requiere de acuerdo con (e) es: Xc:::::: 51 ohmios, que corresponde a una capacidad: 1 X =e

Cea

1 1 C=-=--=----

X)nf

51 2n' 50

62,4 ,uF

Como quiera que el motor era de 1 kW, significa que la capacidad necesaria es del orden de 60 a 70 ,uF/kW, que es el valor que seala la Norma UNE. El lector puede comprobar, sustituyendo (e) en (b), que el valor de la tg es:

tg

cos cp e = --=----

2j2 - 3 sen cp

que en el caso que nos ocupa da lugar a un ngulo e :::::: 45. Es difcil conseguir ngulos ms elevados; sin embargo, es suficiente para un funcionamiento correcto tanto del motor trifsico como monofsico.

e) Motor con espira de sombra


Constituye el modelo ms sencillo y corresponde al tipo de motor monofsico ms pequeo. Est formado por un esttor de polos salientes (Fig. 4.50a) que dispone de un devanado concentrado alimentado por una red monofsica y un rotor en forma de jaula de ardilla.

<1>'1

<1>"1

<1>ce

b)

Figura 4.50.

Motor con espira de sombra y diagrama fasoral de flujos y corrientes.

348

MQUINAS ELCTRICAS

Alrededor de cada polo y abrazando parte del mismo se coloca una espira de sombra; parte del flujo principal del esttor, $;, atraviesa esta espira, induciendo una f.e.m. Eee que a su vez produce una corriente lec en retraso (por presentar una pequea reactancia), que da lugar a un flujo $,c que sumado vectorialmente con $; (Fig. 4.S0b) da como resultado un flujo auxiliar $a desfasado f3 grados en el tiempo, respecto al flujo $;' que atraviesa la otra parte del polo. Ambos flujos $a y se encuentran desplazados tambin en el espacio un ngulo y; en consecuencia se produce un campo giratorio (de naturaleza elptica) que se dirige hacia la espira de sombra, dando origen a un par en el eje. Debido a la pequea magnitud de este par, las aplicaciones de este motor son muy limitadas: ventiladores de uso domstico, equipos de aire acondicionado, tocadiscos, mquinas fotocopiadoras, proyectores de diapositivas, etc. Se fabrican con potencias que llegan hasta l/20 kW.

$;'

EJEMPLO DE APLICACIN 4.16 Un motor asncrono monofsico de 1.800 W, 220 V, 50 Hz, 4 polos, tiene los siguientes parmetros del circuito equivalente: = 3 Q; X, = 5 Q; = 1,5 Q; X; = 2 Q; X~ = 100 Q; se desprecian las prdidas en el hierro y mecnicas. Si el motor se conecta a una red de 220 V, 50 Hz y trabaja con un deslizamiento del 5 por 100, calcular: 1) corriente absorbida por el motor y sufactor de potencia; 2) potencia mecnica desarrollada; 3) par en el eje; 4) rendimiento.

R,

R;

SOLUCIN 1. El circuito equivalente del motor es el mostrado en la Figura 4.43, en el que los valores de las resistencias totales del rotor (propia + carga) para cada campo son:

R~ I,S -=--= 2s 2 O,OS

IS Q

R'2
2(2 - s)

, = O 38S Q 2(2 - O,OS) , (el smbolo


11

1S

De este modo las impedancias Zd y Z son, respectivamente conexin en paralelo): Z d }

significa

.X~ (R~ + .X~) _jSO(IS + JI) 2 2s } 2 IS + j SI


11 11

14,14

L 20,21

Z. = .X __!!_ I } 2

(R'

2(2 - s)

+'

X')
_3_

} 2

=J'SO(O,38S + }'1) = 1 OS L 69 37 Q 0,38S + j SI ' ,

Si se toma la tensin como referencia, la corriente absorbida por el motor ser:

11 =

VI _;_ __ RI +jXI +Zd+Z

220 LO 3 + jS + 14,14 L 20,21 + 1,OS L 69,37

11,07 L -33,13 A

y por tanto, el motor absorbe una corriente de 11,07 A con un f.d.p. de cos 33,13 = = 0,837.

CAPTULO 4.

MQUINAS ASNCRONAS

349

2. Por la regla del divisor de corriente, se obtienen las intensidades I~d e I~i: 50 } = 10,41 L -16,73 15+j51 50 I~i = 11,07 L -33,13) = 10,85 L -32,7 0,385 + j 51 I~d = 11,7 L -33,13 Por consiguiente, de acuerdo con (4.174) se obtiene una potencia mecnica interna total: Pmi = 1.544,24 - 43,01 = 1.501,23 W La potencia de 1.544,24 W indica la potencia mecnica producida por el campo directo, mientras que la de 43,01 W representa la potencia debida al campo inverso. De este modo el campo directo produce un 1,029 de la potencia total, mientras que el campo inverso contribuye al -0,029 de la potencia total desarrollada por el motor. 3. La velocidad de sincronismo es segn (4.166) igual a 1.500 r.p.m., que corresponde a una velocidad mecnica segn (4.71) de 1.425r.p.m., lo que indica un par en el eje de acuerdo con (4.175) de:
T=

l.501,33 =1006N.m l.425 ' 21(-60

4. La potencia elctrica absorbida por el motor de la red ser, de acuerdo con (4.176):
PI

= 220 . 11,070,837 = 2.038,3 W

lo que da lugar a un rendimiento, segn (4.177):


r =

1.501,23 = 73,65 % 2.038,43

que es claramente inferior al que poseen los motores trifsicos. En la prctica el rendimiento de los motores monofsicos oscila entre el 60 y el 80 por 100.

4.14. FUNCIONAMIENTO DEL MOTOR ASNCRONO TRIFSICO ALIMENTADO CON TENSIONES DESEQUILIBRADAS
Cuandose alimenta un motor asncrono trifsico con tensiones desequilibradas, el estudio de su comportamiento requiere el empleo de las componentes simtricas. Si las tensiones simples aplicadas al motor son: VRN, VSN y VTN, la teora de las componentes simtricas permite expresarlas tensiones anteriores como suma de tres juegos de tensiones trifsicas simtricas, el primero de ellos se denomina sistema directo o de secuencia positiva, formado por las tres tensiones: (4.181)

350

MQUINAS ELCTRICAS

donde a es el vector complejo: a = I L 120y por tanto a2 = I L -120. El segundojuego de tensiones se denomina sistema inverso o de secuencia negativa, formado por las tres tensiones: (4.182) y el tercer juego de tensiones se denomina sistema homopolar o de secuencia cero, formado por las tres tensiones iguales en mdulo y fase siguientes: (4.183) Al igualar las tensiones de cada fase con la suma de las respectivas componentes simtricas se obtiene:
VRN

VRNd

VRNi VTN

VRNO

VSN

= a2VRNd + aVRNi +

VRNO

= a VRNd + a2VRNi + VRNO

(4.184)

Estas ecuaciones (4.184) permiten calcular las tensiones desequilibradas: VRN, VSN y VTN aplicadas al motor si se conocen los valores de las componentes directa, inversa y homopolar:
VRNd' VRNi

VRNO'

A partir de (4.184) se pueden obtener las relaciones inversas, es decir, los valores de las componentes simtricas a que da lugar el sistema trifsico desequilibrado original, y que son:
VRNd

= Vid =

VRN + aVSN + a2 VTN


3

VRNi

= Vii =

VRN + a2 VSN + aV TN
3

V RNO= VIO =

RN SN TN _"--_--"'--_---'c_

+V +V 3

(4.185)

Al conectar el motor a la red, las tensiones de secuencia directa darn lugar a la circulacin de corrientes de secuencia directa, y de un modo anlogo, las tensiones de secuencia inversa y homopolar producirn corrientes de secuencia inversa y homopolar, respectivamente. Las tres corrientes homopolares son por definicin iguales en mdulo y fase, por lo que su suma no puede ser cero yen el caso de que existiese hilo neutro retomara esta suma por l; sin embargo, si la conexin del motor es a tres hilos, en estrella o tringulo (como es lo frecuente), no existir camino de retomo, por lo que las corrientes homopolares sern siempre nulas. De este modo solamente circularn por el motor corrientes de secuencia directa e inversa. Como quiera que el estudio de un motor trifsico equilibrado alimentado por tensiones simtricas se puede resolver a partir de su modelo monofsico equivalente, debern desarrollarse los circuitos equivalentes del motor por fase para las tensiones de secuencia directa e inversa. Las corrientes de secuencia directa tendern a mover el rotor de la mquina en sentido directo o positivo, mientras que las corrientes de secuencia inversa tendern a mover el rotor en sentido negativo. Si la velocidad de sincronismo del motor es ni Yel motor gira en el sentido del campo positivo o directo a velocidad n, el deslizamiento de este campo Sd tendr un valor:
Sd

ni -n n = s = -= 1- ni ni

(4.186)

En la Figura 4.51a se muestra el circuito equivalente por fase del motor para el sistema de secuencia directa de tensiones, que coincide con el esquema clsico (se ha supuesto que se desprecian las prdidas en el hierro del motor). En la Figura 4.51b se ha representado el circuito equivalente por fase para el sistema inverso de tensiones. Tngase en cuenta para

CAPTULO 4.

MQUINAS ASNCRONAS

351

+
R' 1- s :~
2
S

IOd VI

II

+
EI
4

Eld ~ X } ~

-R-2-s

1- s

a) Circuito equivalente con secuencia directa

b) Circuito equivalente con secuencia inversa

Figura 4.51. Circuitos equivalentes directo e inverso.

deducir este esquema que el deslizamiento correspondiente del campo inverso s en funcin del deslizamiento del campo directo es igual a: s =
ni - (-n)

n 1+-=2-s
ni

(4.187)

por lo que las resistencias de carga correspondiente a cada circuito son, respectivamente:
R;d = R~ (~ - 1) = R~ 1 - s ; R; = R~ (_1 __

2-s

1) = -R~ 1 - s

2-s

(4.188)

Es decir, la resistencia de carga del circuito equivalente para las tensiones inversas es negativa, lo que era de esperar, ya que la potencia mecnica desarrollada es de sentido contrario a la del campo directo. A partir de los circuitos equivalentes anteriores se pueden calcular todas las magnitudes elctricas y mecnicas del motor para ver su comportamiento. Por ejemplo, la corriente de secuencia directa lid en la fase principal del esttor (fase R) se puede calcular con el circuito equivalente de la Figura 4.5Ia, dando lugar a:
lid Vid =;

z.,

donde

Zld

= (R

+ j XI) + ----1 -+--}

(4.189)

'X

R' 'X' 2+2 }


S

De un modo similar, la corriente de secuencia inversa II en la fase principal del esttor (fase R) se obtiene del circuito de la Figura 4.5Ib, y su valor es: donde
ZI =

(RI

+ jXI) + 1 -+---} 'X

(4.190)

R' ~ } 'X' 2+-2

2-s

A partir de las corrientes anteriores es inmediato calcular las intensidades reducidas del rotor para cada uno de los circuitos de la Figura 4.51 y representadas por I~ e I~,y cuyos valores son: l'2, -- I1, R'
_2_

ix,
+ j(X~ + XI')

(4.191)

2-s

352

MQUINAS ELCTRICAS

Por consiguiente, las potencias mecnicas internas desarrolladas por el motor a las diferentes secuencias son: (Pm)d

= 3R:d/~~

; (Pm)i

= 3R:J2

(4.192)

lo que corresponde a una potencia mecnica total: (4.193) y a un par neto en el eje: T= Pmi
)1(l-S)

3
=
)I .

I [/~~

Rz ----;- 2-s

12 ]

(4.194)

Las potencias activas absorbidas por el motor de la red a las dos secuencias se pueden calcular sumando a cada potencia mecnica las prdidas respectivas en el cobre de los devanados del esttor y el rotor, lo que da lugar a: Pld

= 3R:i:;;

+ 3R;uz/~~ + 3Rcul/~d ; Pli

= 3RJ2

+ 3R;u2/2 + 3Rcul/~

(4.195)

En consecuencia, la potencia elctrica total activa absorbida por el motor de la red ser: PI = Pld + Pli lo que indica un valor del rendimiento del motor:
IJ =-

(4.196)

r:
PI

(4.197)

Para estudiar el funcionamiento de un motor trifsico alimentado con tensiones desequilibradas es interesante partir de las componentes directa e inversa de las tensiones de fase. El grado de desequilibrio de estas tensiones se puede cuantificar mediante el cociente modular:
y=-

VIi

(4.198)

VId que se mide en tanto por ciento. De este modo, fijando el valor de la tensin directa VId en la asignada de la mquina y variando el valor del parmetro y se va cambiando la tensin VIi' lo que permite ver su influencia en el funcionamiento del motor. El anlisis se simplifica tomando la fase de la tensin directa como referencia y eligiendo la fase de la tensin inversa adelantada 60 respecto de la anterior, ya que de este modo se hacen iguales dos de las tensiones de alimentacin. Una vez calculadas las corrientes de secuencia directa e inversa, debern evaluarse las corrientes reales en cada una de las fases de entrada mediante las ecuaciones (4.184) pero aplicadas a corrientes. Con objeto de deducir consecuencias prcticas se va a considerar la resolucin de un ejemplo concreto. EJEMPLO DE APLICACIN 4.17
Supngase un motor asncrono trifsico de 30 kW conectado en estrella a una red de 380 V de tensin compuesta, 50 Hz, 4 polos. Se conoce de los datos delfabricante los siguientes valores: RI

= R; = 0,04 n; XI = X; = 0,3 n; Xt =

10 n

CAPTULO 4.

MQUINAS ASNCRONAS

353

Se desprecian las prdidas en el hierro y mecnicas. Se supone asimismo que el deslizamiento a plena carga del motor es s::: 0,01. Si las tensiones directa e inversa aplicadas al motor por fase son:

Calcular: a) tensiones simples aplicadas en la red de alimentacin; b) tensiones de lnea en la red; c) corrientes directa e inversa absorbidas de la red para la fase de referencia; d) corrientes reales en las diversasfases de la red; e) potencia mecnica desarrollada por el motor a las dos secuencias y potencia mecnica total; f) pares directo, inverso y total en el eje; g) prdidas en el cobre para ambas secuencias; h) potencias elctricas absorbidas de la red para ambas secuencias y total del motor; i) rendimiento del motor. SOLUCIN
a)

El grado de desequilibrio de tensiones es y::: (4.184), las tensiones simples de la red son: VRN::: VRNd + VRNi::: VSN
_ 2

Vii::: Vid

__!2_ ::: 5 % y, de acuerdo con


380

380 19 .fi LO + .fi L 60 :::225,08 L 2,42


_

a VRNd + aVRNi 2

380 .fi L -120 380

19 .fi L

180 - 225,08 L -122,42

0_

VTN::: aVRNd + a VRNi


b)

:::

.fi L

19 120 + L -60 :::208,42 L 120

.fi

Las tensiones de lnea aplicadas al motor sern: VRS ::: VRN - VSN ::: 225,08 L 2,42 - 225,08 L -122,42::: 399 L 30 VST::: VSN - VTN ::: 225,08 L -122,42 - 208,42 L 120 :::370,87 L -92,54 VTR ::: VTN - VRN ::: 208,42 L 120 - 225,08 L 2,42 :::370,87 L 152,54

Que representan unas magnitudes: VRS ::: 399 voltios; VST::: VTR ::: 370,87 voltios. Es decir,el desequilibriode tensionesrepresentadopor el factor y ::: 5 por 100equivale a una diferenciaentre las tensionesde lnea de 399 - 370,87::: 28,13 voltios (::::;7,1 por 100). c) Las impedancias del motor a las secuencias directa e inversa, de acuerdo con (4.188) y (4.l89), valen, respectivamente:
Zld:::

(0,04 + jO,3) +

j 10 jO,3 + 0,04/0,01 ZIi::: (0,04 + jO,3) + l 1 l 0,059 + jO,591 ::: 0,5942 L 84,3

l l -+------

::: 3,32 + j 1,86::: 3,804 L 29Y

j 10 jO,3 + 0,04/(2 - 0,01) Obsrvese que la impedancia inversa del motor es aproximadamente la reactancia de cortocircuito del mismo, en virtud del alto deslizamiento del campo inverso, es decir:

-+--------

354

MQUINAS ELCTRICAS

Zld ~

(XI

+ X~) = j Xcc = j 0,6 = 0,6 L 90. Las corrientes directa e inversa del motor
380 -LO 3,804 L 29,3 19 -L60

sern:

j3

57,67 L -29,3

j3

0,5942 L 84,3

18,46L -24,3

De estos resultados se deduce que un desequilibrio de tensiones definido por el factor


y = 5 por 100 se convierte en un desequilibrio de corrientes de valor: r:x=-=-- ~ 32%
lid

1;

18,46

57,67

Es decir el desequilibrio de tensiones se manifiesta enormemente en el desequilibrio de corrientes. Una forma rpida (pero aproximada) de averiguar la relacin entre los factores anteriores se deduce de la propia definicin y teniendo en cuenta que la impedancia a la secuencia inversa es prcticamente la reactancia de cortocircuito del motor, y as se cumple:
Vid

11; z VI; r:x=-=-=-lid

VI;

Vid Z;

z., ~ --=--=y--z; 1 x; x; VI; VI; Vid Vid

Xcc(Pu)

ZI;

z;

d)

donde Xcc(Pu) representa la reactancia de cortocircuito del motor en valores por unidad. En definitiva, el factor de desequilibrio de corrientes es I/Xcc(Pu) veces el factor de desequilibrio de tensiones. Las corrientes en las diversas fases de la red son: IR

= lid + I; = 57,67 L -29,3 + 18,46 L -24Y = 76,1 L -28,1 Is = a211d + a l., = 52,6 L -167,9 IT = al., + a21; = 49,5 L 108,5

e)

Que son tres corrientes distintasque se desvan respecto de la asignadade 57,67 A. AS, la corriente en la fase R es de 76,1 A, que es casi un 32 por 100 ms elevada que la asignada, la corriente en la fase S es de 52,6 A, que representa un 8,8 por 100 inferior a la asignada, y la corriente en la fase T, que es de 49,5 A, es un 14 por 100 inferior a la asignada. Conviene que el lector advierta estas desviaciones de las corrientes absorbidas por la mquina teniendo en cuenta que el desequilibrio de tensiones es solamente del 5 por 100. Las corrientes reducidas del rotor al esttor para las componentes de secuencia directa e inversa se obtienen aplicando (4.191), resultando ser: JIO ~ 52,2 L -8,1 A I~d = 57,67 L -29Y ---0,04 + j(0,3 + 10) 0,01

C'APTULO 4.

MQUINAS AsNCRONAS

355

jlO I~i = 18,46 L -24,31 ------:::::: 17,92 L -24,2 A 0,04 ---+ j(0,3 + 10) 2 - 0,01 y por consiguiente, las potencias mecnicas debidas a ambas componentes son:
(Pm)d = 3R2 -S-

,l-

/2d =

'2

3 . 0,04'

1 - 0,01 2 0,01 52,2 :::::: 32.366 W; 1 - 0,01


2

(Pm)i = 3R2

, 12_

S '2 S /Zi

= 3 0,04 . 2 _ 0,01 17,92 :::::: -19,18 W;

=> Pmi =

32.366 - 19,18 :::::: 32.347 W

f)

Es decir, el desequilibrio de tensiones provoca una reduccin inapreciable en la potencia mecnica desarrollada por el motor. Teniendo en cuenta que la velocidad de sincronismo del motor vale:
ni

= --

601,
p

= --

60 50
2

1.500 r.p.m.

y como quiera que el deslizamiento del motor es del 1%, la velocidad de giro de la mquina es: n

= nl(1

- s)

1.500(1 - 0,01)

1.485 r.p.m.

De este modo los pares directo, inverso y total del motor son:
Td =

32.366 1.485 2n-60

= 208,13 N.m

=>

= -0,12 N.m 1.485 2n-60 T= 208,13 - 0,12 = 208,01 N.m Ti =

-19,18

=>

g)

Es decir, el desequilibrio de tensiones provoca una reduccin inapreciable en el par mecnico que desarrolla el motor. Las prdidas en el cobre son, respectivamente:
Pcud = 3R~/;~ + 3RI/~d = 3 0,04 (52,22 + 57,672)

726,1 W Pcui = 3R~/; + 3RJ'{ = 3 0,04 (17,922 + 18,462) :::::: 79,44 W


::::::

h)

Las potencias elctricas absorbidas por el motor de la red a las secuencias directa e inversa son, respectivamente: P1d

= 32.366 + 726,1 = 33.092 W

; P'i

= -19,18

+ 79,44

= 60,26 W

i)

que corresponde a una potencia total PI = 33.092 + 60,26 = 33.153 W. El rendimiento del motor es, de este modo:
r

p.
_..!!!!.

PI

= --

32.347

33.153

= 97,57 %

Conclusiones: En la Tabla 4.3 se resumen los principales resultados no solamente para el factor de desequilibrio de tensiones del 5 por 100 con el que se ha resuelto el problema, sino

356

MQUINAS ELCTRICAS

Tabla 4.3. Magnitudes del motor en funcin del ndice de desequilibrio de tensiones

VRN VSN VrN VRS VST VrR


lid

219,39 L 0 219,39 L -120 219,39 L 120 380 L 30 380 L _90 380 L +150 57,67 L -29,3 O 52,20 L -8, I O 57,67 L -29Y 57,67 L 90,7 32366 726,1 O 33.092 97,81 % 208,13

220,5 L 0,5 220,5 L -120,5 217,2 L +120 383,8 L 30 378,1 L -91 57,67 L -29,3 3,69 L -24,)D 52,20 L -8,1 3,58 L -24,2' 61,35 L -29,0 55,64 L 93,78 32366 726,1 3,18 33.095 97,80% 208,12

222,8 L 1,5 212,8 L + 120 391,4 L 30 374,4 L -91,SD 57,67 L -29Y 11,1L -24,3 52,20 L -8, I 10,75L -24,2 68,71 L -28,SD 53,94 L -160 52,1 L 100,7 32359 726,1 28,6 33. 144 97,72% 208,1

225, I L 2,4 208,4 L + 120 399 L 30 .370,9 L -92,5 57,67 L -29,3 18,46 L -24Y 52,20 L -8, I 17,92L -24,2 76,1 L -28,1 52,6 L -167,9 49,S L 108,SD 32.347 726,1 79,44 33.153 97,57% 208,01

227,5 L 3,3 227,5 L -123Y 204,0 L + 120 406,6 L 30 367,4 L -93,6' 57,67 L -29Y 25,8 L -24Y 52,20 L -8, I 25, I L -24,2 83,5 L -27,8 52,3 L -IW 47,8 L

231,1 L4,r 231,1 L -124,7 197,5L +120 418 L 30 362,5 L -95,2 57,67 L -29,3 36,9 L -24,3 52,20 L -8, I 35,85 L -24,2 94,51 L -27,4 53,8 L 172,2 47,4 L 130,3 32.290 726,1 317,8 33.333 96,87% 207,6

222,8 L -122,SD 225,1 L -122,4

378,1 L +150,5 374,4 L +151,5 370,9 L +152,SD 367,4 L + 153,6 362,5 L +155,2

t;

t:

2d

I~; IR

t,
Ir

57,67 L -149,3 56,21 L -152,75

llr

r;

32329 726,1 155,7 33.211 97,34% 207,9

r:
r:
P, T

para los siguientes valores: O(sin desequilibrio), 1,3, 5, 7 Y 10 por 100. De este modo esta tabla puede servir para que el lector examine con detenimiento la influencia del factor de desequilibrio en las diversas magnitudes del motor. En la Figura 4.52 se presenta un grfico realizado con MATLABen el que se representa el factor de desequilibrio de corrientes en funcin del factor de desequilibrio de tensiones. En la Figura 4.53 se presenta un grfico que muestra la evolucin de las corrientes en las diversas fases de la red en funcin del factor de desequilibrio de tensiones. Aspectos prcticos: Se comprueba en este ejemplo que un pequeo desequilibrio en las tensiones de alimentacin de un motor ocasiona una gran variacin en las corrientes absorbidas por la mquina. Parece a primera vista que la potencia y el par del motor no se ven seriamente afectados, pero debe tenerse en cuenta que debido al aumento de las corrientes de secuencia inversa, conforme se eleva el factor de desequilibrio se pueden producir grandes calentamientos en los devanados. Si el motor dispone de un rel magnetotrrnico calibrado para la corriente nominal o asignada de 57,67 A (que corresponde a un factor de desequilibrio del Opor 100), es evidente, como se comprueba en la Tabla 4.3, que para un desequilibrio de tensiones del l por 100 la corriente en la fase R pasa a valer 61,35 A, lo que se traduce en que el motor no podr desarrollar su potencia asignada porque actuara la proteccin trmica del motor al quedar sobrecargada la fase R; esto significa que la mquina solamente podr suministrar una potencia mxima de 57,67/61,35 ;:;::: 94 por 100 de la nominal o asignada, pero si el desequilibrio fuera del 5 por 100, el motor solamente podr dar una potencia del 57,67/76,1 :::::; 75,8 por 100 de la asignada y para un desequilibrio del 10 por 100 la potencia

CAPiTULO 4.

MQUINAS ASNCRONAS

357

70.---,---_.---.,---~--_.--_.----~--_.--~--__,
I , , I , ,

*
e
Q)

60 ------ ------ -------:-------~------:------~------~-------~------:-

:
,
,
,

:
,
,
,

I I

:
, ,

, ,

',

I .l

.JI
,

.JI
I

,
I I I

I
, ,

,
I I I

I
I I I

,
I ,

I
I

I
I

I
I

------

------

-------:-------:-------------.,, " I I , " , , ,


I "

-----:-------:-------r----'"
'"

'1

",
, ,

- - --- - - - - - - - - - - -- - -,-- - - - - -r-- -

-T , ,
,

--

- - -,
, ,
I

- - - --

- .... - - - - - - -r- - - - -I I
I

-r - - - -
,

I I
I

,
I I

,
, I ,

,
I I ,

,
I I I I

,
, , , ,

-----,.-----,
, , I

..------ ..------..,-------,..------r----, I I I
, ,

,
I I I

,
I I ,

,
I I I

I I I

10 - ----- --- --~---- ---:------ -~------~- -----~-- --- -..!- - - - - - -:..- - - - - - ~- - - --

Cociente tension inversa a tension directa en %

Figura 4.52.

Evolucin del desequilibrio de corrientes en funcin del desequilibrio de tensiones.

utilizable sera solamente el 57,67/94,51 ~ 61 por 100 de la asignada, lo que representa una fuerte reduccin en la potencia mecnica que puede suministrar el motor. Afortunadamente, no es frecuente que se tengan grandes desequilibrios en las redes de suministro de energa elctrica, por lo que en la prctica los motores elctricos no deben tener ningn problema de funcionamiento. Pero aun existiendo una red simtrica de alimentacin se pueden producir fuertes desequilibrios, si la lnea que une la red (o transformador) con el
95r---~--~--,----.---.--~--_.----r---~~

:
I

~ 90 -.----------j------+------~------------j------~-------~--.:.,

..
~ ,.:

i:::: i::.. l
~Q)

~ 85 -

------i-... --+------~----.. i.
: : : : :

,r---

~~~r~~j~,~,.I~,<~ --

:'

j:::-

J/~T:<r: :J:.::..::-.1::::
:
:

70 - - - - -- ------~ -------:-.i.. ~-~~ ..~ -~ - - _.

65

------~------~ ------~ ---_ .. ....~. : -- ------~ : : : - _....- - -_.. --.~~~.=-:~.--~------~------ ------~------~------~------~-----: :

; 60

_~-::~-i,<::-;--- ----1---- --~------! - -----j------~---- ---~------f- .--

!:: .:"1,,"z::~:=-;t~::=:::1::;~_~f:;:~t~;;-1_-_-_--~:-: .
450~--L---~2--~3--~4~--5~--~6--~7--~8~--9L-~10
Cociente tension inversa a tension directa en %

Figura 4.53. Evolucin de las corrientes de la red en funcin del desequilibrio de tensiones.

358

MQUINAS ELCTRICAS

motor es de gran longitud y se utilizan cables unipolares en su instalacin, pero sin haber tenido la precaucin de conseguir una simetra en su trazado, 10 que origina un desequilibrio de las reactancias de cada fase. Un ejemplo concreto en ingeniera civil 10constituyen los grupos motobombas que se colocan sumergidos en los pozos para suministro de agua a poblaciones aprovechando las aguas subterrneas, donde las lneas tienen longitudes que en algunos casos superan los 400 m. Existen experiencias en que por este fallo de montaje, los grupos motobombas no pueden funcionar a plena carga y no queda ms remedio que reformar la lnea red-motor, buscando la simetra de los conductores de las tres fases o realizando las trasposiciones correspondientes para igualar las reactancias de la lnea, evitando de este modo los posibles desequilibrios. Este problema no aparece si se emplean cables tripolares, ya que entonces las fases quedan equilibradas debido a la simetra del propio cable.

4.15. MQUINAS ASNCRONAS ESPECIALES


Se van a comentar en este apartado una serie de mquinas elctricas de induccin que por sus caractersticas especficas, por su construccin o por su rgimen de funcionamiento corresponden a aplicaciones muy especiales de la mquina asncrona, estando su campo de trabajo muy limitado.

4.15.1.

Regulador de induccin

El regulador de induccin supone un rgimen especial de funcionamiento de la mquina asncrona convencional y se emple durante muchos aos para controlar la tensin en las redes elctricas; hoy da se ha sustituido casi completamente por transformadores con tomas variables. Dispone de un esttor y un rotor trifsico devanados, en el que puede regularse la posicin de este ltimo respecto del primero por medio de una transmisin por tornillo sinfn. En la Figura 4.54 se muestra el esquema de conexiones del regulador de induccin. Se tiene una mquina asncrona clsica en la que el primario es el rotor (que est bloqueado) y el secundario es el esttor. Al introducir una tensin primaria de VI voltios por fase al rotor se

RI

+
VI

+ V2

R2

SI

T2

Figura 4.54. Regulador de induccin.

CAPTULO

4.

MQUINAS ASNCRONAS

359

produce un campo giratorio en el entrehierro que induce f.e.m.s. en ambos devanados, cuyas magnitudes, de acuerdo con la expresin (4.28), cumplen la relacin:
El E2 KINI KzN2

(4.199)

Estas f.e.m.s. estn desfasadas en el tiempo los mismos grados que forman en el espacio. De acuerdo con el circuito de la Figura 4.54, y en el supuesto de considerar despreciables las cadas de tensin, se cumplir: (4.200) ecuaciones que son el resultado de aplicar el 2. lema de Kirchhoff a las mallas de esttor y rotor. Si los ejes de los devanados forman CI. grados, la 2. a ecuacin anterior se podr representar por el diagrama fasorial de la Figura 4.55. Se observa que el lugar geomtrico de V2 corresponde a los puntos de una circunferencia de radio E2 y centro en P, cuyos valores extremos resultan ser: (4.201) En consecuencia, es posible obtener tensiones variables V2 en mdulo y magnitud partiendo de una tensin VI constante. La regulacin puede hacerse de forma continua, sin necesidad de conmutaciones; por ello, esta mquina requiere poco mantenimiento. Las principales aplicaciones se centran en aquellos casos en que son necesarias tensiones variables con gran margen de regulacin: electroqumica, alimentacin de laboratorios de ensayos elctricos, soldadura por arco, etc.

4.15.2. Selsyns
El trmino selsyn proviene de la contraccin de las palabras inglesas: self-synchronous devices (dispositivos autosincronizados), y son sistemas que sirven para sincronizar las posiciones angulares de dos ejes, en puntos separados, en donde una interconexin mecnica entre ellos resultara impracticable. Existen dos tipos fundamentales de selsyns: a) Losformados por motores asncronos con esttor y rotor trifsicos (rotor devanado), que se emplean para aplicaciones de potencia y que reciben el nombre de ejes elctricos (electric shaft o arbre lectrique). b) Los formados por motores asncronos, con esttor trifsico y rotor monofsico, que se emplean en aplicaciones de control, o de pequea potencia, y que se denominan sincros o selsyns pilotos.

Figura 4.55. Composicin de tensionesen el reguladorde induccin.

360

MQUINAS ELCTRICAS

El principio de funcionamiento de ambos tipos es el mismo y difieren en sus aplicaciones, por lo que aqu nos referiremos nicamente a los selsyns de potencia o ejes elctricos, que se emplean en aquellos accionamientos en los que es necesario obtener la misma velocidad en distintos ejes que no estn unidos entre s por transmisiones mecnicas. Consideremos el esquema de la Figura 4.56, en el que se muestran dos motores primarios 1y 2 que mueven dos ejes independientes 1 y 2 que es preciso que giren en sincronismo mutuo. Se han acoplado mecnicamente a los motores anteriores sendos motores asncronos de anillos rozantes que tienen conectados entre s sus devanados de esttor y rotor. Estos ltimos pueden actuar aisladamente sobre la maniobra y control del grupo o pueden actuar como mquinas auxiliares respecto de los primeros. El eje elctrico lo forman realmente los dos motores asncronos, que son idnticos. Si los dos grupos de mquinas estn sometidos a los mismos pares de carga, los ejes respectivos girarn a la misma velocidad. En el caso de que un motor quede sobrecargado respecto del otro, disminuir su velocidad de tal forma que aparecer una corriente de circulacin entre los rotores respectivos debido a la diferencia de f.e.m.s. inducidas en los secundarios de ambas mquinas; esta corriente originar un par de sincronizacin o igualador que tender a acelerar el motor sobrecargado y a retardar la posicin de la mquina menos cargada, hasta que se igualen las velocidades y posiciones de los ejes respectivos. En consecuencia, el sistema permite obtener un sincronismo en la posicin de dos ejes separados entre s sin necesidad de una conexin mecnica rgida, lo cual es importante en aquellas aplicaciones en que la distancia entre los ejes hace imposible cualquier acoplamiento convencional. Estos sistemas se emplean en las gras de los puertos que presentan un gran glibo, donde es preciso que la traslacin de las ruedas de ambos carriles se produzca a la misma velocidad para evitar descarrilamientos. Tambin es frecuente su aplicacin en puentes gras y en los mecanismos de maniobra para la apertura de las compuertas de los aliviaderos de las presas. El tipo monofsico se emplea en servomecanismos de control de posicin, teledeteccin y otros; por ejemplo, para determinar la inclinacin de los alerones de un avin, indicar la posicin de las barras de control de un reactor nuclear, etc.

4.15.3. Motor de induccin lineal


Un motor de induccin lineal es un tipo de motor asncrono en el que se ha sustituido el movimiento de rotacin de la mquina por uno de traslacin, lo cual le confiere unas peculia-

~ RO---.-----------~---------------1~--------~~------U

CI S

~ To---+--+--.------t--+_-.-----------+--t-~------+__r~~-

MOTOR ASNCRONO I

MOTOR ASNCRONO 2

Figura 4.56. Esquemadel eje elctrico.

CAPTULO

4.

MQUINAS ASNCRONAS

361

ridades caractersticas que lo hacen aplicable a una diversidad de casos prcticos. Este motor se puede considerar que se deriva de la mquina convencional, por desarrollo de la misma al cortarle por una generatriz, como muestra la Figura 4.57. Al aplicar un sistema trifsico de corrientes al esttor se produce un campo magntico que se propaga a la velocidad de sincronismo por el entrehierro de la mquina, de tal forma que aparecen unas corrientes inducidas en el secundario de la mquina, que al reaccionar con el campo inductor producen una fuerza de traslacin que obliga a mover al rotor. El sistema tiene las ventajas de poder trabajar con velocidades ilimitadas, ya que ningn elemento est sometido a velocidades centrfugas. El movimiento lineal se obtiene sin necesidad de utilizar ruedas, engranajes o cualquier otro dispositivo mecnico similar. Sin embargo, el modelo de la Figura 4.57 presenta algunos inconvenientes. En primer lugar, aparecen fuerzas de atraccin entre el esttor y el rotor mucho ms elevadas que la fuerza de traslacin de la mquina; estas fuerzas se equilibran en el motor cilndrico en virtud de su simetra, pero aqu es preciso duplicar el devanado del esttor con objeto de anular estas acciones, como muestra la Figura 4.58; aun as, es preciso igualar los entrehierros de ambas caras para que no existan fuerzas transversales. En segundo lugar, debido a la simetra del devanado se obtienen deformaciones en el campo en las extremidades del miembro ms corto (el primario en la Fig, 4.58); esto conduce a una serie de fenmenostransitoriosque pueden hacer que las caractersticas del motor sean muy diferentes de las que cabra esperar utilizandolas teoras convencionales empleadas a lo largo de este captulo. En el libro de Laithwaite citado en la bibliografa pueden profundizarse los conceptos anteriores, que se salen de los lmites de este texto. Normalmente la jaula de ardilla del elemento secundario es sustituida por una banda de cobre o aluminio continua, en virtud de su mejor conductividad elctrica (Fig, 4.58). En las aplicaciones a la traccin elctrica, el motor lineal se est ensayando como medio de transporte no convencional de alta velocidad; en este caso el primario es el rgano mvil (vagn), estando el secundario fijo, que sirve como sistema de guiado (va); para mejorar el rendimiento y obtener velocidades elevadas (no inferiores a 300 km/h) se emplean imanes superconductores, que provocan una suspensin o levitacin magntica del vehculo, lo que evita todo tipo de rozamientos. La superconductividad se obtiene haciendo circular por los devanados helio lquido a temperaturas que oscilan entre 3 y 4 K. Existen otras aplicaciones para lanzaderas

Figura 4.57.

Gnesis del motor lineal a partir del desarrollo de un motor rotativo.

362

MQUINAS ELCTRICAS

PRIMARIO

SECUNDARIO

Figura 4.58.

Motor lineal con doble alimentacin para eliminar las fuerzas transversales.

de telares, cintas transportadoras con banda metlica, accionamiento para puertas con apertura automtica, etc.

4.16. APNDICE: EL PAR DE ROTACIN DE UN MOTOR ASNCRONO DESDE EL PUNTO DE VISTA FSICO
En el epgrafe 4.7 se ha calculado el par de rotacin de un motor asncrono trifsico a partir de su circuito equivalente. El procedimiento es sencillo y muy didctico, ya que utiliza el potencial de la teora de circuitos para determinar las expresiones del par en funcin de los parmetros de la mquina y permite comprender la forma de las curvas par-velocidad y su influencia en el comportamiento del motor. Otro procedimiento alternativo para explicar la creacin del par en una mquina elctrica es el que se utiliz en el epgrafe 2.10 del Captulo 2, que haca uso de los fasores espaciales de f.m.m. Vamos a analizar en este epgrafe la correlacin entre ambos puntos de vista y el significado fsico de la creacin del par a partir de los fasores espaciales. Considrese el circuito equivalente por fase de un motor asncrono en el que se consideran despreciables las prdidas en el hierro de la mquina, sealado en la Figura 4.59. De
RI jXI

+
VI

II

11'

Ir

jX

R ---+
I

jXI'

El =E

R'2 -- S

1-

Figura 4.59.

Circuito equivalente por fase de un motor asncrono trifsico.

CAPTULO

4.

MQUINAS ASNCRONAS

363

acuerdo con este circuito, si se toma como referencia de fases la f.e.m. El' la expresin de la corriente del rotor reducida al esttor I~ es: l' _ El
2 -

R'
=:) .X'2
2 S

----====

(~~y
u

El

-qJ
2

qJ2

+X?

= arctg -, -_ R2/s

X~

(4.202)

es decir, la corriente reducida I~ se retrasa de la f.e.m. El un ngulo qJ2. Por otro lado, la corriente que circula por la reactancia magnetizante de la rama central vale:
I

= ~ = El
JXI'
X'I

L -90

(4.203)

es decir, la corriente de imanacin se retrasa 90 respecto de la f.e.m. El. Esta corriente 1'1 es la que produce el flujo magntico <1> en el entrehierro del motor. Por otro lado, si se aplica el primer lema de Kirchhoff al nudo A se puede escribir: (4.204) Debe destacarse que la ecuacin anterior est en consonancia con el circuito equivalente de la Figura 4.12, en el que la corriente I~ produce un efecto desmagnetizante respecto de II (es decir, la corriente II entra por el terminal sealado por punto en el circuito del esttor, mientras que la corriente I~sale por el terminal sealado con punto en el rotor). Se puede escribir una ecuacin equivalente a (4.204) considerando una corriente reducida del rotor 1; contraria a I~,es decir: 1;= -I~,tal como se muestra en la Figura 4.59; en este caso la corriente 1;tiene un efecto magnetizante respecto de 11, y de este modo la ecuacin equivalente de (4.204) se puede escribir tambin as: (4.205) lo que significa que el efecto combinado de las corrientes del esttor y del rotor produce la corriente de imanacin 1,1, que genera a su vez el flujo magntico en el entrehierro. En la Figura 4.60 se muestra un diagrama fasorial temporal en el que se han representado las variables ms importantes que aparecen en el circuito equivalente de la Figura 4.59.

Figura 4.60. Diagrama de fasores temporales del circuito de la Figura 4.59.

364

MQUINAS ELCTRICAS

De acuerdo con el circuito equivalente de la Figura 4.59 y el diagrama fasorial de la Figura 4.60, la potencia que atraviesa el entrehierro Pa es la potencia activa total que absorbe el rotor, y se puede expresar as: (4.206) y teniendo en cuenta la expresin (4.93), el par de rotacin que produce la mquina ser: T :::--

r,
nI

mI El t; cos ((J2 :::--=--=-=----=

(4.207)

2n60

2n60

nI

Pero si se tiene en cuenta que la velocidad de sincronismo es n, ::: 60;/p y que la pulsacin de la tensin aplicada de red al esttor es w, = 2nll'la ecuacin (4.207) se transforma en:
pmIEI/~ cos ((J2 T:::-_;__:...._;_--

wI

(4.208)
/1

Ahora bien, del diagrama fasorial de la Figura 4.60 se cumple que /~cos ((J2 ::: que (4.208) se transforma en:

sen 61, por lo

(4.209) Igualmente, de acuerdo con (4.8) el valor de la f.e.m. 2n E,::: j2 KJ]N[$m


El

es: (4.210)

En la ecuacin anterior se ha tenido en cuenta que el factor 4,44 procede del cociente 2n/ j2 y que $m representa el flujo magntico mximo en el entrehierro, Al sustituir (4.210) en (4.209) resulta:
p T::: j2m,K]N/Pm/l

sen 61

(4.211)

pero de acuerdo con los resultados del ejemplo de aplicacin 4.1, el valor de laf.m.m. mxima producida por las mI bobinas del esttor del motor que llevan una corriente eficaz /] es:

.9'=-m] --

j2 n

K,N]/]
p

(4.212)

por lo que (4.211) se puede expresar, en funcin de la f.m.m. anterior, del siguiente modo: (4.213) expresin del par que coincide con la (2.140) demostrada en el epgrafe 2.10 del Captulo 2 (en este caso existe un signo menos cuyo significado es que el par tiende a reducir el ngulo 6J que forman los fasores espaciales de flujo magntico y de f.m.m.). En definitiva, el lector puede comprobar de este modo que las expresiones del par motor de las ecuaciones (4.74) y (2.140) son equivalentes. , Con el fin de que el lector comprenda en profundidad los conceptos de interaccin de las f.m.m.s del motor con el flujo comn e incluso de interaccin de las f.m.m.s entre s, se han

CAPTULO

4.

MQUINAS ASNCRONAS

365

representado en la Figura 4.61 los fasores espaciales de f.m.m. que se producen en un motor asncrono trifsico de dos polos en el mismo instante en que se ha realizado la composicin de fasores temporales de la Figura 4.60. En la Figura 4.61, para sealar el sentido de las corrientes en los conductores del esttor y del rotor se ha utilizado un punto (corriente saliente al plano de la hoja) y una cruz (corriente entrante al plano de la hoja). Las cruces y los puntos tienen diferente tamao, para indicar que la corriente no tiene la misma magnitud en todos ellos sino que se distribuye como una capa de corriente senoidal a lo largo de los conductores tanto del esttor como del rotor (a mayor tamao de cruces o puntos, mayor es la corriente en los conductores correspondientes). Las corrientes del esttor producen un fasor espacial de f.m.m. ,Si'; proporcional a la corriente II' definido por la ecuacin (4.212), Yque en el momento considerado tiene la orientacin sealada en la Figura 4.61 y que se mueve a la velocidad de sincronismo nI en sentido contrario a las agujas del reloj. De un modo anlogo, las corrientes en el rotor producen una f.m.m. resultante (reducida al circuito del esttor) que se ha denominado :7;' o ,7;', segn se considere su efecto desmagnetizante o magnetizante respectivamente. Esta f.m.m. :7;' se mueve respecto del rotor a velocidad n2 = nI - n, pero como el rotor gira mecnicamente a velocidad n, la velocidad absoluta de la f.m.m. :7;' respecto a un referencial en reposo es tambin igual a nI' En definitiva, las f.m.m.s. del esttor ~;;Z;-y del rotor ,o/;' giran a la misma velocidad, es decir, ambas f.m.m.s. estn enclavadas elctricamente. Es interesante que el lector compruebe las analogas geomtricas existentes entre las Figuras 4.60 y 4.61. En la Figura 4.61 se han sealado tambin los polos norte y sur equivalentes a las f.m.m.s, producidas tanto por los devanados del esttor como los del rotor. Se observa que prcticamente la f.m.m, ,7,' acta casi en oposicin a la f.m.m.. ~ del esttor. Ambas f.m.m.s., sumadas geomtricamente de una forma anloga a la ecuacin (4.205), dan lugar a la f.m.m. de imanacin .~: (4.214)

.7,.' = -

.72

Figura 4.61. Diagrama de fasores espaciales de un motor asncrono bipolar.

366

MQUINAS ELCTRICAS

La f.m.m. resultante anterior .:;;;: debe vencer la reluctancia del circuito magntico para producir el flujo comn <D en el entrehierro de la mquina, que gira tambin a la velocidad de sincronismo nI' Este flujo comn es el que genera las f.e.m.s. en los conductores del rotor, cuyas corrientes originan a su vez la f.m.m. del rotor. 7;'. El que esta f.m.m. no se retrase 90 respecto del flujo comn (en el sentido de la rotacin), sino que ambos fasores espaciales formen un ngulo ()2 = 90 + CP2'es debido al flujo de dispersin rotrico (en definitiva, lo que sucede es que el circuito del rotor no es resistivo puro y tiene una reactancia equivalente a ese flujo disperso, como lo seala el circuito elctrico equivalente de la Fig, 4.59). Es decir, la corriente I~ se retrasa CP2respecto de El = E~. De acuerdo con (4.213), el par electromagntico producido en funcin de la f.m.m. del esttor .7;' es de la forma: (4.215) donde se ha llamado constante del par kT al cociente np2/2. La expresin anterior es una forma conveniente para comprender el control vectorial de los motores asncronos trifsicos, que se explica en el epgrafe 7.12.6. Sin embargo, se pueden obtener otras expresiones del par equivalentes a la anterior, y as, de acuerdo con la Figura 4.61, se observa que se cumple la siguiente igualdad:
.1'1

sen

()I

= .7;' . sen

()2

= . 1'2' . sen (90 +C(2) =

'0' . cos CP2

(4.216)

por lo que otra expresin equivalente a (4.215) es: (4.217) que es anloga a la sealada en la ecuacin (2.140) del Captulo 2. Para comprender el significado de la ecuacin anterior, tngase en cuenta que si la tensin de alimentacin VI es constante y se desprecia la cada de tensin en el circuito del esttor, la f.e.m. El en la rama central del circuito equivalente de la Figura 4.59 es constante, lo que significa que el flujo del motor es constante en todo momento y por consiguiente tambin la f.m.m. de imanacin jll' Si el rotor pudiera girar a la velocidad de sincronismo nI' los conductores del rotor no cortaran flujo magntico, por lo que no se generara f.e.m. ni corriente en el rotor; esto es, /~ = .7;' = O,Yde acuerdo con (4.217) no puede haber par de rotacin, es decir, es imposible que el motor pueda girar por s mismo a la velocidad de sincronismo. Al aumentar el par resistente del motor, aumentan el deslizamiento s, la corriente del rotor y los ngulos ()2 y CP2'Para bajos deslizamientos del motor la corriente del rotor crece casi proporcionalmente con el deslizamiento, mientras que el ngulo CP2se mantiene casi constante en CP2~ 0, y el par de rotacin generado de acuerdo (4.217) crecer con la misma ley de aumento de la corriente, es decir, aumenta proporcionalmente con el deslizamiento. En esta situacin el fasor ,7;' de la Figura 4.61 se retrasa 90 respecto a .:;;;:, lo que representa, segn indica la ecuacin (4.216), una condicin ptima de trabajo para un cierto flujo en el entrehierro y una determinada corriente en el rotor. Esta situacin corresponde a la zona estable de la curva par-velocidad del motor sealada en la Figura 4.22 y se produce en la prctica para deslizamientos comprendidos entre el O y ellO por 100. Al aumentar el par resistente del motor, aumenta el deslizamiento y por consiguiente la corriente del rotor, pero tambin aumenta el ngulo CP2'A pesar de que /2 sigue creciendo (y por tanto la f.m.m.. 7;'), el par slo puede aumntar ligeramente hasta llegar, por ltimo, a un estado en que el aumento de la corriente rotrica viene compensado por la disminucin del

CAPTULO

4.

MQUINAS ASNCRONAS

367

cos (/>2 Yel par alcanza su valor mximo (se ha llegado al punto e de la curva par-velocidad de la Fig. 4.22). Un nuevo incremento del deslizamiento a partir de este punto hace que la reduccin del cos (/>2 sea superior al aumento de la corriente rotrica y el par generado se hace cada vez ms pequeo hasta que se produce la parada de la mquina (esta situacin corresponde a la zona CD de la curva de la Fig. 4.22). Puede demostrar el lector, de acuerdo con la expresin (2.135) del Captulo 2, que otra ecuacin del par generado, equivalente a (4.215) y (4.216), en funcin de los fasores espaciales de f.m.m. de esttor y rotor, es: T=

"2 p

110' nDf

2g

.7;'.7; sen (j = K,Y;,7; sen s

. ..'

..... ,

(4.218)

donde (j es el ngulo que forman .7;' y ./'/ (y que es igual a (JI + (j2)' De acuerdo con esta ecuacin, el par generado tiende a reducir el ngulo (j que forman las f.m.m.s. del esttor y del rotor, o de otro modo, las f.m.m.s.. Y; y .7,.' tienden a alinearse, y ese esfuerzo de alineacin, que simplemente se debe a la atraccin de los polos correspondientes que se forman tanto en el esttor como en el rotor de la Figura 4.61, es la causa de que se produzca un par de rotacin en el motor.

PROBLEMAS
4.1. La potencia absorbida por un motor asncrono trifsico de 4 polos, 50 Hz, es de 4,76 kW cuando gira a 1.435 r.p.m. Las prdidas totales en el esttor son de 265 W y las de rozamiento y ventilacin son de 300 W. Calcular: a) El deslizamiento. b) Las prdidas en el cobre del rotor. c) Potencia til en el rbol del motor. el) Rendimiento. [Resp.: a) 4,33 %. b) 194,78 W. c) 4.000 W. d) 84 %.] 4.2. Un motor de induccin trifsico, de 8 polos, 10 CV, 380 V, gira a 720 r.p.m. a plena carga. Si el rendimiento y f.d.p. a esta carga es del 83 por 100 y 0,75, respectivamente, calcular: a) Velocidad de sincronismo del campo giratorio. b) Deslizamiento a plena carga. c) Corriente de lnea. d) Par en el rbol de la mquina. NOTA: JI = 50 Hz. [Resp.: a) 750 r.p.m. b) 4 %. e) 17,9 A. d) 97,6 N.m.] 4.3. Un motor asncrono trifsico de 4 polos, conectado en estrella, se alimenta por una red de 380 V, 50 Hz. La impedancia del esttor es igual a 0,1 + jO,4 Q/fase y la del rotor en reposo reducida al esttor vale 0, I + j 0,3 Q/fase. Calcular: a) Intensidad absorbida en el arranque. b) Corriente a plena carga, si el deslizamiento es del 4 por 100. c) Potencia y par nominal si se desprecian las prdidas mecnicas. d) Rendimiento en el caso anterior si las prdidas en el hierro son iguales a 1.200 W. [Resp.: a) 301,36 A; b) 81,56 A; e) 47,89 kW; 317,6 N.m; d) 90,2 %.] 4.4.' Un motor trifsico de 4 polos, conectado en tringulo, se alimenta por una red de 220 V, 50 Hz. La impedancia del rotor en reposo es igual a 0,2 + j 1,6 n, siendo la impedancia del esttor despreciable. La relacin de transformacin es igual a 2 tm; = m)' Calcular: a) Intensidad absorbida de la red y su f.d.p. para un deslizamiento deIS por 100. b) Potencia y par en el eje en el caso anterior. c) Velocidad a la cual se obtiene el par mximo y par mximo correspondiente. d) Rendimiento del motor cuando trabaja con par mximo. NOTAl Se desprecian las prdidas mecnicas y en el hierro. [Resp.:a) 22,10 A; 0,928. b) 7,43 kW; 49,8 N.m.; c) 1.312,5r.p.m.; 72,2N.m. d) 87,5 %.]

368

MQUINAS ELCTRICAS

4.5. Un motor asncrono trifsico de anillos rozantes de 10 CV, 220/380 V, 50 Hz, 4 polos, tiene los siguientes parmetros del circuito equivalente: R = R~ = 0,5 n; X = X~= 3 n; PFe = Pm = O. El rotor est conectado en estrella y el nmero de espiras del rotor es igual al del esttor, con idnticos factores de devanado. a) Si la red es de 220 V, 50 Hz, cmo se conectar el esttor del motor? Dibujar la placa de bornes con la disposicin de los terminales, indicando sus letras de identificacin. b) Conectado el motor correctamente, de acuerdo con el apartado anterior, cul ser el par de arranque del motor con tensin asignada? Qu corriente absorber el motor de la red en el arranque? e) Cul ser la velocidad del motor a plena carga, es decir, cuando desarrolla 10 CV? d) determinar el par de plena carga y la capacidad de sobrecarga del motor. e) Calcular el valor de la resistencia que debe aadirse en serie con cada fase del rotor para obtener el par mximo de arranque. [Resp.: a) Tringulo; b) 12,5 N.m; 62,65 A; e) 1.452,6 r.p.m.; d) 48,38 N.m; 1.465; e) 5,521 n.] 4.6. Un motor asncrono trifsico conectado en estrella de 3.000 V, 24 polos, 50 Hz, tiene los siguientes parmetros: R = O; X = O; R~ = 0,016 n; X~= 0,265 n. Se obtiene el par de plena carga a la velocidad de 247 r.p.m. Calcular: a) velocidad para par mximo; b) capacidad de sobrecarga: TmJT". [Resp.: a) 234,9 r.p.m. b) 2,61.] Un motor asncrono trifsico de 4 polos, 25 CV, 380 V, 50 Hz, tiene un par de arranque de 322 N.m y un par de plena carga igual a 124 N.m. Determinar: a) El par de arranque cuando la tensin del esttor se reduce a 220 V. b) Tensin que debe aplicarse al esttor para obtener un par de arranque igual al par de plena carga. [Resp.: a) 108 N.m. b) 235,81 V.]

4.7.

4.8. Los parmetros de la rama serie de un motor asncrono trifsico de anillos rozantes conectado en estrella, 380 V, 4 polos, 50 Hz, son: R = R~ = 1 n; XC" = 4 n. Calcular: a) Par de plena carga si el deslizamiento es el 4 por 100. b) Resistencia que debe aadirse a cada fase del rotor, para obtener el par nominal, a la mitad de la velocidad de plena carga con los anillos cortocircuitados. La relacin de transformacin es m = mi = 2, y las prdidas mecnicas son despreciables. [Resp.: a) 33,4 N.m. b) 3 n.] 4.9. Un motor asncrono trifsico de anillos rozantes de 15 kW, 380 V, 50 Hz, 6 polos, tiene los siguientes parmetros (con los anillos deslizantes cortocircuitados): R = R~= = 0,8 n; X = X~= 2 n. Los devanados del esttor y rotor estn conectados en estrella y m" = mi = 2. Calcular: a) Par de arranque. b) Par mximo y velocidad correspondiente. e) Resistencia que debe conectarse en serie, por fase, en el rotor para obtener en el arranque los 2/3 del par mximo. [Resp.: a) 59,4 N.m. b) 804 r.p.m.; 141,3 N.m. e) 2,71 no 0,158 n.] 4.10. Un motor asncrono trifsico de 6 polos, 50 Hz, tiene una resistencia del rotor por fase de 0,2 n y un par mximo de 162 N.m a 875 r.p.m. Calcular: a) El par cuando el deslizamiento es el 4 por 100. b) La resistencia adicional que debe aadirse en el circuito del rotor para obtener los 2/3 del par mximo en el arranque. NOTA: Prescindir de la impedancia del esttor. [Resp.: a) 94 N.m. b) 0,41 03,99 n.]

CAPTULO 4.

MQUINAS AsNCRONAS

369

4.11.

Un motor asncrono trifsico de anillos rozantes de 15 kW, 380 V, 50 Hz, 6 polos, tiene los siguientes parmetros: R = R~ = 0,8 n; X = X~ = 2 n (los anillos estn en cortocircuito). Los devanados del esttor y rotor estn conectados en estrella y mv = mi = 2. Calcular: a) Par de arranque. b) Para mximo y velocidad correspondiente. c) Resistencia que debe conectarse en serie, por fase, en el rotor para obtener en el arranque los 2/3 del par mximo. [Resp.: a) 59,4 N.m. b) 804 r.p.m.; 141,3 N.m c) 2,71 no 0,158 n.]

4.12. Un motor asncrono trifsico en jaula de ardilla, conectado en estrella, de 3,5 kW, 220 V, seis polos, 50 Hz, ha dado los siguientes resultados en unos ensayos: Ensayo de vacio o de rotor libre: tensin compuesta aplicada: 220 V; corriente de lnea del esttor: 3,16 A; potencia absorbida en el ensayo: 590 W. Se sabe tambin que las prdidas mecnicas (rozamiento + ventilacin) a velocidades cercanas a la asignada son de 312 W. (Se pueden despreciar en este ensayo las prdidas en el cobre del esttor. El lector puede comprobar que representa una potencia de 7,2 W, que se puede considerar despreciable). Ensayo de cortocircuito o de rotor bloqueado: tensin compuesta aplicada: 34,3 V; corriente de lnea: 14,5 A; potencia absorbida: 710 W. A la temperatura de funcionamiento, la resistencia entre dos terminales cualesquiera del esttor es de 0,48 ohmios. Si se conecta el motor a una red trifsica de 220 V de lnea y se considera aceptable utilizar el circuito equivalente aproximado del motor, calcular: 1) Parmetros del circuito equivalente aproximado del motor reducido al primario (esttor). 2) Si el motor gira a 960 r.p.m., determinar: a) potencia mecnica til en el eje suministrada por el motor; b) corriente de lnea absorbida por el motor de la red y f.d.p. correspondiente; c) potencia elctrica absorbida por el motor de la red; d) rendimiento del motor; e) par mecnico til en el eje. [Resp.: 1) RFe = 174,7 n; XI' = 41,4 n; R = 0,24 n, R~ = 0,885 n. x; = 0,773 n 2) a) 1.744 W; b) 7,2 A; cos <p = 0,891; e) 71,5 %; d) 2.440 W; e) 17,4 N.m.] 4.13. Un motor de induccin trifsico de 4 polos, 50 Hz, tiene una resistencia del rotor por fase de 0,25 n, siendo la impedancia del esttor despreciable. El par mximo se obtiene para una velocidad de 1.200 r.p.m. Si la capacidad de sobrecarga es igual a 2,1, calcular: a) Velocidad a plena carga o asignada. b) Relacin par de arranque a par asignado o nominal. [Resp.: a) 1.425 r.p.m. b) 0,8173.]

4.14. El circuito equivalente de un motor de induccin trifsico de 4 polos, conectado en estrella, presenta los siguientes valores: R = R~ = 0,85 n; Xcc = 5 n. Si la red tiene una tensin de 380 V, 50 Hz, calcular: a) Corriente de arranque y de plena carga, si el deslizamiento en este ltimo rgimen es del 4 por 100. b) Par de arranque y de plena carga. c) Velocidad del motor cuando consume una corriente mitad de la de arranque y par electromagntico desarrollado por la mquina en ese instante. Prescndase de la rama paralelo del circuito equivalente. [Resp.: a) 41,54 A; 9,68 A. b) 28 N.m; 38 N.m. e) 1.350 r.p.m.; 69,5 N.m.] 4.15. Un motor trifsico de jaula de ardilla, tiene una impedancia del esttor despreciable. La capacidad de sobrecarga, TmJTn, vale 2,5 y el cociente par de arranque a par nominal es igual a 1,5. Calcular el deslizamiento a plena carga y el deslizamiento al cual se obtiene el par mximo. [Resp.: 6,96%; 33,33 %.]

370

MQUINAS ELCTRICAS

4.16. Un motor de induccin trifsico de 4 polos, 50 Hz, tiene una capacidad de sobrecarga de 2,5 y desarrolla su par mximo a 900 r.p.m. Calcular: a) Deslizamiento a plena carga. b) Cul ser la tensin mnima que debe aplicarse a la mquina, expresada en tanto por ciento de la asignada, para obtener el par de plena carga en el arranque? NOTA: Despreciar la impedancia del esttor. [Resp.: a) 8,34 %. b) 76,15 %.] 4.17. Se tiene una estacin de bombeo de agua que lleva una bomba centrfuga que tiene incorporado un motor asncrono trifsico en jaula de ardilla de 15 CV, 380-220 V, 50 Hz, 6 polos, y que tiene los siguientes parmetros: RI = R~ = 0,8 ohmios; XI = X~= 2 ohmios; PFe = Pm = O (se puede prescindir de la rama paralelo del circuito equivalente). a) Si la red es de 380 V, 50 Hz, cmo se conectar el motor? Dibuje el cuadro de bornes. Indique el nombre correcto de los terminales. b) Conectado el motor correctamente, de acuerdo con el apartado a), cul ser el par de arranque del motor con tensin asignada? Si el par resistente por la bomba en el arranque es de 50 N.m, arrancar el motor? e) Si en rgimen permanente el par resistente es igual a 100 N.m, cul ser la velocidad a la que girar el motor? (de las dos soluciones obtenidas tmese la ms lgica). el) Qu corriente absorber el motor en el caso anterior? Cunto valdr la potencia desarrollada por el motor en el eje? e) Si el motor se alimenta por medio de un transformador ideal de relacin 15 kV/380 V 5 por 100, conexin Dy11, a travs de una lnea trifsica de impedancia 0,1 + j 0,5 ohmios/fase, arrancar el motor? (recurdese que el par resistente en el arranque es de 50 N.m. En caso negativo, qu procedimiento sera el ms adecuado para que pueda arrancar el motor? NOTA: La lnea est en el lado de B.T. del transformador y se conecta a la toma de 380 V del mismo. [Resp.: a) Estrella. b) 59,44 N.m, y arrancar el motor, ya que 59,44 N.m > 50 N.m. e) 925 r.p.m. d) 18,08 A; 9,66 kW. e) Con 380 V el motor no arranca. Si se utiliza la toma +5 % del transformador,es decir, si la tensin secundariaes de 380 V + 5 % 380 = = 399 V, se produce un par de 52,55 N.m, que es superior a 50 N.m, por lo que de este modo arrancar el motor.] Un motor asncrono trifsico de 4 polos y rotor devanado tiene los siguientes parmetros del circuito equivalente por fase: RI = 0,1 ohm; XI = 0,5 ohm; R~ = 0,2 ohm; X~= 1 ohm; mv = mi = m = 2. Se desprecia la rama paralelo del circuito equivalente y tambin las prdidas mecnicas. El motor est conectado en tringulo y la tensin de la red es de 380 V, 50 Hz. Calcular: 1) Si la potencia mecnica desarrollada a plena carga por el motor es de 86 kW, cul ser la velocidad del rotor? 2) Cul es la corriente que absorbe el motor de lnea de alimentacin y su f.d.p. cuando desarrolla la plena carga? 3) Qu resistencia debe aadirse al rotor por fase para que la corriente de arranque no sea superior a dos veces la de plena carga? 4) Si estando el motor girando a plena carga (a la velocidad calculada en el apartado 1)se conmutan entre s dos fases de la alimentacin, cul ser el par de frenado desarrollado en ese instante? (Se supone, para la resolucin de este apartado, que no existe resistencia adicional en el rotor.) [Resp. 1) 1.425 r.p.m. 2) 150,62 A; 0,94. 3) 0,321 Q. 4) 123,46 N.m.]

4.18.

4.19. Un motor asncrono trifsico con rotor en jaula de ardilla de 6 p010s, 50 Hz, est conectado en estrella a una red de 380 V. Los parmetros del circuito equivalente del motor son: RI = R~ = 0,5 ohmios; XI = X~ = 2 ohmios; Pm = O (desprciese la rama

CAPTULO

4.

MQUINAS ASNCRONAS

371

paralelo del circuito equivalente del motor). El par resistente de la carga se supone que sigue una ley lineal de la forma: T, = 35 + 0,06n (par en N.m; n: velocidad en r.p.m.) Calcular: 1) Par de arranque y corriente de arranque del motor. 2) Si la tensin de la red se reduce un 10 por 100, podr arrancar el motor? Justificar la respuesta. 3) Con la tensin asignada de 380 V aplicada al motor, a qu velocidad girar el motor con el par resistente sealado? 4) Qu potencia desarrolla el motor en el eje en el caso anterior? NOTA: Para realizar el apartado 3 es preciso resolver una ecuacin de tercer grado en funcin del deslizamiento s. (Sugerencia: el valor de s est comprendido entre el 3 y el 4 por 100). [Resp.: 1) 40,56 N.m; 53,2 A. 2) No arrancar el motor. 3) 960,4 r.p.m. 4) 9,31 kW.j 4.20. Se dispone de un motor asncrono trifsico de rotor devanado (con anillos), de 10 kW, 220/380 V, 4 polos, 50 Hz. Los parmetros del circuito equivalente del motor son: IFe = 11'= O; Pm = O (prdidas mecnicas); R, = R~; X, + X~ =1,5 ohm; m" = mi = m = 2. Se sabe adems que el motor da el par mximo (con los anillos cortocircuitados) a una velocidad de 1.200 r.p.m. La tensin de alimentacin de la red trifsica es de 220 V, 50 Hz. 1) Cmo se deber conectar el motor? Dibujar la placa de bornes, sealando las letras de identificacin de terminales. 2) Calcular la resistencia R, por fase del esttor. 3) Conectado el motor correctamente como se indica en el apartado 1, calcular la intensidad de lnea que absorbe el motor en el momento del arranque con los anillos deslizantes cortocircuitados y par de arranque correspondiente. 4) Si el motor desarrolla una potencia mecnica en el eje de 10 kW (plena carga o asignada), cul ser la velocidad de rotacin correspondiente? (los anillos estn cortocircuitados). 5) Determinar en el caso anterior la corriente de lnea que absorbe el motor y el f.d.p. con el que trabaja la mquina. 6) Si el rotor se conecta en estrella, cul ser la resistencia adicional que debe aadirse al rotor por fase en el arranque para que la intensidad de arranque sea como mximo tres veces la asignada determinada en el apartado 5 y qu par de arranque tendr la mquina entonces? [Resp.: 1) Tringulo. 2) 0,306 n 3) 235,19 A; 107,77 N.m. 4) 1.465,74 r.p.m. 5) 27,59 A; 0,994. 6) 0,935 n, 176,5 N.m.] 4.21. Un motor asncrono trifsico con rotor en jaula de ardilla de 220/380 V, 4 polos, 50 Hz, tiene los siguientes parmetros del circuito equivalente aproximado: R, = = R~ = 0,2 n; Xcc = 0,3 n. Se consideran despreciables las prdidas mecnicas y la rama paralelo del circuito equivalente. Si se conecta el motor a una red trifsica de 380 V de tensin compuesta, 50 Hz: a) Cmo se conectar el motor? Dibujar la caja de bornes, indicando los nombres normalizados de los terminales y la disposicin de los puentes de conexin. b) Potencia mecnica til y par mecnico desarrollado para la velocidad asignada de 1.425 r.p.m. e) Calcular, en las condiciones nominales del apartado anterior, la corriente absorbida por el motor de la lnea, f.d.p. con el que trabaja y rendimiento. d) Estando girando el motor con el par nominal, hasta qu porcentaje se podr reducir la tensin de alimentacin sin que se pare el motor? e) Cul debe ser la tensin mnima para arrancar el motor si mueve un par resistente igual al nominal? [Resp.: a) Estrella; h) 30,94 kW; 207,36 N.m; e) 52,1 A; 0,997; 90,51 %; d) 49,7 % de reduccin; e) 201,78 V.]

372

MQUINAS ELCTRICAS

4.22. Un motor asncrono trifsico con rotor devanado de 220/380 V, 4 polos, 50 Hz, se conecta correctamente a una red trifsica de 220 V de lnea, 50 Hz. La corriente de lnea de plena carga es de 90 A Yse observa que si se realiza el arranque cortocircuitando los anillos, la corriente de arranque es 6,6 veces la asignada o de plena carga, desarrollando un par en esas condiciones que es 2,2 veces el nominal. La resistencia por fase del esttor R es igual a la reducida del rotor R~. Se suponen despreciables las prdidas mecnicas y la rama paralelo del circuito equivalente. Se pide: 1) Conexin del motor. Dibjese la placa de bornes con las letras normalizadas de identificacin de los terminales. 2) Velocidad del motor a plena carga y valores de los parmetros del circuito equivalente: R, R~ Y XC," 3) Potencia mecnica desarrollada por el motor a plena carga, par producido y rendimiento correspondiente. 4) Par mximo desarrollado por el motor con los anillos cortocircuitados y corriente absorbida de la red con su factor de potencia en esas condiciones. 5) Qu resistencia habr que aadir en el rotor por fase para limitar la corriente de arranque a un valor de 300 A de lnea? Qu par de arranque producir el motor en esta situacin? NOTA: mv = mi = 2. [Resp.: 1) Tringulo. 2) 1.424,25 r.p.m.; 0,202 Q; 0,202 Q; 0,499 Q. 3) 30,76 kW; 206,25 N.m.; 90,38 %.4) 624,29 N.m; 426,7 A; 0,83. 5) 0,191 Q; 553,51 N.m.] 4.23. Un motor asncrono con rotor enjaula de ardilla de 220/380 V, 50 Hz, 6 polos, tiene el circuito equivalente por fase mostrado en la Figura P.4.1a. (Se desprecian las prdidas mecnicas y la rama paralelo.) El motor se conecta correctamente para trabajar en una red de 380 V de lnea. a) Si el motor lleva acoplada una caja de engranajes (vase Fig. P.4.1b), con una demultiplicacin de relacin 10:1(es decir, la velocidad se reduce 10 veces, o de otro modo, el par aumenta 10 veces) y mueve un tambor de 50 cm de dimetro sobre el que va arrollando un cable que levanta un peso de 250 kg, cul ser el valor de la velocidad lineal con la que sube el peso de 250 kg? (supngase que la caja de engranajes no ofrece par resistente alguno, sino que nicamente adapta el par del tambor al par en el eje del motor). b) Cul es el rendimiento del motor en el caso anterior?
380V 0,5 n 2 n + 1,5 n 2n

ti R'e

a)

b)

250 kg

Figura P.4.1.

[Resp.: a) 2,428 mis.

b)

90,56 %.]

CAPTULO

4.

MQUINAS ASNCRONAS

373

4.24.

Un motor asncrono trifsico con rotor devanado de 19 kW, 220/380 V, 4 polos, 50 Hz, se conecta correctamente a una red trifsica de 380 V, 50 Hz. Se consideran despreciables: la impedancia del esttor (RI = XI = O), las prdidas mecnicas y la corriente de vaco (no considerar la rama paralelo del circuito equivalente). Se sabe que cuando el motor tiene cortocircuitados los anillos (es decir la resistencia del restato de arranque es nula), las prdidas en el cobre del rotor con par mximo son 8 veces las que tiene con par nominal (plena carga) con un deslizamiento del 3 por 100. Calcular: a) Velocidad del motor para par mximo. b) Corriente que absorbe el motor de la red cuando trabaja a plena carga, par de plena carga y rendimiento del motor en estas condiciones. e) Resistencia que debe aadirse al rotor por fase (mediante el restato de arranque) para que la corriente de arranque sea como mucho igual a dos veces la asignada o nominal. Se sabe que mv = mi = 2. d) Si la resistencia adicional calculada en el apartado anterior permanece fija en el restato de arranque, a qu velocidad girar el motor con un par resistente igual al nominal? Qu corriente absorber el motor de la red en estas condiciones? [Resp.: a) 1.325,72 r.p.m. b) 30,78 A; 124,7 N.m; 96,87 %. e) 0,72 n; d) 830,12 r.p.m.; 30,71 A.] Un motor de induccin trifsico de rotor devanado de 35 kW, 380/Q20 V, 4 polos, 50 Hz, desarrolla su potencia de salida asignada a 1.455 r.p.m. cuando trabaja a la tensin y frecuencia asignadas, estando cortocircuitados sus anillos rozantes. El par mximo que puede desarrollar a la tensin y frecuencia citadas es del 200 por 100 del de plena carga. La resistencia del devanado del rotor por fase es de 0,1 ohmios. La impedancia del esttor es despreciable (RI = XI = O). Se suponen nulas las prdidas mecnicas y se puede omitir en los clculos la rama paralelo del circuito equivalente. Las relaciones de transformacin cumplen la igualdad m; = mi = m. El rotor est conectado en estrella. La red de alimentacin es de 380 V, 50 Hz, estando el esttor conectado en estrella. 1) Calcular la relacin de transformacin m (tmese la ms alta de las dos que se obtienen) y reactancia del rotor a 50 Hz. 2) Calcular la velocidad del motor para par mximo. 3) Resistencia que debe aadirse en serie con cada fase del rotor para obtener el par mximo en el arranque. 4) Si ahora se alimenta el motor a la frecuencia de 60 Hz, ajustando la tensin aplicada de tal forma que el flujo en el entrehierro tenga el mismo valor que a 50 Hz, calcular la tensin que debe aplicarse a 60 Hz. 5) Calcular en el caso anterior la velocidad a la que el motor desarrolla un par de valor igual al de plena carga a 50 Hz, estando los anillos en cortocircuito. [Resp.: 1) m = 1,058; X2 = 0,893 n 2) 1.332 r.p.m. 3) 0,793 n 4) 456 V. 5) 1.755,20 r.p.m.] Se tiene un motor asncrono trifsico con un rotor en jaula de ardilla, que tiene los siguientes datos en su placa de caractersticas: 10 kW, 220/380 V, 50 Hz, 19 A, 1.425 r.p. m., cos cp = 0,90. Se conecta a una red de 380 V, 50 Hz. Se suponen despreciables las prdidas mecnicas y no es necesario considerar la rama paralelo del circuito equivalente. Calcular: a) Parmetros R" R~ YXcc del motor. b) Par de arranque y par de plena carga del motor. Qu tipo de par resistente debe tener el motor para que pueda arrancar? e) Rendimiento del motor con par mximo. d) Velocidad que deber darse al motor por medio de un motor primario externo para que la mquina asncrona funcione como generador entregando su potencia asignada a la red. (Tomar la velocidad ms pequea de las dos posibles). [Resp. a) 0,68 n; 0,486 n. 5,02 n b) 16,82 N.m; 67 N.m; par tipo cuadrtico. e) 79,7 %. d) 1.555,6 r.p.m.]

4.25.

4.26.

374

MQUINAS ELCTRICAS

4.27.

Un motor asncrono trifsico de rotor en jaula de ardilla tiene una placa de caractersticas en la que se leen nicamente los datos siguientes: 380/660 Y, 50 Hz, 585 r.p.m. Los parmetros del circuito equivalente del motor son: IFe = 11' = O; Pm = O (prdidas mecnicas); RI = 0,5 Q; R~ = 0,7 Q; XI = X~= 3 n. Si se conecta el motor a una red de 380 Y trifsica, 50 Hz, indicar: 1)Forma de conexin del esttor del motor y dibujo de la placa de bornes correspondiente. 2) Conectado el motor correctamente, del modo que resulte de acuerdo con el apartado anterior, calcular: si el motor gira a plena carga a 585 r.p.m., el valor de la corriente absorbida por el motor en la lnea, factor de potencia del motor en estas condiciones y la potencia absorbida por el motor de la red. 3) Potencia desarrollada por el motor en las condiciones del apartado anterior, par mecnico en el eje y rendimiento del motor. 4) Si girando la mquina como motor a 585 r.p.m. se intercambian sbitamente dos fases de la red de alimentacin, cul ser en esos momentos el par de frenado desarrollado por la mquina? 5) Si la mquina se hace girar a 615 r.p.m. movida por un motor diesel acoplado a su eje, en el mismo sentido que funcionaba como motor y sin cambiar la secuencia de fases, calcular la potencia mecnica absorbida y la potencia elctrica que la mquina entrega a la red. (Funcionamiento como generador asncrono). [Resp.: 1) Tringulo. 2) 22,6 A; 0,979; 14,56kW. 3) 13,95kW; 227,68 N.m; 95,79 %. 4) 66,42 N.m. 5) 15,69 kW; 15,04 kW.]

4.28. Los parmetros del circuito equivalente de un motor asncrono trifsico en jaula de ardilla, utilizado en una bomba centrfuga para suministro de agua a una fbrica, son los siguientes: RI = R~ = 0,5 ohmios; XI = X~ = 0,75 ohmios (se desprecia la rama paralelo del circuito equivalente y las prdidas mecnicas). La placa de caractersticas del motor seala los siguientes datos: 220/380 Y; 8 CY; 975 r.p.m. El motor se alimenta por medio de una lnea de cobre de 10 mrrr' de seccin de una red de 380 Y de lnea y 50 Hz situada a 3 km de distancia (la resistividad del cobre es de 0,01730 ohmios-rnmvm). 1) Cmo deber conectarse el motor? Dibujar la placa de bornes. 2) Si el par resistente de la bomba es igual a 55 N.m y se supone constante en un amplio margen de velocidad, calcular la velocidad de giro del motor (tmese la ms elevada de las dos posibles). 3) En las condiciones del apartado anterior, calcular la corriente absorbida por el motor de la red y la tensin de lnea en bornes del motor. 4) Se observa que durante el funcionamiento expresado en los apartados 2 y 3 el motor absorbe una corriente superior a la esperada, mientras que la cantidad de agua suministrada era menor a la prevista. Para corregir este defecto se decide plantear una solucin prctica para compensar la cada de tensin en la lnea de 3 km. Para ello se decide alimentar el motor (a principio de lnea por medio de un transformador ideal, de modo que la tensin de alimentacin sea ahora de 450 Y. Si el par de la bomba sigue siendo de 55 N.m, determinar la nueva velocidad del motor, la corriente absorbida y la tensin de lnea que llegar al motor. NOTA: Desprciese la reactancia de la lnea. [Resp.: 1) Estrella. 2) 952,2 r.p.m. 3) 13,53 A; 259,24. 4) 977,5 r.p.m.; 9,29 A; 366,41 Y.] 4.29. Se dispone de un motor asncrono trifsico en rotor de la jaula de ardilla de 4 polos, 220/380 Y, 50 Hz, que tiene los siguientes parmetros del circuito equivalente: RI = 2 ohmios; XI = 5 ohmios; R~ = 1,5 ohmios; X~ = 6 ohmios; se desprecian las prdidas mecnicas y la rama paralela del circuito equivalente. El motor mueve una carga cuyo par resistente es constante y vale 10 N.m. a) Si la red es de 220 Y, 50 Hz, cmo se

CAPTULO 4.

MQUINAS ASNCRONAS

375

conectar el motor? Dibujar el esquema de la caja de bornes, indicando el nombre normalizado de los terminales. b) A qu velocidad girar el motor con el par resistente de 10N.m? Cul ser el rendimiento del motor en estas condiciones. e) Si el motor funciona en rgimen permanente en las condiciones del apartado anterior y se va reduciendo progresivamente la tensin de alimentacin, cul ser la mnima tensin necesaria en la alimentacin antes de que se pare el motor? d) Si se pretende arrancar el motor con el par resistente de 10 N.m, cul ser la mnima tensin necesaria en la red para que pueda arrancar la mquina? [Resp.: a) Tringulo. b) 1.474,14 r.p.m.; 96,1 %. e) 117,48 V. d) 215,67 V.] 4.30. La Figura P.4.2 muestra el esquema de una red de alimentacin a dos gras idnticas de un puerto, representadas por un nico motor (el de elevacin de carga). Los motores empleados son asncronos trifsicos, con rotor en jaula de ardilla de 220/380 V, 6 polos, 50 Hz, conectados en estrella, cuyos parmetros del circuito equivalente (despreciando las prdidas mecnicas y la rama paralelo) son: RI = R~ = 0,2 ohmios, XI = X~ = 0,6 ohmios. Se indican tambin en la figura las impedancias de las redes de distribucin de las gras. La alimentacin se hace al principio de la lnea por medio de una red trifsica de 380 V, 50 Hz. a) Inicialmente arranca la gra nmero 1. Qu par de arranque dar el motor? Si mueve una carga cuyo par resistente es de 100 N.m, cul ser la velocidad final a la que girar el motor? Qu tensin de lnea tendr en bornes el motor? b) Estimando que el motor de la gra nmero 1 gira a velocidad constante (en el valor calculado en el apartado anterior, se arranca el motor de la gra nmero 2, moviendo un par de 80 N.m), en el momento del arranque, cules sern las tensiones en bornes de ambos motores? Arranca el motor de esta gra? e) Una vez que ha arrancado la gra nmero 2 (debe demostrarlo en el apartado anterior), cul ser la velocidad final de rgimen permanente que tomar el motor nmero 2, suponiendo que el par resistente sigue siendo de 80 N.m y que la gra nmero 1 sigue girando a la velocidad calculada en el apartado a)? Cul ser la tensin compuesta en bornes del motor nmero 2? [Resp. a) 134,53 N.m; 984,4 r.p.m; 374,04 V. b) 335,4 V; 271,2 V; Ta2 = 87,52 N.m arranca el motor 2. e) 986,8 r.p.m.; 362,49 V.]
0,2 + jO, I ohm/hilo F Red trifsica 380V 0,3 + jO,2ohmlhilo

Figura P.4.2.

376

MQUINAS ELCTRICAS

BIOGRAFAS
ALGER, Philip L. (1894-1979). Ingeniero americano. Se gradu como ingeniero elctrico en el MIT (1915), Master por la Union College, Schenectady (1920). Trabaj como ingeniero en la General Electric durante 40 aos (entre 1919 y 1959). Su labor ms destacada en la empresa fue en el departamento de mquinas elctricas, estando al frente del departamento de diseo de motores y generadores de induccin. Fue Instructor en el MIT (1915-1916). Desde 1958 a 1970 fue profesor adjunto de ingeniera elctrica y consultor en el Politcnico de Rensselaer. Escribi numerosos artculos tcnicos en relacin con el comportamiento de las mquinas elctricas. Su libro ms famoso fue The Nature of Polyphase lnduction Machines, que es considerado como cita obligada para el estudio de estas mquinas. Escribi varios libros y artculos biogrficos sobre dos grandes ingenieros de la GE: Charles Proteus Steinmetz y Gabriel Kron, con los que trabaj en la vida profesional y cuyos trabajos tanto han influido en los estudios de ingeniera elctrica. Perteneca a numerosas sociedades cientficas: Fellow IEEE (1930), Fellow del ASME, Asociacin americana de amigos de las ciencias y Fellow del lEE. Medalla Lamme en 1959 por sus contribuciones al comportamiento de las mquinas de induccin. Premio Steinmetz (1969). Director del AlEE. Tena varios Doctorados Honoris Causa: St. John's College (1915), Universidad de Colorado (1969). 2. ARAGO, Dominique Francois Jean (1786-1853). Fsico francs. En 1804 ingres en la Escuela Politcnica y al salir de ella fue agregado en calidad de secretario al Bureau des Longitudes, prosiguiendo con Biot la tarea comenzada por Delambre y Mechain de medir el arco de meridiano desde Dunquerque a Barcelona. Profesor de la Escuela Politcnica (1810) Y Director del Observatorio Astronmico (1830). Miembro de la Academia de Ciencias de Pars a los veintitrs aos. Arago, por contribuir en media docena de campos de la ciencia, no pudo emplear su talento a fondo para alcanzar proeza alguna de verdadera importancia en ninguno de estos campos en especial. Trabaj con Biot en ptica hasta 1816, descubriendo la polarizacin rotatoria magntica, la polarizacin cromtica y el polariscopio. Cuando Arago tuvo conocimiento del experimento de Oersted, dio un avance a las propiedades magnticas de las corrientes elctricas y explicando el origen de las fuerzas magnticas. Se distingua por su gran facilidad de palabra, una gran potencia de asimilacin y un gran talento para hacerse inteligible por todo el auditorio. Fue un profesor modelo, a cuya ctedra acudan presurosos gran nmero de alumnos; daba consejos utilsimos a los que se dedicaban al estudio orientndoles en sus investigaciones y sugirindoles mtodos nuevos. Posea una prodigiosa memoria que le permita recordar pasajes enteros que slo haba ledo una vez. Arago tambin gast energas en la poltica, siendo decididamente republicano, y en 1840 lleg a desempear el Ministerio del Interior y ms tarde el de Marina. 3. BAILEY, Benjamin Franklin (1875-1954). Ingeniero americano. Se gradu y doctor en Ingeniera Elctrica en la Universidad de Michigan (1898 y 1907, respectivamente). En 1898 ingres en la GE como ingeniero del Departamento de Ensayos. En 1910 fue profesor ayudante de Ingeniera Elctrica en su Universidad y catedrtico en 1913.Desde 1925hasta su retiro en 1945combin sus trabajo docente como Director del Departamento de Ingeniera Elctrica con el ingeniero consultor, trabajando para las empresas: Fairbanks Morse, Howell Electric Motors, etc. Fund la empresa Bailey Electric en Michigan. Autor de cinco libros de texto y muchos artculos tcnicos. Se le atribuyen diferentes patentes en relacin con sistemas de encendido elctrico para automviles, equipos especiales de encendido para lmparas elctricas y variaciones al motor monofsico de arranque por condensador. 4. BEHREND, Bemard Arthur (1875-1932). Ingeniero suizo. Estudi en Berln, Pars y Londres. Trabaj en Inglaterra como ayudante de Gisbert Kapp. En 1896 ingres en la Compaa suiza de maquinaria elctrica Oerlikon, como ayudante del ingenierojefe; estuvo encargado del laboratorio de ensayos y particip en el diseo de todos los tipos de mquinas elctricas, tanto de c.c. como de c.a., verificando todos sus clculos tericos con los modelos reales. Uno de sus primeros descubrimientos fue encontrar la relacin entre la magnitud y fase de la corriente primaria en un transformador con alta dispersin, aplicando ms tarde sus resultados a los motores de induccin, sentando las bases del diagrama de crculo usado universalmente en la actualidad; el trabajo lo public
1.

CAPTULO 4.

MQUINASASNCRONAS

377

5.

6.

7.

8.

en la revista Elektrotechnische Zeitschrift en enero de 1896, despus de comprobar su teora con los ensayos en un motor real (otros ingenieros como Blondel, Ossanna y Heyland hicieron estudios similares, pero parece que Behrend fue el que contrast mejor los resultados). En 1898 se traslad a los EE.UU., donde dio clases de mquinas elctricas en la Universidad de Wisconsin. En 1900 se incorpor a la empresa Bullock de Cincinnati como director de ingeniera; esta Compaa, que hasta entonces solamente haba fabricado mquinas de C.C., comenz con la ayuda de Behrend a construir todo tipo de mquinas de c.a.: alternadores de polos salientes y motores de induccin. Cuando en 1904la empresa Bullock fue absorbida por Allis-Chalmers, Behrend continu en su puesto en la nueva empresa. En 1908 pas como ingeniero a la Compaa Westinghouse. Fue miembro de un gran nmero de sociedades cientficas y profesionales de Europa y Amrica. Le concedieron el ttulo de Doctor Honoris Causa por la Universidad de Darmstadt en 1931. DOLIVO-DoBROWOLSKY, Mijail Osipowitsch (1861-1919). Ingeniero elctrico ruso. Trabaj como Director de Ingeniera en la Compaa alemana AEG (Allgemeine Electrizitats-Gesellschaft), siendo el responsable del diseo de todas las mquinas elctricas que construa la compaa. Inventor del rotor de doble jaula de ardilla para los motores de induccin. Entre 1889 y 1890 efectu un estudio completo de los transformadores y de las redes trifsicas. Dobrowolsky distingua los montajes en circuito abierto (montaje en estrella) con tres conductores principales y un conductor de retorno comn que se poda suprimir si el consumo era equilibrado, y el montaje en circuito cerrado (montaje en tringulo), con tres conductores solamente. Fue el artfice, junto con Oskar Von Miller (fundador del Museo de la Ciencia de Munich), de la primera instalacin de transporte de energa a gran distancia entre Lauffen y Frankfurt; constaba de un alternador de 32 polos que produca una tensin con fuerza de 86 V Y que con ayuda de un transformador elevaba a 14.000V; la lnea tena una longitud de 175km y se construy para la Exposicin Internacional de 1891. Se le deben tambin inventos especficos con dinamos trifilares. KENNEDY, Alexander Blackie William (1847-1928). Ingeniero ingls. Estudi en la Escuela de Minas de Londres. Fue catedrtico de ingeniera civil y mecnica en University College (1874), creando el primer laboratorio de ensayo de mquinas, preocupado por dar enseanzas prcticas a los estudiantes. Sus primeros trabajos se refieren a cinemtica y dinmica de mquinas. A partir de 1899 se dedic a la ingeniera elctrica. Trabaj en la Westminster Electric Supply Co. y dise numerosas centrales elctricas y sistemas de distribucin de muchas ciudades inglesas. Tambin proyect instalaciones de traccin elctrica. Fue presidente de las siguientes instituciones: Ingenieros Mecnicos (1894), Ingenieros Civiles (1906) y miembro de la Institucin de Ingenieros Elctricos. Elegido Fellow de la Royal Society en 1887. Recibi el Doctorado Honoris Causa por la Universidad de Glasgow (1894), Birmingham (1909) y Liverpool (1913). LANGDON-DAVIES, Walter (1867-1924). Ingeniero ingls. Estudi en la Escuela de Minas de Londres. Invent en 1891 el motor asncrono monofsico. Se traslad ms tarde a Vancouver para trabajar como ingeniero consultor de la British Columbia. A su vuelta a Inglaterra cre la empresa Daysohms Welding para construir equipos dedicados a la soldadura elctrica y desarrollar parte de sus inventos. Tena una gran pasin por los descubrimientos elctricos; sus mtodos de investigacin no eran muy ortodoxos y se dejaba llevar por su instinto e intuicin que la mayor parte de las veces era el camino correcto: construa un prototipo y a continuacin haca pruebas experimentales; entonces correga su teora y adaptaba el modelo a los resultados. Fue miembro del lEE desde 1919. McALLlSTER, Addams Stratton (1875-1946). Ingeniero americano. Se gradu en ingeniera elctrica en la Universidad de Pennsylvania (1901). En este mismo ao obtuvo el ttulo de ingeniero mecnico en la Universidad de Cornell, alcanzando el grado de Doctor en esta Universidad en el ao 1905. Fue profesor ayudante de Fsica en Comell desde 1901 a 1904, compartiendo en este ltimoao el puesto anterior con el de ayudante de ingeniera elctrica. Desde 1905 a 1913 se hizo cargo de la direccin de la misma. Desde 1915 hasta su jubilacin en 1944 ocup diferentes puestosde responsabilidad en la National Bureau of Standards de Washington, compartiendo este trabajo con el de profesor asociado de ingeniera elctrica de la Universidad de Pennsylvania. Escribi varios libros de texto entre los que cabe mencionar el que lleva por ttulo Alternating CurrentMotors, que le dio fama mundial por haberse tomado como libro bsico en muchas uni-

378

MQUINAS ELCTRICAS

9.

10.

11.

versidades del mundo. Fue vicepresidente del IEEE entre 1917 y 1918. Perteneci a gran nmero de organizaciones cientficas: Sociedad Alumbrado, ASME y otras. STURGEON, WiIliam (1783-1850). Fsico ingls. Se educ en el ejrcito. En 1823 puso en prctica la nocin de solenoide expuesta por Ampre. Su propia aportacin fue quiz accidental y consisti en enrollar alrededor de un ncleo de hierro el hilo conductor, observando que aumentaba la fuerza magntica. Sturgeon haba inventado el electroimn, aunque ms tarde Henry mejorara el diseo. Invent tambin un nuevo tipo de galvanmetro y fund el primer diario ingls dedicado a la electricidad. THOMSON,Elihu (1853-1937). Ingeniero americano. Thomson naci en Manchester pero cuando l tena cinco aos su familia se traslad a Philadelphia. En EE.UU. transcurri toda su carrera profesional. Estudio en la Central Higb School y al acabar su formacin ingres en la empresa de E. J. Houston, que era profesor de su Escuela; all desarroll un sistema de alumbrado elctrico por arco y ms tarde fundaron ambos la compaa Thomson-Houston, que en 1892 se asoci con la Edison General Electric Co. para formar la hoy conocida multinacional General Electric. Thomson permaneci como ingeniero consultor en esta empresa el resto de su vida. Dotado de una gran inventiva, Thomson obtuvo patentes por sus mejoras en el diseo de motores de c.a. y transformadores. Invent un generador de alta frecuencia, etc. Hizo contribuciones importantes en radiologa, mejorando los tubos de rayos X y siendo pionero en hacer radiografas estereoscpicas. Se le atribuyen ms de 700 patentes en todos los campos de la ingeniera elctrica. Fue Rector del Massachusetts Institute of Technology entre 1921 y 1923. VEINOTT,Cyril G. (1905-2001). Ingeniero americano. Se gradu en la Universidad de Vermont (1926). Trabaj en la compaa Westinghouse, dedicndose al diseo y desarrollo de motores elctricos de pequea potencia. En 1953 ingres en la Reliance Electric Co., donde fue pionero en la aplicacin de los ordenadores al diseo de motores fraccionarios. Autor de numerosos artculos sobre mquinas elctricas. Escribi varios libros sobre diseo de motores fraccionarios y sobre clculo y construccin de motores de induccin. Fellow del IEEE en 1948. En 1951 recibi el Doctorado Honoris Causa por la Universidad de Vermont. Recibi en 1977 la medalla Tesla del IEEE y en el ao 2000 la medalla de honor.

REFERENCIAS
1. 2. 3. 4. 5. 6. 7. 8. 9. 10. 11. 12. 13. 14. ALGER,P. L.: Nature of Polyphase lnduction Machines. J. Wiley, New York, 1951. COCHRAN,P.: Polyphase Induction Motors: Analysis, Design, and Application. Marcel Dekker, Inc., New York, 1989. COLLILANZI,C.; Ejes elctricos: accionamientos y automatismos. Ed. Tcnicas Rede, Barcelona, 1964. CORTES,M.: Curso Moderno de Mquinas Elctricas Rotativas, tomo III. Ed. Tcnicos Asociados, Barcelona, 1973. CHAPMAN,S. J.: Mquinas Elctricas. McGraw-HilI Latinoamericana, S. A. Bogot, Colombia, 1987. CHATELAlN,J.: Machines lectriques. Trait d'lectricit, Vol. X. Ed. Georgi, Lausanne, 1983. CHILIKIN,M.: Accionamientos elctricos. Ed. Mir, Mosc, 1972. DELTORO, V.: Electric Machines and Power Systems. Prentice Hall, Inc., Englewood Cliffs, N. J., 1985. DRAPER,A.: Electrical Machines. Longman Group, London, 1971. ENGELMAN, R. H.: Static and Rotating Electromagnetic Devices. Marcel Dekker, lnc., New York, 1982. F1TZGERALD, A. E.; KINGSLEY,CH.; UMANS, S. D.: Electric Machinery, Fourth Ed. (S. 1.). McGraw-Hill Book Co., New York, 1985. IVANOV-SMOLENSKY, A. V.: Electric Machines, tomo 2. Mir Publishers, Moscow, 1982. Kosow, 1.: Electric Machinery and Transformers. Prentice Hall, Inc., Englewood Cliffs, New Jersey, 1972. KOSTENKO,M. P.; POTROVSKI: Mquinas Elctricas, tomo Il. Ed. Mir, Mosc, 1975.

CAPTULO

4.

MQUlNAS ASNCRONAS

379

KOVACS,P. K.: Transient Phenomena in Electrical Machines. Elsevier, Akademiai Kiad, Budapest, 1984. 16. LAITHWAITE, E. R.: Mquinas de Induccin Especiales. Ed. Labor, Barclona, 1976. 17. LANGSDORF, A. S.: Teora de las mquinas de CA. Ed. Castillo, Madrid, 1967. 18. MABLEKOS, V. E.: Electric Machine Theory for Power Engineers. Harper & Row Pub., New York, 1980. 19. MATSCH,L. W.: Mquinas Electromagnticas y Electromecnicas. Representaciones y Servicios de Ingeniera S. A., Mxico, 1972. 20. MCPHERSON,G.: An lntroduction 10 Electrical Machines and Transformers. John Wiley & Sons, New York, 1981. 21. MOELLER, F.; KUBLER, E.; WERR, TH.: Electrotecnia general y aplicada, tomo JI, 1." parte. Ed. Labor, Barcelona, 1961. 22. NAGRATH,1. J.; KOTHARI,D. P.: Electric Machines. Tata McGraw-Hill Pub., New De1hi, 1985. 23. SAY, M. G.: Alternating Current Machines. Pitman Pub Co., London, 1976. 24. SLEMON,G. R.; STRAUGHEN,A: Electric Machines. Addison-Wesley Reading, Massachusetts, 1980. 25. SMEATON,R. W.: Motor Application and Maintenance Handbook. McGraw-Hill, New York, 1969. 26. STEIN.R.; HUNT, W. T.: Electric Power System Components: Transformers and Rotating Machines. Van Nostrand Reinhold Co., New York, 1979. 27. THALER,E. J.; WILCOX,M. L.: Mquinas elctricas: Estado dinmico y permanente. Ed. LimusaWiley S.A., Mxico, 1969. 28. VEINOTT,C. G.: Motores Elctricos de Potencia Fraccionaria y Subfraccionaria. Ed. Marcombo, Barcelona, 1978. 29. WILDI, TH.: Tecnologa de los Sistemas Elctricos de Potencia. Ed. Hispano Europea S.A., Barcelona, 1983. 30. YAMAMURA, S.: Theory of Linear lnduction Motors. Halsted Press Book J. Wiley, New York, Uni versity of Tokyo, 1972. 15.

CAPTULO

Mquinas sncronas

5.1. INTRODUCCIN
Las mquinas sncronas son mquinas elctricas cuya velocidad de rotacin 11 (r.p.m.) est vinculada rgidamente con la frecuenciaj de la red de corriente alterna con la cual trabaja, de acuerdo con la expresin: 60j
11=--

(5.1)

donde p es el nmero de pares de polos de la mquina. Las mquinas sncronas, como cualquier otro convertidor electromecnico de la energa, estn sometidas al principio de reciprocidad electromagntica, pudiendo funcionar tanto en rgimen generador como en rgimen motor. Sin embargo, en la prctica de las instalaciones elctricas es ms frecuente su empleo como generadores, para producir energa elctrica de corriente alterna (alternadores) en las centrales elctricas a partir de fuentes primarias de energa hidrulica,trmica o nuclear. Las frecuencias industrialesde las tensiones generadas oscilan entre los 50 Hz en Europa y 60 Hz en gran parte de Amrica. En aplicaciones especiales, como es el caso de la aeronutica, se utilizan frecuencias ms elevadas, del orden de los 400 Hz, lo que trae como consecuencia una reduccin del tamao y peso de los equipos de a bordo. En la generacin de energa elctrica a pequea escala se emplean alternadores acoplados a motores de combustin interna, que se utilizan como equipos de emergencia en hospitales, aeropuertos, salas de ordenadores, centrales telefnicas, etc., y que entran en servicio en el momento que falta la tensin de la red. Tambin se aplican estos grupos en el suministro de energa a instalaciones remotas o aisladas, alejadas de las redes de distribucin, como es el caso de ciertas aplicaciones rurales: riegos por aspersin, granjas, etc. Como ya se ha indicado antes, las mquinas sncronas son susceptibles de funcionar tambin convirtiendo la energa elctrica en mecnica, lo que da lugar al rgimen de marcha como motor sncrono. Estos motores se emplean en aquellos accionamientos industriales que requieren velocidades de transmisin constantes, teniendo adems la ventaja frente a los motores asncronos de poder regular simultneamente el f.d.p. con el cual trabaja, lo que es de gran importancia en cierto tipo de industrias: cemento, metalrgicas, etc., ya que se evita la colocacin de condensadores para reducir la potencia reactiva absorbida por la instalacin. Cuando la mquina sncrona trabaja con f.d.p. capacitivo se dice que funciona como compensador o condensador sncrono. Las centrales elctricas de bombeo constituyen un ejemplo singular de funcionamiento de las mquinas sncronas en sus dos formas bsicas. Estas centrales estn constituidas por un grupo binario: turbina y mquina sncrona. Por el da se

381

382

MQUINAS

ELCTRICAS

aprovecha el salto hidrulico para mover la turbina que hace girar el alternador, generndose energa elctrica. Por la noche o, en general, en las horas de menor consumo, coincidiendo con el valle de la curva de demanda, la mquina sncrona funciona como motor, aprovechando la energa elctrica sobrante de otros tipos de centrales (generalmente trmicas y nucleares), de tal forma que por medio de la turbina se bombea agua de un embalse situado aguas abajo de la presa de la central hasta otro embalse superior, provocando un aumento del nivel de este ltimo, que permite en horas de mayor consumo tener un remanente mayor de energa acumulada, lo que mejora el rendimiento general de toda la instalacin. En este modo de funcionamiento, el motor puede trabajar tambin como condensador sncrono, mejorando el f.d.p, de la red. En este captulo, dedicado al estudio de las mquinas sncronas, comenzamos analizando los aspectos constructivos de las mismas, en sus dos versiones clsicas: de polos salientes, empleadas en las centrales hidrulicas, y de polos lisos o rotor cilndrico, que se utilizan en las centrales trmicas (convencionales o nucleares). Se citan tambin los procedimientos de refrigeracin utilizados para evacuar las prdidas que se producen. Se dan esquemas de los circuitos de excitacin que emplean estas mquinas y el modo en que se realiza la regulacin de tensin. Contina el captulo con el estudio del principio de funcionamiento de un alternador. Primeramente se analiza el comportamiento en vaco, demostrando la expresin de la f.e.m. inducida y recordando la influencia de los factores de devanado en la obtencin de una buena forma de onda de la tensin de salida (que ya se adelant en el Captulo 2). Se explica luego el funcionamiento en carga, analizando con detalle el fenmeno de la reaccin de inducido, tan importante en estas mquinas. A continuacin se construye el diagrama fasorial del alternador, que constituye el mtodo general de clculo de la f.m.m. necesaria en la excitacin para conseguir un funcionamiento con una tensin de salida para una determinada magnitud y f.d.p. de la carga. Se define el concepto de regulacin de tensin y se aplica el diagrama para determinar su valor. Se hace ms tarde un anlisis lineal del alternador, cuyo objetivo est en conseguir un circuito equivalente de estas mquinas. Se explica para ello el mtodo de Behn-Eschenburg o de la impedancia sncrona, que permite traducir las f.m.m.s. en Le.m.s. proporcionales, lo que simplificar ms tarde el tratamiento analtico del comportamiento del alternador. Se describen luego los tipos de ensayos necesarios para la determinacin de la impedancia sncrona. En la pregunta siguiente se hace un anlisis no lineal del alternador para tener en cuenta la saturacin de la mquina. Se introduce el concepto de tringulo de Potier, que permite calcular simultneamente la reactancia de dispersin y la f.m.m. de reaccin de inducido. Los mtodos anteriores se aplican a mquinas de rotor cilndrico. Es por ello que se introduce luego el anlisis de las mquinas de polos salientes, en las que se deben considerar las reacciones tanto en el eje de los polos (eje directo) como en un eje a 90 de aqul (eje cuadratura o transversal). Se explica un mtodo grfico para determinar la orientacin final del fasor de f.e.m. Continuamos despus con el estudio del funcionamiento de un alternador en una red aislada, explicando con detalle el mecanismo de actuacin del regulador de velocidad (governor). Se explica la conexin de un alternador a la red: las condiciones que se deben cumplir y la forma de realizar la maniobra del acoplamiento con un sincronoscopio de lmparas. Se dan frmulas de la potencia activa y reactiva que produce la mquina. Ms tarde se analiza con detalle el funcionamiento de una mquina sncrona conectada a una red de potencia infinita, explicando los efectos que se obtienen al variar la excitacin y al variar la entrada de potencia mecnica actuando sobre el regulador de velocidad. De esta forma se construyen diagramas fasoriales que permiten comprobar los modos de funcionamiento de la mquina tanto como generador o como motor y compensador sncrono.

CAPTULO 5.

MQUINAS SNCRONAS

383

Se analiza luego el funcionamiento en paralelo de alternadores de potencias similares para explicar el procedimiento de variacin de las potencias activas suministradas en funcin de las curvas de estatismo de los reguladores de velocidad. Se explica ms tarde las caractersticas del motor sncrono y sus aplicaciones. Y se comenta la necesidad de incluir devanados amortiguadores en los polos para mejorar el comportamiento transitorio. El captulo finaliza explicando la construccin de un baco que define los lmites de funcionamiento de la mquina sncrona, lo que es de gran inters para los tcnicos que estn al frente de las centrales elctricas y tambin se hace un anlisis simplificado del cortocircuito transitorio del alternador para definir los conceptos de reactancia transitoria y subtransitoria, que tanta importancia tienen en el anlisis dinmico.

5.2. ASPECTOS CONSTRUCTIVOS


Las mquinas sncronas, al igual que los dems tipos de mquinas elctricas, estn constituidas por dos devanados independientes: Un devanado inductor, construido en forma de arrollamiento concentrado o bien distribuido en ranuras, alimentado por corriente continua, que da lugar a los polos de la mquina. b) Un devanado inducido distribuido formando un arrollamiento trifsico recorrido por corriente alterna. En las mquinas pequeas, para potencias que no superan los 10 kVA, el devanado inductor se coloca normalmente en el esttor, en forma concentrada, sobre expansiones magnticas denominadas polos salientes, estando situado el inducido en el rotor, formando generalmente tres fases, las cuales tienen salida al exterior por medio de tres anillos, como se muestra en la Figura 5.1. En las mquinas sncronas grandes, que para el caso de alternadores pueden llegar a 1.000-1.500 MVA, la colocacin de los devanados es inversa a la anterior, de tal forma que los polos quedan situados en el rotor y el devanado trifsico en el esttor. En esta situacin la estructura del rotor se fabrica en dos versiones distintas, ya sea en forma de polos salientes (Fig. 5.2a), ya sea en forma de polos lisos o rotor cilndrico (Fig. 5.2b); en el primer caso los a)

(salida c.a. trifsica)

Figura 5.1. Mquina sncrona de baja potencia con el inducido en el rotor.

384

. MQUINAS ELCTRICAS
DOS ANILLOS DESLIZANTES (entrada de c.c.)

POLOS SALIENTES EN EL ROTOR a) Polos salientes

DEVANADO TRIFSICO DISTRIBUIDO b) Polos lisos

POLOS LISOS EN EL ROTOR

Figura 5.2. Tipos constructivos de mquinas sncronas.

devanados de los polos son concentrados, mientras que para el rotor cilndrico el devanado que se coloca en los polos (generalmente 2 o 4 polos) est distribuido en ranuras, cubriendo una parte de la circunferencia del rotor. La alimentacin del devanado inductor se realiza por medio de dos anillos colocados en la parte mvil de la mquina por los que se introduce una corriente continua exterior. La adopcin de las formas constructivas indicadas en la Figura 5.2 para mquinas de gran potencia presenta diversas ventajas frente a la forma mostrada en la Figura 5.1. Por una parte, un inducido giratorio (Fig. 5.1) requiere tres anillos (caso de mquinas trifsicas) para recoger la tensin generada y enviarla al circuito exterior; estos anillos deben estar ms o menos descubiertos y son difciles de aislar, especialmente para las tensiones elevadas de 6.600 a 30.000 V a las que funcionan normalmente las mquinas sncronas; adems, estos anillos suelen producir perturbaciones debidas a chispas, cortocircuitos, etc.; sin embargo, un inducido fijo (Fig. 5.2) no necesita anillos y sus conductores pueden llevar un aislamiento continuo desde las espiras hasta las barras del cuadro de salida. Debe tenerse en cuenta tambin que es ms difcil aislar los conductores en un inducido giratorio que en un inducido fijo, debido a la fuerza centrfuga ya las vibraciones que produce la rotacin. Cuando el inductor se sita en el rotor, la corriente que lo alimenta debe llegar hasta l por intermedio de dos anillos; como quiera que la tensin de alimentacin del inductor raras veces sobrepasa los 1.000V, Yteniendo en cuenta que la potencia que absorbe es poco importante, los anillos en este caso no producen problemas graves. Otro aspecto a considerar dentro de estas mquinas es la diferencia constructiva del rotor en sus dos formas bsicas mostradas en la Figura 5.2. A este respecto se puede decir que la eleccin entre ambos tipos viene impuesta por la velocidad de rotacin de la mquina, la cual depende a su vez del tipo de motor primario que la hace girar, y as, se distinguen los turbogeneradores, hidrogeneradores y los generadores diesel (ms claramente se dira en estos ltimos: generadores acoplados a motores diesel). Los turbogeneradores estn movidos por turbinas de vapor, las cuales tienen gran rendimiento si se mueven a velocidades elevadas. Estos alternadores tienen un rotor cilndrico (Fig. 5.2b) devanado generalmente para dos polos, lo que corresponde a una velocidad sncrona, de acuerdo con (5.1), de 3.000 r.p.m. a 50 Hz. El eje turbina-alternador es horizontal y se construyen unidades con potencias hasta de 1.000 MVA. En el caso de turbogeneradores

CAPTULO 5.

MQUINAS SNCRONAS

385

empleados en las centrales nucleares, debido a que las temperaturas y presin de vapor en la caldera son menores que en las centrales trmicas convencionales, los generadores son ms econmicos si se construyen con rotor cilndrico pero formando cuatro polos (lo que representa una velocidad de sincronismo de 1.500 r.p.m. a 50 Hz); por otra parte, como el rendimiento econmico global de estas centrales se eleva esencialmente aumentando la potencia del reactor nuclear e instalando para cada uno de ellos un grupo de turboalternadores de potencia adecuada, ste ha sido el estmulo principal para el rpido desarrollo y crecimiento de las potencias de estas mquinas, que se espera que en los prximos aos superen los 2.000 MVA. Los hidrogeneradores estn movidos por turbinas hidrulicas cuyo tipo y velocidad de giro dependen de las caractersticas del salto. En saltos de gran altura se emplean turbinas Pelton que impulsan grupos de eje horizontal que giran a velocidades comprendidas entre 750 y 375 r.p.m. En saltos medios se utilizan turbinas Francis con una disposicin vertical del grupo, de tal forma que las velocidades de sincronismo utilizadas son menores que con alturas elevadas, pudindose llegar a valores cercanos a las 150 r.p.m. En saltos de pequea altura 30 m) se emplean turbinas Kaplan con el eje vertical, donde la velocidad del grupo es generalmente inferior a 100 r.p.m. Las potencias medias de los generadores hidrulicos oscilan entre 150 y 300 MVA y se han llegado a construir unidades de 750 MVA. Debido a las velocidades de giro tan reducidas en comparacin con los turbogeneradores, se construyen en forma de polos salientes (Fig. 5.2a) con un gran nmero de ellos, generalmente entre 20 y 40 polos, aunque se han realizado unidades cercanas a los 100polos. (La central de Bonneville, en EE.UU., tiene alternadores de 60 MVA, 15 kV, 96 polos, 60 Hz, 75 r.p.m.). Los generadores diesel estn movidos por medio de motores de combustin interna a velocidades que no superan las 1.500 r.p.m., habindose construido unidades con potencias prximas a los 20 MVA. Como quiera que la potencia asignada de una mquina es proporcional a su volumen, los turbogeneradores, al ser de gran velocidad, se construyen con un pequeo dimetro y una gran longitud axial, y de esta forma se reducen las fuerzas centrfugas a que estn sometidos los devanados del rotor. Por ejemplo, un alternador de este tipo puede tener de 1 a 2 m de dimetro de rotor y de 10 a 12 m de longitud axial total. (Las velocidades lineales en la periferia del rotor oscilan entre los 125-145 mis en las mquinas tetrapolares y 170-185 mis en los turboalternadores bipolares de gran potencia). Los generadores hidrulicos, como son de menor velocidad, tienen un gran dimetro y una pequea longitud axial. Por ejemplo, un alternador de 200 MVA tiene un dimetro total entre 5-7 m y una longitud axial de 2 a 3 m, pero se han construido unidades con cerca de 20 m de dimetro. Las tensiones generadas por los alternadores no superan los 15 kV para potencias inferiores a 200 MVA; para potencias superiores las tensiones se elevan a 25-30 kV con objeto de reducir el volumen de cobre empleado en la construccin del inducido. Las corrientes de cada fase pueden llegar a 20 kA, por lo que es imprescindible la utilizacin de devanados con circuitos en paralelo. Las corrientes de excitacin pueden alcanzar los 2,5 kA a 1 kV para generadores de 750 MVA. Las prdidas que estas corrientes producen por su efecto Joule en los devanados deben evacuarse de una forma adecuada para evitar daar los aislamientos *. En las pequeas unidades y de baja velocidad es suficiente una refrigeracin por aire que se mueve por conveccin natural; en unidades mayores es preciso recurrir a intercambiadores de

* Un turbogenerador de 1.000 MVA tiene un rotor de 1,25 m de dimetro y 7 m de longitud. El esttor pesa 300 Tm y el rotor 80 Tm. Las prdidas totales a plena carga son del orden de 15 MW, lo que representa una energa calorfica de 13.106 kcal/hora, que debe evacuar al exterior.

386

MQUINAS ELCTRICAS

calor aire-agua colocados en el exterior de la estructura del esttor. En las mquinas de potencias ms elevadas empleadas en los turboalternadores se puede utilizar agua o hidrgeno. Las ventajas del hidrgeno frente al aire son varias, pero se deben destacar las siguientes: a) el hidrgeno tiene una densidad 1/14 veces la del aire, lo que reduce las prdidas por ventilacin y ruido; b) tiene un calor especfico 14 veces el del aire, lo que supone una mayor capacidad de almacenamiento de calor; e) la conductividad trmica es 7 veces superior, lo que reduce los gradientes de temperatura. Al mismo tiempo, la ausencia de oxgeno hace incrementar la vida de los aislantes. Como consecuencia de todo ello, una refrigeracin por hidrgeno al, 2 y 3 kp/crrr' hace que la potencia asignada de estas mquinas pueda elevarse en un 15,30 y 40 por 100, respectivamente, con relacin a una refrigeracin por aire.

5.3.

SISTEMAS DE EXCITACIN

En el epgrafe 5.2 se ha sealado que los devanados que forman los polos de una mquina sncrona se alimentan con corriente continua; esta c.c. procede en los sistemas tradicionales de una dinamo excitatriz del tipo shunt que est montada en el eje del grupo y cuya salida se aplica al rotor del alternador por medio de unos anillos deslizantes con sus correspondientes escobillas. La excitatriz es un generador de c.c. convencional, en el que a veces se sustituye toda o parte de su excitacin por una excitatriz piloto con objeto de mejorar la rapidez de respuesta. En la Figura 5.3 se da una versin de este sistema, donde pueden apreciarse cada uno de estos elementos. (La lnea de puntos indica que las tres mquinas estn situadas en el mismo eje mecnico.) Las mquinas sncronas ms pequeas no suelen tener excitatriz piloto y la excitatriz principal trabaja en forma de derivacin (shunt), alimentando directamente el inductor o campo del alternador. Debido a las dificultades de conmutacin en las dinamos cuando stas giran a grandes velocidades, y en general para evitar la presencia del colector de delgas de las mismas, se han desarrollado desde hace algunos aos excitatrices de c.a. que con ayuda de rectificadores de silicio alimentan con c.c. los polos del alternador. El campo de estas excitatrices proviene generalmente de otra excitatriz de c.a. con imn permanente cuya salida se ha rectificado previamente, como se indica en la Figura 5.4. Generalmente las excitatrices piloto producen una c.a. de una frecuencia cercana a los 400 Hz, mientras que la excitatriz principal es de 50 Hz.

RO--50--T 0------'

ALTERNADOR

Figura 5.3. Sistema de excitacin con dinamo excitatriz.

CAPTULO

5.

MQUINAS SNCRONAS

387

,-----1

RECTIFICADOR TRIFSICO

rv

R S
T

EXCITATRIZ PRINCIPAL DE C.A. ALTERNADOR

EXCITATRlZ PILOTO DE CA

Figura 5.4.

Sistema de excitacin electrnico.

Modernamente se emplea un sistema de excitacin sin escobillas. El esquema es similar al de la Figura 5.4 pero en el cual la excitatriz principal tiene una disposicin inversa a la que aqu se presenta. En este caso el devanado trifsico de la excitatriz est colocado en el rotor y su devanado inductor en el esttor. La salida de c.a. de la excitatriz se convierte en c.c. por medio de rectificadores montados en el eje y que alimentan directamente el rotor del alternador sin necesidad de anillos ni escobillas (rectificadores giratorios). La regulacin de tensin de los alternadores se realizaba en los albores de la electricidad por medio de restatos intercalados en el circuito de campo de la excitatriz principal, que eran maniobrados a mano; ms tarde se construyeron reguladores automticos (tipo Tirrill) que modificaban automticamente la resistencia del circuito de excitacin. Tambin se utilizaron mquinas especiales para la regulacin de tensin, como las amplidinas y rototroles, que eran esencialmente mquinas rotativas de c.c. amplificadoras, que disponan de un nmero de escobillas doble que el de las dinamos convencionales. En la dcada de 1940-1950 se utilizaron tambin amplificadores magnticos. Hoy da la regulacin de tensin es electrnica y se realiza por medio de tiristores o rectificadores controlados de silicio que permiten obtener una c.c. de amplitud variable sin que existan prdidas apreciables, a base de actuar sobre un electrodo auxiliar denominado puerta (gate). En los alternadores modernos empleados en los grupos electrgenos para suministro de energa elctrica a instalaciones aisladas se recurre a la autoexcitacin del alternador, que consiste en obtener la c.c. necesaria para los polos de la propia salida del generador, que luego se rectifica. El cebado de la mquina se realiza merced al magnetismo remanente existente en los polos. En cualquier caso, los generadores disponen de sistemas realimentados que permiten estabilizar la tensin de salida de la mquina, aunque sta trabaje con cargas variables, lo que da una gran autonoma y seguridad al servicio.

5.4. PRINCIPIO DE FUNCIONAMIENTO DE UN ALTERNADOR 5.4.1. Funcionamiento en vaco


Consideremos el esquema simplificado de la mquina sncrona de polos salientes mostrado en la Figura 5.5. Al girar el rotor a la velocidad n, se inducen f.e.m.s. en los arrollamientos de

388

MQUINAS ELCTRICAS

Figura 5.5. Mquina sncrona trifsica. Principio de funcionamiento.

las tres fases del esttor, que van desfasadas en el tiempo 120, que corresponden a la separacin espacial (en grados elctricos) existente entre las bobinas del esttor. Si consideramos que las N espiras de cada fase estn concentradas, y que el flujo concatenado por las mismas vara entre los lmites +<1>m Y -<1>m' el valor medio de la f.e.m. inducida en cada fase, en el transcurso de medio perodo de la corriente alterna, ser: E
med

=T o

21T!2

edt = -

T o

21T!2

edt = -

T o

21T/2 ( -N

d<1 dt dt

(5.2)

es decir:
Emed

2 N =- T

1-<1> d<1>= 4 fN<1>m


+<I>m

(5.3)

Como quiera que el valor eficaz de la f.e.m. es igual al valor medio multiplicado por el coeficiente de forma K de la onda producida, la f.e.m. eficaz E tendr una magnitud: (5.4) Teniendo adems en cuenta que las espiras de cada fase estn distribuidas sobre la periferia del esttor, pudiendo existir al mismo tiempo acortamientos en cada bobina, la f.e.m. resultante vendr afectada por los coeficientes de devanado analizados en el Captulo 2. En consecuencia, la expresin (5.4) se convertir en:
(5.5)

que es la ecuacin general de la f.e.m. inducida en un alternador y que coincide con la (2.112) ya demostrada en el Captulo 2, donde Kd representa el coeficiente de distribucin y Ka el coeficiente de acortamiento de los devanados del inducido. La variacin con el tiempo de la f.e.m. anterior debe ser senoidal; sin embargo, ha de tenerse en cuenta que la forma de la misma es una imagen del modelo de distribucin de la componente radial de la induccin B a lo largo del entrehierro, de acuerdo con la expresin e = Blv. En consecuencia, para obtener una f.e.m. senoidal en los devanados del esttor es necesaria una distribucin senoidal de la induccin a lo largo de la periferia del rotor. Si el entrehierro es de espesor constante, dicha distribucin debe considerarse ms bien de forma trapecial (Fig. 5.6a), es decir, constante debajo de cada polo y rpidamente decreciente hasta las lneas interpolares. Para conseguir que la distribucin de B sea senoidal, habr que aumentar el entrehierro desde el centro del polo hasta uno de sus extremos, ya que de esta forma

CAPTULO 5.

MQUINAS SNCRONAS

389

B~

+
~ ;

!,.J

a) Entrehierro uniforme

b) Entrehierro variable (senoidad)

Figura 5.6. Distribucin de induccin en el entrehierro. aumenta la reluctancia gradualmente, lo que conduce a una forma de B que se acerca ms a la senoidal. En general, no se llegar a la senoide perfecta y por tanto la f.e.m. resultante contendr armnicos. Estos armnicos se reducen por las leyes del devanado del inducido, por medio de la distribucin y acortamiento de las bobinas que lo componen, eomo ya se demostr en el epgrafe 2.9 del Captulo 2. En las mquinas con rotor cilndrico, la obtencin de una forma de B senoidal se logra por una adecuada distribucin del arrollamiento de excitacin a lo largo de la periferia del rotor. Una caracterstica importante del funcionamiento en vaco de la mquina sncrona, la constituye la curva Ea =f(IJ, que expresa la f.e.m. en bornes de la mquina estando desconectada la carga en funcin de la corriente de excitacin o, en definitiva, de los amperios-vuelta/polo que recorren los devanados inductores. La determinacin de esta curva se realiza por va experimental, para lo cual es necesario hacer girar la mquina a la velocidad de sincronismo por medio del motor primario: turbina, diesel, etc., y entonces se debe cambiar gradualmente la corriente de excitacin desde cero hasta un cierto valor mximo correspondiente a la saturacin y desde este ltimo nuevamente hasta cero. Los valores medidos de la f.e.m. Ea en funcin de la corriente de excitacin l. se representan mediante dos ramas, una ascendente y otra descendente, de la caracterstica. Sin embargo, esta divergencia es pequea y depende de la influencia de la histresis en los polos y en el yugo de la mquina. La caracterstica en vaco representa la curva media entre ambas ramas y tiene la forma indicada en la Figura 5.7.

Ea (voltios)

e(amperios)

Figura 5.7. Curva de vaco de un alternador.

390

MQUINAS ELCTRICAS

Se observa que la caracterstica anterior es similar a la curva de magnetizacin del hierro, conclusin lgica si se tiene en cuenta que la curva de vaco corresponde a la solucin experimental del circuito magntico de la mquina, en la que se ha sustituido la f.m.m. por una magnitud proporcional que es la corriente de excitacin y donde se ha intercambiado el flujo por la f.e.m. de acuerdo con la relacin (5.5).

5.4.2. Funcionamiento en carga. Reaccin del inducido


Si estando funcionando un alternador en vaco, con una determinada corriente de excitacin, se cierra el circuito del inducido conectando una impedancia de carga a sus terminales, se obtiene una tensin Ven bornes de la mquina inferior al valor que presentaba en vaco Eo' La reduccin en la tensin de salida del generador es debida a la aparicin de una corriente en el inducido que provoca una cada de tensin en este circuito a la par que produce una f.m.m. que reacciona con la del inductor modificando el flujo del entrehierro de la mquina. La cada de tensin en el circuito del inducido se debe a la impedancia que presentan los arrollamientos de este devanado. Por una parte existe la resistencia R del bobinado, que provoca una cada de tensin muy pequea (del 1 al2 por 100 en los grandes generadores) y que en la mayora de los casos se suele despreciar. Se debe considerar tambin la reactancia del inducido, que se debe al flujo de dispersin del esttor que no interacciona con el flujo del rotor. Este flujo es el que se desarrolla en las cabezas de las bobinas (Fig. 5.8) y dentro de las ranuras donde se sita este devanado. Este flujo de dispersin permite definir un coeficiente de autoinduccin L", que multiplicado por la pulsacin de la corriente, da lugar a la reactancia de dispersin del esttor:
(5.6)

En esta reactancia tiene lugar la cada de tensin inductiva de la mquina sncrona, cuya magnitud a plena carga puede alcanzar valores del 10-15 por lOOde la tensin asignada del inducido. Ms difcil de analizar es el efecto que provoca la f.m.m. del inducido sobre la f.m.m. del inductor, modificando el flujo del entrehierro de la mquina. Este fenmeno se conoce con el nombre de reaccin del inducido y sobre l tiene influencia tanto la magnitud como la fase de la corriente del esttor. Cuando se trata de alternadores trifsicos, las corrientes del inducido al circular por devanados desfasados 120 en el espacio producen una f.m.m. de reaccin que gira en el espacio a la velocidad de sincronismo, es decir, a la misma velocidad que el rotor. Esta f.m.m. influye sobre la f.m.m producida por el devanado inductor y

Flujo de dispersin de ranura

Figura 5.8. Flujos de dispersin en el devanado del inducido.

CAPTULO

5.

MQUINAS SNCRONAS

391

puede deformar, reducir o ampliar el campo de excitacin de la mquina, dependiendo de la fase de las corrientes que recorren el inducido. Para estudiar este fenmeno con ms detalle, considrese el esquema de la Figura 5.9a, en la que se muestra una parte del desarrollo de una mquina sncrona con polos salientes en el rotor, con un devanado inducido de doble capa (dos ramas de bobina en cada ranura). Si se prescinde de los armnicos, las distribuciones de f.m.m. podrn considerarse senoidales. Vamos a analizar la composicin de las f.m.m.s. del inductor y del inducido segn sea la carga resistiva, inductiva o capacitiva:

a) Carga resistiva
Si la carga es resistiva pura, el f.d.p. es la unidad, y si se prescinde de la impedancia del inducido se podr considerar que el desfase entre la f.e.m. y la corriente es (/J = O. Para calcular el sentido y magnitud de las f.e.m.s. inducidas en los conductores habr que aplicar la ley de Faraday en la forma: e = L (v x B), donde v indica un vector velocidad, contrario al sentido de giro del rotor e igual a su velocidad perifrica, lo cual es el resultado de tener en cuenta el movimiento relativo entre ambos circuitos (es como considerar una mquina sncrona de inductor fijo e inducido mvil). Las f.e.m.s. sern mximas cuando los lados de las espiras se encuentren exactamente frente a los centros de los polos, es decir, en el instante representado en la Figura 5.9a. Como el desfase entre la f.e.m, y la corriente es nulo, este instante coincide tambin con los valores mximos de la intensidad, cuyos sentidos sern entrantes al plano de la figura para los conductores situados en ese momento frente a un polo

al

.,.

~ ,....

l -_

b)

Figura 5.9. Reaccin de inducido con carga resistiva.

392

MQUINAS ELCTRICAS

norte. En la Figura 5.9b se muestra la composicin de las f.m.m.s. de ambos circuitos, que sera la misma que la de los flujos que resultaran en la situacin ideal de que el entrehierro fuera uniforme y se prescindiera de la saturacin magntica. Se observa que para una carga resistiva, la reaccin de inducido es transversal, es decir, est desplazada 90 de la f.m.m. del inductor, lo que conduce a una distribucin asimtrica de la f.m.m. resultante debajo de los polos, de tal forma que la induccin que aparece debajo de la arista posterior de la zapata polar aumenta, mientras que disminuye en la parte anterior respecto a los valores que la mquina presentaba en vaco, cuando no exista reaccin de inducido. b) Carga inductiva

Cuando la carga es inductiva pura, el desfase entre la f.e.m. y la corriente es de 90. En este caso, el mximo de las corrientes estar desplazado en el espacio respecto del mximo de las f.e.m.s. en un ngulo de 90 en sentido contrario al del giro del rotor. Como quiera que las f.e.m.s. son mximas cuando los lados de las espiras estn en el centro de los polos, las corrientes sern mximas en el instante representado en la Figura 5.lOa (es decir, cuando el polo norte del rotor se halle adelantado 90 elctricos respecto a la posicin de mxima f.e.m). En la Figura 5.l0b se muestra la composicin de f.m.m.s.; se observa que la f.m.m. de reaccin del inducido se opone a la f.m.m. del inductor, 10 que significa que una carga inductiva pura produce una reaccin antagonista o desmagnetizan te, que tiende a reducir la f.m.m. resultante, haciendo disminuir el flujo en el entrehierro, provocando en consecuencia una reduccin de la f.e.m. inducida.

'. --: ._ .j ._ ._.:'-!. ':: .


-,

.-' L..._
ri,f ; ;

,/,""'TI--------",
.....-;._ .
f ,-,

J ._._.~~~
..

_ ..

b)

-,

~.Ii~ .'.

-, _-......~ ...... ~,,'


0

; 180 ;,------.....
;

Em.rn. del inducido

Figura 5.10.

Reaccin de inducido con carga inductiva.

CAPTULO

5. MQUINAS SNCRONAS

393

e) Carga capacitiva
Cuando la carga es capacitiva pura, la corriente del esttor alcanzar su mximo, 90 elctricos antes de que el polo se enfrente a los conductores que forman las espiras del inducido, que es el momento en que es mxima la f.e.rn, La situacin es similar a la descrita en el caso anterior y se muestra en la Figura 5.11. Obsrvese que en esta ocasin se produce un refuerzo de la f.m.m, del inductor, lo que significa que las cargas capacitivas ayudan a la accin del campo de los polos provocando un efecto magnetizante sobre los mismos. Cuando las cargas no son puras, presentan un desfase comprendido entre -90 y +90 y resultan unas reacciones del inducido que tienen componentes transversales o de eje cuadratura debidas a las componentes activas de las corrientes y otras longitudinales o de eje directo que actan sobre los polos debidas a las componentes reactivas de las intensidades, y cuyo efecto es desmagnetizante o magnetizante dependiendo de si la componente es inductiva o capacitiva, respectivamente. De esta manera en las mquinas sncronas, tanto de polos salientes como de rotor cilndrico, la reaccin del inducido provoca una variacin de la f.m.m. resultante que acta sobre el circuito magntico y que hace variar a su vez la magnitud del flujo en el entrehierro y en consecuencia el valor de la f.e.m. que se obtiene en el inducido. Para obtener una f.e.m, en el esttor de forma senoidal es preciso que el flujo que produce la f.m.m. resultante sea tambin senoidal; para ello es necesario que las f.m.m.s. tanto del esttor como del rotor tengan tambin esta distribucin. Esta condicin se cumple con bastante precisin en las mquinas con rotor cilndrico pero no as en las que tienen polos salientes,
0

a)

/.

,.-.'.
. ..;.._---i i i i i
I

~
F.m.m. del inductor

_._._
,

,;.rs>. ~ ,,' U .......


'

i ; i i i

Figura 5.11.

Reaccin de inducido con carga capacitiva.

394

MQUINAS ELCTRICAS

ya que en stas la variacin de reluctancia del rotor provoca una reaccin del inducido que se aleja de la forma senoidal. Por ello debe reducirse la influencia de esta reaccin frente al campo inductor, lo que se consigue aumentando la reluctancia del circuito magntico de la mquina, para lo cual es preciso aumentar el entrehierro de la misma, aunque no excesivamente para no tener que elevar demasiado la f.m.m. que entonces sera necesaria en el inductor.

5.5. DIAGRAMA FASORIAL DE UN ALTERNADOR. REGULACIN DE TENSIN


El diagrama fasorial de un alternador determina de forma grfica la relacin existente entre la f.e.m. y la tensin en los diversos regmenes de funcionamiento de la mquina, estudiando al mismo tiempo la interaccin entre las f.m.m.s. de excitacin y de inducido que conducen a la f.m.m. resultante que origina el flujo en el entrehierro. En principio, para analizar este diagrama fasorial se va a considerar una mquina sncrona con entrehierro uniforme (rotor cilndrico), ya que entonces la reaccin del inducido no depende de la posicin del rotor por ser la reluctancia idntica en todas las posiciones. Se supondr tambin que la reactancia de dispersin X(J es constante, y que se pueden despreciar las prdidas por histresis en el hierro. Esta ltima condicin equivale a decir que la f.m.m. resultante est en fase con el flujo que ella produce. Consideremos una mquina sncrona funcionando en rgimen generador con una tensin por fase V, que lleva una corriente inductiva en el inducido con un desfase de cp grados. Para determinar la f.e.m. resultante habr que aadir a la tensin terminal las cadas de tensin producidas en la resistencia y reactancia de dispersin, tal como se muestra en el circuito de la Figura 5.l2a, resultando: E, = V + RI + j X"I
(5.7)

cuya composicin geomtrica se muestra en la Figura 5.12a, donde se ha tomado la tensin terminal como referencia en el eje real. El flujo que se necesita para producir la f.e.m. anterior estar adelantado 90 respecto al fasor Er, y si se prescinde de la histresis, la direccin del

X"

al

Figura 5.12. Circuito equivalente de una alternador. Diagrama fasorial,

CAPTULO

5.

MQUINAS SNCRONAS

395

flujo ser tambin la que corresponde a la f.m.m. de excitacin o inductora Fe y de reaccin de inducido F, de tal forma que se cumple: (5.8) * En consecuencia, la f.m.m. de excitacin se obtiene sumando vectorial mente F, y -F, y teniendo en cuenta que esta ltima va en fase con la corriente del inducido que la origina, resulta la composicin fasorial mostrada en la Figura 5.12b. Si con esta excitacin representada por la f.m.m. Fe se deja la mquina en vaco, al no existir reaccin de inducido, es decir, se tiene F = O, la f.m.m. Fe se convierte en excitacin resultante Fe = F, y' el flujo en el entrehierro aumenta colocndose en fase con Fe (Fig, 5.12b) y se determinar por medio de la curva de vaco de la mquina sncrona. El proceso anterior constituye el mtodo general para calcular la f.m.m. necesaria en la excitacin cuando la mquina suministra una corriente I a una tensin V determinada. Realmente el proceso fsico es inverso, ya que partiendo de la f.m.m. de excitacin y conociendo las caractersticas de la carga, se obtendr una V y una I de salida. En la Figura 5.13 se muestra un esquema ilustrativo de las relaciones funcionales que se producen y que incluyen un lazo de realimentacin (la salida interfiere con la entrada). Inicialmente se parte de una corriente de excitacin le que produce una f.m.m. Fe y al girar el rotor se genera una f.e.m. que al conectar. una carga en el inducido da lugar a una corriente de salida 1. Realmente esta corriente 1, mostrada en el esquema de la Figura 5.12h, representa la intensidad de una fase (por ejemplo, la fase R). Existen verdaderamente tres corrientes, IR' 1" Ip en el inducido de mdulo 1, que al estar desfasadas en el tiempo 1200 elctricos y recorrer devanados desfasados 1200 elctricos en el espacio dan lugar a una f.m.m. giratoria F de reaccin de inducido que gira en sincronismo con la f.m.m. Fe de la excitacin. Estas dos f.m.m.s. interaccionan (al girar a la misma velocidad) y dan lugar a una f.m.m. resultante F, que a travs de la curva de imanacin del hierro de la mquina, que es no lineal, produce un flujo final <1>" que es el que origina la f.e.m. resultante E, en el devanado del inducido (en cada fase). Debido despus a la existencia de la resistencia R y reactancia de dispersin Xa de las bobinas del esttor, se obtendr una tensin Valgo inferior a Er'

Figura 5.13.

Relaciones funcionales entre las diversas variables de una mquina sncrona.

* Estas f.rn.m.s. se calculan por polo. Si el inductor tiene N.. espiras/polo recorridas por una corriente 1,. la f.m.rn. F,. ser igual a N..l,. Sin embargo, la f.m.rn. mxima producida en el inducido por polo vendr expresada por la ecuacin (4.47), tomando m = 3 para un alternador trifsico.

396

MQUINAS ELCTRICAS

Se define la regulacin de tensin de una mquina sncrona al cociente:

e = _0 __

E - V V

100%

(5.9)

que expresa el cambio en la tensin de bornes del generador de vaco a plena carga para una determinada excitacin en los polos. Como se observa en la construccin fasorial de la Figura 5.l2b, la regulacin depende de la magnitud de la resistencia y reactancia de dispersin del inducido y tambin del valor de la f.m.m. de reaccin que produce este devanado, que como se ha demostrado en el epgrafe 5.4.2 est ntimamente ligada con el f.d.p. de la carga. Con cargas resistivas y sobre todo con cargas inductivas, debido al efecto desmagnetizante de la f.m.m. de inducido, resulta una disminucin de la tensin conforme crece la corriente de salida, lo que conduce a valores de la regulacin positivos. Para cargas capacitivas, al tener la f.m.m. del inducido un efecto magnetizante, la tensin en carga es superior a la de vaco, lo que conduce a un valor de la regulacin negativo. La curva que representa la tensin en bornes del alternador en funcin de la corriente de la carga se denomina caracterstica exterior. En la Figura 5.l4a se representan tres de estas curvas, que corresponden, respectivamente, a factores de potencia resistivo, inductivo y capacitivo. Se ha partido para su construccin de un valor de la excitacin tal, que se obtiene en vaco la tensin asignada en bornes. Se observa claramente que para cargas resistivas e inductivas la tensin va disminuyendo conforme crece la corriente de la carga. Para cargas capacitivas la tensin aumenta con la intensidad; sin embargo, para corrientes elevadas muy superiores a la asignada aparece tambin una reduccin de la tensin debido a que las cadas de tensin internas (en la resistencia y reactancia de dispersin del inducido) son muy superiores al efecto magnetizante que produce la reaccin de inducido. Todas las curvas cortan al eje de abscisas en un mismo punto lcc que corresponde a la corriente de cortocircuito permanente. Si se desea mantener constante la tensin en bornes de la mquina al variar la corriente del inducido, ser preciso variar de forma adecuada la correspondiente excitacin. Las curvas que muestran las relaciones entre las corrientes de excitacin y las corrientes de carga para una tensin de salida constante se indican en la Figura 5.14b para diversos factores de potencia (estos grficos se denominan curvas o caracterstica de regulacin). En estas curvas leO representa la corriente de excitacin necesaria para producir la tensin asignada en vaco. En

v v"
f.d.p. unidad f.d.p. capacitivo

a) V

= f(!}

en un alternador

b) le

g(I) de un alternador

Figura 5.14. Curvas caractersticas de un alternador para distintos tipos de cargas.

CAPTULO 5.

MQUINAS SNCRONAS

397

consecuencia con la Figura 5.l4b, se observa que con cargas inductivas se requieren excitaciones cada vez mayores a medida que crece la carga, y para con cargas capacitivas ocurre el fenmeno contrario, debido al efecto magnetizante de la f.m.m. del inducido. EJEMPW DE APLICACIN 5.1 Unalternador trifsico conectado en estrella de rotor cilndrico 3.000 kVA, 6.600 V, 50 Hz y 2 polos, tiene una resistencia del inducido de 0,07 a y una reactancia de dispersin de 0,7 alfase. La curva de vaco viene expresada por la ecuacin:
E=----

s.soos,

Fe + 9.300

donde E es la fe.m. por fase generada por la mquina y Fe es la fm.m. de excitacin del inductoren A. v/polo.Lafm.m. de reaccindel inducido a plena carga es de 13.000A.v/polo. Si el alternador funciona a plena carga con factor de potencia 0,8 inductivo, calcular: 1) F.e.m. resultante E; 2) F.m.m. total Fe necesaria en la excitacin. 3) Si manteniendofija la fm.m. anterior se desconecta la carga, cul ser el valor de lafe.m. Ea que aparecer en los terminales de la mquina? SOLUCIN 1. La tensin asignada por fase en bornes del alternador vale:
V=

6.600

j3

= 3.810,5 V

y la corriente de plena carga del alternador es: 1=

v' 3 . VL

r:;

3.000.000 v' 3 . 6.600

r;

= 262,4 A

Si se toma la tensin como referencia de fases, las expresiones fasoriales de tensin y corriente sern:

v = 3.810,5 LO

; 1= 262,4 L -36,87

ya que el arcos 0,8 = 36,87 y la carga es inductiva. Por consiguiente y de acuerdo con (5.7), la f.e.m. resultante E, valdr: Er = 3.810,5 LO + (0,07 + jO,7) 262,4 L -36,87 = 3.942,2 L 1,9 V que corresponde a un mdulo de la f.e.m. resultante de lnea de: 3.942,2

j3

= 6.828,15 V.
2. Para producir la f.e.m. anterior se requerir una f.m.m resultante F, determinada por la curva de vaco, es decir: 3.942,2 =
5.800Fr
I

r,: 9.300

= F, ~ 19.734 A.v/polo

398

MQUINAS ELCTRICAS

y como quiera que esta f.m.m. debe estar adelantada 90 respecto a E, (vase Fig. 5.llb), la expresin fasorial correspondiente ser: F, = 19.734 L 91,9 A.v/polo Ahora bien, segn el enunciado de la f.m.m. de reaccin de inducido a plena carga, vale 13.000A.v/polo y tiene que estar en fase con la corriente de inducido, por 10 que su expresin fasorial ser: F = 13.000 L -36,87 A.v/polo yen consecuenciala f.rn.rn, total necesaria en la excitacin, de acuerdo con (5.8), ser: Fe = F, - F = 19.734 L 91,9 - 13.000 L -36,87 = 29.660 L 112 es decir, Fe es igual a 29.660 A.v/polo. 3. Si dejando fija la f.m.m. anterior, se abre el inducido, la tensin simple que aparecer en bornes de la mquina se deducir de la curva de vaco: 5.800 . 29.660 Eo = 29.660 + 9.300 4.415,5 V

que corresponde a un valor compuesto: Ea = 4.415,5

J3 :::::: 7.648 V.

5.6. ANLISIS LINEAL DE LA MQUINA SNCRONA: EL CIRCUITO EQUIVALENTE 5.6.1. Generalidades


Como ya se ha explicado en el epgrafe anterior, para estudiar el comportamiento de la mquina sncrona es necesario tener en cuenta el efecto de la reaccin de inducido en estas mquinas, lo que requiere, segn se observa en la Figura 5.12, el uso simultneo de magnitudes elctricas: f.e.m., tensin y corriente, con magnitudes magnticas: f.m.m.s. y flujo. Este procedimiento de anlisis, que responde a la denominacin de mtodo general, reproduce fielmente los fenmenos fsicos implicados, pero tiene el inconveniente de que al manejar dos tipos de magnitudes no quede ms remedio que recurrir al uso de diagramas fasoriales como el de la Figura 5.11b para poder explicar las relaciones que intervienen entre las diferentes variables involucradas. Con objeto de simplificar la labor de anlisis de la mquina sncrona, nos interesa conseguir un circuito elctrico equivalente que refleje el comportamiento de aqulla de un modo simple y en el que intervengan parmetros que puedan determinarse fcilmente por ensayos indirectos que consuman poca energa. Otro aspecto a destacar en relacin con el diagrama fasorial de la Figura 5.12b es que los valores de F y X" son desconocidos a priori *, por 10 que resultara del todo inabordable poder predecir las magnitudes de las diversas variables que entran en juego. En el transcurso de la ingeniera elctrica se han ideado diversos mtodos para poder predeterminar el comportamiento de la mquina sncrona y en particular poder calcular la

* En realidad se pueden determinar a partir de los datos de diseo, pero este procedimiento da lugar solamente a aproximaciones muy burdas.

CAPTULO

5.

MQUINAS SNCRONAS

399

regulacin de tensin para diferentes cargas. Debe destacarse que la determinacin de la regulacin en las mquinas sncronas es de vital importancia para disear correctamente el equipo de regulacin automtica de tensin que controla la excitacin. Adems, la magnitud de la regulacin determina el valor de las corrientes de cortocircuito y en consecuencia el tamao y poder de corte que deben tener los disyuntores de maniobra del grupo. Los diferentes mtodos propuestos tienen en cuenta si la mquina es de polos salientes o de rotor cilndrico, si existe o no saturacin, etc. En este epgrafe y en el siguiente 5.7 se van a explicar los mtodos de anlisis de las mquinas sncronas con rotor cilndrico (polos lisos). Primeramente se va a realizar un estudio lineal de la mquina, que nos va a permitir obtener un circuito equivalente, y ms tarde se harn correcciones para tener en cuenta la no linealidad de la curva de imanacin. En el epgrafe 5.8 se har un estudio de las mquinas de polos salientes.

5.6.2. Mtodo de Behn-Eschenburg. Impedancia sncrona


Estemtodo, propuesto en la primera dcada del siglo xx por el ingeniero Behn-Eschenburg,de los talleres suizos Oerlikon, tuvo como objetivo inicial poder predeterminar la cada de tensin de un alternador. Se aplica a mquinas con rotor cilndrico que trabajan en rgimen lineal, lo que significa que los flujos son proporcionales a las f.m.m.s. y en consecuencia puede utilizarseel principio de superposicin.La ventaja de este mtodo es que permite obtener un circuito elctrico equivalente de la mquina sncrona, con las ventajas analticas que supone. Como ya se ha sealado en el epgrafe 5.5, se sabe que en realidad existe un flujo nico en el entrehierro de la mquina sncrona que, como se muestra en la Figura 5.12, es producido por la accin conjunta de las f.m.m.s. de excitacin Fe y de reaccin de inducido F, Sin embargo, resulta ms cmodo considerar que cada f.m.m. produce un flujo independiente que crea a su vez su correspondiente f.e.m. inducida. De esta forma se trabaja nicamente con f.e.m.s. y magnitudes elctricas, dejando a un lado las magnitudes magnticas. En la Figura 5.15 se representa esta idea. Obsrvese que estn implicados tres flujos, a saber:
a)

El flujo de dispersin <Po- que se obtiene en las cabezas de bobina, que est en fase con la corriente del inducido y que da lugar a una cada de tensin en la reactancia del mismo nombre X" y que ya se consider en la construccin del diagrama fasorial de la Figura 5.11, representada por un valor:
(5.10)

es decir, la cada de tensin producida por la reactancia de dispersin se adelanta 90 respecto a la corriente del inducido. b) El flujo de excitacin <Pe producido por la f.m.m. Fe y en fase con ella, que es el causante de la f.e.m. producida en vaco Eo. Esta f.e.m., de acuerdo con la ley de Faraday, estar retrasada 90 respecto al flujo <Pe. e) El flujo de reaccin de inducido <Pi producido por la f.m.m. F, y en fase con la corriente, lo que da lugar a una f.e.m. Ep retrasada 90 respecto de <Pi y por tanto de la corriente del inducido, que se puede expresar as: (5.11) El factor (-j) en la ecuacin anterior indica el retraso de 90 de Ep respecto de 1; el trmino Xp expresa el factor de proporcionalidad entre ambas magnitudes y que se denomina reactancia de reaccin de inducido.

400

MQUINAS ELCTRICAS

~ir L.F

Figura 5.15.

Relaciones funcionales entre las diversas variables de una mquina sncrona.

En la Figura 5.16 se muestra el nuevo diagrama fasorial que se obtiene y en el que se incluyen las f.e.m.s. que se acaban de definir. Se ha dibujado tambin la composicin fasorial de las f.m.m.s de un modo anlogo al que se hizo en la Figura 5.12b. De este modo el lector podr apreciar ms claramente el significado del esquema de la Figura 5.15. En definitiva, lo que se ha conseguido con este artificio es sustituir el tringulo rayado de la Figura 5.16 de f.m.m.s. por otro tringulo semejante de f.e.m.s. en virtud de la proporcionalidad entre ambas magnitudes a consecuencia de considerar el circuito magntico lineal. O de otro modo, la ecuacin original (5.8), que expresaba la resultante de f.m.m.s.: (5.12)

Figura 5.16.

Diagrama fasorial de tensiones y f.m.m.s. de un alternador.

CAPTULO

5. MQUINAS SNCRONAS

401

se ha sustituido por la ecuacin proporcional de f.e.m.s.: Er = Eo + Ep y teniendo en cuenta (5.11), la expresin anterior se puede escribir as: E,. = E - j XpI y sustituyendo el valor anterior de E, en la ecuacin general (5.7) resulta: E - jXpl = V + RI + jXaI que reordenando trminos conduce a la expresin: (5.16) lo que indica que la f.e.m. inducida en vaco Eo, debida a la f.m.m. de excitacin Fe' se puede considerarcomo la resultante de aadir a la tensin V en bornes de la mquina las cadas de tensin por resistencia: RI; en la reactancia de dispersin: j X,,J, y en la reactancia de reaccin de inducido: j Xpl. En la Figura 5.17 se muestra el circuito elctrico que resume el significado de las ecuaciones (5.13) a (5.16). El esquema final de la Figura 5.17b representa el circuito elctrico equivalente por fase de la mquina sncrona. Las dos reactancias en serie Xp + Xa dan lugar a la denominada reactanca sncrona Xs: (5.17) Estareactancia es una magnitud ficticia que representa en un nico trmino los efectos combinados de dispersin y de reaccin de inducido, y que para una mquina no saturada (rgimen lineal) tiene una magnitud constante. De este modo, tenindose en cuenta (5.17), se puede expresar (5.16) as: E (5.15) (5.14) (5.13)

= V + RI

+ jX)

= V + (R + jX)I

(5.18)

en el que la suma R + j X, se define como impedancia sncrona: Zs=R+jX, (5.19)

En definitiva, la Figura 5.17b representa un circuito equivalente tipo Thvenin, constituido por un generador de tensin E en serie con la impedancia sncrona de la mquina.

+
v

a)

b)

Figura 5.17.

Desarrollo del circuito equivalente de una mquina sncrona.

402

MQU/NAS ELCTRICAS

Para hacerse una idea de los valores comparativos de las magnitudes R, Xa y X, se acostumbra en ingeniera elctrica a definirlas en valores por unidad (p.u.) en funcin de una impedancia base tomada como referencia. Si se considera un alternador conectado en estrella con una tensin asignada simple Vn y una corriente asignada In' la impedancia base se define as: (5.20) y los valores por unidad de R, X" Y X, seran:
R(p.u.)

=-

Zb

X,,(p.u.) =

x,
Z;

X,(p.u.)

= Zb

X,

(5.21)

La ventaja de definir las magnitudes en valores p.u. es que resulta ms simple comparar las caractersticas de diferentes mquinas, ya que los valores reales de las impedancias de mquinas de potencias muy dispares son muy desiguales. Sin embargo, los valores p.u. varan en un margen ms estrecho. En una mquina sncrona se observa que el valor de la resistencia del inducido es generalmente del orden de 0,01 p.u., lo que significa que la cada de tensin en esta resistencia a la corriente asignada es del 1 por 100 de la tensin asignada. La reactancia de dispersin vara entre 0,1 y 0,2 p.u., mientras que la reactancia sncrona tiene una magnitud cercana a 1p.u. Teniendo en cuenta estos valores, se observa que la reactancia sncrona es del orden de cien veces la resistencia R del inducido, y es por esta razn por la cual se desprecia el valor de R en el circuito equivalente de la mquina sncrona de la Figura 5.17b; de este modo la impedancia sncrona se convierte en una reactancia sncrona, lo que es norma habitual en el estudio de sistemas elctricos de potencia.

5.6.3.

Caractersticas de vaco y cortocircuito de la mquina sncrona. Determinacin de la impedancia sncrona

Como se ha demostrado en el epgrafe 5.6.2, la red de la Figura 5.17b representa el circuito equivalente de la mquina sncrona. Para estudiar el comportamiento de esta mquina ser necesario determinar los parmetros que se incluyen en este circuito: Ea y Z,. Como en cualquier circuito equivalente de Thvenin, y de acuerdo con [a ecuacin general (5.18), el valor de Ea se podr obtener mediante un ensayo de vaco, ya que en estas circunstancias se cumplir:

Vaco:
1=0
=>

Eo = V (vaco)

(5.22)

es decir, la f.e.m. Ea es la tensin en los terminales de la mquina cuando es nula la corriente del inducido. El clculo de la impedancia sncrona requiere una prueba adicional, denominada ensayo de cortocircuito. En este caso la ecuacin (5.18) nos da:

Cortocircuito:
V=

=>

Ea

= (R + j X)

Icor/o

= Z)cor/o

(5.23)

de donde resulta el valor modular de la impedancia sncrona: (5.24)

CAPTULO

5.

MQUINAS SNCRONAS

403

es decir, la impedancia sncrona es el cociente entre la tensin y la corriente de cortocircuito; se advierte al lector que los valores de todas las magnitudes anteriores deben expresarse por fase. Debe tenerse en cuenta adems un aspecto adicional: que tanto E como Icorro dependen de la corriente de excitacin de la mquina y por consiguiente tambin su cociente. En la Figura 5.18 se muestra el esquema elctrico para realizar ambas pruebas. En el ensayo de vaco se mueve la mquina a la velocidad de sincronismo accionada por un motor primario externo y se deja abierto el interruptor D. Se alimenta el inductor por una fuente auxiliar regulable de c.c. y se van midiendo simultneamente la corriente de excitacin y la tensin de salida. (En el caso de la Figura 5.18 se est midiendo la tensin compuesta de vaco, por lo que el valor de E ser igual a la lectura del voltmetro dividida por )3.) Estas medidas dan lugar a la caracterstica de vaco: Ea =.f(l), que ya se represent en la Figura 5.7 y que se volver a dibujar ms adelante. En el ensayo de cortocircuito se mueve el alternador a la velocidad de sincronismo, se mantiene el rotor desexcitado (le = O)Yse cierra el interruptor D de la Figura 5.18. A continuacin se va elevando gradualmente la corriente de excitacin hasta que los ampermetros del inducido lleguen al 130 por 100 de la corriente asignada. (Tericamente los tres ampermetros deben sealar la misma corriente en virtud de la simetra de los devanados del inducido. En caso contrario, significa que existe una pequea asimetra en los devartados, por lo que la corriente de cortocircuito se tomar como la media aritmtica de las lecturas de los tres ampermetros.) La curva que representa lcc = qJ(l,.) se denomina caracterstica de cortocircuito y es prcticamente una lnea recta, debido a que en estas condiciones el circuito magntico no est saturado porque tanto la excitacin resultante como el flujo son de un valor bajo. No es difcil demostrar lo anterior si se tiene en cuenta el circuito equivalente de la mquina en estas condiciones. En la Figura 5.19a se representa este circuito, donde por simplicidad se ha considerado R = O.Recurdese que el flujo en el entrehierro es el que crea la f.e.m. resultante Er' por lo que analizando el posible valor de E, podemos tener una idea de cul es el valor del flujo. En el esquema de la Figura 5.19a, si la corriente ('orro es del orden de la asignada y como se ha

Fuente de c.c. E

's 2 'u ~ o

Interruptor Motor primario de accionamiento

Figura 5.18.

Esquema elctrico para la realizacin de los ensayos del alternador.

404

MQUINAS ELCTRICAS

Eo

Xicorto

e,

a)

lcorto

b)

Figura 5.19. Circuito equivalente en cortocircuito y diagrama fasorial correspondiente.

indicado anteriormente X,.es del orden de 1 p.u., entonces E ser del orden de la tensin asignada. Ahora bien, X" vara entre 0,1 y 0,2 p.u., por lo que E, tendr un valor comprendido entre ellO y el 20 por 100 de la tensin asignada y el flujo correspondiente tendr un valor de ese orden, trabajando por consiguiente la mquina en la zona lineal de la curva de imanacin. Otra forma ms grfica de comprobar lo anterior es construir el diagrama fasorial de la mquina tal como se muestra en la Figura 5.l9b. Si se parte de la Iconoen el eje imaginario negativo, la f.e.m. Eo estar situada entonces en el eje real y es del orden de la tensin asignada. La f.m.m. necesaria para crear Eo ser la Fe' que est adelantada 90 respecto a Eo' Como quiera que la f.m.m. de reaccin de inducido F va en fase con la corriente de cortocircuito, se comprueba en esta figura que el mdulo de la f.m.m. resultante F r es la diferencia aritmtica entre Fe y F y tiene un valor reducido; el flujo correspondiente ser entonces de escaso valor y es el que genera E, En definitiva, lo que sucede es que en cortocircuito y de acuerdo con el esquema de la Figura 5.l9b, y tal como se adelantaba en el epgrafe 5.4.2, al ser el circuito totalmente inductivo, la f.m.m. de reaccin de inducido F es totalmente desmagnetizante y se opone a la f.m.m. de excitacin Fe' lo que provoca una F, final muy baja. Debe destacarse tambin que una ventaja de estos ensayos: vaco y cortocircuito, es que consumen muy poca energa. Tngase en cuenta que en vaco, al no existir corriente de circulacin, el motor primario deber vencer nicamente las prdidas mecnicas (de rozamiento y ventilacin) y las del hierro de la mquina, mientras que en cortocircuito, al ser el flujo muy reducido, las prdidas en el hierro sern de pequeo valor y el motor primario tendr que vencer solamente las prdidas mecnicas y las del cobre del inducido (3R/2). Teniendo en cuenta el rendimiento de estas mquinas (98 al 99 por 100), el motor primario deber tener una potencia del orden del 2 por 100 de la potencia asignada del alternador a ensayar, lo que facilita la realizacin de estos ensayos en el laboratorio. En la Figura 5.20 se han representado simultneamente las caractersticas de vaco y de cortocircuito de la mquina sncrona. Obsrvese el efecto de saturacin de la curva de vaco y el carcter lineal de la caracterstica de cortocircuito (realmente esta recta tiende a saturarse para valores de la corriente de cortocircuito del orden de 1,2 a 1,3 veces la corriente asignada). Se ha dibujado tambin la denominada recta del entrehierro, que es una recta tangente a la curva de vaco y que coincide con ella en la parte lineal. Esta recta del entrehierro representara en cierto modo el comportamiento en vaco de la mquina si se prescindiera de la satura-

CAPTULO

5. MQUINAS SNCRONAS

405

EO

(por fase)

z,
Vnominal

1 nominal

O'

Figura 5.20.

Caractersticas de vaco y cortocircuito de un alternador. Clculo de la impedancia sncrona.

cindel hierro (en definitiva. si la nica reluctancia a considerar fuera la del entrehierro, y de ah procede su denominacin). De acuerdo con (5.24), la impedancia sncrona se obtiene como cociente entre la f.e.m. El' tomada de la curva de vaco, y de la corriente (orco' tomada de la caracterstica de cortocircuito,para cada valor de la corriente de excitacin, lo que da lugar a la curva de trazo discontinuo de la Figura 5.20. Obsrvese que Z, vara ampliamente debido a la saturacin de la mquina. Para excitaciones pequeas la impedancia sncrona Z, es constante, ya que la caracterstica de vaco coincide con la recta del entrehierro y da lugar a la denominada impedancia sncrona no saturada, cuyo valor sera en la Figura 5.20: Od Zs (no saturada) = O'e (5.25)

Como quiera que generalmente, para reducir el tamao de la mquina, se requiere que el circuito magntico trabaje en el codo de la curva de vaco (que es en definitiva la curva de imanacin),el comportamiento de la mquina, utilizando la impedancia sncrona anterior, diferiragrandemente del valor real en las condiciones normales de funcionamiento. Es por elloque debe utilizarse otro valor ms adecuado de la impedancia sncrona. Pero cul elegir que se adapte mejor a las condiciones reales? En las diferentes Normas e Instrucciones propuestaspor las comisiones electrotcnicas de los diferentes pases se acostumbra a tomar la denominadaimpedancia sncrona saturada (o ajustada), que consiste en partir de la tensin asignada Od, a la que corresponde una corriente de excitacin Oh y que producira una corrienteen el inducido 0'1, y de este modo se tiene: Od Z, (saturada) = Z.. =-

, 0'1

(5.26)

406

MQUINAS ELCTRICAS

Partiendo de este valor, la reactancia sncrona sera igual a: X, = JZ,2-R2 (5.27)

En realidad, y aun empleando el valor de la impedancia sncrona saturada (5.26), al obtener el valor de E mediante (5.18) resultara un valor de la f.e.m. muy superior al real, dando lugar a regulaciones (5.9) demasiado grandes (mtodo pesimista). Esto es debido a que la impedancia sncrona real es inferior a la de partida. Desde un punto de vista estricto, para resolver el problema correctamente deberan repetirse los clculos nuevamente partiendo de un valor de Z, que corresponda a una mayor excitacin, con reserva de verificar si el valor de En al cual se llega corresponde a la le de la cual se ha partido. Mientras no se diga lo contrario, en este texto se resuelven los problemas partiendo de (5.26) sin necesidad de realizar nuevas iteraciones de clculo. No obstante, en el ejemplo de aplicacin 5.2 el lector podr analizar un problema que utiliza un mtodo iterativo para determinar un mejor valor de Zs' Los ingenieros que se dedican al proyecto y construccin de alternadores utilizan con profusin un parmetro denominado relacin de cortocircuito SeR (Short-Circuit Ratio). Esta relacin se define como el cociente entre las corrientes de excitacin necesarias para producir la tensin asignada (o nominal) en vaco y la corriente asignada en cortocircuito. De acuerdo con la Figura 5.20, Oh es la corriente de excitacin que produce la tensin asignada en la curva de vaco, mientras que Oc es la corriente de excitacin correspondiente a la corriente asignada en la caracterstica de cortocircuito. Por consiguiente, la relacin o razn de cortocircuito ser:
Oh SCR=Oc

(5.28)

Vamos a ver que la SCR est ligada ntimamente con el valor de la impedancia sncrona saturada, pues obsrvese que segn (5.26) se tiene:
Od V" Z=-=s

0,/

0,/

(5.29)

que expresada en valores por unidad, tomando como impedancia base la definida en (5.20), (V" es la tensin asignada simple), resulta ser:

z, (p.u.) = Z
es decir:

Z,
b

Vn In O'g = 0,/' = 0,/

V
"

(5.30)

0,/

z, (p.u.)

O'g

(5.31)

y debido al carcter no lineal de la caracterstica de cortocircuito, se cumplir, teniendo en cuenta la Figura 5.20 y la expresin (5.28): Z, (p.u.)
0,/ Oh -=-=SRC O'g Oc

(5.32)

ecuacin que nos indica que la relacin de cortocircuito es la inversa de la impedancia sncrona saturada expresada en valores por unidad.

CAPTULO

5.

MQUINAS SNCRONAS

407

Sin embargo el factor SCR tiene un significado mucho ms profundo que el que aqu se seala, ya que representa en cierto modo una medida del tamao fsico de una mquina sncrona. Vemoslo: Segn (5.5), la f.e.m. generada en vaco por la mquina sncrona es proporcional al producto del nmero de espiras por fase del inducido por el flujo. Si en vaco se induce la tensin asignada se cumplir: (5.33) Ahora bien, el flujo en vaco es el cociente entre la f.m.m. de excitacin (proporcional al segmento Oh de la Figura 5.20) y la reluctancia del circuito magntico. Si se considera despreciable la reluctancia del hierro, la nica reluctancia existente es la del entrehierro, que segn la definicin (1.16) del Captulo I es proporcional al espesor del mismo, g, y por tanto se cumplir: (5.34) de donde se deduce que el segmento [Oh] vale:
[Oh] = K3g N

(5.35)

Ahorabien, el segmento [Oc] de la Figura 5.20 equivale a una f.m.m. de excitacin que segn la caracterstica lineal de cortocircuito ser proporcional a la f.m.m. del inducido para corriente asignada, es decir: (5.36) Por consiguiente, el factor SCR definido en (5.28), teniendo en cuenta (5.35) y (5.36) ser igual a:
Oh SCR=-=K6Oc
g

N2

(5.37)

es decir, la SCR es directamente proporcional al espesor del entrehierro e inversamente proporcional al cuadrado del nmero de espiras por fase del inducido. El factor SCR puede, en consecuencia, aumentarse a base de hacer mayor el entrehierro y reduciendoel nmero de espiras del esttor. Esta reduccin de espiras obliga, segn (5.33), a queaumente el flujo en el entrehierro. Si se fija el valor de la induccin magntica en un valor constantecorrespondiente al codo de la curva de imanacin del material magntico, el nico modode aumentar el flujo es aumentar la longitud axial L del inducido. As, si se dispone de un alternadorde rotor cilndrico y con un alto factor SCR, tendr un entrehierro grande, pocas espiras en el esttor y una longitud axial grande en comparacin con otras mquinas. En definitiva,una alto SCR incrementa el tamao, peso y coste de una mquina sncrona. Generalmente, las mquinas sncronas de gran potencia tienen un alto factor SCR en virtud de requerirun mayor entrehierro para dejar suficiente holgura mecnica para que gire el rotor. En cambio, las mquinas pequeas tienen un factor SCR menor. Los valores tpicos del SCR paraturbogeneradores (alternadores con rotor cilndrico) son de 0,5 a 0,8, mientras que para los generadores hidrulicos (alternadores de polos salientes) es del orden de 1,5.

408

MQUINAS ELCTRICAS

EJEMPLO DE APLICACIN 5.2 Un alternador trifsico conectado en estrella de 165 kVA, 950 V tiene una resistencia del inducido de 0,3 n. Una corriente de excitacin de 15 A produce una corriente en el inducido de 200 A en cortocircuito y una fe.m. de lnea de 780 V en circuito abierto. Calcular: a) impedancia y reactancia sncrona, b) regulacin a plena carga con fd.p. 0,8 inductivo. SOLUCIN
a)

La impedancia sncrona, al estar conectado el alternador en estrella, vale: Z = [780//3J s 200


= 225

'

b)

y la reactancia sncrona ser: X, = JZ} - R2 = J2,252 - 0,32 = 2,23 n. Para calcular la regulacin a plena carga habr que determinar previamente [segn (5.9)J la f.e.m. Eo, Y segn (5.18), se tiene:
E = V + (R + j X,)I

Ahora bien, la corriente de plena carga o asignada del alternador, de acuerdo con sus caractersticas, vale: /=
S 165.000 r; = r; = 100,3 A V 3 . V V 3 . 950

Si se toma la tensin simple como referencia, los valores fasoriales de tensin y corriente sern: 950 V = /3 LO = 450,33 LO ; 1= 100,3 L -36,87

de donde se deduce, aplicando (5.18), un valor de f.e.m.:

Eo = 450,33 LO + (0,3 + j2,23) 100,3 L -36,87

= 629,5 L

14,8

que corresponde a una magnitud de lnea de 629,5/3 = 1.090,33 V Ya una regulacin, segn (5.9):
.;

1.090,33 - 950 950

. 100 = 14,77 %

EJEMPLO DE APLICACIN 5.3 Un alternador trifsico conectado en estrella de /.000 kVA, /.500 V, ha dado los resultados mostrados en la tabla, en unos ensayos de vaco y cortocircuito (la tensin se mide entre dos fases). 500 150 10 900 300 20 1.250 1.500 1.680 1.820 1.920 2.000 2.060 2.100 450 30 600 40 750 50 60 70 80 90 100

CAPTULO 5.

MQUINAS SNCRONAS

409

La resistencia del inducido por fase es de 0, J 5 Q. Calcular: a) F.e.m. necesaria en vaco en el generador cuando trabaja a plena carga con f.d.p. 0,8 inductivo. b) Regulacin de tensin correspondiente. c) Corriente de excitacin necesaria en el inductor. NOTA: Comenzar el clculo con la impedancia sncrona correspondiente a la tensin asignada.Repetir el clculo con una magnitud de Z, que sea autoconsistente con la corriente de excitacin resultante.

SOLUCIN
a)

En la Figura 5.21 se han dibujado a escala las curvas de vaco, cortocircuito e impedancia sncrona. Para la tensin asignada de 1.500 V corresponde una le = 40 A Y para esta excitacin la impedancia sncrona obtenida como cociente entre la f.e.m. de vaco por fase y la corriente de cortocircuito es de 1,44 Q; por tanto, la reactancia sncrona correspondiente ser:

x, = jl,44
E (voltios)

0,152

1,43 Q

2.200 2.000
E

2,0 1,8
'" 1,6 1,4 I,2 1,O

1.800
1.600
Icc

.~
O)

1.400

e,

1.200 1.000 800


600 400 200 O Zs

~ ....."

!
750

6&1' 0,8
450 300 150
0,6

0,4

0,2

10

20

30

40

50 60 t,(amperios)

70

80

90

100

Figura 5.21.

410

MQUINAS ELCTRICAS

La corriente de plena carga del alternador es: S 1=--= 1.000.000 ~ 385 A

J3 . V

J3. 1.500

Si se toma la tensin simple del alternador como referencia, los valores fasoriales de la tensin y la corriente sern:

V = ---

1.500 ~3

L 0 = 866 L 0
.

1 = 385 L -36,87

y aplicando (5.18) se obtiene una f.e.m. Ea:

Eo = 866 LO + (0,15 + j 1,43) 385 L -36,87 ~ 1.307 L 18,08 V


que corresponde a una f.e.m. de lnea de 1.307 J3 = 2.264 V, que por extrapolacin de la curva de vaco de la Figura 5.21 corresponde a una corriente de excitacin de le ~ 130 A, para la cual Z, = 0,67 n, que es muy inferior al valor tomado como referencia. Si se parte, como segunda iteracin, de este valor de la impedancia sncrona que corresponde a una reactancia de 0,653 n, resulta una nueva f.e.m. Eo:

Ea = 866 L 0 + (0,15 + jO,653) 385 L -36,87 ~ 1.076 L 8,79 V


La f.e.m. anterior corresponde a un valor de lnea de 1.076J3 = 1.863,7 V. Para generar esta f.e.m. se requiere, segn la Figura 5.21, una corriente de excitacin de 63 A, para la cual la impedancia sncrona es de 1,1 n, que es superior al valor tomado como referencia. Deber efectuarse, en consecuencia, otro nuevo intento para hacer consistente el valor tomado como referencia de la impedancia, sncrona y el resultante de la excitacin correspondiente. El lector puede comprobar que elegida una Z, = 0,94 n, que corresponde a X, ~ 0,93 n, se obtiene una f.e.m. de lnea de 2.000 V, a la que corresponde le = 80 A, para la cual Z, = 0,94 n; es decir, coincide con la magnitud elegida y en consecuencia el resultado es vlido. La regulacin correspondiente ser: '100=3333% 1.500 ' La corriente de excitacin necesaria se obtiene de la curva de vaco para Ea = 2.000 V, que corresponde a le = 80 A.

b)

E - V 1'=--100= V

2.000 - 1.500

e)

5.7. ANLISIS NO LINEAL DE LA MQUINA SNCRONA: MTODO DE POTIER O DEL F.D.P. NULO. CLCULO DE LA REGULACIN
El mtodo Potier* se aplica a las mquinas sncronas de rotor cilndrico que trabajan en la zona de saturacin. En estas mquinas saturadas la aplicacin del mtodo de Behn-Eschen-

A. Potier: Sur la raction d'induit de alternateurs. Revue d'lectricit. July 28, 1900. pgs. 133-141.

CAPTULO 5.

MQUINAS SNCRONAS

411

burg conduce a errores apreciables, ya que las f.e.m.s. no son ahora proporcionales a las f.m.m.s. debido a la no linealidad de la zona del circuito magntico en que trabaja. El mtodo de Potier determina el valor de la cada en la reactancia de dispersin X"I y la f.m.m. que produce la reaccin de inducido, de tal forma que el clculo de la regulacin se basa en la construccin fasorial general explicada en el epgrafe 5.5. Para calcular la regulacin por el mtodo de Potier es preciso conocer la curva de vaco que representa la f.e.m. E en funcin de la f.m.m. de excitacin Fe' Yadems es necesario realizar un ensayo con carga inductiva pura, representando en un grfico la curva tensin de salidarespecto a la f.rn.m, de excitacin, para una corriente de inducido constante e igual a la intensidad asignada. En la Figura 5.22 se muestra el diagrama de conexiones de este ensayo con carga reactiva o de factor de potencia nulo. En el supuesto de despreciar la resistencia del inducido frente a su reactancia, el diagrama fasorialgeneral de la Figura 5.12b aplicado a la mquina sncrona cuando tiene carga inductiva (Fig. 5.22) da lugar a la construccin mostrada en la Figura 5.23a. Se observa que los fasores V, X) YE, estn en fase e igualmente las f.m.m.s Fe' F, Y -F; en consecuencia, se tiene las siguientes relaciones entre magnitudes: (5.38) Se deduce de la Figura 5.23 que la reaccin de inducido es contraria a la f.m.m, de excitacin, lo que era de esperar al ser la carga inductiva, ya que produce un efecto desmagnetizante. En la Figura 5.23b se ha representado la caracterstica reactiva junto con la de vaco. Se observa que las dos curvas son semejantes, lo cual es lgico, ya que en ambas el circuito magntico es el mismo. El desplazamiento entre ambas curvas se debe a la f.m.m, de reaccin de inducido y a la f.m.m. necesaria para compensar la cada de tensin por reactancia de dispersin. De la curva reactiva se deduce que cuando se tiene la tensin asignada en bornes V, la f.m.m. necesaria en la excitacin Fe es igual a OF; si MF indica la f.m.m. de reaccin de inducidoF, la f.m.m. resultante F" de acuerdo con la l ."ecuacin de (5.38), corresponder al

Fuente de e.e.

Inducido

Interruptor

Carga inductiva variable

Figura 5.22. Ensayo con carga inductiva pura (f.d.p, nulo) del altemador.

412

MQUINAS ELCTRICAS

v
-F F B'
1+-1_

_F..L_

-+l.,

A'

M
jI-~

.s:

F
1

--.j.

a) Diagrama fasorial

b) Curvas de vaco y con carga reactiva

Figura 5.23. Determinacin de la reactancia de Potier.

segmento OM. Esta f.m.m. induce una f.e.m. resultante E, = Me. Teniendo en cuenta la 2: relacin de (5.38) y la representacin de la Figura 5.23b, se deduce que la cada de tensin por reactancia de dispersin viene definida por el segmento CB. Si se supone constante la reactancia X" (independiente de la saturacin de la mquina) y la corriente de carga 1, se pueden obtener los dems puntos de la caracterstica reactiva, desplazando paralelamente el tringulo ABC, denominado tringulo de Potier, de tal forma que si el vrtice C recorre la curva de vaco, el vrtice A describir la curva reactiva. En particular, cuando la base del tringulo se apoya en el eje de abscisas se obtiene el punto A' de la curva de carga. Este punto tiene una gran importancia, ya que corresponde a la excitacin OA' necesaria para que circule la corriente de plena carga en cortocircuito, por lo que la magnitud OA' puede obtenerse de la caracterstica de cortocircuito. Esta propiedad geomtrica permite resolver el problema inverso, que es deducir la cada X"I y la f.m.m. F a la tensin asignada, sin necesidad de conocer la caracterstica completa de cortocircuito. En efecto, conociendo nicamente los puntos A y A' puede construirse el tringulo ABe. que permite determinar las dos magnitudes citadas. Si en la Figura 5.23b se traza en la ordenada correspondiente a la tensin asignada, desde el punto A, el segmento AD = A' O (determinado por la caracterstica de cortocircuito para el valor dado de la corriente I) y se traza la lnea DC paralela a la recta del entrehierro, entonces se obtiene el punto C de interseccin con la de vaco. De esta construccin se observa que:
(5.39)

Conocidas estas magnitudes, podr construirse el diagrama fasorial del alternador de forma anloga a la indicada en el epgrafe 5.5, para deducir posteriormente la regulacin de la mquina y la f.e.m. Eo' Desde el punto de vista riguroso, el segmento CB no da realmente la cada de tensin en la reactancia de dispersin, sino en una nueva reactancia denominada reactancia de Potier, que es algo superior a X". Esto se debe a que la curva de vaco que expresa en general la relacin E = (Fe) no es la misma que la que define la f.e.m. resultante

CAPTULO 5.

MQUINAS SNCRONAS

413

E, en funcin de F, Yque se han considerado idnticas. Esta divergencia se explica por el aumentodel flujo de dispersin del rotor a medida que aumenta la corriente de excitacin. La diferenciaentre la reactancia de dispersin y de Potier no es muy grande en las mquinas de rotor cilndrico, y este mtodo da resultados bastante correctos en los turboaltemadores. Inclusive,la experiencia demuestra que el empleo de este diagrama de Potier en mquinas con polossalientes da resultados bastante aceptables, si se efecta la construccin del tringulo de Potierpara valores de la excitacin correspondientes a 1,2-1,3 veces la tensin asignada. Por ello este mtodo que tericamente slo es exacto para las mquinas de rotor cilndrico, se aplica en la prctia tambin a las mquinas con polos salientes.

EJEMPLO DE APLICACIN

5,4

Un alternador trifsico conectado en estrella de 45 kVA, 220 V, ha dado los siguientes resultados en unos ensayos de vaco y f.d.p. nulo:

120 1,44
3,64

200 2,6
5,2

220 3
5,8

240
3,5

260 4,1
7,5

300
5,9

o
2,2

6,56

Determinar: a) reactancia de dispersin por fase; b) corriente de excitacin equivalente a la reaccin de inducido a plena carga; c) corriente de excitacin necesaria en el inductor para desarrollar la plena carga a la tensin asignada con f.d.p. 0,8 inductivo; d) regulacin de tensin. NOTA: Despreciar la resistencia del inducido.

SOLUCIN
a)

En la Figura 5.24 se muestran las curvas resultantes de los ensayos. Al construir el tringulo de Potier se obtienen los siguientes resultados: 260 - 220
X(JI=

J3

23,1 V=BC

; F=I=

1,7 A=AB

Como quiera que la corriente reactiva del ensayo fue la asignada:


1=

45.000 r; =118A v' 3220

la reactancia de dispersin ser:


X =-=0196Q
(J

23,1 118

'

b)

La f.m.m. de reaccin de inducido expresada en amperios del inductor, de acuerdo con la construccin de la Figura 5.24,' es: Fj = Ij = AB = 1,7A

414

MQUINAS ELCTRICAS

Eo(voltios)

360
Recta de entrehierro
VACO

320 280 240


Xa! C

v'wm

= 220

200

UD

AH
F

F.D.P. NULO

160 120 80 40

7 8 I,(amperios)

Figura 5.24. e)

Para calcular la excitacin necesaria en el inductor se ha de construir el diagrama fasorial del alternador. En la Figura 5.25 se muestra esta representacin, donde se ha tomado como referencia de fases la tensin asignada simple del alternador. Los fasores correspondientes sern: 127 LO 1 = 118 L -36,87

y en consecuencia, la f.e.rn. resultante por fase, segn (5.7), ser:

que corresponde a un mdulo de la f.e.m. de lnea 142,07)3 = 246,1 V. En la curva de vaco se puede medir la f.m.m. resultante que se necesita para producir E, y que corresponde a una corriente de excitacin Ir = 3,65 A. En la Figura 5.25 se muestra la composicin de f.m.m.s. necesaria para calcular la excitacin total que requiere el inductor. De acuerdo con el teorema del coseno, se podr escribir:
( =

Jt; + (2 -

2/J cos (90 + {3)

(1)

CAPTULO 5.

MQUINAS SNCRONAS

415

Figura 5.25. Ahora bien, teniendo en cuenta la composicin grfica de la Figura 5.25 y observando los valores fasoriales obtenidos, resulta:

a = 7,5
d)

tp

= 36,87

= f3

= a + cp = 44,4

Y al sustituir el ngulo anterior en (1) resulta: le = 5 A. La excitacin anterior produce una f.e.m. en vaco (Fig. 5.24) de 284 V, Y en consecuencia la regulacin tendr un valor igual a: 8= 284 - 220 220 . 100=29,1%

5.8. REGULACIN DE TENSIN EN LAS MQUINAS SNCRONAS DE POLOS SALIENTES. TEORA DE LAS DOS REACCIONES
Mientras que el entrehierro en las mquinas sncronas de rotor cilndrico es prcticamente de espesor constante, el de las mquinas de polos salientes es mucho mayor en el eje cuadratura o transversal (regin media entre polos o concretamente lnea interpolar). Debido a la diferencia de reluctancia entre ambos circuitos magnticos, la consideracin de una sola reaccin de inducido en el esttor conduce a resultados poco precisos sobre la regulacin de tensin en estas mquinas. El profesor A. Blondel, de la Escuela de Ingenieros de Caminos de Pars, propuso una teora para estudiar estas mquinas, que fue ampliada ms tarde R. Doherky y C. Nickle y luego por R. H. Park *. En todos estos estudios se consideraba que la reaccin

Vase A. Blondel: Complment a la thorie des alternateurs a deux reactions (Revue Gnrale d'lectricit, ao 1922,tomo 12); R. Doherty y C. Nickle: Synchronous Machines: Part I and 2. An Extension 01Blondel's two reactiontheory. (AlEE Trans., ao 1926).El artculo famoso de R. H. Park se cita en las referencias finales de este captulo.

416

MQUINAS ELCTRICAS

de inducido en los alternadores de polos salientes puede descomponerse en dos componentes, denominadas f.m.m. de reaccin en eje directo o longitudinal y f.m.m. de reaccin en eje cuadratura o transversal. De esta forma se independizan totalmente los dos circuitos magnticos obteniendo, regulaciones que se acercan ms a la realidad. En la Figura 5.26 se muestra un alternador con dos polos salientes y en el que slo se ha considerado una bobina en el esttor aa', justamente para indicar que el eje de la f.m.m. giratoria del inducido tiene la referencia del eje de esta bobina. La f.m.m. de reaccin del inducido F se ha descompuesto en dos valores: F d en el eje directo y F q en el eje cuadratura. Existen, por consiguiente, tres f.m.m.s. que interaccionan en la mquina, a saber: Fe' F d Y Fq' Si se tiene en cuenta la teora lineal de la mquina sncrona desarrollada en el epgrafe 5.6, resulta ms cmodo considerar que cada una de las f.m.m.s. anteriores produce un flujo independiente que crea a su vez su f.e.m. inducida. En la Figura 5.27 se muestran estas ideas, en un diagrama anlogo al de la Figura 5.13 pero teniendo en cuenta ahora que se producen los flujos <Dd y <Dq para crear las f.e.m.s. de reaccin de inducido de eje directo Epd y cuadratura E1Q' que de un modo anlogo a (5.11) tendrn unos valores: (5.40) en las que Id e Iq indican las componentes de las corrientes en los ejes directo y cuadratura. Debe tenerse en cuenta que la f.e.m. de vaco Eo producida por el inductor acta en el eje q, puesto que debe ir retrasada 90 respecto a la lnea de los polos. Si se tiene en cuenta que la resistencia del inducido es R y la reactancia de dispersin es Xrr' para una tensin en bornes V se deber cumplir una igualdad fasorial similar a la indicada en la expresin (5.16), pero que ahora deber estar afectada por las dos componentes de f.e.m. indicadas en (5.40), y as queda: (5.41) Si por simplicidad se considera despreciable la resistencia del inducido y se tiene en cuenta la ltima idntica de (5.40), la ecuacin (5.41) se transforma en: (5.42) Por analoga con la teora de Behn-Eschenburg, en la que se defina la reactancia sncrona (5.17), en este caso resultarn dos reactancias sncronas: una de eje directo Xsd o simplemente Xd y otra de eje cuadratura Xsq o simplemente Xq. De este modo (5.42) se transforma en: (5.43)

Figura 5.26. Alternadorcon polos salientes.

CAPTULO

5. MQUINAS SNCRONAS

417

Figura 5.27.

Relaciones funcionales en una mquina sncrona de polos salientes.

cuyo diagrama fasorial se muestra en la Figura 5.28, donde se ha tomado como referencia en el eje real el eje q, en el que debe quedar alineada la f.e.m. de vaco Eo' Se advierte al lector que la construccin aqu sealada no es tan evidente en su realizacin como a primera vista parece. Las variables conocidas son la tensin V, la corriente 1 y el desfase (/J. Pero los componentes Id e Iq de la corriente 1 del inducido dependen del ngulo t/J, que forma la corriente con la f.e.m. Eo, que an no ha sido obtenida. Es posible hacer frente a esta dificultad con un proceso de ensayo y error; es decir, comenzando con una direccin posible de Eo se construye el diagrama fasoral y se determina la direccin de Eo; si esta direccinno coincide con la de partida se debe repetir el proceso partiendo de otra alineacin y comprobandoel resultado. El proceso debe repetirse hasta que la direccin supuesta coincida con la resultante final. Afortunadamente, existe un procedimiento ms simple y elegante que hace innecesario el mtodo farragoso anterior. Observamos que si se despeja Iq de la ecuacin (5.40) y se sustituye en (5.43), resulta: (5.44) que reagrupando trminos es: (5.45)

b--..;...... .,...

Eo ...... ......;;,_._._._. Eje q (cuadratura)

Figura 5.28. Diagrama fasorial de un alternador de polos salientes.

418

MQUlNAS ELCTRlCAS

e ~\l \)\~ec

c\6~

'

......................

Figura 5.29. Construccin grfica para determinar la direccin de Eo.

En esta ecuacin se observa que el ltimo sumando se adelanta 90 respecto de Id y por consiguiente, de acuerdo con la Figura 5.28, ir en fase con Eo. De este modo la suma de los dos primeros trminos del 2. miembro de la ecuacin (5.45) deber estar en fase con Eo. Esta suma se puede calcular sin necesidad de conocer la alineacin de Eo. En la Figura 5.29 se muestra el proceso a seguir. Se ha tomado la tensin como referencia y se ha dibujado la corriente I con su desfase ({J. Al sumar los dos primeros trminos de (5.45) se obtendr la direccin de Eo Ycon ello podr completarse el grfico de un modo anlogo al de la Figura 5.28. Inclusive, no es necesario para la construccin del diagrama fasorial el clculo de las componentes Id e Iq. En la Figura 5.30 se ha repetido el grfico de la Figura 5.29 para las mismas condiciones de funcionamiento. El segmento ab representa el fasor j Xql y el afijo del mismo, que tiene su final en b, define la direccin de Eo' con lo que queda terminado el ngulo jo La lnea ab se contina entonces hasta el punto e, con la distancia ae = Xdl, o de otro modo, be = (Xci - Xq)/. Entonces se traza una perpendicular ed a la direccin de Eo' de este modo se cumple que bd = be sen 1jJ, es decir: (5.46)
e

Figura 5.30. Diagrama fasoral de un alternador de polos salientes.

CAPTULO

5.

MQUINAS SNCRONAS

419

Por consiguiente, y de acuerdo con (5.45), el punto d sealar el afijo de Eo' habindose completado el diagrama fasorial. Una vez calculado Eo, el valor de la regulacin de tensin del alternador vendr expresado por (5.9), es decir: 8=--100% V
E()- V

(5.47)

La determinacin prctica de Xd y Xq se realiza con un ensayo de deslizamiento (consultar el texto de G. S. Brosan y J. T. Hayden citado en las referencias).
EJEMPLO DE APLICACIN 5.5

Un generador de polos salientes conectado en estrella de 1.500 kVA, 6.600 V, 50 Hz, tiene una resistencia del inducido despreciable. Las reactancias sncronas por fase en los ejes directo y cuadratura son, respectivamente, 23,2 n y 14,5 n. Si la mquina suministra la corriente de plena carga con f.d.p, 0,8 inductivo, calcular: 1) F.e.m. Ea de lnea. 2) Corrientes Id e [,/.

SOLUCiN 1. La tensin simple del generador vale: V= y la corriente de plena carga es:
1=

6.600

j3

= 3.810,5

S
L

150.000

j3 . V J3 . 6.600

=1312A '

Tomando la tensin como referencia de fases, las expresiones fasoriales de tensin y corriente sern:

v = 3.810,5 LO v + jXql

; 1= 131,2 L -36,87

De acuerdo con la construccin de la Figura 5.29, resulta: = 3.810,5 LO + j 14,5131,2 L -36,87 = 5.180,5 L 17

Por consiguiente, el ngulo b de la Figura 5.29 ser igual a 17, siendo la distancia Ob de la Figura 5.30 igual a 5.180,5 V Yel ngulo jJ = <p + b = 36,87 + 17 ~ 54. En consecuencia, de acuerdo con (5.46) se tendr:
bd = (23,2 - 14,5) 131,2 sen 54 = 923,4

y, por tanto, la magnitud de f.e.m. E por fase ser: E = 5.180,5 + 923,4 ~ 6.104 V que corresponde a una f.e.m. de lnea de 10.572,4 V. 2. De acuerdo con la Figura 5.28, las corrientes Id e Iq valdrn:
Id = I sen jJ = 131,2 sen 54 = 106,14 A ; Iq = I cos jJ = 131,2 cos 54 = 77,12 A

420

MQUINAS ELCTRICAS

5.9. FUNCIONAMIENTO DE UN ALTERNADOR EN UNA RED AISLADA 5.9.1. Generalidades


El comportamiento de un generador sncrono bajo carga vara fuertemente dependiendo del factor de potencia de la carga y de si el generador funciona solo o en paralelo con otros alternadores. El primer paso en el estudio del generador sncrono ser el anlisis del comportamiento de la mquina funcionando de manera aislada. En los prximos epgrafes se estudiar la operacin en paralelo. En la Figura 5.31 se muestra el esquema simplificado de un alternador que funciona aisladamente y est alimentando una carga trifsica equilibrada. Se muestran dos controles importantes: por un lado el regulador de tensin, que se incorpora en la excitatriz y que al variar la corriente de campo del generador permite controlar la tensin de salida; por otro lado el motor primario que mueve el alternador (que en el caso de la Figura 5.31 se ha supuesto que es una turbina hidrulica), que lleva un regulador de velocidad que acta sobre la entrada de agua, permitiendo con ello controlar la velocidad del grupo y por consiguiente su frecuencia. Supngase en principio que la mquina sncrona se mueve a velocidad estrictamente constante, por tanto la frecuencia es un parmetro fijo. Al aumentar la carga, es decir, al incrementarse la potencia activa y reactiva tomada del generador, aumentar la corriente de 1 del inducido; como no se ha modificado el regulador de tensin, la corriente de excitacin ser constante, o de otro modo, la f.m.m. Fe permanecer invariable (vase epgrafe 5.5). Ahora bien, como aumenta la intensidad 1, aumenta la f.rn.m. de reaccin de inducido F, lo que da lugar a una f.m.m. resultante menor Fr' una f.e.m. E, menor y una tensin de salida ms baja. La explicacin anterior es ms simple si se considera el circuito equivalente de la mquina sncrona de la Figura 5.17b, que se ha vuelto a repetir en la Figura 5.32, supuesto una resistenENTRADA DELAGUA Subir n _ Mecanismo de cierre de la vena lquida

Subir tensin

t~Bajar tensi:

Lf+

-CSA~A DELAGUA

_1

TURBINA

Regulador de tensin Carga elctrica

Figura 5.31.

Funcionamiento de un alternador en una red aislada.

CAPTULO

5.

MQUINAS SNCRONAS

421

+
Carga

Figura 5.32. Circuito equivalente del sistema de red aislada.

cia del inducido despreciable. La ecuacin que regir el comportamiento elctrico de la mquina, segn (5.18), ser:

v = Eo -

}X)

(5.48)

Si la corriente de excitacin es constante, ser constante la f.e.m. Eo, de tal modo que al aumentar la corriente 1, aumentar la cada de tensin en la reactancia sncrona, lo que se traduce, segn (5.48), en una reduccin de la tensin de salida. En la Figura 5.13 se dibujaron las caractersticas externas de la mquina, y donde puede observarsela variacin de la tensin de salida conforme se modifica la corriente del inducido (para excitacin constante). Obsrvese que para cargas inductivas la cada de tensin es mucho ms fuerte que para cargas resistivas en virtud del efecto desmagnetizante de este tipo de cargas. Sin embargo, para cargas capacitivas se obtiene un aumento de la tensin de salida debido a su efecto magnetizante. En la Figura 5.33 se muestran estos hechos de un modo evidente. Ntese que en los tres diagramas se parte de la misma E Y la misma corriente 1 (misma cada en la reactancia sncrona).Para cargas resistivas (Fig. 5.33a) la reaccin del inducido, representada prcticamentepor la cada XJ, es casi transversal (E forma un ngulo cercano a 90 con} XsI); en el caso de cargas inductivas la reaccin es antagonista (Fig. 5.33b) y para cargas capacitivas la reaccines magnetizante (Fig 5.33c), lo que est de acuerdo con lo estudiado en el epgrafe 5.4.2. Obsrvese en la Figura 5.33 que la tensin menor se obtiene para cargas inductivas. Normalmente interesa que la tensin suministrada a una carga permanezca constante, aunque se modifique el consumo. Como quiera que la f.e.m. E es proporcional al flujo inductor,deber variarse la excitacin para cambiar el flujo y as adaptar la nueva f.e.m. para que se restablezca la tensin al valor asignado o nominal prefijado.

v
v
-jX, 1

a) Carga resistiva

b) Carga inductiva

e) Carga capacitiva

Figura 5.33. Diagramas fasoriales para diversos tipos de cargas. Funcionamiento en isla.

422

MQUINAS ELCTRICAS

Esta accin se realiza con el regulador de tensin de la Figura 5.31, que es simplemente un restato que regula la corriente de campo de la excitatriz, dando sta una tensin de salida que controlar la corriente de excitacin de los polos del alternador. En la Figura 5.14 se represent en un grfico la variacin de la corriente de excitacin en funcin de la corriente de carga para mantener la tensin en el valor nominal. En definitiva, en un alternador que trabaja en una red aislada se tiene: 1. Lafrecuencia depende enteramente de la velocidad del motor primario que mueve la mquina sncrona. 2. El.fd.p. del generador es el.fd.p. de la carga. 3. La tensin de salida depende de: a) la velocidad de giro [la .fe.m. depende, segn (5.5), de lafrecuencia y delflujo]; b) de la corriente de excitacin; c) de la corriente de inducido; d) del.fd.p. de la carga. La potencia suministrada por el generador es la que solicita la carga. Cuando la carga demanda ms potencia, el alternador reduce su velocidad; esta disminucin es detectada por el regulador de velocidad de la turbina (motor primario), que provoca una mayor apertura de entrada del agua, lo que hace aumentar la velocidad de giro del grupo hasta llegar al valor de consigna que impone la frecuencia de salida del generador. Cuando se ha alcanzado esta velocidad, la mquina sncrona entrega al consumidor la nueva carga solicitada.

5.9.2. Funcionamiento del regulador de velocidad


En un alternador, y en general en un sistema elctrico, la potencia demandada vara continuamente y es necesario disponer los mecanismos de regulacin necesarios para adaptar en todo momento la generacin al consumo. Cuando se produce un aumento en la potencia elctrica absorbida por los receptores, en tanto no se modifique la potencia mecnica suministrada por el motor primario (turbina) del alternador, la energa adicional requerida se extrae de la energa cintica almacenada en las masas giratorias del sistema, con lo cual la velocidad de giro del grupo experimentar un descenso que se refleja en la misma proporcin en la frecuencia del generador. Lo contrario sucede en el caso de una reduccin en el consumo de energa elctrica. Este mecanismo constituye, por tanto, un indicador del desequilibrio generacinconsumo que existe en un momento dado y puede ser utilizado como referencia para efectuar la regulacin correspondiente. En una central elctrica, en cada unidad generadora se controla la velocidad del grupo mediante el denominado regulador de velocidad de turbina, que recibe tambin el nombre de regulacin primaria. En la Figura 5.34 se muestra una representacin esquemtica de una central hidroelctrica que consta de un turbina hidrulica que mueve un alternador. El agua se almacena en un embalse mediante una presa para conseguir una energa potencial suficiente; este agua pasa a travs de una tubera forzada y llega a la turbina que mueve el alternador que genera electricidad y cuya tensin se eleva con ayuda de transformadores para enviar la energa elctrica a los consumidores a travs de un red de alta tensin. Se ha destacado en la Figura 5.34 el regulador de velocidad del grupo turbina-alternador por ser el objetivo de este epgrafe. El regulador de velocidad consta de los siguientes elementos:

a)

El pndulo taquimtrico

Que es el rgano sensible a la velocidad de rotacin de la mquina. Es el corazn del sistema generalmente, y como se muestra en la Figura 5.34 est constituido por dos rna-

CAPTULO

5.

MQUINAS SNCRONAS

423

sas m que giran con el rbol de la mquina, bien directamente o a travs de una caja de engranajes con una cierta demultiplicacin. Las masas van unidas a unos collares o anillos a travs de unas barras rgidas. Un collar es fijo (el superior de la Fig. 5.34) Yel otro es mvil y desliza sobre un eje que lleva un resorte r cuya tensin es regulable. Las masas m, al girar, tienden a alejarse (o separarse) por efecto de la fuerza centrfuga venciendo la accin antagonista del muelle o resorte. Como consecuencia del movimiento del anillo deslizante, se produce la actuacin de un servomotor hidrulico que provocar la apertura o cierre del agua de entrada a la turbina (o movimiento similar del distribuidor de la turbina en el caso 'de centrales hidrulicas), que controlar de este modo la potencia suministrada por la mquina. Aunque gran parte de las centrales existentes disponen de este tipo de pndulo (pndulo de Watt), hoy da se van incorporando sensores de velocidad no mecnicos, que miden las revoluciones del rbol del grupo turbina-generador indirectamente a travs de la frecuencia o por medio de tacmetros elctricos.

Estado inicial: AGM Estado intermedio: AG'M' Estado final: A'GM'

hplQl;lM~l:RJ(=Q.

BUS DEL ALTERNADOR

Figura 5.34. Esquema de una central hidroelctrica en el que se destaca el regulador de velocidad.

424

MQUINAS ELCTRICAS

b)

Amplificador hidrulico

Que comprende un distribuidor o vlvula piloto y un pistn principal que hace las veces del servomotor. La vlvula piloto est constituida por un cilindro con cinco orificios de acceso, dos de ellos estn en comunicacin con el recipiente de aceite que sirve para el accionamiento de la vlvula (en forma de drenaje o descarga), otro est en comunicacin con una bomba de aceite para dar presin al mismo y los otros dos estn unidos al servomotor. En el interior del cilindro existen dos mbolos o pistones movidos por el collar deslizante del pndulo a travs de un brazo de palanca LI' Si los pistones SI y S2,Semueven hacia arriba, el aceite a alta presin proveniente de la bomba sale por el orificio a de la vlvula piloto y penetra en la cmara superior del servomotor. Como quiera que entonces la cmara inferior est en comunicacin con el tubo de descarga (presin atmosfrica), se tendr una diferencia de presiones entre ambas cmaras que provocar un movimiento del mbolo S3 hacia abajo. El eje del servomotor acta directamente sobre un inyector que controla la entrada de agua a la turbina mediante la vlvula de aguja V. El movimiento de la vlvula piloto convierte una potencia de nivel bajo en otra de gran valor en el pistn principal del servomotor. Hay que tener en cuenta que la vlvula piloto es del tipo diferencial, de tal forma que las fuerzas que provoca la presin sobre los pistones S, y S2se equilibra en reposo y de este modo se requiere muy poco esfuerzo para cambiar su posicin; con una pequea fuerza aplicada en el nudo M se conseguira un desplazamiento de los pistones que hara actuar el mbolo S3del servomotor con la fuerza suficiente para cerrar o abrir el inyector de la turbina, venciendo la accin antagonista del agua que llega de la tubera forzada. De este modo, el conjunto vlvula piloto-servomotor constituye un buen amplificador hidrulico. e)

Palanca de unin

Representada por la varilla rgida AGM, cuya misin es hacer que se muevan los pistones de la vlvula piloto proporcionalmente al cambio de velocidad del eje turbina-alternador. La parte AG proporciona una realimentacin para el movimiento del inyector que controla la entrada de agua por medio de la vlvula de aguja V. Supngase la situacin de equilibrio inicial representada por la posicin de la varilla AGM. Existe un equilibrio entre el consumo de agua de la turbina, que se convierte en potencia mecnica de entrada al generador con la potencia elctrica suministrada por el mismo (se desprecian las prdidas). Si en estas condiciones se produce un aumento de la potencia elctrica demandada por la red a la cual se ha conectado el alternador, se producir instantneamente una reduccin de la velocidad del grupo. El collar M, bajo la accin del regulador de velocidad o pndulo taquimtrico, va descendiendo lentamente hasta M' arrastrando el vstago MA y la palanca del distribuidor GD. El conducto b se abre y el aceite empuja al mbolo S3 hacia arriba abriendo la vlvula de aguja V de admisin del agua y la velocidad va decreciendo cada vez ms lentamente hasta cesar finalmente su decaimiento. El punto A, que sigue los desplazamientos de la vlvula, llega a A', lleva la palanca AM a la posicin A' M' y el vstago GD sube (G se desplaza a G' y luego vuelve a G). Las fases sucesivas de equilibrio de la varilla rgida AGM son, por consiguiente, AGM-AG'M'-A'GM'. Habiendo cesado de decrecer la velocidad, sta es en este momento inferior a la velocidad inicial y el distribuidor contina actuando y abriendo la vlvula V hasta que el vstago GD, que sube al mismo tiempo que la vlvula, vuelve a su posicin primitiva G cortando la entrada de aceite por el conducto b. Pero en este momento, debido a la entrada de agua (el vstago del servomotor est en A'), la potencia de la turbina ser ligeramente superior a la de la red; la velocidad subir de nuevo y habr un cierre parcial de la vlvula V. La velocidad dejar de

CAPTULO 5.

MQUINAS SNCRONAS

425

subir con la igualdad de la potencia mecnica Pm en la potencia elctrica demandada Pe Yde este modo el distribuidor continuar actuando hasta que la palanca GD pase a su posicin primitiva, y as sucesivamente. Se llega as, despus de algunas oscilaciones, a una nueva posicin de la vlvula en la que se cumple el balance de potenciar Pm = Pe. (Se observa que las oscilaciones se producen siempre, pero el servomotor se adelanta siempre al retomo de los pistones SI y S2 del distribuidor a su posicin de equilibrio amortiguando las oscilaciones). Cuando el sistema alcanza finalmente una posicin de equilibrio, esta situacin ser diferente de la primera; el collar M estar en un punto intermedio entre M y M', lo que significa que la velocidad es inferior a la que se tena en la situacin anterior y el punto A se sita entre A y A'. El consumo de agua es superior al que era antes. A un aumento de la carga ha correspondido una ligera disminucin de velocidad. La regulacin efectuada por este regulador es imperfecta, pero es rpida y estable. Los puntos ACM estn alineados y existe, pues, una relacin entre el recorrido de la vlvula V y el recorrido del collar deslizante M del pndulo taquimtrico. La posicin del vstago de la vlvula determina el consumo de agua en la turbina, es decir, la potencia de la turbina, que coincide en definitiva con la potencia elctrica suministrada por el generador. La posicin del collar deslizante depende de la velocidad de la turbina. Se puede decir entonces que existe una relacin entre la potencia y la velocidad de la turbina. Esta relacin, que se traduce por una curva, lleva el nombre de estatismo y se ha representado en la Figura 5.35a. La curva de estatismo de un alternador nos da la variacin de su velocidad, en definitiva de su frecuencia, cuando pasa de la marcha en vaco a la plena carga, definida por su potencia asignada Pn En la Figura 5.35a se ha utilizado como eje de ordenadas la frecuencia en vez de la velocidad, ya que es una magnitud ms prctica para el ingeniero elctrico. Esta lnea es sensiblemente recta. El punto A corresponde a la marcha en vaco y la frecuencia del generador es fl. El punto B corresponde a la potencia mxima (asignada) del generador y la frecuencia es f2 La frecuencia asignada se toma como la frecuencia media:

fn

=fl+fl

(5.49)

y con ello se define el estatismo por el cociente: J=fl-f2


r

!"

(5.50)

p p
al Relacin frecuencia (velocidadj-potencia

b) Definicin de constante del regulador

Figura 5.35 Recta de estatismo de un alternador.

426

MQUINAS ELCTRICAS

El valor anterior suele expresarse en tanto por ciento y rara vez excede el 4 por 100. En la situacin del ejemplo tratado antes, inicialmente la mquina desarrollaba una potencia Pa correspondiente a un frecuencia fa; al aumentar la potencia demandada a un valor Pb > Pa, el grupo reduce ligeramente su velocidad, dando lugar a una frecuencia Jb < Ja' De la Figura 5.35b se obtienen las siguientes relaciones: (5.51)* donde K se denomina constante del regulador o tambin potencia regulaute y se mide en MW/Hz. Esta constante est relacionada con el estatismo, y teniendo en cuenta (5.50) por la relacin:

Pn_K- __

_n_

- J, - ;_ -

brin

(5.52)

la ecuacin (5.51) permite calcular la frecuencia de trabajo en funcin de la potencia suministrada por la mquina. Un regulador de velocidad con estatismo nulo se denomina regulador asttico, y la respuesta frecuencia-potencia correspondiente es una recta horizontal (vase Fig. 5.36). Esta caracterstica asttica representara un regulador que mantendra la velocidad (frecuencia) del generador constante para cualquier potencia suministrada por la mquina. Indudablemente, esta caracterstica sera ideal para un generador que actuara aisladamente; sin embargo, esta caracterstica crea una indeterminacin en el reparto de potencias cuando se tienen alternadores acoplados en paralelo, lo que resulta indeseable. Prcticamente ocurrira que una de las mquinas se hara cargo de toda la potencia mientras que la otra quedara descargada.

d) Motor de control de potencia


Es el ltimo componente que nos falta por describir de la Figura 5.34. Constituye la denominada regulacin secundaria de la potencia suministrada por al turbina manteniendo la frecuencia constante. Hemos visto que la velocidad de un grupo turbina-alternador, controlada por el pndulo taquimtrico, variaba en funcin de la potencia. Cuando la potencia aumenta, f (frecuencia)

P (potencia)

Figura 5.36.

Respuesta del regulador asttico.

* En los sistemas elctricos de potencia esta ecuacin se define de un modo incremental (!1P = K!1f, ya que no se conoce la ordenada de la recta de la Figura 5.33. Por ello slo se determinan los cambios !1P.

CAPTULO

5.

MQUINAS SNCRONAS

427

la velocidad disminuye, e inversamente, cuando la potencia disminuye, la velocidad aumenta. Estas variaciones estn representadas por la curva de estatismo del regulador (Fig. 5.35). A una potencia dada corresponde una frecuencia determinada. Un regulador montado sobre un generador alimentando una red aislada sera incapaz de mantener la frecuencia a un valor constante y las variaciones de frecuencia seran demasiado importantes para ser toleradas por el usuario. El regulador de velocidad efecta un control primario, que debe ser corregido por un dispositivo especial que devolver la frecuencia a su valor asignado o nominal; es, en definitiva, una correccin que constituye la regulacin secundaria. Veamos ahora cmo podemos obtener este resultado. Volviendo al ejemplo del pndulo taquimtrico, la regulacin secundaria se obtiene haciendo variar la posicin del punto A, es decir, modificando la longitud del vstago AF del servomotor. Sobre este vstago est dispuesto un pin o tornillo que se puede enroscar o desenroscar por la accin de un motor auxiliar de control de potencia (vase Fig. 5.34). En su movimiento el tornillo arrastra el extremo A de la palanca AM. Supongamos ahora que la potencia elctrica demandada por la red aumente y que no tenemos este motor de control que realiza la regulacin secundaria. Segn ya se ha indicado, la velocidad del grupo bajar, el consumo de agua aumentar y la varilla ACM ir a A'CM'. Si actuamos ahora sobre el motor de control de potencia haciendo roscar el tornillo para volver el punto A' a A, la palanca ACM toma su posicin inicial y al mismo tiempo la velocidad adquiere su valor nominal. Es necesario ver que al pasar el punto A' a A, se tiene que aumentar al mismo tiempo la admisin de agua en la turbina, lo que permite obtener el aumento de velocidad que buscamos. (La varilla pasa por la posicin intermedia A'C"M). Si trazamos en la Figura 5.37 la curva de estatismo del alternador, antes de la accin del motor de control y otra despus, notaremos que la curva de estatismo est desplazada paralelamente a ella para trasladarse de la primera posicin a la segunda. Inicialmente la mquina trabajaba con la curva de estatismo AB, el rgimen de funcionamiento estaba representado por el punto e suministrando la potencia P a la frecuencia asignada!". Al aumentar la potencia elctrica demandada a un nuevo valor P' > P, el nuevo punto de funcionamiento (sin actuar el dispositivo de regulacin secundaria) sera el D, que correspondera a la frecuencia 1'. Al actuar sobre el motor de control se consigue obtener una nueva curva de estatismo A'B' paralela a la original, de tal modo que el nuevo punto de equilibrio es el e, suministrando la mquina la nueva potencia P' a la frecuencia asignada!". En definitiva, la accin del motor de control permite mantener la frecuencia asignada de consigna para cualquier potencia suministrada por el alternador. Lo que en definitiva hace la regulacin secundaria es engaar al sistema creyendo que ha habido un cambio en la velocidad, haciendo actuar a la vlvula de admisin de agua.

A'

Recta final

P P P'

Figura 5.37.

Desplazamiento de las rectas de estatismo mediante el motor de control de potencia.

428

MQUINAS ELCTRICAS

EJEMPLO DE APLICACIN 5.6 Un alternador de 2,5 MW, 50 Hz, tiene un estatismo del 5 por 100. El generador tiene en vaco unafrecuencia de 51 Hz. 1) Cul ser lafrecuencia a la quefuncionar el alternador cuando alimenta una carga de 1.000 kW conf.d.p. 0,8 inductivo? 2) Si se aade ahora otra carga que consume 1.000 kW con f.d.p. 0,6 inductivo, cul ser la nueva frecuencia de trabajo? 3) Despus de conectar la segunda carga, que se debera hacer para restablecer la frecuencia de funcionamiento a 50 Hz? SOLUCIN 1. De acuerdo con (5.52), la constante del regulador vale:
K= 2,5

0,0550

1 MWlHz

y segn (5.51) resultar, para una potencia de 1 MW:

f = 51 - 1 . 1 = 50 Hz
2. La segunda carga representa una potencia activa de 600 kW (1.0000,6), que aadida a los 1.000 kW anteriores dan un total de 1,6 MW. La nueva frecuencia de trabajo ser:

f = 51 - 1,6 . 1 = 49,4 Hz
3 Para volver la frecuencia de funcionamiento a 50 Hz, se tendr que actuar sobre la regulacin secundaria del regulador de velocidad, de tal modo que el alternador d en vaco una frecuencia de 51,6 Hz (es decir, 50 + 1,6), Yde este modo se compensar la cada de frecuencia de 1,6 Hz que sufre la mquina cuando se alimentan los 1.600kW.

5.10. ACOPLAMIENTO DE UN ALTERNADOR A LA RED


En el mundo de hoyes muy rara la existencia de un alternador nico que de manera aislada alimente su propia carga; esta situacin slo se presenta en algunas aplicaciones tales como los grupos electrgenos. Es norma general que los alternadores se siten en centrales elctricas al lado de donde se encuentran las fuentes de energa primarias. Con objeto de aumentar el rendimiento y fiabilidad del sistema, las diferentes centrales estn conectadas entre s en paralelo, por medio de lneas de transporte y distribucin. La red as constituida representa un generador gigantesco en el que prcticamente la tensin y la frecuencia se mantienen constantes. Esto se debe a que sobre esta gran red, la introduccin de un nuevo generador no altera los parmetros bsicos anteriores, por representar una potencia muy reducida frente al conjunto total. Por ejemplo, en Espaa la potencia elctrica total instalada del conjunto del pas es de alrededor de 55.000 MW; sin embargo, la potencia unitaria mxima de los alternadores existentes no llega a los 1.000 MW, lo que representa un 2 por 100 sobre el total. En terminologa elctrica, se dice entonces que se dispone de una red de potencia infinita (tensin y frecuencia constantes) a la cual se conectan los diferentes generadores del pas. La conexin en paralelo de un alternador a la red implica una serie de operaciones complejas que constituyen la llamada sincronizacin de la mquina. Es evidente que para que tal conexin se realice sin ninguna perturbacin se hace necesario que el valor

CAPTULO

5.

MQUINAS SNCRONAS

429

instantneo de la tensin del generador tenga igual magnitud y fase que el valor instantneo de la tensin de la red. De esta exigencia se deducen las siguientes condiciones, necesarias para poder acoplar en paralelo un alternador a la red. l. 2. Las secuencias de fases de/ alternador v la red deben ser idnticas. La tensin de/ generador debe tener un valor efica; igual a la tensin de la red y sus fases deben coincidir. 3. Las frecuencias de ambas tensiones deben ser iguales.

Para verificar estas condiciones se emplean en la prctica unos aparatos denominados sincronoscopios, que en el caso ms simple estn formados por tres lmparas incandescentes. En la Figura 5.38 se muestra un circuito de este tipo. La operacin comienza arrancando la mquina por medio del motor primario (turbina. diesel, etc.) hasta una velocidad cercana a la de sincronismo: n = 60fl/). Se introduce entonces la excitacin en el inductor del alternador y se va elevando sta gradualmente hasta que la tensin en bornes del generador coincide con la tensin de la red. En estas condiciones. si la secuencia de las fases del generador y la red son idnticas y las tres lmparas del sincronoscopio se colocan formando los vrtices de un tringulo equiltero con las conexiones indicadas en la Figura 5.38, aparecer una rotacin en la iluminacin de las lmparas. cuyo brillo cambiar con el tiempo. Este efecto puede explicarse con ayuda de la Figura 5.39, en la que se muestran los tres fasores de la tensin de la red, VR, VT, y los del generador, VR" Vs" VI" uno que gira a la velocidad de la pulsacin de la red w y otro a la velocidad de la pulsacin del alternador co'. Dicho de otra manera, si se supone que permanecen fijos los fasores de la red, los fasores del generador girarn a una velocidad co' - w respecto a la red. En el instante sealado en la Figura 5.391a lmpara 1 estar casi apagada y la 3 ser la que brille ms. Conforme gira el sistema fasorial VR" Vs' VT" a la velocidad o' - ca > 0, el brillo

v,

..,

u
LlJ

o so-----~----~------c::

Lmpara 2 Voltmetrode cero

Lmpara l Voltmetro del generador

sncrono

Figura 5.38.

Sincronoscopio de lmparas para acoplar un alternador a la red.

430

MQUINAS ELCTRICAS

Red: Tensiones: VrV~-Vr; pulsacin: Generador: Tensiones: VR, -VI"-VI'; pulsacin:

(!J'-U)

(J)'

V,

Figura 5.39.

Fasores de tensiones de la red y el alternador, respectivamente.

de las lmparas se sucede en el sentido 1 - 2 - 3. Si la velocidad o/ - () es menor que cero, el sentido de la iluminacin se producir en el orden contrario: 1 - 3 - 2. Por consiguiente, la secuencia con la que se produce la mxima iluminacin de las lmparas indicar el modo de actuar sobre el motor primario; es decir, si por ejemplo la iluminacin rota en el sentido 1 - 2 - 3 indica que OJ' > OJ, esto es, la frecuencia del alternador es superior a la de la red y por tanto habr que reducir la velocidad de la mquina motriz (turbina) cerrando la admisin de agua. Cuanto ms cerca est (J) de co', ms lento ser el movimiento de esta sucesin. La igualdad de fase se cumplir cuando sea nula la tensin o iluminacin de la lmpara 1, que es la que est conectada entre bornes homlogos (R - R') Y que se detecta con ms precisin con la ayuda de un voltmetro de cero; en ese instante habr que cerrar el interruptor general y el alternador quedar acoplado a la red. Para asegurar la mayor igualdad entre ambas frecuencias, se debe procurar que el perodo de encendido y apagado de una lmpara sea superior como mnimo a 4 o 5 segundos. Es fcil calcular la d.d.p. existente en bornes de la lmpara l. Si se denomina VF la tensin simple de la red, que es igual a la tensin simple del generador, los valores de las tensiones simples de la fase R para la red y el generador sern, respectivamente: (5.53) Por consiguiente, la d.d.p. en bornes de la lmpara l valdr:
VRN VR'N

VRR

= J2V

[cos (JJt - cos (J)'t]

(5.54)

y teniendo en cuenta la identidad trigonomtrica: cos A - cos B = -2 sen -_ resultar:


vRR'

A+B

sen __

A-B

(5.55)

= -2

J 2V

(J)

sen ---

+ (J)' 2
f

W -

sen .. _--

(1/
f

(5.56)

CAPTULO

5.

MQUINAS SNCRONAS

431

Al valor de la semisuma de pulsaciones se le denomina pulsacin fundamental wo' y por tratarse en la prctica de magnitudes muy cercanas, se puede aceptar que es igual a la pulsacin de cualquiera de ellas:

wO=--2-

w+o/

~ w

(5.57)

A la semidiferencia de frecuencias se la conoce con el nombre de pulsacin de batido wh:


(J)b

w-w' = ---

(5.58)

por lo que (5.56) se transforma en: (5.59) En la Figura 5.40 se ha representado la forma de esta tensin; obsrvese que la onda resultante tiene un valor eficaz doble que el de fase (por lo que si la tensin fase-neutro de la red es de 220 Y, las lmparas se debern prever para 440 Y). La frecuencia de batido representa la frecuencia de la onda envolvente; cuanto menor sea la frecuencia 'de batido tanto mayor ser el perodo de encendido y apagado de las lmparas. Generalmente, la comprobacin de la igualdad en el sentido de sucesin de las fases (R, S, Ty R', S', T') no se hace en las centrales, ya que en stas est verificado durante el tiempo que dura su montaje e instalacin. En los ensayos de laboratorio es preciso realizar esta comprobacin. El lector puede deducir fcilmente que si en el esquema de la Figura 5.38 no estn enfrentados los bornes homlogos, no se obtiene una rotacin de la iluminacin de las lmparas, sino que todas a la vez o se encienden o se apagan simultneamente. En ese caso habr que intercambiar dos terminales del generador en su conexin al interruptor general. En la prctica, en los grandes alternadores se ha sustituido el sincronoscopio de lmparas por otro de aguja (Fig. 5.41) cuyo principio de funcionamiento es el del motor de induccin (con un devanado monofsico en el esttor y un devanado bifsico en el rotor). La posicin de la aguja muestra el ngulo de desfase entre las tensiones del generador y de la red. Cuando las frecuencias son iguales la aguja se para y cuando las frecuencias difieren la aguja gira en uno u otro sentido, dependiendo de si el generador va ms deprisa o ms despacio que la red. La

Figura 5.40.

Tensin detectada por la lmpara I (lmpara de cero) del esquema de la Figura 5.38.

432

MQUINAS ELCTRICAS

t(jJ
O
SINCRONOSCOPIO Figura 5.41. Aspecto externo de un sincronoscopio, velocidad de la aguja est en relacin directa con la diferencia de velocidades (frecuencias). Si co' > J, la aguja gira en sentido contrario a las agujas del reloj yen el cuadro del sincronoscopio se indica que es necesario frenar o retardar la velocidad del grupo actuando sobre el motor primario o mquina motriz que impulsa el generador. La igualdad de fases se determina por la posicin vertical superior de la aguja, que indica el momento en el que hay que proceder al acoplamiento a la red. En las modernas instalaciones se emplean unas columnas de sincronizacin (Fig. 5.42), compuestas por un brazo saliente y giratorio del cuadro general de la central y que tienen dos voltmetros (red y generador), dos frecuencmetros (red y generador) un voltmetro de cero y un sincronoscopio de aguja. En las centrales automticas o con telemando, el acoplamiento se hace automticamente con la ayuda de equipos electrnicos.

~ R

cJ

So----+----~----------------------+__.--R

~ T~--~~r_~--------------,__r-s
T

l
r-----~ ~--_.

R'

S'

T'
: =

Generador sncrono

Figura 5.42. Columna de sincronizacin para el acoplamiento de un alternador a la red.

CAPTULO

5.

MQUINAS

SNCRONAS

433

5.11. POTENCIA ACTIVA Y REACTIVA DESARROLLADA POR UNA MQUINA SNCRONA ACOPLADA A UNA RED DE POTENCIA INFINITA
Consideremos una mquina sncrona de rotor cilndrico no saturada en la que se puede despreciar la resistencia del inducido frente a la reactancia sncrona cuya magnitud se supone constante. El circuito elctrico equivalente por fase de la mquina, teniendo en cuenta el esquema de la Figura 5.17b, estar representado por una f.e.m. E en serie con una reactancia Xs' En la Figura 5.43a se muestra este circuito. Si la mquina suministra una corriente 1 con un desfase ({J inductivo a una red de potencia infinita de V voltios, de acuerdo con (5.18) se cumplir: Eo=V+jX) (5.60)

cuyo diagrama fasorial, tomando la tensin en bornes como referencia, se ha dibujado en la Figura 5.43b. La potencia compleja desarrollada por la mquina ser:

s = 3VI* = 3 VI cos (p + j 3 VI sen ({J = P + j Q

(5.61)

donde 1* expresa el conjugado de 1. Vamos a determinar la potencia anterior en funcin de la f.e.m. Eo' la tensin V y el ngulo b que forman ambas magnitudes. De (5.60) despejamos la corriente: E -V 1=___ (5.62)

jx,

pero segn la Figura 5.43b, las expresiones fasoriales de Ea YV son: Ea = Ea L b = Ea cos por lo que (5.62) se transforma en:
(E cos (j - V) + j Ea sen (j 1 = ---'--------'-jXs

s + j e; sen s ;

V = V L 0

(5.63)

(5.64)

x,

+
V LO

RED DE POTENCIA INFINITA

./

a) Circuito elctrico equivalente

b) Diagrama fasorial

I'~S
.....

Figura 5.43. Mquina sncrona conectada a una red de potencia infinita.

434

MQUINAS ELCTRICAS

que corresponde a un valor conjugado: 1* = (Eo cos (5 - V) - j Eo sen (j -jX, y llevando (5.64) y (5.65) a (5.61), resulta una expresin para la potencia compleja: (5.65)

S = 3V -----"------"-----jx,
que da lugar a:

(Eo cos (j - V) - j E sen (j

(5.66)

3EoV . ElIV cos S = -sen b + J 3 -"----X,

(j -

V2

(5.67)

X,

que de acuerdo con (5.61) corresponde a unos valores de potencia activa y reactiva suministrada por la mquina:

= ---x:- sen b = P',l'

3EoV

.
sen () ;

Q=

3 EoV cos (j - V2 X,

(5.68)

En la Figura 5.44 se ha dibujado la senoide que representa la potencia activa (5.68) en funcin del ngulo (5, que se denomina ngulo de potencia y tambin ngulo de carga. Segn (5.68), la potencia activa mxima vale. (5.69) Si j > O, la potencia activa desarrollada por la mquina es positiva y corresponde al funcionamiento como generador sncrono o alternador (o de otro modo, la mquina recibe potencia mecnica por el eje). Si < 0, la potencia activa es negativa, es decir, la mquina recibe potencia activa de la red y por ello trabaja como motor sncrono entregando potencia mecnica en el eje. (Conviene destacar que la Fig. 5.44 se ha representado con el convenio generador, tomando positiva la potencia activa cuando la mquina suministra energa elctrica a la red; cuando se utiliza la mquina sncrona como motor en los accionamientos elctricos, el convenio de signos de las potencias se toma al contrario del que seala la Fig. 5.44.) En definitiva, y si se tiene en cuenta el diagrama fasorial de la Figura 5.43b, cuando la maquip

Motor Figura 5.44. Potencia activa de una mquina sncrona en funcin del ngulo de carga. Convenio generador.

CAPTULO

5.

MQUINAS SNCRONAS

435

na sncrona trabaja como generador la f.e.m. E se adelanta a la tensin, mientras que cuando la mquina sncrona trabaja como motor la f.e.m. E se retrasa respecto de la tensin. En la Figura 5.45 se muestra una analoga mecnica de la mquina sncrona. Se trata de dos poleas, una interior (rotor) y otra exterior (esttor), unidas por un muelle. Ambas poleas giran a la misma velocidad w (que representa para la polea interior el movimiento mecnico del rotor y para la exterior la velocidad del campo giratorio). En la Figura 5.45a el muelle que une ambas poleas no trabaja, est sin tensar y las marcas A y A' situadas en la polea interior y exterior estn alineadas y no se transmite potencia de una polea a la otra. En el caso de la mquina sncrona, esto significa que los fasores E y V estn alineados, por lo que j = 0, y segn la l." ecuacin (5.68) la potencia activa producida es igual a cero. En el caso de la Figura 5.45b, la polea interior arrastra en su movimiento a la polea exterior; estando la marca A adelantada respecto de A', la potencia se transfiere de la polea interiora la exterior, es decir, del rotor al esttor, y la mquina sncrona trabaja como generador: Eo se adelanta a V. Al contrario, en la situacin de la Figura 5.45c, la polea exterior est arrastrandoa la interior en virtud de que la marca A' se adelanta a la A y se transfiere potencia del esttor al rotor, y la mquina sncrona recibe entonces energa por el esttor y mueve el rotor, es decir, trabaja como motor. En este caso el fasor V que representa el eje del campo giratorio del esttor arrastra el rotor, cuyo eje magntico se representa por el fasor Eo' Obsrvese tambin en (5.68) que la potencia mxima generada (o absorbida) por la mquina se produce para j = 90; este ngulo de potencia representa el lmite de estabilidad permanente de la mquina sncrona. En la realidad, estas mquinas suelen trabajar con un ngulode potencia inferior a 30. Otro aspecto a destacar en (5.68) es el valor de la potencia reactiva producida. Se observa que si se cumple que:

e; cos j>

(5.70)

la mquina sncrona entrega potencia reactiva inductiva a la red, o lo que es lo mismo: la mquinarecibe potencia capacitiva de la red. Se dice entonces que la mquina est sobreexcitada. En el caso de que se cumpla la desigualdad: E cos
j

<V

(5.71)

,/l'y
C)--+ a)

/~EO
.

Vaco

b)

V Generador

el Motor

~')""" ",'.
II~,.".
(j

~"'"

Eo

Figura 5.45. Analogamecnicadel funcionamiento de la mquinasncrona.

436

MQUlNAS ELCTRlCAS

entonces, segn (5.68), la potencia reactiva suministrada por el generador es negativa, es decir, capacitiva, o de un modo equivalente, la mquina recibe potencia inductiva de la red. Se dice entonces que el generador funciona subexcitado.

5.12. FUNCIONAMIENTO DE UNA MQUINA SNCRONA CONECTADA A UNA RED DE POTENCIA INFINITA
Cuando se conecta un alternador a una red de potencia infinita, pasa a formar parte de un sistema que comprende centenares de otros alternadores que alimentan entre todos a millones de cargas. A diferencia de un generador trabajando en una red aislada (epgrafe 5.9), en el que la carga est bien especificada, ahora es imposible saber la naturaleza de la carga (grande o pequea, resistiva o inductiva) conectada a los bornes de un alternador especfico. Qu es, por tanto, lo que determina la potencia suministrada por la mquina? Sabemos que disponemos en el grupo de dos controles: a) el sistema de regulacin de tensin del alternador que controla la corriente de excitacin y que en el caso del generador aislado se utilizaba para regular la tensin de salida, y b) el sistema de regulacin de velocidad del motor primario que se utilizaba en el generador aislado para controlar la frecuencia. Ahora bien, la red a la cual se ha conectado el alternador es de potencia infinita, lo cual indica, como ya se ha sealado, que la frecuencia y la tensin son constantes y estn impuestas por esta red. Es evidente por ello que el comportamiento de un generador en una red aislada va a diferir bastante de su funcionamiento en una red de potencia infinita, y vamos a estudiar a continuacin cmo afectan los controles anteriores al funcionamiento de la mquina.

5.12.1. Efecto de la variacin de excitacin


Consideremos el esquema equivalente de un alternador conectado a una red de potencia infinita tal como se muestra en la Figura 5.46a. Como ya se ha indicado en el epgrafe 5.10, para acoplar esta mquina a la red habr que producir una f.e.m. Eo de igual magnitud y fase que la tensin V de la red. sta ser, por consiguiente, la situacin en los instantes que siguen al acoplamiento. En la Figura 5.46b se muestran los fasores Ea y V, que son idnticos y por tanto no habr corriente de circulacin por el inducido del alternador. Aunque el alternador ha quedado conectado a la red, no suministra (ni recibe) potencia alguna: se dice entonces que trabaja en modo flotante.

RED DE POTENCIA INFINITA

Eo=V

a) Esquema elctrico

b) Diagrama fasorial

Figura 5.46.

Mquina sncrona conectada a una red de potencia infinita en el momento de la conexin (funcionamiento en modo flotante).

CAPTULO 5.

MQUINAS SNCRONAS

437

---+

XI

RED DE POTENCIA INFINITA

_Q<O
a) Esquema elctrico

~ r
J

E.=J'XI
J

.\

h) Diagrama fasorial

Figura 5.47.

Mquina sncrona sobreexcitada conectada a una red de potencia infinita.

Si ahora se aumenta la corriente de excitacin, aumentar la f.e.m. inducida Eo, que al ser superior a la tensin de la red provocar una corriente de circulacin por el inducido, que de acuerdo con (5.62) ser igual a: Eo- V l=--=E, jX
I

t,

(5.72)

y en la Figura 5.47b se muestra el resultado. Es decir, la corriente se retrasa respecto a la diferencia de tensin Ex un ngulo de 90 (debido a la reactancia sncrona). Esta corriente al ser inductiva pura produce un efecto desmagnetizante que tiende a reducir El)para oponerse al aumento de la excitacin. El alternador entrega a la red una potencia reactiva inductiva, o de otro modo, el generador ve la red como si fuera una inductancia. El alternador est sobreexcitado. Contrariamente a lo que podra parecer, la mquina no absorbe ni cede potencia activa: Ve 1 estn en cuadratura (vase Fig. 5.47a). Si se considera ahora el caso inverso, es decir, si se disminuye la corriente de excitacin de forma que EIJ se haga menor que V, aparecer entonces una corriente en el inducido, adelantada 90 respecto de la tensin (Fig. 5.48b), segn determina la ecuacin (5.72); esta corriente al ser capacitiva pura produce un efecto magnetizante que tiende a aumentar E para oponerse a la reduccin de excitacin. El alternador entrega a la red una potencia reactiva capacitiva, o de otro modo, el generador ve la red como si fuera un condensador. O al contrario, como se indica en la Figura 5.48a, el alternador absorbe potencia reactiva inductiva de la red. La mquina funciona subexcitada y no absorbe potencia activa.

_2_,.

XI

RED DE POTENCIA INFINITA

E,=jXsl

"Q>O
Figura 5.48.

a) Esquema elctrico

b) Diagrama fasorial

Mquina sncrona subexcitada conectada a una red de potencia infinita.

438

MQUINAS ELCTRICAS

Este rgimen de funcionamiento en el que la mquina sncrona no cede ni absorbe potencia activa sino slo potencia reactiva de uno u otro signo se conoce con el nombre de compensador sncrono. Este compensador, segn se ha demostrado, trabaja como condensador si est sobreexcitado y acta como bobina si funciona subexcitado. En resumen: la variacin de

la corriente de excitacin provoca un cambio en la potencia reactiva que entrega la mquina pero no afecta a la potencia activa que puede ceder o absorber.

5.12.2. Efectos de la variacin del par mecnico (regulador de velocidad)


Es evidente que la potencia activa suministrada por una mquina sncrona conectada a una red de potencia infinita procede de la potencia mecnica suministrada por la turbina. En rgimen permanente y considerando despreciables las prdidas mecnicas y elctricas del sistema, la potencia elctrica generada coincidir con la potencia mecnica suministrada por la turbina, la cual depende a su vez de la entrada de agua (o vapor en el caso de centrales termoelctricas) a la misma, que viene gobernada como sabemos por la posicin del regulador de velocidad. Si se considera como situacin de partida la mostrada en la Figura 5.46, en la que el alternador est en modo flotante, con E Y V iguales en mdulo y fase, de acuerdo con (5.72) no habr corriente en la mquina sncrona y por tanto la potencia elctrica suministrada por el generador ser igual a cero, lo que indica que la potencia mecnica de entrada ser nula (realmente la turbina recibir una pequea cantidad de agua para vencer las prdidas del grupo). Si en estas condiciones se abre la admisin de agua a la turbina (actuando sobre el regulador de velocidad), el rotor se acelerar como consecuencia del desequilibrio instantneo provocado entre la potencia mecnica y elctrica, y esto hace que la f.e.m. generada se adelante a la tensin de la red en un ngulo (j tal, que la potencia activa de salida se equilibre con la potencia de entrada mecnica. Aunque ambas tensiones tienen el mismo valor eficaz, el desfase entre las mismas determina una tensin Ex (Fig. 5.49b) que, de acuerdo con (5.72), provoca una corriente de circulacin 1 retrasada 90 respecto de aqulla. La potencia elctrica cedida por el generador a la red ser, de acuerdo con (5.68): p = _-

3EoV X,

sen (j

(5.73)

que es funcin del ngulo de potencia (j, lo que indica que si la excitacin permanece constante, es decir, se mantiene fija la f.e.m. Eo' a medida que aumenta la potencia activa, crece el

x,
RED DE

POTENCIA INFINITA

Er;;;;if\ _.
I

\ E,-JX,I
Ii

al Esquema elctrico

b) Diagrama fasoral

Figura 5.49. Efecto de la variacin de la potencia mecnica de entrada.

CAPTULO

5.

MQUINAS SNCRONAS

439

desfase (j entre Vy Eo' En definitiva,se puede decir que la variacin del regulador de velocidad de la turbina provoca un cambio en la potencia activa que entrega la mquina, que se ve reflejada fsicamente como una modificacin en el ngulo (j que forma la fe.m. Eo con la tensin V. Para una determinada excitacin, la potencia activa ser mxima para (j = n/2, que corresponde al lmite de capacidad de sobrecarga esttica o lmite de estabilidad esttica del alternador. Un aumento posterior en la entrada de la mquina motriz (turbina) hace que la potencia activa disminuya y el exceso de potencia se convierte en par de aceleracin que provoca un aumento de velocidad en el generador, salindose del sincronismo. Debe destacarse que al ser la red de potencia infinita, lafrecuencia es constante, lo que indica que la velocidad del alternador no puede variar. Un cambio en el regulador de velocidad modifica solamente el ngulo (j para equilibrar la potencia mecnica de entrada con la elctrica de salida. (Realmente existe un cambio de velocidad en el estado transitorio en el que se produce una variacin en el equilibrio de potencias). En el diagrama de la Figura 5.49b, si adems de variar la entrada de potencia mecnica (actuando sobre el regulador de velocidad), se hace variar tambin la corriente de excitacin, se modificar (segn se ha indicado en 5.12.1) la f.e.rn. E producida por el generador. En la Figura 5.50 se muestra el nuevo diagrama fasorial resultante, en el que se observa la modificacin de la magnitud y fase de la nueva corriente de circulacin. Si no se ha actuado en el regulador de velocidad, el ngulo (j de esta Figura 5.50 debe ser tal que se tendr que seguir cumpliendo la igualdad (5.73) para el nuevo valor alcanzado por Eo' Es interesante conocer el lugar geomtrico de los afijos de las f.e.rn.s. y corrientes que aparecenen la mquina sncrona al variar la excitacin y permanecer constante la entrada al motor primario, es decir, sin que vare la potencia que la mquina entrega a la red. Obsrvese en la Figura 5.50 que si P es constante, de acuerdo con (5.61) ser constante la componente activa de la intensidad:
P = 3 VI cos r.p = 3 V [OA] = K, [OA]

(5.74)

Es decir, al variar la excitacin de la mquina a potencia activa constante, el lugar geomtrico del afijo del fasor 1 se desplaza segn una recta perpendicular a la tensin V. Por otra parte, si se tiene en cuenta la ecuacin (5.68), al ser P constante, se mantendr invariable la proyeccin de la f.e.m. El)sobre un eje perpendicular a V:
P = 3 - E sen

X,

(j

=3-

X,

[BC] = K2 [BC]

(5.75)

m me]. C

Figura 5.50. Diagrama fasorial de la mquina sncrona para estudiar los lugares geomtricos de E e l.

440

MQUINAS ELCTRICAS

Lo anterior indica que al variar la excitacin y ser P constante, el extremo del fasor Eo se desplazara lo largode una recta paralela a V a una distancia E sen b o su equivalenteX,I cos ({J. En la Figura 5.51 se muestra esta composicin geomtrica para cuatro valores de la excitacin. Para el primer valor se obtiene la f.e.m. EO]' que corresponde a una corriente /1 suministrada por el generador a la red (que tiene carcter inductivo por estar en retraso con la tensin). Si se va reduciendo la excitacin, disminuye el valor de la f.e.m., el afijo de la corriente se desplaza sobre la recta, reducindose la componente reactiva de la intensidad, que pasar por un valor nulo correspondiente a la corriente /2 y a la f.e.m. Eo2' Hasta este punto la mquina ha funcionado en rgimen de sobreexcitacin, ya que se est cumpliendo la desigualdad (5.70). Si se reduce an ms la excitacin se invertir el carcter reactivo de la corriente. En la Figura 5.51 se puede observar este efecto para las f.e.m.s. E03 Y Eo4' que corresponden a las corrientes /3 e /4' que tienen para el generador carcter capacitivo. En estas condiciones la mquina est subexcitada. La f.e.m. E04 forma 90 con la tensin y representa el caso lmite que puede conseguirse, pues a partir de ese momento la mquina se desestabiliza. En consecuencia, la magnitud E04 representa la mnima f.e.m. a la que se puede llegar reduciendo la excitacin (para el valor de la potencia mecnica que se ha mantenido constante). Conviene que observe el lector que en todas las situaciones contempladas en la Figura 5.51 las distintas f.e.m.s. se adelantan a las tensiones, lo que ya se advirti con la analoga mecnica de la Figura 5.45. Si se va reduciendo progresivamente la potencia mecnica aplicada al rbol del alternador desde la mquina motriz (lo que se consigue en las turbinas hidrulicas cerrando la admisin de agua), el ngulo b formado por E Y V comienza a disminuir, reducindose al mismo tiempo la potencia activa suministrada por el generador. En el caso ideal de despreciar las prdidas se llegara a la situacin del compensador sncrono (Figs. 5.47 y 5.48), para el cual p = O,es decir, 6 = O;en la prctica Eo Y V formarn un ngulo muy pequeo necesario para
REGIN INESTABLE

SUBEXCITACIN

SOBREEXCITACIN

'O:,.J U:~

-:...::
...:::0,;

ero ~!Z ~:

Figura 5.51.

Lugares geomtricos de E e I en rgimen de subexcitacin y sobreexcitacin.

CAPTULO

5.

MQUINAS SNCRONAS

441

que el motor primario cubra las prdidas del funcionamiento en vaco del generador. Si se desconecta entonces el motor primario del rbol de la mquina sncrona, el ngulo (> se har negativo, ya que entonces el rotor comienza a retrasarse respecto del flujo giratorio del esttor, pero la mquina continuar girando sincrnicamente arrastrada por este campo rotativo. En estas condiciones la mquina comienza a funcionar como motor sncrono transformando la energa elctrica absorbida de la red en energa mecnica en el eje. Si se va aumentando el par en el eje, aumenta la potencia absorbida por la mquina y en consecuencia el rotor se retrasa ms an del esttor, el lmite de estabilidad se obtiene cuando (> = -90; en ese momento la mquina pierde el sincronismo, ya que la potencia mecnica se hace superior a la que absorbe elctricamente de la red. Generalmente se disean los motores de tal forma que para la potencia asignada el ngulo de carga est comprendido entre -20 y -30 elctricos. Si la potencia absorbida por el motor es constante, o en trminos equivalentes, si el motor mueve una carga de par constante, al variar la excitacin se obtiene un lugar geomtrico de las f.e.m.s. generadas y corrientes tal como se muestra en la Figura 5.52. Se han dibujado cuatro situaciones distintas. En el caso primero la f.e.m. es EOI Y la corriente 1I forma un ngulo ({J con la tensin, superior a 90, de este modo la potencia compleja entregada por la mquina sncrona a la red, segn (5.61), tendr una parte real negativa,es decir, la mquina recibe potencia activa de la red y una parte imaginaria positiva, lo que significa que entrega potencia reactiva inductiva a la red, o de otro modo, que recibe potencia capacitiva de la red. La mquina est sobreexcitada. Al reducir la excitacin, se reducela componente reactiva de la intensidad, que pasa por un valor nulo cuando la corriente es 12 (que corresponde a la f.e.m. Eo2)' Ysi la reduccin aumenta, la componente reactiva de la corriente cambia de signo: ste es el caso de las corrientes 13 e 14' En este ltimo caso la f.e.m. E4 forma un ngulo (> = -90 con la tensin y representa la situacin lmite, pues a partir de ese momento una nueva reduccin de la excitacin provocar la desestabilizacin del motor. Si se dibuja en un grfico la corriente del inducido (ordenadas) en funcin de la corrientede excitacin (abcisas) a potencia constante se obtienen unas curvas en forma de V, que se denominan curvas en V de Mordey del motor sncrono. En la parte inferior de la Figura 5.52 se muestra con lnea continua unas curvas de este tipo para una determinada potenciade entrada. Al variar sta se obtiene un haz de curvas en V tal como se seala con las lneas de trazo discontinuo. En todas las situaciones anteriores las f.e.m.s. se retrasan respecto a la tensin V de la red, lo que refleja el comportamiento de la mquina como motor (vase analoga mecnica en la Fig. 5.45), Y la mquina entrega a la red una potencia elctrica negativa o recibe de la red una potencia elctrica que se convierte en mecnica. Esta dualidad de expresiones entrega/recibe que se van subrayando en el texto suele ser motivode confusin. Para evitarlo el ingeniero prefiere utilizar el verbo entregar: cuando la mquinatrabaja como motor, lo que est ms de acuerdo con la transferencia de energa real quese produce en el funcionamiento de la mquina. En la Figura 5.53 se muestran dos esquemas que darn mayor claridad a la explicacin. En el caso de la figura superior la mquina trabaja como generador y se cumplir la siguiente relacin de tensiones: Eo=V+jX) La mquina entregar a la red una potencia activa positiva:
P = 3 VI cos
({J

(5.76)

= (+)

(5.77)

En la situacin de la figura inferior, que corresponde a un comportamiento como motor, la corrienteIg que entrega la mquina a la red forma un ngulo con la tensin superior a 90, Y

442

MQUINAS ELCTRiCAS

SUBEXCITACIN

SOBREEXCITACIN

iV
o~

~ ,~ = E .s
.~ o t: '" o

""u ... ~
,3"0
""

'"

.....
<!)

Figura 5.52. Curvas en V de Mordey.

por ello dar lugar a una potencia activa entregada negativa. Las ecuaciones (5.76) y (5.77) sern ahora: (5.78) En la parte derecha de la figura se muestra el diagrama fasorial de tensiones. Ahora bien, la misma situacin se puede definir con una mayor sencillez utilizando una corriente absorbida por el motor Tm, que es igual y opuesta a la que produca como generador IR. En estas condiciones las ecuaciones equivalentes a (5.78) sern: Ea = V - j XJm

= V = E + j X)m
<fJm

(5.79) (5.80)

P", = 3 VI", cos

= (+)

En el estudio del motor sncrono que se har ms adelante se utilizarn las ecuaciones (5.79) y (5.80) en vez de (5.78), lo que da mayor claridad al significado fsico de las ecuaciones.

CAPTULO 5.

MQUINAS SNCRONAS

443

REDDE POTENCIA INFINITA

a) GENERADOR

x,

+
V RED DE POTENCIA INFINITA

b)MOTOR

Figura 5.53.

Comportamiento de la mquina sncrona como generador y como motor. Diagramas fasoriales.

EJEMPLO DE APLICACIN

5.7

Un alternador trifsico conectado en estrella tiene una impedancia sncrona de valor 0+ j 10 O/fase y est acoplado a una red de potencia infinita de 11 kV de tensin compuesta. Para una determinada excitacin, la mquina entrega a la red una corriente de 250 A con fd.p. unidad. Posteriormente, y manteniendo la potencia activa constante, se eleva la excitacin hasta que la corriente de carga es de 300 A. Calcular: a) F.d.p. cuando suministra 300 A. b) Ee.m.s. y ngulos de carga en ambas situaciones. c) Potencias activas y reactivas suministradas. SOLUCIN
a)

La mquina entrega a la red inicialmente una potencia activa (cos P

(/J

= 1):

J3 . 11.000 2501

~ 4.763 kW

Como la potencia se mantiene constante, cuando el alternador entrega 300 A a la red se cumplir: P = 4.763 kW = de donde se deduce:
COS (/J2

J3 . 11.000 . 300
=:>

COS (/J2

= 0,833

(/J2

= 33,55 inductivo

444

MQUINAS

ELCTRICAS

b)

Si se toma como referencia de fases la tensin simple de la red se cumplir: V

11.000 r; LO
",,3

= 6.351 LO

; II = 250 LO

; Iz = 300 L -33,55

y al aplicar (5.78) se obtiene:


EOI

11.000

J-3 J3

LO

+l 10 . 250 LO = 6.825,2 L 21Y


.

E02 =

11.000

. LO +) 10 . 300 L -33,55 = 8.390,1 L 17,34

que corresponden a valores de lnea y ngulos de carga siguientes:


EOI

(lnea)

11.821,6 V ; 6

= 21,5

; E02 (lnea) = 14.532 V ; b2 = 17,34

e)

La potencia activa suministrada en ambos casos es la misma y ya se calcul en el apartado a): P = 4.763 kW. Las potencias reactivas sern: QI

= O(pues el f.d.p. era la unidad) ; Qz = j3 . 11.000. 300 . sen 33,55 = 3.159kVAR


DE APLICACIN 5.8

EJEMPLO

Un motor sncrono conectado en estrella tiene una impedancia sncrona de 0,6 + )6 n/fase y se conecta a una red de 2.200 V, 50 Hz. Calcular: a) F.e.m. producida cuando la mquina absorbe una potencia elctrica de la red de 200 kW confd.p. 0,8 capacitivo. b) ngulo b de retraso del rotor respecto del esttor. SOLUCIN
a)

Si se toma la tensin como referencia de fases, se tiene: V= 2.200

j3

L 0 = 1.270 L 0

La corriente que absorber el motor de la red ser: 1m

j3 Vcos ip j3 .2.200 . 0,8

200.000

= 65,6 A

que al ser capacitiva corresponde a un valor fasaria1:


1m = 65,6 L +36,87 ; arcos 0,8 = 36,87
b)

De acuerdo con (5.79), sustituyendo la reactancia sncrona por la impedancia, resulta: Eo = 1.270 - (0,6 + ) 6) . 65,6 L 36,87 = 1.513 L _13 En consecuencia, el mdulo de la f.e.m. de lnea ser: E (lnea) = 1.513j3

= 2.620,6 V

Yel ngulo de carga b ser: b = _13, que nos indicaque Eo se retrasa 13respectode V.

CAPTULO

5.

MQUINAS SNCRONAS

445

5.13. FUNCIONAMIENTO EN PARALELO DE ALTERNADORES DE POTENCIAS SIMILARES


Se ha demostrado en el epgrafe 5.9 que cuando un generador trabaja en una red aislada, las potencias activas y reactivas suministradas por el alternador son iguales a las que demanda la carga. La frecuencia de funcionamiento se controla con el regulador de velocidad de turbina y la tensin de salida se modifica actuando sobre la corriente de excitacin. En el caso de que el generador trabaje en una red de potencia infinita (vase epgrafe 5.12), la frecuencia y la tensin del generador son fijas y estn impuestas por la red, de tal modo que la potencia activa suministrada por la mquina se controla mediante el regulador de velocidad, mientras que la potencia reactiva se modifica regulando la corriente de excitacin. Vamos a considerar ahora el funcionamiento de dos alternadores en paralelo que tienen potencias similares y que alimentan una carga. De este modo podremos sacar conclusiones prcticas de la misin de los reguladores de velocidad en estas mquinas. En la Figura 5.54a se muestra el circuito elctrico equivalente despus del acoplamiento. Cada generador se representa por una fuente de f.e.m. Eo en serie con la reactancia sncrona (se desprecian las resistencias de los inducidos). Es evidente que la suma de las potencias activas y reactivas entregadas por los generadores debe ser igual a la que demanda la carga. Vamos a demostrar que el reparto de la potencia activa total entre ambos alternadores es funcin de las pendientes de las curvas frecuencia-potencia de cada mquina y de la posicin de los reguladores de velocidad de los motores primarios. Para verlo de un modo grfico, en la Figura 5.54b se han representado las rectas de estatismo de ambas mquinas (curvas frecuencia-potencia), se ha empleado el mismo eje vertical para representar la frecuencia y dos ejes de abscisas para determinar las potencias suministradas por cada generador. Si PI representa la potencia demandada en un determinado instante por la carga, el reparto de potencias

a)

b)

Figura 5.54.

Funcionamiento en paralelo de alternadores de potencias similares.

446

MQUINAS ELCTRICAS

ser el mostrado en la Figura 5.54b. La mquina 1 suministrar la potencia PI mientras que la mquina 2 suministrar la potencia P2, siendo I, la frecuencia comn de funcionamiento. Si la carga aumenta a un valor PII, la frecuencia desciende poco a poco en ambas mquinas hasta alcanzarun nuevo valorI, para el cual las potenciassuministradaspor los generadoresson ahora P; y P~, de tal modo que P; + P~ = PI/" Se observa en la Figura 5.54b que para cada valor de la frecuencia se tiene un valor definido de las potencias generadas por cada una de las mquinas. Se puede hacer un estudio analtico del reparto si se tiene en cuenta la ecuacin (5.51). A la frecuenciaJ;, comn de funcionamiento se cumplir:
(5.81)

donde JI y J2 representaran las frecuencias de vaco de cada generador (punto A de la Fig. 5.54), que en nuestro esquema coinciden. De este modo (5.81), teniendo en cuenta (5.51), se convierte en: tg tg
al
'Y.2

(5.82)

que nos indica lo que de un modo evidente se observa en la Figura 5.54: las potencias activas se reparten de un modo directamente proporcional a las constantes de los reguladores. Resultar ms cargado el generador que tenga el ngulo a mayor (alternador 1 en el caso de la Fig. 5.54). En definitiva, toma ms carga el generador que tenga la caracterstica frecuenciapotencia ms plana. Se puede proceder a un reparto diferente de las cargas si se acta en la regulacin secundaria de los reguladores. En la Figura 5.55 se ha repetido el esquema de la Figura 5.54b, pero donde ahora se ha reducido la posicin del regulador l. Obsrvese entonces que la misma potencia PI que en el caso de la Figura 5.54b se reparta de un modo desigual, ahora se distribuye de un modo ms equitativo; sin embargo, la frecuenciade funcionamientoj se ha reducido, siendo menor que lala que se obtena en el caso anterior. En definitiva, cuando funcionan dos generadores en paralelo, la reduccin en la posicin del regulador de uno de ellos provoca: 1) Reduccin en lafrecuencia del conjunto; 2) reduccinde la potencia suministrada por ese generador, mientras aumenta la potencia entregada por el otro. Vamos a ver qu ocurre con las potencias reactivas. De acuerdo con (5.68) se tiene: Q = 3 _0_ cos b - 3 X, X,
E V V2

(5.83)

Figura 5.55.

Reparto de potencias entre alternadores conectados en paralelo.

CAPTULO

5.

MQUINAS SNCRONAS

447

Si se modifica la excitacin de las mquinas no cambiar la distribucin de potencias activas; sin embargo, se alterar la divisin de la potencia reactiva. Comprobmoslo: Para la mquina l la potencia activa es igual, segn (5.73), a:
PI = 3 --

EoIV X,I

sen

(5.84)

donde 61 es el ngulo que forman

EOI

y V. Por otro lado, la potencia reactiva ser:


-

QI

EOI V . = -cos (jI X,I

V2 3X,I

(5.85)

Despreciando por el momento cualquier variacin en la tensin V, al aumentar la excitacin de la mquina se eleva la f.e.m. EoI' lo que reducir el sen 61, ya que PI es constante al estar fijado por la entrada del motor primario (turbina 1); 61 disminuir, por lo que cos 61 aumentar y en consecuencia Q 1 aumentar tambin. En realidad el proceso es ms complejo ya que al cambiar la excitacin se modifica la tensin V, lo que hace variar la potencia absorbida por la carga; esto hace cambiar la velocidad del motor primario, alterndose con ello la frecuencia, lo que provocar nuevos cambios adicionales en la carga.

EJEMPLO DE APLICACIN 5.9 Se dispone de dos generadores que alimentan una carga. El generador J tiene en vaco una frecuencia de 51,5 Hz y una pendiente de J MW/Hz. El generador 2 presenta unafrecuencia en vaco de 5J Hz y con la misma pendiente. Los dos generadores alimentan una carga total de 2,5 MW conf.d.p. 0,6 inductivo. Calcular: J) A qufrecuencia funciona el sistema y qu potencias activas suministran cada lino de los generadores? 2) Supngase que se conecta una carga adicional de J MW. Cul sera la nueva frecuencia del sistema y las nuevas potenciasactivas? 3) En la situacin del apartado anterior se aumenta en 0,5 Hz la posicin del reguladordel alternador 2. Determinar la nuevafrecuencia del sistema y el reparto de la potencia entre los dos generadores. SOLUCIN
1. De acuerdo con (5.51), la expresin de las rectas frecuencia-potencia sern:

ya que la pendiente es igual a l. Igualando ambas frecuencias se obtiene:

51,5-PI=51-P2
y adems la suma de las potencias es de 2,5 MW, de donde se deduce. PI = 1,5 MW 2. ; P2 = 1 MW ; f= 50 Hz

Procediendo de un modo similar, teniendo en cuenta que se aade una carga de 1 MW, resulta: P; = 2 MW ; P~ = 1,5 MW ;

f'

= 49,5 Hz. ser:

3.

Al cambiar la posicin del regulador 2 su caracterstica frecuencia-potencia

f=51,5-P2

448

MQUINAS ELCTRICAS

que coincide con la del generador l y, por consiguiente, la potencia total de 3,5 MW se repartir por igual entre ambos generadores.
P;' = P~' = 1,75 MW

lo que da lugar a una frecuencia de 49,75 Hz.


EJEMPLO DE APLICACIN 5.10

Dos alternadores trifsicos trabajan en paralelo y alimentan una carga de 6 MW con j.d.p. 0,8 inductivo. La frecuencia de uno de ellos cae de 51 Hz a 49,75 Hz cuando alimenta una carga de JO MW, y el otro pasa de 51 Hz a 49,5 Hz cuando se carga con 2 MW. Determinar las potencias activas suministradas por cada alternador y el f.d.p. con el que trabaja el primero, si el segundo funciona con un f.d.p. 0,71 inductivo. SOLUCIN De acuerdo con (5.51), las constantes de cada regulador sern:
KI =

10 51-49,75

= 8 MW/Hz ; K2 =

2 51-49,5

= 1,33 MW/Hz

De este modo se tienen las caractersticas de los reguladores:

ya que en paralelo trabajarn con la misma frecuencia. De la ecuacin anterior se desprende que:
PI = 6P2

y como quiera que PI + P2 = 6 MW, se obtiene:


PI

= 5,143 MW

; P2

= 0,857 MW

La potencia activa total es de 6 MW con f.d.p. 0,8 inductivo, y por consiguiente, la potencia reactiva total ser:
QT= PTtg ({JT= 6 0,75 = 4,5 kVAR

que corresponde a una potencia compleja:


ST=6+)4,5

De un modo anlogo, la potencia compleja del 2." generador, que suministra 0,857 MW con f.d.p. 0,71 inductivo, ser:
S2 = 0,857 + ) 0,85

de donde se deduce una potencia compleja para el alternador 1:


SI

= ST -

S2

= 5,143 + ) 3,65 = 6,306 L

35,36

por lo que su f.d.p. ser igual a cos 35,36 = 0,815.

CAPTULO

5.

MQUINAS SNCRONAS

449

EJEMPLO DE APLICACIN

5.11

Dosalternadores distintos conectados en estrella estn acoplados en paralelo alimentando una carga aislada. Ambas mquinas tienen sus resistencias de inducido despreciables. La reactanciasncrona de la primera mquina es de 3 n y la de la segunda vale 4 n. Lasf.e.m.s. porfase generadas por ambas mquinas son iguales a 220 V, estando laf.e.m. E2 del segundo generadorretrasada 20 respecto a la f.e.m. El del primero. Se sabe tambin que la carga absorbe una corriente total de 18,1 A que est retrasada 42,5 respecto de E/, Calcular: a)f.d.p. de la carga; b) corrientes suministradas por cada alternador con susf.d.p. respectivos;c) tensin simple en la barra comn a ambas mquinas; d) potencias activas y reactivas suministraspor los dos generadores; e) potencia activa y reactiva absorbida por la carga. SOLUCIN
a)

El esquema correspondiente al montaje citado es el que se muestra en la Figura 5.54a. Para poder resolver las ecuaciones elctricas que representan el comportamiento de la red es preciso elegir alguna referencia de fases; lo ms cmodo ser tomar la tensin en barras como origen de fases. Respecto a esta referencia se presentarn los valores de los argumentos de todas las magnitudes implicadas. Es conveniente partir de valores de argumentos positivos para evitar errores en los signos de los ngulos. Al resolver despus las ecuaciones se obtendrn los signos reales. De acuerdo con estas premisas, las magnitudes involucradas en el problema respondern a las formas fasoriales siguientes:

Si se denomina <51 el ngulo de potencia del alternador 1, es decir, el ngulo de adelanto de la f.e.m. respecto de la tensin en este generador, y si se tiene en cuenta adems que las f.e.m.s. El y E2 tienen un mdulo de 220 V Y que la fase de E2 se retrasa segn el enunciado 20 respecto de la del primero, las expresiones fasoriales de las f.e.m.s. sern:

Si se denomina (/Jr al desfase de la corriente de carga respecto de la tensin y sabiendo que la corriente de carga se retrasa 43,5 respecto de El y que el mdulo de la corriente total es de 18,I A, se tendr la siguiente expresin fasorial de la corriente de carga:

Aplicando ahora los lemas de Kirchhoff a la red de la Figura 5.54a resulta:


EI=V+)XIII; E2=V+)X212; Ir=II+12

que teniendo en cuenta las expresiones fasoriales sealadas antes y los valores de las reactancias sncronas, da lugar a: 220 L 61 = V + )311 L (/JI= V + 311 L (90 + (/JI) 220 L (JI - 20) = V + )412 L (/J2 = V + 412 L (90 + 18,1 L (61
-

(/J2)

42,so) = I1 L (/JI+ 12 L (/J2

450

MQUINAS ELCTRiCAS

Las tres ecuaciones complejas anteriores se convertirn en seis ecuaciones algebraicas al igualar partes reales e imaginarias en cada una de ellas, resultando: 1) 2) 3) 4) 5) 6) 220 cos 6, = V - 3/, sen ({J, 220 sen 6, = 3/, cos (P, 220 cos (6, - 20) = V - 4/2 sen ({J2 220 sen (6, - 20) = 4/2 COS ({J2 18,1 cos (6, - 42,5) = 1, cos ({J, + /2 COS ({J2 18,1 sen (6, - 42,5) t, sen 6, + /2 sen 62

sustituyendo los valores de las componentes activas de las corrientes que aparecen en las ecuaciones 2) y 4), en la ecuacin 5) se obtiene: 220 220 18,1 cos (6, - 42,so) = sen 6, + - sen (6, - 20) 3 4 y desarrollando las expresiones trigonomtricas anteriores resulta: 112,78 sen 6, = 32,15 cos 6,

= tg 6, = 0,285 = 6, = 15,9

lo que indica que las expresiones de las f.e.m.s. de los generadores son: E, = 220 L 15,9 ; E2 = 220 L -4,1 es decir, la f.e.m. del generador E, se adelanta 15,9respecto a la tensin de barras, lo que indica que trabaja como generador; mientras que la f.e.m. de la mquina E2 se retrasa 4,1 de la tensin, lo que representa, como sabemos, un trabajo como motor sncrono. El desfase ({Jpde acuerdo con la expresin fasorial de la corriente total, corresponde a un valor:
(PT

= (5, - 42,5 = -26,6

es decir, la corriente de carga se retrasa 26,6 de la tensin en bornes. Se trata, por consiguiente, de una carga inductiva cuyo f.d.p. vale: cos
b)
({JT

= cos 26,6 = 0,894 inductivo

Para calcular ahora los valores de las corrientes, vamos a sustituir el valor de 6, encontrado en las ecuaciones 1 a 6, lo que da lugar al juego siguiente: 7) 8) 9) 10) 11) 12) 211,6=V-3/,sen({J, 60,3 = 3/, cos ({J, 219,4 = V - 4/2 sen ({J2 -15,7=4/2cos({J2 16,2 = t. cos ({J, + 12 COS ({J2 -8,1 = I, sen ({J, + /2 sen ({J2

Si se restan las ecuaciones 7) y 9) se obtiene: 13) -7,8 = -3/, sen ({J, + 412 sen ({J2 ecuacin que unida a la 12) permite obtener las componentes reactivas de las corrientes: 14) I, sen ({J, = -3,5 ; /2 sen ({J2 = -4,6

CAPTULO 5.

MQUINAS SNCRONAS

451

Los valores anteriores, junto con las ecuaciones 8) y 10),permiten calcular las expresiones de II e 12: II
0

= 20,1 - j3,5 = 20,4 L

-98

12

= -3,93

- j4,6

= 6,05 L

-130Y

es decir, la mquina l entrega una corriente a la carga que vale 30,4 A Yse retrasa 9,8 a la tensin, trabajando con un f.d.p, 0,985 inductivo. La mquina 2 entrega a la carga una corriente de 6,05 A retrasada 130,5 de la tensin. O de otro modo: la mquina 2 absorbe una corriente 12m igual y opuesta a la expresada en 15), que representa la corriente que absorbe como motor. Esta corriente motora se adelanta 180 -130,5 = 49,5 a la tensin, por lo que considerada as trabaja con un f.d.p. capacitivo de valor 0,649. e) Se puede calcular la tensin, sustituyendo el primer valor 14) en la ecuacin 7), dando lugar a:
0 0 0 0

v = 211,6 d)

10,5 = 201,1 voltios

En resumen, los resultados obtenidos son:

v = 201,1 LO
12 =

; El = 220 L 15,9 ; El = 220 L -4,1 ; li = 20,4 L -9,8 6,05 L -130,5 = 12m = 6,05 L 49,5 ; IT= 18,1 L -26,6

La potencia compleja entregada por la mquina 1 es:


SI

= 3 Vlr = 3
PI

201,1 . 20,4 L 9,8= 12.128 + j2.095

que corresponde a unas potencias:

12.128 W ; QI

= 2.095 VAR

La potencia compleja entregada por la mquina 2 es:


S2

= 3 VIi = 3201,1
P2

. 6,05 L 130,5 = -2.370 + j2.775

que corresponde a unas potencias generadas:

= -2.370 = +2.370

W ; Q2

= +2.775 = -2.775

VAR

o a unas potencias absorbidas como motor: P2m W ; Q2 VAR

De este modo los dos generadores entregan en conjunto:


PT e)

= PI + P2 = 9.758 W

QT

= QI

+ Q2 = 4.870 VAR

De un modo anlogo al anterior resulta una potencia compleja absorbida por el receptor o carga:
ST = 3 VIi = 10.920 L 26,6 = 9.764 + j 4.889

es decir: PT = 9.764 W ;
QT

= 4.889 VAR

que coincide (salvo error de redondeo) con la suma de las potencias entregadas por los generadores. Fsicamente lo que sucede es que el alternador 1 alimenta a la carga y al alternador 2, que trabaja como motor.

452

MQUlNAS ELCTRICAS

5.14. MOTOR SNCRONO. CARACTERSTICAS y APLICACIONES


En el epgrafe 5.12 se ha indicado cmo puede la mquina sncrona pasar del funcionamiento como generador al trabajo como motor desconectando el motor primario de arranque, ejerciendo entonces un par til en el eje transformando la energa elctrica absorbida de la red en energa mecnica de rotacin. La velocidad de giro del motor viene expresada por la relacin:
n=p

60f

(5.86)

que es la de sincronismo de la red. El motor sncrono presenta el grave inconveniente de que el par conserva un sentido nico solamente cuando la mquina se halla ya sincronizada, es decir, cuando el rotor gira a la misma velocidad que el campo del inducido. Si el rotor est en reposo o gira a otra velocidad diferente a la de sincronismo, el par medio que desarrolla al conectarlo a la red es nulo. En efecto, consideremos el esquema de la Figura 5.56, donde se muestran dos conductores del inducido enfrentando a dos polos consecutivos del rotor. En el instante indicado en a) la corriente en el conductor M es saliente al plano de la pgina y en el N es entrante, de tal forma que se ejerce una fuerza sobre los conductores del inducido en el sentido contrario a las agujas del reloj que obliga a girar a los polos (rotor) en sentido opuesto. Si la frecuencia de las corrientes que recorren el esttor es de 50 Hz, al cabo de 1/100 segundos se invierte el sentido de las mismas (Fig. 5.56b), resultando un par en el rotor de sentido contrario al anterior. En consecuencia, el rotor se ve sometido a un par alternativo que impide el arranque de la mquina en reposo. Para obtener un par til en el rbol de la mquina, es preciso que el rotor avance un semipaso polar en el tiempo correspondiente al semiperodo de la corriente de inducido. De esta forma, aunque cambia el sentido de la corriente, tambin cambia la polaridad del polo enfrentado a los conductores, lo que trae consigo el que no se modifique el sentido del par producido. Debido a la inercia del rotor no es posible, partiendo del reposo, obtener este avance de los polos en tan breve espacio de tiempo (1/100 segundo para f = 50 Hz), por lo que se hace necesario utilizar dispositivos especiales de arranque para la puesta en marcha de estos motores.

Rotacin
a) Fuerza en los conductores

Rotacin
b) Fuerza en los conductores medio ciclo despus

Figura 5.56. Principio de funcionamiento del motor sncrono.

CAPTULO

5. MQUINAS SNCRONAS

453

En los motores sncronos que pueden arrancar en vaco, la puesta en marcha se realiza por medio de un motor auxiliar (motor pony), generalmente asncrono con igual nmero de polos que el motor principal, de tal forma que se consigue una velocidad de rotacin casi sncrona y la conexin a la red se realiza empleando equipos de sincronizacin al igual que se haca en el acoplamiento de un alternador a la red. Tambin se pueden emplear para este fin motores de c.c., debido a su ventaja de regulacin de velocidad, o motores asncronos con un par de polos menos que el motor sncrono, y en este caso la conexin a la red se efecta una vez que se ha desconectado el motor auxiliar y el grupo pasa suavemente por la velocidad de sincronismo. Otro procedimiento ms prctico para la puesta en marcha de estos motores consiste en su arranque como asncronos. Para este fin es necesario colocar un arrollamiento en jaula de ardilla sobre los polos de la mquina, como muestra la Figura 5.57. Para efectuar el arranque asncrono el devanado de la excitacin debe estar cerrado sobre una resistencia hmica cuya magnitud sea 10-15 veces superior a la propia. Sera peligroso dejar el arrollamiento inductor abierto, debido a que el campo giratorio podra inducir en l una f.e.m. muy elevada, lo que podra provocar la rotura del aislamiento. Tambin sera un inconveniente cerrar el devanado en cortocircuito, puesto que se creara una elevada corriente monofsica que frenara el rotor. El arranque asncrono se realiza aplicando algunos de los procedimientos explicados en el Captulo 4. Generalmente, debido a que estos motores suelen ser de gran potencia, se aplican las conexiones estrella-tringulo o utilizando un autotransformador. Una vez efectuada la puesta en marcha como asncrono, cuando se obtiene la velocidad de rgimen cercana a la de sincronismo, se conecta la c.c. al devanado de excitacin y entonces se producen unas oscilaciones de velocidad respecto a la sncrona y al cabo de unos perodos se llega a la velocidad asignada. Este proceso constituye la llamada autosincronizacin del motor. Finalizada la operacin del arranque del motor sncrono, se podr ya regular su corriente excitacin para que la mquina trabaje en rgimen de subexcitacin o sobreexcitacin con el fin de regular su f.d.p.; de esta forma esta mquina puede cumplir la doble misin de arrastrar una carga mecnica y compensar la corriente reactiva de la red. Generalmente, la jaula de ardilla colocada en estos motores y que aqu se utiliza para producir un arranque asncrono, se coloca tambin en los generadores y recibe el nombre de devanado amortiguador, ya que reduce las oscilaciones que se producen en los procesos transitorios de las mquinas sncronas: acoplamiento a la red, variaciones bruscas de carga elctrica o mecnica, etc. El efecto de estos devanados amortiguadores en rgimen permanente es nulo, ya que al girar la mquina a la velocidad de sincronismo no se inducen corrientes en los mismos.

Barras del devanado amortiguador

Anillo de cortocircuito de las barras

Devanado inductor Polo saliente

Figura 5.57. Devanado amortiguador colocado en los polos (jaula de ardilla).

454

MQUINAS ELCTRICAS

El motor sncrono puede utilizarse para mover cargas mecnicas, y al igual que el motor asncrono o de induccin, su empleo es muy interesante desde el punto de vista de coste y rendimiento en aquellas aplicaciones que necesitan velocidades bajas, inferiores a 500 r.p.m., para cargas con conexin directa (sin engranajes de reduccin), como en molinos de cemento, papel, mezcladoras y en la propulsin elctrica de buques. Se fabrican para unos valores asignados del f.d.p. de 0,8 en adelanto y unidad. En su versin de potencias inferiores a I CV no utilizan C.c.para la excitacin y su funcionamiento se basa en la variacin de reluctancia del rotor (motores de reluctancia), empleando para este fin una jaula de ardilla de un motor asncrono convencional, a la que se han eliminado algunos dientes. Tambin se emplean motores sncronos de histresis en los que el rotor es cilndrico y utiliza un anillo de un material magntico permanente. Estos motores sncronos fraccionarios se emplean para impulsar relojes elctricos y otros aparatos medidores de tiempo. Para grandes potencias, una de las mayores ventajas de este motor frente al asncrono es la posibilidad de regular el f.d.p. En la Figura 5.58a se muestra una instalacin con una carga activa PI con f.d.p. (cos (/JI) inductivo a la que se ha conectado un motor sncrono que va a absorber una potencia activa P2. Si se desea que el f.d.p. de la instalacin sea cos (/JT> habr que desarrollar una potencia reactiva capacitiva (Fig. 5.58b) en el motor (actuando sobe la excitacin), de tal forma que se debe cumplir: (5.87) En consecuencia, la potencia aparente del motor sncrono y el f.d.p. capacitivo con el que trabaja ser:
SM

Jp; + Q; ;

cos

(/Jo

= P2

SM

(5.88)

Si el motor sncrono no desarrolla ninguna potencia mecnica y se supone que no existen prdidas en la mquina, se tendr P2 = O y la expresin (5.87) se transforma en la potencia reactiva que tendra que entregar a la red trabajando como compensador sncrono. Este tipo

R~----~--------~
u so----+--..... --~
el
tu

CARGA TRIFSICA

:y,

0----+--+--....... --1 COS(PI

'+0

Q > O (inductiva)

Q< O (capacitiva)

Motor sncrono

Qrl=
A
h) Diagrama de potencias

B
QI-Q2

a) Esquema de circuito

Figura 5.58.

Motor sncrono trabajando como condensador.

CAPTULO

5. MQUINAS SNCRONAS

455

de rgimen es el empleado para regular el f.d.p. y la tensin en las grandes lneas de transporte de energa elctrica. EJEMPLO DE APLICACIN 5.12 Un motor sncrono sobreexcitado para absorber 600 kVA mueve una carga mecnica de 260 kW con rendimiento de 0,87. El motor est conectado en paralelo con una carga de 1.000 kVA, con f.d.p. 0,707 inductivo. Calcular el f.d.p. resultante. SOLUCIN El diagrama fasorial del sistema es el indicado en la Figura 5.59. De este esquema se deducen las siguientes relaciones:
lOA]

1.000
-

COS (/>1

1.000 0,707 = 707 kW

[AB] = -

260 = 300 kW 0,87


(/>1:=

[Cfr] = J6002

3002 = 519,6 kVAR ; [BD] = IAE]


[BC] = [BD]-

1.000 sen

707 kVAR

[CD] = 707 - 519,6 = 187,4 kVAR

En consecuencia, el f.d.p. resultante ser: cos


(/JT

= -~

[OB]

[OC] )(707 + 300)2 + (187,4)2

707 + 300

= 0,983

5.15. DIAGRAMA DE LMITES DE FUNCIONAMIENTO DE UNA MQUINA SNCRONA


El diagrama de lmites de funcionamiento de una mquina sncrona es un grfico que representa los valores mximos de potencia activa y reactiva que puede suministrar la mquina funcionando como generador (es menos frecuente pero tambin se construyen estos diagramas para motores sncronos), en funcin de los lmites de calentamiento, tanto del devanado del esttor como del rotor y de la limitacin de potencia de la mquina motriz (turbina), en el supuesto de considerar un funcionamiento a la tensin asignada. La construccin de este grfico se basa en el diagrama fasorial de la mquina sncrona. En la Figura 5.60 se muestra este diagrama para un alternador funcionando con tensin asignada

Figura 5.59.

456

MQUINAS ELCTRICAS

Lugar geomtrico de Eo '" constante

o. e
('1;

.._ 11
o

(JO

'" ...,

,., (JO

v
C
I

= '"
:

('1;

~ ~~
.......,../)
..........

o 3

s ;:;" o

....

Figura 5.60.

Lugares geomtricos de E e 1 en un alternador.

y factor de potencia inductivo. En el dibujo se observa que el lugar geomtrico de los puntos para los cuales la f.e.m. Ea es constante corresponde a circunferencias con centro en C. En la Figura 5.60 se muestra el arco de una de ellas. Ahora bien, la f.e.m. E es proporcional a la corriente de excitacin de la mquina, por consiguiente estas circunferencias representan tambin el lugar geomtrico de los puntos del diagrama para los cuales le = constante. Si la corriente de excitacin se reduce a la mitad del valor que corresponde a la situacin dibujada en la Figura 5.60, el lugar geomtrico ser entonces una circunferencia con centro en e y radio mitad. De este modo, si se trazan circunferencias con diferentes radios, se obtendrn lugares geomtricos para diferentes corrientes de excitacin. El lugar geomtrico de XJ = constante, que en definitiva corresponde al de corriente de inducido I = cte, corresponder a circunferencias con centro en O y radio el valor de la corriente de inducido de que se trate. Como quiera adems que la tensin de la red es constante, la potencia aparente es proporcional a la corriente de inducido y por consiguiente las circunferencias anteriores representan tambin los lugares geomtricos de los puntos de funcionamiento de la mquina para los cuales la potencia aparente es constante. En definitiva, el esquema de la Figura 5.60 puede emplearse para representar potencias no solamente aparentes sino tambin activas y reactivas, como se va a demostrar a continuacin. Obsrvese que en este figura se cumplen las siguientes relaciones: [08]

= XJ

; [AB]

= XJ

cos ep ; lOA]

= XJ

sen ep

(5.89)

Como quiera que las potencias activa, reactiva y aparente suministradas por la mquina sncrona son: P

= 3 VI cos ep

; Q

= 3 VI sen ep

; S = 3 VI

(5.90)

el tringulo OA8 se podr recalibrar como un tringulo de potencias, a base de multiplicar cada lado por el factor de conversin 3 V/X" que es un parmetro constante. Tngase en cuenta para justificar lo anterior que las expresiones (5.90) admiten la siguiente representacin:
P

= 3 VI cos (p = -

3V
X,'

(X,! cos ep)

=-

3V X,

[AB]

(5.91)

CAPTULO 5.

MQUINAS SNCRONAS

457

= 3 VI sen ep = S = 3 VI =

3V X,"

(XI sen ep)


3V

=-

3V

X,

[DA]

(5.92)

3V

X,

(X,!)

=-

X,

[OBJ

(5.93)

En la Figura 5.61 se ha vuelto a dibujar el diagrama de la Figura 5.60, pero en el que cada lado de la figura se ha multiplicado por el factor de conversin citado. Se ha rayado el tringulo que representa las potencias. El eje de ordenadas define las potencias activas, mientras que el eje de abscisas determina las potencias reactivas. El centro de referencia es el punto O. Para el punto B se tiene una potencia activa positiva P y una potencia reactiva positiva Q; la excitacin correspondiente, segn se ha sealado antes, ser proporcional a la distancia MB. Para el punto de trabajo e se tiene una potencia activa positiva Pe' una potencia reactiva negativa Qc y una potencia aparente definida por la distancia De. La corriente de excitacin ser proporcional a la distancia Me. En la prctica de la ingeniera de los sistemas elctricos de potencia se prefiere utilizar diagramas en valores por unidad donde las magnitudes estn referidas a unos valores base. Para las potencias se toma como base la potencia aparente asignada del generador y para las tensiones se elige como base la tensin asignada de la mquina (ver Apndice 3). En el caso de las corrientes de excitacin, se toma como base de referencia la corriente inductora necesaria para producir la tensin asignada en vaco. Las corrientes de inducido utilizan como base, la corriente asignada de la mquina, mientras que para las impedancias se utiliza un valor base, definido en (5.20) como cociente entre la tensin asignada y la corriente asignada. De acuerdocon estas premisas se ha construido en la Figura 5.62 el denominado baco de operacin de un generador sncrono, en el que se muestran los lmites de funcionamiento. El procesoa seguir es el siguiente: l. Se elige el centro O, y se dibujan los ejes de coordenadas X e y correspondientes, respectivamente, a potencia reactiva y activa (en valores por unidad, p.u.). Con la escala que se desee, se trazan Crculoscon centro en O. En el caso de la Figura 5.62 se han dibujado cinco semicircunferencias para valores 0,2; 0,4; 0,6; 0,8 y 1 p.u. de la potencia aparente asignada de la mquina.
P(kW)

3V -Eo

x,

Q(.

Q(kVAR)
sen tp

x,
Q'= 3 VI

Figura 5.61. Diagrama de potencias de un alternador.

458

MQUINAS ELCTRICAS

f.d.p. Excitacin (p.u.) MW(p.u.) inductivo

25
2,3 2

------- ..
...

0,95 .............
0,8

............ _--- ........... _

~ ~ - -.....

- ..........

.s

" '" O
" -c

0._
u

0.6
h

k 0,5

0.4

M~J---~--O-~J-~r-~~-G~~-I~~~----__. 0,7 g 0,4 0.2 O 0.2 0.4 0.6 0,8 MVAR (p.u.)
capacitivo inductivo

MVAR (p.u.)

Figura 5.62. baco de operacin de un alternador. Lmites de funcionamiento. 2. Se localiza el punto M del diagrama. Esto requiere un poco de atencin. Para este punto se tiene una potencia reactiva que segn muestra la Figura 5.61 vale: (5.94) ya que la tensin de funcionamiento escribir tambin as: es la asignada. La expresin anterior se puede

Q,,=3V
;1' 11

V --" X\

(5.95)

que en valor por unidad respecto a la potencia asignada del generador es:

3V QM
(p.lI.)

_I'

= _,_ -::;----V f S,y .> ""

QM

"X,

'1,,11,, Z"
X,
X,

(5.96)

donde 2" representa, segn (5.20). la impedancia base de la mquina. Esta expresin se puede poner tambin:

Q\1
.

(p.u.)

1 1 = - = -_.X,

X,

(p.u.)

(5.97)

2"

CAPTULO

5.

MQUINAS SNCRONAS

459

ya que segn (5.21) el cociente X/Z, representa la reactancia sncrona en valores p.u. Si se considera, por ejemplo, una reactancia sncrona de valor X/Zb = 1,43 p.u., el punto M se localizar para una magnitud de potencia reactiva p.u. segn (5.97):

QM (p.u.)
3.

= -1,43 = 0,7

p.U.

(5.98)

Con centro en M se trazan circunferencias correspondientes a corrientes de excitacin constante. La corriente de excitacin base (de referencia) corresponde al segmento MO, que representa la corriente necesaria en el inductor para que se genere en vaco la tensin asignada. Por este motivo esta distancia representa un valor I p.u. Con esta escala se han trazado en la Figura 5.62 cinco cuadrantes de circunferencia corresponI.\.\e\\\.e\> a los siguientes valores de la excitacin: 0,5 - 1 - 1,5 - 2 - 2,5 p.u. 4. Con centro en O se trazan lneas radiales que representan los valores del f.d.p. de la mquina y que segn la Figura 5.62 corresponden a los ngulos que estas lneas radiales forman con la vertical (eje de potencias activas). En el caso de la Figura 5.62 se han trazado las lneas correspondientes a los f.d.p.: 0,2; 0,4; 0,6; 0,8; 0,9; 0,95. A continuacin se deben dibujar en el baco las lneas y curvas. que representan los lmites de funcionamiento de la mquina en rgimen permanente, que vienen impuestas por:

a) Potencia asignada de la mquina motriz. b) Potencia aparente del alternador, que limita la corriente de circulacin por el inducido, para que en este devanado no se superen las temperaturas admisibles. e) Corriente mxima de excitacin del generador, que est limitada por el calentamiento del devanado del rotor. d) Angulo de carga (j mximo permitido para que se cumplan las condiciones de estabilidad de la mquina.
De este modo el proceso a seguir sera:
a)

b)

e)

Se traza una lnea horizontal con la potencia de la turbina. Realmente este valor corresponde a la mxima potencia activa que puede suministrar el generador. Atendiendo a esto los constructores definen, adems de la potencia aparente asignada del generador (MV A), el f.d.p. asignado; el producto de ambos factores determina la potencia activa mxima, que a lo sumo debe coincidir con la de la turbina o motor de accionamiento. Si se considera el valor clsico de f.d.p. 0,8 inductivo, la potencia activa mxima ser de 0,8 p.u. y es sta la recta horizontal ah que se ha trazado en la Figura 5.62. La limitacin de la potencia aparente del generador (o lo que es equivalente, de corriente en el inducido) viene definida por el arco be correspondiente a la circunferencia de potencia aparente 1 p.u. (centro en O). La limitacin de la corriente de excitacin viene expresada por el arco ed (centro de la circunferencia en el punto M). Para dibujar este arco se ha supuesto un valor mximo de la corriente de excitacin de 2,3 p.u. (vase este valor en el eje vertical), que corresponde en cierto modo a una f.e.m. 2,3 p.u. (es decir, 2,3 veces la tensin asignada). Hay que hacer notar que los arcos correspondientes a la limitacin de la corriente en el inducido, be, y a la limitacin de corriente en el inductor, cd, se cortan en c. Desde este punto hacia el eje de abscisas es ms fuerte la limitacin que impone la corriente de excitacin que la corriente del esttor (en la zona cd, el arco bfcottes-

460

MQUINAS ELCTRICAS

d)

pondiente a la limitacin de corriente por el inducido es exterior al cd que impone el inductor). El lmite de estabilidad en rgimen permanente se obtiene, segn se explic en el epgrafe 5.11, para un ngulo de carga i5 = 90, que corresponde a una recta perpendicular a MO trazada por el punto M. En la prctica se suele reducir esta zona a un valor de i5 menor, para poder hacer frente a posibles variaciones de carga y a inestabilidades de tipo transitorio. Algunos autores simplifican el tema y consideran un valor mximo de c5 del orden de 60. Sin embargo, en la prctica de la ingeniera se recurre a restar un 10 por 1(JO de la potencia asignada de la mquina (MV A) en cada Crculo de excitacin, proyectando el punto resultante hasta la circunferencia. Si se considera la circunferencia de excitacin 1 p.u., se tiene el punto h, que constituye el lmite de estabilidad terico; entonces se resta a la potencia correspondiente a este punto (ordenada Mh) un 0,1 p.u. de la potencia aparente asignada (segmento hk) y esto da lugar al segmento Mk; el punto lmite de funcionamiento se obtiene proyectando el punto k a la circunferencia, dando lugar al punto 1, que define uno de los puntos de la curva que constituye el lmite de estabilidad prctico. Se pueden obtener ms puntos repitiendo el proceso con otras circunferencias de excitacin.

Para que quede ms clara esta construccin, en la Figura 5.63 se ha vuelto a dibujar el tramo ag a mayor escala. El origen es el punto M, y en ordenadas tenemos potencias activas. Se han trazado semicircunferencias con centro en M (origen de excitacin). La distancia MA representa la potencia aparente asignada del alternador (1 p.u.). Se toma un lO por 100 de MA, es decir, 0,1 p.u., y se obtiene el punto B, que se proyecta hacia la circunferencia de radio 1 p.u., dando lugar al punto C. El proceso se repite para el punto B, que al restar 0,1 p.u. nos da el punto D, que proyectado da lugar al E. Tngase en cuenta que la magnitud a restar es siempre la misma: 0,1 p.u. de la potencia aparente asignada. Es por esto que al llegar a la semicircunferencia de radio MF correspondiente a una potencia 0,1 p.u. y restar la misma cantidad se obtiene el punto M, que proyectado en la circunferencia da lugar a G. Es decir, el lmite de estabilidad prctico no llega a M sino que se queda a una distancia 0,1 p.u. de l.

Figura 5.63. Detalle del proceso de construccin del lmite de estabilidad prctico.

CAPTULO 5.

MQUINAS SNCRONAS

461

EJEMPLO DE APUCACIN 5.13


Un turbogenerador de 10 MVA, 2.400 V, 50 Hz, conectado en estrella, tiene una reactancia sncrona de 0,7 O. La corriente mxima del rotor corresponde a un valor para el cual la mquina producira en vaco una tensin 2,1 veces la asignada (se considera el circuito magntico lineal). La turbina que mueve el generador es de 7,5 MW y el ngulo de carga mximo para funcionamiento estable se supone igual a 60. a) Dibujar el diagrama que muestra los lmites de funcionamiento de la mquina. b) Puede el generador entregar una comente de lnea de 2.300 A con f.d.p. 0,32 inductivo? c) Si el generador entrega una potencia activa de 7 MW, cal es la potencia reactiva mxima que puede entregar simultneamente con f.d.p. inductivo?

SOLUCIN
a)

La corriente asignada del generador es: 1m =

V 3 2.400

r:;

10 106

2.406 A

Vn

2.400

J3

1.386 V

De acuerdo con los valores anteriores la impedancia base valdr:


Z =--=05760 b 2.406 '

1.386

por lo que la reactancia sncrona p.u. tendr una magnitud:


0,7

X, (p.u.) = 0,576 = 1,22 p.u. El grfico que muestra los lmites de funcionamiento de la mquina es el mostrado en la Figura 5.64.
Excitacin (p.u.) 2,1 -----------------______ MW (p.u.)
-, -,

--

'......,' ... ............ "

0,9

f.d.p. inductivo

0,6

0,4 0,32

0,4

0,2

0,2

0,4 0,6

0,8

MVAR(p.u.)

P = 0,306 MW(p.u,); Q = 0,906 MVAR(p.u,); S =0,956MVAL 71,34;cos71,34= 0,32

Figura 5.64.

462

MQUINAS ELCTRICAS

El punto M corresponde a una potencia reactiva p.u.:

QM

= x, (p.u.) = -1,22 = 0,82

p.u.

La distancia MO corresponde a una tensin de vaco l p.u. En el dibujo se han trazado dos cuadrantes de circunferencia que representan los valores de excitacin 1 p.u. y 2,1 p.u., respectivamente, y esta ltima circunferencia representa la limitacin de la corriente en el rotor. La potencia de la turbina es de 7,5 MW, que corresponde a un valor p.u.:

P (p.u.)

= - = 0,75
10

7,5

b)

De acuerdo con estos valores se han trazado las lneas lmites de la Figura 5.64. El tramo ab corresponde a la limitacin de la turbina; be corresponde a la limitacin de potencia aparente o corriente de excitacin, que segn se muestra en la figura corresponde a un valor de 2,1 p.u. En la parte izquierda se ha trazado la recta Ma con un 15= 60, que limita la estabilidad. En estas condiciones el generador entrega unas potencias activa y reactiva: p

= J3 . 2.400 . 2.300 . 0,32 = 3,06 MW = 0,306 p.u. Q = P tg l.fJ = 3,06 . 2,96 = 9,06 MV AR = 0,906 p.u.

que corresponde al punto A del diagrama de la Figura 5.64. Se observa que este punto est dentro de la curva de potencia aparente mxima pero est fuera de la curva de corriente de excitacin mxima; por tanto, el generador no puede funcionar en estas condiciones, ya que provocara el calentamiento del rotor. Se puede comprobar lo anterior de un modo analtico, ya que la f.e.m. necesaria en estas condiciones sera: E,

= V + jX)

= 1.386 LO + jO,7 . 2.300 L -71,3

= 2.956,6

L 10

que corresponde a un valor por unidad:

Eo (p. u.)

= --

2.956,6 1.386

= 2,13

p.u.

e)

que es superior a la circunferencia de corriente de excitacin mxima (2,1 p.u.). Si el generador entrega una potencia activa de 7 MW, es decir, 0,7 p.u., la mxima potencia reactiva que puede suministrar es de 7,14 MV AR (punto B en el diagrama de la Figura 5.64). Este punto se encuentra entrando con una potencia activa 0,7 p.u. en el diagrama y trazando una horizontal (potencia activa constante) hasta encontrar el lmite del recinto (0,714 p.u.).

5.16. TRANSITORIO DE CORTOCIRCUITO DE UNA MQUINA SNCRONA


La condicin transitoria ms grave que le puede ocurrir a un generador sncrono es el cortocircuito trifsico. En el estudio de sistemas elctricos de potencia se dice entonces que se ha producido una falta o fallo de cortocircuito. El anlisis riguroso del cortocircuito es difcil y

CAPTULO

5.

MQUINAS SNCRONAS

463

se sale de los lmites de este texto. Nuestro inters reside en comprender de un modo cualitativo los fenmenos fsicos que se producen y que dan lugar a la definicin de unas reactancias sncronas nuevas que tienen importancia en el comportamiento transitorio de estas mquinas. Para facilitar el estudio se va a considerar que se parte de un generador trabajando en vaco y que en un momento determinado se produce un cortocircuito trifsico entre los tres terminales del inducido. En la Figura 5.65 se presenta un oscilograma que muestra la naturaleza general de la variacin de la corriente en una de las tres fases en el momento del cortocircuito (en las otras dos fases, las formas de onda diferirn de la que aquf se muestra, a causa del hecho de que en cualquier instante de tiempo la suma de las corrientes de las tres fases debe ser igual a cero). Se observa en este oscilograma que los valores de pico de la corriente definen las envolventes representadas por las curvas ab y ef Por lo tanto, la curva cd, equidistante entre los envolventes, representa una corriente unidireccional, es decir, de C.C., que cae lentamente, sobrela que se superpone otra de c.a. que tambin se amortigua con el tiempo, indicada por la formade onda de la Figura 5.66. Esta ltima curva se construye situando las amplitudes de la componente de c.a. de la Figura 5.66 por encima y por debajo del eje c'd' (que ocupa el lugar de la curva cd en la Figura 5.65). Las envolventes de la nueva curva, una de ellas marcada por a'b', sern simtricas alrededor del eje de tiempo. Se denomina a esta seal componente simtrica de la c.a. de cortocircuito. La aparicin de estas componentes de c.c. se basa en el concepto de flujo atrapado, un conceptorelacionado con la ley de Lenz. Esta ley establece que un cambio en el flujo induce f.e.m.s.que provocan a su vez corrientes en los circuitos afectados que tienden a oponerse a los cambios del flujo. Por ello, cuando se produce un cortocircuito en un alternador, fluirn corrientes en las tres fases del inducido, en el devanado de excitacin y en los devanados amortiguadores(vase Fig. 5.57) que intentarn mantener el flujo en el mismo valor que tena en el instante previo a la falta. En otras palabras, el flujo es atrapado por la mquina. Cuando se produce el cortocircuito, la componente alterna de la corriente salta a un valor muy alto, perola corriente total no puede cambiar en ese instante (para mantener el flujo constante), de ahque aparezca una c.c. suficientemente grande para que la suma de las componentes de c.a. y de c.c. instantneamente despus de la falta sea igual a la corriente alterna que circulaba antes de la falta (que si se parte del generador en vaco tena un valor previo nulo). Las componentesde c.c. se extinguen rpidamente pero inicialmente pueden llegar a alcanzar el 50 o 60 por 100 de los valores de la c.a. un instante despus de la falta. De este modo la corrienteinicial total puede llegar a valer de 1,5 a 1,6 veces la magnitud de la c.a. sola. Si se analiza ahora la componente simtrica de la corriente alterna (Fig. 5.66), se observa quepuede dividirse en tres perodos: durante algo as como el primer ciclo despus de la falta, la componente de la c.a. es muy grande y decae rpidamente. Corresponde al denomina-

i(t)

Figura 5.65. Oscilograma de la corriente de cortocircuito en una fase.

464

MQUINAS ELCTRICAS

a'

m n' e'

-n -11 - n-l1- ~ 11 -" "


lJ U
\1

n 1\ 11 1\ Tf ---t;::-.Ji U, U V V lJ V V V V Vd' i

horto

Figura 5.66. Componente simtrica de la corriente de cortocircuito. do perodo subtransitorio. Concluido este perodo, la corriente sigue disminuyendo ms lentamente hasta alcanzar un rgimen permanente final; corresponde al perodo transitorio. Finalmente, el tiempo que transcurre desde cuando la corriente alcanza el rgimen permanente se conoce como perodo de estado estacionario. En el perodo subtransitorio, la corriente de cortocircuito est limitada en el esttor nicamente por la reactancia de dispersin. Al no poder cambiar el flujo instantneamente, para contrarrestar el efecto desmagnetizante de la corriente en el inducido aparecern corrientes en el devanado de excitacin y en los devanados amortiguadores, que tendern a conservar el flujo atrapado. Estos devanados actan en este momento como si fueran secundarios de un transformador en el que el primario es el esttor. Si se denominan las reactancias de estos bobinados Xe (excitacin o inductor), Xa (amortiguador) y es Xp la reactancia de reaccin de inducido, el circuito elctrico equivalente en este perodo (supuesta una relacin de transformacin unidad) es el que se muestra en la Figura 5.67a. Las tres reactancias Xe' Xa y Xp se asocian en paralelo y estn conectadas en serie con la reactancia de dispersin X". La reactancia equivalente de este circuito se denomina reactancia subtransitoria y su valor ser: X"=X
s

+----"1 1 1 -+-+-

(5.99)

x, x,
___Q_

Xp

De este modo, el valor eficaz de la corriente en el perodo, de acuerdo con la Figura, 5.67a ser:
I" =

X" s
l

(5.100)

y este valor corresponde en la Figura 5.66 al cociente c a' El efecto del devanado amortiguador desaparece despus de los primeros ciclos a causa de que la resistencia de este bobinado es relativamente mayor que la del devanado de excitacin, por ello en el perodo transitorio siguiente se puede considerar un circuito equivalente de la mquina tal como el mostrado en la Figura 5.67b. La reactancia de esta red se denomina reactancia transitoria X; y su valor ser:
X' = X + __e
s

/j2.

"Xe

XX _p_ +Xp

(5.101)

CAPTULO 5.

MQUINAS SNCRONAS

465

s .:;
.~

g
a)

b)

e)

Figura 5.67.

Circuitos equivalentes para los diversos perodos de tiempo,

Por tanto, el valor eficaz de la corriente en el inducido en el perodo transitorio ser:

r
l

= __Q

X;

(5.102)

que corresponde al cociente c m' en la Figura 5.66. Despus del perodo transitorio desaparece el efecto de la corriente de excitacin, dando lugar al circuito equivalente de la Figura 5.67c, en el que se tiene la reactancia sncrona normal X,:

/j2

x, = X" + Xp
correspondiendoa una corriente de cortocircuito de rgimen permanente:
1 corto
l

(5.103)

Eo =X,

(5.104)

y que corresponde al cociente c n' en la Figura 5.66. En resumen, el valor eficaz de la c.a. de cortocircuito vara continuamente en funcin del tiempo.Si I" es el valor subtransitorio, I' el transitorio e Icorto el permanente, entonces el valor eficazde la corriente de cortocircuito se puede expresar as: (5.105) T" Y T' tienen dimensiones de tiempo y representan, respectivamente, las constantes de tiempo subtransitoria y transitoria. Los valores aproximados son TI! = 0,03 segundos y T' = 1 segundo.Los valores p.u. de las reactancias para un turbogenerador son del orden de:
X, = 1 a 2 ; X; = 0,22 a 0,35 ; X;' = 0,1 a 0,25

/j2

Mientrasque para un alternador hidrulico (polos salientes) son: X, = 0,5 a 1,5 ; X; = 0,2 a 0,5 ;

X;'

= 0,13 a 0,35

466

MQUINAS ELCTRICAS

EJEMPLO DE APLICACIN 5.14 Un alternador trifsico de 50 MVA, 13,2 kV, 50 Hz, est funcionando en vaco a la tensin asignada, cuando se produce un cortocircuito trifsico en sus terminales. Las reactancias por unidad referidas a los valores nominales de la mquina son: X, = J; X; = 0,3; X;' = 0,15; y las constantes de tiempo son: TU = 0,03 segundos; T' = 1,1 segundos. Si se desprecia la componente continua en la corriente de cortocircuito, calcular: a) Valor eficaz de la corriente alterna en el instante posterior a lafalta. b) Valor de la corriente despus de transcurrir dos ciclos de la tensin de red y tambin despus de 5 segundos. SOLUCIN
a)

La corriente inicial es la subtransitoria: l' = Eo 1 = X;' 0,15 = 6'67 p.u.

Como quiera que la corriente asignada vale:


J=

50 106

J3 . 12,2 . 10

2.187 A
3

b)

entonces el valor real de I' ser de 6,67' 2.187 = 14.587 A. Las corrientes transitoria y permanente de cortocircuito son:
I"

= 0,3 = 3,33 p.U.

; (OrlO

= 1=

1 p.u.

por lo que segn (5.105) la corriente i (t) valdr: i(t) = 3,34 e-110m + 2,33 Para 2
t
e-IIJ1

+ 1 (en valores p.u.)

= 2 ciclos = - = 0,04 segundos resulta un valor:


50 i(t

= 0,04 segundos) = 0,88 + 2,247 + 1 = 4,127 p.u.

que corresponde a una intensidad absoluta de 9.025 A. Para t = 5 segundos resulta una corriente:
(t

= 5) = O + 0,025 + 1 =

1,025 p.u.

es decir, 2.241 A, que es prcticamente la corriente de cortocircuito permanente.

PROBLEMAS
5.1. Un alternador trifsico de 6 polos, 1.000 r.p.m. tiene un esttor de 54 ranuras sobre el que se sita un devanado de paso diametral que contiene 10 conductores/ranura. Si el flujo por polo es de 2 . 10-2 Wb y su forma senoidal, determinar la f.e.m. inducida por fase. [Resp.: 383,2 V.]

CAPTULO

5.

MQUINAS SNCRONAS

467

5.2.

Un alternador trifsico de 750 r.p.m., 50 Hz, est diseado para generar una f.e.m. de 3.500 V/fase. El devanado del esttor tiene 120 ranuras con 24 conductores/ranura y la anchura de las bobinas es de 12 ranuras. Calcular el flujo mximo por polo si su distribucin es senoidal. [Resp.: 0,0361 Wb).] Un alternador trifsico de 1.500 kVA, 6.600 V, conectado en estrella, tiene una curva de vaco definida por la ecuacin:

5.3.

E _ 12.210 F" 0- 4.250 + Fe


donde Eo se expresa en tensin de lnea y Fe representa la f.m.m. de excitacin en A.v/polo. La resistencia y reactancia de dispersin del inducido por fase son 0,6 O y 2,3 O, respectivamente. Se obtiene la corriente de plena carga en cortocircuito con una excitacin de 2.500 A.v/polo (ste es un modo de dar la reaccin del inducido a plena carga). Determinar: a) F.e.m. E, de lnea a plena carga y con f.d.p. 0,8 inductivo. b) Corriente de excitacin necesaria en el inductor cuando la mquina est girando a plena carga con f.d.p. 0,8 inductivo, si se sabe adems que la mquina tiene polos salientes devanados con 190 espiras cada uno. e) Si en la situacin del apartado anterior se desconecta repentinamente la carga, cul ser el valor de la tensin de lnea que aparecer en bornes de la mquina? d) Cunto vale la regulacin de tensin de la mquina? [Resp.: a) 7.030,7 V. b) 40 A. e) 7.835 V. d) 18,71 %.] 5.4. Un alternador trifsico de 5.000 kV A, 6.600 V, conectado en estrella, tiene una curva de vaco definida por la ecuacin:
E o

= _7._40_0_ .....:.le 85 + le

donde E se expresa en voltios por fase e le representa la corriente de excitacin. La resistencia y reactancia de disposicin del inducido por fase son 0,2 O y 1 O, respectivamente. Se obtiene la corriente de plena carga en cortocircuito con una excitacin de 20 A. Calcular: a) Margen de excitacin necesario para dar una tensin asignada constante desde vaco a la plena carga con f.d.p. 0,6 inductivo. b) Si las prdidas en el hierro, por friccin y rozamiento con el aire ascienden a un total de 100 kW y las bobinas de campo estn alimentadas con una excitatriz a 200 V, calcular el rendimiento a plena carga con f.d.p. 0,6. [Resp.: a) 90,23 A en vaco; 129,6 A a plena carga. b) 92,6 %.] 5.5. Un alternador trifsico conectado en estrella de 1.000 kV A, 4.600 V, tiene una impedancia sncrona de 2 + j 20 O/fase. Determinar la regulacin a plena carga con factores de potencia: a) unidad, b) 0,75 inductivo. [Resp.: a) 44,6 %. b) 81,4 %.] Un generador sncrono trifsico cia sncrona de 0,4 + j 6 O/fase. tra una potencia de 1.000 kW b) unidad, e) 0,866 capacitivo. [Resp.: a) 9,7 %. b) 1,85 %. e) conectado en estrella de 6.600 V, tiene una impedanCalcular la regulacin de la mquina cuando suminisa la tensin asignada con f.d.p.: a) 0,866 inductivo, -5,95 %.]

5.6.

468

MQUINAS ELCTRICAS

5.7.

Un alternador trifsico conectado en estrella de 4.000 kV A, 6.600 V, ha dado los siguientes resultados en unos ensayos de vaco y cortocircuito:

10 960 232

20 1.920 464

30 2.800

40 3.440

60 4.220

80 4.600

100 4.800

La resistencia del inducido es despreciable. Calcular: a) Regulacin a plena carga con f.d.p. 0,8 inductivo. b) Corriente de excitacin necesaria en el caso anterior. NOTA: Tmese como reactancia sncrona la primera aproximacin obtenida de los ensayos, es decir, la que corresponde a la excitacin necesaria para obtener la tensin asignada en vaco. [Resp.: a) 21,33 %. b) 81 A] 5.8. Un alternador trifsico conectado en estrella de 1.000 kVA, 6.600 V ha dado los siguientes resultados en un ensayo de vaco:

2.000 4.300

3.250 6.600

4.025 7.250

5.200 7.920

6.500 8.580

Por el mtodo de Potier se ha determinado el valor de la reactancia de dispersin y la f.m.m. de reaccin de inducido a plena carga, resultando los valores: Xa = 3,1 O; F; = 2.000 Av/polo. Calcular: a) F.m.m. de excitacin necesaria a plena carga con f.d.p. 0,8 inductivo. b) Regulacin de tensin. [Resp.: a) 5.120 Av/polo. b) 19,7%.] 5.9. Un generador sncrono trifsico conectado en estrella de 1.500 kV A, 6.600 V ha dado los siguientes resultando en un ensayo de vaco:

1.848 1.000

3.696 2.000

5.511 3.500

6.600 5.000

7.128 6.000

7.590 7.000

7.986 8.000

En un ensayo de cortocircuito se obtiene la corriente de plena carga para una excitacin de 2.500 Av/polo. En un ensayo con carga reactiva se necesitan 8.250 Av/polo para que circule la corriente asignada a la tensin de rgimen de 6.600 V. Calcular: a) F.m.m. de excitacin necesaria en los polos cuando el generador suministra la plena carga con f.d.p. 0,8 inductivo. b) F.e.m. correspondiente en vaco. e) Regulacin de tensin del alternador en este rgimen. [Resp.: a) 7.330 Av/polo. b) 7.700 V. e) 17 %.]

CAPTULO 5.

MQUINAS SNCRONAS

469

5.10.

Un alternador trifsico de 1.000 kV A, 11.000 V, conectado en estrella, tiene una resistencia de inducido despreciable. Los ensayos de vaco y con carga reactiva pura a la intensidad asignada han dado los siguientes resultados: 20 5.800 25 55 70 90

7.000 12.500 13.750 15.000 1.400 8.500 10.500 12.400

Determinar por el mtodo de Potier: a) Cada de tensin en la reactancia de dispersin a plena carga. b) F.m.m. de reaccin de inducido con corriente asignada. e) F.e.m. resultante E, cuando la mquina funciona a plena carga con tensin asignada y f.d.p. 0,8 inductivo. d) F.m.m. necesaria en la excitacin en el caso anterior. e) F.e.m. E que producir en vaco la excitacin anterior. f) Regulacin de tensin de la mquina en las condiciones anteriores. [Resp.: a) 1.155 V/fase. b) 13 A e) 12.305 V. d) 64 A e) 13.200 V. f) 20%.] 5.11. Un alternador trifsico de 5.000 kV A, 6.600 V, conectado en estrella, ha dado los siguientes resultados en unos ensayos de vaco y f.d.p. nulo con corriente de plena carga: 3.200 3.100 5.000 4.900 7.500 10.000 14.000 6.600 7.500 5.800 8.300 7.000

1.850 4.250

La resistencia del inducido es despreciable. Determinar por el mtodo de Potier: a) Cada de tensin a plena carga en la reactancia de dispersin del inducido. b) F.m.m. de reaccin de inducido con corriente asignada. e) Repetir los apartados anteriores para una corriente de carga de 500 A d) Determinar las f.m.m.s. de excitacin necesarias en el inductor cuando la mquina suministra una corriente de 500 A a la tensin asignada, con factores de potencia: unidad, 0,9 capacitivo, 0,71 inductivo. e) Calcular en las condiciones del apartado anterior las f.e.m.s. producidas en vaco. [Resp.: a) 520 V/fase. b) 2.300 Av/polo. e) 594 V/fase; 2.630 Av/polo. d) 8.465; 6.616; 11.352 Av/polo. e) 7.000 V; 6.000 V; 7.900 V.] 5.12. Un alternador trifsico conectado en estrella de 6.600 V ha dado los siguientes resultados en un ensayo de vaco: 10 3.000 15 4.200 21 5.400 31 6.600 39 7.200 50 7.700 65 8.100

En cortocircuito se necesita una excitacin de 24 A para que circule la corriente de plena carga. En un ensayo con carga reactiva se obtienen 6.100 V con una corriente 125 por 100 de la asignada, para una excitacin de 66 A La resistencia del inducido es

470

MQUINAS ELCTRICAS

despreciable. Calcular: a) Cada de tensin a plena carga en la reactancia de dispersin del inducido. b) F.m.m. del inducido con corriente asignada. e) F.m.m. necesaria en la excitacin cuando el generador suministra la plena carga con f.d.p. unidad y 0,8 inductivo. d) F.e.m.s. producidas en vaco en las condiciones del apartado anterior. e) Regulaciones de tensin correspondientes. [Resp.: a) 554 V/fase. b) 21 A. e) 40,7 A. d) 54,5 A. e) 10,6 %; 19,7 %.] 5.13. Un alternador trifsico conectado en estrella de 5.000 kVA, 6.600 V, tiene una resistencia del inducido despreciable. El ensayo de vaco a la velocidad de sincronismo ha dado los siguientes resultados:
24 3.000 35 4.200 50 5.400 71 6.600 90 7.300 120 8.000 140 8.300

En cortocircuito es necesaria una corriente de excitacin de 37 A para que circule una corriente de 300 A en el inducido. En un ensayo con carga reactiva y corriente asignada se obtiene una tensin de 6.000 V para una excitacin de J 30 A Cuando la mquina suministra los 3/4 de la plena Caiga a la tensin asignada con f.d.p. 0,8 inductivo, se pide: a) Cada de tensin en la reactancia de dispersin. b) F.m.m. de reaccin de inducido. e) Excitacin necesaria en el inductor. d) F.e.m. que producir la excitacin anterior al dejar la mquina en circuito abierto. e) Regulacin de tensin correspondiente. [Resp.: a) 476 V. b) 33,8 A e) 121 A d) 8.050 V. e) 22 %.] 5.14. Un alternador trifsico tiene una impedancia sncrona de O + j 5 Q y est conectado a una red de potencia infinita de 6.600 V. La excitacin es tal que la f.e.m. inducida en vaco es de 6.000 V. Determinar la potencia activa mxima que en estas condiciones podr suministrar la mquina, sin que exista prdida de estabilidad. Hallar tambin la comente de inducido y el f.d.p. para dicha carga. [Resp.: 7.920 kW; 1.029,5 A; 0,673 capacitivo.]

5.15. Un alternador trifsico tiene una impedancia sncrona de O+ j Ion y est conectado a una red de potencia infinita de 11.000 V suministrando una corriente de 220 A con f.d.p. unidad. Sin cambiar la entrada de potencia a la mquina motriz, se eleva la f.e.m. un 25 por 100. Calcular: a) Intensidad del inducido y f.d.p. en estas condiciones. b) Potencia activa mxima que podr ceder la mquina a la red antes de perder el sincronismo, con el nuevo valor de la excitacin. e) Intensidad y f.d.p. en las condiciones del apartado anterior. [Resp.: a) 281,5 A; 0,781 inductivo. b) 16 MW. e) 1.053 A; 0,797 capacitivo.] 5.16. Un alternador trifsico conectado en estrella tiene una resistencia de inducido despreciable y una reactancia sncrona de 30 Q/fase. Est acoplado a una red de potencia infinita de 11 kV y desarrolla 4.000 kVA con f.d.p. unidad. Si se aumenta la f.e.m. un 20 por 100, permaneciendo constante la entrada de potencia a la mquina motriz, determinar el nuevo f.d.p. con que trabajar la mquina y la potencia aparente que suministra. [Resp.: 0,937 inductivo; 4.270 kVA]

CAPTULO

5.

MQUINAS SNCRONAS

471

5.17.

Un alternador trifsico conectado en estrella tiene una resistencia del inducido despreciable y una reactancia sncrona de 8 Q/fase. La curva de vaco est definida por la ecuacin: E = _20_,2_4_0_' _::le o 42 + le donde E expresa la f.e.m. de lnea e le la corriente de excitacin. Se conecta el generador a una red de potencia infinita de 11 kV suministrando en un momento dado una potencia activa de 3.810 kW con f.d.p. unidad. En esta situacin se aumenta la corriente de excitacin un 50 por 100 sin modificar la apertura de distribuidor de turbina. Calcular: a) Intensidad del inducido y f.d.p. en estas condiciones. b) Potencia activa mxima que podr ceder la mquina a la red antes de perder el sincronismo con el nuevo valor de la excitacin. c) Intensidad y f.d.p. en el caso anterior. [Resp.: a) 246,7 A; 0,81 inductivo. b) 18,28 MW. c) 1.245,5 A; 0,77 capacitivo.]

5.18.

Un generador sncrono trifsico conectado en estrella de 6.600 V, 50 Hz, tiene una resistencia del inducido despreciable y una reactancia sncrona constante. La curva de vaco est definida por la ecuacin.

12.210 'le E =---o 85 + le


donde Eo expresa la f.e.m. de lnea e le la corriente de excitacin. Se conecta la mquina a una red de potencia infinita; una vez efectuado el acoplamiento y sin cambiar la corriente de excitacin, se abre el distribuidor de agua a la turbina hasta que el alternador suministra a la red una potencia activa de 10 MW. En esta situacin se aumenta la corriente de excitacin un 50 por 100 respecto al valor de conexin sin modificar la potencia de entrada a la mquina motriz, comprobndose entonces que se obtiene un f.d.p. 0,8 inductivo. Calcular: a) Reactancia sncrona del alternador. b) F.d.p. con el que trabaja la mquina antes de cambiar la excitacin y entregando la potencia de 10MW. [Resp. a) 0,935 n. b) 0,994 inductivo.] 5.19. Un alternador trifsico conectado en estrella tiene una resistencia de inducido despreciable y una reactancia sncrona de 10 Q/fase. Est acoplado a una red de potencia infinita de 11 kV y se sabe que desarrolla una potencia con f.d.p. 0,673 inductivo, siendo el ngulo de potencia J :: 10. Calcular: a) F.e.m. de lnea producida por el generador. b) Potencia activa que suministra a la red. [Resp.: a) l3.856 V. b) 2.647 kW.] Dos alternadores idnticos de 2.000 kV A funcionan en paralelo alimentando una carga aislada. El regulador de la primera mquina es tal que la frecuencia cae uniformemente de 50 Hz en vaco a 48 Hz a plena carga. La correspondiente cada uniforme de velocidad de la segunda mquina es de 50 Hz a 47,5 Hz. a) Cmo se distribuirn entre los dos generadores una potencia activa consumida por la carga de 2.700 kW? b) Cul es la potencia activa mxima con f.d.p. unidad que puede suministrarse sin sobrecargar ninguno de los alternadores? [Resp.: a) 1.500 kW; 1.200 kW. b) 3,600 kW.]

5,20.

472

MQUINAS ELCTRICAS

5.21. Dos alternadores trifsicos funcionan en paralelo suministrando a una carga una potencia de 6 MW con f.d.p. 0,8 inductivo. La frecuencia de uno de ellos cae uniformemente de 51 Hz en vaco a 49,75 Hz cuando se carga con una potencia activa de 10 MW, y en el otro la frecuencia pasa de 51 Hz a 49,5 Hz cuando se carga con 2 MW. Determinar las potencias activas suministradas por cada generador y el f.d.p. con el que trabaja el primero, si el f.d.p. del segundo es de 0,71 inductivo. [Resp.: 5.143 kW; 857 kW; 0,815.] 5.22. Dos alternadores idnticos de 15 MVA, 6,6 kV, 50 Hz, conectados en estrella, estn acoplados en paralelo, suministrando en conjunto a una red aislada una potencia de 20 MW con f.d.p. 0,8 inductivo. Ambos generadores tienen resistencias de inducido despreciables y reactancias sncronas de un valor de 2,83 Q/fase. Sabiendo que la potencia activa se reparte por igual entre ambos generadores y que el primero tiene una f.e.m. de 11.484V de lnea, calcular: 1)Corrientes suministradas por cada generador con sus f.d.p. respectivos. 2) F.e.m. generada por el segundo alternador. NOTA: Se supone que la tensin comn en barras de ambos generadores permanece constante en el valor nominal de 6.600 V. [Resp.: 1) 1.203 A; 0,726; 1.000 A; 0,874. 2) 9.960 V.] 5.23. Dos alternadores idnticos conectados en estrella estn acoplados en paralelo alimentando una carga aislada. Ambas mquinas tienen sus resistencias de inducido despreciables y sus reactancias sncronas son de 10 Q/fase. Las f.e.m.s. generadas por cada alternador son El = 6.700 V/fase y E2 = 6.500 V/fase, estando la f.e.m. E2 adelantndose 10 elctricos respecto a El. Si la carga absorbe una corriente total de 500 A que est desfasada 37 en retraso respecto a la f.e.m. El' calcular: a) Tensin en la barra comn a ambas mquinas en voltios por fase. b) Corrientes suministradas por cada alternador con sus f.d.p. e) F.d.p. de la carga. [Resp.: a) 5.245 V. b) 219 A; 0,836; 290 A; 0,978. e) 0,933 inductivo.] 5.24. Dos alternadores idnticos de 5.000 kVA, 6,6 kV, conectados en estrella, funcionan en paralelo con las mismas excitaciones y se reparten por igual una potencia activa de 8 MW a 6,6 kV con f.d.p. 0,8 inductivo. Las resistencias de los inducidos son despreciables y las reactancias sncronas por fase valen 17,4 Q. a) Calcular las f.e.m.s. de lnea de cada generador. b) Si la f.e.m. de uno de los generadores se reduce un 15 por 100, determinar la f.e.m. que tendr que generarse en el otro para evitar un cambio en la tensin en barras y un suministro adicional de vapor a cada uno. e) Calcular en las condiciones del apartado anterior las corrientes suministradas por cada generador y sus f.d.p. [Resp. a) 17.935 V Yson iguales para cada generador. b) 20.922 V. e) 380 A; 0,922; 515 A; 0,68.] 5.25. Un motor sncrono trifsico conectado en estrella, de 75 kW, 500 V, tiene una impedancia sncrona Zs = 0,03 + j 3 Q/fase. Si funciona a plena carga con un f.d.p. 0,8 capacitivo y rendimiento del 90 por 100, calcular la f.e.m. inducida Eo Yla potencia activa absorbida de la red. [Resp.: 309 V; 83,33 kW.] 5.26. Un motor sncrono trifsico de 4 polos tiene una resistencia de inducido despreciable y una reactancia sncrona de 3 Q/fase. Est conectado a una red de 2.000 V, 50 Hz. La excitacin es constante y produce una f.e.m. de 1.150 V/fase. Calcular la potencia

CAPTULO 5.

MQUINAS SNCRONAS

473

activa absorbida de la lnea, el factor de potencia y el par desarrollado en el eje si la corriente del inducido es de 200 A. [Resp.: 668 kW; 0,963 inductivo; 4.250 N.m.] 5.27. Un motor sncrono trifsico de 6.600 V conectado en estrella, trabaja con tensin constante y excitacin constante. Su impedancia sncrona es 2 + j 20 Q/fase. Cuando la entrada es de 1.000 kW, el f.d.p. es de 0,8 capacitivo. Hallar el f.d.p. cuando se aumenta la entrada a 1.500 kW. [Resp.: 0,935 capacitivo.] 5.28. Un motor sncrono trifsico de 400 V, 6 polos, 50 Hz, conectado en estrella, tiene una impedancia sncrona de 0,5 + j 4 Q/fase. Absorbe una corriente de 15 A con f.d.p. unidad cuando funciona con una cierta excitacin. Si se aumenta el par de carga hasta que la corriente de lnea sea de 60 A, permaneciendo constante la excitacin, hallar el par total desarrollado y el nuevo f.d.p. [Resp.: 362 N.m; 0,91 capacitivo.] 5.29. Una carga de 250 kVA tiene un f.d.p. de 0,65 inductivo. Se conecta a la misma red un motor sncrono de 75 kW de rendimiento 88 por 100 para elevar el factor de potencia de la instalacin a 0,85 inductivo. a) Calcular la potencia aparente del motor sncrono y el f.d.p. con el que trabaja. b) Si la tensin de alimentacin es de 380 V y el motor tiene una impedancia sncrona de + jO,5 Q/fase (conexin estrella), determinar la f.e.m. Eo inducida en esta mquina. [Resp.: a) 92,7 kVA; 0,919 capacitivo. b) 442,5 V.]

5.30. Una industria absorbe una potencia activa de 2.000 kW con f.d.p. 0,6 inductivo de una red de 6.000 V. Se coloca un motor sncrono conectado en estrella que va a desarrollar una potencia activa de 400 kW con rendimiento 0,8 para elevar el f.d.p. de la instalacin a la unidad. a) Determinar la potencia aparente del motor sncrono y el f.d.p. con el que trabaja. b) Si el motor tiene una resistencia de inducido despreciable y una reactancia sncrona de 2 Q/fase, estando la curva de vaco determinada por la ecuacin:
E _ 9.000' le 030 + le

donde Eo se expresa en voltios de lnea e le en amperios de excitacin, calcular la f.e.m. E del motor y la excitacin necesaria en el inductor. [Resp.: a) 2.713 kVA; 0,184 capacitivo. b) 6.890 V; 98 A.] BIOGRAFAS
1. ALEXANDERSON, Ernest Frederik Werner (1878-1975). Ingeniero elctrico sueco-americano. Emigr a los Estados Unidos en 1901 y al ao siguiente ingres en la GeneralElectric Company en Schenectady bajo la direccin de Charles Proteus Steinmetz. En 1906 desarroll un alternador de alta frecuencia que revolucion las comunicaciones por radio. En 1916 patent un dispositivo de sincronizacin selectiva para receptores de radio. Invent la amplidina, que es una mquina de c.c. de campo transversal, derivada de la dinamo Rosenberg, que se utiliz en sistemas de control automtico en instalaciones industriales, para la regulacin de la excitacin de alternadores y en sistemas de direccin de tiro para artillera. Hoy da las mquinas de campo transversal han sido sustituidas completamente por elementos semiconductores de la familia SCR (rectificadores controlados de silicio). En 1948 se jubil en la GE y continu como ingeniero consultor para la RCA (Radio Corporation of

474

MQUINAS ELCTRICAS

America). Contribuy enormemente al desarrollo de la TV. Estaba en posesin de ms de 300 patentes en los campos de la ingeniera elctrica y de las telecomunicaciones. 2. BEHN-EsCHENBURG, Hans (1864-1938). Ingeniero elctrico suizo. Naci ellO de enero de 1864en Zurich. Muri el 18 de mayo de 1938en Zurich. Estudi entre los aos 1886-1888en el Politcnico de Zurich y los dos aos siguientes ampli estudios en Berln. De vuelta a Zurich en 1891 fue ayudante del gran fsico Heinrich Friedich Weber. que haba sido contratado para dirigir el nuevo Instituto de Fsica del Politcnico y que se haba dotado con grandes medios materiales; de hecho, Weber fue contratado por H.von Helmholtz para efectuar las medidas de la red de transporte de energa elctrica trifsica de Lauffen a Frankfurt con motivo de la Exposicin Electrotcnica Internacional en esta ltima ciudad, y que iba a ser la primera red trifsica del mundo. BehnEschenburg fue el ayudante de Weber en la realizacin de las medidas, lo que supuso una gran labor de aprendizaje tcnico. Como quiera que c.E.L. Brown, director de ingeniera de los talleres suizos Oerlikon y que haba diseado los alternadores de Lauffen en el ro Neckar, al finalizar la Exposicin se asoci con Walter Boveri para fundar la hoy multinacional Brown-Boveri, se eligi como nuevo director de Talleres Oerlikon a E. Huber-Stockar y se contrat tambin a Behn-Eschenburg como director del Departamento elctrico. En esta empresa se encarg del diseo y construccin de las mquinas elctricas, logrando perfeccionamientos notables. Prepar una plataforma de ensayos para comprobar el funcionamiento de las mquinas que salan de los talleres y elabor diversos protocolos de medidas, lo que le llev a estudiar con rigor el comportamiento de las diversas mquinas elctricas. Sus trabajos fueron tan destacados que en el ao 1913 fue nombrado director general de Talleres Oerlikon y ms tarde, en 1919, delegado en el Consejo de Administracin. Entre los aos 1892 a 1922 public gran cantidad de artculos tcnicos en la revista alemana Elektrotechnische Zeitschrift, donde expuso sus investigaciones; en particular desarroll mtodos indirectos para predeterminar la cada de tensin de las mquinas rotativas de c.a. y de los transformadores. Analiz con rigor la reaccin de inducido de alternadores, sustituyendo aqulla por una reactancia principal que al sumarse a la reactancia de dispersin da lugar a la denominada reactancia sncrona del alternador, lo que permite predecir con exactitud (en las mquinas no saturadas) la cada de tensin de la mquina sncrona, y que debido a ello se conoce como mtodo de Behn-Eschenbug o de la impedancia sncrona. Especialista en motores elctricos de traccin, fue el artfice de la electrificacin ferroviaria suiza utilizando una frecuencia de 50/3 Hz. Estudi la conmutacin de los motores monofsicos con conmutador y la recuperacin de energa del mismo. En 1919 recibi el nombramiento como Doctor Honoris Causa de la Politcnica de Zurich. 3. BLONDEL, Andr (1863-1938). Fsico e ingeniero francs. Estudi en la Escuela Politcnica y ms tarde en l'Ecole des Ponts et Chausses (1888) y en la Facultad de Ciencias de Pars (1889). Ingeniero del Servicio Central de Balizas y Faros (1889) Ydesde 1893catedrtico de Electrotecnia en la Escuela de Ingenieros de Caminos de Pars. Entre 1893 y 1902 dise unos galvanmetros especiales que l denomin oscilgrafos y que podan registrar seales de hasta 30 kHz. Invent tambin un histeresmetro y un analizador de armnicos. Con su oscilgrafo hizo importantes estudios de la forma de la onda de los alternadores y del arco voltaico. En 1912 invent un radiofaro, que en la dcada siguiente fue el modelo que se instal en todas las costas francesas. Sent las bases de la metrologa moderna, definiendo unidades fotomtricas y haciendo un estudio de diferentes tipos de proyectores y alumbrado pblico. En electrotecnia su principal contribucin se refiere a las mquinas sncronas, influido por Alfred Potier, que fue profesor suyo de Fsica en la Politcnica. Analiz con rigor el acoplamiento en paralelo de alternadores, determinando los repartos de las diversas potencias entre los mismos. Adapt el mtodo de Potier, que permita calcular la reaccin de inducido en alternadores de polos lisos, para el caso de alternadores de polos salientes, considerando dos tipos de reacciones: la longitudinal y la transversal. Invent bacos para resolver la ecuacin de cambio de condiciones de las lneas areas. Pblico tambin un tratado de 1.700 pginas sobre traccin elctrica. Su producin cientfica le hizo ganar grandes premios: Medalla del Instituto Franklin, la de Lord Kelvin y la del Instituto Electrotcnico de Montefiore. Fue miembro de la Academia de Ciencias de Pars (1913) Yrepresentante francs del AlEE (American Institute of Electrical Engineers).

CAPTULO

5.

MQUINAS SNCRONAS

475

4. CONCORDIA, Charles (1908-). Ingeniero americano. Ingres en 1926 en la General Electric, empresa en la que ejerci toda su carrera profesional hasta su jubilacin en 1973. Fue pionero en la aplicacin de los ordenadores analgicos y digitales para resolver problemas de ingeniera. Sus contribuciones ms importantes se refieren a las mquinas sncronas, dinmica de los sistemas elctricos interconectados, incluyendo estudios sobre la estabilidad, operacin, control y planificacin de los mismos. Escribi en 1951 un magnfico texto sobre mquinas sncronas que fue texto de consulta obligado para los especialistas en esta materia. Fue responsable del diseo y construccin del primer analizador de redes de la General Electrie. Recibi el Premio Coffin en 1942 por sus contribuciones al diseo de accionamientos para tneles de viento (aerodinmica). Medalla Lamme en 1962 por sus aportaciones en mquinas elctricas. Fellow del IEEE en 1947. Doctor Honoris Causa por la Union College en 1971. Fue autor o coautor de ms de 130 artculos tcnicos. Se le atribuyen siete patentes en relacin con la ingeniera elctrica. En 1999 recibi la Medalla de Honor del IEEE por sus grandes contribuciones en el rea de dinmica de los sistemas elctricos de potencia. 5. DOHERTY, Robert E. (1885-1950). Ingeniero americano. Estudi en la Universidad de lllinois (1909). Master en Ciencias por la Union College. Entre 1909 y 1931 trabaj en la General Electrie, Schenectady, en la que fue ayudante de CP. Steinmetz durante seis aos. Fue ingeniero consultor de la empresa y director de formacin de la Compaa. En 1931dej la GE para incorporarse como profesor en el Departamento de Ingeniera Elctrica de la Universidad de Yale. Desde 1936 hasta su fallecimiento fue Presidente del Instituto Carnegie de Tecnologa. Medalla Lamme del IEEE en 1937 por sus trabajos sobre mquinas elctricas. Public numerosos artculos tcnicos, con especial nfasis en las mquinas sncronas, en las que era una autoridad mundial. Perteneci a un gran nmero de sociedades cientficas y estaba en posesin de ttulos acadmicos honorficos: Doctor en Artes por Yale (1931), Doctor en Leyes por Pittsburg (1936), Doctor en Ciencias por el Waysnesburg College (1948). 6. DUDDELL, William du Bois (1872-1917). Fsico francs. Prcticamente toda su formacin e investigacin la realiz en Inglaterra.. Estudi en el City and Guilds Central Technical College (1896), siendo alumno de los profesores Ayrton y Mather. Estando dibujando punto por punto la forma de onda de un arco en c.a., l quiso automatizar las medidas, y para ello tom un oscilgrafo tipo Blondel, al que le hizo algunas mejoras para conseguir un amortiguamiento crtico (1898). Ms tarde hizo estudios sobre el arco elctrico, descubriendo el arco musical, que le dio la fama y notoriedad mundial; su invento consista en hacer pasar una corriente de alta frecuencia a travs de un arco utilizando un condensador y una bobina en paralelo con aqul: el arco se vea sometido de este modo a oscilaciones espontneas. Duddell hizo un estudio completo del fenmeno y prepar una serie de circuitos oscilantes con pulsadores, simulando un piano que permita variar a voluntad la frecuencia propia del arco, con lo que pudo tocar el himno nacional ingls (God save (he Queen). A Duddell se le debe tambin la invencin de un alternador de alta frecuencia de 120 kHz, de un galvanmetro de vibracin y de un ampermetro trmico especial que permita medir con precisin las corrientes de alta frecuencia. Fue elegido miembro de la Royal Society en 1907, Presidente de la Comisin Internacional de Telegrafa en 1914 y Presidente del lEE desde 1912 hasta 1914, sustituyendo a Ferranti. Muri en la Primera Guerra Mundial. 7. LAMME, Benjamin G. (1864-1924). Ingeniero americano. Ingres en la Compaa Westinghouse en 1893 y en esta empresa trabaj toda su vida. Comenz como ingeniero de laboratorio para realizar ensayos de mquinas elctricas y en el ao 1903 ya era director de ingeniera. Lamme jug un papel de extraordinaria importancia para el desarrollo de esta Compaa. Fue pionero en el desarrollo y construccin de mquinas elctricas: motores de induccin, alternadores polifsicos, conmutatrices, motores monofsicos con conmutador para traccin elctrica. Fue uno de los ingenieros responsables (el otro era Charles Scott) del diseo del generador de 5.000 CV instalado en las cataratas del Nigara en 1895. (Ambos ingenieros tenan solamente veintinueve aos cuando realizaron este proyecto en 1893. Esta instalacin tuvo la importancia histrica de ser la primera en los EE.UU. de c.a. y tipo bifsica a 2.200 V, 30 Hz). Proyect y construy lneas de traccin elctrica en New York, New Haven y Hartford. Era un experto matemtico y escribi gran nmero de artculos sobre ingeniera elctrica.

476

MQUINAS ELCTRICAS

8. MORDEY, WilIiam Morris (1856-1938). Ingeniero elctrico britnico. A los catorce aos ingres en el Servicio de Telgrafos de Gran Bretaa. En 1881 se incorpor a la Compaa inglesa Brush como encargado del departamento de ensayos elctricos. En esta fbrica se construan dinamos para alumbrado elctrico de lmparas de arco con la patente americana de Brush de Cleveland; tambin se fabricaban lmparas incandescentes. Mordey, que tena gran habilidad para la construccin de mquinas elctricas, enseguida fue el responsable del diseo de las mismas. A finales del siglo XIX el alumbrado sola realizarse con lmparas de arco alimentadas por mquinas Gramme, Brush, Jablochkoff y Siemens. A partir de los trabajos tericos de Rowland y sobre todo de J. Hopkinson se lleg a un conocimiento del circuito magntico de las mquinas elctricas, que permiti un diseo cientfico de las mismas. Mordey comenz a disear dinamos tipo Schukert en su fbrica y ms tarde inici el diseo y construccin de alternadores, en vista del desarrollo de la c.a. promovida en Inglaterra por Ferranti y por Westinghouse en Estados Unidos. A Mordey se le debe el invento del devanado compound o compuesto en las dinamos. Desarroll el trabajo en paralelo de los alternadores y el funcionamiento de estas mquinas como motores sncronos. Fue el primer ingeniero que se dio cuenta de que las curvas que relacionan la corriente de inducido con la corriente de excitacin de las mquinas sncronas tenan forma de V, y de ah que reciban actualmente el nombre de curvas en V de Mordey. Fue Presidente del Instituto de Ingenieros elctricos ingleses (lEE) en el bienio 1908-1909. 9. POTIER, Alfred (1840-1905). Ingeniero francs. Estudi en la Politcnica y despus en la Escuela de Ingenieros de Minas (1859). En 1881 ocup la ctedra de Fsica en la Escuela Politcnica y desde 1893 fue catedrtico de electricidad en la Escuela de Ingenieros de Minas, ctedra que se estableci expresamente para l. Colabor con Michcl Lvy y Bertrand en la confeccin del mapa geolgico de Francia. Realiz investigaciones en Fsica: polarizacin de la luz, revisin del principio de Huygens, etc.; escribi tambin varias memorias didcticas sobre termodinmica. En electricidad investig sobre la teora de la pila voltaica y la determinacin del equivalente electroqumico de la plata. En electrotecnia estudiel comportamiento de la mquina de c.c. y realiz trabajos importantes sobre la reaccin de inducido de los alternadores. En la Exposicin de Electricidad de Pars fue uno de los que ms trabaj en la eleccin de un sistema coherente de unidades elctricas. Tambin fue uno de los fundadores de la revista L'clairage lectrique,que ms tarde, tras diversas transformaciones, dara lugar a la actual Rvue gnrale d' lectricit. Desde 1891fue miembro de la Academia de Ciencias de Pars. 10. RUDENBERG, Reinhold (1883-1961). Ingeniero germano-americano. Estudi ingeniera elctrica y mecnica en Hannover (1906). Fue ayudante de Ludwig Prandtl en Gotinga, conocido por sus investigaciones en aerodinmica. Entre 1908 y 1936 trabaj en la Compaa Siemens-Schuckert de Berln como ingeniero de laboratorio para ensayo de mquinas elctricas, donde lleg a director de ingeniera. En esta etapa profesional invent el motor trifsico con conmutador y escobillas ajustables y el autoarranque por corrientes de Foucault de un motor de induccin. En 1916proyect un alternador para una central hidrulica de 60 MVA, un rcord para esa poca. Ide conductores huecos para redes de transporte. En 1913 fue nombrado profesor de electrotecnia en Berln, donde ms tarde, en 1919, fue catedrtico y profesor honorfico en 1927. En 1936 se traslad a Inglaterra, trabajando hasta 1938 como ingeniero consultor de la General Electric. En 1939 se fue a los EE.UU., donde fue contratado como catedrtico de ingeniera elctrica en Harvard. En 1952, al llegar sujubilacin, fue profesor visitante en Berkeley, Ro de Janeiro, Sao Paulo y Montevideo. Escribi ms de 100 publicaciones sobre electricidad: anlisis de la conmutacin en las mquinas de C.C., motor de c.a. con conmutador, diseo de disyuntores y teora del arco elctrico. Escribi dos libros de texto. En particular, el que lleva por ttulo, Comportamiento transitorio de sistemas elctricos de potencia, cuya primera edicin data de 1923 y que fue traducido a varios idiomas, fue un libro de consulta obligada en muchas escuelas de ingenieros y tiene captulos que an hoy da se pueden considerar de actualidad. Se le atribuyen ms de 300 patentes en todas las ramas de la ingeniera elctrica. 11. SKINNER, Charles Edward (1865-1950). Ingeniero americano. Se gradu en ingeniera mecnica en la Universidad de Ohio (1890). Doctoren Ciencias por la misma Universidad en 1927.En 1890 ingres en la Compaa Westinghouse. Fue uno de los ingenieros responsables de las instalaciones

CAPTULO 5.

MQUINASSNCRONAS

477

12.

elctricas de la central del Nigara en el perodo 1893-95. Permaneci en la Westinghouse toda su vida. En esta Compaa planific y construy los laboratorios de investigacin: ensayo de materiales, fsico-qumica y el de A.T. Se retir como Director Adjunto de Ingeniera. Representante americano de la Comisin Electrotcnica Internacional, de la Sociedad Internacional de Materiales, etc. Perteneci al Comit americano de patrones y normas. Presidente del IEEE (1931-1932). Miembro de diversas sociedades cientficas: IEEE, Instituto Franklin, AS ME, Sociedad americana para el avance de la Ciencia, etc. WHITEHEAD, John Boswell (1872-1955). Ingeniero americano. Se gradu en la Universidad John Hopkins (1893), doctorndose en 1902. Fue uno de los responsables de las instalaciones elctricas de la central del Nigara. Trabaj en esa poca para la Westinghouse; ms tarde pas a la empresa Niagara Fall Power. En 1898 fue contratado como profesor de la Universidad John Hopkins y se encarg de crear los estudios de Ingeniera Elctrica. En 1910 obtuvo el nombramiento de catedrtico de Ingeniera Elctrica, desde 1920 a 1938 fue Director del Departamento de Electrotecnia y a partir de 1938 hasta su jubilacin en 1942 fue director de la Escuela de Ingeniera. Escribi diversos artculos sobre electromagnetismo, transformadores y traccin elctrica. Premiado con la medalla del Instituto Edison en 1941. Perteneci a un gran nmero de comits cientficos americanos. Cooper con la fuerza areas americanas en investigaciones electrnicas aplicadas. Presidente del IEEE entre 1933 y 1934.

REFERENCIAS
1. 2. 3. 4. 5. 6. 7. 8. 9. lO. 11. 12. 13. 14. 15. 16. 17. 18. 19. 20. 21. ASINEL: Textos sobre Centrales Termoelctricas Convencionales y Nucleares. Grupo D, Volumen 01: Alternadores; Volumen 2: Funcionamiento del alternador. Asinel, Madrid, 1985. BEWLEY, L. V.: Alternating Current Machinery. MacMillan CO., New York,1949. BROSAN,G. S.; HAYDEN,J. T.: Advanced Electrical Power and Machines. Isaac Pitman & Sons Ltd., London, 1966. CONCORDIA, CH.: Synchronous Machines. John Wiley & Sons, New York, 1951. CORTES,M.: Curso Moderno de Mquinas Elctricas Rotativas, tomo 111. Ed. Tcnicos Asociados, Barcelona, 1973. CRARY, S. B.: Power System Stability, Vols. I y Il. John Wiley & Sons, New York, 1947. CHAPMAN,S. J.: Mquinas Elctricas. McGraw-Hill Latinoamericana, S. A., Bogot, Colombia, 1987. CHATELAIN, J.: Machines lectriques. Trait d'lectricit, Vol. X. Ed. Georgi, Lausanne, 1983. DEL TORO, V.: Electric Machines and Power Systems. Prentice Hall, Inc., Englewood Cliffs, N. J., 1985. DRAPER,A.: Electrical Machines. Longman Group, London, 1971. ELGERD,O. l.: Electric Energy Systems Theory: An Introduction. Tata McGraw-HiII Pub., New Delhi, 1971. EL HAWARY,M. E.: Principles of Electric Machines with Power Electronic Applications. Reston Book, Prentice Hall, Englewood Cliffs, N. J., 1986. FITZGERALD, A. E.; KINGSLEYCH.;UMANS,S. D.: Electric Machinery, Fourth Ed. (S.I.). McGrawHill Book Co., New York, 1985. GEMLICH,D. K.; HAMMOND,S. B.: Electromechanical Systems, McGraw-Hill, New York, 1967. HINDMARSH,1.: Electrical Machines. Pergamon Press, New York, 1965. IEEE: Test Procedures for Synchronous Machines, n.? 115. Institute of Electrical and Electronic Engineers, New York, 1965. IVANOV-SMOLENSKY, A. V.: Electric Machines, tomo 2. Mir Pub., Moscow, 1982. JONES,C. V.: The Unified Theory of Electrical Machines. Plenum Pub., Corp., New York, 1968. KIMBARK,E. W.: Power System Stability, Vol. Il: Synchronous Machines. John Wiley & Sons, New York, 1956; reimpreso por Ed. Dover en 1968. Kosow, I.: Electric Machinery and Transformers. Prentice Hall, Inc., Englewood Cliffs, New Jersey, 1972. KOSTENKO, M. P.; PIOSTROVSKI: Mquinas Elctricas, tomo 11. Ed. Mir, Mosc, 1975.

478
22. 23. 24. 25. 26. 27. 28. 29. 30. 31. 32. 33. 34. 35. 36.

MQUINASELCTRICAS
LANGSPORF, A. S.: Teora de las mquinas de CA. Ed. Castillo, Madrid, 1967. LAWRENCE, R. R.; RICHARDS, H. E.: Principies of Alternating-Current Machinery, 4: ed. McGraw-Hill, New York, 1953. LIWSCHITZ-GARIK, M.; WHIPPLE,C. Electric Machinery, Vol. 2. Van Nostrand Co., New York, 1946. MABLEKOS,V. E.: Electric Machine Theoryfor Power Engineers. Harper & Row Pub., New York 1980. MATSCH, L. W.: Mquinas electromagnticas y electromecnicas. Representaciones y Servicios de Ingeniera S. A., Mxico, 1972. MCPHERSON,G.: An Introduction to Electrical Machines and Transformers. John Wiley & Sons, New York, 1981. NAGRATH,I. J.; KOTHARI,D. P.: Electric Machines. Tata McGraw-HilI Pub., New Delhi, 1985. PARK, R. H.: Two-Reactance Theory of Synchronous Machines Generalized Methods of Analysis, Pt. 1, AlEE Trans., Vol. 48, July, 1929, pgs. 716-730. PuCHSTEIN,A. F.; LLOYD,R.; CONRAD,A. G.: Alternating Current Machines. Wiley lnterscience, New York, 3: ed., 1954. SAY, M. G.: Alternating Current Machines. Pitman Pub. Co., London, 1976. SLEMON,G. R.; STRAUGHEN, A.: Electric Machines. Addison-Wesley, Massachusetts, 1980. STEIN, R.; HUNT, W. T.: Electric Power System Components: Transformers and Rotating Machines. Van Nostrand Reinhold Co., New York, 1979. THALER,E. J.; WILCOX,M. L.: Mquinas elctricas: Estado dinmico y permanente. Ed. LimusaWiley S. A., Mxico, 1969. WESTINGHOUSE: Electrical Transmission and Distribution Reference Book. Westinghouse Co., East Pittsburg, Pennsylvania, 1964. WILDI, TH.: Tecnologa de los Sistemas Elctricos de Potencia. Ed. Hispano Europea S. A., Barcelona, 1983.

c.

CAPTULO 6

Mquinas de corriente continua

6.1. INTRODUCCIN
Las mquinas de c.c. tienen una gran importancia histrica debido a que su empleo como generadores o dinamos representaron el primer procedimiento para producir energa elctrica a gran escala. Su etapa de desarrollo abarca el perodo comprendido entre los aos 1830 y 1880. Es necesario hacer notar que la dinamo, aunque es una invencin ms moderna que el alternador, fue desarrollada antes que ste; tngase en cuenta que la pila de Volta data del ao 1800 y que los descubrimientos de Oersted (electromagnetismo) y Faraday (induccin magntica) son de los aos 1820 y 1830, respectivamente. Es lgico, en consecuencia, que la etapa experimental de desarrollo de la dinamo, que comienza en 1830, tienda a buscar un tipo de generador que suministre una forma de corriente similar a la que producan las pilas galvnicas. En aquella poca, la corriente alterna, consecuencia inmediata de la ley de induccin de Faraday, constitua solamente un experimento fsico sin ningn inters prctico. El desarrollo de la mquina de c.c. se centra durante mucho tiempo en la bsqueda de procedimientos que transforman la c.a. inducida en una espira, al girar dentro de un campo magntico, en corriente unidireccional o de polaridad constante (c.c.). La primera idea del conmutador o colector de delgas encargado de la rectificacin mecnica de la tensin del devanado del rotor surge en 1831 y se debe a Pixii. Sin embargo, tuvieron que transcurrir 36 aos ms para que Gramme construyera en 1867una dinamo con inducido en anillo dotada de un colector de delgas como el que actualmente se conoce. Posteriormente surgi la idea del devanado en tambor para aprovechar ms eficazmente el arrollamiento del inducido y se propusieron los diferentes mtodos de autoexcitacin de estas mquinas, de tal forma que en 1886se lleg a una configuracin fsica, cuyos rasgos fundamentales coinciden con los de las mquinas modernas. El desarrollo de los motores de c.c. sigue una lnea histrica paralela a la de las dinamos y su empleo se debe al principio de reciprocidad ya formulado por Faraday y Lenz. El modo de funcionamiento ms caracterstico de las mquinas de c.c. lo constituye su empleo como motor. La ventaja fundamental de los motores de c.c. frente a los motores de c.a. ha sido su mayor grado de flexibilidad para el control de la velocidad y del par, lo cual ha hecho muy interesante su aplicacin en diversos accionamientos industriales: trenes de laminacin, telares, traccin elctrica, etc. Sin embargo, debe destacarse que debido al desarrollo tan espectacular de la electrnica de potencia, su aplicacin incluso en estos campos, en los que mantena su primaca hasta finales del siglo xx, se ha ido reduciendo en pro de los motoresde c.a., cuyo coste de fabricacin y mantenimiento es ms reducido. En el Captulo 7, dedicado a los accionamientos elctricos, se explican los mtodos de control moderno de ambos tipos de motores. Por otro lado, el empleo de la mquina de c.c. como generador o 479

480

MQUINAS ELCTRICAS

dinamo est prcticamente obsoleto debido a que la corriente alterna presenta ms ventajas para la generacin, transporte y distribucin de la energa elctrica que la corriente continua, debido a la sencillez y economa que supone el uso de transformadores para convertir tensiones de un valor a otro. En la actualidad, cuando se necesita c.c. para una aplicacin determinada se recurre al empleo de rectificadores de silicio, que transforman la c.a. de la red en c.c. de forma esttica y con un gran rendimiento.

6.2. ASPECTOS CONSTRUCTIVOS


La mquina de c.c. est constituida por una parte fija o esttor y una parte mvil o rotor. En la Figura 6.1 se muestra un esquema bsico de la forma fsica que presenta este tipo de mquinas. El esttor est formado por la culata (1), que pertenece al circuito magntico inductor y que ejerce la funcin de soporte mecnico del conjunto. En las mquinas de pequea potencia se constriJye de hierro fundido pero en las mquinas grandes se realiza con plancha de acero curvada sobre un molde cilndrico y posteriormente soldada en su base. La culata o carcasa contiene tambin los pies (13) sobre los que se apoyar la mquina. La culata est perforada en diversos puntos de su periferia para fijar los polos, los cuales estn constituidos por los ncleos polares (2), modernamente realizados en chapas de acero convenientemente apiladas sobre las que se coloca el devanado inductor o de excitacin (8). La parte de los polos prxima al rotor presenta una expansin magntica denominada zapata polar (3). Para mejorar la conmutacin, estas mquinas suelen llevar tambin unos polos intermedios (4) y (5), que reciben tambin el nombre de interpolos, polos auxiliares o polos de conmutacin; el devanado de estos polos se conecta en serie con el inducido. El rotor est formado por el inducido (6) y el colector de delgas o conmutador (10). El inducido se construye con discos de chapa de acero al silicio convenientemente ranurado para alojar en l el correspondiente devanado. En la Figura 6.1 el arrollamiento del inducido (7) es del tipo en anillo, que hoy da ya no se emplea por el poco aprovechamiento que se obtiene del cobre. El devanadoen anillo fue el primero que se utiliz en la clebre dinamo de Gramme y tiene la ventaja pedaggica de poder comprender ms claramente el funcionamiento de estas

Figura 6.1. Aspectos constructivos de una mquina de c.c.

CAPTULO 6.

MQUINAS DE CORRIENTE CONTINUA

481

mquinas debido a la facilidad con que pueden observarse sus circuitos. En la actualidad, y como ya se indic en el Captulo 2, los arrollamientos son en tambor, disponiendo ambos lados activos de las espiras en las generatrices externas del apilamiento del rotor. Los devanados de las mquinas de c.c. son cerrados, lo cual indica que el bobinado se cierra sobre s mismo sin principio ni fin. Los devanados pueden ser imbricados y ondulados, dependiendo de si se cruzan o no las partes de la bobina observadas desde el lado del colector. En la Figura 6.2 se muestran ambos tipos de devanados dibujados en perspectiva y en forma desarrollada en inducidos de tambor. Se observa en ambos casos que las bobinas que forman los devanados (imbricado u ondulado) constan de dos lados activos que se sitan debajo de polos de diferente nombre con objeto de obtener la mayor f.e.m. posible. El colector de delgas es el rgano que caracteriza especficamente a estas mquinas * y es el encargado de la conversin mecnica de la c.a. inducida en las bobinas en c.c. de salida. Est formado por lminas de cobre o delgas cuya seccin transversal tiene la forma de cola de milano, como puede observarse en la Figura 6.3. Las delgas estn aisladas entre s y del cubo del colector por medio de un dielctrico de mica. La fijacin del conjunto se consigue merced a la presin que ejercen unos anillos extremos de forma cnica. Para facilitar la soldadura de las conexiones de las bobinas del inducido con el colector, las delgas presentan en un extremo un taln con unas hendiduras convenientemente fresadas para alojar los conductores terminales. Despus de que se conecta el colector al inducido, se procede a la rectificacin de aqul por medio del tomo, hasta dejar su superficie perfectamente cilndrica. La extraccin o suministro de corriente al colector se realiza por medio de escobillas de grafito, aunque modernamente se emplean los tipos electrografticos y metalografticos. Las
B YI LEY: y = YI- Y2

desarrollo

LEY: y

= YI + Y2

desarrollo

Figura 6.2. Devanados imbricado y ondulado.

Se exceptan las mquinas de c.a. con conmutador, que se estudian al final de este captulo.

482

MQUINAS ELCTRICAS

_,_-

Conexin del inducido a la delga

Aislamiento de mica

Tomillo de apriete

Anillos aislantes de mica

Figura 6.3. Seccin de un colector de delgas.

escobillas permanecen inmviles en el espacio, dispuestas en los portaescobillas, y de esta manera, mientras gira el rotor, las escobillas conservan una posicin invariable con respecto a los polos de la mquina. La Figura 6.4 muestra la disposicin tpica de una escobilla alojada dentro del portaescobillas. La presin de la escobilla sobre el colector debe ser del orden de 0,1 a 0,15 Kp/cm', lo que se consigue por la accin de un muelle en espiral cuya presin se ajusta por medio de una palanca dispuesta al efecto. El portaescobillas suele ser de bronce o latn y est unido al collar o soporte por medio de un buln convenientemente aislado. Las escobillas de la misma polaridad van juntas en paralelo al mismo terminal de salida para su conexin al exterior. Para informacin del lector, en la Tabla 6.1 se da una relacin de las caractersticas ms importantes de motores de c.c. para potencias comprendidas entre 18 y 257 kW con una tensin de servicio de 440 V. De acuerdo con la Norma UNE-EN 60034-8, los bornes de las mquinas de c.c. se designan con diferentes letras segn se trate del inducido o de los devanados inductores, de acuerdo con la relacin que se seala a continuacin. Inducido: letras Al y A2; polos auxiliares o de
Muelle

Tomillo de apriete Posiciones de apriete

Portaescobillas

Figura 6.4. Escobilla, portaescobilla y muelle de ajuste de la presin de contacto.

CAPTULO

6.

MQUINAS DE CORRIENTE CONTINUA

483

Tabla 6.1.

Caractersticas de motores de c.c.

18 23 28 35 46 58 70 87 lOO 123 152 176 215 257

91 91 92 92 92,5 92,5 93 93 93,5 93,5 94 94 95 95

45 57 69 86 113 142 171 213 243 299 367 426 514 615

240 260 290 320 370 420 480 530 570 640 720 790 860 920

8,5 7,3 5,4 4,6 3,6 3 2,4 2,1 1,8 1,6 1,2 1,1 1 0,8

0,83 0,94 1,51 1,71 2,08 2,47 4,87 5,41 8,19 9,25 13,54 15,54 23,32 27,29

510 560 650 705 840 910 1.275 1.350 1.550 1.650 2.050 2.380 3.100 3.280

compensacin: letras Bl y B2; devanado de compensacin: letras Cl y C2; devanado inductor en serie: letras Dl y D2; devanado inductor en paralelo o independiente: letras El y E2. La red se identifica con las letras P para el terminal positivo y la N para el terminal negativo.

6.3. PRINCIPIO DE FUNCIONAMIENTO


Como se ha indicado en los prrafos precedentes, la mquina de c.c. puede funcionar tanto en rgimen generador como en rgimen motor. Para comprender el principio de generacin de la f.e.m. en las espiras del rotor, se va a considerar el inducido en forma de anillo que se indica en la Figura 6.5. En este devanado, al girar el rotor, se induce una f.e.m. en los conductores dispuestos en la cara exterior del ncleo al ser cortados por el flujo del esttor. En los conductores interiores no aparece ninguna f.e.m., ya que no les atraviesa el flujo de los polos, al estar sus lneas de fuerza limitadas al circuito de baja reluctancia del anillo. Analizando la figura anterior se observa que el sentido de las f.e.m.s. de los conductores situadosdebajo del polo norte son de signo contrario a la de los conductores situados debajo del polo sur (el sentido de estas f.e.m.s. se obtiene aplicando la conocida expresin: e = (V x B) L a estos conductores). Como quiera que la estructura de la mquina es simtrica, las f.e.m.s. correspondientes a la parte izquierda del inducido sern opuestas a las de la parte derecha, y en consecuencia no circular ninguna corriente por el arrollamiento. Para utilizar la f.e.m. del inducido y llevarla a un circuito exterior se han de conectar unas escobillas de salida A y B, situadas en el eje transversal de los polos para que puedan aprove-

484

MQUINAS ELCTRICAS

EJE TRANSVERSAL O EJE CUADRATURA (Lnea neutra)

Figura 6.5. Mquina de c.c. con inducido en anillo (tipo mquina de Gramme).

char la mxima f.e.m. del devanado. Estas escobillas dividen el arrollamiento en dos ramas en paralelo con una misma f.e.m. En cada una de estas ramas, las f.e.m.s. deben tener el mismo sentido, pues de lo contrario no se utilizaran totalmente las f.e.m.s. generadas en el arrollamiento (como caso lmite, si las escobillas se disponen bajo los centros de los polos, la tensin entre ellos sera nula). El eje que forma la alineacin de las escobillas se denomina lnea neutra. Esta lnea tiene gran importancia, pues indica las posiciones en las que se produce la inversin de f.e.m. en las bobinas del inducido pasando las espiras correspondientes de una rama paralelo a la otra. Nota prctica: La posicin exacta de la lnea neutra se determina experimentalmente moviendo el collar de las escobillas hasta encontrar el punto en el que se producen las mnimas chispas en el colector de delgas. En los inducidos en anillo y tambin en los que llevan un devanado imbricado simple el nmero de circuitos derivados coincide con el de polos. Si se denomina 2p el nmero de polos y 2e el nmero de circuitos derivados o ramas en paralelo, se tendr: IMBRICADO SIMPLE 2e = 2p
(6.1)

En los devanados ondulados simples, el nmero de circuitos derivados es siempre igual a 2, sea cual sea el nmero de polos de la mquina, es decir: ONDULADO SIMPLE 2e = 2
(6.2)

Existen tambin devanados imbricados mltiples y ondulados en series paralelas (de Arnold) en los que el nmero de circuitos derivados sigue leyes diferentes a las anteriores, pero que no se considerarn en este texto. Para calcular la f.e.m. producida en el inducido de una mquina de c.c. debe tenerse en cuenta que en cada bobina del arrollamiento se obtiene una f.e.m. alterna, de tal forma que en un semiperiodo de la misma el flujo concatenado vara entre los lmites +<1> y -<1> (flujo de los

CAPTULO

6.

MQUINAS DE CORRIENTE CONTINUA

485

polos); en consecuencia, el valor medio de la f.e.m. obtenida en la bobina en el tiempo mencionado ser: . 2 1+<1> 4<l> E med = - d<l> =T T
-<1>

(6.3)

donde T indica el periodo de la corriente. Como quiera que la frecuencia de la tensin generada va ligada al nmero de polos 2p y a la velocidad de rotacin n en r.p.m. por la relacin:
np

f= 60 la f.e.m. media en una espira del inducido ser:


E
med

(6.4)

= 4<l> pn 60

(6.5)

Como quiera que las escobillas de la mquina recogen las f.e.m.s. inducidas en las distintas bobinas durante un semiperiodo, la f.e.m. resultante en el inducido ser igual a la suma de las f.e.m.s. medias de las distintas bobinas que componen cada rama en paralelo del devanado. Si ste consta de Z conductores que forman Z/2 bobinas y est dividido por las escobillas en 2e circuitos derivados, se tendrn en cada rama Z/4e bobinas conectadas en serie, que teniendo en cuenta la relacin (6.5) producirn una f.e.m. resultante de magnitud: E

=-

Z<l> 60 e

= KEn<l>

(6.6)

en la cual KE = !:_ l!.. es una constante determinada para cada mquina. De esta expresin se 60 e deduce que la f.e.m. puede regularse variando la velocidad del rotor o cambiando el flujo inductor mediante el ajuste de la corriente de excitacin de los polos. En los generadores o dinamos esta f.e.m. se obtiene como consecuencia del movimiento del rotor por la accin de una energa mecnica de entrada y puede aprovecharse en un circuito exterior conectando una carga elctrica que har circular corriente por el inducido. En los motores, el giro de la mquina es el resultado de la interaccin del flujo del inductor con las corrientes del inducido al conectar ste a una red de c.c., lo que provoca una f.e.m. de reaccin en el rotor que se opone al sentido de la corriente y que recibe por ello el nombre de fuerza contraelectromotriz (f.c.e.m.). En cualquier caso, funcione la mquina de c.c. como generador o como motor, el paso de una corriente continua por los conductores del inducido provoca en el rotor un par electromagntico que tiene carcter resistente para el trabajo como generador y carcter motor cuando la mquina mueve una carga mecnica (es decir, cuando funciona como motor de c.c.). Para calcular la magnitud de este par debe tenerse en cuenta que si es I, la corriente total del inducido, la corriente que circula por los conductores del rotor en una mquina con 2e circuitos derivados ser 1/2e. Si denominamos Bmed el valor medio de la induccin de los polos en los conductores del inducido y L a la longitud de los mismos, la fuerza media resultante en cada conductor, de acuerdo con la ley de Laplace, ser:
(6.7)

486

MQUINAS ELCTRICAS

y que tiene sentido tangencial al rotor. Si se denomina R al radio del rotor y Z al nmero de conductores del inducido, se obtendr un par resultante de magnitud: (6.8) Como quiera que la superficie del inducido comprendida en el paso polar de la mquina es igual a:
S.=--

2nRL 2p

el valor del flujo por polo ser:


<1>=B
med

--

2nRL 2p

(6.9)

expresin que llevada a (6.8) determina el par de la mquina en funcin del flujo por polo:
T=-

2n e

I p - Z<1>1 = KT1<1>

'

(6.10)*

donde KT = !:.... '!_ es una constante determinada para cada mquina. 2n e Si se tiene en cuenta (6.6), la ecuacin anterior puede expresarse en funcin de la f.e.m. del inducido, resultando:
El T=--'-

2n-

(N.m)

(6.lla)

60

Este par ser resistente en el caso de transformacin de energa mecnica en elctrica, es decir, en un generador, y de rotacin en el caso de un motor, es decir, cuando se transforma energa elctrica en mecnica. En cualquier caso, el numerador de (6.lla) representa la potencia electromagntica que se aplica a la mquina (en rgimen generador) o que se extrae de ella (rgimen motor), y que es en definitiva el producto del par por la velocidad angular, es decir: Pa,= El

= Tw = T . 2n 60

(W)

(6.llb)

Para comprender mejor el fenmeno de produccin de par en las mquinas de c.c., en la Figura 6.6 se ha representado una mquina de c.c. funcionando como generador. La mquina es movida por medio de un motor primario en sentido contrario a las agujas del reloj, y se *

Es interesante observar al comparar (6.10) con (6.6) que la relacin entre las constantes K,y KE es: 2n

K=-K
t:

60

1
(j)

De hecho, si se expresa (6.6) en funcin de la velocidad angular

(en rad/s) resulta:

Z p 271/1 E =- - <1> = K .(1)<1>

2n e 60

7'

que utiliza la misma Kr que la ecuacin del par (6.10): T = KT1;<1>.

CAPTULO

6.

MQUINAS DE CORRIENTE CONTINUA

487

Par electromagntico

t
t

+
E R

Figura 6.6. Mquina de c.c. funcionando como generador.

producen entonces unas f.e.m.s. en los conductores cuyos sentidos se muestran en la misma figura. Al conectar una resistencia de carga entre las escobillas aparecen uas corrientes de circulacin en los conductores del inducido, que al reaccionar con el campo magntico inductor provocan un par electromagntico que se opone a la rotacin y que por tanto tiene carcter resistente respecto a la accin del motor primario. Para mantener la velocidad de la dinamo, el par del motor primario ha de ser suficiente para equilibrar este par resistente, junto con el par de prdidas, debido a los efectos de rozamiento, ventilacin, etc. Cuando la mquina de c.c. funciona como motor, se debe aplicar una tensin de c.c. de alimentacin al inducido, que provoca una corriente de circulacin por los conductores de este devanado, cuyos sentidos se muestran en el esquema de la Figura 6.7. La interaccin de estas corrientes con el flujo inductor origina un par de rotacin en sentido contrario a las agujas del reloj que obliga a girar a la mquina. Esto conduce a la aparicin de una f.e.m. en el inducido cuyo sentido es idntico al estudiado para el caso de la dinamo, puesto que la rotacin en ambos casos es coincidente. En consecuencia, la f.e.m. engendrada se opone a la corriente que circula por los conductores y por ello recibe el nombre de fuerza contraelectromotriz: El movimiento del motor se

Ji
ti

Par electromagntico

Figura 6.7. Mquina de c.c. funcionando como motor.

488

MQUINAS ELCTRICAS

mantendr en tanto que el par electromagntico de rotacin producido sea superior al par resistente ejercido por la carga mecnica conectada al rbol de la mquina. EJEMPLO DE APLICACIN 6.1 Unamquina de c.c. de 8 polos tiene un inducido bobinado con un arrollamiento imbricado de 960 conductores. El flujo por polo es de 40 mWb y la velocidad de rotacin es de 400 r.p.m. Calcular: a) f.e.m. generada; b) si el arrollamientofuera ondulado, a qu velocidad debera girar el rotor para obtener una f.e.m. igual a 400 V?; c) cul sera el par electromagntico desarrolladopor la mquina en el caso anterior para una corriente del inducido de 25 A? SOLUCIN
a)

En el caso de que el arrollamiento sea imbricado, el nmero de circuitos derivados coincide con el de polos. Al sustituir valores en la expresin (6.6) se obtiene: 400 4 E =960 . O 40 - = 256 V 60 '4

b)

Cuando el arrollamiento es ondulado, el nmero de circuitos derivados es igual a 2, y al aplicar (6.6) resulta: 400

=-

n 4 960 . 0,04 - = 2,56n 60 1

c)

de donde se deduce que la velocidad de giro debe ser de 156,25 r.p.m. De acuerdo con la expresin (6.10), se obtiene:
T

=- -

1 4

2n 1

960 0,04 . 25 = 611,15 N.m

6.4.

REACCIN DE INDUCIDO

Cuando una mquina de c.c. funciona en vaco (trabajando, por ejemplo, como generador), no existe corriente en el inducido y el flujo en el entrehierro est producido nicamente por la f.m.m. del inductor. Cuando se cierra el circuito del inducido sobre una resistencia de carga aparece una corriente de circulacin por los conductores del rotor que dan lugar a una f.m.m. que combinada con la del esttor producen el flujo resultante en el entrehierro de la mquina. Se conoce con el nombre de reaccin del inducido al efecto que ejerce la f.m.m. de este devanado sobre la f.m.m. del inductor, y que hace variar la forma y magnitud del flujo en el entrehierro respecto a los valores que la mquina presentaba en vaco. Para estudiar el fenmeno de la reaccin del inducido se va a considerar, por simplicidad, una mquina bipolar (Fig. 6.8) trabajando en la zona no saturada de su caracterstica magntica *. De esta forma, podr aplicarse el principio de superposicin tanto a los diagramas de f.m.m. como a los que representan las distribuciones de flujo magntico en el entrehierro. Dicho de otra manera, en vez de combinar las f.m.m.s. del inductor y del inducido para obtener el flujo resultante, se

* Por razones de sencillez, en la Figura 6.8 se ha representado el inducido sin colector de delgas, con las escobillas apoyadas directamente sobre los conductores del rotor.

CAPTULO

6.

MQUINAS DE CORRIENTE CONTINUA

489

Rotacin

Figura 6.8.

Reaccin de inducido en una mquina de c.c.

obtendr ste sumando las distribuciones del campo magntico que produce cada f.m.m. actuando independientemente. Cuando la mquina trabaja en vaco, solamente acta la excitacin de los polos, de esta forma se obtiene una distribucin de campo magntico en el entrehierro que es constante y mxima debajo de cada polo y que decrece rpidamente en el espacio interpolar hasta hacerse cero en la lnea neutra. En la Figura 6.9 se muestra un esquema desarrollado de la mquina de la Figura 6.8 con la distribucin de la induccin magntica correspondiente en vaco (sin corriente en el inducido). Tericamente la curva anterior debiera tener una amplitud constante y de diferente signo debajo de cada polo y ser nula en el espacio interpolar, pero esto no es as debido a los flujos de dispersin que aparecen en los cuernos polares, que hacen que la forma de Bia) sea en la prctica de forma trapezoidal, tal como se muestra en la Figura 6.9. Al cerrar el circuito del inducido, se originan unas corrientes que producen una f.m.m. de forma triangular, como puede demostrarse analticamente aplicando el teorema de Ampre a recintos cerrados cuya anchura sea igual a un paso polar (vase Cap. 2, problema 2.6 del final del captulo). El eje de esta f.m.m. coincide con el de la lnea de escobillas, de tal forma que si stas se disponen en la lnea neutra, la f.m.m. del inducido ser mxima en esta lnea interpolar; en consecuencia, la f.m.m. de reaccin de inducido tiene carcter transversal respecto a la f.m.m. del inductor (vase Fig. 6.8).

Figura 6.9. Distribucin de la induccin magntica en el entrehierro producida por los polos.

490

MQUINAS ELCTRICAS

En la Figura 6.10 se representa la distribucin de la f.m.m. del inducido sobre la periferia del entrehierro. El esquema corresponde al desarrollo de la mquina bipolar de la Figura 6.8, en la que ahora se muestran los polos a trazos para indicar que su accin no se tiene en cuenta. Esta distribucin se puede intuir fcilmente observando que desde el punto de vista magntico se puede considerar que los conductores estn conectados entre s tal como se muestra con las lneas de trazos de la Figura 6.10. De esta manera los conductores situados entre los ejes verticales CC, y DD, actan como si se tratara de bobinas concntricas que producen una f.m.m. que tiene su mximo sobre el eje AA,. De forma similar, las corrientes en los conductores situados a la izquierda de CC, ya la derecha de DD, producen una f.m.m. que tiene su mximo sobre el eje BB,. Con relacin a esta distribucin de f.m.m, se tendr una correspondientedistribucin radial de la induccin, cuyo diagrama se ha representado tambin en la Figura 6.10. Se observa que esta curva B(a) presenta unas depresiones en los espacios interpolares debido a que en estas zonas la reluctancia es mucho mayor que debajo de los polos por existir un entrehierro mayor. Para considerar el efecto de la reaccin del inducido sobre la distribucin del flujo inductor, habr que superponer las curvas de las Figuras 6.9 y 6.10, resultando el diagrama de la Figura 6.11. Examinando la induccin resultante en el entrehierro, se pueden deducir importantes consecuencias. En primer lugar, la reaccin del inducido deforma la curva de induccin debajo de cada polo, reforzando el campo en un lado del polo y debilitndolo en el otro, es decir, se forma exactamente el mismo campo transversal del inducido que en el caso de la mquina sncrona con carga resistiva. Si la mquina no est saturada, esta magnetizacin transversal no modifica su f.e.m., ya que el flujo se conserva constante; sin embargo, si existe saturacin, la B resultante tiene un valor inferior al previsto en las salidas de los polos, lo que hace que el flujo total disminuya y aparezca un efecto desmagnetizante que reduce el valor de la f.e.m. de salida. Otro efecto a considerar en esta situacin es la posible elevacin de tensin entre delgas consecutivas motivada por el paso de las espiras correspondientes por la zona de refuerzo del flujo de los polos, lo que puede conducir a un chisporroteo en el colector. Por ltimo, otra consecuencia tambin importante que se deduce claramente de la Figura 6.11 es el desplazamiento que sufre la lnea neutra debido a la reaccin del inducido; cuando la mquina trabaja en vaco, la lnea neutra magntica coincide con la lnea neutra

Figura 6.10. F.m.m. en el entrehierro e induccin producida por el inducido.

CAPTULO

6.

MQUINAS DE CORRIENTE CONTINUA

491

Figura 6.11.

Deformacindel campo magntico en el entrehierro debido a la reaccin del inducido.

geomtrica o lnea media entre polos (punto N); sin embargo, cuando circula corriente por el inducido y estando funcionando la mquina como generador (caso de la Fig. 6.11), la lnea neutra magntica se adelanta un ngulo O (punto M) respecto del sentido de giro del rotor. En el caso del funcionamiento como motor, un razonamiento anlogo indicara que la lnea neutra magntica se retrasa respecto a la geomtrica. Este desplazamiento de la lnea neutra magntica lleva consigo un fuerte chisporroteo en el colector, ya que durante la conmutacin, la escobilla correspondiente pondr en cortocircuito una seccin del devanado en la cual se induce una cierta f.e.m. por existir flujo en esa zona. Para evitar este fenmeno habr que desplazar las escobillas hasta encontrar la lnea neutra real, es decir, en el esquema de la Figura 6.11 habr que pasar la escobilla de la posicin N a la M (adelantando las escobillas cuando la mquina funciona como generador y retrasndolas cuando trabaja como motor). En la Figura 6.12 se muestra ms claramente esta operacin. Hay que hacer notar que todos los conductores que se encuentran a la izquierda del eje de escobillas CD llevan corriente saliente al plano de la pgina, mientras que todos los situados a

Figura 6.12.

Desplazamiento de las escobillas hasta la lnea neutra real.

492

MQUINAS ELCTRICAS

su derecha llevan corriente entrante. De esta forma, y como se ha indicado en los prrafos anteriores, se producir una f.m.m. de reaccin de inducido F que coincide con el eje de escobillas, que puede descomponerse en dos partes: una longitudinal o de eje directo Fd que tiene carcter desmagnetizante o antagonista, pues se opone a la f.m.m. de excitacin Fe' y que puede considerarse que est producida por los conductores comprendidos en el ngulo 2e (grados elctricos), como se muestra en la Figura 6.13a, y otra componente transversal F, producida por los dems conductores agrupados, como se indica en la Figura 6.13b, abarcando (180o-2e) grados elctricos. Para calcular la magnitud de ambas f.m.m.s. se va a considerar un inducido de Z conductores distribuidos en 2e circuitos derivados que transportan una corriente total l, teniendo la mquina 2p polos. En estas circunstancia se tiene: Corriente/conductor Conductores/polo =

t,
2e
Z 2p I

Amperio-conductores/polo = - 2e 2p rivueltas por polo ser: Amperivueltas/polo 11 Z rz =_ -'--_ = _,_ 2 2e 2p


0

(6.12)

y como hacen falta dos conductores del inducido para formar una espira, el nmero de ampe-

8pe

(6.13)

Como cada polo ocupa una extensin de 180 y hay solo 2(}grados elctricos ocupados por conductores responsables de la reaccin desmagnetizante, la magnitud de la f.m.m. Fd correspondiente ser:

.97,;=---

IZ

2e

8pe 180"

[A.v/polo]

(6.14)

a) Antagonista

b) Transversal

Figura 6.13.

F.m.m.s. antagonista y transversal.

CAPTULO

6.

MQUINAS DE CORRIENTE CONTINUA

493

De forma anloga, la f.m.m. transversal est producida por los conductores comprendidos en un ngulo 180-20, resultando un valor de F,: (6.15) En resumen, y concretando los prrafos anteriores, se puede decir que: a) Cuando las escobillas estn situadas en la lnea neutra geomtrica, la reaccin del inducido es totalmente transversal y sufm.m. viene expresada por la relacin (6.13), lo que conduce a un desplazamiento de la lnea neutra magntica que provoca un chispeo en el colector. b) Si se desplazan las escobillasa la lnea neutra magntica verdadera,se evita el chisporroteo del colector pero aparece una reaccin antagonista, definida por (6.14), que se opone a la accin del inductor y que debe ser compensada por un aumento idntico en lafm.m. de los polos.

Como quiera que la reaccin del inducido es proporcional a la corriente de carga, el desplazamiento de las escobillas debera ser variable con el rgimen de carga de la mquina. Esto supondra una operacin de gran complejidad que los constructores han intentado evitar. Por ello, en la prctica, en las mquinas de mediana y gran potencia se impide el desplazamiento de la lnea neutra atenuando el efecto de la reaccin transversal. La solucin ms eficaz consiste en neutralizar la reaccin del inducido a lo largo de toda la periferia de ste, mediante la incorporacin de un arrollamiento de compensacin. Para ello, en las extremidades polares se practican paralelamente al eje de la mquina unas ranuras en las cuales se colocan unos conductores dispuestos en serie con el circuito exterior, de tal forma que la corriente circule en ellos en sentido opuesto a la de los conductores del inducido que estn debajo. En la Figura 6.14 se muestra un esquema de este tipo que contiene 4 conductores por polo, que casi compensan el efecto de reaccin transversal del inducido. En el caso ideal habr que incorporar en el devanado de compensacin tantos conductores como existan en el inducido y de esta forma la reaccin transversal total de la mquina ser nula y no habr desplazamiento de la lnea neutra. Como quiera que estos arrollamientos elevan considerablemente el costo de una mquina, aumentando tambin las prdidas en el cobre, solamente se emplean en las mquinas de potencia elevada que tengan que soportar fluctuaciones bruscas de carga.

Figura 6.14. Devanadode compensacin.

494

MQUINAS EL.:cTRICAS

En la mayora de las mquinas de c.c., para eliminar el desplazamiento de la lnea neutra geomtrica con las variaciones de carga y asegurar una mejor conmutacin, se emplean los llamados polos auxiliares, interpolos o polos de conmutacin, que son pequeos ncleos magnticos que se colocan en la lnea neutra terica, que van provistos de un devanado que se conecta en serie con el inducido, y que producen un campo magntico opuesto al de la reaccin transversal. En la Figura 6.15 se muestra una mquina bipolar que lleva colocados los interpolos correspondientes. En el caso a) la mquina funciona como generador yen el caso h) la mquina funciona como motor. En ambas situaciones se ha mantenido el mismo sentido de giro del rotor (contrario a las agujas del reloj) y la misma polaridad de los polos principales; sin embargo, el sentido de la corriente en el inducido en ambasfiguras es opuesto. Si se considera el comportamiento como generador (Fig. 6.15a), el sentido de giro lo impone el motor primario externo que mueve el rotor; las corrientes en el inducido se determinan con la ley de Faraday: e = (v x B) . 1 (que corresponde a un sentido de la corriente definido por la regla de Fleming de la mano derecha), que da lugar a corrientes salientes al plano de la pgina en los conductores situados en la semicircunferencia superior y corrientes entrantes en los conductores situados en la semicircunferencia inferior. Es por ello que la f.m.m. de reaccin de inducido es transversal y se dirige de izquierda a derecha. Como quiera que los interpolos deben producir f.m.m.s. de sentido contrario al anterior, el sentido de las corrientes en estos polos auxiliares (yen este caso de funcionamiento como generador) debe ser el sealado en la Figura 6.15a. Obsrvese que en el funcionamiento como generador la

polaridad magntica de un interpolo es la misma que la del polo principal que le precede en el sentido de rotacin de la mquina (Fig. 6.150).
Si se considera el comportamiento como motor (Fig. 6.15b), el sentido de giro es debido a la reaccin de las corrientes del inducido con el flujo inductor de los polos y que viene expresado por la ley de Laplace: F = (i x 1) . B (que corresponde a un sentido de la fuerza en los conductores definido por la regla de Fleming de la mano izquierda); como quiera que se

_1;

-1,

Fuente

de e.e.
b)

a)

Generador

Motor

Figura 6.15.

Mquina de c.c. con polos auxiliares o de conmutacin.

CAPTULO

6. MQUINAS DE CORRIENTE CONTINUA

495

ha invertido la corriente en el inducido respecto a la que llevaba la mquina cuando funcionaba como generador (comparar Figs. 6.15a y b), el sentido de giro del rotor es el mismo en ambos casos. De este modo en el comportamiento como motor (Fig. 6.15b) la f.m.m. de reaccin de inducido es transversal y dirigida de derecha a izquierda. Como quiera que los interpolos deben producir f.m.m.s. de sentido contrario al anterior, el sentido de las corrientes en estos polos auxiliares (en el caso de funcionamiento como motor) debe ser el sealado en la Figura 6.15b. Obsrvese que en el funcionamiento como motor la polaridad magntica de
un interpolo es la misma que la del polo principal que le sigue en el sentido de rotacin de la mquina (Fig. 6.15b).

Debe tenerse en cuenta que, como ya se ha indicado antes, los interpolos deben ir en serie con el inducido para que su accin sea proporcional a la corriente Ji y compensar de este modo la reaccin transversal del inducido; adems, para que el flujo producido por los interpolos guarde la proporcionalidad con la corriente es preciso que el circuito magntico trabaje en la zona lineal, lo que requiere un mayor entrehierro interpolo-inducido que el de los polos principales. Por otra parte, hay que sealar tambin, y como ya se indicar en el epgrafe 6.5, que la misin de los polos auxiliares es tambin otra, como es producir una induccin adicional en la lnea neutra para que se genere una f.e.m. de conmutacin que evite el chisporroteo en el colector de delgas.
EJEMPLO DE APLICACIN 6.2

Determinar el nmero por polo de: a) amperivueltas transversales, b) amperivueltas antagonistas o de desmagnetizacin, y e) las espiras en serie para equilibrar los amperivueltas antagonistas en el caso de una dinamo de 500 conductores, 6 polos, devanado ondulado, que lleva una corriente total de inducido de 200 A. El ngulo geomtrico de adelanto de escobillas es de 10 y el coeficiente de dispersin o de Hopkinson es igual a 1,3. NOTA: El coeficiente de Hopkinson define la relacin entre el flujo que produce un polo y el flujo que llega al inducido.

SOLUCIN Como la mquina tiene 6 polos, el ngulo elctrico de adelanto de escobillas valdr:

o = p = 3 . 10 = 30
a)

La expresin (6.15) da la magnitud de la f.m.m. de reaccin transversal, y teniendo en cuenta que para un devanado ondulado se tiene 2c = 2, resulta:

t Z ( 1 - --20) = 200 . 500 ( 1 - -2 . 300) = 2.778 A.v/polo .7 = -'r 8pc 180 8.3. 1 180
b)

Para calcular la f.m.m desmagnetizante habr que aplicar la expresin (6.14) :


IZ 20 200 . 500 2 . 30 .;;; = -= = 1.389 A.v/polo 8pc 1800 8 . 3 . 1 1800

e)

Los amperivueltas por polo que habr que aadir al inductor para compensar la f.m.m. antagonista, teniendo en cuenta el coeficiente de dispersin, sern:

.~ =

1.389 1,3 = 1.805 A.v/polo

496

MQUINAS ELCTRICAS

Como las espiras necesarias se van a colocar en serie con el inducido, habr que poner en cada polo: N

= - = -1

.::;:;; 1.805 200

= 9 espiras

6.5.

CONMUTACIN

Se entiende por conmutacin el conjunto de fenmenos vinculados con la variacin de corriente en las espiras del inducido al pasar stas por la zona donde se las cierra en cortocircuito por las escobillas colocadas en el colector. Una buena conmutacin debe realizarse sin la formacin de chispas en el colector, mientras que una mala conmutacin, concurrente con la formacin de chispas, produce, para un trabajo prolongado de la mquina, un deterioro notable de la superficie del colector que perturba el buen funcionamiento de la mquina. El chisporroteo entre las escobillas y el colector obedece a causas mecnicas y elctricas. Entre las primeras figuran: defectuoso ajuste de las escobillas con el colector, resalte de algunas delgas, insuficiente equilibrado del rotor, etc.; todos estos factores empeoran el contacto entre las escobillas y el colector. La causa elctrica fundamental del chisporroteo la constituye la elevacin de la tensin entre delgas adyacentes del colector, que, en especial, puede ser provocada por los fenmenos de autoinduccin de las secciones del arrollamiento del inducido. Debido a la gran cantidad de factores que intervienen en el proceso de la conmutacin, resulta muy difcil hacer un anlisis riguroso y matemtico del mismo, y por ello nos limitaremos aqu a lo esencial para llegar a un comprensin sucinta del fenmeno. La teora que se va a exponer se basa en la hiptesis de considerar un colector mecnicamente perfecto con un devanado del inducido en el que se van a omitir en principio las f.e.m.s. inducidas en las espiras conmutadas y donde se van a despreciar las resistencias de las mismas y de sus hilos de conexin a las delgas frente a la resistencia de contacto escobilla-colector. En la Figura 6.16 se muestra el proceso de conmutacin de una seccin e del inducido de una mquina de c.c. En el instante inicial, representado por la posicin a), la corriente I, de salida de la escobilla se toma de la delga 3; la corriente en la seccin e considerada es 1/2 y tiene sentido de derecha a izquierda. En un instante intermedio (posicin b) la seccin e est pasando por la lnea neutra y debe invertir su sentido, apareciendo el reparto de corrientes que se indica en la misma figura. El momento en que la comente de la bobina es nula coincide precisamente con media duracin de la conmutacin. El final de la misma se obtiene cuando la escobilla deja de hacer contacto con la delga 3 (posicin e), en cuyo instante la corriente en la seccin e se ha invertido y vuelve a tomar el valor de partida 1/2. El intervalo de tiempo necesario para la conmutacin de la seccin se denomina perodo T de conmutacin. Si se denomina Re a la resistencia de contacto de la escobilla con la delga cuando estn totalmente unidas (en toda su superficie), se observa en la Figura 6.16 que en el instante inicial t = O,la resistencia de transicin de contacto es igual a R,.. Conforme se aleja paulatinamente la escobilla de la delga 3, disminuye su superficie de contacto proporcionalmente al tiempo t transcurrido desde el instante en que comenz la conmutacin, y siendo la resistencia de contacto inversamente proporcional a esa superficie, la resistencia de transicin llegar a ser infinita al final del perodo T de conmutacin. Si se denomina R a la resistencia de transicin entre la delga 3 y la escobilla, se cumplir: R = R T
C-

T
-t

(6.16)

CAPTULO

6.

MQUINAS DE CORRIENTE CONTINUA

497

a ) Instante inicial t = O

b) Instante intermedio t

e) Instante final t

=T

Figura 6.16.

Proceso de conmutacin en una seccin del inducido.

Razonando de forma anloga, la resistencia R2, de transicin entre la delga 4 y la escobilla, disminuye de manera inversamente proporcional al tiempo t: R2 = R e

T
t

(6.17)

Si se tiene en cuenta que en las hiptesis de partida se desprecian las resistencias de las espirasconmutadas y sus hilos de conexin y se consideran nulas las posibles f.e.m.s. inducidasen la seccin e, al aplicar los lemas de Kirchhoff al circuito de la Figura 6.16 se obtiene: (6.18) de donde resulta:
T-t il =].-I t y en consecuencia la corriente i en la seccin conmutada vendr expresada por: i = il ~

(6.19)

=~

(1 - 2 ~)

(6.20)

498

MQUINAS ELCTRICAS

que es la ecuacin de una recta. En la Figura 6.17 se muestra el diagrama de variacin de esta corriente, que recibe el nombre de conmutacin rectilnea o lineal. En la prctica la conmutacin lineal no se da nunca; esto es debido a que es inevitable la aparicin de una f.e.m. de autoinduccin y de induccin mutua en la seccin conmutada por la variacin de corriente existente en la propia seccin y en la adyacente (o adyacentes cuando las escobillas tienen contacto con dos o ms delgas). Esta f.e.m., denominada fuerza electromotriz reactiva e.; tiene la forma general:
d<l> e = -N-r dt

(6.21)

donde N representa el nmero de espiras de la seccin conmutada y <l> es el flujo total que atraviesa a la seccin objeto de estudio, que en parte proviene de su propio flujo y que en parte se debe a los flujos que llegan de las otras secciones conmutadas. Si se consideran lineales las variaciones de las corrientes, la f.e.m. er tendr un valor proporcional a la corriente I, *; en consecuencia, para una determinada corriente de inducido, la magnitud de e, ser constante en el perodo de conmutacin, como as se ha representado en la Figura 6.17 por medio de una paralela al eje de abscisas. Esta f.e.m. e., al oponerse, segn la ley de Lenz, a la causa que la produce, retardar el proceso de conmutacin, lo que lleva consigo el que la corriente i pasar por cero en un tiempo superior al terico T /2. En la Figura 6.17 se ha representado este fenmeno por una curva de trazo delgado. Como quiera que el tiempo total de la conmutacin debe ser constante e igual a T, el paso de la corriente de Oa -//2 deber ser ms forzado y rpido que en el caso lineal, lo que indica que la magnitud de e, experimenta un aumento. Adems, si se tiene en cuenta que la superficie de contacto delga-escobilla est disminuyendo, se producir una alta densidad de corriente a la salida de la delga que se aleja, lo que da lugar al chisporroteo en el colector. Para evitar el chisporroteo originado por la aparicin de e r' ser necesario compensar esta f.e.m. por otra de signo opuesto e igual magnitud (por tanto, proporcional tambin a la corriente 1); esto se consigue incorporando en la zona de conmutacin (lnea neutra) un polo
e

----------------------------i----- ._-1/2 Retardada

e,

.........

1.j_1l~!...

1/2

1=0

t=

Figura 6.17.

Variacin de la corriente en la conmutacin.

di 1 Por ejemplo, la f.e.m. propia de autoinduccin ser de la forma: e = -L - = -L .: = -K, l, , dt T '

CAPTULO 6.

MQUINAS DE CORRIENTE CONTINUA

499

auxiliar que cree una determinada induccin Be en la seccin conmutada. De esta forma, si N es el nmero de espiras de la seccin, L la longitud de sus conductores (o la longitud del polo auxiliar, si la de ste es menor) y v la velocidad tangencial de los mismos, la f.e.m. adicional que resulte, denominada fuerza electromotriz de conmutacin ec' tendr una magnitud: e,. = Bc2LvN (6.22)

Como el signo de esta f.e.m. ec debe ser opuesto a e, (vase Fig. 6.17), la polaridad del polo de conmutacin en el caso de un generador ser, naturalmente, opuesta a la del polo del cual viene la bobina. Para que la magnitud de e( coincida con la de e, tendrn que variar ambas con la misma ley; como quiera que e, es proporcional a la corriente del inducido [j, habr que hacer que el' Ypor tanto Be sea proporcional tambin a esa corriente; esto se logra, para todos los regmenes de funcionamiento, conectando el arrollamiento de los polos auxiliares en serie con el devanado del inducido. Adems, para lograr una relacin directa entre la f.rn.m. de los polos auxiliares y la induccin Be correspondiente ser necesario que el circuito magntico de estos polos est no saturado; esto se logra aumentando el entrehierro de los mismos o tambin trabajando con inducciones bajas (inferiores a 1,2 Teslas). En la Figura 6.18 se muestra la disposicin bsica de una mquina de c.c. en la que se ha intercalado un polo auxiliar o de conmutacin (normalmente hay tantos como polos principales). Obsrves~ que el arrollamiento del interpolo va en serie con el inducido, con un entrehierro mayor que el de los polos principales. Para calcular el nmero de espiras necesario en los interpolas hay que tener en cuenta que ellos deben realizar una doble misin: por una parte, y de acuerdo con los prrafos anteriores, tienen que producir una densidad de flujo Be en las espiras conmutadas que engendre una f.e.m. de conmutacin el' que se oponga a la f.e.m. de reactancia er; pero adems, y de acuerdo con el epgrafe 6.4, deben contrarrestar la reaccin transversal del inducido, que tiende a modificar la posicin de la lnea neutra. Como quiera que ambas acciones conducen a polaridades coincidentes (como puede comprobarse fcilmente comparando los esquemas de las Figs. 6.15 y 6.18), la f.m.m. necesaria en los polos auxiliares, .7;;ux' ser igual a la suma de la

Figura 6.18.

Detalle de la conexin inducido-polo auxiliar o de conmutacin.

500

MQUINAS ELCTRICAS

f.m.m. de reaccin transversal, .:;;; ms la f.m.m. necesaria para crear Be' Yque denominaremos Ys; por tanto: (6.23) Si no hay desplazamiento de escobillas toda la reaccin del inducido es transversal; en consecuencia, y de acuerdo con (6.13), se obtiene: IZ 9'= _'8pc
I

A.v/polo

(6.24)

Para calcular ,iTa supondremos que el entrehierro de los polos auxiliares baux es elevado; de esta forma se podr despreciar la reluctancia del hierro frente a la del entrehierro, resultando:

.~ = H

baux =

Be

baux

(6.25)

Po

donde HB indica la intensidad de campo magntico en el entrehierro de los polos auxiliares. Llevando (6.24) y (6.25) a (6.23) queda:
,9;;"ux = _,

8pc

Z + ----"baux
Po

A.v/polo

(6.26)

Generalmente g;;;ux es del orden de 1,2 a 1,3 veces ,7,Si Naux son las espiras por polo de los arrollamientos correspondientes que estn recorridos por la corriente I, del inducido, resulta: Naux =-i~ux

t,

(6.27)

que determina el nmero de espiras por polo necesario en los polos auxiliares. EJEMPLO DE APLICACIN 6.3. Un generador de c.c. de 6 polos, con un arrollamiento imbricado de 720 conductores, suministra una corriente de 600 A. La densidad deflujo en el entrehierro de los polos auxiliares es de 0,372 Wb/m2 y el entrehierro correspondiente es de 6 mm. Despreciando la reluctancia del hierro y la dispersin deflujo magntico, calcular el nmero (porpolo) de espiras necesarias en los polos auxiliares. SOLUCIN De acuerdo con la expresin (6.26) se tiene:
7;,ux

= _' Z + ----"baux 8pc Po

A.v/polo

y al sustituir valores resulta: ,


"~ux

600 = --780 8.3.3

0,372 -3 7 6 . 10 4n . 10-

= 8.276

A.v/polo

CAPTULO

6.

MQUINAS DE CORRIENTE CONTINUA

501

puesto que en un arrollamiento imbricado se cumple que 2p = 2e. En consecuencia, el nmero de espiras necesario ser:
N .. = -U/_

8.276 = 14 espiras 600

6.6. GENERADORES DE C.C.: ASPECTOS GENERALES


Los generadores de c.c. o dinamos convierten una energa mecnica de entrada en energa elctrica de salida en forma de corriente continua. En la actualidad, estos generadores han cado en desuso y han sido sustituidos por rectificadores, generalmente de silicio, que transforman c.a. de la red en C.C., en forma esttica y con mayor rendimiento. Conviene, sin embargo, analizar el funcionamiento bsico de las dinamos para comprender claramente el comportamiento de los motores de C.C., cuyo estudio se realiza en el epgrafe 6.7. Desde el punto de vista de circuito elctrico, las mquinas de c.c. constan de un inductor o excitacin,colocado en el esttor, y de un inducido giratorio provisto de colector de delgas. La representacin de ambos devanados es la indicada en la Figura 6.19. El devanado de excitacin est formado por los arrollamientos de todos los polos conectados en serie, a los que se aplica una tensin de alimentacin de c.c. que produce una corriente de circulacin le' que da lugar a una f.m.m. que origina el flujo <1> en el entrehierro de la mquina. El inducido gira dentro del campo magntico del inductor y genera, merced a la combinacin colector-escobillas, una f.e.m. continua E en vaco cuya magnitud viene definida por la expresin (6.6). Al conectar una carga elctrica exterior aparece una corriente l de circulacin que provoca una cada de tensin en el inducido, que se debe en parte a la resistencia propia de este devanado R y en parte a la resistencia que presentan los contactos escobillas-colector. Si se denomina Vese la cada de tensin por par de escobillas (que suele considerarse del orden de 2 voltios), la aplicacin del 2. lema de Kirchhoff al circuito del inducido de la Figura 6.19 conduce a la siguiente ecuacin: E = V + RJ + Vese (6.28)

donde V indica la tensin terminal en bornes de la mquina. En esta ecuacin se ha supuesto despreciable la posible accin desmagnetizante del inducido sobre el inductor. Cuando se considera esta accin, se suele expresar como tanto por ciento de reduccin en la f.m.m. de excitacin, o su equivalente en cada de tensin del inducido. En el caso de que la mquina
-------1,

+
v

r"-:

, , , , ,

____e_e!

CARGA

potencia mecnica INDUCIDO

V. Figura 6.19. Generadorde c.c. o dinamo.

INDUCTOR

502

MQUINAS ELCTRICAS

posea polos auxiliares que anulen la reaccin del inducido, la expresin (6.28) seguir siendo vlida si se incluyen en R; las resistencias de los arrollamientos de estos polos, que como ya se ha comentado en el epgrafe 6.5 estn en serie con el inducido. Para determinar el proceso de transformacin de energa mecnica en elctrica en un generador de c.c. se va a considerar el esquema de la Figura 6.19 en el que se tienen las siguientes ecuaciones de circuito: Inductor:

V" = RJe V""


(6.29) (6.30)

Inducido: E = V + RJ; + Al multiplicar por l la segunda ecuacin anterior resulta: El; = VI; + RJ2 + v"sc(

que expresa el balance de potencias en el inducido de una dinamo. Los trminos anteriores representan: P2 = VIi: Potencia elctrica de salida suministrada por el generador. Peu = R,l2: Prdidas en el cobre del inducido. Pese = V"s);: Prdidas en los contactos de las escobillas. Potencia electromagntica desarrollada por la mquina.
De acuerdo con esta nomenclatura, el balance de potencias en el inducido se convierte en:

(6.31) Para calcular la potencia mecnica de entrada habr que sumar a la potencia electromagntica anterior, Pa' las restantes prdidas, que abarcan:
a)

Las prdidas en el cobre del arrollamiento de excitacin, definidas por: (6.32)


Las prdidas mecnicas Pm debidas al rozamiento y ventilacin. Las prdidas en el hierro PFe, que slo existen en el apilamiento del rotor, debido a la magnetizacin cclica que aparece por su movimiento a pesar de ser el flujo inductor constante.

b)
c)

En consecuencia, y de acuerdo con lo anterior, la potencia mecnica de entrada PI al rbol de un generador de c.c. ser: (6.33) donde Po est determinado por la ecuacin (6.31). En la Figura 6.20 se muestra un esquema simplificado de este balance de potencias. Este diagrama energtico es vlido para todas las mquinas de c.c. en las que la potencia de excitacin proviene de la misma mquina. Hay que tener en cuenta que los circuitos inductor e inducido pueden estar conectados entre s, constituyendo una sola unidad, o pueden estar separados, en cuyo caso la excitacin procede de una fuente exterior y por tanto no interviene en el balance general. Desde el punto de vista de comportamiento y condiciones de trabajo, tiene una gran importancia la forma en que se conectan entre s los devanados inductor e inducido, y as se distinguen:
a)

Mquinas con excitacin independiente, en las que el devanado inductor es alimentado mediante una fuente de alimentacin externa a la mquina, que puede ser, por ejemplo, una batera de acumuladores.

CAPTULO

6.

MQUINAS DE CORRIENTE CONTINUA

503

Potencia elctrica de salida P2 = VI;

Figura 6.20. Balancede potenciasen el generadorde c.c.


b)

Mquinas autoexcitadas, en las que la mquina se excita a s misma tomando la corriente inductora del propio inducido (caso de funcionamiento como generador) o de la misma red que alimenta el inducido (caso de trabajar como motor).

Las mquinas de c.c. con autoexcitacin se clasifican a su vez en: Mquinas serie, en las que el inductor est en serie con el inducido; en este caso el devanado de excitacin est preparado con pocas espiras de hilo grueso, ya que circular por l la corriente total de la mquina. 2. Mquinas shunt o derivacin, en las que el devanado inductor se conecta directamente a los terminales de la mquina, quedando en paralelo (derivacin) con el inducido; en este caso el devanado de excitacin est formado por arrollamientos de hilo delgado con gran nmero de espiras. 3. Mquinas compound o compuestas, en las que la excitacin total est repartida entre dos devanados, uno colocado en serie (de pocas espiras de hilo grueso) y otro colocado en paralelo con el inducido (de muchas espiras de hilo delgado). Segn que el devanado en derivacin est conectado directamente a las escobillas del inducido o despus del devanado en serie, se obtienen las mquinas compuestas con corta o larga derivacin, respectivamente. En la Figura 6.21 se muestran esquemticamente los diferentes tipos de mquinas de c.c. citadas y que pueden emplearse en su versin de generador o motor (se han sealado tambin las letras normalizadas de designacin de los terminales de acuerdo con la Norma UNE-EN 60034-8) . Por razones de sencillez, no se han representado las resistencias de los devanados del inducido, polos auxiliares, etc. La autoexcitacin de los generadores de c.c. fue descubierta por W. Siemens sobre el ao
1866, y se basa en el magnetismo remanente de los polos. Considrese, por ejemplo, una

l.

mquinashunt; al girar el rotor por medio de un motor primario, si no existe ningn flujo en lospolos no aparecer ninguna f.e.m. en las escobillas y en consecuencia no existir transformacinde energa mecano-elctrica; ahora bien, si previamente se han magnetizado los polos (con ayuda de una fuente de c.c. exterior), al mover el rotor se producir en bornes una pequeaf.e.m.; si el devanado inductor est conectado adecuadamente al inducido, se obtendr una corriente en el arrollamiento inductot, que reforzar el magnetismo remanente ya existente,que a su vez provocar un aumento de f.e.m., y as sucesivamente, hasta el instante

504

MQUINAS ELCTRICAS

E1n8
a) Excitacin independiente

b) Excitacin serie

,....--+-------0+
Cl C2

corta derivacin
:
!

l~~_~_~~~i:::~~_i_~_~ .
d) Excitacin compuesta (compound)

e) Excitacin derivacin (shunt)

Figura 6.21. Tiposde excitacinde las mquinasde c.c. en que, debido a la saturacin del circuito magntico de la mquina, se detiene el proceso de autoexcitacin, llegndose a un rgimen de funcionamiento estable. Si la conexin del inductor al inducido no fuera la adecuada, se debilitara el magnetismo remanente y la mquina tendera a desexcitarse.

6.7. GENERADORES DE C.C.: CARACTERSTICAS DE SERVICIO


Las propiedades de los generadores se analizan con la ayuda de las caractersticas que establecen la dependencia entre las magnitudes principales que determinan el funcionamiento de la mquina. Cada uno de los principales tipos de excitacin: independiente, serie, derivacin y compuesto, impone a la mquina caractersticas de funcionamiento distintas, que determinan la clase de servicio al que se adapta cada una de ellas. Estas caractersticas quedan perfectamente reflejadas por medio de grficos, destacando las siguientes: Caracterstica de vaco E =f(le), que representa la relacin entre la f.e.m. generada por la dinamo y la corriente de excitacin, cuando la mquina funciona en vaco; es decir, el inducido no alimenta ninguna carga. 2. Caracterstica en carga V =f(le), que representa la relacin entre la tensin terminal y la corriente de excitacin para una intensidad de carga 1 constante. En particular, cuando 1 es igual a cero se obtiene la curva de vaco. 3. Caracterstica externa V =f(l), que representa la tensin en bornes en funcin de la corriente de carga, para una intensidad de excitacin constante. 4. Caracterstica de regulacin le =f(/), que representa la relacin entre la corriente de excitacin y la corriente de carga, para una tensin en bornes constante. l. De las curvas anteriores merecen especial atencin las caractersticas de vaco y externa, y son las que se van a estudiar a continuacin para cada tipo de generador.

CAPTULO

6.

MQUINAS DE CORRIENTE CONTINUA

505

a)

Caractersticas de un generador con excitacin independiente

En este tipo de generador, la corriente inductora se obtiene de una fuente de c.c. externa a la mquina. Es el tipo de excitacin ms antiguo y hoy 'se emplea nicamente en casos muy especiales. El esquema bsico de conexiones es el que se indica en la Figura 6.22, donde se ha dispuesto de un restato en serie con el inductor para regular la corriente de excitacin. Para determinar la curva de vaco se hace girar el rotor a velocidad constante, manteniendo desconectada la carga. La corriente de excitacin se va aumentando gradualmente desde cero hasta el valor mximo permitido, anotando simultneamente la f.e.m. generada E, que se mide con ayuda de un voltmetro. Al representar la relacin E = f(le) se obtiene la rama ascendente A de la curva de la Figura 6.23. Se repiten luego las medidas disminuyendo la corriente le hasta llegar a cero y al dibujar la curva E = f(I) se obtiene la rama descendente B de la Figura 6.23a. La diferencia entre ambas ramas se debe a la histresis de los polos. La curva media entre las mismas representa la caracterstica de vaco del generador de c.c. Se observa en esta curva que la mquina produce una f.e.m. ER sin corriente de excitacin (le = O), lo cual se debe al magnetismo remanente que aparece en los polos. Si se tiene en cuenta la ecuacin (6.6), que expresa el valor de la f.e.m. generada, es evidente que para una determinada velocidad de giro, la f.e.m. es proporcional al flujo por polo, y como quiera tambin que la f.m.m. del inductor (A.v/polo) es proporcional a la corriente de excitacin, se comprueba que la forma de la caracterstica de vaco es del mismo tipo que la curva de magnetizacin de los materiales magnticos. Otro aspecto a considerar en la curva de vaco es que la f.e.m. generada resulta, de acuerdo con (6.6): (6.34) es decir, la f.e.m. es directamente proporcional al flujo magntico que producen los polos y a la velocidad de rotacin de la mquina. Es por ello que si se considera que la velocidad de giro es diferente, por ejemplo ni, la f.e.m. producida para el mismo valor del flujo ser: (6.35) y al dividir entre s las expresiones (6.34) y (6.35) resulta:

E
E'

ni

(6.36)

INDUCTOR

CARGA

Reostato de excitacin

Potencia mecnica

11

INDUCIDO

+
Figura 6.22. Ensayo de vaco de una dinamo con excitacin independiente.

506

MQUINAS ELCTRICAS

E (voltios)
n

= constante

Caracterstica de vaco

n'< n

E'

= E!n'/n)

le (amperios) a) h)

Figura 6.23. Curva de vaco de un generador de c.c. para distintas velocidades. Es decir, para los mismos valores de los flujos, las f.e.m.s. son proporcionales a las velocidades de giro respectivas. Esto significa que si se conoce la curva de vaco de una mquina para una velocidad n, se puede deducir la curva de vaco a otra velocidad n' sin ms que aplicar la ecuacin (6.36). En la Figura 6.23b se muestra esta idea; la curva de vaco de trazo grueso est definida para la velocidad n y se ha deducido la curva de vaco para la velocidad n', de tal manera que E' = E(n'/n) para cada valor de las abscisas. Para determinar la caracterstica externa de un generador con excitacin independiente habr que conectar la carga; cerrando el interruptor de la Figura 6.22 y variando la misma se representar la relacin V = f(l) cuando se mantiene constante tanto la corriente de excitacin como la velocidad de giro del motor primario. En principio, si no se considera la reaccin del inducido, la f.e.m. generada E ser constante y la variacin de la tensin en bornes con la corriente del inducido vendr expresada por la relacin (6.28), es decir:

V = E - R/ - Vese

(6.37)

donde I es la intensidad del inducido y que coincide con la corriente de carga. La expresin anterior representa la ecuacin de una recta. Si se tiene en cuenta la reaccin del inducido, al desplazar las escobillas para obtener una buena conmutacin, aparece una f.m.m. desmagnetizante que acta sobre el valor de la f.e.m. engendrada y cuya magnitud puede ser no lineal con la corriente si la mquina trabaja en la zona de saturacin. En la Figura 6.24 se muestra la caracterstica externa de un generador con excitacin independiente, donde pueden apreciarse cada una de las cadas anteriores. Se observa que en la zona normal de trabajo, que llega hasta la corriente asignada, la caracterstica es prcticamente lineal. Hay que hacer notar que si no se considera la reaccin del inducido por estar compensada por los polos auxiliares, hay que introducir en la resistencia R de la ecuacin (6.37) la resistencia de estos polos.

b)

Caractersticas de un generador derivacin

El esquema de conexiones de este generador es el mostrado en la Figura 6.25; en este caso el devanado inductor est conectado en paralelo con el inducido, y se regula la excitacin por medio de un restato conectado en serie con el inductor.

CAPTULO

6.

MQUINAS

DE CORRIENTE

CONTINUA

507

V(voltios)

.:=::-----0------------ ------------0 ... _.. ~

Cada debida a la reaccin del inducido Cada debida a la resistencia de inducido y escobillas

1nomina

Ji (amperios)

Figura 6.24.

Caracterstica externa de un generador de c.c. con excitacin independiente.

Para determinar la f.e.m. que produce la dinamo en vaco habr que utilizar la caracterstica en vaco de la mquina (Fig. 6.26) junto con la caracterstica del inductor.que representa la relacin E = RJe (donde Re es la resistencia total del circuito de excitacin). Esta caracterstica del inductor es una recta de pendiente Re: tga=R
E =e le

(6.37)

La f.e.m. final engendrada correspondeal punto P, solucin comn a la curva de vaco y a la recta del inductor. Para cada valor de la resistencia del circuito de excitacin se obtendr una recta de pendiente distinta cuya interseccin con la curva de vaco determinar la f.e.m. resultante. Si la resistencia Re se va desplazando por la curva de vaco hasta llegar al origen, para una cierta resistencia Re = Rcr' denominada resistencia crtica, la recta del inductor ser tangente a la parte inicial de la curva de vaco y, en consecuencia, en estas condiciones la dinamo prcticamente no se excita. Por ello la resistencia mxima del restato debe proyectarse para que al sumarse con la resistencia del devanado inductor no se supere la magnitud R"r' La Figura 6.26 puede servir tambin para explicar el proceso de autoexcitacin de estos generadores. Considrese una resistencia total del circuito inductor Re definida por el ngulo a. En vaco, toda la corriente de inducido pasa a travs del devanado inductor, por lo que li =1" adems, en estas condiciones la corriente comn anterior es de pequeo valor, por lo que se pueden considerar despreciables tanto la cada de tensin en la resistencia del inducido como
Reostato de

v
INDUCTOR N INDUCIDO

CARGA

Figura 6.25. Generador con excitacin en derivacin o shunt.

508

MQUINAS ELCTRICAS

E E
~ __ r-----

Curva de vaco

E=RJe

Figura 6.26.

Determinacin de la tensin de funcionamiento de una dinamo derivacin.

la propia reaccin del inducido; de este modo la d.d.p. en bornes de la mquina (tensin entre M y N en la Fig. 6.25) coincide con la f.e.m. generada. Si se considera que la resistencia total del circuito de excitacin es Re (devanado inductor + restato de campo) y que la inductancia del devanado inductor es Le' de acuerdo con el 2. lema de Kichhoff se tiene: (6.38) Inicialmente la corriente le = O,por lo que la f.e.m. generada ER se debe al magnetismo remanente de los polos y que corresponde al segmento OA de la Figura 6.26; de este modo, de acuerdo con (6.38), en este momento se cumple:
ER=Ldt

.u,

(6.39)

Por consiguiente, al ser ER un valor positivo, la dlJdt ser positiva y la corriente le comienza a aumentar hasta alcanzar el valor OB, lo que a su vez hace que la f.e.m. aumente hasta el valor Be; de este modo, y teniendo en cuenta que: E-Rl
=Ldt

.u,

e e

(6.40)

el primer miembro de la ecuacin anterior representa la diferencia entre la curva de vaco y la recta de la resistencia del inductor, que siempre da un valor positivo, ya que la curva de vaco est por encima de la recta Re' lo que hace que dlJdt > O, por lo que la corriente le va aumentando en todo momento; en nuestro caso la corriente alcanzar el valor OD, que producir la f.e.m. FD, y as sucesivamente (el recorrido que sigue la mquina est sealado por la lnea quebrada de trazo discontinuo de la Fig. 6.26). Cuando la f.e.m. alcance el valor dado por la interseccin de la recta de campo con la curva de vaco (punto P de la Fig. 6.26), de acuerdo con (6.40) el trmino Ldlrdt ser cero, por lo que no puede haber ningn aumento posterior de la corriente le' la f.e.m. deja de

CAPTULO

6.

MQUINAS DE CORRIENTE CONTINUA

509

aumentar y el punto P determina los valores finales tanto de la tensin en vaco (f.e.m.) como de la corriente de excitacin, cumplindose en este punto la condicin: E=RJe (6.41)

Nota prctica: Hay que destacar que para producir la autoexcitacin de un generador shunt deben asegurarse las concordancias de los sentidos del devanado de campo con la tensin producida. Es decir, la corriente del devanado de campo debe producir un flujo magntico queayude a aumentar (reforzar) el magnetismo remanente; de otro modo el flujo que producira el devanado inductor tendera a anular el magnetismo remanente existente y la dinamo no producira tensin. En el argot elctrico se dice que la dinamo no se ceba. Cuando esto sucedepor error en las conexiones, es necesario proceder a imantar con c.c. exterior los polos de la mquina y volver a montar el circuito nuevamente, invirtiendo bien sea el devanado de campo o el sentido de giro de la mquina. En un generador derivacin, la caracterstica externa V = f(l) representa una variacin de la tensin terminal superior a la del generador con excitacin independiente; esto se debe a que hay que aadir ahora el efecto de la cada de tensin producida por las'escobillas, resistencia del inducido y su reaccin, con la disminucin correspondiente en la corriente de excitacin. En la Figura 6.27 se muestra la caracterstica externa de un generador shunt en comparacin con un generador con excitacin independiente, donde se aprecia lo anteriormente sealado. El generador derivacin es el tipo que se emplea ms frecuentemente en la prctica, ya queno necesita una excitacin separada y en los lmites de carga normales no presenta cadas de tensin elevadas. De cualquier modo, variando la resistencia del restato de excitacin se puede ajustar la tensin terminal para compensar la cada de tensin producida por la carga.

e) Caractersticas de un generador serie


En este tipo de generador el inductor y el inducido van en serie, tal como se muestra en la Figura6.28. Cuando la mquina est en vaco, al ser le = 1 = l = O,se obtiene una f.e.m. muy pequea que se debe al magnetismo remanente.

1nominal

Figura 6.27. Caracterstica externade un generadorshunt.

510

MQUINAS ELCTRICAS

INDUCTOR

v
v
INDUCIDO

A a)

o
b)

Figura 6.28. Generador con excitacin serie y caracterstica externa correspondiente.

Al conectar la carga, si sta tiene una gran resistencia, la mquina puede no excitarse; la reduccin de esta resistencia, Rv va acompaada con un incremento relativamente elevado de la tensin terminal. En la Figura 6.28b se presenta la curva de vaco de la mquina construida con el inductor conectado como excitacin independiente. Tambin se observa la forma de la caracterstica externa V = f(/). Para una corriente OA, se genera una f.e.m. A C y la correspondiente tensin es AB. La diferencia entre ambas magnitudes es BC y representa, en consecuencia, la cada de tensin en la resistencia de inducido, reaccin en el mismo y contacto de escobillas. De la caracterstica externa se deduce que la tensin del generador vara bruscamente con la carga, por ello en la prctica este tipo de dinamo no se emplea.

d)

Caractersticas de un generador con excitacin compuesta

En la Figura 6.29 se representa el esquema de circuito de este tipo de generador, que puede hacerse con corta o larga derivacin. Generalmente las f.m.m.s. de los devanados serie y derivacin suelen ser del mismo signo, es decir, aditivas, pero puede realizarse una conexin sustractiva o diferencial.

v
Reostato de E

v
INDUCIDO
Inominal

a)

b)

Figura 6.29. Generador compound y caractersticas externas.

CAPTULO

6. MQUINAS DE CORRIENTE CONTINUA

511

La caracterstica exterior de este generador, en caso aditivo, se obtiene como suma de las caractersticas derivacin y serie, pudiendo dar curvas prcticamente planas, lo que indica la constancia de la tensin terminal con la corriente de carga. Si se incrementa el nmero de espiras en serie, la tensin terminal puede aumentar con la carga, lo que da lugar a la caracterstica hipercompuesta. En caso contrario, la tensin puede reducirse con la carga como en el generador derivacin, dando lugar a la caracterstica hipocompuesta. La conexin diferencial presenta una cada de tensin elevada con el aumento de la carga, lo que hace til su aplicacin en generadores para soldadura en corriente continua. En la Figura 6.29b se muestran las caractersticas externas para excitaciones aditivas: hipercompuesta e hipocompuesta y de tipo diferencial.

EJEMPLO

DE APLICACIN

6.4

La caracterstica de vacio de un generador de c.c. tipo derivacin que gira a 1.000 r.p.m. se expresa en la siguiente tabla de valores:

E
:
..

20 0,56

40 1,12

60 1,76

80 2,6

100 4

110 5

120 6,3

Calcular la tensin de la mquina en circuito abierto con una resistencia total del circuito de excitacin de: a) 22 n. b] 30 Q. c) Encontrar la magnitud de la resistencia crtica a la velocidad asignada de 1.000 r.p.m. d] Determinar la resistencia del circuito de campo para que la mquina suministre 100 Ven vacio a la velocidad asignada. e) Hallar la velocidad crtica de la mquina en el apartado anterior.

SOLUCIN
a)

En la Figura 6.30 se muestra la caracterstica de vaco de la dinamo derivacin a 1.000 r.p.m. Para determinar la tensin en vaco (f.e.m.) con Re = 22 Q, se construye la recta del inductor, definida por la ecuacin:

v = E = R),. = 22 le
Para ello, en la abscisa correspondiente a le = I A, se tomar V = E = 22 V (punto A en la Fig. 6.30). La interseccin de la recta anterior con la curva de vacfo da una f.e.m. igual a 110 V (punto B). De forma anloga al caso anterior se construye ahora la recta V = 30 le Y se obtiene la interseccin en el punto C, que corresponde a una f.e.m, aproximada de 82 V. La resistencia crtica para n = 1.000 r.p.m. corresponde a la tangente a la curva de vaco a). Midiendo su valor en la Figura 6.30 resulta una Rer = 35 Q. Para E = 100 V se obtiene una corriente de excitacin de 4 A; en consecuencia, la resistencia de excitacin necesaria ser: Re = 100/4 = 25 Q.

b)
c) d)

512

MQUINAS ELCTRICAS

140
Re 220
VACO

I I

120
llOV 100

R" 35 O

R,~300

JI

....,... l.iO"I""
n

1.000r.p.m.

82V

C ~\.

80 60 40 20
A

n' -

714,3 r.p.m.

le 2 3 4 5
6 7

Figura 6.30. e)

La velocidad crtica para Re = R:r = 25 n corresponder a la curva de vaco especificada para una velocidad n' tal que esta curva sea tangente (para le ~ O)a la recta V = E = 25 le. Teniendo en cuenta que la resistencia crtica para n = 1.000 r.p.m. es de Rer = 35 n, y como quiera que las curvas de vaco y las tangentes respectivas coinciden para le ~ O, se puede escribir: Para n Para n'

= 1.000 r.p.m. E = kEnrJ> = R,}, = (nueva velocidad) E' = kEn' rJ> = R:Je
=;> =;>

por lo que al dividir entre s las f.e.m.s. anteriores resulta:


E
=;>

nrJ> Rje
_=;>

1.000
--

E'

n'rJ>

R:Je

n'

35 =25

=;>

n'

25 = 1.000 - ~ 7143 r.p.m. 35'

Para comprobar la relacin anterior de un modo grfico, en la Figura 6.30 se ha construido la nueva curva de vaco correspondiente a la velocidad anterior (curva a trazos discontinuos). Las ordenadas (f.e.m.s.) de esta nueva curva son 714,311.000 ~ ~ 0,714 veces la original especificada a 1.000 r.p.m. El lector puede comprobar que la recta V = 25 le es tangente en el origen a esta curva de vaco. EJEMPLO DE APLICACIN 6.5 Un generador de c.c. compuesto de larga derivacin de 200 V, 10 kW, 4 polos tiene una resistencia del inducido, polos auxiliares, campo en serie y campo derivacin de 0,1; 0,03; 0,07 y 100 n, respectivamente. Calcular el valor de la f.e.m. generada cuando la mquina trabaja a plena carga. Si la velocidad asignada es igual a 1.500 r.p.m. y el inducido tiene un

CAPTULO

6. MQUINAS DE CORRIENTE CONTINUA

513

devanado imbricado con 720 conductores, determinar elflujo por polo en estas condiciones. Prescndase de la cada de tensin en escobillas. SOLUCIN El esquema de circuito es el mostrado en la Figura 6.31, donde se muestran los valores de las resistencias de los devanados.

le

l/
0,1

100 n

v=

200

Figura 6.31.

A plena carga, la corriente l vale:


P 10.000 1=-=--= V 200

50A

Para una tensin terminal constante igual a 200 V, la corriente de excitacin en el campo derivacin es:
l =-=2A e 100

200

En consecuencia, la corriente de inducido vale: I = l + le = 50 + 2 = 52 A La f.e.m. generada por la mquina tendr una magnitud: E = V + RJ donde en R hay que incluir la resistencia del campo inducido, campo en serie y polos auxiliares, resultando: E

= 200 + (0,07 + 0,1 + 0,03) 52 = 210,4 V

Para calcular el valor del flujo por polo correspondiente hay que tener en cuenta que para un devanado imbricado el nmero de circuitos derivados coincide con el nmero de polos. Al aplicar despus la ecuacin general de la f.e.m. resulta: n p 1.500 2 E = 210 4 = - Z<1> - = -720<1>, 60 e 60 2 de donde resulta un flujo de 11,68 mWb.

514

MQUINAS ELCTRICAS

6.8. MOTORES DE C.C.: ASPECTOS GENERALES


Un motor de c.c. transforma una energa elctrica de entrada en una energa mecnica de salida. Esencialmente consiste en una dinamo trabajando en rgimen inverso, lo que est de acuerdo con el principio de reciprocidad electromagntica formulado por Faraday y Lenz. Para comprender este principio bsico de reciprocidad en el funcionamiento de una mquina de C.C., se va a considerar una dinamo derivacin que suministra energa elctrica a una red de c.c. de tensin constante, tal como se indica en la Figura 6.32a. De acuerdo con la expresin (6.28), la aplicacin del 2. lema de Kirchhoff al circuito del inducido conduce a la siguiente ecuacin: E = V + RJ + que denominando V; = V +

v:.sc

(6.42)

v:.sc a la tensin neta en el inducido nos da una corriente f:


[=--'

E-V
R

(6.43)

Si la f.e.m. E es superior a la d.d.p. V; (prcticamente esta tensin coincide con la de la red), el sentido de la corriente en el inducido coincide con el de E; en consecuencia, la mquina trabaja como generador suministrando una potencia electromagntica El; La mquina crea entonces un par resistente que se opone al de rotacin, es decir, contrario al movimiento del motor primario. Si se disminuye la f.e.m. del generador, reduciendo la velocidad de rotacin o la excitacin del inductor, cuando E se hace menor que la tensin V;, la corriente I del inducido cambiar de sentido, como as lo expresa la ecuacin (6.43); se dice entonces que la mquina produce una fuerza contra-electromotriz, ya que E se opone a la corriente I. En esta situacin la mquina trabaja como motor y se produce un par electromagntico que coincide con el de rotacin, lo que indica que el par ha pasado de ser resistente a ser motor. Desconectando el motor primario, la mquina de c.c. continuar girando en el mismo sentido que lo haca

Red dec.c.

v = constante
E

Red de c.c.

v = constante
E

+-R

+-R

a) Generador

b) Motor

Figura 6.32.

Funcionamiento de la mquina de c.c. como generador o como motor.

CAPTULO

6.

MQUINAS DE CORRIENTE CONTINUA

515

cuando actuaba como generador pero desarrollando ella ahora su propio par motor. El que la mquina conserve el mismo sentido de giro trabajando como generador o como motor se debe a que se ha cambiado la polaridad de la corriente en uno solo de los devanados. En las Figuras 6.32a y b se muestra esta accin, donde puede observarse que en ambos casos la corriente de excitacin tiene el mismo sentido, pero que sin embargo la corriente I ha cambiado de signo. Si en el rgimen de funcionamiento como motor se considera positivo el signo de la corriente 1; absorbida, al aplicar el 2. lema de Kirchhoff al circuito del inducido de la Figura 6.32b se obtiene:

v=E

+ R/ +

v",e

(6.44)

que no es ms que la ecuacin (6.43) en la que se han cambiado los signos de I, V; Y E. Para determinar el proceso de transformacin de energa mecnica en elctrica en el motor de c.c. se ha de obtener una ecuacin del balance de potencias. Para ello, si se multiplican ambos miembros de (6.44) por 1; resulta: (6.45) cuyos trminos significan: Potencia elctrica absorbida por el inducido del motor. Peu; = R/2: Prdidas en el cobre del inducido. Pe. = V,. ".1;: Prdidas en los contactos de las escobillas.
1e

Potencia electromagntica

desarrollada por la mquina.

De acuerdo con esta nomenclatura, el balance de potencias en el inducido se convierte en: (6.46) La potencia electromagntica Pa expresa la potencia mecnica total que produce el motor, y que da lugar, al dividir por la velocidad de giro, al par interno desarrollado por la mquina y cuya expresin ya se calcul en el epgrafe 6.3, definida por la ecuacin (6.11):

T=__
2nn 60

[N.m]

(6.47)

Si se sustituye en esta ecuacin la f.e.m. E por su valor (6.6), se obtiene la siguiente expresin: (6.48) que coincide con la (6.10) demostrada por otro procedimiento. Esta ecuacin indica que el par es directamente proporcional al flujo en el entrehierro y a la corriente del inducido. Para calcular la potencia mecnica til en el rbol del motor habr que restar a la potencia electromagntica, Pu' las prdidas rotricas, que abarcan las prdidas en el hierro PFe Y las prdidas mecnicas p,,, de rozamiento y ventilacin. En consecuencia, la potencia til del motor P2 se podr poner: (6.49)

516

MQUINAS ELCTRICAS

La potencia absorbida por la mquina PI' en el caso de tener el motor excitacin independiente, ser igual a la potencia que llega al inducido P; sin embargo, en las mquinas autoexcitadas la potencia de entrada tendr que compensar tambin las prdidas en el circuito de excitacin Pex debidas al efecto Joule en el cobre del inductor, resultando: (6.50) En la Figura 6.33 se muestra de una forma esquemtica el reparto de potencias en el motor de c.c. La potencia de entrada PI es el producto de la tensin de alimentacin por la corriente absorbida, y el rendimiento del motor ser: (6.51)

Potencia mecnica de salida P2

Figura 6.33. Balance de potencias en el motor de c.c.

EJEMPLO

DE APLICACIN

6.6

Un motor tipo derivacin de 500 V consume 4 A en vaco. La resistencia del inducido, incluidas las escobillas, es de 0,2 n y la corriente de excitacin es de 1 A. Calcular la potencia de salida y el rendimiento cuando la corriente de entrada es de: a) 20 A; b) 100 A. SOLUCIN
a)

El motor en vaco consume una potencia: Po

VIo

= 500

. 4 = 2.000 W

Esta potencia se emplea en cubrir las prdidas en el hierro PFe, las prdidas mecnicas Pm, las prdidas en el inducido (cobre, escobillas) y las prdidas de excitacin. Exceptuando las prdidas en el inducido, que son proporcionales al cuadrado de la intensidad del inducido, todas las dems permanecen constantes cualquiera que sea el rgimen de trabajo de la mquina. Con los datos del problema se tiene: Prdidas en el inducido en vaco: RJ2

= 0,2

(4 - 1)2

1,8 W

Prdidas en el inductor: Yl, = 500 . 1 = 500 W

CAPTULO

6.

MQUINAS DE CORRIENTE CONTINUA

517

Se observa que en vaco prcticamente se pueden despreciar las prdidas en el inducido y en consecuencia se puede poner: Po = 2.000 W = PFe +

r; + 500

::;. PFe + Pm = 1.500 W

Cuando la mquina absorbe una corriente de 20 A, la intensidad del inducido vale: I, = 1 - le = 20 - 1 = 19 A. La f.e.m. generada es: E = V - R/ = 500 - 0,2 19 = 496,2 V, y por tanto la potencia electromagntica es: Pa = EI = 496,2 . 19 = 9.427,8 W y la potencia mecnica de salida, de acuerdo con (6.49), ser: P2 = 9.427,8 - 1.500 = 7.927,8 W Como la potencia elctrica de entrada es:
PI

= V . 1 = 500 . 20 =
r

10.000 W

el rendimiento del motor en este rgimen ser:

= - = -PI

P2

7.927,8 10.000

= 79,27 %

b)

De forma similar, cuando la corriente absorbida es de 100 A, resulta: E = V - R/ = 500 - 0,2 (100 - 1) = 480,2 V La potencia electromagntica es: Pa = EI

= 480,299 = 47.540 W
= 47.540 - 1.500 = 46.040 W

y la potencia mecnica de salida ser: P2 =

P, - (PFe + Pm)

Como la potencia absorbida por el motor de la red es:


PI = V l = 500 . 100 = 50.000 W

el rendimiento de la mquina ser ahora:


r

= - = -PI

P2

46.040

50.000

= 92,08 %

6.9. MOTORES DE C.C.: CARACTERSTICAS DE FUNCIONAMIENTO


Los diferentes tipos de motores de c.c. se clasifican de acuerdo con el tipo de excitacin, de forma anloga a la que se haca con los generadores, y as se tiene: motores con excitacin independiente, serie, derivacin y compuesta. En cada caso el par electromagntico viene determinado por la expresin (6.48); (6.52)

518

MQUINAS ELCTRICAS

Desde un punto de vista prctico, estos motores de c.c. presentan una gran ventaja sobre los motores de c.a., debido a su posibilidad de regulacin de velocidad. De acuerdo con la ecuacin (6.44), si se incluye en R la resistencia del inducido y escobillas se tiene: V= E + RJ Teniendo en cuenta la expresin general de la f.e.m.: E (6.53)

= 60 - Zct> -c = K:nct> .
V-RI

(6.54)

y al sustituir en (6.53), despejando la velocidad n, se obtiene:


11=--'-'

KEct>

(6.55)

que indica la posibilidad de regular la velocidad de un motor de c.c. a base de controlar las siguientes variables: El flujo por polo producido por la corriente de excitacin. Al disminuir el flujo, aumenta la velocidad de rotacin, y de ah el peligro de poner en marcha el motor sin conectar la excitacin, ya que dar lugar a un embalamiento del motor, limitado nicamente por el magnetismo remanente de los polos. b) La tensin de alimentacin V,aplicada al motor. Al disminuir/aumentar la tensin de alimentacin se reduce/aumenta la velocidad, tal como seala la ecuacin (6.55). c) La resistencia del circuito del inducido, lo que se consigue conectando en serie con este devanado una resistencia o restato variable. Al aumentar/disminuir la resistencia del inducido, la velocidad disminuye/aumenta, como cabe deducir de la ecuacin
(6.55).

a)

Estos mtodos se emplean en la prctica de forma independiente o combinada para obtener unas caractersticas adecuadas al rgimen de trabajo que se necesita, aunque hoy da se han sustituido por procedimientos electrnicos (vase Cap. 7). Para invertir el sentido de rotacin de un motor de c.c. es necesario invertir nicamente la corriente en uno solo de los devanados: inducido o inductor. Si se modifican los sentidos de la corriente de ambos circuitos, el sentido de rotacin del motor no se modifica [lo que se justifica por la aplicacin de la Ley de Laplace: F = I(L x B) a los conductores del inducido]. Un aspecto importante en los motores de c.c. lo constituye el proceso de arranque. Hay que tener en cuenta que, de acuerdo con (6.53), la corriente del inducido tiene una magnitud:

/.,= -- R

V-E
,

1" (arranque directo)

=R

V
I

(6.56)

Como quiera que en el momento de la puesta en marcha la velocidad es nula, se tiene en ese momento una f.c.e.rn. tambin nula, por lo que la corriente del inducido en ese instante ser muy elevada, debido a que entonces el motor ofrece a la tensin de la red slo una pequea resistencia R. Para proteger el motor contra esta corriente de arranque tan fuerte se utilizan restatos de arranque conectados en serie con el inducido. En la Figura 6.34 se muestra un circuito de este tipo en el que se ha considerado que la excitacin es independiente. La resistencia total de arranque se subdivide en diversos tramos con conexiones a unos terminales o plots, de tal forma que en el arranque se tiene la mxima resistencia en serie con el inducido, la cual se va eliminando sucesivamente (plots 1 - 2 - 3 - 4) conforme el motor va

CAPTULO

6.

MQUINAS DE CORRIENTE CONTINUA

519

Restato de arranque
Off

(parada)

-n
INDUCTOR

Figura 6.34.

Restato de arranque y su conexin.

aumentando de velocidad, hasta que al cabo de un cierto tiempo el rotor queda conectado directamente a la red. Hoy da el arranque se hace automticamente utilizando contactares y rels de tiempo, para conmutacin de las resistencias. En los diversos esquemas de funcionamiento de las mquinas de c.c. se ampliar esta idea, explicando tambin las llamadas caractersticas mecnicas que expresan la relacin n = J(T), que tanta importancia tiene en el diseo de los accionamientos elctricos.

6.9.1. Motores de c.c. con excitacin independiente y derivacin. Sistema de regulacin Ward-Leonard
Los esquema de conexiones para el arranque y regulacin de velocidad de estos motores de c.c. son similares entre s y se muestran en la Figura 6.35. En el caso de excitacin independiente (Fig. 6.35a), los circuitos del inductor y del inducido se alimentan de fuentes distintas, mientras que en el caso de motor derivacin (shunt) las fuentes coinciden. Es por ello que si la
Restato de arranque Restato de arranque
++

+
v

-/i

+
v
E

t,
R

R,
E

+
Inductor

+
a) Motor con excitacin independiente

Ve
b) Motor con excitacin derivacin

Figura 6.35. Motores con excitacin independiente y derivacin.

520

MQUINAS ELCTRICAS

tensin de alimentacin al inducido se supone constante (V = ~) no hay diferencia prctica en el comportamiento de estos dos tipos de motores, y por ello en este epgrafe se van a estudiar ambas mquinas al mismo tiempo. En principio la explicacin se cie al motor derivacin, pero cuando existan diferencias con el motor con excitacin independiente se tratar ste de modo concreto. En cualquier caso, durante la puesta en marcha interesa que el flujo en el entrehierro posea su valor mximo, de esta forma el motor podr desarrollar el par de arranque necesario con la corriente mnima posible en el inducido. Es por ello que desde el primer instante de la puesta en marcha, el devanado de excitacin debe de estar conectado a la tensin de la red; adems, el restato R, conectado en serie con el inductor deber tener la mnima resistencia para que sea tambin mxima la corriente de excitacin o campo. Para deducir analticamente las caractersticas n = JeT) de estos motores se va a suponer inicialmente que trabajan en la zona lineal de la curva de magnetizacin. De acuerdo con las expresiones (6.52), (6.53) y (6.54), se tiene: (6.57) de donde resulta: (6.58) que conduce a una expresin de la velocidad en funcin del par:
n=--'-'=--V-

V-RI

KE<D

KE<D

KEKT<D2

'

(6.59)

Estando funcionando el motor derivacin con un par resistente determinado, al aumentar ste se producir un frenado en la mquina, disminuyendo la velocidad del rotor; como consecuencia de ello la f.c.e.m. E = KEn<D del motor se reducir, por lo que segn (6.58) aumentar la corriente absorbida por el inducido y el par de la mquina T = KT<D l se elevar igualndose con el nuevo par resistente ofrecido por la carga. De acuerdo con la expresin (6.59), la curva par-velocidad n = JeT) del motor derivacin (y por extensin, del motor con excitacin independiente) es una lnea recta, como as se muestra en la Figura 6.36a. La ecuacin de esta recta para los valores asignados de tensin aplicada, resistencia asignada del inductor y resistencia asignada del inductor (sin resistencia adicional en el circuito de campo) se denomina caracterstica natural de la mquina. La velocidad del motor en vaco (para T = O)viene definida por el trmino no = lIKE<D V de la ecuacin (6.59). Para altos pares de carga, aumenta la corriente del rotor y por ello se reduce el flujo resultante a consecuencia de la reaccin del inducido; de ah que en realidad la caracterstica n = JeT) de estos motores se desve ligeramente de la recta anterior. La cada de velocidad con el aumento del par en la curva anterior es muy pequea, lo que indica que los motores derivacin presentan una caracterstica de carga dura o rgida y por ello se utilizan en aquellas aplicaciones que requieran una velocidad casi constante: ventiladores, bombas centrfugas, cintas transportadoras, mquinas herramientas, etc. De acuerdo con la expresin (6.59), y como ya se ha indicado en el epgrafe 6.9, la regulacin de velocidad de los motores derivacin e independiente se consigue de las formas siguientes: a) ajustando la tensin del inducido; b) variando la resistencia del circuito del inducido (aunque este procedimiento es poco prctico por las prdidas que se producen, y de
ah su bajo rendimiento); c) cambiando la resistencias del circuito de excitacin, que regula a su vez el flujo del motor. Estudiemos a continuacin cada uno de estos mtodos.

CAPTULO

6.

MQUINAS DE CORRIENTE CONTINUA

521

Vnom= v Y < Vnom V2< VI V3< V2< VI T (par)


b) Variacin de la tensin aplicada

n (velocidad)

n0r-

__
V,U)/r!

=V

T(par)
a) Caracterstica natural

Rs1

= Rsmin

T(par)
) Variacin del flujo inductor

Figura 6.36. Caractersticas par-velocidad de un motorderivacino shunt.

a) Regulacin de velocidad por cambio en la tensin aplicada al inducido


Este sistema de control solamente se puede aplicar al motor con excitacin independiente, ya que es el que tiene separados los circuitos de excitacin y de inducido. Al reducir la tensin de alimentacin V, y de acuerdo con (6.58), la corriente I, disminuye, por lo que el par desarrollado por el motor se reduce y, al hacerse inferior al par resistente, se produce una disminucin de la velocidad de la mquina. Al contrario, si aumenta la tensin aplicada, se produce una elevacin de la velocidad. En la Figura 6.36b se muestran las curvas (rectas) par-velocidad que se obtienen cuando se regula la velocidad de un motor de c.c. con excitacin independiente empleando este procedimiento. Se observa que la velocidad en vaco no = l/KE$ V va cambiando con este mtodo, ya que sta es proporcional a la tensin aplicada; sin embargo, no se modifican las pendientes de las rectas respectivas (porque no se altera ni el valor del flujo ni la resistencia en serie del inducido). Es decir, resultan una serie de rectas paralelas a la racterstica natural de la mquina y que estn por debajo de sta, ya que la tensin aplicada se va disminuyendo por debajo de la tensin asignada (que sera la mxima prevista para no daar los aislamientos de la mquina).

522

MQUINAS ELCTRICAS

b) Regulacin de velocidad por cambio en la resistencia en serie con el inducido


Al introducir una resistencia adicional en el circuito del inducido se produce un aumento muy fuerte en la pendiente de la caracterstica par-velocidad de la mquina, lo que est justificado por la ecuacin (6.59). Este sistema es vlido tanto para los motores con excitacin derivacin como para motores con excitacin independiente. En la Figura 6.36c se muestra la evolucin de las rectas correspondientes. Como ya se ha indicado, este procedimiento de control de velocidad es bastante antieconmico, ya que al circular la elevada corriente de inducido por esta resistencia adicional (restato) las prdidas por efecto Joule son muy elevadas, y es por ello que solamente se utiliza en motores de pequea potencia. Obsrvese en la Figura 6.36c que todas las rectas que se obtienen (que se denominan caractersticas artificiales o reostticas) pasan por el punto correspondiente a la velocidad de vaco de la mquina no (ya que esta velocidad depende solamente de la tensin aplicada V y del flujo inductor <1.

e) Regulacin de velocidad por cambio en la resistencia en serie con el inductor


Al introducir una resistencia adicional en el circuito del inductor se produce una disminucin en el flujo inductor de la mquina (tanto si el motor es con excitacin independiente o derivacin), lo que se traduce en un aumento en la velocidad del motor. Para comprender la sucesin de fenmenos que resultan, tngase en cuenta que al introducir una resistencia adicional en el circuito inductor con ayuda de la resistencia o restato variable R, de la Figura 6.35 se produce una disminucin tanto de la corriente de excitacin como del flujo inductor. Como consecuencia de ello, se reduce la f.c.e.m. del motor, lo que provoca un aumento de la corriente del inducido absorbida por la mquina. El par del motor T = KT<1>I aumenta, ya que la disminucin de flujo est compensada con creces por el aumento en la corriente del inducido. Como consecuencia del aumento del par motor se produce una elevacin de la velocidad de la mquina. La explicacin analtica de lo anterior se justifica con la ayuda de la ecuacin (6.59), que volvemos a repetir aqu: (6.60) Es evidente que al reducirse el flujo se produce un aumento de la velocidad en vaco no, que se deduce de la ecuacin anterior: (6.61) y un aumento sustancial de la pendiente de las rectas correspondientes, que vienen definidas por el valor del coeficiente que afecta al par en la ecuacin (6.60): (6.62) De este modo, si se desprecia la reaccin de inducido de la mquina, al variar la resistencia del inductor se obtienen un haz de rectas que estn por encima de la caracterstica natural del motor, tal como se muestra en la Figura 6.36d. En resumen:

CAPTULO

6.

MQUINAS DE CORRIENTE CONTINUA

523

En la regulacin de velocidad por ajuste de la tensin aplicada al inducido (caso de motores con excitacin independiente), cuanto menor es la tensin aplicada, menor es la velocidad, y cuanto mayor es la tensin, ms elevada es la velocidad que alcanza el motor. Existe una velocidad mxima que puede alcanzar la mquina con este procedimiento, y corresponde al valor mximo de la tensin permitida (tensin asignada). b) La regulacin por variacin de la resistencia del inducido solamente se aplica a pequeas mquinas en virtud de su bajo rendimiento. Cuanto mayor es la resistencia que se aade al circuito del inducido, tanto menor ser la velocidad que adquiere la mquina. En definitiva, se produce una mayor cada de tensin en el circuito serie aadido, por lo que llegar una tensin menor al motor. Esto es, resulta una variante al caso anterior de control por tensin en el inducido. c) En la regulacin por resistencia de excitacin, cuanto mayor es la resistencia de este circuito menor es elflujo y mayor es la velocidad, ya la inversa, cuanto mayor es el flujo menor es la velocidad. Existe una velocidad mnima que puede adquirirse con este mtodo y que se presenta cuando circula por el circuito de campo la mxima corriente permitida por los devanados. Cuando un motor de c.c. funciona con sus valores de tensin, corriente de excitacin y potencia asignadas o nominales se dice que gira a la velocidad base (asignada). La regulacin por control de la tensin aplicada al inducido se realiza para velocidades inferiores a la velocidad base, pero no para velocidades superiores a ella, ya que requerira una tensin mayor que la asignada, por lo que podra daarse el inducido. La regulacin por control de la corriente de excitacin se emplea para velocidades superiores a la velocidad base. Cuando se requiere un accionamiento elctrico que tenga una regulacin de velocidad amplia (por ejemplo, del orden de 30 al), el motor ms adecuado es el de excitacin independiente, en el que se controla tanto la tensin del inducido como la corriente de excitacin de un modo independiente. Si la tensin en los bornes se puede regular suavemente desde cero hasta un valor mximo, es posible obtener una gama extensa de velocidades sin necesidad de emplear restatos de arranque, lo que hace que este sistema tenga un gran rendimiento. Existen varios esquemas de regulacin, pero el ms conocido se denomina sistema WardLeonard. Consiste en un motor asncrono trifsico que lleva acoplado mecnicamente un generador de c.c. de excitacin separada, cuyo ajuste permite obtener diferentes tensiones de salida en escobillas. El generador alimenta posteriormente el inducido de un motor de c.c. con excitacin independiente (Fig. 6.37). La tensin necesaria para la alimentacin de los inductores se obtiene de un pequeo generador en derivacin (excitatriz) acoplado al mismo eje mecnico del grupo motor de c.a.-generador de c.c. La polaridad del inductor del generador principal se puede invertir con objeto de proceder al cambio del sentido de marcha del motor de c.c. Durante la puesta en marcha se acta sobre el restato del generador R~ para que la tensin de salida del generador sea reducida y evitar as fuertes corrientes de arranque. Para una determinada corriente I suministrada por el generador y absorbida por el motor, la variacin de velocidad desde cero a un lmite inferior ni' que es la velocidad base se efecta a flujo constante en el valor correspondiente a plena excitacin (corriente de excitacin asignada), aumentando progresivamente la tensin del generador V hasta alcanzar la tensin asignada del motor. En estas condiciones, el par desarrollado por el motor es:
T

a)

= K/!JI = CI (constante)

(6.63)

524

MQUINAS ELCTRICAS

Excitacin del motor

Exeitatriz

~+-----

Generador de e.c. (dinamo)

Motor de e.e,

S----+f----

T-----4----Red de e.a.

a) ESQUEMA FSICO

Excitacin del motor Restato para ajustar la tensin de la excitatriz R ---Generador de e.e. Motor de e.e.

.... +-1----S----....._---T-----..... ---

b) ESQUEMA ELCTRICO

Figura 6.37.

Sistema de regulacin de velocidad Ward-Leonard.

Si se desprecia la cada de tensin en el circuito del inducido, la velocidad del motor viene expresada por: (6.64) donde e2 es una constante. La potencia desarrollada por el motor en estas condiciones ser: P=TZnn =

60

el - e2v = A . V
60

2n

(6.65)

en la que A representa una constante. De este modo, en esta fase, el par se mantiene constante y la potencia va aumentando linealmente con la velocidad, que a su vez va elevndose conforme aumenta la tensin aplicada V. Este modo de funcionamiento est representado por la parte izquierda de las curvas par, potencia mostradas en la Figura 6.38.

CAPTULO 6.

MQUINAS DE CORRIENTE CONTINUA

525

T P

Regin l (Par constante) Par T

Regin 2 , Potencia constante) ,

,,

, , , , , , ,, , , ,

" 'Potencia P : fII' :

"

)... ,," :
:

-----
base)
n2= 2n

, , , ,

ni (velocidad

Regulacin por control de tensin

Regulacin por control de flujo

Figura 6.38.

Regulacin de velocidad a potencia constante y a par constante.

A partir de la velocidad base ni' el control de la misma se lleva a cabo manteniendo constantela tensin aplicada (que en la etapa anterior haba alcanzado el valor asignado) y reduciendoel flujo del motor, y de este modo se cumple:
P =

v"J = costante

; T

= --

n 2n60

= v" = costante = KE<Pn

(6.66)

10 que indica que en esta segunda etapa la potencia se mantiene constante y que al mismo tiempo,siendo constante el producto del flujo por la velocidad (ecuacin de una hiprbola), el par se ir reduciendo de un modo hiperblico a medida que aumenta la velocidad. Esta situacinest representada en la parte derecha de la Figura 6.38, donde la velocidad del motor va aumentandohasta llegar a su valor mximo, definido por n2 El sistema Ward-Leonard se ha empleado ampliamente en la industria para el accionamientode los trenes de laminacin (fbricas siderrgicas), en los ascensores de velocidad media y elevada en grandes edificios, para accionar mquinas de arrastre de papel, gras, etc. Hoyda existen grupos Ward-Leonard estticos a base de convertidores electrnicos que suministran una tensin variable para alimentar tanto el inductor como el inducido del motor eliminando el sistema rotativo motor de c.a.-dinamo, lo que mejora el rendimiento de grupo y reduce el nivel de ruido.

6,9.2. Motor de c.c. con excitacin serie


El esquemade conexiones de este tipo de motor es el indicado en la Figura 6.39. El flujo de la mquina depende de la corriente del inducido J = J y en consecuencia depende de la carga. Si no haysaturacin en el circuito magntico, el flujo es directamente proporcional a la corriente f, y la caracterstica de carga se puede obtener de las ecuaciones bsicas (6.52), (6.53) y (6.54):
j,

V=E+RJj

(6.67)

526

MQUINAS ELCTRICAS

Restato de arranque

1 = I

1,

Inductor E

-1"

Figura 6.39.

Motor serie de c.c.

que en el supuesto de que se cumpla la proporcionalidad

<D = Kf resulta:
(6.68)

lo que conduce a una caracterstica del par, teniendo en cuenta (6.55):

(6.69)

La curva ti = f(T) tiene forma hiperblica y se aparta tanto ms de ella cuanto ms se satura la mquina. En la Figura 6.40 se muestra la representacin de esta caracterstica (el par de arranque es el que tiene la mquina para n = O,y que de acuerdo con (6.69) vale ~(/r = Ilja2V2 Si se desprecia la reaccin del inducido y la saturacin magntica, se puede considerar que al duplicarse el par aplicado al eje del motor, la corriente consumida por el mismo aumenta [de

n (velocidad)

!.'/'I
Figura 6.40.

T(par)

Curvas par-velocidad de un motor serie.

CAPTULO

6.

MQUINAS DE CORRIENTE CONTINUA

527

acuerdo con la primera ecuacin (6.68)] slo un 140 por 100 del valor original, y el nmero de revoluciones, si se tiene en cuenta (6.69), cae hasta un 70 por 100 del valor primitivo. En un motor derivacin esta misma sobrecarga no altera prcticamente la velocidad, pero en cambio, la mquina consumira una corriente doble de la inicial. El motor en serie puede, por tanto, soportar elevadas sobrecargas, aumentando con ello slo moderadamente la corriente. Esto constituye su ms valiosa propiedad. Al disminuir el par resistente, el motor reduce lentamente su consumo de corriente, aunque su velocidad se eleva rpidamente y, para cargas inferiores al 25 por 100 de la asignada, esta velocidad adquiere valores peligrosos para la integridad del motor. Por esta razn el motor serie no debe ser arrancado en vaco o con una carga pequea. Las propiedades tan valiosas de este motor lo hacen apropiado para la traccin elctrica: trenes, tranvas, trolebuses y tambin en gras donde son necesarios altos pares a bajas velocidades y viceversa. La regulacin de la velocidad de estos motores, a diferencia con el motor derivacin, se realiza solamente por control de la tensin aplicada al motor. Este procedimiento puede realizarse de manera econmica si se dispone por lo menos de dos motores (pueden ser tambin cuatro o seis), como sucede en los ferrocarriles elctricos urbanos o interurbanos. Cada coche motor va equipado con dos motores serie, uno acoplado al boje (o bogie) delantero que impulsa las ruedas motrices delanteras y otro acoplado al boje trasero impulsando sus respectivas ruedas traseras (Fig. 6.41). Las velocidades de ambos motores son iguales en todo momento. La variacin de velocidad se consigue con la conexin serie-paralelo de ambos motores, de esta forma pueden obtenerse dos velocidades bsicas de trabajo con un buen rendimiento energtico. Inicialmente los motores estn conectados en serie a travs de una resistencia variable que se va eliminando gradualmente mediante un combinador de tambor* hasta que se obtiene una tensin en bornes de cada motor, mitad de la lnea. Con ello se obtiene la primera posicin de marcha. En este momento, al no existir ninguna resistencia externa en el circuito, se obtiene un gran rendimiento del conjunto. Cuando se desea aumentar la velocidad del vehculo se cambia la conexin en serie de los motores y se pasa a paralelo insertando al mismo tiempo entre ellos y la lnea una resistencia exterior. Esta resistencia se va eliminando poco a poco hasta que los motores funcionan a plena tensin de lnea, obteniendo la segunda posicin estable de funcionamiento. Para comparar ambos tipos de conexiones se van a suponer despreciables las resistencias de los circuitos de ambos motores y se va a admitir que el circuito magntico no est saturado. De esta forma las ecuaciones generales del par y la velocidad en cada mquina son: n= V-R/
~ -'-

KEct> KEct> KEKf T = KTct>l = KTKI2 = ei2

= --'-

el .:

1;
(6.70)

donde el y el son constantes, K, expresa la proporcionalidad entre el flujo y la corriente y V indica la tensin en bornes de cada motor. Si se considera una carga que ofrece un par constante, de acuerdo con (6.70) las corrientes del inducido en cada tipo de acoplamiento son

Un combinador (controller) consiste en un tambor metlico con conexiones adecuadas para realizar por medio de u*na manivela de accionamiento las operaciones de arranque, regulacin de velocidad e inversin en los motores de traccin. Hoy da se ha sustituido totalmente por dispositivos automticos electrnicos.

528

MQUINAS ELCTRICAS

Catenaria y alimentacin

<
R, ~

I~

I~

Bogie l

Bogie 2

Carril

VI2

Carril
a) CONEXIN SERIE DE LOS MOTORES b) CONEXIN PARALELO DE LOS MOTORES

Figura 6.41.

Conexin en serie y en paralelo de motores serie de c.c. en traccin elctrica.

idnticas, resultando el esquema de reparto de intensidades que se muestra en la Figura 6.41, y se cumple:
a)

CONEXIN SERIE: Vi

= VI2
n =C1s

t, = T
VI2 1 Ti = 1 (6.72)

(6.71)

b)

CONEXIN PARALELO: Vi = V

Es decir, a igualdad de par, con la conexin paralelo puede obtenerse una velocidad doble que con el acoplamiento serie.

6.9.3. Motor de c.c. con excitacin compuesta


El esquema de conexiones de este motor es el indicado en la Figura 6.42a. El devanado de excitacin serie puede conectarse de forma que refuerce el campo derivacin (aditivo) o que se oponga al mismo (diferencial). La corriente del devanado derivacin es constante, mientras que la intensidad del arrollamiento serie aumenta con la carga; de esta forma, se obtiene un flujo por polo que aumenta tambin con la carga, pero no tan rpidamente como en el motor serie. La caracterstica mecnica de estos motores es la mostrada en la Figura 6.42b, y es intermedia entre las curvas del motor derivacin y serie.

CAPTULO

6.

MQUINAS DE CORRIENTE CONTINUA

529

+
v

Restato de arranque
-J

11 (velocidad)

Inductor paralelo

a)

b)

Figura 6.42.

Motor con excitacin compuesta. Curvas par-velocidad de los motores de c.c.

EJEMPLO

DE APLICACIN

6.7

Un motor tipo derivacin de 240 V tiene una resistencia de inducido (incluyendo las escobillas y los polos auxiliares) de 0,04 Q y una resistencia de campo o excitacin de 100 Q. a) Hallar el valor de la resistencia que debe aadirse al circuito inductor para incrementar la velocidad de 1.200 a 1.500 r.p.m. cuando la corriente de alimentacin sea de 200 A. b) Con una resistencia de campo como en el caso a), hallar la velocidad cuando la corriente de alimentacin sea de 100 A. Si la mquina gira como generador para dar 200 A a 240 V, hallar: c) la corriente de campo a 1.200 r.p.m., d) la velocidad cuando la corriente de excitacin sea de 2 A. NOTA: La curva de magnetizacin se supone lineal.

SOLUCIN
a)

El esquema de la mquina es el mostrado en la Figura 6.43. Con Re = 100 Q la corriente de excitacin vale 2401100 = 2,4 A. Si la corriente de alimentacin es de 200 A, la intensidad que atraviesa el inducido es:
/=I-/e=200-2,4=

197,6 A

-J

R,

v = 240 voltios
Inductor

Figura 6.43.

530

MQUINAS ELCTRICAS

En consecuencia, la f.e.m. del motor en estas condiciones ser: E = V - RJ = 240 - 0,04 . 197,6 = 232,1 V De acuerdo con la expresin (6.54), la f.e.m. es igual a: E = KEn<D y como quiera que se considera lineal la curva de magnetizacin, se podr poner:

Es decir, la f.e.m, es directamente proporcional a la velocidad y a la corriente de excitacin. Si se denominan E' e 1; los valores de la f.e.m. y corriente en el inductor a 1.500 r.p.m., resultar la siguiente proporcionalidad:
E E'

n(, n'/,:'

232,1
E'

1.200 . 2,4 1.500' 1;

y por tanto:
E' = 120,88

t;

(1)

Ahora bien, las magnitudesE' e 1; respondena las siguientesecuacionesde circuito:


E'

V - RJ

; I, = 1 - 1;

y se puede poner: E' = V - R (I - Ji) que sustituyendo valores conduce a la ecuacin:


E'

= 240 -

0,04 (200 - 1;) = 232 + 0,04 . ;

y teniendo en cuenta (1) se obtienen los valores:


E' = 232,1 V ; 1; = 1,92 A

En consecuencia, la resistencia total del circuito de excitacin ser: R'=-=-=125n e /,: 1,92 Como el inductor tena 100 n de resistencia, habr que aadir una resistencia en el circuito de 125 - 100 = 25 n. b) La corriente del inducido ser en este caso:
I = 1 - 1,: = 100 - 1,92 = 98,08 A V 240

que conduce a una f.e.m. E":


El! = V - RJ = 240 - 0,04 . 98,08 = 236,07 V

Y aplicando ahora la relacin de proporcionalidad de fe. m.s. queda: 1.200' 2,4


n": 1,92

de donde resulta una velocidad de 1.525,6 r.p.m.

CAPTULO

6.

MQUINAS DE CORRIENTE CONTINUA

531

c)

Denominando E~a la f.e.m. del generador, al aplicar la proporcionalidad de f.e.m.s. resulta: E


E~

nI"
n~/<,g

232, l

1.200 . 2,4 1.200 . l.,

e,

es decir: Ee = 96,71 leg' En un generador se cumplen adems las siguientes ecuaciones de circuito: 1, = I + leg ; E = V + RJ = V + R(1 + I"g) Sustituyendo valores queda: E~ = 240 + 0,04 (200 + Ieg) con la expresin E, = 96,71 leg dan como soluciones:
Eg
d)

= 248 + 0,04 ('R' que junto

= 248,1 V

; leg

= 2,56 A

En este caso, la corriente de inducido es igual a 200 + 2 = 202 A, que conduce a una f.e.m. E;:
E~ = 240 + 0,04 . 202 = 249,08 V

y al aplicar la proporcin de f.e.m.s. se obtiene: 232,2 248,08


EJEMPLO DE APLICACIN

1.200' 2,4
/12

; n = 1.539,1 r.p.m.

6.8

Un motor tipo serie de /10 V cuando gira a 1.500 r.p.m. y consume 35 A desarrolla un par electromagntico de 20 N.m. La resistencia del motor es de 0,3 Q. A qu velocidad girar el motor: a) cuando el par desarrollado sea de 15 N.m, suponiendo que elflujo sea proporcional a la corriente; b) cuando lo corriente sea el doble, suponiendo que el flujo aumente un 30%? SOLUCIN
a)

El esquema de circuito es el que se muestra en la Figura 6.44. La f.c.e.m. del motor cuando absorbe una corriente de 35 A es: E = V - R/ = 110 - 0,3 . 35 = 99,5 V

+ v=
110 voltios

I=I R=O,3!l Inductor

-1,.

Figura 6.44.

532

MQUINAS ELCTRICAS

Igualando la potencia electromagntica a la potencia mecnica, resulta:


2nn EI=TI

60

99535 ,

2nn = 20-

60

es decir, n = 1.662,8 r.p.m. Si se denominan E', l/ y n' la f.c.e.m., la corriente del inducido y la velocidad de rotacin del motor cuando desarrolla un par de 15 N.m, se cumplir: E'!' = T' - 60 = (V - RI')!'
I 11

2nn'

que al sustituir valores da:


2nn' 15 . = (1 10 - O 3 . I')/.'

60

'

(1)

Teniendo en cuenta que en el supuesto de considerar el flujo proporcional a la corriente, la f.e.m. puede ponerse: E

= KEn<l> = KJn
1.662,8 . 35 I:n'
=>

y al comparar las f.e.m.s. en los dos regmenes de funcionamiento se obtiene:

E
E' es decir:

ni

99,5
=>

E'

t;

E'

;:::; -/.'n'
I

1 585

E' = 110 - 0,31 . I' = I

1 I'n' 585
I

y teniendo en cuenta la relacin (1), resultan los siguientes valores:

l/ = 30,31 A
b)

; n' = 1.947,3 r.p.m.

Si la corriente es el doble, la f.c.e.m. tendr un valor:

E"

= V-

RJ/'

= 110 - 0,3 (2 . 35) = 89 V

Al expresar las f.c.e.m.s. en funcin de los flujos, resulta:

y teniendo en cuenta que <1>" = 1,3 <1>, queda la relacin:

99,5
89

1.662,8
1,3n"

que da un valor de 1.144,1 r.p.m.

6.10. MOTOR DE C.C.: MTODOS DE FRENADO


Cuando se utilizan los motores de c.c. en los accionamientos elctricos no; solamente se necesita arrancar y regular la velocidad con suavidad y precisin, sino que tambin a menudo se requiere parar el motor rpidamente. Este rgimen de funcionamiento se denomina frena-

CAPTULO

6.

MQUINAS DE CORRIENTE CONTINUA

533

do, y se puede realizar por procedimientos mecnicos o elctricos. En el primer caso se utiliza la fuerza de rozamiento entre una llanta rotrica acoplada al accionamiento y unas zapatas que la aprisionan con mayor o menor fuerza para conseguir la parada de la mquina en el momento requerido; en el segundo caso se produce el frenado elctrico haciendo funcionar el motor de c.c. como generador cuya energa suministrada procede de la energa cintica de todas las masas giratorias acopladas al rotor. Una particularidad del frenado elctrico es que su accin desaparece cuando la mquina se detiene, por lo que deben utilizarse en combinacin con frenos mecnicos para conseguir mantener la carga del accionamiento en reposo o paro total. El frenado elctrico que se explica en este epgrafe se aplica normalmente en traccin elctrica y en los aparatos de elevacin y transporte, como es el caso de ascensores, montacargas, gras y otros. Existen tres mtodos de frenado elctrico, a saber: a) regenerativo o por recuperacin de energa, b) dinmico o reosttico y e) a contracorriente.

a) Frenado regenerativo o por recuperacin de energa (regenerative braking):


Este rgimen de frenado solamente es posible si la mquina de c.c. que funciona en rgimen motor se mueve por el mecanismo accionado con una velocidad superior a Ia-correspondiente a su marcha en vaco no. Considrese, por ejemplo, un motor de c.c. con excitacin independiente (o tambin derivacin) como se muestra en la Figura 6.45a; si se denomina R a la resistencia total del circuito del inducido, el valor de la corriente absorbida por la mquina en las condiciones de funcionamiento como motor viene expresada por: V-E _ __ V-KEnIP r_ - __ =--_
1

(6.73)

En la expresin anterior E es la f.c.e.m. del motor, cuyo valor es inferior a la tensin aplicada V, por lo que la corriente absorbida I tiene el sentido mostrado en la Figura 6.45a. De acuerdo con la expresin (6.59), la caracterstica par-velocidad de la mquina en rgimen motor viene definida por: (6.74)
Restato de

+
v

-J

a) Motor con excitacin independiente

b) Curvas par-velocidad

Figura 6.45.

Motor con excitacin independiente. Frenado regenerativo o por recuperacin de energa.

534

MQUINAS ELCTRICAS

que es la ecuacin de una recta en un sistema de coordenadas n = f(T) cuya pendiente es proporcional a la resistencia total del circuito del rotor. Si la resistencia adicional de arranque es cero, la resistencia del inducido es R, = R Yla recta correspondiente es la CBAD sealada en la Figura 6.45b, que se denomina caracterstica mecnica natural del motor. En esta misma figura se muestran otras dos rectas con mayor resistencia en el circuito del inducido, R2 y RJ. Si se considera un rgimen permanente en el que el inducido no tiene ninguna resistencia adicional en serie y se denomina T, al par resistente de la carga, el punto correspondiente de funcionamiento en esta situacin se encuentra en el primer cuadrante de la caracterstica par-velocidad del motor, y es el sealado por A en la Figura 6.45b. Para un determinado flujo inductor <D, existe una velocidad para la cual la f.c.e.m. E del motor es igual a la tensin de red V y cuyo valor es la velocidad de vaco no, de tal modo que segn (6.73) se cumple: (6.75) La mquina funciona entonces en el punto de trabajo B, lmite entre el primero y segundo cuadrante de la curva par-velocidad de la Figura 6.45b. En este punto de trabajo, y de acuerdo con (6.73), la corriente del inducido I, es igual a cero, por lo que el motor no puede desarrollar ningn par electromagntico en el eje (ya que T = KrcJ>1 = O). Ahora bien, si por cualquier procedimiento se hace girar la mquina a una velocidad n superior a la de vaco no' es evidente que la f.c.e.m. del motor E ser superior a la tensin aplicada V, de tal modo que la corriente I, del inducido cambiar de signo respecto al mostrado en la Figura 6.45a. En esta situacin la mquina de c.c. trabajar como generador y la red actuar como un receptor de energa, frenando a la mquina. La ecuacin caracterstica parvelocidad vendr definida por la ecuacin (6.74) (pero en la zona negativa de los pares), que es en definitiva la continuacin de la caracterstica mecnica en rgimen motor que se extiende al segundo cuadrante del sistema de coordenadas n = f(T). Es por ello que en rgimen de frenado regenerativo, el punto de trabajo en la curva par-velocidad se traslada al segundo cuadrante y se ha sealado por C en la Figura 6.45b. La mquina funcionar como generador en el tramo BC de la caracterstica par-velocidad. Debe sealarse tambin que cuanto mayor sea la resistencia R del circuito del inducido, tanto mayor ser la velocidad de rotacin en rgimen generador para un mismo par de frenado, como as lo puede comprobar el lector en la Figura 6.45b para diversos valores de las resistencias de inducido. Este tipo de frenado se utiliza en la traccin elctrica cuando un tren baja por una pendiente elevada. Al superar el tren la velocidad de vaco no, se devuelve energa a la red y esta energa es aprovechada por los diversos ferrocarriles que estn circulando por la misma va alimentados por la misma catenaria que el tren considerado. Tambin este tipo de frenado es posible en los accionamientos de mquinas de elevacin y transporte durante el descenso de las cargas, limitando la velocidad de cada de las mismas. El frenado regenerativo de un motor serie es algo ms complicado. Tngase en cuenta que las curvas par-velocidad de un motor serie estn limitadas al primer cuadrante, y obsrvese en la Figura 6.40 que en este tipo de motores, cuando el par tiende a cero la velocidad tiende a un valor muy elevado. Esto significa que el motor serie no puede pasar por s mismo del rgimen motor al rgimen de frenado regenerativo aumentando simplemente la velocidad. Esto se debe a que al aumentar la velocidad se produce, segn indica la ecuacin (6.69), una reduccin de par, lo que se traduce, segn seala (6.68), en una reduccin de la corriente del inducido, que es la misma que atraviesa los polos inductores del motor y de este modo el flujo

CAPTULO

6.

MQUINAS DE CORRIENTE CONTINUA

535

inductor disminuye, por lo que la f.c.e.m. del motor E se acerca ms al valor de la tensin de alimentacin de la red V, pero sin conseguir sobrepasarla. Es por ello que para producir el frenado regenerativo de un motor serie debe desconectarse el devanado de excitacin de su unin con el inducido y se debe conectar a un pequeo generador externo, que puede ser incluso la propia red, aunque en este caso, teniendo en cuenta que en un motor serie el devanado inductor tiene poca resistencia elctrica, debe limitarse la corriente que circula a su travs, introduciendo una resistencia externa en serie con este devanado; de este modo el comportamiento del motor serie en rgimen de frenado regenerativo es similar al de motor con excitacin independiente comentado anteriormente.

b) Frenado reosttico o dinmico (rheostatic o dynamic braking)


El frenado dinmico hace uso del comportamiento de un motor como generador. Si se desconecta el inducido de un motor de la red de alimentacin mientras permanece conectada la excitacin, la mquina no producir par motor (ya que l = O) Ydebido a la accin del par resistente se acabar parando al cabo de un cierto tiempo, que depender de la energa cintica almacenada en el sistema rotativo. Pero si al separar el inducido de la red se carga inmediatamente con una resistencia de carga, manteniendo en todo momento la excitacin del inductor, la accin de frenado aumentar enormemente. En esta situacin la energa producida por el motor que funciona como generador es transformada en calor por efecto Joule en las resistencias de carga conectadas al inducido del motor. En la Figura 6.46a se muestra el esquema elctrico correspondiente para un motor con excitacin independiente. Cuando el conmutador S de la Figura 6.46a est situado en la posicin 1, el inducido se alimenta de la red de c.c. y la mquina funciona en rgimen motor absorbiendo una corriente I, de la red en el sentido sealado. Al pasar el conmutador S a la posicin 2 se carga el inducido sobre una resistencia externa Rex,' lo que provocar una inversin en el sentido de la corriente. Si se considera que en el momento de la conmutacin la f.c.e.m. del motor es igual a E, el valor de la corriente I, es:
I=---I R + Rex'

(6.76)

Restato de arranque
-/;

n (velocidad)

+ v.
a) Motor con excitacin independiente b) Curvas par-velocidad

Figura 6.46.

Frenado reosttico de un motor de c.c. con excitacin independiente.

536

MQUINAS ELCTRICAS

es decir, tiene un valor negativo y por consiguiente el par de frenado desarrollado por la mquina es negativo y de valor:
(6.77)

esto es, el par de frenado es proporcional a la velocidad e inversamente proporcional a la resistencia total del circuito del inducido R +Rexr . De la ecuacin anterior se obtiene el valor de la velocidad n: n=T(R

+ Rexr) KTKEct>2

(6.78)

En este caso de frenado reosttico o dinmico, las caractersticas par-velocidad para diversas resistencias de carga externas son las mostradas en la Figura 6.46b, que son rectas que pasan por el origen de coordenadas y que tienen lugar en el segundo cuadrante. Supngase que la mquina est funcionando como motor en el punto de trabajo A. En el momento de conectar el inducido sobre la resistencia externa Rexr comienza el rgimen de frenado dinmico, y como debido a la inercia del rotor, la velocidad no puede cambiar instantneamente, no habr variacin en la f.c.e.m. del motor y el punto de funcionamiento se traslada a D, en el que se tiene un par de frenado TI' A partir de este momento la velocidad de rotacin se va reduciendo siguiendo la recta DO, hasta llegar a pararse en el punto O. En este momento debe desconectarse el motor de la red, ya que en caso contrario la mquina puede comenzar a girar en sentido contrario siguiendo el tramo de la recta OF del cuarto cuadrante. Para conseguir el frenado dinmico de un motor serie hay que tener en cuenta que si se conserva el mismo sentido de rotacin y las mismas conexiones entre el inductor y el inducido, la mquina en rgimen generador es imposible de cebar, ya que la corriente inductora tiende a anular el magnetismo remanente de los polos. Una mquina serie slo se puede cebar como generador si su sentido de rotacin es opuesto al que tena como motor. Si se desea cebar con el mismo sentido de rotacin que tena funcionando como motor, es necesario invertir las conexiones entre el inductor y el inducido, tal como se seala en la Figura 6.47, para conservar de este modo el sentido de la corriente inductora (pese a la inversin de la corriente en el inducido) que tena en rgimen motor. En algunos casos el frenado dinmico se realiza separando los circuitos inductor e inducido y funcionando el conjunto como una mquina con excitacin independiente, de un modo anlogo al sealado anteriormente.

+
v

I-

Inductor

Inducido

t,
Inducido
b) Conexiones en rgimen de frenado dinmico

a) Conexiones como motor serie

Figura 6.47. Conexiones para el frenado reosttico de un motor serie de c.c.

CAPTULO

6. MQUINAS DE CORRIENTE CONTINUA

537

e)

Frenado a contracorriente (counter current braking o plugging)

Este procedimiento de frenado se emplea cuando es necesario parar repentinamente un motor para luego poder invertir su marcha, como en ciertas aplicaciones de trenes de laminacin de aceras y gras de puerto. Sin alterar las conexiones del inductor, se aplica tensin al inducido en sentido inverso, lo que ocasiona un frenado brusco, el motor se para y luego inicia la marcha en sentido inverso. En algunos casos se abre el interruptor principal en el instante en que el rotor pasa a la situacin de reposo, o sea, justamente antes de que el motor comience a girar en sentido contrario. En la Figura 6.48a se muestra el esquema elctrico de frenado a contracorriente o contramarcha de un motor con excitacin independiente. Inicialmente el conmutador S est en la posicin 1 y la mquina funciona como motor absorbiendo el inducido una corriente I en el sentido mostrado en la Figura 6.48a. Si la resistencia limitadora de la corriente del inducido es R, = 0, el valor de la corriente anterior es: 1=-1

V-E
R

(6.79)

lo que da lugar a la curva par-velocidad expresada en (6.74), y que para un par resistente T, corresponde al punto de funcionamiento A de la caracterstica mecnica natural de la Figura 6.48b. Cuando el conmutador S pasa a la posicin 2, se invierte la polaridad de la tensin aplicada al motor, y como en los primeros instantes el rotor continuar girando en el mismo sentido que llevaba previamente, la f.c.e.m. del motor E permanecer invariable y se producir una corriente de sentido contrario al sealado en la Figura 6.48a, cuyo valor es: I=--1

V+E

R+Rr

(6.80)

donde R, es una resistencia que se coloca en serie con el inducido para limitar la fuerte corriente producida en el perodo de frenado y que en la prctica suele emplearse una resis-

n (velocidad) R;
-Ji

Restato limitador

+
{l

s
v---u...

V
2

~ _o---<Y'~

+ v.

a) Motor con excitacin independiente

b) Curvas par-velocidad

Figura 6.48.

Frenado a contracorriente de un motor de c.c. con excitacin independiente.

538

MQUINAS ELCTRICAS

tencia doble que la utilizada para limitar la corriente de arranque en rgimen motor. La caracterstica par-velocidad en este perodo de frenado viene definida por: (6.81) que corresponde a la recta DFG que transcurre por el segundo y tercer cuadrante de las caractersticas mecnicas de la Figura 6.48b, en la que se observa que para T = O se tiene
n = -(V/KE<l = -no'

Si se supone que la mquina est trabajando inicialmente como motor en el punto A en la caracterstica mecnica natural, al producirse la conmutacin, adems de cambiar la polaridad de la tensin aplicada a la mquina se introduce la resistencia limitadora R, en el circuito del inducido, y la mquina pasar a trabajar en el punto D del segundo cuadrante, ejerciendo un par de frenado TI' Este par de frenado provoca una deceleracin del rotor y la velocidad de la mquina se va reduciendo siguiendo la recta DF. Al pasar el motor por el punto F (velocidad cero) se desconecta el motor de la red. En caso de que no se realice esta operacin, la mquina se ir acelerando en sentido contrario. Un proceso anlogo al anterior se emplea para frenar un motor serie. En cualquier caso, el frenado a contracorriente es un mtodo de muy bajo rendimiento debido a la gran cantidad de energa disipada en las resistencias colocadas en serie con el inducido.

6.11. FUNCIONAMIENTO DE UNA MQUINA DE C.C. EN CUATRO CUADRANTES


Como ha comprobado el lector en los epgrafes anteriores, el motor de c.c. es una mquina elctrica muy verstil que permite una excelente regulacin de velocidad y que puede funcionar con ambos sentidos de giro, produciendo pares electromagnticos a favor o en contra del sentido de rotacin, y es por ello que ha sido la mquina motriz por excelencia en aplicaciones de velocidad variable, conocidas modernamente como accionamientos elctricos (electric drives en ingls): traccin elctrica, trenes de laminacin, maquinaria de elevacin y transporte y otros. Para describir los modos de funcionamiento de una mquina de c.c. se utiliza la representacin en cuatro cuadrantes mostrada en la Figura 6.49. En el eje de abscisas se representa el par electromagntico que genera la mquina y que tambin sirve para representar la corriente que atraviesa el inducido, al ser proporcional el par a esta corriente. En el eje de ordenadas se representa la velocidad de giro, bien sea en rad/s o en r.p.m.; si se supone el flujo inductor constante, la f.c.e.m. del motor es proporcional a la velocidad, por lo que tambin el eje de ordenadas puede servir para representar la f.c.e.m. del motor, y si se tiene en cuenta que, debido a la pequea cada de tensin en el inducido, se cumple E :::::; V, el eje de ordenadas representa tambin el eje de tensiones. Es decir, hay proporcionalidad de las caractersticas mecnicas n = j(T) con V = f2(J). De acuerdo con lo anterior, en la Figura 6.49 se ha sealado el eje de abscisas con las magnitudes Te I, y el eje de ordenadas con n y V. Estas magnitudes se toman como positivas en el primer cuadrante, que corresponde al funcionamiento como motor girando en sentido directo o positivo (antihorario). La potencia mecnica desarrollada por el motor es el producto del par por la velocidad angular de giro, de tal modo que si estas dos magnitudes son positivas, su producto ser positivo, indicando con ello que la potencia mecnica es positiva

CAPTULO

6.

MQUINAS DE CORRIENTE CONTINUA

539

n,V

-li

_Ii

~V(red)

E -

+
Figura 6.49. Funcionamiento de la mquina de c.c. en cuatro cuadrantes.

cuando sale energa mecnica por el eje. De un modo anlogo, si Ve I son positivos, ser tambin positivo su producto, lo que indica que la potencia elctrica es positiva cuando la mquina absorbe energa de la red. Para facilitar el estudio se ha incluido en cada cuadrante de la Figura 6.49 el circuito del inducido con los sentidos de la corriente y las polaridades tanto de la f.c.e.m. del motor como de la tensin aplicada a la mquina. En el cuadrante 1, la velocidad y el par son positivos, y por consiguiente son positivos tanto la tensin como la corriente del inducido. De este modo es positiva la potencia elctrica que absorbe la mquina y tambin la potencia mecnica que sale del eje del motor. Se produce una transformacin de energa elctrica en energa mecnica. Cuando la mquina trabaja en el cuadrante 2, la velocidad es positiva (giro directo) y por ello son tambin positivas tanto la tensin aplicada como la f.c.e.m, del motor; el par electromagntico y la corriente de inducido son negativos. En consecuencia, la mquina absorbe potencia mecnica por el eje (valor negativo) y se devuelve energa elctrica a la red (valor positivo). Se ha producido un cambio en el sentido del flujo de energa en la mquina, que ahora funciona como generador en rgimen de frenado regenerativo o por recuperacin, aunque tambin esta energa elctrica puede disiparse en una resistencia externa, como sucede en el frenado reosttico o dinmico. En el cuadrante 3, tanto el par como la velocidad son negativos, por lo que son negativos la tensin aplicada y la corriente absorbida. De este modo tanto la potencia mecnica como la potencia elctrica son magnitudes positivas. La mquina trabaja como motor girando en sentido inverso. Es evidente que para que se invierta el sentido de la corriente se debe cumplir que IVI > IEI, y de este modo el flujo energtico se dirige de la red al motor. Cuando la mquina trabaja en el cuadrante 4, el par es positivo y la velocidad negativa, por lo que la potencia mecnica es negativa y procede del mecanismo accionado por el motor. La f.c.e.m.

540

MQUINAS ELCTRICAS

Tabla 6.2. Signos de las diversas magnitudes en los cuatro cuadrantes

Motor directo Frenado directo

+ +

+ +

+ +

Motorinverso
Frenado inverso

3
4

es negativa y la corriente del inducido es positiva, lo que se verifica si IEI > IVI, y se devuelve energa elctrica a la red. La mquina funciona en rgimen generador como freno regenerativo en sentido inverso, aunque tambin se puede frenar la mquina disipando la energa elctrica sobre unas resistencias externas. En la Tabla 6.2 se resumen los conceptos anteriores, sealando los signos correspondientes de las diversas magnitudes implicadas, tanto elctricas como mecnicas. Debe destacarse que el uso de restatos de arranque y regulacin por medio de resistencias elctricas sealados en este captulo, aunque son mtodos muy didcticos para comprender su aplicacin en el control de las mquinas de c.c., en la actualidad han cado en desuso, debido a que en estas resistencias se disipan grandes cantidades de energa en calor por efecto Joule, y de ah los bajos rendimientos de estos accionamientos. En la actualidad, con el gran avance de la electrnica de potencia, el control de los motores de c.c. se realiza con convertidores electrnicos: rectificadores fijos, rectificadores controlados y choppers, cuyo estudio especfico se realiza en el Captulo 7, dedicado a los accionamientos elctricos. Se puede adelantar aqu, a modo informativo, que los rectificadores convierten la c.a. en c.c. y solamente permiten el trabajo de la mquina de c.c. en el primer cuadrante. Los rectificadores controlados transforman la c.a. de la red en c.c. regulable, que incluso pueden modificar el signo de la tensin de salida variando una seal de control de encendido, y es por ello que pueden trabajar en los cuadrantes 1 y 4; incluso aadiendo un grupo dual en oposicin, una mquina de c.c. puede trabajar en los cuatro cuadrantes. El chopper es otro tipo de convertidor electrnico que transforma una c.c. en otra c.c. de valor medio regulable, permitiendo el trabajo de una mquina de c.c. en dos o en cuatro cuadrantes. Estos convertidores electrnicos se utilizan con profusin en traccin elctrica. Debe destacarse que desde la dcada de 1980, y debido a la incorporacin del microprocesador en los equipos de control industrial, se utilizan tambin en los accionamientos elctricos de velocidad variable los motores de c.a. asncronos y sncronos, de tal modo que se puede conseguir el funcionamiento en cuatro cuadrantes de un modo similar a los motores de c.c. Los mtodos de regulacin se explican en el Captulo 7, dedicado a los accionamientos elctricos.

6.12. MOTOR MONOFSICO DE C.A. CON COLECTOR DE DELGAS


Un motor de c.a. con colector es bsicamente un motor de c.c. tipo serie que se ha adaptado a la corriente alterna mediante un adecuado diseo de su estructura elctrica y magntica; su

CAPTULO

6.

MQUINAS DE CORRIENTE CONTINUA

541

principio de funcionamiento se basa en la propiedad que tiene un motor de c.c. de conservar un sentido de giro determinado, cualquiera que sea la polaridad de la tensin de alimentacin. En la Figura 6.50 puede apreciarse esta propiedad con una alimentacin de c.a. En el caso a) se representan los sentidos de las corrientes en el del inducido y los polos para el semiperodo positivo de la onda senoidal. En la Figura 6.50b se muestran estos mismos sentidos para el semiperodo negativo de la onda. En ambos casos el par resultante tiene sentido contrario a las agujas del reloj. A diferencia con los motores de c.c., los motores de c.a. con colector se construyen con chapas magnticas para reducir las prdidas en el hierro que aparecen debido a la alternancia del flujo. Generalmente se emplea el motor tipo serie, debido a que el par que se obtiene en este caso es superior al tipo derivacin (por ser superior el factor de potencia). Con la disposicin serie se tiene una reactancia del devanado inductor inferior a la que existira con el tipo derivacin; hay que tener en cuenta que el coeficiente de autoinduccin es proporcional al cuadrado del nmero de espiras, y como se ha explicado en este captulo, este nmero es inferior en un inductor serie que en un inductor derivacin, por lo que el motor serie presentar un mejor f.d.p. que el motor derivacin, lo que se traduce en una mayor magnitud del par de carga. Para mejorar an ms el f.d.p. del motor se emplean alimentaciones de baja frecuencia que se obtienen de la red principal de 50 Hz, duplicando o triplicando el perodo de la onda, resultando frecuencias de 50/2 = 25 Hz y 50/3 = 162/3 Hz. La f.e.m. inducida en los conductores del rotor de estos motores se debe en parte, a la variacin del flujo inductor (f.e.m. por accin transformadora) y en parte al movimiento del rotor (f.e.m. de movimiento o velocidad). La existencia de esta doble f.e.m., hace que la conmutacin sea difcil. Para mejorar la conmutacin se reduce la reaccin del inducido empleando arrollamientos de compensacin, al igual que se haca en los motores de c.c. En la Figura 6.51 se muestra el esquema simplificado de un motor serie de c.a. que incorpora un arrollamiento compensador que va conectado en serie con el inducido. De todas formas, pese a la utilizacin de los devanados compensadores, la conmutacin en estos motores suele ser difcil debido a la aparicin de la f.e.m. transformadora en la bobina cortocircuitada por las escobillas, que produce una corriente de circulacin intensa que origina un fuerte chisporroteo en el colector. Para mejorar la conmutacin se suelen intercalar resistencias entre la unin de dos bobinas y el colector para reducir la corriente de cortocircuito, y tambin se recurre al empleo de

+
a) Semiperiodo Oa T/2

+
b) Semiperiodo T/2 a T

Figura 6.50.

Motor monofsico con colector de delgas. Principio de funcionamiento.

542

MQUINAS

ELCTRICAS

Red

0v

Devanado principal

Figura 6.51.

Motor de c.a. serie con devanado de compensacin.

tensiones reducidas en el colector para que disminuya la d.d.p. entre delgas consecutivas, y de ah que estos motores tengan una tensin asignada lmite entre 250 y 300 V. Otro procedimiento para mejorar la conmutacin es emplear frecuencias de alimentacin reducidas, que disminuyen la f.e.m. de accin transformador y que contribuyen adems, como se ha indicado en los prrafos anteriores, a mejorar el f.d.p. de la mquina. Los motores serie de c.a. con colector tienen sus mayores aplicaciones en la traccin elctrica. En la Europa central (Alemania, Suiza, Dinamarca) se emplean con frecuencias de 162/, Hz. En EE. UU. se utilizan 25 Hz. En Espaa se emplea corriente continua de 600 V a 1.500 V en trenes metropolitanos y de 3.000 V de c.c. en ferrocarriles de gran recorrido; sin embargo, el tren espaol de alta velocidad (AVE) tiene una alimentacin de c.a. monofsica a 25 kV, 50 Hz. El arranque de estos motores serie se realiza por medio de un transformador reductor cuyo primario recoge la tensin de la lnea de la catenaria y cuyo secundario tiene diversas tomas que van cerrndose por medio de contactares convenientemente regulados. En la Figura 6.52 se muestra esta idea. En el momento de la puesta en marcha se cierra el contacto 1,que aplica una pequea tensin al motor, lo que provoca el arranque del mismo, posteriormente se irn cerrando los sucesivoscontactos, teniendo la precaucin de ir abriendo con anterioridad los precedentes para no dejar en cortocircuito ninguna parte del transformador.

'V Pantgrafo

Motor serie de c.a.

Figura 6.52.

Arranque de un motor de c.a. monofsico en traccin elctrica.

CAPTULO

6.

MQUINAS DE CORRIENTE CONTINUA

543

Al cerrar el contacto 4 se tiene aplicada la mxima tensin al motor y ste desarrollar la velocidad ms elevada. Este sistema de arranque tiene un gran rendimiento al no tener que disipar prdidas por efecto Joule en restatos de arranque como ocurra en el caso de los motores de c.c. Existen tambin motores serie de c.a. de pequea potencia, generalmente de una fraccin de CV, que se disean para funcionar indistintamente con c.c. o c.a. y que por ello reciben el nombre de motores universales. Se proyectan para tensiones comprendidas entre los 25 y 250 V, con frecuencias desde cero hasta 50 Hz. Los rendimientos suelen ser bajos (60-80 por 100) y las velocidades elevadas; generalmente la velocidad asignada es del orden de las 7.000 r.p.m., pero pueden obtenerse velocidades en vaco que lleguen a las 15.000-20.000r.p.m. Estos motores se utilizan con mucha frecuencia en aplicaciones domsticas: batidoras, taladradoras elctricas, etc.

PROBLEMAS
6.1. Un generador de c.c. de 4 polos tiene un inducido con 564 conductores que gira a 800 r.p.m., siendo el flujo por polo de 20 mWb. La corriente que circula por los conductores es igual a 60 A Calcular la corriente total, la f.e.m. y la potencia electromagntica desarrollada (El) si el devanado es: a) ondulado, b) imbricado. [Resp.: a) 120 A; 301 V; 36,12 kW. b) 240 A; 150,5 V; 36,12 kW.] Un motor de c.c. tetrapo1artiene un inducido bobinado con un arrollamiento imbricado de 888 conductores. Las escobillas estn desplazadas de la lnea neutra en sentido contrario al movimiento del rotor un ngulo de 5 grados geomtricos. Si la corriente total del inducido es igual a 90 A, calcular: a) Los amperivueltas por polo antagonistas y transversales. b) La corriente adicional necesaria en los polos para compensar la accin desmagnetizante, si el devanado de excitacin tiene 1.200 espiras/polo. [Resp.: a) 555 Av/polo; 4.440 A.v/polo. b) 0,4625 A]

6.2.

6.3. Estimar el nmero de espiras que necesita cada uno de los polos de conmutacin de un generador de 6 polos que suministra 200 kW a 200 V, sabiendo que el inducido tiene 540 conductores en conexin imbricada, el entrehierro interpolar es de I cm y la densidad de flujo en el mismo es de 0,3 Wb/rrr'. Despreciar la reluctancia del hierro y la dispersin. [Resp.: 10 espiras.] 6.4. Un generador tipo derivacin desarrolla una f.e.m. de 130 V. Cuando se conecta una carga, la tensin terminal baja a 120 V. Hallar la corriente de carga si la resistencia del circuito de campo es de 10 Q y la resistencia total del inducido de 0,05 Q. Prescndase de la reaccin del inducido. [Resp.: 188 A] 6.5. Un generador tipo derivacin de 4 polos, 1.500 r.p.m. tiene una curva de vaco (a velocidad asignada) definida por la siguiente tabla de valores:

o
6

0,1 20

0,4
80

0,6 120 200

1,14 220

1,32 240

1,56 260

2,4 300

3,04 320

Calcular: a) tensin en vaco que desarrollar la mquina cuando la resistencia total del circuito de excitacin sea de 125 Q; b) valor crtico de la resistencia del

544

MQUINAS ELCTRICAS

circuito de campo; e) tensin en vaco a 1.000 r.p.m. con una resistencia del inductor igual a 125 n; d) velocidad a la cual se hace crtica la resistencia de campo de 125 n. [Resp.: a) 298 V. b) 200 n e) 155 V; 925 r.p.m.] 6.6. Un generador tipo derivacin tiene una caracterstica de circuito abierto expresada por la ecuacin: E = 200le
K +Ie

Para una corriente de excitacin de 1,5 A se obtiene una f.e.m. en vaco de 150 V. Determinar el valor de la resistencia crtica del devanado inductor en derivacin y la tensin en vaco cuando la resistencia del campo es de 200 n. [Resp.: 400 n, 100 V.] 6.7. La caracterstica en vaco de un generador derivacin que gira a 1.200 r.p.m. es:
47

85
0,4

103
0,6

114
0,8

122

127 1,2

135
1,6

141
2,0

0,2

1,0

La resistencia del devanado inductor es de 55 n. Determinar: a) Resistencia que hay que aadir al circuito de campo para que la mquina genere una tensin de 120 V en vaco cuando gira a 1.200 r.p.m. b) Tensin en circuito abierto cuando se aade al inductor una resistencia de 20 y la velocidad se reduce a 800 r.p.m. [Resp.: a) 71 n b) 83 V.]

6.8.

Un generador tipo derivacin de 200 V debe mantener una tensin constante entre terminales para todas las cargas. A plena carga, la velocidad cae un 10 por 100 y la cada de tensin en el inducido es de 10 V. La corriente de excitacin en vaco es de 4 A Y la curva de vaco viene definida por la siguiente tabla de valores:
56 0,8

114
1,6

132
2

160

180

200 4

216 4,8

240 6,3

2,6

3,2

Hallar el cambio en la resistencia del circuito de excitacin desde vaco a plena carga. [Resp.: 50 a 34 n.] 6.9. Un generador tipo compuesto, de gran derivacin, suministra una corriente de carga de 50 A a 500 V Ytiene unas resistencias de inducido, campo en serie y campo en derivacin de 0,05; 0,03 n y 250 n, respectivamente. Calcular la f.e.m. generada y la corriente en el inducido. Considrese una cada de contacto de 1 V por escobilla. [Resp.: 506,2 V; 52 A] 6.10. Un motor tipo derivacin de 240 V tiene una resistencia del inducido de 0,2 n. Calcular: a) el valor de la resistencia que debe introducirse en el circuito del inducido para limitar la corriente de arranque a 40 A; b) f.e.m. generada cuando el motor est giran-

CAPTULO

6.

MQUINAS DE CORRIENTE CONTINUA

545

do a velocidad constante con esta resistencia adicional en el circuito para una corriente del inducido igual a 30 A. [Resp.: a) 5,8 Q. b) 60 V.] 6.1]. Un motor tipo derivacin de 250 V gira en vaco a 1.000 r.p.m. y absorbe una comente de 5 A. La resistencia total del inducido es de 0,2 Q y la del campo en derivacin de 250 Q. Calcular la velocidad cuando est cargado y tome una corriente de 50 A, sabiendo que la reaccin del inducido debilita el campo un 3 por 100. [Resp.: 994 r.p.m.]

6.12. Un motor tipo derivacin de 250 V tiene una resistencia de inducido de 0,5 Q Yuna resistencia de campo de 250 Q. Cuando mueve a 600 r.p.m. una carga cuyo par es constante, el inducido absorbe 20 A. Si se desea elevar la velocidad de 600 a 800 r.p.m., qu resistencia debe insertarse en el circuito de excitacin, suponiendo que la curva de magnetizacin sea una lnea recta? [Resp.: 88 Q.] 6.13. Un motor tipo derivacin de 250 V tiene una corriente de inducido de 20 A cuando gira a 1.000 r.p.m., venciendo el par de plena carga. La resistencia del inducido es de 0,5 Q. Qu resistencia debe insertarse en serie con el inducido para reducir la velocidad a 500 r.p.m. con el mismo par, y cul ser la velocidad si el par de carga se reduce a la mitad, estando dicha resistencia en circuito? Supngase que el flujo permanece constante. [Resp.: 6 Q; 771 r.p.m.] 6.14. Un motor tipo derivacin de 7,5 kW, 460 V, tiene una entrada de 8.500 W cuando desarrolla un par en el eje de 78,3 N.m a 900 r.p.m. En qu tanto por ciento debe reducirse el campo para aumentar la velocidad a 1.050 r.p.m. con un par en el eje de 60,7 N.m? La resistencia del inducido es de 1 Q, la resistencia del circuito de campo a 900 r.p.m. es de 770 Q y las prdidas mecnicas y en el hierro son constantes. Prescndase de la reaccin de inducido. [Resp.: 14 %.] 6.15. Un motor tipo derivacin de 250 V, con un flujo inductor constante, mueve una carga cuyo par vara con el cubo de la velocidad. Cuando gira a 500 r.p.m. el inducido absorbe 40 A. Hallar la velocidad a que girar si se conecta una resistencia de 25 Q en serie con el inducido. Prescndase de las prdidas del motor. [Resp.: 250 r.p.m.] 6.16. Se dispone de un motor derivacin de 200 V, cuya excitacin se mantiene constante. El circuito del inducido tiene en serie un restato de arranque. Calcular el nmero de secciones de este restato y las resistencias de cada seccin, sabiendo que la resistencia del inducido es de 0,5 Q, y que las corrientes en este circuito deben estar comprendidas: a) entre 25 A Y50 A; b) entre 30 A Y50 A. Sugerencia: Se deben demostrar las siguientes relaciones:

donde RT representa la resistencia total del circuito del inducido: propia + restato de arranque; R es la resistencia del inducido; RK_1 y RK son las resistencias totales del

546

MQUINAS ELCTRICAS

circuito del inducido hasta las secciones k - l Y k, respectivamente, del restato de arranque; y es el cociente Im/lmn del inducido en el proceso de arranque. [Resp: a) 3 secciones; 2 1 0,5 b) 4 secciones; 1,6 0,96 n, 0,57 n, 0,346 n.]

n o, n

6.17.

Un motor serie, con un circuito magntico no saturado y con una resistencia de inducido despreciable, absorbe 50 A a 500 V cuando gira a una cierta velocidad con una carga dada. Si el par de carga vara con el cubo de la velocidad, hallar la resistencia adicional colocada en serie con el motor para poder reducir la velocidad: a) un 50 por 100; b) un 20 por 100. [Resp.: a) 23,3 n. b) 6 n.] Un motor serie de 240 V tiene una resistencia de 0,2 n. A la velocidad de 1.800 r.p.m. absorbe 40 A. Hallar la resistencia que debe aadirse: a) para limitar la velocidad a 3.600 r.p.m. cuando la corriente sea de lOA, suponiendo un flujo proporcional a la corriente entre 10 y 40 A; b) para que la velocidad sea de 900 r.p.m. para una corriente de 60 A, sabiendo que el flujo a 60 A es un 18 por 100 mayor que el flujo a 40 A, a qu velocidad girar el motor cuando se conecte directamente a la lnea y absorba 60 A? [Resp.: a) 12,2 b) 1,52 n, 1.500 r.p.m.]

6.18.

6.19.

Un motor serie de 4 polos gira normalmente a 600 r.p.m. con una alimentacin a 250 V, absorbiendo 20 A. Todas las bobinas de campo estn conectadas en serie. Estimar la velocidad y la corriente consumida por el motor si las bobinas se vuelven a conectar en dos grupos en paralelo de dos bobinas en serie. El par de carga es de tipo ventilador, aumentando con el cuadrado de la velocidad. Supngase que el flujo es directamente proporcional a la intensidad de excitacin y prescndase de las prdidas de potencia y cadas de tensin. [Resp.: 713,5 r.p.m.; 33,6 A.l Un motor serie que tiene una resistencia de 1 n entre terminales mueve un ventilador, para el cual el par vara con el cuadrado de la velocidad. A 220 V el conjunto gira a 300 r.p.m. y absorbe 25 A. Debe aumentarse la velocidad a 400 r.p.m. aumentando la tensin. Hallar la tensin y la corriente para los casos lmites siguientes: a) cuando el circuito magntico est saturado, es decir, para flujo constante; b) cuando el circuito magntico est no saturado, es decir, cuando el flujo sea directamente proporcional a la corriente. [Resp.: a) 44,3 A; 304,5 V. b) 33,3 A; 380 V.]

6.20.

BIOGRAFAS
1.

ARNOLD, Engelbert (1856-1911). Ingeniero suizo-alemn. Estudi en el Politcnico de Zurich (1878). En 1880 fue contratado por el Politcnico de Riga como ayudante del profesor Dettman, y junto con este profesor fundaron una empresa elctrica que se dedic a la construccin de dinamos con inducidos dentados. En 1891 pas a formar parte de la compaa suiza Oerlikon, como director de Ingenieria, donde tuvo la oportunidad de proyectar mquinas elctricas de gran tamao. Ms tarde acept el puesto de catedrtico de electrotecnia de la Universidad Tcnica de Karlsruhe. Este Centro, bajo la direccin de Amold, adquiri gran fama en todo el mundo y en sus aulas se formaron multitud de ingenieros de todos los pases. En 1906fue nombrado Doctor Honoris Causa por la Universidad de Hannover. Fue uno de los pioneros que estudiaron con profundidad los devanados elctricos de las mquinas de c.c. y los problemas de conmutacin. Escribi diversos libros sobre mquinas elctricas que fueron referencia en muchas empresas constructoras de

CAPTULO

6.

MQUINAS DE CORRIENTE CONTINUA

547

maquinaria elctrica y en universidades tecnolgicas. Su obra pstuma fue Die Weschselstromtechnik, compuesta de cinco volmenes que se escribieron entre los aos 1903 y 1909 Y donde particip su ayudante, el profesor Jens Lassen de La Cour. 2. BARLOW,Peter (1780-1862). Matemtico, fsico y astrnomo ingls. Fue profesor de la Academia militar de Woolwich (1806-1846) y miembro de la Royal Society en 1823. Cultiv fundamentalmente las matemticas y la fsica, inventando procedimientos para evitar la interferencia de las grandes masas del hierro de un barco sobre la brjula. Invent un telescopio acromtico e hizo estudios notables sobre construccin de lneas frreas. En 1828 construy la famosa rueda que lleva su nombre, que era una mquina dinamoelctrica y que sirvi para demostrar la accin dinmica de una corriente elctrica. 3. BRUSH,Charles Francis (1849-1929). Ingeniero e inventor americano. Se gradu como ingeniero de Minas en la Universidad de Michigan. Desde 1869 a 1871 se dedic al anlisis qumico, y ms tarde, entre 1871 y 1875, ocup la direccin de una importante industria siderrgica. En 1876 perfeccion notablemente la dinamo elctrica, patentando un generador que llevaba su nombre que produca una tensin variable controlada por la carga y una corriente constante que se utilizaba para el alumbrado de arco, con las lmparas conectadas en serie. En 1879 invent un regulador de tensin. Fue premiado con la medalla Rumford en 1899 y recibi la medalla Edison en 1913. 4. GRAMME,Zenobe Theophile (1826-190 I l. Inventor belga. De formacin eminentemente autodidacta, en 1856 se traslad a Pars y trabaj en una sociedad industrial. Dotado de una gran intuicin y habilidad para la invencin. En 1867 obtuvo una patente por una mquina de c.a. y dos aos ms tarde, en 1869, patent su famosa dinamo, que daba tensiones ms elevadas que las conocidas hasta entonces y que resolva la rectificacin con el colector de delgas. En 1870 se uni al tambin inventor Hippolyte Fontaine, para explotar su dinamo. En la Exposicin Internacional de Viena presentaron una dinamo Gramme de gran tamao y pudieron demostrar que era reversible y que se poda usar como motor. Hizo tambin estudios sobre transporte de energa elctrica. 5. HJORTH,Soren (1801-1870). Ingeniero civil dans. Fund en 1844 la primera Sociedad danesa de ferrocarriles y fue ms tarde el director del primer ferrocarril del Reino de Dinamarca. En 1848 se traslad a Londres a la fbrica Robinson y Cia. Pimlico, construyendo en sus talleres un motor elctrico alternativo con electroimanes que recibi el nico premio sobre mquinas elctricas en la Exposicin Universal de Londres de 1851. El mismo ao volvi a Copenhague, haciendo investigaciones que le llevaron a enunciar el principio dinamoelctrico. Este proyecto est depositado en los archivos de la Sociedad Real de las Ciencias; describe una mquina con un inductor de imanes permanentes pero que lleva arrolladas una serie de bobinas, por las cuales hace circular la corriente producida por la misma mquina. El proyecto fue realizado cuando Hjorth volvi a Londres en 1854 y se hicieron dos prototipos en la fbrica de W.T. Henley. Este invento se describe en dos patentes inglesas: la n." 2.198, de 1854, y la n." 806, de II de abril de 1855; esta ltima llevaba por ttulo: An improved magnetoelectric battery. Fue, sin embargo, Werner Van Siemens quien desarroll tcnica y comercialmente esta idea en la construccin de mquinas de c.c. y a quien se le atribuye por eso la paternidad del invento, pero el descubrimiento de Siemens tuvo lugar en 1866, es decir, once aos ms tarde que Hjorth. 6. HOBART,Henry Metcalf (1868-1946). Ingeniero americano. Se gradu en el MIT en 1889. En ese mismo ao ingres en la Thomson-Houston. En 1894 pas a la General Electric, en la que fue ayudante de c.P. Steinmetz. Entre 1895 y 1913 trabaj en Berln y Londres. En esta misma ciudad fue profesor en el University College (1908-1911). Autor de ms de una docena de libros de ingeniera elctrica, fundamentalmente en relacin con las mquinas elctricas, muchos de los cuales fueron traducidos al francs y al alemn. Escribi gran cantidad de artculos tcnicos. Se le atribuyen cerca de 30 patentes en el campo de la ingeniera elctrica. 7. HOPKINSON, Edward (1859-1922). Fsico ingles. Estudi en el Owens College. En 1881 se doctor en Ciencias en la Universidad de Londres, en la rama de Electricidad y Magnetismo. Al ao siguiente fue ayudante de Sir Williams Siemens. Realiz ensayos en Porthus sobre el primer tranva elctrico del Reino Unido. Pionero en los estudios y proyectos de traccin elctrica, fue el responsable del diseo de locomotoras elctricas para los ferrocarriles ingleses. Recibi la medalla de oro de la Institucin de Ingenieros Civiles en 1893 por sus trabajos en traccin elctrica. Premio

548

MQUINAS ELCTRICAS

8.

9.

10.

11.

12.

Telford y medalla George Stephenson. Se le atribuyen gran nmero de patentes en el campo de la ingeniera elctrica, muchas de ellas en colaboracin con su distinguido hermano el profesor John Hopkinson: como la dinamo y el motor Manchester, mejoras en el diseo de los circuitos magnticos de las mquinas elctricas, etc. Fue Presidente de la Institucin de Ingenieros Mecnicos de Inglaterra. JACOBI, Moritz Herman Von (1801-1875). Fsico alemn, hermano del matemtico Carlos Gustavo Jacobi. Trabaj como arquitecto en Konisgsberg y despus fue profesor de arquitectura en Dorsal. En 1837 fue llamado por el zar a San Petersburgo; en ese mismo ao descubri la galvanoplastia, que le dio celebridad mundial. En 1839 construy un motor elctrico con electroimanes que emple para propulsar un barco con ruedas sobre el ro Neva. En 1840 construy una lnea telegrfica para la residencia del zar, pudiendo comprobar entonces la inutilidad del empleo de un doble hilo, ya que se poda hacer el retomo por tierra. Invent el restato. Fue miembro de la Academia de Ciencias de San Petersburgo. LANGSDORF, Alexander Suss (1878-1974). Ingeniero americano. Estudi en la Universidad de Washington. Director del Departamento de Ingeniera Elctrica de esa Universidad en 1904. Fue tambin decano de la Escuela de Arquitectura (1916). Entre 1920 y 1926 dej la enseanza para trabajar en la industria privada, pero volvi nuevamente a la Universidad, siendo nombrado director de las Escuelas de Ingeniera y Arquitectura. En 1950 se retir como catedrtico y decano emrito de ingeniera elctrica. Langsdorf escribi entre 1915y 1920dos libros sobre mquinas de c.c. y c.a. que le haran mundialmente famoso y que fueron traducidos a varios idiomas, entre ellos el espaol y el chino. PACINOTTI, Antonio (1841-1912). Fsico italiano. Estudi en Pisa, donde su padre era catedrtico de Fsica. En 1860 construy una dinamo con inducido en anillo, dotada con un primitivo conmutador. Una descripcin de esta mquina fue publicada en 1865en la revista II Nuava Cimenta (3 de mayo de 1865), donde demostraba la reversibilidad de esta mquina. Desde 1864 a 1873 fue profesor de fsica en Bolonia. En 1873 present en la Exposicin de Viena tres modelos de su mquina y fue donde vio la dinamo de Gramme, que era muy similar. En el Jurado se encontraba lord Kelvin, y aunque se reconoci que la mquina de Pacinotti era anterior a la de Gramme, sta tena algunas modificaciones que la hacan distinta, por lo que Pacinotti solamente recibi un premio de progreso. En 1874 se traslad a la Universidad de Cagliari, donde sigui con sus ensayos elctricos. Ms tarde, en 1882, volvi a la universidad de Pisa para ocuparse de la ctedra de fsica tecnolgica. En 1883 propuso una aplicacin de su inducido en anillo para transmitir a distancia movimientos angulares; en realidad lo que propona era un indicador bifsico con campo giratorio. Sin embargo, sera el genio de Ferraris el que descubrira ms tarde la esencia del campo giratorio. VARLEY, Samuel Alfred (1832-1921). Fsico ingls. En 1852 ingres en la Compaa inglesa de telgrafos. En su primera etapa trabaj, junto a su hermano Cromwell, en el desarrollo de diversos inventos telegrficos. En 1866invent una dinamo autoexcitada, anloga a la construida por Charles Wheatstone y Werner V. Siemens. Por el desarrollo de esta mquina recibi la medalla de oro de la Exposicin Internacional de 1885. Desarroll la dinamo compound en 1878, lo que fue motivo de litigio con la Compaa anglo-americana Brush. En 1892 la Cmara de los Lores se decidi finalmente a favor de Varley. Propuso la subdivisin del ncleo de hierro en los transformadores para reducir las prdidas por corrientes de Foucault. Fue uno de los miembros fundadores del lEE en 1871, nombrndose a Sir Williams Siemens el primer presidente. WARD-LEONARD, H. (1861-1915). Ingeniero elctrico americano. Naci en 1861 en Cincinnati muri en abril de 1915 en New York. Se gradu a los veintids aos en el Massachusetts Institute afTechnolagyen 1883. Al acabar su carrera trabaj dos aos con Edison y en 1887fue contratado como ingeniero ayudante en la Western Electric Light Ca. de Chicago. En 1888 cre la empresa Leonard-Izard, realizando importantes proyectos sobre centrales elctricas y traccin elctrica. En 1891 patent el clebre control de velocidad de motores de c.c. y que desde entonces lleva su nombre. Este invento 10 justific ms tarde con un artculo publicado en la revista Transactions AlEE de 1896,Vol. 13,pags. 377-386, que llevaba por ttulo: Valts versus Ohms-the speed regulation af Electric Matars. Se deben a l otros inventos como el frenado por recuperacin de energa

CAPTULO

6.

MQUINAS DE CORRIENTE CONTINUA

549

o regenerativo, el disyuntor de doble brazo, diversos sistemas de alumbrado para trenes, etc.; en total alrededor de 100 patentes en diversos campos de la ingeniera elctrica. Premiado con la medalla de oro en la Exposicin de Pars de 1900 y tambin en la de San Louis de 1904. Fellowdel American lnstitute of Electrical Engineers, falleci en el mes de abril de 1915 en el hotel Astor de New York durante un banquete que celebraba el Instituto Americano de Ingenieros Elctricos (AlEE). 13. WILSON, Ernest (1863-1932). Ingeniero ingls. De familia humilde, empez trabajando en una escuela de aprendices de la empresa Geenwood y Batley, donde vio construir alternadores tipo Brush y Ferranti. Segua tambin clases nocturnas en el Yorkshire College y debido a su buen aprovechamiento obtuvo una beca para estudiar en Alemania. En 1886 se matricul en la Kdniglische Technische Hochschule en Hannover, donde fue alumno de Kohlrausch, teniendo un aprendizaje eficaz en ensayos de mquinas y en el idioma alemn. Al volver a Inglaterra en 1887 ingres en la empresa que los hermanos Siemens tenan en este pas y lleg a ser ayudante personal de Alexander Siemens. Wilson trabaj en el diseo de los equipos elctricos de las primeras locomotoras construidas para los ferrocarriles de la ciudad de Londres. Durante este perodo de tiempo l asista a las clases de John Perry y Silvanus Thompson en el Findsbury Technical College. Interesado por el diseo de mquinas elctricas, saba que en el King's College se haba creado el Laboratorio Siemens y nombrado catedrtico de ingeniera elctrica a John Hopkinson en 1890, quien a su vez eligi a Wilson como profesor de Laboratorio en. 1891. En 1898 era profesor ayudante y al ao siguiente, debido a la trgica muerte en accidente de montaa del profesor Hopkinson, Wilson pas a sustituirle en su ctedra, llevndola con dignidad hasta su jubilacin en 1930. Como profesor era infatigable y lleg a publicar ms de 80 artculos propios y 30 ms en colaboracin. Su clebre libro sobre traccin elctrica se public en 1897 y fue tomado como de consulta en muchos centros extranjeros. En 1888 invent el motor de c.a. polifsico con colector de delgas. En 1891 invent el motor de c.a. con conmutador con conexin serie, y H. Gorges patent en ese mismo ao otro con conexin shunt,

REFERENCIAS
1. 2. 3. 4. 5. 6. 7. 8. 9. 10. 11. 12. 13. 14. 15. CHAPMAN,S. J.: Mquinas elctricas. McGraw-Hill Latinoamericana, S. A., Bogot, 1987. CHATELAIN,J.: Trait d'lectricit, Vol. X: Machines lectriques. Editions Georgi, Lausanne, 1983. CHILIKIN,M.: Accionamientos elctricos. Ed. Mir, Mosc, 1972. CORTESCHERTA,M.: Teora general de mquinas elctricas. UNED, Madrid, 1976. DAWES, CH. L.: Tratado de electricidad, tomo 1: Corriente continua. Ed. G. Gili, Barcelona, 1966. DEL TORO, V.: Electric Machines and Power Systems. Prentice Hall Inc., Englewood Cliffs, N. J., 1985. EL-HAWARY,M. E.: Principies of Electric Machines with Power Electronic Applications. Reston Books, Prentice-Hall, Englewood Cliff, N. J., 1986. FITZGERALD, A. E.; KINGSLEY,Ch.: Electric Machinery, 2: edicin. McGraw-Hill Book, New York, 1961. FOUILLE,A.: Electrotecnia para ingenieros, tomo II: Mquinas elctricas. Ed. Aguilar, Madrid, 1965. GEHMLICH, D. K.; HAMMOND, S. B.: Electromechanical Systems. McGraw-Hill Book, New York, 1967. GRAY, A.; WALLACE,G. A.: Electrotecnia. Fundamentos tericos y aplicaciones prcticas. Ed. Aguilar, Madrid, 1967. HINDMARSH: Electrical Machines. Pergamon Press, Elnsford, N. Y., 1965. IVANOV-SMOLENSKY, A.: Electrical Machines, tomo 3. Mir Publishers, Moscow 1983. JUREK,S. F.: Electrical Machinesfor Technicians and Technician Engineers. Longman, London, 1972. KASATKIN,A.; PEREKALIN, M.: Curso de Electrotecnia. Ed. Cartago, Buenos Aires, 1960.

550
16. 17. 18. 19. 20. 21. 22.

MQUINAS ELCTRICAS

2:1.
24. 25. 26. 27. 28. 29.

Kosow, l.: Electric Machinery and Transformers. Prentice Hall. Englewood Cliffs, N. J., 1972. EKOSTENKO, M. P.; PIOTROVSKI, L. M.: Mquinas elctricas, tomo 1. Ed. Mir, Mosc, 1975. LANGSDORF, A. S.: Principios de mquinas de e.c. Ed. Castillo, Madrid, 1964. LrwscHITz-GARIK, M.: Direct Current Machines. Van Nostrand Co., Princeton, N. J., 1946. MATscH, L. W.: Mquinas electromagnticas v electromecnicas. Representaciones y Servicios de Ingeniera, S. A., Mxico, 1974. NAGRATH,1. J.; KOTHARr,D. P.: Electric Machines. Tata McGraw-Hill Publishing Co., New Delhi, 1985. PARKERSMITH,S.: Problemas de Ingeniera Elctrica. Ed. Selecciones Cientficas. Madrid, 1961. RIMINI, c.: Electrotecnia General. tomo 11. Ed. G. Gil de Mxico, 1955. SANJURJONAVARRO,R.: Mquinas elctricas. McGraw-Hill Interamericana de Espaa, S. A., Madrid, 1989. SEGUrFR, G.; NOTELET, F.: Etectrotechuique lndustrielle. Technique et Docurnentation, Pars,
1977.

SISKIND,CH.: Direct Current Machinery. McGraw-Hill, New York, 1952. SI.EMON,G. R.; STRAUGHEN, A.: Electric Machines. Addison-Wexley Pub., Reading, Massachusetts, 1980.

STErN,R.: HUNT.Jr. W. T.: Electric Power Svstem Components. Transjormers and Rotating Machines. Van Nostrand Reinhold Co., New York, 1979. VESHENEVSKI. S.: Caractersticas de los motores en el accionamiento elctrico. Ed. Mir, Mosc,
1972.

30.

TRENKMANN, H.: Teora, Clculo y Construccin de las Mquinas de c.e. Escuela del Tcnico Electricista, Tomo IV. Ed. Labor. Barcelona, 1962.

CAPTULO 7

Accionamientos

elctricos

7.1. INTRODUCCIN
En los captulos anteriores de este libro dedicado a las mquinas elctricas se ha destacado la importancia del control de velocidad de los motores elctricos. Las tcnicas estudiadas en' estos captulos han explicado los mtodos convencionales de regulacin y control de mquinas elctricas en los que se utilizaban restatos de arranque y regulacin, combinacin motorgenerador(grupo Ward-Leonard) y transformadores de tensin variable. Con el desarrollo tan espectacular de la electrnica desde el descubrimiento del transistor en 1947 por los Laboratorios Bell, y sobre todo gracias a la aparicin en el mercado en 1957,lanzado por los Laboratorios de la General Electric, del tiristor o SCR (semiconductor controlled rectifier), un dispositivo semiconductor capaz de realizar una conmutacin controlada con altos niveles de potencia, dio comienzo a una nueva era en la utilizacin de circuitos de control esttico para conseguir una regulacin en las mquinas elctricas con una alta fiabilidad y rendimiento. Este rea de la ingeniera elctrica se conoce con el nombre de electrnica de potencia o tambin electrnica industrial, y se ha desarrollado enormemente en los ltimos aos debido al descubrimiento de nuevos dispositivos de conmutacin como: interruptores de apagado por puerta o GTO (gafe turn off switchesi, transistores de unin bipolar o BJT (bipolar junction transistorsi, transistores bipolares de puerta aislada o IGBT (insulated gafe bipolar transistors), transistores MOSFET (metal-oxide semiconductor field-effect transistorsi, tiristores controlados MOS o MCT (MOS-controlled Thyristors), etc. En principio la electrnica de potencia se utiliz para obtener c.c. de las redes de c.a., lo que era necesario para la alimentacin de motores de C.C., para las industrias electroqumicas, que necesitabaneste tipo de energa en los procesos de electrlisis, galvanoplastia, etc. Los dispositivoselectrnicos fueron en principio vlvulas electrnicas o tubos de vaco, despus rectificadores de vapor de mercurio y por fin los rectificadores semiconductores. A continuacin se procedi a la regulacin de velocidad de los motores de e.e. por medio de grupos giratorios Ward-Leonard,tiratrones (vlvulas de gas con electrodo de control) y ms tarde con tiristores (tiratrones semiconductores o de estado slido). Siempre que se requera una regulacin de velocidadse acuda a los motores de c.c. por su facilidad en el control electrnico. Como ya se sabe,los motores de c.c. son mucho ms caros que los motores de c.a. asncronos a igualdad de potencia,y adems tienen una parte muy frgil, que es el conmutador o colector de delgas, que requera un cuidadoso mantenimiento. La regulacin de velocidad de los motores asncronos permaneci en estos aos desconocida por la dificultad en el control electrnico. La invencin del microprocesadoren 1971permiti vencer estas dificultades y en la actualidad se fabrican convertidoreselectrnicos excelentes para la regulacin y control de motores asncronos, que estn sustituyendo a los motores de c.c. en aplicaciones de velocidad variable. 551

552

MQUINAS ELCTRICAS

Este captulo, dedicado a los accionamientos elctricos, representa una simbiosis entre la electrotecnia y la electrnica y pretende dar una visin moderna aunque simplificada de los procedimientos existentes para la regulacin y control de motores elctricos. Comienza el captulo explicando de un modo breve el funcionamiento de los principales dispositivos electrnicos que se utilizan en el control de mquinas elctricas: diodos, tiristores, etc. A continuacin se describen los principales tipos de convertidores que se emplean en la regulacin: rectificadores clsicos, rectificadores controlados mono y trifsicos, recortadores o troceadores (conocidos en ingls como choppers), inversores y cicloconvertidores. Finalmente se aplican estos conocimientos para estudiar el comportamiento de los motores elctricos alimentados por convertidores electrnicos, comenzando con la regulacin de los motores de c.c. y despus con los de corriente alterna tipos asncrono y sncrono. El fin primordial de los accionamientos elctricos es adaptar la electrnica a las mquinas elctricas para conseguir relaciones par/velocidad que se adapten a las diferentes condiciones de servicio. Los convertidores electrnicos consiguen puestas en marcha suaves de los motores, evitando las grandes corrientes de arranque tradicionales; pueden ajustar su rampa de aceleracin hasta la velocidad de rgimen con relativa facilidad debido a las unidades de control, que incorporan circuitos integrados digitales y microprocesadores, permitiendo asimismo con interfaces adecuadas un control por ordenador desde una consola alejada de la mquina.

7.2.

DISPOSITIVOS SEMICONDUCTORES DE POTENCIA

La electrnica de potencia est basada en el empleo de dispositivos semiconductores que trabajan en conmutacin, esto es, dispositivos que en virtud de sus caractersticas de uso actan como simples interruptores estticos que pueden estar abiertos o cerrados, es decir, o bien bloquean la tensin aplicada o bien conducen la corriente sin ninguna restriccin. Se considera que no hay estados intermedios, excepto el breve tiempo durante el que se produce el perodo transitorio de la conmutacin. Los dispositivos empleados: diodos y transistores de potencia, tiristores, GTO, IGBT, etc., se basan en el uso de materiales semiconductores. Para estudiar su comportamiento no es necesario profundizar en los fenmenos que ocurren internamente en el semiconductor, sino ms bien analizar y comprender sus formas de actuacin, conociendo sus respuestas en forma de curvas caractersticas especficas y las limitaciones reales que presentan estos componentes. El objetivo, por consiguiente, de este epgrafe es exponer de una forma simplificada los diversos dispositivos utilizados en electrnica de potencia y la forma en que trabajan. Como quiera que se basan en el empleo de materiales semiconductores, consideramos que es importante iniciar este apartado explicando lo que son estos materiales.

7.2.1. Introduccin
Los semiconductores son materiales que tienen una resistividad cuyo valor est comprendido entre la de un conductor y un aislador. Los dos semiconductores ms importantes en electrnica son el germanio y el silicio; estos elementos estn situados en la columna IV de la Tabla Peridica y tienen 4 electrones de valencia en su ltima capa. La estructura cristalina de ambos elementos es un tetraedro en el que cada tomo comparte un electrn de valencia con cuatro tomos vecinos formando enlaces covalentes; de este modo la capa electrnica de cada tomo tiene un total de ocho electrones: cuatro propios y otros cuatro pertenecientes a los tomos vecinos; en esta situacin es imposible distinguir cules de los electrones de valencia

CAPTULO 7.

ACCIONAMIENTOS ELCTRICOS

553

son propios y cules extraos, ya que ambos se han convertido en electrones comunes del cristal semiconductor. A temperatura cercana al cero absoluto, un semiconductor se comporta como un aislador, y no conduce la corriente porque no hay electrones libres; pero al elevarse la temperatura, el enlace de los electrones de valencia se debilita y algunos de ellos, en virtud del movimiento trmico, pueden abandonar sus tomos. El electrn escapado se convierte en libre y all donde estuvo hasta entonces se origina un sitio libre; este sitio libre en el enlace interatmico del semiconductor se llama convencionalmente hueco. Cuanto ms alta es la temperatura del semiconductor, tanto mayor es el nmero de electrones libres y de huecos que surgen en l. De este modo, la formacin de un hueco en la masa del semiconductor se debe a la salida de un electrn de valencia de la envoltura del tomo y corresponde a la aparicin de una carga elctrica positiva igual a la carga negativa del electrn. En un semiconductorpuro o intrnseco el nmero de electrones que se liberan en cada instante es igual al nmero de huecos que se forman en ese mismo tiempo. Su nmero total a la temperatura ambiente es relativamente pequeo, por eso la conductividad elctrica de este semiconductor, denominada intrnseca, es pequea (por ejemplo, en el caso del germanio, hay un electrn [hueco] libre por cada 5 . l O'" tomos de germanio). Sin embargo, el hecho sobresaliente de un semiconductor intrnseco es que, al aadir aunque slo sea una cantidad insignificante de impureza en forma de tomos'de otros elementos, su conductividad se eleva enormemente. Este proceso se denomina dopado, y segn sea la estructura atmica de estos elementos que se aaden, la conduccin en el semiconductor se hace bien por electrones o bien por huecos. As, si se sustituye en un cristal semiconductor de silicio un tomo del mismo por un tomo de un elemento de valencia cinco (por ejemplo, de antimonio), se producirn cuatro enlaces o apareamientos entre electrones del antimonio y del semiconductor, pero el quinto electrn de valencia del antimonio sobra y queda libre. Cuantos ms tomos de antimonio se introduzcan en el semiconductor, tantos ms electrones libres habr en su masa. El antimonio constituye de este modo un elemento donador de electrones. El semiconductor creado se demonina extrnseco de tipo N (de tipo Negativo), ya que la conduccin se produce debido a este excedente de electrones de un modo anlogo al proceso de conduccin en los metales. En la Figura 7.1a se representa de un modo grfico el material N. Obsrvese el gran nmero de electrones libres en este material, representados por los signos (-), en la masa de silicio, y que constituyen los portadores mayoritarios de este tipo de semiconductor. Los iones donadores del material N son positivos, ya que han perdido un electrn, y por ello se han representado por crculos blancos con un signo (+) en su interior. Tambin existe un nmero
Portadores mayoritarios

Masa de silicio

Portadores minoritarios
a) Material tipo N

Portadores mayoritarios

Iones aceptadores

b) Material tipo P

Figura 7.1. Materialessemiconductores.

554

MQUINAS ELCTRICAS

escaso de huecos generados trmicamente, y que se han sealado con signos (+) en la masa de silicio; representan los portadores minoritarios del semiconductor tipo N. Algo totalmente distinto sucede si el dopado se realiza con un elemento trivalente, por ejemplo, de indio. En este caso, cada tomo del metal indio, con sus tres electrones de valencia, enlaza solamente con tres de los tomos vecinos del semiconductor; para enlazar con el cuarto tomo le falta un electrn, y aS, se origina un hueco; ste, como es natural, puede llenarse con un electrn cualquiera desprendido de un enlace de valencia del semiconductor. Cuantos ms tomos de indio se introduzcan en el cristal de silicio, tantos ms huecos habr en su masa, de tal modo que al romperse los enlaces entre tomos, los electrones correspondientes se movern de hueco en hueco en toda la masa del semiconductor, y as en cada instante el nmero de huecos ser mayor que el nmero de electrones libres. El indio constituye de este modo un elemento aceptador de electrones. El semiconductor creado se demonina extrnseco de tipo P (de tipo Positivo), lo que significa que el fenmeno de la conduccin de corriente en la masa del semiconductor tipo P va acompaado de la continua generacin y desaparicin de cargas positivas (huecos) que se comportan de este modo como portadores de corriente. En la Figura 7.1b se representa de un modo grfico el material P. Obsrvese el gran nmero de huecos libres en este material, representados por los signos (+) en la masa de silicio y que constituyen los portadores mayoritarios de este tipo de semiconductor. Los iones aceptadores del material P quedan de este modo cargados negativamente y por ello se han representado por Crculosblancos con un signo (-) en su interior. Tambin existe un nmero escaso de electrones generados trmicamente y que se han sealado con signos (-) en la masa de silicio; representan los portadores minoritarios del semiconductor tipo P. Cuando se realiza una unin entre los materiales P y N, tal como se indica en la Figura 7.2a, se dispone de un nmero preponderante de electrones libres en la regin N y de huecos en la regin P. Los electrones libres de la regin N se difundirn hacia la regin P, mientras que los huecos de la regin P se difundirn hacia la regin N. Los electrones libres que procedan de la zona N llegarn a una regin en que la proporcin de huecos es grande, lo que les har recombinarse rpidamente con ellos, y viceversa, los huecos procedentes de la

Material P

Regin de transicin Material N

Campo elctrico exterior

Campo elctrico exterior

+----

p +

Difusin de huecos hacia la regin N

La regin de transicin se estrecha

1,

La regin de transicin se ensancha

Difusin de electrones hacia la regin P

~------~

+ .-

----------~
Iv
Uniones

+
v
e) Unin PN con tensin inversa

a) Unin PN

b) Unin PN con tensin directa

Figura 7.2.

PN.

CAPTULO

7.

ACCIONAMIENTOS ELCTRICOS

555

zona P se recombinarn con los electrones. Debido a este proceso, aparece una regin de transicin (zona de deplexin) en la que quedan al descubierto iones negativos en el lado Pe iones positivos en el lado N, formndose de este modo una barrera de potencial que se opone a posteriores recombinaciones por difusin de portadores mayoritarios. Existe al mismo tiempo una corriente de arrastre debida a los portadores minoritarios, ya que si un electrn de la regin P se acerca a la unin, caer bajo la influencia de la barrera de potencial que en ella existe y pasar a la zona N, y del mismo modo, si un hueco de la regin N se acerca a la unin, la barrera de potencial lo empujar a la regin P. Como consecuencia de todo ello no hay corriente neta, ya que la corriente de difusion de los portadores mayoritarios es equilibrada con la corriente de arrastre de los portadores minoritarios, como consecuencia del campo elctrico de la barrera. Si se aplica una d.d.p. externa a la unin PN con la polaridad mostrada en la Figura 7.2b (se dice entonces que la polarizacin es directa () positiva), se produce una circulacin de corriente que aumenta mucho con la tensin aplicada; esto se debe a que la pila produce un campo elctrico que hace que los electrones de la regin N se desplacen hacia la regin P (en definitiva, del menos al ms de la pila), mientras que los huecos de la regin P se movern al encuentro de los electrones, desde el terminal positivo al negativo. Al encontrarse en el lmite de las regiones, zona de deplexin de la unin PN, los electrones se comportan como si saltasen a los huecos y como resultado unos y otros dejan de existir. El contacto metlico unido al polo negativo de la pila puede ceder a la regin tipo N una cantidad de electrones prcticamente ilimitada, respondiendo as a la disminucin de electrones en esta regin, y el contacto unido al polo positivo de la pila puede aceptar de la regin tipo P la misma cantidad de electrones, lo que equivale a la introduccin en ella de la cantidad de huecos correspondiente. En este caso la zona de deplexin se estrecha, aumentando el nmero de portadores que se difunden a travs de la unin, lo que da como resultado la circulacin de una corriente directa a travs del diodo, que se incrementa conforme aumenta la d.d.p. aplicada por la pila. Cuando se intercambian los polos de la pila (Fig. 7.2c), circula una dbil corriente l , de saturacin. En este caso el comportamiento de las cargas elctricas en el diodo es otro. Ahora el campo elctrico aplicado por la pila hace que los electrones de la regin N se alejen de la unin PN y se desplacen hacia el contacto positivo del diodo, y los huecos de la regin P, hacia el contacto negativo; como resultado, la zona de deplexin se ensancha y se forma una regin pobre en electrones y huecos y que por ello ofrece gran resistencia a la corriente. No obstante, en esta zona tendr lugar un pequeo intercambio de portadores de corriente entre las regiones de la unin, por eso pasa corriente a travs de la unin, pero su intensidad es mucho menor que la directa. Esta corriente recibe el nombre de corriente inversa. Los diversos dispositivos semiconductores que se explican en este epgrafe utilizan diversas composiciones de materiales P y N, formando uniones scmiconductoras similares a la aqu explicada, que en el caso ms simple del diodo es una nica unin P-N, en el caso del transistor son dos uniones P-N-P o N-P-N y en el caso de los tiristores son tres uniones P-N-P-N. Cada dispositivo tiene sus propiedades caractersticas, dependiendo de la disposicin de uniones semiconductoras y de la forma en que se inyecta corriente a las mismas o se polarizan mediante tensiones adecuadas.

7.2.2. Diodos rectificadores


Estos dispositivos se basan en una unin semiconductora tipo P-N. Si se considera simplemente una barra de silicio, dopada en un lado para obtener un semiconductor tipo P y en el otro un semiconductor tipo N, la unin PN representa la frontera entre ambas configuracio-

556

MQUINAS ELCTRICAS

nes. El diodo tiene por ello dos terminales, el que se conecta al material P es el nodo (A) y el que se conecta al material N es el ctodo (K). En la Figura 7.3 se muestra en a) la unin P-N del diodo rectificador y en b) el smbolo utilizado para su representacin. La forma externa del encapsulado de un diodo suele ser cilndrica para los diodos de seal y los rectificadores de pequea corriente y se construye con vidrio, metal o plstico; el ctodo suele distinguirse con una banda de color o un punto, aunque a veces el encapsulado lleva grabado el smbolo del diodo para la fcil identificacin de los terminales. Para rectificadores de alta corriente, el chip rectificador se monta directamente sobre un metal base, cuyo ctodo acaba en forma de tomillo (Fig. 7.3c) para que pueda roscarse sobre una placa metlica (el chasis del equipo o en disipadores de calor especiales) de tal forma que se tenga un camino fcil para la conduccin trmica y se disipe el calor producido en la unin. La conexin del nodo se hace con un hilo conductor unido a la parte superior del chip y sale hacia fuera a travs de un sellado de cermica o vidrio. Los diodos pueden ser de baja potencia, denominados entonces diodos de seal, y de gran potencia (para corrientes superiores a 1 A), recibiendo entonces el nombre de rectificadores. La caracterstica tensin-corriente de un diodo real es la indicada en la Figura 7 Aa. Como ya se ha indicado, cuando se aplica una polarizacin positiva, el diodo conduce de tal forma que la corriente aumenta mucho con la tensin aplicada. Cuando la polarizacin es negativa, es decir, se intercambian los polos de la pila, el diodo lleva una dbil corriente l, de saturacin. Esta corriente recibe el nombre de corriente inversa del diodo. En la Figura 7Aa se observa la caracterstica 1 = f(V) del diodo y se ve que la respuesta es no lineal. Se puede obtener un modelo lineal o circuito equivalente del diodo en la zona correspondiente a polarizaciones directas o positivas y que es til en algunas aplicaciones (el modelo puede completarse para extenderlo a la curva completa). La idea para realizar el modelo es sustituir la curva de la respuesta real por varios segmentos que se adapten lo ms posible a la misma; entonces cada segmento representa una propiedad lineal sobre su rango. Para la caracterstica directa del diodo de la Figura 7.4a, se observa que la lnea AB corta al eje de abscisas para un valor de tensin de Va = 0,7 V, que representa el valor de la barrera de potencial para un diodo de silicio. La lnea OA se considera como corriente cero, lo que indica que el diodo no comienza a conducir hasta que no se aplique un potencial externo del orden de 0,7 V, opuesto a la batera. Es fcil deducir que el circuito equivalente podr representarse por el esquema indicado en la parte inferior de la Figura 7 Aa. El valor Va es la interseccin de la recta AB con el eje de tensiones y RJ es la resistencia directa, que viene representada por la inversa de la pendiente del tramo AB. Este modelo puede ser til cuando el diodo trabaja con pequeas tensiones y corrientes, como sucede con el diodo de seal que generalmente se

a) Unin PN

b) Smbolo del diodo

*
K

e) Forma constructiva

Figura 7.3. Diodorectificador.

CAPTULO 7.

ACCIONAMIENTOS ELCTRICOS

557

B imx

tg = i/R

a) Caracterstica real de un diodo

b) Caracterstica ideal de un diodo

Figura 7.4. Curvas real e ideal del diodo.

utiliza en la modificacin y control de informacin, como es el caso de la electrnica digital, donde los diodos se pueden emplear como puertas lgicas para direccionar la informacin. Tngase en cuenta que el diodo no empieza realmente a conducir hasta que no se aplique una tensin directa de por lo menos 0,7 V; es claro que si se aplica una d.d.p. inferior a este valor el diodo no conducir, lo que podra interpretarse como que la seal externa es de sentido inverso, lo cual no es cierto. En consecuencia, cuando se empleen diodos como detectores de seal ser preciso que las d.d.p. que se apliquen superen el umbral de 0,7 V para obtener una informacin til; con seales inferiores quedar una incertidumbre en el conocimiento de la polaridad de la tensin aplicada. Cuando se emplean diodos para grandes potencias, como es el caso de la rectificacin, es claro que la d.d.p. de 0,7 V ms la cada interna del diodo, que pueden suponer un total de 1 V, no representarn ms que un pequeo porcentaje de la tensin total, por ello lo que se hace en rectificacin es considerar el diodo como ideal con la respuesta representada en la Figura 7.2b; es decir, el diodo conduce cuando se aplica tensin directa, presentando una resistencia nula, y no conduce con polarizacin inversa, presentando entonces resistencia infinita; este comportamiento hace que se pueda considerar el diodo como un interruptor de dos posiciones (conduccin/no conduccin, o en ingls: ON/OFF). Esta simplificacin es muy ventajosa en el estudio de los circuitos rectificadores, que se har ms adelante. La corriente directa i est limitada por el calentamiento en el material semiconductor. En la actualidad existen diodos rectificadores que pueden llevar una corriente de 5.000 amperios y con tensiones inversas de pico de 5 kV.

7.2.3. Tiristor

rectificador controlado

El siguiente grupo o categora de dispositivos son los rectificadores controlados, que tambin se denominan rectificadores controlados de silicio o semiconductor (SCR: Semiconductor Controlled Rectifiers) o simplemente tirstores. Estos elementos son dispositivos de tres terminales: nodo (A), ctodo (K) y puerta (G), correspondientes a tres uniones semiconductoras (P-N-P-N). En la Figura 7.5a se muestra la composicin interna de un tiristor, en la Figura 7.5b el smbolo correspondiente y en la Figura 7.5c un tipo de configuracin externa tpica.

558

MQUINAS ELCTRICAS

K K
al Composicin interna bJ Smbolo el Forma constructiva

Figura 7.5. Tirislor. Los tiristores tienen propiedades semejantes a los diodos rectificadores cuando funcionan con tensiones negativas: al aplicar una tensin inversa (nodo negativo respecto al ctodo), la unin central 12 tiene polarizacin positiva mientras que las uniones JI y 13 quedan polarizadas inversamente y es por ello que circula entonces una pequea corriente de dispersin por el circuito. Si se aumenta la tensin inversa hasta un valor conocido como tensin de ruptura inversa VRH (reverse breakdown voltage), se produce un efecto de avalancha en las uniones 11 y 13 y la corriente se eleva bruscamente, pudiendo causar la destruccin del tiristor. Cuando se aplican tensiones positivas (nodo positivo respecto al ctodo), la corriente que llevan es despreciable y que se debe a la pequea corriente de dispersin de la unin central 12, que queda polarizada inversamente. Sin embargo, si la tensin directa nodo-ctodo alcanza un valor crtico, denominado tensin de ruptura directa VIII (forward breakdown voltage], el tiristor pasa al estado de conduccin. Si habiendo aplicado tensiones positivas se introduce una corriente en el electrodo de puerta (en el sentido puerta-ctodo), se reduce la tensin de ruptura directa; para una corriente de puerta suficientemente elevada (del orden de 20 a 200 mA) se elimina la regin de bloqueo directo y el tiristor inicia su conduccin comportndose como un diodo. Cuando el dispositivo est conduciendo, se puede eliminar o desconectar la corriente de puerta y el tiristor sigue en conduccin, estado ON (lo que indica que en la prctica es suficiente aplicar un pequeo impulso de corriente a la puerta durante un tiempo del orden de 10 a 50 microsegundos para iniciar la conduccin del tiristor, sin ser necesario mantener constantemente la corriente de puerta). Sin embargo, si la corriente principal o de nodo es inferior a un valor crtico, denominado corriente de mantenimiento 1" (que es del orden de 20 mA para un tiristor de 500 Y, 100 A), el dispositivo pasa al estado de bloqueo o no conduccin. En la Figura 7.6 se muestran las curvas caractersticas reales e ideales de los tiristores, donde se aprecian los hechos mencionados. Tericamente el apagado del tiristor se produce de una forma natural cuando la corriente de nodo es nula (en realidad cuando es inferior a 1,,), y esta forma de desconexin se denomina por ello apagado por conmutacin natural o conmutacin en lnea; esta ltima denominacin procede de que al trabajar el tiristor en redes de c.a., la corriente pasar por cero al final de cada semi ciclo positivo, producindose una desconexin natural en sincronismo con la tensin de lnea. Esta forma de actuar es la que tienen los tiristores cuando trabajan como rectificadores controlados para suministrar una c.c. regulada a partir de una red de alimentacin de c.a. Sin embargo, cuando se utiliza el tiristor en una red de C.C., la corriente no pasa por cero de una forma natural (no hay semiciclos negativos), y en este caso se deben usar circuitos especiales para lograr el apagado del dispositivo, a base de hacer que la corriente principal se anule, inyectando corriente en sentido contrario (es decir, entre ctodo y nodo); se dice en-

CAPTULO 7.

ACCIONAMIENTOS ELCTRICOS

559

Estado de conduccin

.J
~aso de bloqueo ) a conduccin

o
Bloqueo inverso Caracterstica real de un tiristor Bloqueo directo

a)

b)

Caracterstica ideal de un tiristor

Figura 7.6. Curvas real e ideal de un tiristor.

tonces que el apagado del tiristor, estado OFF, se produce por conmutacin forzada. Esta forma de trabajar es la que se utiliza en los inversores y choppers. Otro aspecto a considerar es que, si se aplica una tensin directa inmediatamente despus de anularse la corriente de nodo, al tiristor no le dar tiempo a bloquear la tensin directa y comenzar a conducir sin haber aplicado ningn impulso de disparo a la puerta. Es necesario por ello mantener el dispositivo polarizado inversamente durante un tiempo finito antes de aplicar tensiones directas. Este perodo se conoce como tiempo de apagado t off del tiristor (que es del orden de 10 a 100 microsegundos). Tambin se puede producir un encendido intempestivo del tiristor, si se aplica una tensin directa muy rpida, lo que se conoce como dv/dt de encendido; para evitar este problema se suele colocar un circuito serie resistencia-condensador, que se conecta en paralelo con los terminales nodo-ctodo y que se denomina red snubber. Otro aspecto a considerar es que si la corriente de nodo sube muy rpidamente (di/di alta), se puede averiar o destruir el tiristor; para evitar este problema suelen colocarse inductancias en serie con el dispositivo. En la prctica del control de motores elctricos suele ser suficiente la inductancia de los propios devanados de la mquina, no necesitndose ninguna bobina adicional. La cada de tensin directa en los tiristores cuando estn conduciendo es algo superior a la de los diodos rectificadores, ya que en este caso oscila entre 1 y 2,5 V, dependiendo de la tensin de alimentacin del circuito. Los valores lmites de corriente y tensin mxima de pico en estos componentes son comparables con los de los diodos. En la industria se utilizan tiristores que pueden llevar corrientes de 6.000 A, con tensiones inversas mximas de 12 kV, pero con frecuencias de conmutacin muy limitadas que no suelen superar los 300 a 400 Hz. Su aplicacin ms importante est como rectificadores e inversores controlados en instalaciones de c.c. en alta tensin (HVDC, high voltage direct current, corriente continua en alta tensin) y en el accionamiento de grandes motores (hasta 25-30 MW). Un dispositivo SCR especial que equivale a dos tiristores conectados en paralelo-inverso es el triac. En la Figura 7.7 se muestra el smbolo correspondiente. El triac funciona como un tiristor tanto en en los semiperodos positivos como negativos; es decir, cuando TI es positivo respectoa Tl, el triac entra en conduccinal aplicar un impulso de puerta positivo. Al contrario, si n es positivo respecto a TI, el dispositivose activa al aplicar un impulso negativo a la puerta. El triac se utiliza ampliamente en aplicaciones de pequea potencia en el control de alumbrado de lmparas de incandescencia, en la regulacin de velocidad de licuadoras, batidoras domsticas y tambin en el control de taladros elctricos de mano. El triac es ms

560

MQUINAS ELCTRICAS

TI

TI

ri

r:
al
Smbolo del triac b) Circuito equivalente

ri
el Forma constructiva

Figura 7.7. Triac: Smbolo, circuito y forma constructiva.

econmico que el empleo de dos tiristores en oposicin; sin embargo, su capacidad dv/dt es ms bien reducida y con tiempos de apagado elevados, y por ello no se fabrica ni se emplea en alta potencia.

7.2.4. Tiristor de apagado por puerta (GTO)


El tiristor de apagado por puerta (gate-turn-off), conocido vulgarmente por sus iniciales OTO, puede pasar al estado de conduccin (encendido) aplicando un impulso positivo a la corriente de puerta, al igual que en los tiristores convencionales, pero tienen adems la ventaja frente a estos ltimos de que pueden desconectarse (apagado) tambin aplicando a la puerta un impulso de corriente pero de signo negativo. Debe destacarse, sin embargo, que la corriente que debe aplicarse a la puerta para producir el apagado del OTO es muy elevada, y corresponde a una relacin Afie pequea y que en la prctica vara entre 3 y 5. Por ejemplo, para un OTO tpico de 2.500 V, 1.000 A, se requiere un pico de corriente de puerta negativa del orden de 250 A. Como quiera que en el proceso de apagado tanto la corriente como la tensin tienen valores elevados, las prdidas por conmutacin son grandes y es por este motivo por lo que sus aplicaciones se limitan a circuitos con frecuencias de conmutacin inferiores a 1 kHz. Se fabrican OTO con valores asignados o nominales de 6 kV y corrientes de 6 kA. En la Figura 7.8 se indica el smbolo de este tiristor, junto con las curvas reales e ideales que rigen su funcionamiento.

ON ON
) \pagado

..-1
VAK

Paso de bloqueo a \nduccin y viceversa

VAK

_!}_o~~E~n~ce~n~d~id~o~J~IO~F:F

--+
ic

K
a) Smbolo del GTO

,
b) Curvas reales

O Bloqueo inverso

~ ~OFF

el Curvas ideales

Figura 7.8. Tiristor GTO. Smbolo, curvas reales e ideales.

CAPTULO 7.

ACCIONAMIENTOS ELCTRICOS

561

7.2.5. Transistor bipolar de unin (BJT)


El transistor de unin bipolar (bipolarjunction transistor) es un dispositivo de tres terminales, formado por dos uniones PN creadas en un nico cristal semiconductor.Esta disposicin, junto con una polarizacin adecuada, permite utilizar el transistor como amplificador de corriente. El transistor de unin se denomina tambin transistor bipolar para distinguirlo de otros transistores empleados en electrnica, como son el transistor de efecto de campo (FET), transistor uniunin (UJT), etc. El transistor fue descubierto en diciembre de 1947 por los fsicos americanos Bardeen y Brattain y el nombre procede de la contraccin de la expresin inglesa Transfer resistor. Hay dos tipos de configuraciones del transistor bipolar, el NPN y el PNP. En la Figura 7.9 se representan ambos tipos junto con los smbolos empleados para su identificacin en los esquemas. El transistor NPN (Fig. 7.9a) tiene una regin tipo P entre dos tipo N, mientras que el transistor PNP (Fig. 7.9b) tiene una regin tipo N entre dos tipos P. Los tres terminales externos se conocen con el nombre de emisor, base y colector. Hoy da los transistores son casi todos de silicio y la configuracin ms frecuente es la NPN. En los smbolos de identificacin, la flecha representa el emisor y seala el sentido real del flujo de cargas positivas en la regin del emisor para cada tipo de transistor. Tambin se han mostrado en la Figura 7.9 los sentidos reales de las intensidades en la base y colector. En electrnica de potencia los transistores trabajan siempre en conmutacin, es decir, en las zonas de corte y saturacin. Para ello los terminales de colector y emisor se conectan al circuito principal y el terminal de base se toma como electrodo de control. En la Figura 7.10 se muestra un transistor de potencia utilizado en control de motores, junto con las curvas caractersticas reales e ideales del mismo. Si la corriente de base es cero, iB = O, el transistor est en corte o apagado (OFF), actuando como un interruptor abierto. Sin embargo, si se aplica una corriente a la base suficientemente grande para llevar el transistor a la zona de saturacin, entonces el transistor conduce, estado ON, y acta como un interruptor cerrado. En esta situacin la tensin vCE(sat) es del orden de 1 o 2 V, por lo que las prdidas por conduccin en el BJT son relativamente pequeas. Debe destacarse que el transistor es un dispositivo activo controlado por corriente, ya que la corriente de base determina si el estado del transistor es abierto o cerrado. Para mantener el transistor en conduccin, la corriente de base debe ser suficiente (es necesario que la corriente de base iB sea superior a ic/hFE, siendo ic la corriente de colector y hFE la denominada ganancia de corriente en c.c. del transistor). Los transistores preparados para tensiones y corrientes elevadas se conocen con el nombre de
C

E
a) Transistor NPN b) Transistor PNP

Figura 7.9. Tipos de transistores.

562

MQUINAS ELCTRICAS

Saturacin _-----i86

ic

__ ----i
e
__

ON
83

Estado de conduccin

__ ----i82
--__ i81

~
~, Paso de bloqueo a ~conduccin y viceversa

Corte

ie = O

o
a)

VCE(sat)

o
b)

t.

OFF

No conduccin
e)

Figura 7.10. Transistor de potencia: Forma constructiva, curvas reales e ideales.

transistores de potencia, y en la prctica se construyen mediante conexiones especiales entre transistores como el tipo Darlington, que es una combinacin tipo paralelo entre dos transistores y que se fabrican directamente en una misma pastilla de circuito integrado o chip. Existen transistores de potencia preparados para tensiones de 1.500 y y corrientes de 1.000 A, con tiempos de conmutacin que van desde 400 ns hasta 5 us.

7.2.6. Transistor MOSFET


El transistor de potencia MOSFET (metal oxide semiconductor-field effect transistor) es un transistor que conmuta muy rpidamente (del orden de 200 ns), por lo que se utiliza en las aplicaciones con alta frecuencia (del orden de l MHz) y baja potencia (unos pocos kW). Existen diversos nombres comerciales para este tipo de transistores, como HEXFET (de la empresa americana Intemational Rectifier), SINMOS (Siemens) y TIMOS (Motorola). En la Figura 7.11 se muestra el smbolo correspondiente y las curvas de comportamiento real e ideal. Los tres terminales se denominan: drenador D (drain), fuente S (source) y puerta G (gate). La corriente circula en el sentido drenador-fuente, El dispositivo tiene muy poca capacidad de bloqueo a la tensin inversa y por ello lleva integrado en su interior un diodo en sentido inverso. Al contrario que un transistor bipolar BJT, que es un dispositivo que dispara por corriente, un MOSFET es un dispositivo controlado por tensin (esta tensin es del orden de 5 Y). Cuando se aplica una tensin positiva entre la puerta y la fuente Ves positiva, el transistor entra en conduccin. La puerta est aislada por una capa de xido de silicio Si02, por lo que la impedancia de entrada de este dispositivo es muy elevada, lo que tiene la ventaja de que se puede actuar directamente con lgica digital CMOS o TTL, ya que la corriente de actuacin de puerta es menor de 1 mA. El MOSFET conmuta muy rpidamente y las prdidas en este proceso son despreciables; sin embargo, cuando conduce (estado ON) su cada de tensin es alta, por lo que las prdidas por conduccin son elevadas. Por ejemplo, para un MOSFET de 400 Y, lOA, la cada de tensin es de 2,5 Y Yesta cada se incrementa con la temperatura y la corriente. Existen en el mercado MOSFET que llegan a 600 Y, 50 A y 50 nanosegundos de tiempo de conmutacin.

CAPTULO

7.

ACCIONAMIENTOS ELCTRICOS

563

+
VDS

Ves i[) S a) ON ves6 io ON h)

vess
vC;S4 ves3 ves2 OFF O
e)

_.I
Paso de bloqueo a ~ondUCCin y viceversa

vesl

vos

OFF

I'DS

dl

Figura 7.11.

Transistor MOSFET: Smbolo, forma constructiva y curvas reales e ideales.

7.2.7. Transistor bipolar de puerta aislada IGBT


El transistor bipolar de puerta aislada IGBT (insulated gate bipolar transistor) es un dispositivo semiconductor de potencia hbrido, que combina las propiedades del transistor de unin BJT y del MOSFET. La puerta es tipo MOSFET, lo que le da una gran impedancia de entrada y la conduccin o encendido del IGBT se hace por tensin como en el MOSFET. Al igual que el BJT, el IGBT tiene una cada de tensin en el estado de conduccin que es reducida. La velocidad de conmutacin es significativamente ms pequea que el MOSFET y del orden del BJT. El smbolo es el mostrado en la Figura 7.12 y tambin sus curvas de funcionamiento. Existen en el mercado IGBT de 2.000 V, 3.000 A Y que son valores muy superiores a los transistores de unin BJT. La frecuencia de conmutacin es del orden de 20 kHz.
io

+
VDS

i
Ves

iD ON Paso de bloqueo a \OlldUCCin y viceversa

O
VDS

OFF

VDS

al

h)

el

Figura 7.12. Transistor bipolar de puerta aislada IGBT: Smbolo, curvas reales e ideales.

564

MQUINAS ELCTRICAS

7.2.8. Tiristor MCT


El tiristor controlado MOS, denominado por sus iniciales MCT (MOS-eontrolled thyristor), es un semiconductor de potencia hbrido que combina las propiedades del MOSFET y del tiristor. Es el componente electrnico ms reciente (ao 1988) y su smbolo es el mostrado en la Figura 7.13. El MCT es bsicamente un tiristor que se enciende o se apaga por medio de una puerta tipo MOSFET. Tiene una alta di/dt (del orden de 100 A/ps) y un tiempo de apagado de 1,5 us. Estas caractersticas, tan superiores a los dems dispositivos comentados anteriormente, le hacen que se considere el dispositivo de conmutacin ideal, por lo que se le augura un gran futuro para su uso en accionamientos de motores de media y alta potencia. La primera generacin de MCT es de 600 Y, 75 A, pero se espera que en un futuro cercano sean ms elevadas.

7.2.9. Funcionamiento ideal de los dispositivos semiconductores


Los dispositivos semiconductores de potencia hasta aqu mencionados corresponden a la mayora de los tipos existentes en el mercado. Pese a esta gran diversidad, se pueden clasificar en las tres categoras fundamentales mostradas en la Figura 7.14, a saber: a) diodos rectificadores, que slo pueden conducir corriente en un sentido (de nodo a ctodo) y no disponen de electrodo de control; b) tiristores, que conducen en un solo sentido, siempre que se aplique una corriente de control al electrodo de puerta. Los triaes se pueden incluir en esta categora y pueden conducir la corriente en ambos sentidos actuando sobre la corriente de puerta. En ambos dispositivos SCR (rectificadores controlados de silicio) se pasa al estado de desconexin o apagado por conmutacin natural, al anularse la corriente de nodo; e) dispositivos autoconmutados, en el que se incluyen de una forma genrica todos los dems, es decir: GTO, BJT, MOSFET, IGBT y MCT. Se han representado en la Figura 7.14e para mayor sencillez por un nico smbolo, en forma de interruptor al que se le aade una flecha para indicar la propiedad unidireccional de la corriente en todos ellos. Disponen de un electrodo de control denominado puerta (o base, en el caso de los BJT) que permite tanto el apagado como el encendido de los mismos (autoconmutados). Evidentemente, cada uno de los dispositivos estudiados tiene propiedades especficas, como capacidad de llevar corriente, bloqueo a la tensin inversa, tiempo de encendido y apagado, frecuencia mxima de conmutacin, etc., pero a nivel funcional se comportan como simples interruptores estticos, que permiten la circulacin de corriente en el sentido de la flecha cuando entran en conduccin (estado ON) y no dejan pasar la corriente cuando estn

Figura 7.13. Tiristor MCT: Smbolo.

CAPTULO 7. ACCIONAMIENTOS ELCTRICOS

565

:*
q
K a) Diodo rectificador

;~
A G
K

+
v

;\

----

G (puerta)

- I
e) Dispositivo autoconmutado

b) Tiristor

Figura 7.14.

Categoras de dispositivos semiconductores de potencia.

apagados (estado OFF). En lo que sigue en este captulo se supondr que estos interruptores estticos cumplen las condiciones ideales siguientes: a) En el estado OFF no dejan pasar la corriente a travs de ellos, bloqueando tanto las tensiones directas como inversas aplicadas. b) En el estado ON conducen las corrientes permitidas sin que se produzcan cadas de tensin apreciables. c) La conmutacin encendido-apagado (ON-OFF) o viceversa se produce instantneamente al dar seal al electrodo de control (puerta o base en su caso). d) La potencia necesaria para actuar (disparar) el interruptor es despreciable.

7.3. CONVERTIDORES ELECTRNICOS DE POTENCIA 7.3.1. Introduccin


Los convertidores electrnicos de potencia estn basados en el uso de dispositivos semiconductorescomo los explicados en el epgrafe 7.2 y que trabajan en conmutacin comportndose como interruptores estticos que pueden estar cerrados o abiertos y que se considera a partir de ahora que tienen un funcionamiento ideal. Antes de sealar los diversos tipos de convertidores conmutados es interesante comprender la ventaja que ellos presentan frente a los sistemas clsicos convencionales basados en ajustar la tensin en una carga, que puede ser una resistencia, un motor, etc., introduciendo una resistencia variable en serie con la carga para provocar una cada de tensin intermedia. Paraello se va a considerar el ejemplo mostrado en la Figura 7.15, en el que se dispone de una fuentede c.c. de 200 V que debe suministrar una potencia de 500 W a una resistencia elctrica de 20 ohmios. En el caso de la Figura 7.15a se considera el sistema clsico que utiliza una resistencia en serie y en la Figura 7.15b se considera un interruptor electrnico sealado por un smbolo como el indicado en la Figura 7.14c que se va a abrir y cerrar de un modo peridico, haciendo variar de este modo el valor medio de la tensin de c.c. que llega a la resistencia de carga. Vamos a comparar los rendimientos de ambos circuitos. Indudablemente, si se conecta la resistencia de carga de 20 ohmios directamente a la fuente de c.c. de 200 V, la potencia disipada en la carga sera: P

=R

. /2

= ....:'.. =R

V2

2002 20

= 2.000 W

Para limitar la potencia en esta carga es necesario incluir una resistencia en serie R, tal como se indica en la Figura 7.15a. Podemos determinar el valor de esta resistencia calculando

566

MQUINAS ELCTRICAS

Resistencia en serie

a)

Resistencia de carga

b)

Resistencia de carga

~ ~------------~
200Y-e)

TI

.........
o

TI

= 0,25 T

Figura 7.15.

Estudio comparativo de un divisor de tensin y un interruptor electrnico.

previamente la tensin que debe llegar a la carga VR para que la potencia disipada en R sea igual al valor requerido de 500 W. De acuerdo con la ecuacin anterior, se debe cumplir: P = 500 = R . [2 = V; = V;
R
=>

20

VR =

J 20 . 500 = 100 V

y en consecuencia la cada de tensin que deber haber en la resistencia en serie ser de 200 - 100 = 100 voltios, lo que indica que R, debe ser igual a R y por tanto igual 20 ohmios y en R, se disiparn tambin 500 W; de este modo el rendimiento energtico del circuito es del 50 por 100, ya que la fuente de c.c. debe producir 1.000W, de los cuales 500 W se disipan en R, y los otros 500 W se disiparn en la resistencia de carga R. Si se utiliza el circuito de la Figura 7.15b, es indudable que si el interruptor electrnico S est siempre cerrado, la potencia disipada en la resistencia de carga sera, como ya se ha indicado antes, de 2.000 W; por otro lado, si el interruptor est abierto, la resistencia de carga no consumir energa; ahora bien, si el interruptor se cierra y se abre peridicamente y se grada el cociente TI/T (vase Fig. 7.15c), se podr lograr la potencia deseada de 500 W. Como quiera que se requiere una potencia de 500 W frente a 2.000 W, el cociente T/T deber ser: -=--=025 T 2.000
TI 500 '

Es decir, si el interruptor o conmutador electrnico est cerrado el 25 por 100 del tiempo, entonces la potencia media disipada en la resistencia de carga ser el 25 por 100 de 2.000 W, esto es, 500 W, que es el valor que se deseaba conseguir. Es evidente que con este procedimiento toda la energa que produce la fuente de c.c. se entrega a la carga, sin ninguna prdida intermedia. En la prctica, como quiera que el interruptor no es ideal, existe alguna prdida en este dispositivo pero de valor muy reducido; por ejemplo, si se considera que el

CAPTULO 7.

ACCIONAMIENTOS ELCTRICOS

567

interruptor electrnico es un transistor de unin cuya cada en conduccin es del orden de 2 V, Y teniendo en cuenta que la corriente que absorbe la resistencia de carga en directo es 1 = 200/20 = 10 A, entonces la potencia perdida en el transistor ser de 2 . 10 = 20 W, pero como esto ocurre durante un 25 por 100 del tiempo de funcionamiento, la potencia media perdida ser slo del 25 %20 = 5 W, y de este modo de los 500 W que entrega la fuente, 5 W se disiparn en el interruptor electrnico (transistor) y llegarn a la carga 495 W, lo que representa un rendimiento energtico del 99 por 100. El anlisis anterior es muy simplificado pero permite comprender por qu los convertidores electrnicos se han impuesto en el control y regulacin de mquinas elctricas. Los diversos tipos de conversin que se utilizan en la prctica se pueden clasificar en funcin de los tipos de seales, bien sean de c.a. o de C.C., que utilizan y responden a los grupos o clasificaciones siguientes: a) Conversin corriente alterna a corriente continua: En este caso se dispone de una red de c.a. que se transforma por medio de una rectificacin en una seal unidireccional, que ms tarde se filtra por medio de bobinas y condensadores para obtener una salida aplanada que representa una onda de c.c. casi pura. Los convertidores utilizados se denominan rectificadores. Se puede variar la salida de .c.C. utilizando un rectificador controlado o tiristor; en esta situacin el convertidor se puede utilizar, como se ver ms adelante, en la regulacin de velocidad de motores de c.c. h) Conversin corriente continua a corriente continua: Una tensin de c.c. se transforma en otra c.c. de amplitud menor e incluso mayor. Los convertidores que realizan esta aplicacin se denominan choppers o recortadores. Este sistema se utiliza en la regulacin de velocidad de trenes metroplitanos que disponen de una alimentacin de 600 V Y que recortando esta seal, de un modo anlogo al sealado en la Figura 7.15c, pueden regular la velocidad de los motores de c.c. encargados de realizar la traccin del tren. e) Conversin de corriente continua a corriente alterna: Se dispone de una red de c.c. que se conmuta de una forma adecuada para obtener corriente alterna. Los convertidores que realizan esta operacin se denominan inversores y tambin onduladores. La salida de estos inversores est formada por trozos de ondas rectangulares o tambin escalonadas y se utilizan para el control de velocidad de motores de c.a. a partir de redes de C.C.; para ello se modifica la frecuencia de la onda de c.a. por tcnicas de conmutacin dentro del inversor. La forma de onda se puede aproximar a la senoidal modulando el ancho de impulso PWM (pulse width modulation). Existen en la actualidad muchos trenes alimentados por una red de c.c. que llevan, sin embargo, motores de c.a. asncronos, cuyo arranque y regulacin de velocidad se consigue por medio de inversores. d) Conversin de corriente alterna a corriente alterna: Existen tres tipos de convertidores que se incluyen en este grupo, a saber:
1.

Conversin c.a-c.c.-c.a.: En este caso la conversin de corriente alterna a corriente alterna incluye una etapa intermedia de c.c. Para pasar de la corriente alterna de entrada a corriente continua se utilizan rectificadores fijos o controlados (tiristores); en la etapa siguiente se transforma la corriente continua intermedia en corriente alterna por medio de inversores. Este sistema se utiliza con frecuencia para el control de velocidad de motores de c.a. sncronos y asncronos. Por ejemplo, de este modo se arrancan y regulan los trenes de alta velocidad espaoles (AVE); a partir de una red de 25 kV de c.a. monofsica, se obtiene c.c.

568

MQUINAS ELCTRICAS

por medio de rectificadores controlados, y en la etapa siguiente, por medio de inversores, se consigue una c.a. trifsica de frecuencia y tensin variable. 2. Conversin c.a. a c.a. por control de fase: Se utiliza una tensin de c.a. que se conmuta peridicamente una vez en cada semiciclo, dando lugar a una c.a. controlada en fase de la misma frecuencia que la de entrada pero de menor amplitud. Los dispositivos empleados son los triacs o dos tiristores en paralelo-inverso. Este sistema se utiliza para el control de alumbrado incandescente en iluminacin industrial y domstica. Tambin se emplea en la regulacin de velocidad de motores monofsicos de c.a. con colector de delgas (de pequea potencia), como en batidoras domsticas, taladros elctricos, etc. Este sistema se utiliza poco en la regulacin de velocidad de motores trifsicos debido a que solamente admiten el control de la tensin y no de la frecuencia, por lo que no son adecuados para accionamientos elctricos importantes, ya que al reducir solamente la tensin aplicada a un motor se reduce el par electromagntico que produce la mquina, que depende, como se sabe, del cuadrado de la tensin aplicada. 3. Conversin directa c.a.-c.a.: En este caso se obtiene una c.a. de frecuencia variable a partir de trozos de una c.a. de entrada; los convertidores se denominan entonces cicloconvertidores. La entrada generalmente es una red trifsica, y la salida, tambin trifsica, tiene una frecuencia menor que la correspondiente de entrada. Este tipo de conversin no es muy atractivo comercialmente porque requiere un equipo de control bastante complejo. Se utiliza en la regulacin de velocidad de motores de c.a. trifsicos de gran potencia.

7.4. RECTIFICADORES 7.4.1. Rectificador monofsico media onda con carga resistiva
Constituye el ms simple de los rectificadores. Considrese una tensin alterna senoidal:
Vs

Vm sen wt

= Jzv

sen wt

(7.1)

donde Vm expresa el valor mximo o de pico y Vel valor eficaz. Si se aplica esta seal a un circuito serie formado por un diodo y una carga resistiva (Fig. 7.16), el diodo conducir cuando est polarizado directamente, es decir, en aquellos semiciclos de la seal generadora en que el nodo sea positivo frente el ctodo, lo que corresponde a los semiciclos positivos, ya que entonces el terminal A del generador (aplicado al nodo del diodo) es ms positivo que el terminal B (que por medio de R est conectado al ctodo). Durante estos semiciclos positivos, si se supone el diodo ideal (es decir, sin cadas internas de tensin), la tensin del generador aparecer en la carga vR' mientras que la d.d.p. en el diodo ser cero. En los semiciclos negativos de la tensin del generador el terminal B ser ms positivo que el A, quedando el diodo polarizado inversamente y no dejando pasar ninguna corriente a travs de l. Durante estos semiperodos no se tendr ninguna tensin en la carga y la tensin de la red quedar aplicada al diodo. Partiendo de una tensin alterna senoidal, se obtiene una tensin unidireccional en la carga (resistencia) del tipo semionda o media onda (curva discontinua de la Fig. 7.16), que da origen al nombre con que se designa el rectificador. La onda es unidireccional en el sentido de que no cambia de polaridad; sin embargo, es oscilante debido que lleva incorporadas componentes alternas. Estas componentes alternas se

CAPTULO

7.

ACCIONAMIENTOS ELCTRICOS

569

pueden obtener desarrollando en serie de Fourier la funcin peridica correspondiente a Como el lector puede demostrar fcilmente, se obtiene la siguiente expresin: vR = -

vR'

v,,, + -v,,, sen w( ti

2V,,, ~
-

L.

l
-2--

n~2.4.6

n -

cos ruot

(7.2)

Esta ecuacin demuestra que la salida del rectificador de media onda tiene un trmino de corriente continua (V,,/n) que es el valor medio Ve, de la tensin vR; otro trmino (V,,/2) sen on que demuestra que la salida contiene energa a la frecuencia de la red, y una serie de trminos de orden superior cuyas frecuencias son mltiplos pares de la frecuencia de la red (armnicos pares). A medida que aumenta la frecuencia de un armnico, su amplitud decrece. En resumen, la tensin media de la c.c. resultante vale:

v'T = - = -n que corresponder a una corriente:


1 _ V" _

Vm

j2v = 0,318 v,,, n


v'1l
nR
=

(7.3)

j2v
nR

cCR

(7.4)

que ser la corriente media del diodo. Si se observa la Figura 7.16, la tensin inversa de pico en el diodo (PIV) ser igual a la mxima de la red v'1l = por tanto, la PIV del diodo se elegir para que soporte esta tensin inversa mxima. Para poder juzgar la calidad de onda que suministra un rectificador, se suele definir lo que se denomina factor de rizado, que viene a indicar el porcentaje de c.a. que lleva superpuesta la C.C.; este factor r viene dado por el cociente:

j2v;

r =

valor eficaz de la componente alterna de la tensin de salida valor medio de la tensin de salida

(7.5)

I ciclo = T seg = 2n rad.


Vs

\{,
wt

+
R vR vR

v
I I

"",)

Vm
wt

I I

Figura 7.16. Rectificadormedia onda.

570

MQUINAS ELCTRICAS

Como quiera que la componente alterna de salida se llama el rizado, si ste tiene un valor eficaz Vr y es Vec el valor medio de la continua, la expresin (7.5) se puede poner: (7.6) Para calcular este cociente, consideremos una tensin instantnea vR en la salida que se compone de una tensin continua v.'e y una tensin instantnea de alterna vac' y se tendr entonces: (7.7) Por definicin de valor eficaz de una seal peridica el lector puede demostrar que el cuadrado del valor eficaz de la tensin total es igual a la suma de los cuadrados de los valores eficaces de las tensiones componentes. Si el valor eficaz de la tensin vR es Vef y el de vae es Vr' la ecuacin (7.7), expresada como relacin entre valores eficaces, ser:

V,} = V; + V,~
en consecuencia:

(7.8)*

v
-1
T y teniendo en cuenta que

JV

2. ef -

2 ce

(7.9)

pero el valor eficaz de la semionda total (sealada por trazo discontinuo en la Fig. 7.16) vale:

lT v (t)
2

dt =

1 2n

In

Jo

(7.10)

\1.,= v,. resulta:


(7.11)

y por tanto el rizado ser:

r=-=--= \l.c V;,,/n

Vr

0,38 V;"

1,21

(7.12)

Otro aspecto a considerar en los rectificadores es la llamada eficacia o rendimiento de la rectificacin, definida como el cociente entre la potencia de corriente continua entregada a la carga Pee Yla potencia total que se enva a la misma P, es decir: (7.13)
y como quiera que se cumple: 1= V
_!!!.

2R

(7.14)

* Es fcil deducir esta relacin teniendo en cuenta el sentido fsico del valor eficaz. Si a la resistencia de carga R se le aplica una d.d.p. continua V;,., la potencia disipada ser V,",/R. Cuando se aplica la c.a. V,. se disipa: V;IR. Si se aplican las dos tensiones, es decir, una total del valor eficaz, Vse disipar: V21R. Al igualar esta ltima potencia a la suma de las otras dos se obtiene la ecuacin (7.8).

CAPTULO

7.

ACCIONAMIENTOS ELCTRICOS

571

resultar: (7.15) Si el generador del circuito de la Figura 7.15 fuera el secundario de un transformador, la

tensin eficaz V sera Vj)2 y la intensidad eficaz 1, de acuerdo con (7.14), sera igual a v'"f2R. En consecuencia, el factor de utilizacin f; del transformador, definido como cociente entre la potencia de c.c. de la carga y la potencia de c.a. del transformador, valdr:

f, = P

P.e

(7.16)

transformador

El significado de este rendimiento se puede explicar mejor utilizando un ejemplo numrico. Si se usa un transformador de 1 kV A en un rectificador media onda con carga resistiva (1 kW), la potencia de c.c. disponible en la carga ser slo de 287 W (sin considerar las prdidas de potencia en el diodo). Otro aspecto a considerar es que la corriente continua de la carga pasa. por el secundario del transformador, pudiendo saturar el ncleo del mismo. Para prevenir este efecto el diseo de transformadores utilizados en equipos de rectificacin debe calcularse con inducciones bajas: 9 a 11 kgauss, en vez de 14 a 15 kgauss que es lo normal para chapas de grano orientado. En conclusin, se puede establecer que los rectificadores media onda tienen las siguientes desventajas: J) Rizado muy grande. 2) Rendimiento de la conversin o relacin de rectificacin pequea. 3) Factor de utilizacin del transformador bajo. 4) Posibilidad de saturacin

de C.c.del secundario del transformador. EJEMPLO DE APLICACIN 7.1 El circuito rectificador de la Figura 7.16 se conecta a una alimentacin de c.a. de tensin eficaz 30 V, 50 Hz. Si la potencia de c.c. en la carga R es de lOOW, calcular: a) Tensin de c.c. en la carga Vee. b) Resistencia de la carga. c) Potencia aparente suministrada por la fuente de alimentacin de c.a.
SOLUCIN a) La tensin mxima o de cresta de la alimentacin es)2 una tensin sinusoidal), es decir: veces el valor eficaz (por ser

v'1l
La tensin

VJ2

= 30J2 = 42,43 voltios


42,43

v"e

viene expresada por la relacin (7.3), es decir:

V. ce.
b)

= -- 1! = -- n =
P.e

vJ2

135 voltios
'

Como quiera que la potencia

en la carga es de 100 W, la intensidad de c.c. ser:

Pe! 100 . (,. = = -_ = 7,4 amperIos Vec 13,5

S72

MQUINAS ELCTRICAS

En consecuencia, la resistencia que ofrece la carga ser:

V:.t 13,5 R = - = -_ = I 82 ohmios Icc 7,4 ,


e) La intensidad eficaz de la corriente que da el secundario del transformador, teniendo en cuenta (7.14), es:
Vm 42,43 1= - = -= 11 66 A 2R 2 1,82 '

En consecuencia, la potencia aparente entregada por la fuente de c.a. ser: S = VI = 30 . 11,66 = 349,8 VA.

7.4.2. Rectificador monofsico media onda con carga inductiva


El comportamiento de un rectificador (yen general de un convertidor esttico) depende en bastante medida del tipo de carga que alimenta. En el epgrafe anterior se ha analizado el funcionamiento de un rectificador monofsico media onda que alimenta una carga resistiva. Se observa que en este caso la corriente en la resistencia de carga tiene la misma forma que la tensin. En la prctica de la electrnica de potencia, las cargas suelen ser motores elctricos cuyos devanados ofrecen resistencia e inductancia. Se va a estudiar en este epgrafe la influencia que ejerce esta inductancia en la forma de la onda de corriente en la carga y en el modo en que se produce la conduccin en el diodo rectificador. Considrese para ello el esquema de la Figura 7.17, que muestra un rectificador monofsico media onda que alimenta una resistencia R en serie con una inductancia L. Para calcular en este caso la corriente en la carga se debe aplicar el 2. lema de Kirchhoff al circuito de la Figura 7.17a, mientras el diodo D est en conduccin, lo que da lugar a la siguiente ecuacin diferencial:
di L dt

+ Ri = V sen ost
m

(7.17)

cuya solucin es la suma de una corriente estacionaria o de rgimen permanente (respuesta forzada o solucin particular de la ecuacin diferencial anterior) i/t) y una corriente transitoria it(t) (respuesta natural, o solucin de la ecuacin diferencial homognea). La corriente de rgimen permanente se determina en la forma clsica que se emplea en la teora de circuitos de c.a. (es decir, en el dominio fasorial), lo que da lugar a:
Vm it) = - sen (on - 8)

(7.18a)

donde los valores de Z y 8 son: 8 = arctgLw R (7.18b)

La corriente transitoria es la solucin de la ecuacin diferencial homognea, es decir, haciendo el 2. trmino de (7.17) igual a cero, lo que conduce a un valor:
0

iJt)

= Ae -T

con

= L/ R

(7.18c)

CAPTULO 7.

ACCIONAMIENTOS ELCTRICOS

573

wt

b)

~ +
VL

+
L
Vo wt

= Ldi/dt
+
VR

s = v'n

senwt

= Ri

e)

al
WI

d)

Figura 7.17.

Rectificador media onda con carga inductiva.

donde A es una constante de integracin y r un parmetro del circuito, denominado constante de tiempo, y que es igual a L/R. Por consiguiente, la solucin completa de (7.17) es:
i(t)

= ip(t) + it(t) = -

Vm

sen (wt - 8) + Ae

.:
T

(7.19a)

Si se considera que en t = O la corriente total anterior i(t) es igual a cero, se obtiene que A = (Vm sen 8)/Z, de donde se deduce finalmente una expresin para la corriente:
i(t) = Vm

sen (cot - 8) + sen 8 e

-~J
T

(7.19b)

cuya evolucin con el tiempo se ha sealado en la Figura 7.17c. A partir de la expresin anterior se puede calcular el valor del tiempo fa O su correspondiente ngulo tjJ, para el cual la corriente i(t) se hace cero (vase Figs. 7.17c y d). Es importante darse cuenta en las ondas de la Figura 7.17 que, al ser el circuito inductivo, la corriente circula ms all de T12, es decir, de n radianes, por lo que el diodo D sigue conduciendo hasta el tiempo ta en el que se anula la corriente en el circuito, pese a que la tensin existente entre TI2 y fa es negativa. La tensin en la carga vo(t) tiene la forma mostrada en la Figura 7.17d, cuyo valor medio es:
Vec =
_!!!_

2n

v loJ; sen 8 d8 = V
o

_!!!_

2n

[1 - cos tjJ]

(7.20)

Comparando esta ecuacin con la (7.3) se observa que el valor medio de la tensin en la carga se reduce al aumentar el valor de la inductancia del circuito, y por ello no es muy til para rectificar grandes potencias. Una forma de mejorar el circuito anterior para que sea til en aplicaciones de pequea potencia es colocar un segundo diodo D en paralelo con la carga, tal como se muestra en la Figura 7.18a. Este diodo recibe el nombre de diodo volante o de libre

574

MQUINAS ELCTRICAS

+
b)

__j=iO

iJ

+
(i){

vo R

el

circuito equivalente para O < 1 < T/2

el
conduce D conduce

circuito equivalente para T/2 < t < T

o;

Figura 7.18.

Rectificador media onda con diodo volante.

circulacin tfreewheeling diode) y su misin es conseguir que la tensin de salida "o del circuito no tenga ningn tramo negativo. El funcionamiento de este circuito es el siguiente: durante el semiciclo positivo de la tensin de alimentacin entra en conduccin el diodo principal D, mientras que el D no conduce porque est polarizado inversamente, dando lugar al circuito equivalente de la Figura 7.18b. Cuando wt = n; la tensin de alimentacin invierte su polaridad, por lo que el diodo D se corta (anulndose la corriente principal i) y la corriente de carga io se cierra por el camino que ofrece el diodo volante DIV' de acuerdo con el circuito equivalente de la Figura 7.18c; de este modo se consigue, como se muestra en la Figura 7.18e, que la tensin en la carga va tenga la misma forma que en el caso del rectificador media onda con carga resistiva (comparar la Fig. 7.18e con la Fig. 7.16), con un valor medio de c.c. expresado por v"c = Vrn/n, tal como se sealaba en (7.3). En la Figura 7 .18f se muestra la forma de onda de la corriente en la carga io' El tramo correspondiente al semi perodo positivo responde, al igual que en el caso anterior, a la ecuacin (7.19b), que es la solucin del circuito equivalente de la Figura 7 .18b, mientras que el tramo de corriente correspondiente al semi perodo negativo de Vo se calcula a partir del circuito equivalente de la Figura 7.18c, ya que el diodo volante cortocircuita la carga. La ecuacin diferencial de este circuito viene expresada por:
di 0= L- + Ri dt (7.21)

Si se redefine el origen de tiempos cuando se inicia este semi perodo, la condicin inicial de la ecuacin anterior es que i(t = O) 1(T/2) /1' que es el valor de la corriente del primer

CAPTULO

7.

ACCIONAMIENTOS ELCTRICOS

575

semiciclo en t = T/2, como se muestra en el esquema de la Figura 7.181 La solucin de (7.21), teniendo en cuenta la condicin inicial anterior, es anloga a (7.18c) y vale: (7.22) que es una seal exponencial que se reduce a cero en t = ta, como se muestra en la Figura 7.18f. Dependiendo de la constante de tiempo, la corriente puede ser continua o no; es decir, si ta es superior a T, la corriente de carga io no se anula nunca (conduccin continua), o si ta es inferior a T, la corriente de carga es discontinua, ya que se anula antes de finalizar el ciclo (conduccin discontinua). En la prctica es deseable una conduccin continua porque da lugar a una corriente de carga con menor rizado. Es fcil comprobar que con grandes inductancias se produce una corriente io de carga prcticamente constante. En la Figura 7.19 se muestran los primeros ciclos de la tensin de alimentacin v" de la tensin en la carga "o y de la corriente en la carga io que se produciran con altas inductancias. La curva inferior de la corriente de carga io se ha determinado a partir de la curva de la Figura 7.181, teniendo en cuenta que si la conduccin es continua, la corriente inicial de cada semiperodo es la final del semiperodo anterior. Obsrvese que al cabo de muy pocos ciclos (se han supuesto tres en el caso de la Fig. 7.19) la corriente alcanza el rgimen permanente, dando lugar a una forma de onda prcticamente plana debido al filtrado que produce la inductancia, de tal modo que los valores medios de la tensin y de la corriente de carga vendrn reflejados por expresiones anlogas a (7.3) y (7.4), es decir:
V=~ ce

v
t:

l. _ Vcc_ Vm ce R - nR

(7.23)

Jt

wt

wt conduce conduce D

Figura 7.19.

Rectificador media onda con carga altamente inductiva.

576

MQUINAS ELCTRICAS

La explicacin matemtica de este efecto de filtrado o alisamiento en la onda de corriente es sencilla. Tngase en cuenta que de acuerdo con (7.2) la tensin Vo de la semionda que aparece en la carga es de la forma: Vm + \1;" sen wt - 2\1;" vo(t) = tt 2 ti ~ L.
,,=2,4.6 -2--

n - 1

cos ruot

(7.24)

que consta de una componente continua Vec = Vn,ln, un armnico fundamental y una serie de armnicos pares. La impedancia que ofrece la carga a los armnicos ser de la forma siguiente: Z

= R + jnLw

=R2

nLw + (nLw)2 L 0" ; 8" = arctg R

(7.25)

donde n es el orden del armnico. Para calcular la expresin de la corriente instantnea en la carga en rgimen permanente io(t) debe tenerse en cuenta que la componente de corriente continua se obtiene a partir de la componente de tensin continua de (7.24) dividiendo por la resistencia R (ya que en c.c. la reactancia es cero), que corresponde al valor sealado en (7.23). Las componentes de la corriente alterna se obtienen dividiendo los trminos de tensin de (7.24) por las respectivas impedancias que se ofrecen a los diversos armnicos expresadas en (7.25) y teniendo en cuenta el desfase que ellas producen en la onda de tensin, lo que da lugar a la expresin final siguiente: V'" io(t) = - + nR 2
2V
_-==n=, ===0

J R2V+ (LW)2sen (wt m

8,) (7.26)

nJR2 + (nLw)2 ,,=2,4,6 n - I

00

1
-2-

cos (nwt - 8n)

Es inmediato comprobar, a partir de la ecuacin anterior, que la inductancia de carga ofrece una impedancia elevada a los armnicos altos. En la prctica se pueden considerar despreciables los armnicos de corriente superiores al fundamental, lo que equivale a decir que en el caso de la expresin (7.26) la corriente de carga se compone de una componente de c.c. y un primer armnico de primer orden o fundamental, y sta es la justificacin de la curva de corriente mostrada en la Figura 7.19, que tiene un pequeo rizado de c.a. Cuando se trata de rectificadores doble onda y trifsicos como los que se vern ms tarde en este captulo, la tensin de salida tiene un rizado muy pequeo, por lo que si la carga es muy inductiva, la corriente correspondiente tiene una forma prcticamente plana, sin apenas rizado, y as se considerar en el anlisis correspondiente. EJEMPLO DE APLICACIN 7.2 En el circuito rectificador media onda de la Figura 7.1710 tensin eficaz de alimentacin es de 100 V. 50 Hz, la resistencia vale 20 ohmios y la inductancia 0,637 Henrios. Calcular: a) expresin de la corriente instantnea de carga; b) tiempo ta de apagado del diodo, es decir, tiempo correspondiente a la anulacin de la corriente de la carga; c) determinar la tensin y corriente media de c.c. en la carga; d) se coloca un diodo volante en paralelo con la carga anterior de acuerdo con el circuito de la Figura 7.18; calcular la expresin instantnea de la corriente de carga en rgimen permanente; e) determinar en la situacin del apartado anterior la corriente media de c.c. en la carga.

CAPTULO 7.

ACCIONAMIENTOS ELCTRICOS

577

SOLUCIN
a)

De acuerdo con la expresin (7.19b), la corriente de carga viene expresada por:


i(t)

Vm { sen (wt =Z

fJ) + sen fJ . e _:} r

y teniendo en cuenta los valores del problema resulta:

Leo = 0,637 . 2n . 50 = 200 ohmios ; Z = ) R2 + (LW)2 = )202 + 2002 fJ


L

201 ohmios

= arctg - = arctg R

Lw

200 20

1,47 radianes ; sen fJ

= sen 1,47 = 0,995

L 0,637 = - = -= 0,0318 segundos R 20

que da lugar a una corriente instantnea:


i(t)

100)2 {sen (wt = ---

= 0,704 [sen (wt


b)

__ - 1,47) + 0,995' e 0.0318 = 201 - 1,47) + 0,995e-31,45t] ; w = Zn> 50 ~ 314 rad/s


t }

Para calcular el tiempo de apagado del diodo debe igualarse a cero la corriente anterior, es decir:
i(t) =

0,704 [sen (314ta - 1,47) + 0,995e-31.45t] = O

La determinacin de ta requiere un proceso de clculo iterativo. Es interesante comenzar con un valor de prueba suficientemente aproximado, que se obtiene despreciando el trmino exponencial de la corriente frente al trmino senoidal, lo que conduce al valor siguiente:
i(t)

=O

=>

sen (314ta - 1,47) = O


=>

=>

314ta - 1,47 =

tt

=>

4,61 ta = -- = 0,0147 segundos 314

e)

El lector puede demostrar que el valor exacto del tiempo de apagado es = wta = 3140,0167 = 5,244 radianes. La tensin media viene expresada por la ecuacin (7.20), lo que da lugar a un valor:
t, = 0,0167 segundos,que corresponde a un argumento ljJ

Vee = - [1 - cos 1jJ] = -2n 2n

Vm

100)2

(1 - cos 5,244) ~ 11,1 voltios

que corresponde a una corriente media en la carga: Ice = Vee 11,1 = -~ 0,555 amperios 20

578

MQUINAS ELCTRICAS

d)

En este caso la corriente de rgimen permanente en la carga viene expresada por (7.26):
lO

'() t

= -v'n +
nR

J v'n sen (()) wt 2 R2 + (LW)2

--===' ==

2V" ~ 1 L... -2-- cos (nwt nJR2 + (nLw)2 n=2.4.6 n - l

()n)

y al sustituir valores y limitando el desarrollo al armnico de orden 4 resulta:


o(!) = 100)2 + 100)2 sen 20n 2J202 + 2002

((1)t _ actg 200) _


20

nJ202

2 100)2 + 4002
20

. -- 1 cos ( Lcot - arctg 400) - 2 . 100)2 -- 1 22-1 20 nJ202+800242-1 que correponde numricamente a:

800) cos ( 4wt - arctg -

o(t) = 2,25 + 0,35 sen (314t - 1,47) - 0,075 cos (628t - 1,52)-

- 0,0075 cos (1.256t - 1,55) donde los argumentos estn expresados en radianes. La expresin anterior es aceptable aproximarla a:
(t) =

2,25 +0,35 sen (314t - 1,47)

e)

que corresponde a un valor de c.c. y a un primer armnico causante del rizado de la seal de salida. Obsrvese que el rizado es pequeo debido a que la inductancia es elevada, por lo que prcticamente la corriente es constante e igual a I,c' lo que justifica la forma de onda explicada en la Figura 7.19. El valor medio de la c.c. en la carga corresponde al primer sumando de la expresin anterior, es decir: Icc = 2,25 amperios.

7.4.3. Rectificador monofsico de doble onda (onda completa) con carga resistiva
Existen dos tipos de circuito que rectifican la onda completa de la red. Uno de ellos emplea un transformador con toma central y el otro es un puente rectificador o puente de Graetz. Veamos el funcionamiento de cada uno de ellos. a)

Rectificador con transformador con toma central

Este tipo de rectificador emplea un transformador cuyo secundario tiene una conexin central que es el punto comn de retomo del circuito rectificador. En la Figura 7.20 se muestra el esquema del circuito, que utiliza dos diodos conectados a los extremos del secundario del transformador. Las tensiones secundarias se miden desde el centro b, es decir, de b a a y desde b a c. Para evitar confusiones, la tensin del devanado secundario destinado a este rectificador se especifica, por ejemplo: 25-0-25 V; esto significa que de b a a se tienen 25 V eficaces y tambin desde b a e; entre e y a hay 50 V eficaces. Durante el semiciclo positivo de la tensin de red el terminal a es ms positivo que b y a su vez ste es ms positivo que e, lo que se representa en el esquema con las polaridades indicadas. En este caso el diodo DI queda polarizado directamente y el diodo D2 est con polarizacin inversa; en consecuencia, el

CAPTULO

7.

ACCJONAMIENTOS ELCTRICOS

579

~ ~
...

I ciclo

= T seg = 2 rr rad.

3rr
d

Transformador

D2 R

+
vR

I ciclo = Tseg = 2 t: rad.

Vs

= v

," ,
I I \

/ .... ,y'
\ \

Figura 7.20. Rectificador monofsico doble onda con transformador de toma central.

diodo DI conduce y el D2 queda bloqueado, y de esta manera se produce una corriente en la malla formada por adeb y la tensin v, de la mitad superior del transformador aparece en la carga R (en el supuesto de considerar el diodo ideal). La tensin inversa a que est sometido el diodo D2 es la d.d.p. en sus bornes, entre e y d, es decir, entre e y a al estar conduciendo DI y suponerlo ideal. En consecuencia, si el valor mximo de la tensin de cada semidevanado es Vm, la tensin inversa de pico del diodo D2 (VIP) es igual a 2Vm Durante el semiciclo negativo se invierten los papeles representados por DI y D2. En este caso e ser ms positivo que b y este terminal ser a su vez ms positivo que a; por tanto, el diodo D2 conducir y el DI estar bloqueado. La corriente que va a la carga se cerrar por la malla cdeb, lo que indica que el sentido de la corriente en R no ha cambiado. Ambas mitades de la onda de c.a. se han rectificado y dan lugar a una tensin en la carga tal como se indica en la Figura 7.20. El desarrollo de Fourier de esta tensin rectificada doble onda vR es de la forma: 2'-':" - 4'-':" ~ vR = L...
t:
ti
11=2.4.6

-2--

n - 1

COS nwt

(7.27)

que si se compara con la expresin (7.2) de la rectificacin media onda, se observa que el trmino de c.c. es ahora doble que en el caso anterior, es decir:

v',c = -

2Vn
ti

= 0,637 ,-,:"

(7.28)

Por otra parte, se observa en (7.27) que ahora no existe trmino fundamental a la frecuencia de la lnea, siendo el segundo armnico el contenido en la frecuencia ms baja; por consiguiente, la rectificacin doble onda tendr menor rizado (es decir, menos componente alterna) que la media onda. La corriente continua que atravesar la carga ser:

1 = V,c=2V,,,
ce

nR

(7.29)

580

MQUINAS ELCTRICAS

El valor eficaz de la tensin doble onda ser doble que en el caso de la media onda y su magnitud se puede obtener aplicando la definicin de valor eficaz, resultando: Ver=

J~r

v\t) dt =

-2
2n

in v;,
o

serr'

e de =

v' 2

V ~

(7.30)

De una forma anloga a (2.9), la tensin de rizado ser:

v, "

J v~- V>
r

J(fi)' - c:-)'
0,308Vm

"0,308 V.

(7.31)

y por tanto el rizado valdr:

= -v, =

v,

2Vm/n

0,484

(7.32)

Como quiera que la corriente eficaz vale: (7.33) la eficacia o rendimiento de la rectificacin expresada en (7.13) tiene ahora un valor:
IJ = r P,

r;

= --

VeJcc
Vef

(2Vm/n) . (2Vm/nR)
(Vm/j2) . (Vm/j2R)

=- =
n2

811

(7.34)

'

El clculo del factor de utilizacin del transformador para un rectificador doble onda se realiza teniendo en cuenta que, desde el punto de vista de cada semidevanado secundario, el grupo se comporta como dos rectificadores media onda y por tanto el factor ser doble que el de media onda expresado en (7.16), es decir, 0,574; sin embargo, la aplicacin directa de (7.16) conduce a un valor de 0,811, que expresa el factor de utilizacin del transformador en bloque. En el diseo elctrico del transformador, el factor 0,574 se considera que slo afecta al secundario, mientras que el 0,811 se refiere al primario, lo que conduce a un valor medio de 0,693. Esto indica que un transformador de 1 kVA puede entregar 693 W de corriente continua a una carga resistiva. De todas las expresiones anteriores se deduce que la rectificacin doble onda reduce el rizado de la tensin de salida y mejora la utilizacin del transformador. Adems, como cada devanado secundario funciona solamente medio ciclo, circulando intensidades continuas de sentido opuesto, el flujo neto producido por Ice en el secundario se cancela, por lo que no hay problemas de saturaciones en el ncleo del transformador. b)

Rectificador en puente de Graetz

Es otra variante del rectificador doble onda. En este caso el circuito (Fig. 7.21) necesita cuatro diodos pero el transformador no necesita toma central, y adems la tensin secundaria total es V(eficaz) o Vm su valor mximo, por lo que la tensin inversa de pico de cada diodo es slo V,,, en vez de 2 v'n que se obtena en el caso anterior. Durante el semiciclo positivo de la tensin alterna del secundario del transformador slo conducen los diodos DI y D3, que son los que estn polarizados directamente (vase Fig. 7.21), Yla corriente circula a travs de la carga R de e a d. Durante el semiciclo negativo, conducen los diodos D2 y D4 Y el sentido de la corriente en R sigue siendo el mismo. En cada uno de los casos la tensin del transformador se

CAPTULO 7.

ACCIONAMIENTOS ELCTRICOS

581

Transformador

a Semiciclos positivos

El
~
Primario
~ ~a

D3 b

Semiciclos negativos

Tensin y corriente en la carga

Figura 7.21. Rectificador monofsico doble onda en puente.

refleja en la carga pero invirtiendo el semiciclo negativo. Se obtiene, por tanto, en la carga una seal doble onda cuyo valor de continua ser igual que (7.28), es decir:
Vee = 2Vm = 0,637Vm = 0,637Vt: 2V = 0,9 V
11:

(7.35)

siendo V el valor eficaz de la tensin del transformador. El factor de rizado ser 0,484 y el rendimiento de la rectificacin valdr 0,811. Sin embargo, en este circuito la corriente y la tensin del secundario son alternos puros sin ninguna consideracin para el flujo de c.c. (no hay efecto de saturacin), de tal forma que el factor de utilizacin es 0,811 (no hace falta considerar dos factores). Si se tiene el ejemplo de un transformador de 1kVA, quiere decir que se entregan a la carga 811 W, y por ello este tipo de rectificador puente es el mejor circuito desde el punto de vista de la rectificacin completa, aunque tiene la desventaja de necesitar cuatro diodos en vez de dos. En la Tabla 7.1 se resumen las propiedades de los rectificadores monofsicos.
Tabla 7.1. Relaciones fundamentales en los trasformadores monofsicos

Tensin de lnea (eficaz) N." de diodos Tensin inversa de pico Tensin continua Ve, Rizado (r) Factor de utilizacin del trafo Frecuencia del armnico principal

V 1

V(cada mitad)
2 2Vm 0,637V,,, 0,484 0,693

V 4 0,637Vm 0,484 0,811 2f

V,,,
0,318V,,, 1,211 0,287 f

V:"

2f

582

MQUINAS ELCTRICAS

EJEMPLO

DE APLICACIN

7.3

Un transformador con tensiones eficaces de l27 Ven el devanado primario y 250 Ven el devanado secundario alimenta un puente rectificador que tiene conectada una resistencia de carga de lOOQ. Determinar: a) la tensin y corriente continua en la carga; b) la corriente y potencia en el primario del transformador si se considera ste ideal. SOLUCIN
a)

El valor de la tensin mxima en el secundario es: V,,, = la tensin v'c' de acuerdo con (7.35), ser:

fi 250 = 353,55 voltios, y

v'c = 0,637
1

V,,,

= 0,637 . 353,55 = 225,21 voltios


V"

por lo que la corriente continua en la carga ser:

= 225,21 = 2 25 A
100 '

b)

y esta carga desarrolla una potencia de valor ~( = v'Jc = 225,21 2,25 La intensidad eficaz en el secundario del transformador es:
1=

= 507 W.

-R = -100'= 2 5 A = lo-

Vej

250

y teniendo en cuenta que la relacin de transformacin es m = 127/250 = 0,508, la corriente 1] primaria ser:
1]

= -::z = 0,508 = 4,92 A

lo

2,5

dando una potencia: S =

v] 1]

= 127 . 4,92 ;:::; 625 VA.

7.4.4. Rectificador monofsico de doble onda (onda completa) con carga inductiva
En la Figura 7.22 se muestra un rectificador de doble onda que es idntico al de la Figura 7.21, pero donde el puente de diodos se ha dibujado con otra topologa. En este caso, la carga es de tipo inductivo y se considera que L tiene un valor elevado (tericamente inifinito); de este modo la corriente de carga tendr un rizado despreciable y se podr suponer que su forma es totalmente plana. Con esta suposicin, las curvas de comportamiento del circuito son las mostradas en la parte derecha de la Figura 7.22. La tensin en la carga es una doble onda rectificada, al igual que en el caso anterior, pero al ser la carga altamente inductiva, la corriente io es constante e igual a (.(.,de tal modo que los dos diodos que conducen en cada semiciclo llevan una corriente constante. La corriente en el secundario del transformador es alterna y de forma rectangular, con una amplitud en cada serniciclo de valor icc. De acuerdo con las expresiones (7.28) y (7.29), los valores de la tensin media de c.c. y de la corriente media de c.c. vendrn expresadas por:
V
ce

2V
=_______!!!_

(7.36)

CAPTULO

7.

ACCIONAMIENTOS ELCTRICOS

583

b)

Transformador

llio D4
L + wt e)

~
Re~

R Primario

va

io
tt

D3 2n: 3n
wt d)

O
t,

al n
O

2n'

r--p:
Icc:

'3n

4n:

wt

: el

Figura 7.22.

Rectificador monofsico en puente con carga inductiva.

EJEMPLO DE APLICACIN 7.4 El circuitorectificadorde la Figura 7.22 tiene los siguientesparmetros: Vs = 220 sen cot; f = 50 Hz, R = 100 Q; L = 1 H. a) Calcular la expresin instantnea de la corriente de carga ioCt); b) Desarrollar en serie de Fourier la corriente iit) en el lado de corriente alterna; c) Calcular el valor eficaz de la corriente del armnico fundamental lis Y tambin el valor eficaz de la corriente Is' SOLUCIN
a)

j2 .

De acuerdo con (7.27), la onda rectificada tiene el siguiente desarrollo en serie de Fourier:
va(l) = -

2Vm
tt

- -

4V,,, ~ n

-2--

,,=2.4.6

n - 1

cos non

La impedancia compleja de la carga para el armnico de orden n es de la forma:


Z" = R + j(nLm) = Z" L
C{Jn

Por consiguiente, la corriente io(l)de la carga, teniendo en cuenta la expresin de la tensin vo(t), ser:
lo(l)= -

2V,,, 4Vm ~ 1 - L. -2-- cos (non nR nZ" n=2.4.6 n - 1

C{J,,)

donde se ha tenido en cuenta que la componente de c.c. de la corriente es el cociente de la componente de c.c. de la tensin dividido por la resistencia y las componentes de c.a. de la corriente son el cociente de las componentes de c.a. de la tensin divididas por las impedancias correspondientes.

584

MQUINAS ELCTRICAS

Como quiera que los valores de las impedancias complejas de los tres primeros armnicos son: Z2

= R + j(2Lw) = Z4 = R + j(4Lw) =

100 + j628 lOO+ j

= 635,9 L 80,95 1.256 = 1.260 L 85,45

Z6 = R + j(6Lw) = 100 + j 1.884 = 1.886,7 L 86,96


donde se ha tenido en cuenta que w = 2nf ~ 314 rad/s. La expresin de la corriente de carga, teniendo en cuenta que V:n = 220 = 311,1 voltios, ser:

j2 .

ioCt)=

2311,1 4311,1 [1 1 -. -2- cos (2wt - 80,95) + tt : 100 tt 635,9 2 - 1 + _1_. _1_ cos (4wt - 8545) + ... 1.260 42 - 1 '

es decir:
io(t) = 1,98 - 0,208 cos (2wt - 80,95) - 0,021 cos (4wt - 85,45) -

- 0,006 cos (6wt - 86,96) + ... expresin que es aceptable aproximar a:


o(t) ~ 1,98 = (, b)

que confirma la forma de onda de la Figura 7.22d. Si se parte de la onda rectangular de la Figura 7.22e, en la que (c = 1,98 A, el desarrollo en serie de Fourier es de la forma:
i (r) = s

41('(' ( sen wt + 1 sen 3wt + -1 sen 5wt + 1 sen 7wt + ... ) t: 3 5 7

e)

El armnico fundamental de la seal anterior tiene una valor mximo 41cJn, que corresponde a un valor eficaz: 1,

1 41ee 1 4 1,98 . ::; - = ::; --= 1,78 ampenos y2 tt y2 t:

y el valor eficaz de la corriente rectangular de la Figura 7.22e se calcula aplicando la definicin de valor eficaz de una onda peridica, lo que conduce a: 1 s

f2n 1:'. de = lec = 1,98 amperios = _!_ 2n Jo

El lector puede comprobar este resultado a partir de los valores eficaces de los armnicos incluidos en la corriente is(t), es decir: 1 = JI2+/2+12+/2+
s
J

...
7

41 [ 1 + =-'_.

nj2

(1)2 + (1)2 - + (1)2 ~ + ... J"2 3 5 7


- 1cc

_~[nj2J- nj2 4

CAPTULO

7.

ACCIONAMIENTOS ELCTRICOS

585

7.4.5. Rectificadores trifsicos


La conversin de grandes potencias de alterna a continua lleva consigo un funcionamiento con circuitos de tres fases, ya que estos sistemas son los que se emplean en la generacin y distribucin de energa elctrica. Del mismo modo que un rectificador monofsico de doble onda produce dos impulsos de corriente por ciclo, un circuito trifsico media onda producir tres, un doble onda seis, etc. El factor de rizado se reducir y al incrementarse el nmero de fases las frecuencias de rizado de las componentes armnicas se vern aumentadas, lo cual lleva consigo una facilidad en el filtrado de la corriente mayor que en el caso de un rectificador monofsico. En la rectificacin de potencia se utilizan circuitos hexafsicos y dodecafsicos que se obtienen por acoplamientos especiales entre transformadores trifsicos; la idea es mejorar in situ la forma de onda de la salida para no emplear filtros posteriores, que en el caso de grandes intensidades encareceran el coste de los equipos. En este epgrafe se estudiar primeramente la rectificacin trifsica de media onda y doble onda (3 impulsos y 6 impulsos por ciclo, respectivamente), para finalizar con un rectificador especial de 12 impulsos. Los transformadores empleados para los dos primeros montajes son conexin Dy, es decir, tringulo-estrella; esto es debido a que en cada momento slo conduce un diodo rectificador, lo que provoca, por ejemplo, en el caso del circuito semi onda un desequilibrio fase-neutro que en una conexin Yy originara flujos alternos homopolares que saltaran por el aire y el neutro de la estrella se desplazara y producira un desequilibrio en las tensiones simples. Algunos proyectistas utilizan tambin la conexin zig-zag en secundario, que evita tambin estos problemas. En nuestro caso se elegir la conexin Dy, que es la disposicin ms frecuente en la rectificacin trifsica. En el caso de 12 impulsos se emplear un transformador con dos secundarios y conexin Dyd.

a) Rectificacin trifsica media onda


Considrese el esquema de la Figura 7.23a, donde un transformador Dy con la relacin adecuada alimenta el grupo rectificador carga. Los diodos rectificadores se colocan en cada fase y la carga se conecta entre el punto comn de los diodos (ctodos) y el neutro de la estrella. En este circuito cada diodo conducir siempre que su nodo sea ms positivo que su ctodo (es decir, est polarizado directamente). En la Figura 7.23b se indica, a la izquierda, las tensiones simples instantneas (fase-neutro) del secundario, que son ondas senoidales desfasadas 2n/3 radianes (120) y de valor mximo V:". En el instante to, correspondiente a la posicin representada por el punto A, la tensin mayor es VRN, el diodo correspondiente a esa fase Di quedar polarizado directamente y conducir hasta el instante tI' representado por el punto B, en el que la tensin V5N = VRN; inmediatamente despus, VSN> VRN, y el diodo DI deja de conducir al quedar polarizado inversamente y conduce el D2 hasta el punto indicado por C, en donde es sustituido por D3, y as sucesivamente. En la Figura 7.23c se indica con trazo grueso la forma de onda de la seal que se obtiene en bornes de la resistencia R. Se observa que cada diodo conduce durante 1/3 de ciclo, es decir, durante un perodo de 2n/3 radianes. Al igual que se ha hecho en los casos anteriores, se puede obtener el desarrollo en serie de Fourier de esta onda para ver sus componentes, resultando:

VR

3)3v:"
2n

(1 -

L
,,=3,6,9.

-2--

n - 1

nti cos cos non ) 3

(7.37)

586

MQUINAS ELCTRICAS

TRANSFORMADOR

DI

+ Primario Secundario
a)

s'o---------------------~

b)

Figura 7.23.

Rectificador trifsico media onda.

cuya componente de tensin continua es:


(7.38)

Se observa tambin en (7.37) que el primer armnico que aparece es de orden 3, y le siguen los mltiplos de tres pero con amplitudes mucho menores. El valor eficaz de la tensin total de salida en la carga se obtiene aplicando su definicin, y resulta:
-1

I
o

v2(t) dt ::

-2n/3

1 fsn/6
n/6

V2 serr' O dO :: 0,841 V ni m

(7.39)

donde debido a la simetra slo se ha integrado la porcin de ciclo correspondiente al tramo AB (conduccin del diodo DI). La tensin de rizado, de forma anloga a (2.9), resulta ser:
(7.40)

En consecuencia, el rizado valdr:


r:: -::

Vr Vcc

0,151 V:n :: 0183 0,827V:,. ,

(7.41)

Comparando este valor con el que se obtuvo en (7.32) para el rectificador monofsico tipo doble onda, que era de 0,484, se observa una gran reduccin en el rizado y en consecuencia la tensin de salida es de mejor calidad. Respecto al clculo de las tensiones inversas de pico

CAPTULO

7.

ACCIONAMIENTOS ELCTRICOS

587

(PIV) que deben soportar los diodos, se observa en la Figura 7.23 que cuando un diodo no conduce queda aplicada en sus bornes la tensin compuesta del secundario del transformador, lo que supone un valor mximo de

,J3,-,:", siendo ,-,:" el valor de pico de la tensin de una fase.

b) Rectificador trifsico doble onda (puente)


Este puente rectificador trifsico es muy clsico en aplicaciones en alta potencia y responde al esquema de circuito mostrado en la Figura 7.24a. Utiliza seis diodos que conducen dos a dos. Estos diodos se han numerado en la Figura 7.24a de acuerdo con la secuencia de conduccin

TRANSFORMADOR

=-

----+

'

DI M .....

,, , , ,:D3

---

--------, . ,
D5

'
+

a)

Primario

Secundario

D4

D6

-,, ,, ,, ,, ,,: D2
l

________ 12

, ,, ,,, ,, J

Diodos Tensiones compuestas

il

56
I']S

61
IRs

12
\'n

23

34

45

56

61
"RS

vsr

"s

vr

vn

vns

J3\;"
17)

lec

O
i4

rrJ3

7rrJ3

llw'3

WI

e)

4n/3

5n/3

lec

2n

WI

d)

iR
O

w'3

2rr13

4n/3

5n/3

2n

7n/3

3n

11n/3

i:

WI

e)

t.; Figura 7.24. Rectificador trifsico doble onda.

588

MQUINAS ELCTRICAS

de los mismos y cada uno de ellos funciona durante 120. La secuencia de conduccin es 12, 23, 34, 45, 56 y 61. Los dos diodos que conducen en cada momento son los que tienen aplicada la tensin instantnea compuesta o de lnea (fase a fase) ms elevada. La tensin de lnea es veces la tensin de fase del secundario del transformador conectado en estrella. En la Figura 7.24b se han dibujado las tensiones instantneas de lnea bsicas vRS' VST' vTR Ysus opuestas VSR' vTS YVRT (obsrvese que sus valores de pico son Vm, siendo V", la tensin mxima de una fase del secundario del transformador). Supngase una situacin inicial en la que estn conduciendo los diodos D5 y D6; al llegar al instante to, correspondiente al punto A, la tensin compuesta mayor del puente es la vRS' por lo que DI comienza a conducir por tener polarizacin positiva, el diodo D6 sigue conduciendo y D5 deja de conducir por quedar polarizado inversamente. Es decir, a partir de to conducen D6 y DI Yla corriente sigue el camino sealado por la lnea de trazo discontinuo mostrada en la Figura 7.24a, por lo que la tensin en bornes de la resistencia de carga es la d.d.p. en bornes de los diodos D6 y DI, es decir, VRS. Esta situacin contina durante 60, hasta el instante ti (punto B), en el que se hace mayor la tensin VRT' por lo que el diodo D6 queda sometido a polarizacin inversa y pasa a conducir el diodo D2; es decir, a partir de ti conducen los diodos DI y D2 y lo hacen hasta el instante t2 (punto C), en el que conducen D2 y D3, y as sucesivamente; de este modo el diodo D2 ha conducido desde el punto A hasta el punto e (es decir, durante 120). En resumen, la tensin v R en la carga resistiva est formada por tramos de 60 de las respectivas tensiones de lnea que son ms elevadas en cada momento. De este modo, en un ciclo completo de 360 existen 6 tramos de onda, y es por lo que este rectificador recibe el nombre de 6 impulsos (debe destacarse, sin embargo, que cada diodo conduce durante 120). Los tiempos correspondientes a los puntos A, B, e, etc., se denominan instantes de conmutacin. En la Figura 7.24b se muestra en trazo grueso la onda de tensin en la carga resistiva, y cuyo desarrollo en serie de Fourier nos da:

fi

fi

vR

_ 3(fiv,,,) ( 1n

-2-11=6.12,18,

2 cos nn cos ncot ) n -1 6

(7.42)

que indica que el valor de componente continua de la tensin de salida es:


(7.43)

El primer armnico que aparece es de orden 6 y le siguen sus mltiplos pero con una amplitud muy reducida. El valor eficaz de la tensin total de salida en la carga se obtiene aplicando su definicin, y resulta: 1

f2n/3
-n/3

(fiVm

sen e)2 de = 1,6554Vm

(7.44)

nl3

donde debido a la simetra slo se ha integrado la porcin de ciclo correspondiente al tramo AB (conduccin de los diodos 6 y 1), La tensin de rizado, de forma anloga a (2.9), resulta ser:
(7.45)

CAPTULO 7.

ACCIONAMIENTOS ELCTRICOS

589

En consecuencia, el rizado valdr:

r =

Vr = v'e

0,063 v'n 0,038 1,6542v'71

(7.46)

que es un valor muy reducido, lo que indica que la salida es una seal prcticamente de c.c. Para ver la tensin inversa de pico a que se ven sometidos los diodos, se observa en la Figura 7.24a que cuando un diodo no conduce, la tensin existente entre sus terminales corresponde al valor de pico de la tensin compuesta, es decir, m, siendo Vm el valor mximo de la tensin de una fase. En las Figura 7.24c y d se han dibujado tambin las ondas de corriente en los diodos DI Y D4 que coinciden con la forma de onda de la corriente en la carga, es decir, de amplitud lec Yque tienen una duracin de 120. En la Figura 7.24e se ha representado tambin la corriente en la fase R del secundario del transformador y que se obtienepor la diferencia iR(t) = i,(t) - i4(t), que resulta de aplicar el primer lema de Kirchhoff en el nudo M de la Figura 7.24a. En las instalaciones de traccin elctrica se requiere una c.c. que tenga muy poco rizado, ya que se traduce en un menor coste de los filtros que alisan la c.c. de salida. Para ello se construyenrectificadores con ms de 6 impulsos por ciclo. En el esquema de la Figura 7.25 se muestra un rectificador tpico de 12 impulsos; est formado por un transformador con un devanado primario en tringulo y dos secundarios, uno en estrella y el otro en tringulo, es decir, conexin Dyd (y que debe estar diseado con las mismas tensiones de lnea secundarias). De este modo existe una diferencia de fases de 30 en las tensiones de salida. Como quieraque cada grupo rectificador produce una seal similar a la indicada en la Figura 7.24b, al sumar las tensiones secundarias obtenidas por cada uno de ellos, segn el esquema de la Figura7.25, resulta una c.c. de 12 impulsos por ciclo, lo que indica que los nicos armnicos queresultarn son el 12,24, 36, etc., ya que los armnicos de orden 6 se anulan al sumarse las salidasde ambos rectificadores porque estn en oposicin de fase (tngase en cuenta que un desfasede 30 para la seal de frecuencia fundamental se convierte en 6 . 30 = 180 para la de sexto armnico). Con este tipo de montaje se reducen tambin los armnicos de las corrientes en el lado primario del transformador, lo que se traduce en una menor penetracin de armnicos en la red. El valor de la tensin media de c.c. ser el doble que la de un rectificador trifsico en puente, es decir, Vee = 3,31 Vm' siendo Vm la tensin de pico de cada fase secundaria. Todos los tipos de rectificadores trifsicos estudiados hasta aqu, a saber: media onda o de 3 impulsos, doble onda o de 6 impulsos y dodecafsico o de 12 impulsos, tienen de por s muy poco rizado, lo que indica que la componente alterna es muy reducida. Las formas de onda de las corrientes de carga, cuando alimentan resistencias, son idnticas a las ondas de tensin que se han dibujado en las Figuras 7.23, 7.24 Y7.25. Cuando adems alimentan cargas muy inductivas, se obtiene un aplanamiento adicional en las ondas de corriente en virtud del filtrado que producen las inductancias (como ya se demostr en los epgrafes 7.4.2 y 7.4.4), por lo que las ondas de corriente, tanto en la carga como en los diodos, son prcticamente planas y coinciden con el valor medio de la corriente continua. Las corrientes en el secundario (o secundarios) de los transformadores tienen formas escalonadas,que se acercan mucho a la forma senoidal, lo que se traduce en un menor impacto de armnicos en la red, y de ah que para grandes potencias se impongan los rectificadores trifsicos frente a los monofsicos.

J'3v

590

MQUINAS ELCTRICAS

R'

S'

T'

Primario

Secundario 1

TRANSFORMADOR + R

Secundario 2

Figura 7.25.

Rectificador dodecafsico.

EJEMPLO DE APLICACIN 7.5 En la Figura 7.26 se muestra la onda producida por un rectificador de qfases. Es una onda peridica de perodo Zn/q. Calcular la expresin del desarrollo en serie de Fourier de la onda. Particularizarpara un rectificador trifsico media onda (q = 3) y de doble onda (q = 6).

Figura 7.26.

Onda producida por un rectificador de q fases.

CAPTULO

7.

ACCIONAMIENTOS ELCTRICOS

591

SOLUCIN
El desarrollo en serie de Fourier de la onda sealada en la figura viene expresado por:

a
v(t) =
__J!

La
I

ll

cos non +

L b; sen non
I

oc

donde los valores de los coeficientes [teniendo en cuenta que segn la Figura 7.26 la tensin es de la forma v(t) = Vm cos cot y que al hacer el cambio cot = (J se convierte en v(J) = Vm COS (J] se calculan as:

= -2 f+T12 T -T12 2 f+T/2 an = ao T


-T/2

Vm cos cot dt

Vm

COS iot

= - l fXlq Vm COS (J d(J niq -atlq 1 fnlq cos non dt = V:1l COS (J cos niq -nlq

n(J d(J

2 b, = T

f+Tl2 -T12

1 Vm COS cu sen non dt = ttlq

f -nlq

V:1l COS (J sen

nf) d(J

La tensin v((J) viene expresada, de acuerdo con la Figura 7.26, por: v(J) = Vm cos (J. Al sustituir este valor en las integrales anteriores y teniendo en cuenta que sen niq = - sen (-ni q), se obtiene para el coeficiente ao: ao = ttlq

f"lq
-atlq

2qVm t: Vm COS (J d(J = -sen t: q

El valor del coeficiente all nos da:


an =

-1 nlq

fnlq

-rrlq

Vm

COS

(J cos n(J d(J = _____!'!_ qV 2n

-tuq
n

[cos (n + 1) (J + cos (n - 1) (J] d(J =

= qV:n [sen (n + 1) (J + sen (n - 1) (J]"'''


2n n + 1 n - 1
=nl q

= qVm [sen (n + 1) nlq + sen (n - 1) nlq]


n+ 1 n - 1

donde se ha tenido en cuenta que sen n/q = -sen (-nlq). obtiene:

Operando la expresin anterior se

a =ti

qVm 2n

(n - 1) sen (n

+ 1) nlq + (n + 1) sen (nn2 - I

1) nlq

-2qV:" n(n2
-

1)

ntt n cos - sen q q

Se ha tenido en cuenta que para n

3q, etc., se cumple sen q De un modo anlogo, los valores de los coeficientes bn sern: btl = - 1

= O, q, 2q,

nn

= O.

frrlq
-nlq

Vm

COS

nlq

qV (J sen n(J d(J = _____!'!_ 2n


[COS

fnlq
-nlq

[sen (n + 1) (J + sen (n - 1) (J] d(J =

= qV", 2n

(n + 1) (J + cos (n -1) (J]nlq = O n+1 n- 1

-nlq

592

MQUINAS ELCTRICAS

donde se ha tenido en cuenta que cos nlq = cos (-n/q). Por consiguiente, el desarrollo en serie de Fourier de la onda de la Figura 7.26 vendr expresado por:
v(t) = -

qVm

n 3Vm

sen - q

ti

;, L,
n=q.2q,3q, ...

2qVm
2

tdn - 1) 6Vm
2

nn n cos - sen - cos ncot


q q

Para el caso del rectificador trifsico media onda (q = 3) nos dar:


v(t) = -

sen - n 3

tt

n=3,6,9, ... n=3~9,

ce

ntt
COS -

n(n - 1)

n sen - cos non = 3 3

= 3~Vm

(1 -

n2 ~ 1 cos ~n cos nwt)

expresin que coincide con la sealada en (7.37). Para el caso del rectificador trifsico doble onda (q = 6) el desarrollo en serie de Fourier nos da:
v(t)

6V' n = ____!!!. sen - tt 6


= 3V~
tt

00

12V~1
2

n=6,12,18,.

n(n - 1)

ntt n cos - sen - cos non = 6 6

(1 _ I
n=6,12,18, ...

_2_ cos nn cos nwt) n2 - 1 6

expresin que coincide con la (7.31), en la que V~ representa ahora la tensin compuesta de pico, que es igual a veces la tensin de pico de una fase, lo que da lugar a:

J3
n

v(t) = ---

6J3Vm

sen - 6

tt

n=6,12,18, ... n=6,~18,

12J3vm
2

nin - 1)

cos - sen - cos non = 6 6

ntt

ti

= 3;;Vm

(1 -

n2 ~ l cos :n cos nwt)

que coincide con la (7.42).

7.5. RECTIFICADORES CONTROLADOS 7.5.1. Introduccin


En el epgrafe 7.4 se han estudiado los diversos tipos de circuitos rectificadores monofsicos y trifsicos y con cargas de tipos resistivo e inductivo. Estos circuitos convierten una tensin de c.a. en una tensin de c.c. de valor medio constante. Si los diodos rectificadores de estos circuitos se sustituyen por tiristores, se obtienen tensiones de c.c. cuyo valor medio se puede controlar. Recurdese, segn se analiz en el epgrafe 7.2.3, que un tiristor tiene tres terminales o electrodos: nodo, ctodo y puerta; bsicamente un tiristor se comporta como un diodo, es decir, bloquea las tensiones inversas (ctodo positivo respecto al nodo) y puede conducir con tensiones directas, pero solamente a partir del instante en que se aplica una corriente positiva entre el terminal de puerta y el ctodo. Esta ligera diferencia con los diodos tiene una profunda importancia prctica, no solamente por disponer de una rectificacin controlada para transformar la c.a. en C.C., sino tambin porque puede convertir la c.c. en c.a., trabajando

CAPTULO 7.

ACCIONAMIENTOS ELCTRICOS

593

en este caso como inversor (denominado tambin ondulador), y es por ello que reciben el nombregenrico de convertidores con control de fase. Sabemos que el tiristor deja de conducir,es decir, se apaga, cuando se anula la corriente principal nodo-ctodo. Si la red es de c.a. esteapagadose hace de forma natural al final de cada ciclo y se denomina por ello conmutacin naturalo conmutacinpor lnea, pero en algunos casos (por ejemplo, si se desea interrumpir la corrienteen un instante especfico) se debe obligar artificialmente a que la corriente del tiristor valgacero, lo que requiere realizar una conmutacinforzada, y que se consigue en la prctica cargandoun condensador en un circuito auxiliar y descargndolo en el tiristor en el sentido ctodo-nodo,es decir, contrario al sentido de la corriente principal. En el caso de utilizar tiristorestipo GTO, el apagado se logra aplicando a la puerta una corriente negativa. Los convertidores con control de fase que se incluyen en este epgrafe pueden ser monofsicosy trifsicos y cada uno de ellos se subdivide en funcin de la polaridad de la tensin va y la corriente ia que suministran a la carga, y as se tienen: convertidores de un cuadrante, de doscuadrantes y de cuatro cuadrantes, como se muestra en la Figura 7.27. Los convertidores de un cuadrante o semiconvertidores funcionan solamente como rectificadores controlados, es decir, la polaridad de la tensin va es fija y el sentido de la corriente iaes nico y no pueden trabajarcomo inversores conmutados por lnea; trabajan como rectificadores en los cuadrantes 1 y III, tal como se seala en la Figura 7.27a.

+
JI

Tensin de c.a.

* f

Tensin dec.c. (polaridad nica) III

io
a)

+
IV

va

Tensin de C.a.

Tensin de c.c. (polaridad doble)

io
b)

+
IV

JI Tensin de c.a.

Tensin dec.c. (polaridad doble de V e 1) 111

io e)

+
IV

Figura 7.27.

Tipos

de convertidores

controlados

594

MQUINAS ELCTRICAS

Los convertidores de dos cuadrantes o convertidores completos tienen una polaridad de la tensin que puede ser positiva o negativa, pero el sentido de la corriente es nico; trabajan como rectificadores en el cuadrante I y como inversores conmutados por lnea en el cuadrante IV (Fig. 7.27b). Los convertidores de cuatro cuadrantes son en realidad dos convertidores acoplados de tal forma que funcionan como rectificadores en los cuadrantes I y III Y como inversores en los cuadrantes 11y IV; se denominan convertidores duales o dobles y es la versin en electrnica de potencia de los clsicos grupos rotativos Ward-Leonard. Estos convertidores con control de fase se utilizan sobre todo en la regulacin de velocidad de motores de c.c. en industrias metalrgicas y papeleras (trenes de laminacin), en obras civiles (plantas para dosificacin automtica de ridos), en la industria alimentaria (mquinas mezcladoras y dosificadoras), etc. Suelen incluir tcnicas de control en lazo cerrado (feedback) para ajustar pares motores, rampas de aceleracin y deceleracin e incluso posicionado de ejes en mquinas herramientas y otro tipo de mecanismos.

7.5.2. Principio de funcionamiento de un rectificador con control de fase


Para comprender el funcionamiento de un rectificador controlado se va a considerar el caso ms simple mostrado en la Figura 7.28, que representa una fuente o red de c.a. con un tiristor que alimenta una carga resistiva. Es un rectificador controlado media onda. Durante los semiciclos positivos de la tensin de alimentacin, el tiristor S tiene polarizacin positiva; si se aplica un impulso de corriente ic a la puerta en el instante wt = o; que se mide a partir del momento en que es cero la tensin de la red, el tiristor conduce y la tensin de alimentacin se transfiere a la carga resistiva (se considera despreciable la cada de tensin en el tiristor, que es del orden de 1 V). Cuando la tensin aplicada empieza a ser negativa en wt = ti, el nodo del tiristor se hace negativo respecto a su ctodo (polarizacin inversa o negativa) y el tiristor se apaga o deja de conducir. El ngulo o: se denomina ngulo de encendido o de retardo del tiristor y se va aplicando de una forma sncrona en todos los semiciclos positivos de la tensin de red. Durante el intervalo en el que conduce el tiristor, denominado intervalo de conduccin, la tensin en la carga coincide con la tensin de la red. En la Figura 7.28 se han dibujado en b) la onda senoidal de la red, en c) los impulsos de disparo que se aplican de un modo sncrono, y en d) las ondas de tensin y corriente en la carga. Si la tensin de la red es de la forma v, = V;" sen wt, la tensin media V;, que se obtiene en la carga vale: V (1 + cos V. = - 1 V sen wt d(wt) = _!!!. (( 2n , 2n
ni

In

IX)

(7.47)

y la corriente media en la carga ser (.c = V,)R. Si el ngulo de encendido se vara entre Oy n, la tensin media en la carga variar entre un valor mximo v.i y cero (es evidente que cuando a = 0, el tiristor acta como un rectificador), de tal modo que la regin de trabajo corresponde al primer cuadrante mostrado en la Figura 7.28a. Si la carga es de tipo inductivo, entonces, al igual que suceda con el circuito rectificador media onda (epgrafe 7.4.2), se producir un retraso de la corriente frente a la tensin, y para evitar incursiones negativas en la tensin de carga deber colocarse un diodo de libre circulacin en paralelo con la carga para que disipe la energa magntica almacenada correspondiente. De todos modos, no es necesario profundizar ms en este circuito, ya que tiene pocas aplicaciones industriales debido al alto nivel de rizado de la seal de salida.

CAPTULO

7.

ACCIONAMIENTOS ELCTRICOS

595

TII
a)

IV

Figura 7.28.

Rectificador controlado media onda con carga resistiva.

7.5.3. Convertidor monofsico en puente completo con carga inductiva


Un convertidor monofsico en puente completo est constituido, como se muestra en la Figura 7.29a, por cuatro tiristores. Se va a analizar el comportamiento de este circuito si la carga es altamente inductiva, lo que supone que la corriente de carga tendr un rizado prcticamente despreciable. Durante los semiciclos positivos, los tiristores SI y S2 estn polarizados directamente y cuando se encienden simultneamente para un valor wt = '1., la tensin que aparecer en la carga "o ser la tensin de alimentacin v, = VIII sen en. Debido a la inductancia de la carga, los tiristores continuarn conduciendo ms all de n radianes, aunque la tensin de la red sea en esos momentos negativa debido a que la corriente de carga no se ha anulado (esto se debe al alto valor de la inductancia L). Durante los semiciclos negativos de la tensin de red, los tiristores S3 y S4 quedan polarizados directamente pero no empezarn a conducir hasta que no se aplique una seal a los electrodos de puerta. Al producirse el encendido en el instante correspondiente a tt + y. radianes, los tiristores S3 y S4 comenzarn a conducir, provocando el apagado inmediato de los tiristores SI y S2 por quedar polarizados inversamente. En este momento la tensin de la carga I'() es la opuesta a la tensin de red, es decir -v" El efecto del control de fase con carga inductiva es que la tensin de la carga tiene intervalos durante los cuales su tensin instantnea es negativa (Fig. 7.29d), lo que diferencia este circuito con el puente de diodos de la Figura 7.22, en el que la transferencia de conduccin de corriente de unos diodos a otros se realizaba en el preciso momento en que la tensin de alimentacin pasaba por cero voltios. El valor medio de la tensin en la carga se obtiene por simple integracin de la curva de la Figura 7.29d, resultando ser:

V:,

1 = ~2
n

fY+IT
x

V:" sen

(J){

2V dtcot = ~_m cos


n

y.

(7.48a)

596

MQUINAS ELCTRICAS

t,
u "O "O
<U

----:++ lO
SI
S3

1
-o

R
ic L

:,::

<U

j
+
II io

vo

io

s;. Si

s~S4
lec WI

e)

+
III a)
<X

IV

l.<

4 n lec
WI

f)

Figura 7.29. Convertidor monofsico en puente completo con carga inductiva y ex < 90.

y denominando Ved) a la tensin de c.c. que se obtendra utilizando diodos en vez de tiristores, cuyo valor se demostr en (7.28) y (7.36), la ecuacin anterior se puede escribir as: Vec =

-2V,n cos IX =
tt.

VecO COS

IX

(7.48b)

De la expresin anterior se deduce que se puede regular la tensin media que alimenta una carga (por ejemplo, un motor de c.c.) ajustando solamente el ngulo de encendido IX del tiristor. Obsrvese que para ngulos de encendido comprendidos en el intervalo O < IX < n la tensin Vec es positiva, y si se tiene en cuenta adems el sentido obligado de conduccin de corriente en los tiristores, la corriente de carga io tendr el sentido mostrado en la Figura 7.29a. El valor medio de esta corriente de carga vendr expresado por: (7.49) En la Figura 7.2ge se muestra la forma de la corriente de la carga io en rgimen permanente, que se supone constante y de amplitud l,.e (debido al alto valor de la inductancia que ha filtrado todos los armnicos de corrriente), La Figura 7.29frepresenta la forma de onda de la corriente alterna de alimentacin t; que prcticamente es una onda rectangular de amplitud (,. El valor de la potencia media V'('('e suministrada por convertidor ser siempre positiva, lo que significa que la energa se transfiere en el sentido red-carga; se dice entonces que el convertidor funciona en modo rectificador y su zona de funcionamiento corresponde al primer cuadrante indicado en la Figura 7.29a.

CAPTULO

7.

ACCIONAMIENTOS ELCTRICOS

597

Analizando la onda rectangular de corriente i, que se produce en la red de alimentacin, y representada en la Figura 7.29f, se puede demostrar que su desarrollo en serie de Fourier es de la forma: ,et) =

I - Ice
n= 1.3.5 ....

sen (nwt - cfJn)

; cfJn

nn

= na

(7.50)

donde cfJn es el denominado ngulo de desplazamiento del armnico de orden n (el cos cfJn se denominafactor de desplazamiento y coincide con el concepto de f.d.p. que se utiliza en la teora clsica de circuitos de c.a. senoidal). En la Figura 7.29f se ha dibujado con trazo discontinuo el armnico fundamental de corriente de la red isl' Ycuya amplitud mxima vale: (7.51) Se observa que su ngulo de desplazamiento cfJl de retraso con la tensin coincide con el ngulo IY. de encendido de los tiristores. Al ser la tensin de alimentacin Vs sinusoidal de pulsacin co, la nica corriente que contribuye a producir un flujo de potencia media es la debida al primer armnico anterior. Si se denomina V, = Vm/j2 el valor eficaz de la tensin de c.a. de alimentacin e lsl el valor eficaz de la corriente del armnico fundamental, la potencia media (en definitiva, potencia activa) que se suministra al convertidor, teniendo en cuenta las expresiones (7.48) y (7.51), ser: P

V/si

COS cfJl

(fi) . C~

Icc) cos

l:

=~

v.J; cos l: = v.J;

(7.52)

es decir, la potencia activa que absorbe la instalacin coincide, como era de esperar, con la potencia suministrada (o absorbida) por la carga en el lado de la c.c. La potencia aparente S en la red de c.a. es el producto del valor eficaz de la tensin de la red V, por el valor eficaz 1, de la corriente is entregada por la misma. El valor eficaz de esta corriente, teniendo en cuenta la forma indicada en la Figura 7.291, vale: I, =

-IIHn
n ,

I~. d(wt)

= Icc

(7.53)

De este modo se puede definir elfactor de potencia del convertidor por el cociente PIS, cuyo valor es: fd.p.
4

=P ec

V/sI COS V/s

cfJl

= t.. - cos cfJl =


t,

nj2l
Ice

----'---

cos

cfJl

= 0,9

COS cfJl

= 0,9 cos

O'.

(7.54)

lo que significa que el f.d.p. no coincide con el factor de desplazamiento. El cociente IJI, se denomina factor de distorsin y es una medida del contenido de armnicos que tiene la corriente alterna de la red. La expresin (7.54) nos indica que el f.d.p. de la instalacin es el productodel factor de desplazamiento por el factor de distorsin. De acuerdo con la expresin anterior, el f.d.p. de un circuito con rectificadores controlados puede ser muy pequeo, en particular cuando la tensin de salida es baja, es decir, para ngulos de encendido a grandes.

598

MQUINAS ELCTRICAS

En la Figura 7.30 se muestra, para mayor claridad, el circuito equivalente de un convertidor trabajando como rectificador controlado. Se observa que el convertidor absorbe potencia activa y reactiva. La potencia activa absorbida de la red coincide, como es lgico, con la potencia consumida por la carga de c.c. (se exceptan las prdidas elctricas en el convertidor). La potencia aparente absorbida por la instalacin S es el producto de la tensin eficaz de alterna V, por la corriente eficaz absorbida de la red 1" y de ah se puede deducir el valor de la potencia reactiva absorbida por el convertidor a travs de la expresin: Q=JS1_p2 (7.55)

Desde luego, conviene tener en cuenta que hablar de potencias reactivas en este tipo de circuitos con corrientes no sinusoidales no tiene sentido fsico, pero se puede considerar un anlisis simplificado si la corriente rectangular absorbida por la red se puede sustituir por el armnico fundamental, y en este caso la expresin anterior se transforma en: (7.56) En este caso tiene sentido real el diagrama fasorial mostrado en la parte inferior de la Figura 7.30. En definitiva, la aproximacin anterior significa hacer coincidir el f.d.p. de la instalacin con el factor de desplazamiento del armnico fundamental. Para mayor claridad se han superpuesto en esta figura las ondas de tensin y corriente que se mostraban en las Figuras 7.29b y f, donde se observa que el desfase entre las ondas v, e id coincide con el ngulo de encendido a. En la Figura 7.30 se muestra en la parte de la derecha el circuito equivalente de la rama de c.c. El generador de tensin representa la salida de c.c. del convertidor, el diodo en serie sirve para indicar que la corriente fluye solamente en un sentido y la inductancia en serie nos recuerda que la carga tiene una inductancia elevada. El mismo circuito de la Figura 7.29 puede funcionar tambin en modo inversor. Este tipo de funcionamiento se consigue si el ngulo de encendido de los tiristores se sita en el rango 90 < o: < 180 En este caso la tensin media v:.'c' de acuerdo con (7.48), ser negativa, lo cual no significa que se vaya a producir una corriente en la carga de signo negativo, ya que como sabemos el tiristor solamente puede llevar corriente
0 0

Figura 7.30.

Circuito equivalente de un convertidor trabajando como rectificador controlado.

CAPTULO

7.

ACCIONAMIENTOS ELCTRICOS

599

en el sentido nodo-ctodo. En esta situacin, la corriente en la carga tendera a anularse. Ahora bien, se puede obligar a que circule corriente en la carga de signo positivo pese a ser la tensin V:e negativa, siempre que se aada en el circuito de carga un generador de f.e.m. E de magnitud y polaridad adecuada, tal como se seala en la Figura 7.31 a. Esta f.e.m. externa debe ser ligeramente superior al valor medio V:, de la tensin que produce el convertidor para que pueda circular corriente por el circuito. De este modo, aplicando el 2. lema de Kirchhoff a la malla de salida de la Figura 7.31a se obtiene una corriente media le(:

E + V,,, = Rl

=:>

lec

= E+V:
R

'o

amperios

(7.57a)

Se debe destacar en la expresin anterior que V:e es el valor que se obtiene en la ecuacin (7.48), Yal ser CI. mayor de 90, su magnitud ser negativa. Evidentemente, si CI. = 90, se tiene Vee = OYel inversor acta como un cortocircuito respecto de la malla de salida. Para mantener Ice constante en presencia de un ngulo de encendido (X ajustable en el puente, se necesita variar bien sea el valor de R, de E o de ambos a la vez. Si la f.e.m. es constante, entonces un incremento en el ngulo (X provocar una reduccin de Icc debido al aumento negativo de Vee. La corriente lec valdr cero para un valor de CI. expresado por:

I . = E - (2V,,/n) cos el.= O


C(

==>

cos

(X

=2Vm

nE

(7.57b)

Il

III a)

Figura 7.31.

Convertidor monofsico en puente completo con carga inductiva ya>

90.

600

MQUINAS ELCTRICAS

En la Figura 7.31d se muestra la onda Vo que se obtiene en el lado de c.c., cuyo valor medio es Vee; en la Figura 7.31e se representa la corriente media Ice en la carga (aplanada por el alto valor de la inductancia), y en la Figura 7.3lfse muestra la forma de onda de la corriente en la red de c.a. Cuando el convertidor funciona como inversor, el producto VeJee ser negativo, lo que significa que existir una transferencia de energa de la carga a la red. Esta forma de trabajar se produce en el frenado regenerativo o por recuperacin de energa de los motores de c.c. y corresponde al cuadrante IV mostrado en la Figura 7.3Ia. Esta operacin requiere en la prctica invertir previamente la excitacin del motor para hacer que funcione como generador. En la Figura 7.32 se muestra para mayor claridad el circuito equivalente de un convertidor trabajando como inversor y que es anlogo al que se explic en la Figura 7.30. Para analizar el balance de potencias en el circuito, se considera que la corriente rectangular de la red de c.a. se puede aproximar por su armnico fundamental I\)' cuyo valor se expresaba en (7.51); de este modo, y como ya se ha indicado antes, se cumplir cos cP I = cos IX, por lo que la potencia activa de la red se determinar igual que en (7.52), es decir: P

v'n)(4) = VJSI cos cjJl = ( j2 . nj2

cos

IX

=;-2 Vn/e

COS IX

= v'Jce

(7.58)

es decir, la potencia generada en el lado de c.c. coincide con la potencia activa entregada por el convertidor a la red. La potencia disipada por efecto Joule en la resistencia R del circuito de la Figura 7.321a debe suministrar la batera E, pero no se ha considerado aqu por ser externa al convertidor. Es evidente, por consiguiente, que la potencia activa P de la red procede de la fuente de c.c. E y as se ha sealado con una flecha de transferencia de potencia en la Figura 7.32; sin embargo, la potencia reactiva del sistema procede de la red de c.a. y es necesaria para el funcionamiento del convertidor. De ah que este inversor no pueda funcionar de modo aislado, se necesita la red de c.a. para transferir la energa del lado de c.c. al lado de c.a. (es una situacin similar al funcionamiento de una mquina asncrona como generador de induccin). Viendo el convertidor desde el lado de la red de c.a., la corriente fundamental Isl

.
L

+
~'(

Figura 7.32. Circuito equivalente de un convertidor trabajando como inversor.

CAPTULO

7.

ACCIONAMIENTOS ELCTRICOS

601

absorbida est retrasada un ngulo a de la tensin Vs (vase parte inferior de la Fig. 7.32), pero para analizar el funcionamiento del convertidor como inversor se puede considerar que existe una corriente contraria 1:, = -1st que se entrega a la red. De ah que se pueda considerar el convertidor como un equipo que entrega a la red una potencia reactiva capacitiva expresada por:

Q = V,/;, sen (n

<P,) = Vi;, sen (rt -

a)

(7.59a)

ya que la corriente 1;, se adelanta respecto de potencia reactiva inductiva:

s'

o que la red entrega al convertidor una


(7.59b)

Q = V,!S' sen <p, = V'/S' sen a

ya que en este caso la corriente Id se retrasa respecto de Vs El convertidor en puente completo hasta aqu estudiado tiene la posibilidad de funcionar en dos cuadrantes, pero en aquellas situaciones en las que se desea que la corriente de carga circule en ambos sentidos con tensin de carga reversible, se necesita el uso de dos puentes de tiristores conectados en antiparalelo, como se muestra en la Figura 7.33; este convertidor se conoce con el nombre de convertidor dual o doble y puede trabajar en los cuatro cuadrantes. Se utilizan en accionamientos de velocidad variable y alta potencia. Estos convertidores dobles pueden trabajar simultneamente o no. En el funcionamiento simultneo o modo de corriente de circulacin interna, ambos convertidores funcionan a la vez, uno como rectificador controlado y el otro como inversor. En el funcionamiento no simultneo o modo de corriente de circulacin libre solamente funciona a un tiempo un convertidor, dejando al otro en situacin de bloqueo. Cuando se utiliza el modo de funcionamiento simultneo, los convertidores disponen de un control de disparo de los tiristores, que obligan a que la suma de sus tensiones medias de c.c. sea igual a cero, para que de esta forma no circule corriente continua en la malla que forman ambos convertidores, es decir:
(7.60)

vs

Red de c.a.

Id
~~ "2~

L/2
".........

L/2
".".".,..,

!t,

VVV

I .-ce:

. VVy

----.,

v"c1

+~ _
....... ----,

. ~~ ~~ f--3 i.g ~ L: B 11"e2


R
, U . , u

'o
E

~~

-.1-

Ll.

lec

~~ ~?

Figura 7.33. Convertidordual o doble.

602

MQUINAS ELCTRICAS

Si se denominan IX, y 1X2 los ngulos de disparo de ambos convertidores, los valores de las tensiones medias de c.c. son:
(7.61a)

por lo que al sustituir en (7.60) se obtiene:


(7.61b)

lo que significa que si un convertidor trabaja como rectificador, el otro trabaja como inversor. Esta forma dual de trabajar hace que sea imposible el conseguir que las tensiones instantneas suministradas por ambos convertidores sean iguales, lo que provova una corriente alterna de circulacin en la malla de los mismos y que se limita colocando las inductancias Lf2 mostradas en la Figura 7.33. Existen tambin puentes semiconvertidores que estn constituidos, como se muestra en la Figura 7.34, por un puente monofsico similar a los explicados en las Figuras 7.29 y 7.31 pero en el que los dos tiristores inferiores se han sustituido por dos diodos rectificadores. Generalmente, en este caso los semiconvertidores incorporan un diodo de recuperacin de energa para evitar tensiones negativas en la tensin de salida. Se puede demostrar que la tensin de salida es de la forma: Vec =

-1 fn
tt
o:

Vm sen wt d(wt)

V = ___!:! (1 + cos IX)


n

(7.62)

De acuerdo con la ecuacin anterior la tensin de salida Vee es siempre positiva, y de este modo la potencia VeJee es positiva, lo que indica que la potencia fluye siempre en el sentido red-carga. Los semiconvertidores no pueden de este modo invertir la potencia y solamente trabajan en el cuadrante l. EJEMPLO DE APLICACIN 7.6 Se utiliza un convertidor monofsico en puente completo similar al de la Figura 7.29 para controlar la velocidad de un motor de c.c. de 150 V, 1.500 r.p.m., con excitacin separada. El convertidorse conecta a una red monofsica de 220 V,50 Hz. El inducido tiene una resistencia de 0,5 ohmios y una inductancia de 5 mH. Se supone que el motor tiene un circuito magn-

t,
+

SI

_v, = ~s

TI rol

vo

+
E S4 S2

Figura 7.34.

Semiconvertidor monofsico.

CAPTULO 7.

ACCIONAMIENTOS ELCTRICOS

603

tico lineal y que la constante k<D de la f.c.e.m. del motor es de 0,1 V/r.p.m. (Recurdese que la [.c.e.m. del motor viene expresada por: E = k<Dn). l. Funcionamiento del convertidor como rectificador controlado. Mquina de c.c. en rgimen motor: La mquina de c.c. funciona como motor girando a J .200 r.p.m. y lleva una corriente de inducido de 50 A. Suponiendo que el rizado de la corriente del motor es despreciable debido a la alta inductancia del inducido: a) determinar el ngulo (X de encendido que requieren los tiristores; b) potencia elctrica absorbida por el motor; c)jd.p. que presenta el grupo convertidor-motor a la red de c.a. 2. Funcionamiento del convertidor en modo inversor. Frenado por recuperacin de energa: Se cambia la polaridad de la f.c.e.m. del motor invirtiendo la corriente en el devanado de excitacin: a) determinar el ngulo de encendido de los tiristores para mantener la corriente de inducido en 50 A con una velocidad de 1.200 r.p. m.; b) calcular la potencia que se devuelve a la red de c.a. para la velocidad de 1.200 r.p.m.

SOLUCIN 1. Funcionamiento rgimen motor.


a)

del convertidor como rectificador controlado: Mquina de c.c. en

La f.c.e.m. del motor a 1.200 r.p.m. vale: E = 0,1 . 1.500 = 150 V, por lo que la tensin media que debe aplicarse al inducido es: Ve! = E + RIce De la ecuacin (7.48) resulta: Vee = 175 =

150 + 0,5' 50 = 175 V

2V

cos

IX

2}2 220

cos

IX

=>

IX

= 27,93

b)

La potencia elctrica absorbida por el inducido del motor (se desprecian las prdidas en el circuito de excitacin) es: P = VeJcc = 175 . 50 = 8.750 W

c)

De acuerdo con la ecuacin (7.54), el f.d.p. que ofrece el grupo convertidor-motor es:
f.d.p. = 0,9 cos
IX

= 0,9 cos 27,93 = 0,795

2. Funcionamiento energa.
a)

del convertidor en modo inversor. Frenado por recuperacin de

En el momento en el que se cambia la polaridad a la excitacin, como la velocidad es la misma de 1.500 r.p.m., la f.c.e.m. no cambiar y su valor ser el mismo que en el caso anterior, es decir, 150 V. La tensin en bornes del motor que acta como generador ser, de acuerdo con (7.57): E + Ve! = RI
=>

Vec = 0,5 . 50 - 150 = -125 voltios

y al sustituir en (7.48) resulta un ngulo de conduccin:

V:_,

= -125 = ---;;-cos IX =

2v'n

2)2220
neos

=>

IX

129,13

604 b)

MQUINAS ELCTRICAS

La potencia que entrega la mquina de c.c. hacia el grupo convertidor-red de c.a. en el momento de frenado por recuperacin es:
PJ

=E

. (c

150 . 50 = 7.500 W

Las prdidas en la resistencia del inducido son: Pe" = R . l;e = 0.5 . 502 = 1.250 W, por lo que la potencia entregada a la red vale: P = 7.500 - 1.250 = 6.250 W, valor que coincide con el producto v"Jee tal como seala la expresin (7.58): P = Vee . lec = = 125 . 50 = 6.250 W.

7.5.4.

Convertidor trifsico en puente completo

En accionamientos elctricos de alta potencia conviene utilizar convertidores trifsicos, ya que producen mayores tensiones de salidas y de menor rizado que los monofsicos y tambin contribuyen a un equilibrado mayor de la red en comparacin con estos ltimos. Existen convertidores trifsicos media onda equivalentes a los rectificadores de este nombre cuyo esquema elctrico se vio en la Figura 7.23, pero en el que ahora se sustituyen los diodos por tiristores. En este epgrafe solamente se van a explicar los convertidores en forma de puente completo y cuyo esquema elctrico se muestra en la Figura 7.35a, y que es equivalente al rectificador en puente de la Figura 7.24 en versin controlada o con tiristores. Se supone, como es habitual, que la carga es fuertemente inductiva. Los tiristores del puente se encienden a intervalos de 60 (es decir, n13) en la sucesin indicada por sus nmeros, tal como se muestra en la Figura 7.35c. El origen del ngulo rJ. de encendidose ha fijadoen el punto A, que correspondeal instanteen que se producirala conmutacin natural de haber utilizado diodos rectificadoresen vez de tiristores(repasar epgrafe 7.4.5). Las ondas senoidales representan las diversas tensiones compuestas de la red, que a partir del punto A son: vRS' vRP vST' vTR Y vTS (desfasadas entre s 60) y cuyos valores mximos son Vm, siendo Vm la tensin de pico de cada una de las fases del secundario del transformador. Para comprender el funcionamiento del circuito se ha dibujado en la Figura 7.35b la seal de salida (onda de trazo ms grueso), donde se ha supuesto un ngulo de encendido de 0, dando lugar a un comportamiento como el rectificador clsico. Para explicar esta figura se supone que se parte de una situacin previa en la que estaban conduciendo los tiristores S5 y S6. Al llegar al origen de tiempos en el puntoA, la tensin compuesta de mayor valor es la vRS' por lo que si se dispara el tiristor SI aplicando un impulso a su puerta (vase corriente Gl en la Figura 7.35c), SI comienza a conducir por estar polarizado en directo, lo que provoca al mismo tiempo el apagado del tiristor S5 al quedar polarizado en inverso, por lo que a partir de A son los tiristores S6 y SI los que conducen. De esta forma la tensin de la red, que en ese momento es vRS' se aplica a la carga. Cuando se llega al instante representado por el punto B (60 despus que A) se dispara S2 (al aplicar un impulso de corriente G2 a su puerta), que comienza a conducir por estar polarizado en directo, pero al mismo tiempo se apaga el tiristor S6 por quedar polarizado en inverso, es decir, a partir del punto B los tiristores que conducen son SI y S2 y la tensin VRT en ese nuevo tramo que se inicia llega a la carga. Este proceso contina indefinidamente y la onda de tensin que se obtiene a la salida es idntica a la que produca el rectificador en puente de la Figura 7.24 con el mismo valor medio que se indicaba en (7.43), y que de acuerdo con la nomenclatura utilizada en (7.48b) se denomina YecO' y cuyo valor es:

J3

VecO=

3(J3Vm) n

(7.63)

CAPTULO 7.

ACCIONAMIENTOS ELCTRICOS

605

SI TRANSFORMADOR

S3

S5

+
R

Primario

Secundario

al

wt

b)

Corriente de puertaiG

el

Figura 7.35.

Puente rectificador controlado. Disparo en O.

donde v'n representa el valor mximo de la tensin simple del secundario del transformador. La corriente continua media en la carga ser de amplitud constante, debido al aplanado que produce la inductancia L, y su valor es lcc = Vc)R. Es indudable que para conseguir la forma de onda mostrada en la Figura 7.35b se requiere unagran precisin en los impulsos de disparo; por ejemplo, si se dispara el tiristor SI un poco antes de que la onda llegue al punto A, el tiristor no puede iniciar su conduccin porque su polarizacines todava negativa. En la prctica, para evitar este problema se utilizan impulsosde disparo de mayor anchura y adems suele retrasarse el disparo de 10 a 15 respecto al punto A. Vamos a analizar a continuacin el comportamiento de este convertidor cuando los impulsosde disparo se retrasan respecto al punto de conmutacin natural. En la Figura 7.36a se muestranlas ondas de tensiones correspondientes. Al igual que antes, se supone una situacin inicialen la que estaban conduciendo los tiristores S5 y S6. Al llegar al punto A, en vez de proceder al disparo del tiristar SI, retrasamos su disparo un ngulo exprximo a los 45 (Fig. 7.36a), es decir, cuando la onda pasa por el punto B. Es indudable que S5 y S6 habrn seguidoconduciendo hasta el punto B, pero en ese momento, debido al disparo del tiristor SI

606

MQUINAS ELCTRICAS

(y teniendo en cuenta que queda polarizado directamente), se produce su encendido, provocando el apagado inmediato de SS (por quedar ste polarizado inversamente). La tensin existente en la carga, que hasta el punto B era VTS' pasa a ser v RS' Este proceso se va repitiendo conforme se van disparando los siguientes tiristores cada 60, dando lugar a una forma de la onda de salida representada por los tramos de trazo grueso sealados en la Figura 7.36a. Obsrvese que el ngulo de encendido o: impone el instante de inicio en la conduccin del nuevo tiristor (siempre que su polarizacin sea positiva) pero no influye en el perodo de conduccin de los tiristores, que sigue siendo de 120. Obsrvese tambin que los tramos de onda que se obtienen solamente duran 60. Para determinar la tensin media V:c que se obtiene en este convertidor para el ngulo de encendido ex, se puede calcular el valor medio de un tramo repetitivo de la onda peridica de salida. Si se toma el tramo correspondiente a los puntos B' y e (de duracin n/3), resulta que la tensin en esa zona es vRS = fivm sen ou, y como quiera que los ngulos correspondientes a estos puntos (medidos desde el instante en el que la tensin vRS pasa por cero) son, respectiti 2n vamente: 08 = - + ex; Oc = - + ex, resulta: 3 . 3

v. =
ce

~ 1 n/3 H:!Jr
3

f>+3 fiv

sen wt d(wt)

3(J3V) m cos ex n

(7.64a)

Origen de ngulos de encendido

S2 12

S3

S4 23
34

S5

S6

SI
56

45

J3 v,,,

WI

a)

i,

rr/3

()(

-+
I nl3 + IY.
n+1Y. 7nl3 + ex wt
e)
VRN

b)

i:

14

.0

4n/3 + ()(

2n+

IY.

mt

d)

Figura 7.36.

Puente rectificador controlado. Seales para un ngulo de disparo de 45.

CAPTULO

7.

ACCIONAMIENTOS ELCTRICOS

607

y teniendo en cuenta que segn (7.63) la expresin fraccionaria de (7.64a) es la tensin del rectificador (es decir, el valor de la tensin para a = 0), se puede escribir:
Ve(' =

~.c()

3(J3~,.)
tt

cos a = VecO cos a

(7.64b)

En la Figura 7.36b se ha dibujado la corriente il en el tiristor SI, en e) la corriente i4 en el tiristor S4 y en d) la corriente iR en la fase R del secundario del transformador. Tngase en cuenta que las corrientes en cada uno de los tiristores tienen una duracin de 120 y su amplitud coincide con la corriente media de la carga lcc. La corriente de lnea en la fase R se obtiene por la diferencia iit) = il - i4 (aplicar el primer lema de Kirchhoff en el nudo M del circuito puente de la Fig. 7.35a). En las fases S y Tse obtienen ondas anlogas. El desarrollo en serie de Fourier de la corriente de lnea, tomando ahora como referencia de ngulos la tensin simple VRN, tal como se indica en la Figura 7.36d, es de la forma:
iR(t) =

L.
n= 1.3.5""

_" ntt

41..

sen - sen (non - na) 3

ntt

(7.65)

por lo que el valor eficaz de la componente del primer armnico (n = 1) de la corriente de lnea vale: (7.66) que se retrasa a grados de la tensin de fase del secundario del transformador, en nuestro caso de VRN En la Figura 7.36d se ha representado la tensin simple VRN, que como se sabe para un sistema de secuencia directa est retrasada 30 respecto de la tensin compuesta VRS. La corriente iR es la onda escalonada resultante de la resta iR = t. - i4' La componente fundamental de la corriente anterior iLI se retrasa un ngulo a respecto de VRN. La potencia activa que suministra el secundario del transformador vale:
p =

J3VJLI

cos a

(7.67)

y teniendo en cuenta (7.64) y (7.66), y recordando que Vm era la tensin mxima de una fase, la expresin (7.67) se transforma en:

es decir, la potencia activa entregada por el transformador coincide con la potencia suministrada a la carga de c.c. (se desprecian las prdidas en el convertidor). La ecuacin anterior representa la ecuacin del balance energtico del convertidor. El valor eficaz de la corriente de la red incluyendo sus armnicos impares se puede obtener directamente a partir de la curva de la Figura 7.36d, resultando ser:
1 n

5n

>+6

Ice . d(J)t) =

(7.69)

C(+~ 6

608

MQUINAS ELCTRICAS

Por consiguiente, el factor de potencia de la instalacin definido por el cociente entre la potencia activa y la aparente, teniendo en cuenta (7.64), (7.69) Yla expresin del valor eficaz de la tensin compuesta de la red en funcin de la tensin mxima simple, ser:

(7.70)

As pues, en un convertidor trifsico el factor de potencia se ve menos reducido por las corrientes armnicas que en el convertidor monofsico (vase ecuacin 7.54). Sin embargo, el f.d.p. de la alimentacin puede ser mucho menor que la unidad cuando el rectificador trabaja con una tensin de salida pequea. Dependiendo de la potencia del convertidor y de su colocacin, puede ser necesario reducir los armnicos de corriente en la red de alimentacin. Esto se consigue utilizando un filtro de entrada, que consiste en colocar una inductancia en serie con condensadores en paralelo entre las diferentes fases del rectificador. De un modo anlogo al convertidor monofsico, el puente trifsico de la Figura 7.35a puede funcionar tambin en modo inversor. Este tipo de funcionamiento se consigue si el ngulo de encendido de los tiristores se sita en el rango 90 < IX < 180. En la Figura 7.37 se muestra la onda de tensin correspondiente para un ngulo de encendido prximo a los 135. En este caso, y como seala la ecuacin (7.64), la tensin media de c.c. es negativa, y si la carga contiene un generador de tensin, habr una conversin de la c.c. de este generador a la c.a. de la red, lo que se utiliza en la prctica en el frenado por recuperacin de energa de los motores de c.c. En la prctica el ngulo de encendido no suele sobrepasar los 150-165, para dar tiempo al apagado de los tiristores y evitar de este modo una conmutacin para un ngulo superior a los 180, que provocara un cortocircuito en la red de alimentacin. En algunas aplicaciones se utilizan tambin semiconvertidores trifsicos que responden al circuito de la Figura 7.38, que es anlogo al de la Figura 7.35a pero donde los tres tiristores inferiores del puente se han sustituido aqu por diodos rectificadores y donde se ha aadido un
Tiristores que se encienden-e-c-e- SI Origen de ngulos ex = 135

S2

S3

S4

S5

S6

Figura 7.37.

Puente rectificador controlado. Onda de tensin para un ngulo de disparo de 135.

CAPTULO 7.

ACCIONAMIENTOS ELCTRICOS

609

il

i
S3
M

TRANSFORMADOR

iR

jo

SI

SS

+
R Dw
Vo

i4

L Primario Secundario

S4

S6

S2

Figura 7.38.

Semiconvertidor trifsico o puente rectificador semicontrolado.

diodo de libre circulacin en paralelo con la carga para evitar tensiones negativas en la misma. Se puede demostrar que la tensin media de c.c. que se obtiene en este-caso viene expresada por: Vee = 3(.J3V,,,) (1 + cos a) 2n (7.71)

y el convertidor solamente puede trabajar en el cuadrante 1. En accionamientos elctricos que requieren la regulacin de velocidad en los cuatro cuadrantes se emplean convertidores trifsicos dobles, como se muestra en la Figura 7.39; consisten en dos convertidores trifsicos conectados en oposicin o paralelo-inverso. Al igual que en el caso de los convertidores monofsicos, los ngulos de encendido de ambos convertidoresestn relacionadospor la ecuacin (7.6Ib) y pueden funcionar los dos a un tiempo, o modo de funcionamiento simultneo, o uno cada vez, denominado entonces modo de funcionamiento no simultneo. En el primer caso, mientras un convertidor funciona como

Lf/2

Lf/2

Icc~

c:o

c:o~ le"

:..... _ ---: R: , ,:, , , , , , , :,, L:, ,

.
:

o.J

!
oJ

o.J

C"

Ic'o

C"

' ' ' ... --

- ,,: --_.
t Ice

o.J

oJ

o.J

Figura 7.39.

Convertidor trifsico doble o dual.

610

MQUINAS ELCTRICAS

rectificador controlado el otro trabaja como inversor; en el segundo caso, cada uno de los convertidores puede funcionar como rectificador o como inversor, uno de ellos trabaja en los cuadrantes I y IV y regula la velocidad en un sentido y el otro trabaja en los cuadrantes 11y 111 regulando la velocidad en sentido contrario. Antes de finalizar este epgrafe dedicado a los convertidores trifsicos controlados en fase es interesante comentar una aplicacin interesante de los mismos en el transporte de energa elctrica en corriente continua a alta tensin HVDC (high voltage direct current). En la Figura 7.40 se representa el esquema simplificado de este sistema de transporte, que consiste en dos redes de c.a. trifsicas dotadas de sendos convertidores trifsicos cuyas salidas se unen por medio de una red de transporte de c.c. Las aplicaciones modernas de los sistemas HVDC comenzaron en 1954 en Suecia empleando rectificadores controlados de vapor de mercurio; se instal una lnea submarina de c.c. de 100 kV entre la isla de Gotland en el Bltico y una ciudad costera del continente sueco. La distancia fue de 96 km y con una potencia transmitida de 90 MW. A partir de esta fecha se han construido muchas redes de c.c. en alta tensin sobre todo empleando lneas areas de gran longitud, como es el caso de Canad, Estados Unidos, Zaire y Mozambique. Existen tambin lnea submarinas que enlazan Inglaterra y Francia, por el estrecho de Calais. En Japn hay lneas de c.c. uniendo varias islas y hay otros ejemplos en Nueva Zelanda. Como ejemplo de este tipo de instalaciones se puede citar la Pacific Intertie, inaugurada en 1970, que es una lnea bipolar de 400 kV que une Celilo en Oregn con Los ngeles en California. Es una lnea area de 1.370 km y que transmite una potencia total de 1.440 MW. El circuito equivalente de esta instalacin (de un polo respecto de tierra) responde al esquema de la Figura 7.40. Si se desea transmitir potencia desde el centro I (Celilo) al centro 2 (Los ngeles), se producen 400 kV en la 1."ciudad con una corriente de 1.800A; como la resistencia de la lnea de c.c. es aproximadamente de 19 ohmios, llegan a Los ngeles aproximadamente 366 kV. La potencia generada en Celilo es de 720 MW, llegando a Los ngeles 658,44 MW. Como la lnea es de doble circuito, la potencia total transportadaes de 1.440MW. Para transferir energa en sentido contrario, se producen en Los ngeles -400 kV, llegando a Celilo -366 kV. Cuando un centro trabaja en modo rectificador, el otro trabaja en modo inversor. Otro ejemplo notable de instalacin HVDC es el que existe en la central hidroelctrica de Itaip, en la frontera entre Brasil y Paraguay. Es actualmente la central hidroelctrica ms grande del mundo y consiste en 18 alternadores de 700 MVA cada uno, nueve de ellos son de 50 Hz para suministro de energa a Paraguay y los otros nueve de 60 Hz para suministro elctrico de Brasil. Entre ambas instalaciones de c.a. existe un enlace asncrono de c.c. para convertir la energa elctrica trifsica de 50 Hz de Paraguaya c.c. y sta a trifsica de 60 Hz para Brasil; de esta forma, Paraguay, que no necesita tanta energa elctrica, puede enviar su excedente de energa a Brasil.
Centro 1 Centro 2

Red trifsica

,..

t
Rectificador inversor

"R

yyy

+ -

...... Red

trifsica

~
Inversor rectificador

Figura 7.40.

Transporte en c.c. en alta tensin.

CAPTULO

7.

ACCIONAMIENTOS ELCTRICOS

611

EJEMPLO DE APLICACIN 7.7 Se utiliza un convertidor trifsico dual (Fig. 7.39) para controlar la velocidad de un motor de e.e. con excitacin independiente. El convertidor se alimenta de una red trifsica de 380 V, 50 Hz. La resistencia del inducido del motor se supone despreciable y su inductancia esde elevado valorpara poder considerarplana la corriente que absorbe el motor. El convertidor 1 (el de la izquierda de la Fig. 7.39) trabaja como rectificador y suministra una corriente de 180A. El convertidor 2 (el de la derecha de la Fig. 7.39) trabaja como inversor y absorbe del otro una corriente de 30 A. La tensin en bornes del motor es de 500 V. Calcular: a) potenciassuministradas por ambos convertidores; b) potencia activa absorbida por el conjunto de la red trifsica; c) ngulos de encendido de ambos convertidores; d) potencia reactiva absorbida de la red trifsica. NOTA: Considerar que el fd.p. coincide con el factor de desplazamiento.
SOLUCIN
a)

La tensin simple o de fase de la red vale:

Vfase = VI = j3

380

= 219,39 voltios les:

La potencia suministrada por el convertidor PI =

V,Jccl = 500 . 180 = 90 kW V,Jcc2 = 500 . 30 = 15 kW

La potencia absorbida por el convertidor 2 y que ste devuelve a la red es:


P2 =

b)

La potencia activa que absorben los convertidores de la red trifsica ser: P = PI - P2 = 90 - 15 = 75 kW

c)

El ngulo de encendido del convertidor

1, de acuerdo con (7.64), ser:

El ngulo de encendido del convertidor 2 ser:


=>

-500 =

3(j3

J2'
n

219,39)

cos x,

es decir,

(X2

= 180 -

XI

= 180 - 13 = 167. = 90 tg 13 = 20,78 kVAR

d) La potencia reactiva absorbida por el convertidor 1 de la red trifsica es:


QI = PI tg
XI

La potencia reactiva absorbida por el convertidor 2, teniendo en cuenta que entrega a la red una potencia activa de 15 kW, es decir, absorbe de la red una potencia activa de -15 kW, es:

Qz = P2 tg x2 = -15, tg 167 = 3,46 kVAR

612

MQUINAS ELCTRICAS

Es importante que se observe que las potencias activas se restan (P = PI - P2), mientras que las potencias reactivas se suman (Q = QI + Q2 ). La razn es que los convertidores con conmutacin natural siempre absorben potencia reactiva, tanto si funcionan como rectificadores o si funcionan como inversores.

EJEMPLO DE APLICACIN 7.8 La Figura 7.40 muestra un sistema de transporte en c.c. a alta tensin. Se supone que eL centro 1 acta como rectificador y el centro 2 trabaja como inversor. Las redes trifsicas se supone que tienen una tensin compuesta de 380 kV. El centro 1 produce una tensin de salida de 400 kV con una corriente de 1.800 A, siendo la resistencia de Lared de c.c. de 19 ohmios. CalcuLar:a) ngulos de encendido de los tiristores de los centros 1 y 2; b) potencias activas y reactivas de cada centro convertidor. SOLUCIN
a) El ngulo de encendido del convertidor 1, de acuerdo con (7.64), ser:

La d.d.p. en el centro 2 vale: Vcc2 =

V:cI -

RIce

= 400

- 19 . 1.800

= 365,8

kV

Este centro funciona como inversor, por lo que se tiene:

b)

La potencia transferida desde el centro 1 hacia el centro 2 vale: PI = Veclla = 400 . 1.800 = 720 MW La potencia que entrega el centro 2 a la red trifsica 2 vale: P2 = Vec2(e

= 365,8

. 1.800

= 658,44

MW

La potencia reactiva absorbida por el centro 1 de la red trifsica 1, teniendo en cuenta que (J( = 38,8, vale: Q

= PI tg (J(I = 720

. tg 38,8 ~ 579 MV AR

El centro 2 entrega a la red trifsica 2 una potencia activa de 658,44 MW, o de otro modo, recibe de la red trifsica una potencia activa de 658,44 MW, por lo que la potencia reactiva recibida de la red trifsica 2 ser: Q2 = P2 tg (J(2 = -658,44 . tg 135,5 ~ 647 MV AR

CAPTULO 7.

ACCIONAMIENTOS ELCTRICOS

613

7.6. REGULADORES DE CORRIENTE ALTERNA 7.6.1. Introduccin


Un regulador de c.a. est constituido por dos tiristores en oposicin o simplemente un triac, que permite variar el valor eficaz de la tensin alterna que llega a una carga. Las aplicaciones ms importantes de estos reguladores son: control del flujo luminoso de lmparas elctricas e instalaciones de alumbrado, regulacin de calefactores elctricos, cambiadores de tomas de transformadores y en control de velocidad de motores asncronos. Los tipos de control utilizados son: el control de fase y el control on-off o encendidoapagado. En el control de fase, los tiristores actan enviando a la carga ciclos parciales (parte de la onda) de la tensin alterna de la red; en el control on-off, los tiristores trabajan de tal modo que envan a la carga ciclos completos de la red pero de un modo intermitente. Los sistemas controlados dependen del tipo de red, y as, existen reguladores monofsicos y trifsicos. Teniendo en cuenta que trabajan con alimentacin de c.a., la conmutacin o apagado de los tiristores o triacs se produce de un modo natural, y por ello no se requieren circuitos especiales de apagado. .

7.6.2. Regulador con control de fase y carga resistiva


En la Figura 7.41a se muestra un generador de c.a. o red monofsica que tiene una tensin instantnea v, = Vs sen cot que alimenta una resistencia R a travs de un triac o dos tiristores en oposicin. Al aplicar impulsos de disparo en el circuito de puerta del triac cada 180 (n radianes) y regulando el ngulo de encendido, se puede controlar la tensin eficaz que llega a la carga. En la Figura 7.41b se ha dibujado la onda de c.a. de la red, en e) los impulsos de disparo y en d) las formas de onda de la tensin y la corriente en la resistencia R. Tngase en cuenta que debido al carcter bidireccional del triac, la seal de salida es similar a la que se obtena en un rectificador controlado (vase epgrafe 7.5.2), pero incluyendo ahora formas de onda similares en los semiciclos negativos. El valor de la tensin eficaz en la carga vale:

j2

-2 fn (j2v, sen
2n
o:

IX sen 2IXJI/2 wt)2 d(wt) = Vs 1 - - +--

2n

(7.72)

Por consiguiente, variando el ngulo IX entre Oy n, la tensin eficaz Va variar desde Vs a O.La corriente eficaz en la carga ser igual a la corriente eficaz de la red, y su valor es:
Vo 10=1 s =R

(7.73)

De este modo la potencia disipada en la carga vale: (7.74) y la potencia aparente que entrega el generador de c.a. es: So

= V,l, = V/o

(7.75)

614

MQUINAS ELCTRICAS

Triae

rv \~\. = sen .ot


~n

mt

e)

a)

el)

Figura 7.41.

Regulador con control de fase y carga resistiva.

por lo que el f.d.p. de la instalacin vale: P V.I V. f.d.p, = ~ = ~.J o = __(_.J = 1 sen 2 .et.JII2 _ ~ +
ti

V/o

V.

2n

(7.76)

lo que indica que el f.d.p. solamente es igual a 1 cuando el ngulo de encendido et. es igual a cero. El valor del f.d.p. disminuye al aumentar el ngulo de encendido del uiac. Debido a que la carga es resistiva, la forma de onda de la corriente es idntica a la de tensin en la carga y van en fase. El apagado del triac se produce de un modo natural cuando la corriente se anula en cada semiciclo.

7.6.3.

Regulador con control de fase y carga inductiva

El esquema correspondiente se muestra en la Figura 7.42a, en h) se ha dibujado la tensin de la red, en e) los impulsos de disparo, en d) la corriente en la carga y en e) la tensin en la carga. Debido a la inductancia de la carga, la corriente en el triac no se anula en OJt = tt, como suceda en el caso anterior, sino para un ngulo ou = /) (/J se denomina ngulo de extincin). El ngulo de conduccin del triac <> = f3 - 'Y. depende del ngulo de encendido 'Y. y del argumento de la impedancia de carga O. Durante el intervalo de conduccin, es decir, para 'l. < W{ < {J, la ecuacin elctrica del circuito es: dio V 2V, sen cot = R10 + L dt

(7.77)

CAPTULO 7.

ACCIONAMIENTOS ELCTRICOS

615

v,

wt Triac b)

WI

...___'io

el
WI

+
'\_.., l~r =

v",

sen (1)1

4rr d)

a)

wt e)

Figura 7.42. Regulador con control de fase y carga inductiva.

que se resuelve de un modo anlogo al seguido en el epgrafe (7.4.2), pero en este caso la condicin inicial es io = Opara wt = a. El lector puede demostrar que la solucin es:
io(t) = __ Z

J2v

[sen (wt - 8) - sen


;

(a - 8) . e,/tgO .

e-rlr] L
(7.78)

donde Z

= J R2 + (LJ)2

0= arctg -

Lw

,=-

En la Figura 7.42d se muestra la forma de onda de la corriente de carga io(t), que adems de anularse para wt = a, tambin se hace cero para wt = {3, siendo {3 el ngulo de extincin, y que se puede calcular igualando a cero (7.78), lo que da lugar a: sen (/3 - 8) = sen (o: - O) . pero teniendo en cuenta que
e"'tgO . e-wrlC,,,

(7.79a)

en = {3 y

J, = tg 0, la ecuacin anterior se transforma en:


{J-,

sen ({3 - O) = sen (ex o empleando el ngulo de conduccin


--_
= <5

8) . e -go

(7.79b)

{3 - a, se obtiene:
= tg (o: - 8) (7.79c)

sen

etgO -

cos <5

ecuacin trascendente que permite calcular <5 por aproximaciones sucesivas. Este ngulo de conduccin <5 debe ser inferior a n radianes para que el tiristor S1del triac se apague antes de

616

MQUINAS ELCTRICAS

encender el tiristor S2 en wt = IX + ti. Tngase en cuenta que si se dispara el tiristor SI en = IX, el triac comenzar a conducir en sentido positivo hasta que se anule la corriente de carga, de manera que si en el instante n + IX aqulla an no se ha anulado, el tiristor S2 no entrar en conduccin aunque se enve el correspondiente impulso de disparo a su puerta. En consecuencia, el ngulo de conduccin de cada tiristor del triac debe ser como mximo igual a n, es decir, un semiperodo, y de este modo cada tiristor del triac funcionar de un modo independiente del otro. Si en el primer miembro de la ecuacin (7.79c) se toma b S tt, se deduce del 2. miembro que debe cumplirse IX ;::: e. Obsrvese que si se aplica la condicin lmite mnima de hacer IX = e, la corriente de carga io(t) expresada en (7.78) sera una senoide pura, ya que se anula el trmino exponencial, resultando ser:
iot

io(t) = __

J2v: sen (wt z


o

e)

(7.80)

o sea, la misma corriente que circulara por el circuito sin que existiera el triac; es decir, como si la carga inductiva se conectara directamente a la red. Si el ngulo de encendido IX fuese inferior a es decir, IX < la ecuacin (7.79c) nos indica que b > n, esto es, el ngulo de conduccin es superior a la duracin de un semiciclo, por lo que el tiristor S2 del triac no comenzara a conducir aunque se aplique un impulso de disparo a su puerta en el instante t: + IX; el circuito trabajara de este modo como rectificador controlado para la parte positiva de la onda (es decir, slo actuaria el tiristor positivo). Para evitar esta situacin se suele aplicar un tren de impulsos a la puerta del triac o impulsos de mayor duracin, de modo que si en el inicio de estos impulsos de disparo se verifica IX < e, el tiristor SI no comenzar a conducir hasta que no se llegue al instante en que cat = IX = e, y por ello en el rango O < IX S e la corriente io(t) viene expresada por (7.80), sin que los impulsos de disparo modifiquen la forma de onda de la corriente. A partir de IX > e la corriente es discontinua, tal como se muestra en la Figura 7.42d, y su valor obedece a la ecuacin (7.78). Este sistema de control, que permite ajustar el valor eficaz de la tensin de c.a. que llega a una carga, se utiliza en la prctica para la regulacin de velocidad de motores universales o motores monofsicos con colector de delgas, como es el caso de los taladros elctricos, aspiradoras y batidoras domsticas y tambin en el control del flujo luminoso de lmparas de incandescencia. Utilizando tres triacs o tres juegos de dos tiristores cada uno, se puede regular la velocidad de motores asncronos trifsicos; hay que tener en cuenta en este caso que como el par desarrollado por estos motores depende del cuadrado de la tensin aplicada, este sistema de regulacin solamente se utiliza para motores de potencia pequea y conviene adems que el par resistente sea del tipo cuadrtico, como es el caso de los ventiladores y bombas centrifugas, para que el par resistente en el arranque sea de pequeo valor.

e,

e,

7.6.4. Regulador on-off


El principio de este sistema de control se puede comprender fcilmente con el esquema de la Figura 7.43, donde una red de c.a. monofsica alimenta una carga inductiva a travs de un triac. Para regular la potencia de la carga, en vez de controlar el ngulo de disparo del triac, lo que se hace es aplicar impulsos sncronos a su puerta en el momento en que la tensin pasa por cero durante una serie de ciclos y desconectar los impulsos de disparo durante otra serie de ciclos. En el caso de la Figura 7.43 se observa que la tensin de alimentacin se ha conectado a la carga durante n ciclos consecutivos y se ha desconectado

CAPTULO

7.

ACCIONAMIENTOS ELCTRICOS

617

v,
b)

WI

e)

wt

d)
a)
WI

Figura 7.43.

Regulador con control on-off del triac.

durante los siguientes m ciclos. De este modo el valor eficaz de la tensin aplicada a la carga resistiva ser: n 2n(n + m)

In
a

(J2v

sen

wt)2 d(wt)

= v,

In = vsJk .v-;;;;;;

(7.81)

donde k es igual a nlcn + m), y esta constante se denomina ciclo de trabajo (duty cye/e) del regulador. Este tipo de control se utiliza en la regulacin de velocidad de motores con mucha inercia mecnica y tambin en la regulacin de temperatura de calderas de agua con mucha inercia trmica (de grandes dimensiones); en este caso la resistencia R de la Figura 7.43 representa el elemento calefactor que recibe ciclos variables de la onda de la red de c.a., consiguindose de esta manera fijar la temperatura del fluido de la caldera en un valor muy constante. Inicialmente, si el agua de la caldera est muy fra, el triac recibe impulsos de disparo todo el tiempo, de tal modo que la resistencia de calefaccin de la caldera funciona a toda potencia, pues recibe todos los ciclos de la red; conforme va aumentando la temperatura del agua, van disminuyendo el nmero de impulsos aplicados a la puerta, reducindose el nmero de ciclos activos n respecto de los pasivos m. Cuando la temperatura del agua es cercana a la de consigna (set pont), entonces el valor de n se reduce sensiblemente y es el suficiente para mantener la temperatura en el valor deseado. Este sistema de control recibe por ello el nombre de proporcional, ya que cuanto mayor es el error de temperatura, es decir, la diferencia entre la temperatura de consigna y la real que tiene la caldera, tanto mayor es la potencia desarrollada por la resistencia de calefaccin.

7.7. CONVERTIDORES C.C. A C.C. (CHOPPERS O RECORTADORES)


En algunas aplicaciones se dispone de una fuente de c.c. constante y se requiere una tensin de c.c. variable. Un ejemplo de ello es la alimentacin de motores de c.c. en los trenes metropolitanos. En estas instalaciones se procede primeramente a transformar la c.a. trifsica de la

618

MQUINAS ELCTRICAS

red de distribucin de media tensin en c.c. por medio de rectificadores trifsicos (o hexafsicos e incluso dodecafsicos). La c.c. obtenida es de amplitud constante y se aplica entre un cable sustentador denominado catenaria (terminal positivo) y el carril (terminal negativo). Para regular la velocidad de los motores de traccin es preciso transformar la c.c. de amplitud constante que recoge el pantgrafo del coche motor en tensin continua de amplitud media variable que se aplica al motor. En este epgrafe se estudian algunos circuitos de conmutacin que pueden realizar esta transformacin y que reciben el nombre genrico de choppers o troceadores, ya que utilizan interruptores estticos para recortar o trocear la tensin de alimentacin y conseguir un control del valor medio de la tensin de salida. Los diversos tipos de choppers difieren entre s segn sea el nmero de cuadrantes del diagrama tensin-corriente en los que son capaces de trabajar.

7.7.1. Chopper directo o reductor de tensin


La Figura 7.44 muestra el esquema elctrico de un chopper directo que es capaz de suministrar una tensin de c.c. ajustable Vo a una carga genrica (por ejemplo, un motor de c.c. definido por una R, una L y una f.c.e.m. E) a partir de una fuente de c.c. de amplitud constante \1" de tal modo que se cumple esta limitacin: V:-c ::s; V,(donde ~c representa el valor medio de la tensin continua de salida); SI es un interruptoresttico autoconmutado cuyo cierre o apertura (conduccin o no conduccin)depende de la seal de control que se aplica a su terminal de puerta G. En la prctica SI puede ser un transistor bipolar de unin, un MOSFET de potencia, un GTO e incluso un tiristor con conmutacin forzada; DI es un diodo rectificador. Supngase una condicin inicial para la cual la corriente de la carga es lmn Yque SI est apagado. Cuando en t = O se enciende SI, la tensin de alimentacin V,se conecta a la carga y por tanto Vo = Vs; tal como se muestra en la Figura 7.44b, el diodo DI queda polarizado en inverso y la intensidad circula por la carga. El tiempo durante el cual est cerrado SI se
"o
G

...
SI Chopper +

v,l
----.
i,

io

V,

DI

1:
Vo
L

el

j
al

+
E
d)

i-:
tON

,2T

Figura 7.44. Chopper directo o reductor de tensin.

CAPTULO

7.

ACCIONAMIENTOS ELCTRICOS

619

denomina tiempo de encendido tON" Durante este tiempo, y siempre que V, > E, se produce un aumento exponencial de la corriente de carga regido por la ecuacin:

. dio Vs, == R1 + L -dt + E


1)

(7.82)

cuya solucin, teniendo en cuenta que io(t) == I"'in' es de la forma:


lo t

. ()

==

",ine

-l/r

, + -R-

V-E

(1

-e

-l/r)

(7.83)

donde r == L1R es la constante de tiempo del circuito. En la Figura 7.44c se muestra la forma de onda de la corriente anterior, cuya solucin es vlida para O ~ t < tON' En el instante t == tON se bloquea el interruptor esttico SI aplicando una seal al electrodo de control G. La corriente habr alcanzado un valor mximo io(t == tON) == Irruix ' En ese momento, debido al apagado de SI, la corriente de la carga se cierra por el diodo volante DI cortocircuitando la tensin de la carga, por lo que vI)== O; esta situacin contina hasta t == T. La corriente en este perodo de tiempo responde a la solucin de la ecuacin:

==Rlo

dio + L- +E
dt

(7.84)

t == tON'

que tomando la condicin inicial io(t == O) == Imx Yredefiniendo el origen de tiempos a partir de da lugar a la siguiente solucin:

i (t) == I . e-liT o
max

E - (1 R

e-liT)

(7.85)

cuya expresin es vlida para tON ~ t < T. La corriente en la carga va disminuyendo con el tiempo y al final de este perodo tendr un valor io{l == tOFF == T - tON) == Imin, como as se indica en la Figura 7.44c. En t == T, el interruptor SI se vuelve a encender nuevamente y el ciclo se repite indefinidamente. En la Figura 7.44b se muestra la forma de onda de la tensin en la carga y en la Figura 7.44c la onda de corriente que se ha supuesto que su valor nunca pasa por cero, lo que supone un modo de funcionamiento continuo (y que se consigue en la prctica siempre que se cumpla que UR T). Se observa que la tensin de alimentacin de carga Vil es un recorte de la tensin de alimentacin de c.c. V, y de ah el nombre de este tipo de convertidores: choppers en ingls, recortadores o troceadores en castellano. El valor medio de la tensin en la carga se obtiene fcilmente de la Figura 7.44b, resultando ser: \/" ==

T tONV

== kV,

; donde k ==

tON

(7.86)

en el que el parmetro k define el ciclo de trabajo (duty cycle) del chopper. La corriente media en la carga (c viene expresada por:

t: == -R--

\/' -c -

(7.87)

donde \/" es el valor medio de la tensin en la carga expresada en (7.86). Como quiera adems que la corriente de alimentacin i, es igual a la corriente de carga io durante el

620

MQUINAS ELCTRICAS

perodo ON y cero durante el perodo OFF, como se muestra en la Figura 7.44d, su valor medio ser: (7.88) De este modo, y teniendo en cuenta adems (7.86), se puede escribir: T

1 k

(7.89)

que expresa el principio de conservacin de la energa del chopper (VJs = VeJa,)' es decir, la potencia suministrada por la fuente es igual a la potencia absorbida por la carga. De acuerdo con (7.89), el chopper directo se puede considerar como un transformador de corriente continua con un primario de tensin V, y corriente t, Y un secundario de tensin Vec y corriente (e' de tal modo que la relacin de transformacin, de acuerdo con (7.89), es igual a l/k. El perodo de las seales dibujadas en la Figura 7.44 es el inverso de la frecuencia (T = 1/f), donde f es la frecuencia en Hertz del interruptor recortador o troceador. Las frecuencias de conmutacin prcticas dependen del tipo y tamao del interruptor esttico utilizado. Los valores tpicos estn comprendidos en el rango de 0,2 a 1 kHz para dispositivos de alta corriente de apagado por puerta (GTO) o en el rango de 3 a 100 kHz para transistores de baja corriente. En algunos dispositivos conviene utilizar una frecuencia de conmutacin superior a 20 kHz para evitar el ruido audible que producen las fuerzas magnticas oscilantes del sistema. Sin embargo, hay que tener en cuenta que las prdidas de potencia en el interruptor esttico aumentan con la frecuencia de conmutacin.

EJEMPLO

DE APLICACIN

7.9

El circuito chopper de la Figura 7.44 tiene los siguientes parmetros: V, = 120 V; E = 30 V; L = 2 mH; R = 0,4 Q; T = 2,5 ms; tON = I ms. Calcular: a) corrientes mxima y mnima en la carga; b) tensin media y corriente media en la carga.

SOLUCIN a) La corriente io(t) en el perodo O :::; t < tONviene expresada por (7.83):

i (r) = I .e:" +
o
mm

V-E
_5 __

(1 _ e-th)

que para t = tONconduce a un valor Im,:

(a)
b) La corriente io(t) en el perodo tON:::;t < T viene expresada por (7.85):

E io(t) = Imax ' e-th - - (1 - e-ti!) R

CAPTULO

7. ACCIONAMIENTOS ELCTRICOS

621

que para t = T -

tON conduce

a un valor mn:

(h)
De las ecuaciones
(a)

(h)

se obtiene:

Imx

V (1 - e-~_!.- V, E) =t 1e
r

; t.

- 1 =;V (et~N -!.1


e' -

(e)

que teniendo en cuenta que el valor de la constante de tiempo es r = UR = 2/0,4 = 5 ms y los datos del enunciado, da lugar a:
I . = ---'max R

V, l-e-r

1-er'

_!. V

tON- -E.)

120 =0,4

l-e-s I _~ l-es

30 ) = 63 21 A 120 ' 30) =2739A 120 '

l.
mm

=-

R!.

V, (et~N _ 1 E) ---e -1'


t

V,

=-

120 1 ---0,4 ~ eS-1

(eL

Se observa que al ser la corriente mnima mayor que cero, la conduccin es continua. El lector puede deducir a partir de la segunda ecuacin (e) el tiempo t~N que se requerira para que mn llegara a valer cero, y que sera:
=

mn

=>

tON -1 V eT E) -!.-- - V ( e' - 1 s

que en nuestro caso da un valor:


t~N

= r . In 1+- E (e!T - 1)] = 5 . In [30 1+ [ V, lW

(e 5' -

2.5

1)] = 0,81 ms

e)

Como quiera que el tiempo de cierre real tON = 1 ms es superior al valor anterior de 0,81 ms, la conduccin ser continua. La tensin media en la carga viene dada por (7.86):
Vec = tON

V =s

1 120 = 48 voltios 2,5

por lo que la corriente media ser, segn (7.87): (.('= --Vc(' - E

48 - 30 . = --= 45 ampenos 0,4

Es evidente que si las curvas exponenciales de la Figura 7.44e se hubieran aproximado por lneas rectas, la corriente media en la carga se podra haber obtenido como media aritmtica entre las corrientes Imx e Imfn es decir:
!

63,21 + 27,39
2

45,3 amperios

622

MQUINAS ELCTRICAS

que prcticamente coincide con el valor anterior. La aproximacin ser tanto ms vlida cuanto mayor sea la constante de tiempo del circuito frente al perodo T (en nuestro caso, r = 5 ms y T = 2,5 ms, por lo que r = 2T).

7.7.2. Chopper inverso o elevador de tensin


En el chopper directo estudiado en el epgrafe anterior se transfiere energa desde la fuente V, hasta la carga (motor de c.c.), pero en la prctica es tambin importante poder efectuar el proceso inverso, es decir, transferir energa desde una fuente de c.c. de poca tensin a otra de mayor tensin. Por ejemplo, en los trenes metropolitanos es de gran inters devolver energa a la red de c.c. por parte de los motores de traccin, lo que se traduce en un frenado en los mismos por recuperacin de energa o regenerativo. La Figura 7.45 muestra un circuito que es capaz de funcionar de este modo. Los sentidos de referencia de las tensiones y corrientes se han elegido igual que en el caso del chopper directo (vase Fig. 7.44). Debido a que la f.c.e.rn. E del motor de la Figura 7.45a es inferior a la tensin de alimentacin V" no sucede nada hasta que no se cierra el interruptor esttico 52. Al entrar en conduccin 52, se cortocircuita la carga y la tensin en la misma es igual a cero, "o = O. La f.c.e.rn. E hace que la corriente en la carga io empiece a aumentar en sentido negativo, almacenando energa magntica en la inductancia L del circuito durante el tiempo de conduccin (O -::;; t < tON)' La ecuacin de funcionamiento en este perodo es:
0=

.
R10

dio +L- +E
dt

(7.90)

Cuando 52 pasa al estado de bloqueo en t = tON' la f.e.m. inducida en la inductancia L se suma a la f.c.e.m. E, lo que hace que la corriente de la carga contine circulando a travs del diodo D2 hacia la fuente V,. Durante el perodo de bloqueo (tON -::;; t < T), la ecuacin de funcionamiento del circuito es:
11 Ri ....., _

dio

.....;

non

Considerando como instante de inicio de funcionamiento el momento en el que io devuelve corriente a la red a travs del diodo D2, las curvas de tensin y corriente en la carga son las

D2

v,

c_--.
52

-- :
+
io

vo

T h)

i.\

v,

D2

S2

va

Chopper

io

+
E

al

Figura 7.45. Chopper inverso o elevador de tensin.

CAPTULO

7.

ACCIONAMIENTOS ELCTRICOS

623

mostradas en la Figura 7.45b y c. Como quiera que en el epgrafe siguiente combinaremos este chopper inverso con otro directo para formar un chopper de dos cuadrantes, se ha designado en las Figura 7.45b Y e con tON el intervalo de tiempo correspondiente a la conduccin del diodo D2 (y no de cierre del interruptor esttico S2). Durante este intervalo la difdt en la carga es positiva, como as se deduce de (7.91), y el mdulo de la corriente negativa io decrece. En t = tON se cierra S2 y la di.Jd: en la carga se hace negativa de acuerdo con (7.90), por lo que aumenta el valor negativo de la corriente de carga (vase Fig. 7.45c). Cuando el diodo D2 conduce, la tensin en la carga Vo = Vs y cuando se cierra S21a tensin Vo = 0, como as se seala en la Figura 7.45b. El valor medio de la tensin en la carga, de acuerdo con esta figura, es:

V. = -T' V = k Vs' . donde k = - T


ce

tON

tON

(7.92)

y el valor medio de la corriente en la carga es:

t; = --R-

Vec -E

(7.93)

Debido a nuestra eleccin de los sentidos de las corrientes y del tiempo, las expresiones anteriores son idnticas a las que se obtenan en el caso del chopper directo [comparar (7.92) y (7.93) con (7.86) y (7.87)]. El valor medio de la corriente en la carga viene expresado tambin por (7.88), es decir: (7.94)

7.7.3. Choppers de dos y cuatro cuadrantes


En la prctica se necesita una combinacin de choppers reductores y elevadores. Un esquema elctrico que responde a esta doble combinacin es el que se muestra en la Figura 7.46. Este circuito es capaz de suministrar un valor de la tensin media de la carga Vee positivo, siempre
SI
Vo G

D2 ~
~IO

s2.~
io

-+~llll~

-r~llll~

~llW~_'

+ R S2
G

lo
+ E

02 SI

DI S2

1
V

I
a)

Figura 7.46.

Chopper de dos cuadrantes.

624

MQUINAS ELCTRICAS

que la corriente de la carga pueda circular en ambos sentidos, con una transicin por cero. Esto significa que el sistema puede funcionar en los dos primeros cuadrantes del diagrama tensin-corriente (vo - in)' tal como se seala en la Figura 7.46. En este chopper, los interruptores estticos SI y S2 se cierran o encienden alternativamente, tal como se muestra en la Figura 7.46b, con el pequeo intervalo de apagado del interruptor. Cuando funciona con una corriente io positiva, el interruptor SI y el diodo DI actan como un chopper directo o reductor. Ni SI ni DI pueden funcionar con este sentido de la corriente. Similarmente, cuando la corriente io es negativa, el interruptor S2 y el diodo D2 trabajan como un chopper inverso o elevador, independientemente de S I Y D l. La Figura 7.46c muestra una forma de onda tpica de la corriente durante la transicin de la corriente de valores positivos a negativos. Cuando la corriente es positiva, sta se cierra a travs de SI Y DI (funcionamiento en el primer cuadrante), mientras que cuando la corriente es negativa, sta circula a travs de S2 y D2 (funcionamiento en el segundo cuadrante). Las relaciones de las tensiones y corrientes para este chopper de dos cuadrantes son las mismas que las de los choppers directo e inverso, a saber:
Vce

= tT

ON

Vs

= kV .\"

'(T

. J

Vee - E =-R

(7.95)

En aquellos casos en los que se necesita producir tanto tensiones como corrientes en ambos sentidos, es necesario utilizar un chopper de cuatro cuadrantes. Un ejemplo prctico podra ser el control de un brazo de robot accionado por un motor de c.c. en el que se necesita una corriente positiva para el perodo de aceleracin y una corriente negativa para el perodo de frenado. Es preciso disponer tambin de una tensin positiva para regular la velocidad en sentido directo o positivo y una tensin negativa para invertir la velocidad. Se puede lograr este funcionamiento en cuatro cuadrantes utilizando dos sistemas como el sealado en la Figura 7.46, pero conectado segn se muestra en la Figura 7.47. La combinacin SI, DI, S2 y D2 se utiliza para el funcionamiento en los cuadrantes I y 2, mientras que se requieren los S3, D3, S4 y D4 para el funcionamiento en los cuadrantes 3 y 4. Igualmente el chopper de cuatro cuadrantes se utiliza con profusin en la regulacin de velocidad de motores elctricos de c.c. empleados en la traccin elctrica. El motor que lleva la traccin puede funcionar en ambos
i,~---+

SI

D2

S3

.... G

D4

E
V,

+
DI

Vo
D3 S4

52

---G

-G

Figura 7.47. Chopper de cuatrocuadrantes.

CAPTULO

7.

ACCIONAMIENTOS ELCTRICOS

625

sentidosen los cuadrantes 1 y 3, empleando los cuadrantes 2 y 4 para frenado regenerativo o por acumulacin de energa.
EJEMPLO DE APLICACIN 7.10

Se dispone de un chopper de dos cuadrantes como el indicado en la Figura 7.46 para controlar la velocidad de un motor serie de c.c.; la constante kll> del motor vale 0,1 V/r.p.m. (la fe.m. del motor viene expresada por E = kll>n); la resistencia del inducido del motor es de 0,2 ohmios, siendo la inductancia lo suficientemente elevada como para considerar que la corriente de carga es plana y de amplitud constante; lafrecuencia del chopper es constante y vale /.000 Hz y la tensin de alimentacin es de 100 V. a) El chopper SI y el diodo DI se emplean para trabajar la mquina como motor a una velocidad de 500 r.p.m. y con una corriente de carga media de lOO A (que se supone de valor constante, es decir, libre de rizado). Calcular el tiempo tONdel chopper, la potencia desarrollada por el motor y absorbida de la red. b) El chopper S2 y el diodo S2 se utilizan para que la mquina funcione en rgimen de frenado regenerativo a la misma velocidad y con una corriente de -100 A. Calcular el tiempo tONdel chopper, la potencia desarrollada por el motor y la que se tj.evuelve a la red.

SOLUCIN
a)

La tensin media Vee en bornes del motor vale:

= 0,1 . 500 + 0,2' 100 = 70 voltios y teniendo en cuenta (7.95) y que T vale T = 11f = 111.000= 1O~3s =
V
cr

Vee = E + RIce

1 ms, resulta:

= -tON V T s

=>

70

tON =100 T

=> -

tON = 0,7 T

=>

tON= 0,7 . 1 = 0,7 ms

por lo que la corriente media absorbida de la red, teniendo en cuenta (7.94), vale:
Is tON =. 100 = 70 A T Ice = O7 '

La potencia mecnica desarrollada por el motor vale: Pm = E . lec = (0,1 500) . 100 = 5.000 W
y la potencia absorbida de la red es: P, = V, . 1, = 100' 70 = 7.000 W

b)

Tngase en cuenta que en la resistencia del inducido se disipa una potencia Pcu = R . I(~.= 0,2 . 1002 = 2.000 W, y de este modo la mquina absorbe de la red 7.000 W, de los cuales se pierden por efecto Joule 2.000 W y quedan 5.000 W para desarrollar potencia mecnica el motor. La tensin media Vec en bornes del motor (rgimen generador) vale: Vec = E - RIce = 0,1 . 500 - 0,2 . 100 = 30 voltios y teniendo en cuenta (7.95) resulta:
V = tONV
ce

=>

30 = tOTN 100

=>

tOTN = 0,3

=>

tON= 0,3 . 1 = 0,3 ms

626

MQUINAS ELCTRICAS

Debe destacarse que el tiempo anterior representa ahora el tiempo de conduccin del diodo D2, lo que quiere decir que el tiempo de conduccin del chopper S2 es T - tON = l - 0,3 = 0,7 ms. La corriente media entregada a la red, teniendo en cuenta (7.94), vale: 1s
tON =T I ce = O ' 3 . 100 = 30 A

La potencia desarrollada por el motor es ahora una potencia elctrica generada: Pe

= E
P,

Icc = (0,1 500) . (100) = 5.000 W

y la potencia elctrica que entrega a la red vale:

V, '1,

100 30 = 3.000 W

Tngase en cuenta que en la resistencia del inducido se disipa, al igual que antes, una potencia P,." = R . I}c = 0,2 . 1002 = 2.000 W, y de este modo la mquina en rgimen de frenado produce o genera una potencia elctrica de 5.000 W, de los cuales se pierden por efecto Joule 2.000 W y quedan 3.000 W para entregar a la red.

7.8.

CONVERTIDORES C.C. A C.A. (ONDULADORES O INVERSORES)

El propsito de un ondulador (onduleur en francs) o inversor (inverter en ingls) es convertir la energa elctrica de una fuente de tensin o corriente continua en una salida de c.a. de frecuencia variable, con tensin o corriente ajustable. La tensin obtenida tiene la forma de una onda peridica que est formada por tramos rectangulares o escalonados procedentes de la tensin de c.c. de alimentacin, lo cual es suficiente para aplicaciones de mediana y pequea potencia. Para potencias elevadas es preciso mejorar la onda para que se acerque ms a la forma sinusoidal, lo que se consigue en la prctica utilizando tcnicas de conmutacin dentro de los interruptores estticos. Los inversores pueden ser monofsicos o trifsicos y con salida en tensin o en corriente. En este epgrafe solamente se estudian los inversores de tensin. Los inversores estticos se utilizan en una variedad de aplicaciones como: calentamiento por induccin, regulacin de velocidad de motores de c.a. (tanto sncronos como asncronos). En sistemas de alimentacin ininterrumpida o SAl (UPS en ingls: uninterrumpible power supplies) para ordenadores y otros. El sistema de alimentacin de entrada puede ser una batera de acumuladores, clulas solares, pilas de combustible u otras fuentes de C.C., como por ejemplo la salida de un rectificador fijo o controlado.

7.8.1. Inversores monofsicos


En la Figura 7.48a se muestra un inversor monofsico media onda que servir para comprender el principio de funcionamiento de los mismos. La alimentacin es una fuente de c.c. doble con toma central y tensiones VJ2 a cada lado. Los interruptores estticos SI y S2 pueden ser tiristores, OTO, transistores bipolares de unin (BJT) o MOSFET. Los diodos ID 1 Y D2 se utilizan como diodos de libre circulacin para devolver la energa reactiva de la carga a la red de c.c. en los instantes en que estn abiertos los interruptores estticos Sl Y S2.

CAPTULO

7.

ACCIONAMIENTOS ELCTRICOS

627

v,
2"

a)

e
DI Figura 7.48. SI D2 S2 DI SI D2 S2 DI SI D2 S2

Inversor monofsico media onda.

Los interruptores estticos se abren alternativamente cada T/2 segundos, siendo T el perodo de la seal resultante y cuya inversa! = liT es la frecuencia de la tensin generada de c.a. En las Figuras 7.48b y e se muestran los impulsos de disparo que se envan a las puertas de los interruptores estticos, que se mantienen durante medio ciclo (debe tenerse en cuenta que antes de aplicar los impulsos de corriente i~2 al interruptor S2 debe haber finalizado la secuencia de los impulsos i~1 sobre SI para evitar la coincidencia en la conduccin de SI y S2, que provocara un cortocircuito en la red de alimentacin). Entre O :s;; t < TI2 conduce el interruptor esttico S 1 debido a la aplicacin de los impulsos MI' de este modo la tensin en la carga es Vo = VJ2. Durante el siguiente semiperiodo TI2 :s;; t < T conduce S2 (debido a los impulsos ig2), lo que da lugar a una tensin de carga vlJ = - V/2. Esta secuencia se va repitiendo continuamente, por lo que la tensin en la carga tendr la forma de onda rectangular mostrada en la Figura 7.48d. Si la carga es inductiva, como es el caso de la Figura 7.48a, la corriente io se retrasa respecto de la tensin de salida Vo como se seala en la Figura 7.48e y corresponden a ciclos de carga y descarga del circuito R-L. Obsrvese que en los instantes previos a t = TI2 (punto A de la Fig. 7.48e), el interruptor SI est cerrado y se tiene una corriente io positiva que en t = TI2 es igual a +1. En ese momento se abre SI y se cierra S2; como la corriente ilJ no puede ser discontinua, deber seguir siendo positiva durante un cierto tiempo pese a la inversin de signo de Vo (que ha pasado a valer - V/2), y es por ello que en el tramo AB la corriente de retorno la debe suministrar el diodo D2. En el punto B la corriente io = O, por lo que comienza a conducir S2 hasta el punto C, que corresponde a una corriente io = -L En ese instante, que corresponde a t = T, se abre S2 y se cierra SI y la corriente de retorno la suministra el diodo DI, desde e hasta D, y desde este punto hasta E, la corriente es positiva y la suministra SI, y as sucesivamente. En definitiva, los diodos volantes de realimentacin conducen cuando la tensin y la corriente en la carga tienen polaridades opuestas. Se puede demostrar que el desarrollo en serie de Fourier de la onda de tensin de la Figura 7.48d del inversor monofsico media onda es de la forma:

Vo

=
11::::

L
1.3.5 ...

'2V _, sen ncot nn

(7.96)

628

MQUINAS ELCTRICAS

donde co = 2nf es la pulsacin de la tensin alterna producida por el inversor y f = liT es la frecuencia. En la expresin anterior para n = 1 se obtiene el valor de la componente fundamental, cuyo valor eficaz es:
VI =

j2n

2V,

(7.97)

En la Figura 7.49a se muestra otro tipo de inversor monofsico, que corresponde al inversor en puente. Los interruptores estticos SI y S2 conducen durante el primer semiperodo y los S3 y S4 conducen durante el segundo semiperodo. En la Figura 7.49b se muestra el tipo de onda que se obtiene entre los terminales A y N; al cerrar SI la tensin de VAN es la tensin del generador V" mientras que cuando conduce S41a tensin VAN vale cero. De un modo anlogo se obtiene la tensin entre los terminales B y N, cuya onda se muestra en la Figura 7.49c. La tensin entre los terminales de salida A y B vale: Vo = VAN - VBN' cuya onda se ha dibujado en la Figura 7.49d. Es una onda peridica cuya amplitud mxima coincide con la tensin de alimentacin V,. En el caso de que la carga sea resistiva la seal de corriente coincidir en su forma con la onda de tensin, pero en el caso de que la carga sea inductiva la corriente se retrasar de la tensin y, como ocurra en el caso de la Figura 7.48, habr perodos de conduccin de los diodos volantes DI, D2, D3 Y D4 para devolver la energa inductiva a la red de c.c. Se puede demostrar que el desarrollo en serie de Fourier de la onda de tensin de la Figura 7.49d del inversor monofsico media onda es de la forma:
"o =
L..
n= 1.3.5...

;,

ntt

4Vs

sen non

(7.98)

que corresponde a un valor doble que el del inversor monofsico media onda. Como en el caso anterior, la frecuencia del inversor depende de la frecuencia de conmutacin de los interruptores estticos.

7.8.2.

Inversor trifsico en puente

Este inversor produce una tensin alterna trifsica a partir de una alimentacin de c.c. El esquema correspondiente es el mostrado en la Figura 7.50. El funcionamiento de este inversor

DI e) +

v,
D4 ti)

Figura 7.49. Inversor monofsico en puente.

CAPTULO

7.

ACCIONAMIENTOS ELCTRICOS

629

Figura 7.50. Inversortrifsicoen puente. es esencialmente el mismo que el monofsico en puente comentado en el epgrafe anterior; es como si al tipo monofsico en puente se le hubiera aadido otra rama, de tal forma que los terminales R, S y T se conectan alternativamente (por medio de los interruptores estticos) a los terminales positivo y negativo de la fuente de c.c., dando lugar a la obtencin de una tensin trifsica siempre que los impulsos de disparo de los interruptores se desfasen 120 entre s y entren en conduccin siguiendo un determinado orden cclico. En la Figura 7.50 la numeracin de cada interruptor corresponde al orden de estas secuencias de disparo para obtener un ciclo completo de la tensin de salida. El control de la frecuencia de esta tensin viene definido por la frecuencia de disparo de los interruptores estticos. Tomando como referencia de tensiones el borne N negativo de la fuente y suponiendo una conmutacin instantnea se obtienen las ondas de las tensiones VRN' VSN Y VTN' mostradas en la Figura 7.51, que tienen forma rectangular con un valor mximo igual a la tensin de la fuente Vs. Las tensiones compuestas de salida se obtienen mediante las ecuaciones siguientes: (7.99) En la Figura 7.51d se ha dibujado nicamente la primera tensin compuesta anterior, cuyo desarrollo en serie de Fourier es de la forma: vRS = que al desarrollar nos da: v
= -RS

L n=1.3.5

ce

4V,
nti

cos - sen non 6

nit

(7.100)

213 V (sen ost - -l sen 5wt - -1 sen 7wt + - 1 sen llwt


t:
s

11

+ ...)

(7.101)

Es decir, las tensiones compuestas no tienen armnicos de orden tres ni mltiplos de tres, lo cual era de prever puesto que estos armnicos de las tensiones simples VRN' VSN Y VTN' van en fase entre s, es decir, son homopolares, y al hacer las diferencias de tensiones sealadas en (7.99) estas componentes armnicas se anulan. Se observa adems que el primer armnico que aparece es el de orden 5 y tiene un valor que es el 20 por 100 del fundamental.

7.8.3. Control de la tensin de salida de un inversor


En muchas aplicaciones industriales es necesario regular la tensin de salida de un inversor; por ejemplo, para controlar la velocidad de giro de un motor asncrono o de induccin se

630

MQUINAS ELCTRICAS

S4

SI

a) wt

S6

S3

S6

S3

S6

b)

wt

wt
d)
WI

IIIIIIIIIIIIIIII
2345612345612345 Secuencia de los impulsos de disparo

e)

Figura 7.51. Tensiones del inversor trifsico en puente.

necesita variar la tensin de alimentacin e incluso la frecuencia (manteniendo entonces el cociente V/fconstante para conseguir un flujo de entrehierro constante y evitar de este modo saturaciones en el ncleo de hierro). Es indudable que se puede regular la tensin de salida del inversor ajustando la tensin de c.c. de entrada, pero al ser sta una accin externa al inversor no se va a desarrollar aqu; en nuestro caso solamente se va a estudiar el control de la tensin actuando sobre los instantes de conmutacin de los interruptores estticos, para lograr ajustar la anchura de los impulsos rectangulares de la seal de salida, procedimiento que se denomina en ingls pulse width-modulated (PWM) o modulacin por la anchura de los impulsos, y que pueden clasificarse en tres categoras, a saber: a) modulacin de un solo impulso; b) modulacin de impulsos mltiples, y e) modulacin senoidal de impulsos.

a) Modulacin de un solo impulso


En este tipo de control se produce un solo impulso rectangular por semiciclo cuya anchura se puede ajustar actuando sobre los instantes de encendido de los interruptores estticos del inversor. Se dispone para ello, y como se muestra en la Figura 7.52, de dos tensiones de control, una de las cuales es una seal de referencia de forma rectangular con un valor de pico VR y frecuencia f igual a la frecuencia que debe producir el inversor; esta tensin tiene la misin de onda moduladora. La otra tensin se comporta como portadora y es una onda triangular con un valor de pico VT y frecuencia fT. Los instantes de encendido de los interruptores estticos vienen definidos por los puntos de interseccin de las dos ondas anteriores. La anchura de los impulsos de salida corresponde

CAPTULO

7. ACCIONAMIENTOS ELCTRICOS

631

a) Tensiones de control

O=wt

11,

V,

O 11- 15 -2-

15 nI2 11+15 -2- 11

b) Tensin de salida

311/2 .5 211 wt

del inversor

Figura 7.52.

Modulacin de un solo impulso.

al perodo de tiempo durante el cual se cumple la desigualdad vr(t) < vR(t) en el semiperodo positivo y al contrario en los serniciclos negativos. Los dos parmetros de:control de la tensin producida son: ndice de amplitud: M

VR Vr

ndice de frecuencia: N

=.&_
f

(7.102)

El ndice de amplitud se emplea para ajustar la amplitud de la tensin que produce el inversor; el ndice de frecuencia representa el nmero de impulsos rectangulares que contiene la onda en cada semiciclo. En este caso, como solamente se quiere producir un solo impulso por semiciclo, el ndice N es igual al, por lo que la frecuencia de la onda triangular coincide con la de la onda rectangular. En la Figura 7.52b se muestra la forma escalonada de la tensin de salida, cuya tensin mxima es + V, para el impulso positivo y - V, para el impulso negativo, siendo V, la tensin de c.c. de alimentacin del inversor. Para comprender cmo la amplitud de la seal rectangular de referencia controla la anchura de los impulsos generados, obsrvese que la ecuacin de la recta AB del primer tramo de la onda triangular es de la forma:
VeO) = --

2Vr
ti

8 + Vr

(7.103)

El punto e correspondiente a la interseccin de las ondas triangular y rectangular se deduce de la ecuacin anterior haciendo V(8c) = VR, resultando ser:

8e = ~ (1 _ V 2 Vr
IDBI

R)

(7.104)

Si se denomina b la anchura del impulso, la distancia IDBI es igual a la mitad de la anchura de un impulso, de donde se deduce:

= "2 = "2 -"2

tt

n(

1 - Vr

VR)

="2

tt VR Vr

=>

s =n

VR Vr

(7.105)

La ecuacin anterior indica que la anchura del impulso es funcin lineal de la tensin de la seal rectangular o de referencia VR, o en definitiva, teniendo en cuenta (7.102), la anchura

632

MQUINAS ELCTRICAS

del impulso es proporcional al ndice de amplitud M; de este modo, si M = 1, es decir, VR = Vr> el impulso de salida del inversor tiene una anchura de n radianes (anlogo a la salida del inversor clsico sin control de la Fig. 7.49), mientras que si M = O, es decir, VR = O, la anchura del impulso es cero. De esta forma, variando el ndice de amplitud entre O y l se regula el ancho de los impulsos de salida del inversor entre O y n, lo que se traduce en una variacin de la tensin alterna del mismo, como claramente indica el desarrollo en serie de Fourier de la onda de la Figura 7.52b, que responde a la ecuacin:
Vo

L
n=I,3.5 ...

4V
_s

nb sen - sen nwt

(7.106)

ntt

Hay que tener en cuenta, para intentar conseguir bajas la tensin de salida. Es fcil cociente entre las amplitudes

sin embargo, que si el ngulo anterior b se reduce demasiado tensiones, aumenta enormemente el contenido de armnicos de demostrar lo anterior a partir de la ecuacin (7.106), ya que el del tercer armnico frente al primero o fundamental vale: 3b 2 sen3b 2 2 3 - 4 sen 2b 2 3

4\1, -senb3 = 3n bl 4\1, -senn

s
2

s 3 sen-

(7.107)

Es decir, conforme disminuye b, la amplitud del tercer armnico es ms elevada respecto de la amplitud del armnico fundamental.

b)

Modulacin de impulsos mltiples

Una forma de reducir el contenido de armnicos del inversor anterior es obtener varios impulsos de salida en cada semiciclo; para ello es preciso aumentar la frecuencia de la onda portadora triangular frente a la moduladora rectangular, o de otro modo, segn (7.102), debe aumentarse el ndice de frecuencia N. En la Figura 7.53 se ha considerado un ndice de frecuencia
I

fr

Onda rectangular de referencia (onda moduladora)


a) Tensiones de control
0)(

Onda triangular (onda portadora) Impulsos rectangulares


mI

b) Tensin de salida

del inversor

2n

Figura 7.53.

Modulacin de impulsos mltiples.

CAPTULO

7. ACCIONAMIENTOS ELCTRICOS

633

igual a 3, es decir, la onda triangular tiene triple frecuencia que la rectangular, por lo que se obtienen tres impulsos de salida por semiciclo. Si se consideran N impulsos en cada semiciclo de anchura 15, la distancia entre impulsos es n/N con una anchura de cada impulso 6, que puede demostrarse de una forma similar a (7.105), que vale:

() = - - = N V

tt VR

M ya que M

=-

VR Vr

(7.108)

De este modo con el ndice de modulacin M se gobierna la anchura de los impulsos y al variar M entre Oy I la anchura vara entre O y ti/N, Se puede demostrar (vase ejemplo de aplicacin 7.11) que el desarrollo en serie de Fourier de la seal de salida de un inversor con modulacin de impulsos mltiples es de la forma:
Vo

L
11=1.3,5,...

4V,
ntt

sen 2

nl5 N-l

sen

nn(2i+ 2N

1)} sen non


1M M

(7.109)

i=O

donde N indica el nmero de impulsos en cada semiciclo y 15la anchura de cada impulso. El valor eficaz de la tensin de e.a. obtenida es: - 1 n/N

fiv+~ V,2 d(wt)


__:r__~
2N

= V, f:15 .

= V,

(7.110)

e) Modulacin senoidal de los impulsos


Este mtodo utiliza varios impulsos en cada semiciclo, pero en vez de ser todos del mismo ancho, como en el caso anterior, su anchura se hace variar de forma senoidal, de tal modo que los impulsos ms cercanos al pico de la onda senoidal son ms anchos y los cercanos al paso por cero de la onda senoidal son ms estrechos, Este tipo de modulacin tiene un contenido de armnicos muy reducido y por este motivo se ha impuesto en la mayora de los inversores de gran potencia que se emplean en el control de velocidad de motores asncronos. Para conseguir este tipo de modulacin es evidente que la onda moduladora de referencia debe ser una sinusoide con un valor de pico VR y frecuencia f igual a la frecuencia que debe producir el inversor, lo que corresponde a una expresin instantnea vR(t)= VR sen cot (con t = 2nf). La tensin portadora ser, al igual que en el caso anterior, una serie de ondas triangulares con un valor de pico Vr y frecuenciaj., De este modo, de acuerdo con (7.108) la anchura del impulso valdr:
t: VR n VR sen on 6 = - - = - -'-'----

N Vr

Vr

(7.111)

Es decir, el ancho de cada impulso depende de la posicin del impulso dentro de la onda senoidal; para aplicar la expresin anterior a un caso concreto, se debe considerar el valor del argumento wt para el tiempo correspondiente al centro de cada impulso. Generalmente en los inversores trifsicos el valor del ndice de frecuencia N suele ser 6, aunque es vlido cualquier nmero mltiplo de 3 (es decir, la onda triangular tiene una frecuencia mltiplo de 3 de la onda senoidal). Para ver la forma en la que se configuran los impulsos, en la Figura 7.54 se han mostrado tres composiciones diferentes de modulacin senoidal con N = 6 en todas ellas. En cada semiperodo existen 6 ondas triangulares positivas para el semiciclo positivo y otras 6 negativas para el semiciclo negativo, Los puntos de

634

MQUINAS ELCTRICAS

~nda

tnangUl~Onda

sinusoidal de referencIa

If~IIVIIVIIV!1~
a)

\ ,~

Tens~~x~:~ahda

./

b)

Vo
...

_--

..
-v,

....
I\A

--

..
;:~
!::

\ 1\ 1\ 1\ 1\ 1\ 1\ 1\

JtJrHttJ 'V

1I
1

Vi Vi Vi rl-\JjJ_

1\ 1\)

e)

Vo

'

Tensin de salida mitad y frecuencia mitad

v, .

,,'1""'''1''"''._,,'1''"',,'''''.

r--r-..

Figura 7.54.

Modulacin senoidal de la anchura de los impulsos,

interseccin de la serie de ondas triangulares con la onda senoidal determinan los instantes de encendido de los interruptores estticos del inversor, En el caso a) se ha considerado un ndice de amplitud igual a l (es decir, la onda senoidal de referencia tiene el mismo valor de pico que las ondas triangulares) y por ello la tensin de salida mostrada en la parte inferior tiene impulsos anchos y la componente fundamental de esta tensin representa la tensin ms elevada que puede producir el inversor. Los impulsos de la tensin de salida varan entre V" siendo sta el valor de la tensin continua de la red de alimentacin del inversor. En el caso b) la seal senoidal de referencia se ha reducido a la

CAPTULO

7.

ACCiONAMIENTOS ELCTRICOS

635

mitad, por lo que se consigue una onda fundamental en el inversor cuya amplitud es la mitad que en el caso anterior. En el caso e) se ha considerado una onda senoidal de referencia de amplitud mitad y de frecuencia mitad (doble perodo), y es por ello que el nmero de impulsos en cada semiperodo se ha duplicado, dando lugar a una componente fundamental de mitad de amplitud y frecuencia. Evidentemente, en los inversores trifsicos es necesario emplear tres ondas moduladoras senoidales desfasadas 120 para conseguir los instantes de disparo de los 6 interruptores estticos que existen en el inversor.
0

EJEMPLO DE APLICACIN 7.11


Determinar eLdesarrollo en serie de Fourier de Latensin de salida de un inversor con modulacin de impulsos mltiples del tipo sealado en LaFigura 7.53; considerar N impulsos de anchura 3 por semiciclo y una tensin de e.e. de alimentacin del inversor de vaLor V,. Si eL nmero de impuLsospor semiciclo es igual a 5 y eLndice de moduLacin de amplitud vaLeM = 0,2, calcuLar las magnitudes de Los once primeros armnicos en funcin de la tensin de alimentacin.

SOLUCIN El desarrollo en serie de Fourier viene expresado por:


a x oc v(t) = _2 + '" a cos non + '" 2 L.II L.nb sen non
I 1

donde los valores de los coeficientes son:


ao 4 =-

To

iT/2 v(t) dt = -2 in v(O) dO


no
4 bn = -

; an

4 =-

Tu

iT/2 v(t) cos non dt = -2 in v(O) cos nO dO


no
o
ll

Ton

iT/2 v(t) sen non dt = -2 in v(O) sen nO dO

Por el tipo de simetra de la onda de la Figura 7.53, se anulan los trminos ao, a Y los trminos pares b, Y los valores de los otros coeficientes b; son:
2V h" = --:;;

[fiN+~
_1'__~
2N2

sen nO dO +

f~+~
:l1r__~
2N2

sen nO dO + ~_~ sen nO dO +


2N2

f~+~
]

7Tr_+~
'N ') 7n
(j ---

- - sen nO dO +

f.~+~
9n
2N2

2N .,

2N2

---

- sen ne de + '"

donde se ha tenido en cuenta que los centros de los impulsos se producen para los valores de () igual a: n/2N, 3n/2N, 5rr/2N, 7n/2N, etc. Resolviendo la primera integral anterior se obtiene:
b,

= 2V, f2:\;+:;
ti _1'__~
2N2

ncl

sen ne de = 2V, [_ cos nO


tt

]n

2+:
_1'__~
2N2

= 2V,
ttn

[cos n (~_~)
2N

_ cos n

] (~+~) =
2N 2

=-

4Vs
nl

n3 ttn sen - sen2 2N

636

MQUINAS ELCTRICAS

y de un modo anlogo se obtienen las dems, resultando:


b = _, sen "nn 2

4V

ns ":'
i=O

nn(2i+l) sen ---2N

por lo que el desarrollo en serie de Fourier es de la forma:


Vo

'le

{4V,

n=I,3,5,...

ttn

sen 2

-s: L
i=O

sen

nn(2i+l)} 2N

sen rut

Para N = 5 los coeficientes b, son de la forma:


4V, n(j [nn b = _ sen sen n

ttn

10

+ sen -

3nn

10

+ sen -

Sttn

10

+ sen -

Tnn

10

+ sen -

9nnJ

10

n n n El valor de (jes, segn (7.108): () = - M = - 0,2 = -, y el primer coeficiente de la serie tiene N 5 25 un valor:
b1

=4 _ V,. sen -tt


n

[n

50

3n 5n Ttt sen - + sen - + sen - + sen - sen 10 10 10 10 10

9nJ = 0,2587 V
' b, 1 = -0,2388V,

y operando los dems, se obtienen los valores siguientes:


b} = 0,0983V, ; b5 = O,0787V, ; b., = 0,0957V, ; b = 0,2453V,

7.9.

CONVERTIDORES

C.A. A C.A. (CICLOCONVERTIDORES)

Un cicloconvertidor es un sistema de interruptores estticos que es capaz de convertir una seal de c.a. de tensin y frecuencia constante (como la de la red elctrica) en una tensin de c.a. de tensin y frecuencia variable, pero sin pasar por ninguna etapa intermedia de c.c. Se puede considerar que un cicloconvertidor monofsico est formado por dos convertidores conectados en oposicin, tal como se muestra en la Figura 7.55a. Los convertidores P y N son rectificadores controlados positivos y negativos, respectivamente. Si solamente trabaja el convertidor P, la tensin de salida es positiva, y de modo anlogo, si solamente trabaja el convertidor N, la tensin ser negativa. La frecuencia con que se suministran los impulsos de disparo a las puertas de los tiristores fija la frecuencia de salida del cicloconvertidor. Supngase que los ngulos de encendido de los tiristores de ambos puentes sean iguales entre s y de valor cero, es decir, :.t.p = fI'N = 0 y con una frecuencia 1/3 de la correspondiente de la red; entonces el convertidor P funcionar durante 1,5 ciclos y el convertidor N funcionar durante otros 1,5ciclos, dando lugar a la forma de onda mostrada en la Figura 7.55c. Si el ngulo de encendido de los tiristores es otro, por ejemplo, r:t.p = r:t.N = 60 y con la misma frecuencia anterior, la forma de onda ser la mostrada en la Figura 7.55d. Indudablemente, las Figuras 7.55c y d son tensiones alternas que se obtienen en la carga de frecuencia 1/3 de la correspondiente a la red de alimentacin y cuyo valor eficaz depende de los ngulos de encendido de los convertidores. Ambas ondas tienen un contenido de armnicos importante, que se puede reducir si se van modificando gradualmente los ngulos de encendido de los tiristores dentro de cada semiciclo de la seal de salida e incluso utilizando una red de c.a. trifsica para alimentar dos convertidores trifsicos.

CAPTULO 7.

ACCIONAMIENTOS ELCTRICOS

637

Convertidor P

Convertidor

N)6

1\ 1\ 1\ 1\ b). I\JVV V VWf


e)

'{e2

V, ]
1>---1--0

&foo " rJ.p=~'~\I"


Vo

~~/)f\f\

VVV

on

a)

mi

Figura 7.55.

Principio de funcionamiento de un cicloconvertidor monofsico.

En la Figura 7.56a se muestra un cicloconvertidor que partiendo de una red trifsica alimenta una carga monofsica. Los dos convertidores, P y N, son trifsicos; el primero de ellos se utiliza para generar la parte positiva de la tensin de la carga y el otro para conformar la parte negativa de la misma. Sabemos del epgrafe 7.5 que la tensin media de salida de un rectificador controlado vara con el coseno del ngulo de encendido de sus tiristores [vase expresiones (7.48b) y (7.64b)], Yes por ello que si se van cambiando los ngulos de encendido de un modo adecuado se puede llegar a construir una tensin media de salida que se acerque a la forma sinusoidal. En la Figura 7.56b se muestra la tensin de salida obtenida cuando los ngulos de encendido se modifican de acuerdo con los valores mostrados en su parte inferior (los ngulos de encendido superiores a 90 significan los perodos de conduccin del convertidor negativo N; realmente, respecto a su propia referencia, los ngulos de encendido son IXN = 180 Cl.p). Si se desea transformar una alimentacin trifsica con salida tambin trifsica es preciso utilizar tres grupos convertidores (uno por cada fase) como los indicados en la Figura 7.56a. Cada fase de la salida se alimenta por dos rectificadores trifsicos, uno que suministra la tensin durante el semiciclo positivo y el otro para el semiciclo negativo. Los ngulos de encendido de los tiristores se controlan de tal modo que se utilizan diveros sectores sucesivos de las tres tensiones de entrada para producir las tres ondas de salida. Los cicloconvertidores son tiles para producir una salida a una frecuencia regulable baja. Prcticamente, la mxima frecuencia de salida es alrededor de un tercio de la frecuencia de alimentacin trifsica. Los cicloconvertidores se utilizan para el control de velocidad de grandes motores asncronos o de induccin.
0 0 -

7.10. ACCIONAMIENTOS ELCTRICOS 7.10.1. Generalidades


Un accionamiento elctrico es un sistema capaz de convertir energa elctrica en mecnica, produciendo un trabajo til y manteniendo el control sobre el proceso de conversin. En la

638

MQUINAS ELCTRICAS

a)

oJ

oJ

'7

'7
oJ

'2

r>

'2

r~
~

CONVERTIDOR P

-:)0
+10

CONVERTIDOR N CARGA

b)
mI

(J.p

= 90

r. ... t t t t
...

............... ...
30

60 30 0

60

90 (XN = 90

120 60

150 180 150 120 90 30 0 30 60 90


0

t t t t

CONVERTIDOR P

CONVERTIDOR N

Figura 7.56.

Cicloconvertidor trifsico a monofsico.

Figura 7.57 se muestra el diagrama de bloques simplificado de un accionamiento elctrico regulado electrnicamente. La red elctrica alimenta, por medio de un convertidor electrnico, a un motor elctrico que mueve la carga directamente o por medio de una transmisin mecnica. Existe un sistema de medida que recibe seales de la mquina accionada y del motor elctrico y cuyas salidas se envan al sistema de control que acta sobre el convertidor. La carga constituye el sistema mecnico empleado para realizar el trabajo productivo. Por ejemplo, en un tren elctrico, mquinas herramientas, trenes de laminacin en las fbricas metalrgicas o en industrias papeleras, gras, excavadoras, etc., normalmente las necesidades de la carga vienen definidas por unos requerimientos especficos de par y velocidad. Para seleccionar adecuadamente un accionamiento elctrico para una aplicacin concreta, se deben conocer las demandas que requiere la carga. Debe elegirse un motor elctrico con unas caractersticas par-velocidad y velocidad-corriente que se adapten a aqulla. Esto significa que para que las caractersticas del motor sean compatibles con los requerimientos de la carga, aqul debe funcionar sin que se sobrepasen los valores mximos de corriente y/o potencia. Las curvas par-velocidad que presenta el motor cuando la alimentacin se ajusta a sus condiciones asignadas o nominales se denomina caracterstica natural. Generalmente la caracterstica natural del motor elctrico no es compatible con las solicitaciones que requiere la carga, y es por ello que se hace necesario colocar un convertidor electrnico entre la red de alimentacin y el motor elctrico. El convertidor electrnico, o simplemente convertidor, controla el flujo de potencia que se transfiere de la red al motor, de tal modo que sus curvas caractersticas se adapten a la carga.

CAPTULO 7.

ACCIONAMIENTOS ELCTRICOS

639

RED ELCTRICA

CONVERTIDOR ELECTRNICO

MOTOR ELCTRICO

SISTEMA MECNICO

SEAL DE CONSIGNA

SISTEMA DE CONTROL

SISTEMA DE MEDIDA

Figura 7.57. Diagrama de bloques de un accionamiento elctrico.

Para realizar esta misin el convertidor dispone de un sistema de control que trabaja con niveles muy bajos de tensin y potencia, y que incorpora transistores y circuitos integrados, y tambin a veces sistemas microprocesadores. La seal de consigna, que define el punto de funcionamiento del accionamiento, representa una entrada al equipo de control. Existe un aislamiento elctrico entre el circuito de potencia del convertidor y el sistema de control del mismo; esto se debe a dos razones fundamentales: por una parte evitar que una avera del circuito de potencia afecte al circuito de control e incluso al operador que est ajustando los valores de las seales de consigna de este ltimo, y por otra, intentar que los armnicos generados en el circuito de potencia no lleguen al circuito de control, ya que podran provocar por interferencia electromagntica un funcionamiento anmalo del mismo. Un accionamiento elctrico dispone tambin de elementos sensores o transductores que detectan la velocidad de giro, la corriente del motor, etc., lo cual es imprescindible tanto para la proteccin del convertidor como para un funcionamiento correcto del mismo, sobre todo cuando el equipo de control acta como un sistema realimentado. El accionamiento elctrico completo mostrado en la Figura 7.57, que incluye la red, el convertidor con su equipo de control y sus entradas tanto de consigna como las que proceden de los sensores y el conjunto motor-carga, se puede considerar como un sistema integral. El punto de funcionamiento del motor puede variar si se producen cambios en alguno de los elementos del conjunto. Estos cambios o perturbaciones afectan al sistema completo, ya que todos los componentes estn interconectados elctrica o mecnicamente. Por ejemplo, la seleccin de las caractersticas ms adecuadas de la red depende del tipo y potencia del motor que se quiera utilizar, o al contrario, hay que elegir el tipo de motor en funcin de las caractersticas de la red de alimentacin. El convertidor a utilizar debe tener muy en cuenta el comportamiento de la carga, pero tambin tiene influencia en su seleccin las limitaciones de la red elctrica; por ejemplo, y de acuerdo con el Reglamento Electrotcnico de B.T., es posible que haya que limitar los efectos de las corrientes de arranquedel motor del accionamiento,ya que stas pueden provocar oscilaciones de tensin no admisibles por la compaa suministradora, aunque las puedan soportar el conjunto motor-carga. En algunos casos, el convertidor ms adecuado tcnica y econmicamente, desde el punto de vista del accionamiento tiene que desecharse porque introduce demasidados armnicos en la red o porque trabaja con un factor de potencia demasiado pequeo.

7.10.2. Funcionamiento en cuatro cuadrantes


En las mquinas elctricas, generalmente se expresa el par en funcin de la velocidad o el deslizamiento, y es por ello que la velocidad se considera como variable independiente y se

640

MQUINAS ELCTRICAS

representa en el eje de abscisas, mientras que el par es la variable dependiente y se lleva al eje de ordenadas. Sin embargo, en los accionamientos elctricos el par se toma como variable independiente y por tanto se asigna al eje X, mientras que la velocidad es ahora la variable dependiente y se representa en el eje Y. La razn de este convenio es que cuando se selecciona un accionamiento, lo primero que debe conocerse es el par que requiere el motor para poder mover la carga, siendo un problema posterior el determinar la velocidad a que trabajar el conjunto. Las caractersticas de un accionamiento pueden expresarse a travs de un diagrama parvelocidad, cuyos lmites se representan en un diagrama de cuatro cuadrantes, para incluir ambos signos del par y de la velocidad. Para comprender este tipo de representacin consideremos el esquema de la Figura 7.58, que representa el accionamiento de un ascensor. Se dispone de un motor que a travs de una caja de engranajes reduce la velocidad para mover un torno o polea sobre el que se arrolla un cable que por una parte sujeta la cabina o caja del ascensor y por la otra lleva un contrapeso, que suponemos que ejerce una fuerza superior a la cabina vaca pero inferior al peso de la cabina cargada (en la prctica el contrapeso suele tener un peso mitad de la cabina cargada). La fuerza que debe ejercerse sobre el cable (en definitiva, el par necesario) para mover el ascensor puede tener un signo u otro dependiendo de los valores relativos de las masas del contrapeso y la cabina (llena o vaca) y del sentido del movimiento: subida o bajada. Como el ascensor debe subir o bajar, el accionamiento tiene que ser capaz de funcionar en todos los cuadrantes de un plano fuerza-velocidad de traslacin de la cabina o, en definitiva, de un plano proporcional par-velocidad angular del motor. En la Figura 7.59 se muestra un diagrama de cuatro cuadrantes que servir para explicar el funcionamiento del accionamiento. En el primer cuadrante se supone que el ascensor est cargado y que est subiendo. El par motor T es positivo, ya que tiene el mismo sentido que el de la velocidad angular. El par resistente T, acta en sentido opuesto al de rotacin y es negativo. La potencia mecnica desarrollada por la mquina, que es el producto del par motor T por la velocidad angular del mismo w, tendr signo positivo. En este cuadrante 1el accionamiento trabaja como motaren sentido directo o positivo. En el segundo cuadrante el ascensor sube, pero sin carga, movido por la accin del contrapeso, que al pesar ms que la cabina vaca, tira de ella, y en estas condiciones la velocidad de

CAJA DE VELOCIDAD CONTRAPESO

CABINA

Figura 7.58.

Accionamiento de un ascensor.

CAPTULO 7.

ACCIONAMIENTOS ELCTRICOS

641

CUADRANTE 11

CUADRANTE
I

FRENADO EN SENTIDO DIRECTO

MOTOR GIRANDO EN SENTIDO DIRECTO

-T

+T

T~ CUADRANTE
III

w~r,.
CUADRANTE IV FRENADO EN SENTIDO INVERSO

MOTOR GIRANDO EN SENTIDO INVERSO CONTRAPESO CABINA VACA Figura 7.59. -w

CABINA LLENA

Funcionamiento en cuatro cuadrantes de un accionamiento elctrico.

subida puede alcanzar valores elevados y peligrosos. Para evitar esto, el par motor debe actuar en sentido opuesto al de rotacin, lo que significa que su potencia mecnica P = Twes negativa. La mquina trabaja como freno desarrollando un par que se opone al movimiento frenando la carga. Se dice que el cuadrante 1Irepresenta el comportamiento del accionamiento en rgimen de frenado en sentido directo. En el tercer cuadrante, el ascensor baja con la cabina vaca. El contrapeso evita el ernbalamiento de la cabina. Para que el ascensor baje, el par motor debe actuar en el mismo sentido que el movimiento. De este modo tanto el par como la velocidad son negativos, por lo que la potencia mecnica P = Tt ejercida por el accionamiento tiene signo positivo. El cuadrante 111representa el comportamiento del accionamiento girando como motor en sentido inverso. En el cuarto cuadrante el ascensor est cargado. La cabina baja por su propio peso, sin necesidad del motor, pero la velocidad de cada podra alcanzar valores peligrosos. Por ello, la mquina elctrica debe desarrollar un par que se oponga a la aceleracin de la cabina. El motor acta como freno, siendo el producto Tt negativo. El cuadrante IV representa el comportamiento del accionamiento como freno en sentido inverso.

7.10.3. Dinmica de la combinacin motor-carga. Estabilidad


Cuando se examinan los regmenes transitorios en los accionamientos elctricos, se desprecia en primera aproximacin los procesos electromagnticos, ya que stos se desarrollan ms

642

MQUINAS ELCTRICAS

rpidamente que los procesos mecnicos. Si se denomina Tel par electromagntico producido por el motor, T,. al par resistente que ofrece la carga, J al momento polar de inercia de las masas giratorias (motor + carga) y t la velocidad angular de rotacin, la dinmica del cuerpo en rotacin nos ofrece la ecuacin de equilibrio de momentos:
T-T=J-

,.

dw dt

(7.112)

que nos indica que el par de rotacin desarrollado por el motor se equilibra con el par resistente y por el momento dinmico. De la ecuacin anterior se observa que: 1. Cuando T> T, 2. 3. > O;es decir, tiene lugar la aceleracin del motor. dt dw Cuando T < T, => - < O; lo que indica que el motor sufre una deceleracin. dt dw Cuando T = T => - = O;y en este caso el motor trabaja en rgimen permanente en r dt un estado de equilibrio.
=> -

dw

De acuerdo con estas condiciones, es evidente que un accionamiento elctrico alcanza un estado de equilibrio a una determinada velocidad, cuando se cumple la igualdad entre el par mecnico que ejerce el motor y el par resistente que ofrece la carga, es decir cuando T = T', Cuando se produce una perturbacin en el sistema (por ejemplo, un cambio del par resistente que ofrece la carga), deja de cumplirse la igualdad anterior y el grupo motorcarga buscar un nuevo punto de trabajo para adaptarse a estas nuevas condiciones. Se dice que el sistema es estable, si al producirse una perturbacin, se llega a alcanzar una nueva condicin de equilibrio. En caso contrario, es decir, si la perturbacin provoca un incremento continuado de la velocidad o una reduccin de la misma hasta llegar a parar el accionamiento, se dice que el sistema es inestable, ya que es incapaz de alcanzar una velocidad de equilibrio. Supngase que el accionamiento se encuentra en una situacin de equilibrio para un punto de funcionamiento especfico para el cual se tiene un par motor T, un par resistente T, Yuna velocidad de giro w. Entonces se cumplir: T

= T,

=>

dw dt

= O, es decir: w = cte.

(7.113)

Al producirse una perturbacin, es decir, una variacin en la alimentacin del motor, en la carga o en cualquier parte del accionamiento, se producirn cambios en los pares motor y resistente y tambin en la velocidad. Si se supone que estos cambios son pequeos y se designan, respectivamente, por !!.T, !!.Tr y !!.w a partir del tiempo t contado desde que se produjo la perturbacin, entonces la nueva condicin de equilibrio vendr expresada por:
(7.114)

CAPTULO

7.

ACCIONAMIENTOS ELCTRICOS

643

Pero teniendo en cuenta la igualdad de (7.112) se obtiene: }-

d/1(J) = /1T- /1T dt r

(7.115)

La ecuacin anterior representa la ecuacin dinmica del accionamiento en funcin de sus desviaciones respecto a su punto de equilibrio o trabajo. Los incrementos anteriores, al ser pequeos, se pueden expresar como funciones lineales de las variaciones de velocidad, es decir:

dT /1T = - /1(1) dw

/1T =
r

dT
_r

di

/1w

(7.116)

donde dT/dw y dT,Jdw representan, respectivamente, las derivadas de los pares motor y resistente en el punto de equilibrio, que son en definitiva las pendientes de las curvas T = f(w) y T, = cp(1). Sustituyendo (7.116) en (7.115) se obtiene: } cuya solucin es:

d/1w dt

=-

dT

deo

/1w -

dT
_r

dw

/1w

(7.117)

/1w(t) = /1(1)0 . e-j d,~-JW

[dT dT]
I

(7.118)

donde /1w()es el valor inicial de la desviacin de velocidad (es decir, para t = O). Para que el sistema sea estable, es necesario que el trmino exponencial de la ecuacin anterior tienda a cero conforme el tiempo tienda a infinito, de este modo el incremento de velocidad se anular con el tiempo. En definitiva, lo que ocurrir es que el motor volver a su velocidad inicial, es decir, la que tena en el punto de equilibrio previo. De este modo, para que el sistema sea estable deber cumplirse la siguiente condicin:

ar, dT --->0 d(1) dw

=>

st;

dT ->d(1) dw

(7.119)

La ecuacin anterior nos indica que el accionamiento ser estable si al producirse una perturbacin que aumente la velocidad del sistema, el nuevo par que ofrezca la carga T, debe superar al par motor T; de este modo se produce una deceleracin o frenado, haciendo que el grupo vuelva a alcanzar la velocidad de equilibrio que tena anteriormente. De un modo anlogo, si se produce una reduccin en la velocidad del sistema, el nuevo par motor T debe ser mayor que el par resistente ofrecido por la carga Tr, Y de este modo el sistema retoma a la velocidad estable. Para comprender la relacin (7.119), en la Figura 7.60 se muestra la caracterstica parvelocidad de un motor asncrono que mueve dos tipos de pares resistentes, uno de magnitud constante y otro que vara con el cuadrado de la velocidad (par ventilador). Para el par resistente 1, de valor constante, existen dos puntos de interseccin con la curva del motor: A y B. Vamos a analizar la estabilidad permanente del punto A. Al producirse un aumento en la velocidad, se observa que el par de la carga se hace mayor que el par motor y esto provocar una deceleracin del motor que devolver la mquina a su punto de funcionamiento en A. De una forma similar, una reduccin de velocidad hace que el par motor sea superior al par resistente, lo que provocar una aceleracin del grupo que devolver el sistema al punto de equilibrio A. Por consiguiente, el punto A representa un punto de funcionamiento estable. En

644

MQUINAS ELCTRICAS

0,9 0,8
-e

I A
Par motor

"Y' ~
I

'c "" ::l


S o,
-e '" ' o

'"

0,7 0,6 dTldOJ...... 0,5 0,4 0,3 0,2 0,1

-e

rj ~/d(D
I
..t! ro'

,
~
I I

BIj}_

!j

,
Par resistente

!\ ....
21

,
I

,'~
1

I Par resistente 1
1,5 2 2,5 Par por unidad 3 3,5

0,5

Figura 7.60. Curvas par-velocidad de diversas combinaciones motor-carga.

el caso del punto B, si se produce un aumento de velocidad, el par motor se hace mayor que el par de carga, provocando una aceleracin del motor y el punto de funcionamiento se alejar del punto B (el equilibrio se alcanzar en el punto A, si el par resistente no ha variado). Similarmente, una disminucin de la velocidad hace que el par resistente sea mayor que el par motor, lo que provocar una deceleracin y el punto de trabajo se alejar de B; de hecho, la mquina tender a pararse. Por ello, el punto B es un punto de funcionamiento inestable. Vamos a analizar ahora el punto de equilibrio e, que se obtiene con el par resistente 2 y que es de tipo cuadrtico. Si se aplican los razonamientos anteriores,podemos deducir que e es un punto de funcionamiento estable. Obsrvese que aunque los puntos B y e estn dentro de la misma zona de trabajo de la curva par-velocidad del motor, tienen un comportamiento totalmente distinto, ya que el punto e representa unfuncionamiento estable, mientras que el punto B no lo es. Esto demuestra que la estabilidadde un punto de equilibrio depende no solamente de la curva caracterstica del motor sino tambin de la curva par-velocidad del par resistente o de carga. Obsrvese que los puntos B y e estn situados en la zona del motor que tiene pendiente positiva y suele representar una regin donde la mayora de las cargas tienen un funcionamiento inestable, por lo que generalmente se denomina regin de funcionamiento inestable.

7.11. ACCIONAMIENTOS ELCTRICOS CON MOTORES DE C.C. 7.11.1. Generalidades


La aplicacin principal de los convertidores electrnicos descritos anteriormente en este captulo es la regulacin de velocidad de los motores elctricos. Como quiera que histricamente los primeros motores que se utilizaron en los accionamientos elctricos de velocidad variable fueron de corriente continua, este epgrafe se va a dedicar al control electrnico de este tipo de motores. Recurdese del Captulo 6 que el motor de c.c. est formado por un devanado inductor (excitacin o campo) que est situado en el esttor y un inducido que est situado en

CAPTULO 7.

ACCIONAMIENTOS ELCTRICOS

645

el rotor y que dispone de un colector de delgas o conmutador, cuya representacin simplificada se muestra en la Figura 7.61. Las frmulas fundamentales que gobiernan el comportamiento de los motores de c.c., y que ya se demostraron en el Captulo 6, son: Tensin aplicada al inducido: F.c.e.m.: Par electromagntico: Potencia mecnica: Curva de imanacin lineal: V = E + RJ E = kEnct> T = kTct>I P = Tw = El;
<1> = kJe

(7.120a) (7. 120b) (7.120c) (7.120d) (7.120e)

En las ecuaciones anteriores conviene hacer dos advertencias: a) se han supuesto despreciables las prdidas en el hierro y mecnicas para que coincida la potencia electromagntica con la potencia mecnica; b) se ha supuesto el circuito magntico lineal y por ello existe proporcionalidad entre el flujo que producen los polos y la corriente de excitacin. De acuerdo con las ecuaciones (7. 120a) y (7.120b) se puede escribir: E V - RJ n =-=---'-' k!J kE<I> (7.121)

lo que significa que la velocidad de un motor de c.c. se puede variar de los modos siguientes: a) regulando la tensin aplicada al inducido, que se denomina control por tensin de inducido; b) regulando el flujo magntico a travs de la corriente aplicada al devanado inductor, y que se denomina control por campo o excitacin. Debe tenerse en cuenta tambin que la corriente absorbida por el inducido I, depende directamente del par electromagntico desarrollado por el motor. La velocidad que corresponde a los valores nominales de tensin, corriente de excitacin y corriente de inducido se denomina velocidad base; de este modo, y tal como se describi en el epgrafe 6.9d del captulo anterior al estudiar el sistema de regulacin electromecnico Ward Leonard, en la prctica la regulacin de velocidad por debajo de la velocidad base se realiza manteniendo constante la corriente de campo le (y la corriente I adaptada al par resistente) y variando la tensin aplicada al inducido V, mientras que la regulacin de velocidad por encima de la velocidad base se hace manteniendo la tensin V en su valor nominal y reduciendo le por debajo de su valor nominal. En la Figura 7.62 se muestran las curvas obtenidas, que dan lugar a dos regiones. En la regin 1, que cubre el rango entre la velocidad cero y la velocidad base o nominal, la regulacin se hace por control de la tensin aplicada al inducido, el motor trabaja a par constante y la potencia absorbida de la red es proporcional a la velocidad. En la regin 2, que cubre el

Figura 7.61. Partes constitutivas de un motor de c.c.

646

MQUINAS ELCTRICAS

Regin I (Par constante) Par T

Regin 2 , (Potencia constante)

, ,, , ~

: .: Potencia P:
~

),~,
,

----T
,
Velocidad base)
11~=

,, , , , ,

,,, ,, ,

"
Figura 7.62.

11] (velocidad

2n

Regiones de regulacin de velocidad de un motor de e.e,

rango entre la velocidad base y una velocidad mxima que suele ser doble que la nominal, la regulacin se realiza por control de la corriente de excitacin del inductor; en esta zona la mquina trabaja a potencia constante mientras que el par se va reduciendo conforme aumenta la velocidad, al ser P = Tea = constante, y es evidente que la rama correspondienteal par sigue la ecuacin de una hiprbola y de ah la forma que se muestra en la Figura 7.62. Los tipos de convertidores electrnicos que se utilizan para realizar las regulaciones anteriores, que representan en cierto modo convertidores estticos Ward Leonard, dependen del tipo de red existente. Si la red de alimentacines de c.a. se necesitarn rectificadorescontrolados para transformar la c.a. de la red en c.c. de amplitud variable que se utilizar para alimentar el inducido o el inductor de los motores de c.c. Sin embargo, si la red de alimentacines de c.c., se debern usar choppers o recortadores para transformar la tensin de c.c. de alimentacin de amplitud constante (producida en la prctica por rectificadores fijos) en tensin de c.c. de amplitud variable.

7.11.2.

Regulacin de la velocidad de motores de c.c. por medio de rectificadores controlados

En la Figura 7.63 se representa de un modo simplificado el esquema correspondiente que utiliza rectificadores controlados para alimentar el inducido o el inductor o ambos a la vez. La red de c.a. puede ser monofsica o trifsica, por lo que los convertidores electrnicos sern, respectivamente, mono o trifsicos. Generalmente el uso de redes monofsicas se restringe a
1, mx imo de la tensin simple de la red)

---+
+
E R R"

v'n (Valor

RED MONOFSICA '__,---' O TRIFSICA

+
V=V.,

RECTIFICADOR CONTROLADO

INDUCTOR RECTIFICADOR MOTOR DE e.e. CONTROLADO (INDUCIDO)

Figura 7.63.

Regulacin de velocidad de motores de c.c. con rectificadores controlados.

CAPTULO 7.

ACCIONAMIENTOS ELCTRICOS

647

motores cuyas potencias nominales no superen 5 kW; para potencias mayores se recurre siempre a redes trifsicas para la alimentacin del sistema convertidor, ya que se consigue un mejor equilibrado de las fases. Como ya se indic en el estudio de los convertidores electrnicos en forma de rectificadores controlados, si todos los dispositivos de conmutacin son tiristores, el convertidor se denomina completo. En el caso de que la mitad de ellos sean tiristores y la otra mitad diodos, entonces se trata de semiconvertidores. Recurdese que el retraso en el encendido de los tiristores determina el valor medio de la tensin de salida del convertidor de acuerdo con las expresiones siguientes:
Convertidor monofsico: Semiconvertidor monofsico: 2Vm V:,c = -- cos a

(7.122a) (7.122b)

Vec = Vm (l + cos a) n V:". = V .. =


ce

Convertidor trifsico:

3 ()3v;,,) cos a n 3()3V;,)


2n (1 + cos a)

(7.122c)

Semiconvertidor

trifsico:

(7.122d)

Que corresponden, respectivamente, a las expresiones ya demostradas en (7.47), (7.62), (7.64) Y(7.71). En estas ecuaciones Vm es el valor mximo de la tensin de la red en el caso de que se trate de sistemas monofsicos y representa la tensin mxima de la tensin simple o de fase de la red en el caso de que se trate de sistemas trifsicos. Recurdese asimismo que las expresiones anteriores solamente son vlidas si la conduccin de los tiristores es continua, lo que se produce siempre que la inductancia correspondiente de la mquina sea elevada; en caso contrario lo ms conveniente es aadir una gran inductancia en serie con el circuito. En la Figura 7.64 se muestra de un modo grfico las relaciones anteriores; dependiendo de que se trate de un convertidor completo o un serniconvertidor.El valor mximo de la tensin de salida VecO(que es el valor que resulta para un ngulo de disparo a = O)vale 2V;"In 0(3)3) V,,,In,por lo que las expresiones anteriores obedecen las siguientes expresiones generales:
Convertidores: Semiconvertidores: Vcc = Ved)
COs

Vee = Ved) (l + cos a)

(7.123a) (7.123b)

-',:'!

------------..

Figura 7.64. Tensin de salida de un convertidor en funcin del ngulo de encendido.

648

MQUINAS ELCTRICAS

Conviene destacar al ver la Figura 7.64 que siempre que se utilicen convertidores completos, la tensin media resultante que producen tiene valores positivos para ngulos de encendido 0 < (X < 90 y valores negativos para 180 < y. < 360. Sin embargo, los semiconvertidores solamente dan una tensin media de c.c. de polaridad constante y generalmente requieren el uso de un diodo de libre circulacin para retornar la energa inductiva del circuito a la red. En definitiva, en el caso de convertidores completos la tensin de salida puede variar entre + Vcco y -V;c(), mientras que en el caso de los semiconvertidores la tensin de salida vara solamente entre O y V;.d). Como quiera que para conseguir un control integral de un motor de c.c. conviene disponer de ambas polaridades de alimentacin a la mquina, en lo que sigue se supondr que se trabaja siempre con convertidores completos. En el caso de que se trabaje a par constante (regin l de la Fig. 7.62), el convertidor que alimenta al inducido, sea monofsico o trifsico, dispondr de una tensin de salida que segn (7.123a) es de la forma: (7.124) Ahora bien, de acuerdo con (7.121) se tiene: (7.125) En la expresin anterior, la tensin aplicada al motor Ves igual a la tensin producida por el convertidor, es decir, V = Vec. Despejando la corriente de inducido en funcin del par de la ecuacin (7.120c) resulta:
n =

v-

RI

kif>"

1 ( T ) = kE<D V - R kT<D

(7.126)

y al sustituir (7.124) en (7.126) se obtiene:


n = V - RI l ( " =V o cos k E<D kE<D ce
IX -

R I kT<D

T )

(7.127)

Al ser el par T electromagntico constante en esta regin, es evidente que la velocidad n del rotor del motor de c.c. depende del ngulo de encendido, de tal modo que conforme IX aumenta la velocidad decrece, y a la inversa, al disminuir IX la velocidad aumenta, que corresponde en cada caso a una reduccin o elevacin de la tensin V aplicada al inducido del motor. En el caso de que el motor trabaje a potencia constante (regin 2 de la Fig. 7.62), el convertidor que alimenta al inducido, sea monofsico o trifsico, se regula para que suministre la tensin nominal a la mquina. El inductor tiene que estar alimentado por un rectificador controlado para poder reducir la corriente de excitacin. Como quiera que la corriente del inducido es constante (adaptada al valor de la carga mecnica), el mantener la potencia mecnica P constante significa que:
P = EI
=>

E = constante = kEn<D

(7.128)

y si se supone que el circuito magntico es lineal, al sustituir el flujo magntico de la ecuacin (7 .120e) en la ecuacin anterior, resulta:
n=---

(7.129)

kEk/J

CAPTULO

7.

ACCIONAMIENTOS ELCTRICOS

649

y teniendo en cuenta que en el inductor se cumple en rgimen permanente que I, = VJRe' siendo Ve la tensin aplicada al inductor que se extrae de la salida de un convertidor completo, por lo que la ecuacin anterior se transforma finalmente en: (7.130) Es evidente entonces que al aumentar el ngulo de encendido del convertidor que alimenta al circuito de excitacin se incrementa la velocidad del motor en la regin correspondiente a potencia de la mquina constante. En cualquier caso, esta doble regulacin de par y de potencia con este sistema de control solamente permite que el sistema trabaje en un nico cuadrante. Para que la mquina trabaje en dos cuadrantes es preciso utilizar un convertidor doble o dual cuyos circuitos responden a los esquemas ya estudiados en las Figura 7.33 y 7.39; en ambos casos los ngulos de encendido de los convertidores deben estar relacionados por la ecuacin (7.62). Se puede lograr tambin un funcionamiento en dos cuadrantes con un nico convertidor completo empleando el esquema de la Figura 7.65. No se ha dibujado la alimentacin del inductor que produce un flujo inductor constante, por lo que.la tensin Ve es fija en ambas figuras (es decir, se supone un funcionamiento a par constante). En el caso a) el rectificador controlado que alimenta al inducido tiene un ngulo de encendido que vara en el rango 0 < 'X < 90, por lo que la polaridad de la tensin es la mostrada y es superior en cada momento a la f.c.e.m. del motor, y por ello la potencia elctrica que absorbe el motor es positiva, transformndose la energa elctrica en mecnica. Para poder invertir el sentido de transferencia de la energa, al no poder cambiar el sentido de la corriente en el inducido debido a que el rectificador solamente conduce en un solo sentido, la nica alternativa posible es invertir tanto los signos de V como de la f.c.e.m. E, de tal modo que adems se cumpla que IEI > IVI, como se muestra en la Figura 7.65b. Para invertir la salida del rectificador es preciso que el ngulo de encendido vare en el rango 180 < 'X < 360, yen esta situacin el rectificador trabaja como inversor conmutado por la red o por lnea. La inversin de la f.c.e.m. E puede lograrse de varias formas:
a) b)

Que la carga mecnica que mueve el motor de c.c. provoque en el rotor un movimiento en sentido inverso (es decir, que trabaje en el cuadrante IV de la Fig. 7.59). Que se invierta la corriente en el circuito de excitacin girando el motor en sentido directo, lo que se logra de una forma automtica si el inductor se alimenta con un rectificador controlado haciendo que el convertidor funcione con ngulos de encendido comprendidos entre 90 y 180, o si el rectificador es fijo o no controlado,

'i_
+
v
n

'i_
v

'e

al oo<:x < 90; v E; Motor

b) 18()o<:x < 360; E > V; Frenado regenerativo

-'e

Figura 7.65.

Funcionamiento en dos cuadrantes de un motor de c.c.

650

MQUINAS ELCTRICAS

conmutando los dos terminales entre s del inductor respecto a la alimentacin de su rectificador. (Tambin se pueden invertir los terminales del inducido, pero no es conveniente abrir este circuito por la gran corriente que lleva). En ambos casos se lograr una inversin en el sentido de transferencia de la energa siempre que adems se cumpla la condicin IEI > IVI y la mquina devolver energa elctrica a la red, lo que provocar su frenado regenerativo o por acumulacin de energa. Es indudable que el funcionamiento en cuatro cuadrantes requerir el uso de dos convertidores para el inducido y dos para el inductor. Cada uno de ellos responder al esquema de la Figura 7.63: uno de ellos tendr los tiristores como se seala en esa figura y el otro tendr los tiristores en posicin o polaridad contraria. De este modo, con un grupo la mquina podr funcionar en los cuadrantes 1 y IV Ycon el otro en los cuadrantes JI y IlI. EJEMPLO DE APLICACIN 7.12 Un motor de c.c. con excitacin independiente de 20 kW, 500 V, 1.000 r.p.m. dispone de dos convertidores electrnicos trifsicos en puente completo controlados por fase para regular su velocidad, uno para alimentar el inducido y otro para el inductor, con un esquema similar al mostrado en la Figura 7.63. La red trifsica tiene una tensin de lnea de 380 V, 50 Hz. La resistenciadel inducidoes R = 0,2 n y la resistenciadel inductores Re = 150 n. Se supone que el circuitomagntico es lineal, de tal modo que la constanteproducto ser k.k, = 1,l V/A-raells. La mquina trabaja en rgimen de conduccin continua y el rizado de la corriente es despreciable. a) Si el devanado de excitacin trabaja con mxima corriente desarrollando unpar de 150 N.m a 1.000 r.p.m., calcular el ngulo de encendido 'X del convertidor del inducido. b) Si el devanado inductor sigue trabajando con corriente mxima y mueve la misma carga anterior, que tiene un par de 150 N.m, y se ajusta el ngulo de encendido del inducido en el valor 'X = O, cul ser la velocidad a la que girar el motor? c) Para el caso anterior, determinar el ngulo de encendido 'Xc del convertidor que alimenta el circuito de excitacin si se desea elevar la velocidad al doble de la nominal, es decir, a 2.000 r.p.m. SOLUCIN
a)

La tensin eficaz de fase de la red vale:


~~ =

380 :; = 219,39 voltios '1.13

que corresponde a un valor mximo: Vm = 219,39 = 310,27 voltios. La tensin media suministrada por el circuito de excitacin es mxima cuando el ngulo de encendido es de 0, por lo que, de acuerdo con (7 .122c), se tiene: Ve = VecO =

J2 .

3(J3V
rr

m)

3(J3 310,27) . = 513,18 voltios rr

por lo que la corriente mxima de excitacin valdr:


le

= - = --Re

\1"

513,18 150

= 3,42 amperios

CAPTULO

7.

ACCIONAMIENTOS ELCTRICOS

651

El par electromagntico,teniendo en cuenta (7.120e) y (7.120e), se puede expresar as:


T

= k;

. <D . 1;

k . k . le . I

Pero al ser el par igual a 150N.m, la constante producto kT' k = 1,1 Yla corriente de excitacin mxima igual a 3,42 A, de la ecuacin anterior resulta:
1; =

150 = 39,87 amperios 1,1 3,42

La f.c.e.m. del motor, teniendo en cuenta (7. 120e), (7. 120d), y (7.120e), es igual a:

de donde se deduce: E

1.000 1,I . 3,42 . 2 -= 393,96 voltios 60 39,87 = 401,93 voltios

De este modo la tensin necesaria para alimentar el inducido sera:

v = E + RJ; = 393,96 + 0,2'


y de (7. 122c) se deduce: 401,93
b)

V:,O COS

:t; = 513,18 cos (/.,

cos:t;

= 0,783

:t; = 38,44

Si x; = O, en el convertidor del inducido se obtiene la siguiente tensin:


V =

v',O = 513,18 voltios


0,2 . 39,87 = 505,21 voltios

que da lugar a una f.c.e.m.: E = V - RJ de donde se deduce:


(1)

= 513,18 -

E 505,21 = --= = 134,29 rad/s (kk)/e 1,1' 3,42

es decir:
(J) = e)

260

t:

= n=

60 134,29 = 1.282,4 r.p.m. 2

La f.c.e.m. ser la misma que en el caso anterior, es decir, 505,21 voltios, por lo que se tiene:
E

= (kTk)I,,(J)

= 505,21 = 1,1 . 1,.: 2 @ =

2.000

le

= 2,19 amperios

10 que requiere una tensin en el inductor:

Ve = RJe = 150 . 2,19 = 328,94 voltios de donde se deduce: 328,94 =


V'co

cos:te = 513,18 cos

(/.e

cos!Xc

= 0,641 =

!Xc = 50,13

652

MQUINAS ELCTRICAS

7.11.3.

Regulacin de la velocidad de motores de c.c. por medio de choppers

Como ya se ha sealado, los choppers son convertidores electrnicos que transforman una tensin de c.c. de magnitud fija en c.c. de magnitud variable, por lo que son tiles en la regulacin de motores elctricos de c.c. Un ejemplo clsico de ello es la traccin elctrica. Los trenes elctricos, trolebuses y tranvas han funcionado y en muchos casos siguen funcionando con una alimentacin de c.c. y empleando motores con excitacin serie debido a las grandes ventajas que ofrecen stos por su alto par de arranque. AS, en Espaa los trenes metropolitanos funcionan a una tensin comprendida entre los 600 V Y 1.500 V, los ferrocarriles de va estrecha funcionaban a l.500 V de c.c. (en la actualidad 3.000 V) Y los trenes elctricos de va normal disponen de 3.000 V de C.C., que es la tensin normalizada en RENFE. Hasta hace unos aos la regulacin de velocidad se lograba con la incorporacin de restatos de arranque y restatos de regulacin, con los grandes inconvenientes de las prdidas que se producan por efecto Joule. Para simplificar el trabajo del conductor estos restatos se agrupaban en un combinador (control/er), que era un tambor que dispona de una serie de contactos unidos a las resistencias de arranque y regulacin para que las maniobras se hicieran en el orden correcto, lo que evitaba combinaciones prohibidas. Estos combinadores era frecuente verlos en los tranvas y trolebuses, hoy ya desaparecidos pero que an se encuentran en muchos pases europeos. Los convertidores electrmcos han sustituido a estos clsicos restatos, permitiendo la regulacin de velocidad con elevados rendimientos. Una aplicacin ms moderna de los choppers est en el control del arranque y regulacin de velocidad de prototipos de vehculos elctricos terrestres como automviles y autobuses, evitando la contaminacin ambiental que producen los motores de combustin interna; en estos casos la fuente de alimentacin es una batera de acumuladores elctricos que a travs de choppers alimenta el motor de c.c. del vehculo. Los choppers tienen grandes ventajas, como alto rendimiento, control flexible, peso ligero, tamao pequeo, respuesta rpida, y siendo posible tambin el funcionamiento en varios cuadrantes. En la Figura 7.66 se muestra un chopper de dos cuadrantes que es repeticin del esquema de la Figura 7.46. En este chopper, los interruptores estticos SI y S2 se cierran o encienden alternativamente. Cuando la corriente que alimenta el motor tiene el sentido positivo mostrado en la Figura 7.66, la corriente de alimentacin de la fuente V, pasa a travs de SI retornando la energa inductiva por el diodo DI. Se dice entonces que el chopper es directo o reductor y el motor funciona en el primer cuadrante. Cuando la corriente io tiene sentido contrario, la corriente circula por el interruptor esttico S2 y el diodo D2, se dice entonces que el chopper es inverso o elevador y el motor devuelve energa a la red trabajando en el segundo cuadrante. Recurdese de la expresin (7.95) que los valores medios de la tensin y la corriente suministrada por el chopper son los sealados a continuacin:
V =
ce

ON

V = kV
','ce

V -E . 1 = _cc__ R

(7.131)

donde tON representa el tiempo de cierre del interruptor esttico SI, mientras que el tiempo T - tON' es decir, el resto del ciclo, corresponde al tiempo de cierre del interruptor esttico S2. El parmetro k = t()NIT es el denominado ciclo de trabajo del chopper. Como consecuencia de ello, si el chopper tiene frecuencia constante (es decir, Tconstante), al variar el tiempo tON se puede regular la tensin media que suministra al motor de C.C., o en definitivaIa velocidad de la mquina. Se pueden utilizar tambin en el control de motores de c.c. choppers

CAPTULO 7.

ACCIONAMIENTOS ELCTRICOS

653

SI

D2

+ Vs

+
DI
S2

r c. Vo

Ji"
+ E

V.

r----t---t----t---t------FIT,
o
tON

Figura 7.66.

Chopper de dos cuadrantes.

de cuatro cuadrantes como el indicado en la Figura 7.47. Recurdese que los interruptores estticos del chopper pueden ser GTO, IGBT y tambin transistores MOSFET. Nota informativa: Aplicacin de los choppers a la traccin elctrica espaola.

1. RENFE
En la Figura 7.67 se muestra de un modo simplificado el esquema elctrico de un coche motor de la unidad de tren S/446 de Renfe para cercanas como ejemplo de un sistema de traccin elctrica que utiliza choppers. Este tren se ajusta a la composicin Mc-T-Mc (Me: coche motor con cabina, T: coche remolque), es decir, tiene dos coches motores y un remolque. La catenaria tiene una tensin de 3.000 V respecto al carril (en algunos pases se emplea una alimentacin por tercer carril). En cada boje (o bogie) del coche motor existen dos motores de c.c. de 4 polos de 300 kW de potencia, y como hay dos bojes en cada coche motor, se tiene una potencia total de 1.200 kW. El tren tiene dos coches motores, por lo que la potencia total instalada en motores elctricos en estas unidades de tren es de 2.400 kW. El chopper de traccin tiene una frecuencia nominal de 300 Hz (aunque funciona en los saltos de 75-150 y 300 Hz) para tener en cuenta las condiciones de arranque. El sistema incorpora tambin un chopper de frenado a 600 Hz que introduce una resistencia elctrica para disipacin de energa reosttica y un filtro a la entrada del pantgrafo para eliminar lo ms posible los armnicos de red, que consiste en una inductancia de ncleo de aire de 17 mH y un
3.000 V c.c.

Figura 7.67.

Esquema elctrico de un coche motor de tren equipado con choppers.

654

MQUINAS ELCTRICAS

condensador de 600 )lF. Este tren tiene un esfuerzo de traccin medio de 127,8 kilonewtons (kN) y est diseado para velocidades mximas de solamente 100 km/h. La aceleracin en recta y horizontal es de 0,75 rn/s" (O a 60 km/h) y de 0,55 m/s? (O a 100 km/h). La deceleracin mxima de servicio es de 0.7 mise solamente con freno elctrico y de 1 rn/s? (con freno combinado).

2. Metropolitano de Madrid
La serie 5.500 del Metro de Madrid se alimenta por una catenaria de 600 V de c.c. (+20-30 por 100) con motores de traccin (l/boje) AEG-ABS 3324, con una potencia continua de 211 kW (a 1.900 r.p. m.) y una potencia unihoraria de 235 kW (a 1.825 r.p.m.). El chopper de traccin es del tipo AEG-I.200 A, con frecuencia nominal o asignada de 250 Hz. El filtro de red est formado por una inductancia L = 2 mH y un condensador C = 30 !lF. El freno elctrico es de recuperacin y est combinado con frenado reosttico. La velocidad mxima del tren es de 70 km/h, con una aceleracin-deceleracin de 1,05 mis".

EJEMPLO DE APLICACIN 7.13 Un motor de c.c. con excitacin independiente de 600 V, 1.500 r.p.m., 200 A, tiene una resistencia de inducido R = 0,2 n. Este motor est controlado por un chopper de 600 V, 400 Hz. a) Calcular la velocidad del motor para un ciclo de trabajo k = 0,5 y un par resistente mitad del nominal. b) r: Cul ser la velocidad del motor cuando funcione en rgimen de frenado regenerativo con k = 0,5 y moviendo una carga con par nominal? NOTA: Elflujo magntico del inductor se mantiene constante y la inductancia del inducido es lo suficientemente elevada para considerar plana la corriente absorbida por el motor. SOLUCIN
a)

En condiciones nominales. la tensin aplicada al motor es de 600 V Y la corriente del inducido 200 A, por lo que la f.c.e.m. del motor vale: El = V - R(, = 600 - O, 15 200 = = 570 voltios. Si el par resistente es mitad del nominal, la corriente absorbida ser tambin mitad de la nominal, es decir, de 100 A, Y como adems el ciclo de trabajo es igual a 0,5, de las ecuaciones (7.131) se deduce: El = kV, - Rl, = 0,5 . 600 - 0,15 . 100 = 285 voltios pero teniendo en cuenta que la f.c.e.m. del motor viene expresada por E = kE<1>n, es decir, la f.c.e.m, es directamente proporcional a la velocidad en las condiciones de flujo constante. Como quiera que la velocidad nominal es de 1.500 r.p.m., se tendr: El = kE es decir: 570 285 1.500
.

<1> . ni

570 = k; . <1> . 1.500

; E2 = kl: . <1> .

112

285 = kE

<1>

/12

n2 = 750 r.p.m.

b)

En la situacin de frenado la corriente tendr signo negativo; si adems mueve un par nominal, la corriente entregada por el motor al chopper y de ah a la red ser de 200 A. Como el ciclo de trabajo es de 0,5, la f.c.e.m. del motor en esta situacin ser:

E, = kV, - Rl; = 0,5600

- 0,15' (-200) = 330 voltios

CAPTULO

7.

ACCIONAMIENTOS ELCTRICOS

655

y por consiguiente, al comparar con los datos nominales, se tendr: 570 330 1.500

= n3

= 868,42 r.p.m.

7.11.4. Regulacin de motores de e.e, mediante realimentacin


En los epgrafes anteriores se ha analizado la regulacin de velocidad de los motores de c.c. utilizando tanto rectificadores controlados como choppers. En ambos casos la configuracin empleada se conoce en la ingeniera de control como sistemas de lazo abierto (open loop en ingls). En los sistemas de lazo abierto se comprueba que cuando varan las condiciones de trabajo de la mquina, tambin cambia la respuesta de la misma, y de este modo, si por ejemplo se producen cambios en el par resistente aplicado al motor, aparecen tambin cambios en la velocidad de giro. Este comportamiento, que en muchos accionamientos no tiene demasiada importancia, en otros es inadmisible, ya que repercute en el proceso de fabricacin que llueve el accionamiento. Se puede conseguir una mayor estabilidad de la velocidad del motor 'y de la respuesta dinmica que tiene el mismo, utilizando las tcnicas de los sistemas realimentados (feedback en ingls) o de lazo cerrado (closed loop). Un motor de c.c. con realimentacin tiene, como se indica en el diagrama de bloques de la Figura 7.68, una entrada de referencia o de consigna n * y dispone en el eje motor-carga de un tacogenerador (o simplemente tacmetro elctrico) que detecta la velocidad real de la mquina n. El funcionamiento del sistema es tal que aunque se produzcan cambios en el par resistente que ofrece la carga, la velocidad del motor se ajustar rpidamente al valor de consigna asignado. Para simplificar el circuito se ha supuesto que la mquina se alimenta con excitacin constante a travs de un rectificador monofsico, por lo que la regulacin de velocidad se consigue mediante la variacin de la tensin de c.c. aplicada al inducido, que procede de un convertidor electrnico, que en el caso de que est alimentado por una red trifsica ser un rectificador controlado, o cuando la red sea de c.c., que ser un chopper. El taco generador es una dinamo de imn permanente que produce una f.e.m. proporcional a la velocidad y que tiene salidas tpicas del orden de 10 V cada 1.000 r.p.m.; la tensin del tacmetro es, por consiguiente, proporcional a la velocidad de giro de la mquina, y como quiera que a bajas velocidades esta tensin tiene mucho rizado, se alisa su salida por medio de un filtro adecuado. En la Figura 7.68 se ha denominado n a la salida del tacmetro (realmente es una tensin proporcional a la velocidad del motor). El funcionamiento del circuito es el siguiente: La salida del tacmetro n se compara con la seal de referencia n * en el nudo restador, mostrado en la parte superior izquierda de la Figura 7.68. La salida de este nudo es una tensin de error de velocidad en = n* - n, que se aplica a un controlador de velocidad, que suele ser del tipo proporcional-integral (PI) y cuya misin es estabilizar el comportamiento del motor o en general del accionamiento, mejorando la respuesta transitoria y reduciendo el error de velocidad a cero por efecto de la accin integral. La tensin de salida del controlador de velocidad Ve se lleva a un circuito generador de impulsos de disparo que modifica el ngulo de encendido (J. de los componentes activos (tiristores, GTO, etc.) del convertidor. Si se parte de una situacin estacionaria o de rgimen permanente en la que el motor est girando a la velocidad de consigna n*, al aumentar el par resistente del motor se producir una reduccin de su velocidad de giro, que llegar a tener un valor n, lo que da lugar a una seal de error de velocidad en tanto ms elevada cuanto menor sea la velocidad de la mqui-

656

MQUINAS ELCTRICAS

Red trifsica Controlador de velocidad Generador de impulsos de disparo

r-v

Velocidad de consigna

JL
n Velocidad real del motor

Convertidor

!
le'" +

Red monofsica

~~
Rectificador INDUCTOR

Filtro

Figura 7.68.

Control de la velocidad de un motor de c.c. con realimentacin de velocidad.

na n respecto a la de referencia n *. La salida del controlador de corriente Ve aumenta y como consecuencia de ello se reduce el ngulo de encendido II que produce el generador de impulsos (en el caso de que se trate de un convertidor constituido por rectificadores controlados), o acta aumentando el ciclo de trabajo (duty cycle) en el caso de que el convertidor sea un chopper. Como consecuencia de ello aumenta la tensin de salida Vee del convertidor que alimenta el inducido del motor de c.c., lo que eleva la corriente del inducido I provocando un aumento del par electromagntico producido por el motor, haciendo que se eleve su velocidad de giro hasta conseguir anular el error de velocidad, logrando que la velocidad real de la mquina n coincida con la velocidad de consigna o referencia n*. En esta situacin se habr alcanzado una condicin de equilibrio, al haber conseguido reducir el error de velocidad a cero para el nuevo ngulo de conduccin de los tiristores. El sistema de control analizado de la Figura 7.68, aunque es muy simplificado, sirve para conocer las ventajas del control realimentado, que permite mejorar el comportamiento transitorio de la mquina con una gran precisin y una alta estabilidad de funcionamiento.El problema de este circuito es que no tiene proteccin de sobrecorriente; es decir, cuando se produce un aumento del par resistente, sobre todo si ste es brusco (operacinde arranque, frenado, etc.), se produce un aumento de la tensin de salida del convertidor que se aplica al inducido, pero teniendo en cuenta que tanto la inductancia del mismo L como su resistencia R son de pequeo valor e incluso tambin su cociente, que define la constante de tiempo r = L/R, la variacin de la corriente de inducido con el tiempo puede incluir transitorios con valores de pico peligrosos para la vida de los tiristores del convertidor. Es por ello conveniente ampliar el esquema de la

CAPTULO

7.

ACCIONAMIENTOS ELCTRICOS

657

Figura 7.68 introduciendo un lazo de realimentacin de corriente que limite la intensidad del inducido a valores seguros. En la Figura 7.69 se muestra un esquema de control realimentado de un motor de c.c. con dos lazos de control: de velocidad y de corriente. Si se analiza la parte superior del circuito de control del inducido, se observa que el error de velocidad pasa a travs del controlador de velocidad y luego a un Iimitador de corriente (que es un circuito electrnico con caractersticas de saturacin). La salida de este bloque se denomina l* y es la corriente de referencia o consigna del inducido. La corriente del inducido J se detecta con un transductorde corriente, que en los casos ms simples es un shunt, aunque modernamente es un dispositivo de efecto Hall (que produce una tensin proporcional a la corriente). La salida de este transductor se filtra convenientemente y se compara con la corriente de inducido de referencia, dando lugar a una tensin eque se aplica a un controlador de corriente tipo PI (proporcional-integral)para reducir el error a cero y mejorar la respuesta dinmica del lazo. Es la salida de este controlador "".la que pasa por el generador de impulsos de disparo para ajustar los ngulos de encendido de los tiristores del convertidor que alimenta al inducido. Para ver el efecto combinado de ambos lazos de realimentacin supngase, por ejemplo, la operacin de arranque del motor. En esta situacin la velocidad del motor es n = O y la

Red trifsica

Velocidad de consigna

Controlador de velocidad

Limitador de corriente

Controlador de corriente

Generador de impulsos de disparo

Jl_ n

Velocidad real del motor

r-'---",
: R,.
, !.. .... ,

: :,.-.-- I
, I

--- ..1.-------1

r:

Convertidor

- V;:'e
~oo~~

1oI1------{II)Transductor

RI@'Filtro II _ ~,(.'
+ .__ __ _-_ __ -_ --_ -- -- -- - -_ --- E

= Vce -

,
RI
I I ,

Controlador de excrtacion '. -

Generador de Impulsos de dis aro

Convertidor

INDUCIDO

Filtro

Figura 7.69. Control de la velocidad de un motor de c.c. con realimentacin de velocidad y de corriente.

658

MQUINAS ELCTRICAS

velocidad de consigna es n*, por lo que el error de velocidad es mximo y su valor es: e = n* - n, lo que provocar la saturacin del limitador de corriente, cuya salida ser l* = '/:"x' y de este modo la corriente del inducido no podr sobrepasar este valor, protegiendo tanto al motor como al convertidor. Conforme el motor se vaya acelerando, el error de velocidad se ir reduciendo y el limitador de corriente se saldr de la zona de saturacin, dando lugar a una (* inferior al valor mximo permitido. Evidentemente, si con el esquema de la Figura 7.69 se mantiene la excitacin constante en su valor nominal o de consigna, la regulacin de velocidad solamente se puede realizar controlando la tensin del inducido, lo que permite una regulacin por debajo de la velocidad base (vase Fig. 7.62). Por encima de esta velocidad base, la regulacin de velocidad se consigue actuando sobre la corriente de excitacin. De este modo una regulacin completa de velocidad requiere que el inductor del motor est alimentado tambin por un convertidor, para poder modificar la corriente de excitacin Esta fase de la regulacin se denomina control por debilitamiento del campo (jield weakening en ingls), que se puede realizar de una forma manual ajustando el valor de consigna de la entrada E* al circuito de control del inductor o de forma automtica mediante la realimentacin de la corriente del inducido con los bloques sealados con lnea discontinua en la Figura 7.69; en este caso suele fijarse el valor de la f.c.e.m. de consigna entre el 80 y el 90 por 100 de la tensin nominal o asignada del inducido. El valor de la f.c.e.m. real del motor, que viene expresada por E = v:., - R/, se compara con E* en el circuito restador sealado en la parte central izquierda de la Figura 7.69, cuya salida se aplica a un controlador de excitacin y que acta sobre el generador de impulsos del inductor, variando de este modo la corriente de excitacin (,. Por debajo de la velocidad base, el controlador de excitacin se satura debido al gran valor de la tensin de error ee' haciendo que la tensin aplicada al inductor sea la nominal o asignada, lo que asegura que por debajo de la velocidad base, la corriente de excitacin ser la nominal. Si la velocidad de consigna n* se ajusta para un valor superior a la velocidad base, en el circuito de control del inducido se producir la saturacin, dando lugar a una '/;"x que limitar el ngulo de encendido de los tiristores del convertidor para suministrar la tensin nominal v:. e al inducido. El motor se acelerar y la f.c.e.m. del mismo aumentar, lo que provocar una reduccin de la tensin de error e. del lazo de control de campo, y como consecuencia de ello se reducir la corriente de excitacin, hasta que la velocidad real n coincida con la impuesta por la consigna n*. El circuito de excitacin de la Figura 7.69 puede incluir tambin proteccin por corriente de una forma anloga al circuito del inducido. Si el convertidor del inducido admite ngulos de encendido superiores a 90, el circuito de la Figura 7.69 se puede emplear para que la mquina trabaje en dos cuadrantes. Colocando otro convertidor en conexin antiparalelo o dual, podr lograrse una regulacin de velocidad en los cuatro cuadrantes; es decir, la mquina admitir el trabajo como motor en los dos sentidos de giro y con frenado regenerativo en ambos casos.

'e'

7.12. ACCIONAMIENTOS ELCTRICOS CON MOTORES DE C.A. ASNCRONOS 7.12.1. Introduccin

Los motores asncronos o de induccin, en particular los de rotor en jaula de ardilla, tienen grandes ventajas frente a los motores de C.C., como son la robustez, mantenimiento reducido,

CAPTULO

7.

ACCIONAMIENTOS ELCTRICOS

659

mayor fiabilidad, menor coste y peso por unidad de potencia. Como ya se sabe, el mayor inconveniente del motor de c.c. es su colector de delgas, que requiere un mantenimiento continuo debido al chispeo que en l se produce por los fenmenos asociados a la conmutacin, ya estudiados en el Captulo 6. La traccin elctrica y los trenes de laminacin de las empresas metalrgicas requeran motores con una buena regulacin de velocidad, por lo que estas aplicaciones estuvieron dominadas hasta fechas muy recientes por motores de c.c. El motor asncrono se utilizaba como mquina elctrica robusta para transformar la energa elctrica en mecnica con un gran rendimiento y poco mantenimiento pero siempre que el proceso productivo no necesitara regulacin de velocidad. Con la llegada del tiristor al mercado en el ao 1957, el aumento posterior de sus capacidades de tensin y corriente, el descubrimiento sucesivo de otros componentes electrnicos como el GTO, los IGBT, los transistores de potencia MOSFET e incluso el microprocesador (1971) se hizo posible el desarrollo de convertidores electrnicos para la regulacin de velocidad de los motores de induccin, que se incorporan cada vez con ms frecuencia en los sistemas que eran desde antao un campo exclusivo de los motores de c.c. Hoy en da, como deca un importante fabricante de mquinas elctricas, para pequeas potencias se vende a la vez en un solo conjunto: cobre, hierro y silicio, indicando con ello que los fabricantes al suministrar el motor asncrono clsico (realizado con bobinas de cobre en un circuito magntico de hierro) incorporan como valor aadido el convertidor electrnico (componentes de silicio) para la regulacin de velocidad y casi al mismo precio que costaba nicamente el motor de induccin hace unos pocos aos. Para ver un ejemplo concreto de esta evolucin o trasvase del motor de c.c. hacia el motor de c.a., es interesante sealar que el sistema de transporte suburbano de Madrid (METRO), alimentado por una red de c.c. de 600 V (yen las nuevas lneas a 1.500 V), Yque en la dcada de 1980 toda su traccin se basaba en motores de C.C., en la actualidad (ao 2003) estas mquinas solamente representan el 40 por 100 del total; es decir, en unos veinte aos, ms de la mitad de los motores de c.c. se han sustituido por motores de c.a. asncronos. Es de esperar que a este ritmo de cambio dentro de otros veinte aos la traccin elctrica se realizar totalmente con motores de c.a. Para comprender los principios bsicos de la regulacin de velocidad de los motores asncronos, debe recordarse del Captulo 4 que la velocidad mecnica de giro de estas mquinas en r.p.m. viene expresada por:
Il

= ni (1 - s) = _._ (1 - s)
p

60f

(7.132)

y como se sealaba en el epgrafe 4.1 1, Y de acuerdo con la expresin anterior, existen tres procedimientos para cambiar la velocidad 11, a saber: a) variar el nmero de polos de la mquina; h) cambiar la frecuencia de alimentacin I; e) modificar el deslizamiento s. El primer procedimiento, ya descrito en 4.11.1, no es un sistema de regulacin de velocidad en un sentido estricto sino una modificacin de la velocidad pero discreta, es decir, a saltos; por ejemplo, la conexin Dahlander permite conseguir dos velocidades de giro en una misma mquina mediante dos combinaciones distintas del devanado, pero nada ms. De este modo los procedimientos prcticos para regular la velocidad de un motor asncrono se basan fundamentalmente en el control de la frecuencia y del deslizamiento, y ste a su vez se puede controlar indirectamente ajustando la tensin de alimentacin aplicada a la mquina. De acuerdo con lo anterior, los accionamientos elctricos con motores de induccin se clasifican en las tres categoras principales siguientes: 1. Alimentacin con frecuencia constante y tensin variable: Este procedimiento se denomina tambin control por tensin del esttor. Con este mtodo se regula nica-

660

MQUINAS ELCTRICAS

mente la tensin aplicada al esttor; esta accin se puede conseguir en la prctica utilizando autotransformadores o incluyendo en serie con el esttor del motor una serie de impedancias para provocar una cada de tensin intermedia entre la red y el motor, pero a nivel electrnico se consigue empleando los reguladores de c.a. estudiados en el epgrafe 7.6. 2. Alimentacin con tensin y frecuencia variable: En este caso la alimentacin al esttor consiste en una tensin que se vara a la vez que la frecuencia, es decir, haciendo que el cociente E,/f, ~ V, /f, sea constante, para conseguir mantener inalterable el flujo magntico de la mquina y evitar saturaciones. Para efectuar esta operacin existen en la prctica dos sistemas: uno de ellos pasa por una etapa intermedia de c.c. y el otro emplea cicloconvertidores. En el primer caso se emplean dos convertidores electrnicos, uno para rectificar la c.a. de la red y transformarla en C.C., que va seguido de un inversor para convertir la c.c. en c.a. de frecuencia y tensin variable. En el caso de los cicloconvertidores se efecta una transformacin de la c.a. de la red en una c.a. de tensin y frecuencia variable sin pasar por la etapa intermedia de C.C.; generalmente este sistema se emplea para potencias grandes. 3. Recuperacin de la potencia de deslizamiento: Este mtodo consiste en variar la resistencia efectiva del circuito del rotor utilizando convertidores electrnicos, que se unen a los anillos del rotor para recuperar la potencia de deslizamiento que se produce a la frecuencia del rotor f2 = sf., Es obvio que este mtodo solamente se puede emplear cuando el motor tiene el rotor devanado o con anillos deslizantes.

7.12.2.

Regulacin de velocidad por control de la tensin de lnea aplicada al esttor

Se puede regular la velocidad de un motor asncrono variando la tensin que alimenta al devanado del esttor. En la Figura 7.70 se muestra el esquema correspondiente, que utiliza dos tiristores por fase colocados en oposicin, que en el caso de pequeos motores se suelen sustituir por triacs, de acuerdo con lo analizado en el epgrafe 7.6. La tensin que llega al esttor puede regularse fcilmente variando el ngulo de encendido de los tiristores. Este mtodo de control es sencillo y se puede aplicar a motores que poseen una jaula de ardilla con resistencia elevada, lo que se consigue utilizando bronce en vez de cobre en la construccin de las barras del rotor (las normas americanas NEMA, National Electrical Manufactures'

Figura 7.70. Controlde velocidadde un motorasncronopor mediode un reguladorde c.a.

CAPTULO 7.

ACCIONAMIENTOS ELCTRICOS

661

Association denominan a esta clase de motor como tipo D). El comportamiento de este tipo de

accionamiento no es muy bueno debido a los armnicos que introduce en la red y por el bajo factor de potencia que se consigue. Las curvas par-velocidad de estos motores se parecen a las que tienen los motores con rotor devanado en los que se haya incluido una resistencia adicional en serie por medio de los anillos; por ello el par mximo se obtiene para deslizamientos altos, del orden del 50 por 100 (s = 0,50) y con deslizamientos a plena carga bastante elevados y que oscilan entre el8 y el15 por 100, por lo que el rendimiento de estas mquinas es pequeo. En la Figura 7.71 se muestran las curvas par-velocidad de un motor asncrono para diversas tensiones de alimentacin. Hay que tener en cuenta que la expresin del par electromagntico producido por un motor asncrono es de la forma: (7.133)

es decir, el par depende del cuadrado de la tensin aplicada al esttor, y es por ello que si las cargas ofrecen un alto par de arranque existirn problemas de regulacin de velocidad. Sin embargo,este sistema puede ser til para cargas con pares resistentestipo ventilador (ventiladores y bombas centrfugas),como el sealadoen la Figura 7.71. Las distintascurvas de par se han calibradoen funcin del porcentajede la tensinproducidafrente a la nominal Vln, lo que correspondeen defmitivaa distintosngulosde encendidode los tiristores.Las velocidadesna' n; nc' . son las diversas velocidadesde equilibrioque se obtienen para las distintas tensiones aplicadas.

7.12.3. Regulacin de velocidad por control de la tensin y frecuencia de lnea. Control escalar
De acuerdo con la ecuacin (7.132), es evidente que un mtodo simple de cambiar la velocidad de giro de un motor asncrono es cambiar la frecuencia de alimentacin 1; que llega al esttor, pues sta modifica la velocidad de sincronismo del campo magntico giratorio y por ende la velocidad mecnica de giro, que es cercana a aqulla en virtud del pequeo valor del
VI

= IOO%Vn

Figura 7.71.

Curvas par-velocidad de un motor asncrono para diversos valores de la tensin.

662

MQUINAS ELCTRICAS

deslizamiento en este tipo de mquinas. Sin embargo, hay que tener en cuenta que el flujo magntico en el entrehierro es directamente proporcional a la f.e.m. inducida en cada devanado del esttor e inversamente a la frecuencia. Recurdese que la f.e.m. El inducida por fase en el devanado l (esttor) es de la forma: (7.134) Por consiguiente, una reduccin en la frecuencia de alimentacinj, produce un aumento del flujo magntico <1>",. Para evitar la saturacin del ncleo magntico debido al aumento del flujo, deber aumentarse proporcionalmente la f.e.m. El' es decir, hay que mantener el cociente EJj constante. En este sistema de regulacin de velocidad se controla, por lo tanto, la magnitud del flujo magntico y por ello recibe el nombre de control escalar, frente al moderno control vectorial, que se explica en el epgrafe 7.12.6, en el que se regula tanto el mdulo como la fase del flujo magntico, lo que se traduce en una mejor respuesta dinmica del motor. Para estudiar el comportamiento del motor para un cociente E/f constante, vamos a considerar el circuito equivalente exacto por fase de un motor asncrono, mostrado en la Figura 7.72, en el que se desprecian las prdidas en el hieno. La f.e.m. El es la d.d.p. en la rama central, por lo que la corriente en la reactancia magnetizante XII vale:
l

El El I El 1 = - = -= -- = constante 1' XII 2nj;LII 2nLII JI

(7.135)

Es decir, si se conserva constante el cociente El/Ji' la corriente magnetizante I1I permanece constante y el flujo magntico mutuo del motor <D", no vara. Vamos a analizar el comportamiento mecnico del motor a travs de sus curvas par-velocidad en el caso de que se mantenga constante el cociente E/JI' De acuerdo con la expresin (4.74) del Captulo 4, el par electromagntico producido por una mquina asncrona vale:

T=--ni 2nDel circuito de la Figura 7.72, la corriente

(7.136)

60
a: (7.137)

1; es igual

'1-

Figura 7.72. Circuito equivalente de la mquina asncrona.

CAPTULO

7.

ACCIONAMIENTOS ELCTRICOS

663

valor que al sustituir en (7.136) nos da: (7.138)

pero teniendo en cuenta que la velocidad de sincronismo ni es igual a 60J/p, la ecuacin anterior se puede escribir, de una forma equivalente, de este modo: (7.139) Para una determinada frecuencia de alimentacin Ji' la f.e.m. El se mantiene constante (en virtud de que el cociente E/JI es constante), y de la expresin anterior se puede calcular el valor del deslizamiento, para el cual se obtiene el par mximo del siguiente modo:
dT -=0 ds

R; s =-= '" X~

(7.140)

que difiere de la (4.77) calculada en el Captulo 4 debido a que all se utiliz el circuito equivalente aproximado del motor. Sustituyendo (7.140) en (7.139) se obtiene el valor del par mximo:
mlP E~ ---l

2nf 2X~

(7.141)

pero teniendo en cuenta que X~ = L~2nJI' siendo L; la inductancia de dispersin del rotor reducida al esttor, el par mximo se puede expresar del siguiente modo: (7.142) de donde se deduce que si el cociente E/JI es constante, el valor del par mximo ser constante para todas las frecuencias de alimentacin al motor. Por otro lado, si el motor trabaja para un deslizamiento inferior a sm' y en especial para bajos deslizamientos, se cumple la desigualdad R~ sX~, por lo que la ecuacin del par (7.139) se transforma en: (7.143) lo que indica que para bajos deslizamientos, el par es proporcional a s, es decir, la curva parvelocidad en esta zona es lineal. En la Figura 7.73 se muestran las curvas par-velocidad de un motor asncrono cuando se mantiene constante el cociente El/JI (curvas E)Jla, Elb/Jlb' EIJJlc' ...), Yen las que se aprecia que el valor del par mximo Tmxpermanece inalterable para las diversas curvas y que la zona situada a la derecha del deslizamiento correspondiente al par mximo es prcticamente lineal (para mayor claridad se han omitido las zonas de las curvas de pares que quedan a la izquierda de los puntos correspondientes a los pares mximos respectivos).

664

MQUINAS ELCTRICAS

Por otro lado, si se tiene en cuenta que segn la ecuacin (4.5) del Captulo 4 el deslizamiento del motor es el cociente entre la frecuencia del rotorj, y la frecuencia del esttorj., al sustituir en (7.143) resulta: (7.144) Es decir, para un mismo par resistente aplicado al motor, la frecuenciade las corrientes del rotor se mantiene constante. Esto es, para los puntos A, B, e, ... de la Figura 7.73, en los que el par es el mismo para las diversas combinaciones E/J;, la frecuenciaj, del rotor es la misma en todos los puntos de trabajo. De aqu se deduce, de acuerdo con la expresin (4.12) del Captulo 4, que al ses ], proporcional a la diferencia entre la velocidad de sincronismo y la velocidad del rotor, denominada velocidad de deslizamiento n2 = ni - n, esta diferencia ser constante para todos los puntos de trabajo que tienen el mismo par, lo que se aprecia claramente en la Figura 7.73. Esta propiedad es muy importante, ya que permite determinar la velocidad a la que trabaja el motor en cada punto, si se conoce el valor de la velocidad de deslizamiento de solamente uno de ellos. Supngase, por ejemplo, que en el caso de la Figura 7.73, para la curva primera situada ms a la derecha que la frecuencia aplicada al motor es la nominalj., = 50 Hz y que la mquina tiene 4 polos (por lo que la velocidad de sincronismo en el punto A, es nla = = 60 . !IJp = 60 . 50/2 = l.500 r.p.m.), si en el punto de trabajo A la velocidad es de l.440 r.p.m., la velocidad de deslizamiento es n2 = l.500 - 1.440 = 60 r.p.m.; si en esta situacin la frecuencia de la alimentacin cambia y pasa a valer t; = 30 Hz, la velocidad sncrona correspondiente al punto El ser de nle = 60,30/2 = 900 r.p.m. y la velocidad del motor en el punto D ser: 900 - 60 = 840 r.p.m., y de un modo anlogo para otras frecuencias de trabajo. En la prctica, la forma ms sencilla de regular la f.e.m. El y su frecuenciaj', asociada es controlando la tensin de alimentacin VI' De acuerdo con el circuito equivalente de la mquina asncrona mostrado en la Figura 7.72, en el circuito del esttor se cumple:

v, = El + RIII + jXIII
y teniendo en cuenta las ecuaciones (7.134) Y(7.144) se puede escribir: 1 IEII 1 IVI RIII jXIIl1 <l>m = 4,44 klNI !I = 4,44 klNI

(7.145)

7:- T -T

(7.146)

+-- Reduccin de frecuencia

t.; = 50 Hz
A

------.

Par resistente T,.

Deslizamiento s Velocidad n

Figura 7.73. Curvas par-velocidad de un motor asncrono para El/JI constante.

CAPTULO

7.

ACCIONAMIENTOS ELCTRICOS

665

A la frecuencia nominal, los trminos RI II/JI Yj XJI /JI son despreciables frente a VJJp por lo que se puede considerar vlida la siguiente aproximacin: (7.147) De este modo, al variar la frecuencia de alimentacin JI siempre que se cambie al mismo tiempo la tensin aplicada VI se conseguir una buena regulacin de la velocidad del motor. A bajas frecuencias la aproximacin anterior no es vlida; tngase en cuenta que el trmino inductivo j XI l/JI = jLdl2nJI I /JI =j Ldl2nII no depende de la frecuencia, por lo que su magnitud seguir siendo despreciable; sin embargo, el trmino resistivo RII/JI aumentar su valor relativo al reducirse la frecuencia. Es por ello que para mantener el mismo flujo magntico en el entrehierro, se deber elevar el cociente V/JI en bajas frecuencias, tal como se seala en la Figura 7.74. La realizacin prctica de este sistema de control requiere el uso de dos convertidores electrnicos: un rectificador controlado y un inversor de conmutacin forzada (jorce-commutated inverter en ingls), tal como se seala en la Figura 7.75. El rectificador controlado transformar la tensin trifsica de la red en una etapa intermedia de c.c., de tal modo que se puede regular la tensin que llega al inversor modificando el ngulo de 'encendido de los tiristores del rectificador controlado. El inversor produce una tensin trifsica cuya frecuencia depende de la secuencia de impulsos que se aplican a las puertas de sus tiristores, de este modo al motor asncrono llega una tensin variable en frecuencia y tensin. Aunque el esquema de la Figura 7.75 suele ser el clsico, en algunos casos se utiliza un rectificador fijo y el control de la tensin se realiza con la ayuda de un chopper colocado en la etapa de c.c. En otros casos el inversor genera una salida de tensin y frecuencia variable empleando tcnicas de modulacin PWM con objeto de que las tensiones sean lo ms senoidales posibles, y que incluyen adems mtodos de control sofisticados para provocar una eliminacin selectiva de armnicos. El circuito Le del esquema de la Figura 7.75 es un filtro para alisar la c.c. que se obtiene a la salida del rectificador controlado. En el caso de que interese que el motor funcione en rgimen regenerativo, deber incorporarse al circuito un inversor adicional, que se seala en la Figura 7.75 por un recuadro discontinuo, y que se conecta en antiparalelo con el rectificador controlado, formando ambos un grupo dual; de este modo, cuando el motor trabaja en rgimen regenerativo (cuadrante II)
1

JI
Figura 7.74. Variacin de la tensin en funcin de la frecuencia en porcentaje de los valores nominales.

666

MQUINAS ELCTRICAS

GRUPO REGENERATIVO FILTRO


L

1 v

!
1 ..

+
e

GRUPO RECTIFICADOR CONTROLADO

GRUPO INVERSOR

Figura 7.75.

Regulacin de velocidad por control

V/JI'

circula una corriente negativa a travs del grupo inversor situado a la salida, que alimenta al motor, y esta corriente pasa por el grupo dual devolviendo energa a la red trifsica a travs del inversor superior. El motor puede funcionar tambin en los cuadrantes III y IV si se invierte la secuencia de fases del grupo inversor, cuya salida se aplica al motor. Las caractersticas par-velocidad de este sistema de regulacin se muestran en la Figura 7.76. Se considera que a la frecuencia nominal o frecuencia base, la tensin aplicada al motor es la tensin nominal. Por debajo de esta frecuencia, el flujo se mantiene constante, variando la frecuencia pero haciendo que sea constante el cociente tensin/frecuencia (recurdese que a bajas frecuencias se aumenta esta relacin), con lo que se consigue que el par mximo sea el mismo para todas las curvas par-velocidad que resultan para las frecuencias inferiores a la nominal o base (vase ejemplo de aplicacin 7.14). Por encima de la frecuencia nominal no se puede realizar el mismo tipo de control de conservar inalterable la relacin V/JI' ya que obligara a aumentar la tensin por encima de su valor nominal, y es por ello que la tensin se fija en el valor nominal y se aumenta la frecuencia paulatinamente en esta zona; esto provoca una disminucin del flujo magntico en el entrehierro, lo que se traduce en una reduccin del valor par mximo en todo el rango de variacin superior,como puede observarse en la parte derecha de la Figura 7.76. Esta accin corresponde al control por reduccin de campo explicado en los motores de c.c., por lo que es posible funcionar en esta zona en un rgimen de potencia constante, como as se ha sealado con las curvasde trazado discontinuomostradasen la Figura7.76. Se observantambin en esta figura las diferentesvelocidadesde funcionamientocuando el motor mueve una carga con par cuadrtico. Cuando se requiere un control muy preciso de la velocidad de un accionamiento movido por un motor asncrono, el circuito de la Figura 7.75 no es capaz de cumplir tal objetivo y debe completarse con algn lazo de realimentacin que incorpore una medida de la velocidad del motor. En la Figura 7.77 se muestra el circuito correspondiente. En este esquema se fija la velocidad de consigna desde el exterior por medio de la tensin de referencia n*; el tacogenerador acoplado al eje del motor produce una f.e.m. proporcional a la velocidad real de la

CAPTULO

7.

ACCIONAMIENTOS ELCTRICOS

667

Par constante T Curva par-velocidad a la frecuencia base

'O E

... o

n Velocidad

Figura 7.76. Curvaspar-velocidad por control VJf

mquina y que se ha denominado n. En la parte inferior izquierda hay un nudo sumador en el que se restan ambas tensiones (velocidades), dando lugar a la tensin de error de velocidad n* - n, que se procesa a travs de un controlador proporcional integral PI para reducir el error a cero y mejorar la dinmica del lazo de control; la parte proporcional de este controlador incluye un factor p/60, siendo p el nmero de pares de polos del motor, para convertir la diferencia de velocidadesmedida en r.p.m. en una frecuencia equivalente del circuito del rotor o frecuenciade deslizamientof2 A continuacin existe un regulador de la frecuencia anterior, que es simplemente un !imitador,que fija el valor de la frecuencia de consigna del rotorf2* para que no sea superior a la que corresponde a la condicin de par mximo, limitando de este modo la corriente del inversor a un valor permisible. A continuacin existe otro nudo sumador que suma la frecuencia1;*con la frecuencia elctrica de giro del rotor np/60, generando de este modo la frecuencia de consigna en el valorfl* = f/ + np/60, que debe producir el inversor que alimenta el motor. Por otro lado, esta sealfl* se lleva a un generador de funciones para producir la seal de referencia de tensin V* que debe aplicarse a la mquina y que regula los impulsos de disparo de rectificador controlado, haciendo que la mquina funcione con el cociente V */1;* constante. Si la frecuencia1;* que llega al generador de funciones es inferior a la frecuencia (velocidad)base, este generadorsuministrauna tensinproporcionalde la forma V* = k . fl* + Yo' donde Vo representa la tensin necesaria que debe aplicarse al esttor, para que el motor trabaje con flujo nominal a velocidad cero, y donde k se elige de tal modo que la mquina trabaje a la tensin nominal o asignada cuando el valor de la frecuencia es la nominal. Cuandofl* es superior a la frecuencia base j" se produce una saturacin del bloque generador de funciones, lo que limita el valor de V * a su valor mximo, que es igual a la tensin nominal. De este modo, por debajo de la frecuencia base se tiene una regulacin simultnea de V* yfl*' la primera de las cuales se utiliza para cambiar la salida del rectificador controlado y la segunda para modificar la frecuencia del inversor, manteniendo en cada momento el cociente V1*/fl* constante. Por encima de la frecuencia base, el valor de V* es constante y la regulacin de velocidad se hace a travs de la frecuencia de salida del inversor y que va fijando la frecuencia fl*' Si se considera que la mquina est girando en una situacin determinada a velocidad n y se aumenta la velocidad de consigna n*, se produce un error de velocidad positivo que satura
1 1

668

MQUINAS ELCTRICAS

Rectificador controlado

Inversor Filtro

~ e:: ~ ... ~

2
v,

ls

-+

t
V1 Va

+
Vee

+
Vec

JI
-+

MOTOR ASNCRONO

VI

Generador impulsos de disparo del rectificador controlado '-----,r-----' Generador de funcin VIJ Velocidad de consigna

Generador impulsos de disparo del inversor '----._-1

'k[.
; i

J; ~---------

.... JI

*
=

np

+ 60

Controlador PI

Reguladordel deslizamiento

Eje

Velocidad real del motor


n

Carga mecnica

Figura 7.77.

Regulacin

V/JI con realimentacin.

el regulador de la frecuencia del rotorf2' dando lugar a una salida mxima def{; de este modo el motor se acelera aumentando la corriente de la mquina hasta el valor mximo permitido por el inversor, lo que da lugar al mximo par de aceleracin posible hasta que se consigue reducir el error de velocidad a cero. El accionamiento que mueve el motor alcanza el rgimen permanente para un valor de fz* para el cual se cumple el balance de pares motor y resistente. El proceso inverso tiene lugar si se reduce la velocidad de consigna n*. Nota informativa: La traccin elctrica espaola con motores asncronos.

a) Locomotora S252 de Siemens. Lnea de alta velocidad Madrid-Sevilla (AVE)


Un ejemplo de regulacin de velocidad de motores asncronos en la traccin elctrica espaola se muestra en la Figura 7.78. Corresponde a la locomotora S252 de Siemens para la lnea de alta velocidad Madrid-Sevilla (AVE). Este coche motor se utiliza para la traccin del Talgo en esta lnea. La alimentacin de la catenaria es una red de c.a. monofsica de 25 kV, 50 Hz. Se tiene un transformador de entrada que alimenta dos circuitos idnticos, que constan a su vez de un rectificador controlado monofsico para transformar la c.a. de 25 kV de la alimentacin de la catenaria en c.c. a 2,6 kV (etapa intermedia), y un inversor trifsico se encarga de convertir esta corriente continua en c.a. trifsica de amplitud y frecuencia variable utilizando

CAPTULO 7.

ACCIONAMIENTOS ELCTRICOS

669

Catenaria 25 kV; 50 Hz

}~
Transformador de entrada

}~
controlado Chopper Inversor de frenado 25 kV- 50 Hz 'V 1.400 kW; 2.050 V; 530 A c.u.

Figura 7.78.

Esquema elctrico de la locomotora Siemens S252.

la modulacin de anchura de impulso (PWM). Cada equipo alimenta a dos motores trifsicos asncronos (situados en un mismo boje del coche motor) con una potencia de 10400 kW cada uno, lo que representa un total de 2.800 kW por boje, es decir, 5.600 kW en total. La velocidad mxima del tren es de 220 km/h. Existen centros de transformacinen el recorrido (separados entre s unos 40 km) alimentados por redes de muy alta tensin (de 220 kV) para reducir al mximo las interferencias con los sistemas elctricos cercanos; en cada centro se dispone de dos transformadores monofsicos de 20 MVA cada uno y salida a 25 kV, que alimentan transversalmente a dos catenarias distintas.

b) Tren ICE 350E de Siemens. Lnea de alta velocidad Madrid-Barcelona (AVE)


El ICE 350E de la compaa alemana Siemens es una evolucin del ICE 3 de la misma empresa, capaz de alcanzar los 350 km/h y que incorpora el concepto de traccin distribuida. Sin cabezas tractoras, todo el equipo elctrico va repartido a lo largo del tren, bajo el bastidor de todos los coches y con un 50 por 100 de los ejes motorizados. Este sistema, al margen de aumentar el espacio disponible para los viajeros, permite una mejor adherencia, mayor aceleracin y capacidad de superar pendientes ms pronunciadas. El tren tiene una longitud de de 200 m y 8 coches, con un peso mximo por eje de 15 toneladas. Dispone de 16 motores asncronos de 550 kW, lo que da lugar a un total de 8.800 kW. El sistema de alimentacin a la catenaria es el denominado 2 x 25 kV, 50 Hz. La alimentacin a los trenes se realiza a travs de la catenaria a +25 kV, la lnea de retorno puesta a tierra y un alimentador o feeder negativo a -25 kV. Este sistema 2 x 25 kV implica el

670

MQUINAS ELCTRICAS

tendido de un conductor negativo a lo largo de la lnea que, en combinacin con el conductor de retorno, debidamente conectado a los carriles a distancias regulares, produce el efecto de disminuir las perturbaciones electromagnticas originadas por el sistema. A lo largo de la lnea exiten ocho subestaciones elctricas de traccin en las que se dispone de dos transformadores monofsicos de 60 MV A en cada una. Seis subestaciones se alimentan de 400 kV Y dos de ellas a 220 kV. Para conseguir que la corriente de retorno de cada tren circule por el feeder negativo, se colocan autotransformadores cada 15 km conectados entre catenaria y feeder negativo con su punto medio unido al carril. Otro sistema de regulacin que se utiliza para el control de los motores asncronos es el uso de cicloconvertidores. En la Figura 7.79 se muestra de una forma simplificada el esquema correspondiente; en este caso dicho sistema electrnico produce una tensin trifsica variable en amplitud y frecuencia directamente, sin que se requiera una etapa de c.c. intermedia. El inconveniente de los cicloconvertidores es que el rango de variacin de la frecuencia producida vara solamente entre O y 1/3 de la frecuencia de alimentacin de aqullos. Este sistema de regulacin se emplea principalmente en motores de gran potencia, debido a la complejidad del equipo de control de disparo de los tiristores. Obsrvese que se requieren 12 tiristores por cada fase, que deben ir perfectamente desfasados y al mismo tiempo sincronizados entre s. 7.14

EJEMPLO

DE APLICACIN

Se dispone de un motor asncrono trifsico de 4 polos que tiene los siguientes parmetros del circuito equivalente medidos a 50 Hz: R, = 0,1 ohmios; R; = 0,2 ohmios; X, = 0,4 ohmios; X; = 0,35 ohmios.
RED TRIFSICA

ill
Devanado secundario 1

Devanado primario Devanado secundario 2 Devanado secundario 3

Figura 7.79.

Control de velocidad por medio de un cicloconvertidor.

CAPTULO

7.

ACCIONAMIENTOS ELCTRICOS

671

Se desprecian las prdidas mecnicas y la rama paralelo del circuito equivalente. Calcular la velocidad de sincronismo, velocidad para par mximo y par mximo correspondiente y par de arranque, cuando las tensiones por fase y frecuencias aplicadas al motor son: a) V, = 200 voltios;f, =50 Hz; b) V, = 160 voltios; i, =40 Hz; e) V, = 80 voltios; f, = 20 Hz; d) V, = 40 voltios; f, = lO Hz; e) V, = 200 voltios; I, = 60 Hz. 2. Qu tensin debera aplicarse al esttor (porfase) para el caso anterior e) para que el par mximo producido coincida con el que se obtiene a 50 Hz? SOLUCIN
la)

l.

Tensin: V) = 200 voltios; frecuencla j, = SOHz

La velocidad de sincronismo viene expresada por:


ni =--

60fl 6050 =-= l.500 r.p.m. p 2

El deslizamiento para par mximo vale:


s =
m

R~ JR2 I + (XI +X')2 2

=--;====

0,2

JO , 12 + O75 2 ,

Por consiguiente, la velocidad para par mximo ser:


n = ni (1 -

s) = l.500 (1 - 0,264) = 1.103,5 r.p.m.

y el par mximo tendr un valor:

445,9 N.m

El par de arranque se obtiene de la expresin general del par, haciendo s = 1, resultando un valor:
T= 3R'V2
2 I

=>

2n;~s'[(RI+
=> T =
a

~~)+(XI+X~?J
= 234,16 N.m

3 0,2' 2002 2n --,1 .[(0,1 + 0,2i + 0,752] 600 1500

lb) Tensin: V) = 160 voltios; frecuencia f = 40 Hz El proceso de clculo es idntico al anterior pero teniendo en cuenta que al cambiar la frecuencia de alimentacin se modifican las reactancias de la mquina. En este caso se tiene:

XI + X2'

= X = 0,75 (e

40 50 = 0,6 ohmios

672

MQUINAS ELCTRICAS

y aplicando las mismas expresiones que en el apartado anterior resulta:


ni

1.200 r.p.m. ;

nmx

= 805 r.p.m.

; Tmx

= 431,43 N.m

T, = 275,62 N.m

Se observa que el par mximo se reduce levemente respecto al caso anterior y que el par de arranque aumenta porque la curva par-velocidad correspondiente se ha desplazado a la izquierda respecto al caso anterior. le) Tensin: VI = 80 voltios; frecuencia j', = 20 Hz

Ahora la reactancia de cortocircuito a esta frecuencia vale: XI + X;

= Xc, = 0,75 50 = 0,3 ohmios = 367,08 N.m


; T,,= 339,53 N.m

20

y aplicando las mismas expresiones que en el apartado anterior resulta:


ni

= 600 r.p.m.

nmx

= 220,53 r.p.m. ; Tmx

se observa que a esta frecuencia tan reducida el par mximo se reduce sensiblemente respecto al caso a) de frecuencia nominal de 50 Hz. Id) Tensin: VI = 40 voltios; frecuenca f = 10 Hz

La reactancia de cortocircuito a esta frecuencia vale: XI + X;

= Xcc = 0,75 ~ = 0,15 ohmios


50 r.p.m. ; Tmx

10

y aplicando las mismas expresiones que en el apartado anterior resulta:


ni

= 300 r.p.m.

nmx

= -32,82

= 272,57 N.m

; Tu

= 271,62 N.m

Se observa que a esta frecuencia tan reducida el par mximo es muy inferior al caso a) de frecuencia nominal de 50 Hz. le) Tensin: VI

= 200 voltios; frecuenca j', = 60 Hz


XI + X;

En este caso la reactancia de cortocircuito a esta frecuencia vale:

= X" = 0,75 ~ = 0,9 ohmios


50

60

y aplicando las mismas expresiones que en el apartado anterior resulta:


ni =

1.800 r.p.m. ; nmx = 1.402,4 r.p.m. ; Tmx = 316,56 N.m ; Ta = 141,47 N.m

Se observa que para esta frecuencia superior a la nominal tambin el par mximo se reduce respecto al obtenido en el caso a) de frecuencia nominal de 50 Hz. Es instructivo que el lector dibuje las curvas par-velocidad que se obtienen en este problema y las compare con las curvas de la Figura 7.76. En esta figura el par mximo se conserva constante paraJ < Jbase debido a que el flujo se supone constante o de otro modo el cociente E/JI' pero en este problema se ha conservado constante realmente el cociente V/JI' se excepta el caso e) y por ello las curvas par-velocidad que quedan a la izquierda de la correspondiente a la frecuencia base (de 50 Hz) van disminuyendo su par mximo conformedisminuye la frecuencia de alimentacin.

CAPTULO

7.

ACCIONAMIENTOS ELCTRICOS

673

2. De acuerdo con los resultados expuestos en el apartado anterior, para que el par mximo se conserve a frecuencias inferiores a la nominal, es preciso aumentar la relacin VI/JI correspondiente; en el caso que se solicita en el problema se tiene: Caso a: Caso c:
VI = 200 voltios; JI = 50 Hz. Tmx = 445,9 VI = 80 voltios.j', = 20 Hz. Tmx = 367,08

N.m N.m

El par mximo es proporcional al cuadrado de la tensin aplicada, es decir:


Tmx =
ni 2 [ RI + 2n 60

3V~

JRI

= kVI + (XI + X2)

12J

Como quiera que con 80 V, 20 Hz se tiene un par mximo de 367,08 N.m, para conseguir un par mximo de 445,9 N.m a 20 Hz se requerir una tensin VI que se deduce a continuacin:

El lector puede comprobar que para una frecuencia de 10Hz, la tensin necesaria en el esttor para conservar el par mximo, teniendo en cuenta los resultado del apartado Id), sera:
--

445,9 = 51,16 voltios 272,57

Estos resultados justifican la forma de la curva de la Figura 7.74, que a bajas frecuencias aumenta la relacin V/JI'

7.12.4. Regulacin de velocidad por control esttico de una resistencia adicional en el rotor
En el epgrafe 4.9.2 del Captulo 4 se estudi el arranque de motores asncronos con rotor devanado, en el que se introduca un restato trifsico en los anillos del rotor para reducir la corriente de arranque. Es evidente que al variar la resistencia del restato se puede regular la velocidad de giro del motor. En la Figura 4.34 se representaron las curvas par-velocidad para cuatro resistencias adicionales distintas. La resistencia externa se puede variar estticamente empleando el circuito mostrado en la Figura 7.80. En este montaje, la potencia que llega al rotor se rectifica por medio de un puente trifsico de diodos y la inductancia L se utiliza como elemento de filtro para alisar la c.c. que sale del rectificador. La resistencia externa Rex est en paralelo con un chopper, de tal modo que el valor de la resistencia efectiva que se aade al rotor depende del parmetro k o ciclo de trabajo del chopper. Recurdese, de acuerdo con el epgrafe 7.7, que el ciclo de trabajo de un chopper o troceador est definido por:
k=T
tON

(7.148)

siendo tON el tiempo de cierre del interruptor esttico S y T el perodo del mismo. La frecuencia del chopper J = liT se elige cercana a 1 kHz.

674

MAQUINAS ELCTRICAS

RO-----~~--------------s 0-------+-..... -----TO-----~~~_.----------Red trifsica de c.a. MOTOR ASNCRONO DE ROTOR DEVANADO

Anillos deslizantes

Filtro

+
G --

ro

Q)

>< Q)
ro ' c: ';:;

~
Q)

c:r:

Rectificador

Chopper

Figura 7.80. Regulacin de velocidad con chopper y resistencia externa en el rotor.

Si en el circuito de la Figura 7.80 se desprecia el rizado de la corriente (c' la energa absorbida por la Rex durante un ciclo de funcionamiento del chopper viene definida por: (7.149) lo que significa que la potencia absorbida por la resistencia externa durante el perodo T vale: (7.150)
y teniendo en cuenta la definicin (7.148) se puede escribir:

(7.151) donde Ri, representa la resistencia efectiva externa, que vale: (7.152) Cuando k = O,el interruptor esttico del chopper est abierto todo el tiempo y se tiene R,~ = Rex Ycuando k = 1el interruptor esttico S est cerrado, por lo que la resistencia externa queda cortocircuitada y = O;en este caso la resistencia total del rotor por fase es nicamente la resistencia de cada fase del devanado rotrico del motor. Al variar k entre Oy 1, la resistencia efectiva vara en el rango O< < Rer' dando lugar a las curvas par-velocidad mostradas en la Figura 7.81.

R:;

CAPTULO

7.

ACCIONAMIENTOS ELCTRICOS

675

._._.-

TI'

Figura 7.81.

Curvas par-velocidad para diversos valores del ciclo de trabajo k del chopper.

Si como se seal en el Captulo 4, se denomina E2 a la f.e.m. inducida Rorfase en el rotor en reposo y la f.e.m. inducida en el rotor mvil con deslizamiento s se denomina E2s' la relacin entre ambas f.e.m.s. se seal en (4.10) y vale: (7.153) La frecuencia del rotor J2 respecto de la frecuencia de las corrientes del esttor JI' para un deslizamiento s vale: (7.154) La tensin rectificada V;,c del circuito de la Figura 7.80, teniendo en cuenta (7.43), es: (7.155) donde se ha tenido en cuenta que v'/1 = fiE2s = sfiE2, que es la tensin mxima por fase que alimenta al puente de diodos. Si se denomina P" a lapoteneia del entrehierro (es decir, la potencia que se transfiere del esttor al rotor), la potencia disipada en el circuito del rotor, de acuerdo con (4.68), vale: (7.156) donde sPa se denomina potencia de deslizamiento. Si se desprecian las prdidas en el cobre del devanado del rotor frente a la potencia disipada en la resistencia externa Rex' se puede escribir: (7.157) y teniendo en cuenta que segn (4.73) se cumple:
P = T 2n 11

111

60

= T w

(7.158)

676

MQUINAS ELCTRICAS

donde w, representa la velocidad de sincronismo del campo giratorio en rad/s, al sustituir (7.158) en (7.157) y teniendo en cuenta (7.155), resulta: (7.159) es decir: (7.160) lo que significa que el par electromagntico desarrollado por el motor es proporcional a la corriente continua rectificada en el circuito del rotor, o de otro modo, la corriente lcc es una imagen del par motor, consecuencia importante a tener en cuenta cuando se realizan regulaciones de velocidad utilizando sistemas realimentados. Este mtodo de regulacin de velocidad por control esttico de la resistencia del rotor tiene el grave inconveniente de que el rendimiento es pequeo, sobre todo con altos deslizamientos; sin embargo, el sistema tiene una electrnica muy simple y por ello encuentra aplicaciones en mecanismos de elevacin y transporte tales como gras, cabrestantes, montacargas, etc.

7.12.5. Regulacin de velocidad por recuperacin de la potencia de deslizamiento


En el sistema de regulacin analizado en el epgrafe anterior, la potencia de deslizamiento sPa se transformaba, segn (7.156), en potencia disipada en las resistencias elctricas del rotor y de ah su bajo rendimiento por la gran energa disipada en prdidas por efecto Joule del rotor. Tngase en cuenta que el efecto de la resistencia o resistencias externas es producir tensiones a la frecuencia j, = si, que se oponen a las f.e.m.s. inducidas en los devanados rotricos. Sin embargo, si en vez de colocar un restato de arranque en los anillos del rotor, se introducen en los mismos una f.e.m. trifsica a la frecuenciaj', = si, que tenga la posibilidad de ajustar tanto su magnitud como su fase, se podr aumentar el rendimiento del conjunto. La forma prctica de introducir esta f.e.m. externa es mediante un montaje denominado recuperacin de la energa de deslizamiento, por el que la potencia sPa se devuelve a la red. En la historia de la ingeniera elctrica se han empleado diferentes esquemas para regular el deslizamiento de una forma activa utilizando la conexin en cascada de varias mquinas elctricas rotativas, como los sistemas Kramer (1908), Scherbius (1912) Ytambin el motor Schrage (1914) de escobilllas deslizantes, todos ellos en la actualidad ya obsoletos. Sin embargo, la electrnica de potencia ha permitido la reproduccin de alguno de ellos sin utilizar mquinas electromecnicas. En la Figura 7.82 se muestra el accionamiento elctrico Kramer esttico, que permite una regulacin de velocidad por debajo de la correspondiente al sincronismo y de ah que algunos autores denominen a este montaje como cascada subsncrona. Como se indica en la Figura 7.82, el sistema esttico Kramer tiene un rectificador en puente trifsico que transforma la c.a. del rotor a la frecuencia de deslizamiento 12 = sf, en c.c. y dispone de una inductancia de filtrado y un inversor conmutado por lnea con tiristores que devuelve la energa del rotor a la red a travs de un transformador trifsico. Si se supone que en el motor asncrono los factores de devanado de esttor y rotor son iguales entre s (o

CAPTULO 7.

ACCIONAMIENTOS ELCTRICOS

677

S T

Red trifsica de c.a.

/11

VI(tensin simple de la red) MOTOR ASNCRONO DE ROTOR DEVANADO Filtro


L

sPu

ir

VI

lp

Primario (NI espiras)


mT=N/N2

o::
O

el

Secundario (N2 espiras)

~ ~
Z

-<

o::
O
(1)

E2s = sEz

sr,

sPa

/21

=t>

=t>

lt

-< o::

RECTIFICADOR

INVERSOR

Figura 7.82. AccionamientoKramer esttico.

simplemente que los factores son iguales al), entonces la relacin de transformacin entre esttor y rotor es la relacin de espiras, es decir:
El NI ~=~=m E2 Nz y teniendo en cuenta (7.153), la f.e.m. por fase en el rotor mvil ser igual a:

(7.161)

E2" = sE2 =

El
S~

(7.162)

Si se denomina VI a la tensin eficaz por fase aplicada al esttor del motor y se desprecian las cadas de tensin de los devanados del esttor y del rotor, entonces El = VI' por lo que la tensin Ve, que se obtiene a la salida del puente rectificador de la Figura 7.82, de acuerdo con (7.155) y (7.162), ser:
Vcc=----

3J6 sV
n

(7.163)

Si se consideraahora el inversorde la Figura 7.82 conectadoa la red a travs de un transformador de relacin mT (es importanteobservaren la Fig. 7.82 que se ha denominadoprimario al devanado conectadoa la red y secundario al devanadoque est unido al inversor,aunquela transferenciade energava en sentido inverso),la tensin V del inversorest relacionadacon la tensin simple de la red VI y con el ngulo 1/. de encendido de los tiristores por la expresin (7.64b):

(7.164)

678

MQUINAS ELCTRICAS

Si se desprecia la resistencia de la inductancia de filtrado, entonces en la malla elctrica intermedia de c.c. se cumple:

\1;, + Vi = O
y sustituyendo (7.163) y (7.164) en (7.165) resulta:

(7.165)

--

3)6
JI

-- + -m

sV

3}6 -VI
JI

cos

"Y.

=O

m[

m s = - - cos
f/11

"Y.

= -m.1 . cos

"Y.

(7.166)

donde 111;, = m/mi., El ngulo y. de encendido debe ser superior a 9()0 como as le corresponde a un rectificador controlado trabajando en el modo inversor. De este modo, segn (7.166) al variar y. entre 90 y 18()0, el deslizamiento s cambiar entre O Y mA Si se elige m = 111[, entonces m;, = 1 Y se podr variar el deslizamiento del motor entre Oy 1; es decir, la velocidad del motor se podr regular entre la velocidad de sincronismo y cero. Por consiguiente, la velocidad del motor se puede regular en la zona subsncrona simplemente ajustando el ngulo de encendido de los tiristores del inversor. Si se siguen despreciando las prdidas de potencia del motor, la potencia de deslizamiento vale, segn (7.157):

sp" = \l;J

===>

P = --lf

v.:
S

(7.167)

Por consiguiente, el valor del par electromagntico ta adems (7.158), ser:

producido por el motor, teniendo en cuen-

p"
(1) I

\1;.,1,,,
.1'(1) I

(7.168)

donde (J), es la velocidad angular de sincronismo del campo magntico giratorio en rad/s. Sustituyendo (7.163) en (7.168) resulta: T

= 3}6 sV

= 3)6 Vf"

(7.169)

que nos indica que el par es proporcional a la corriente continua 1". En la Figura 7.83 se muestran las curvas par-velocidad para diversos ngulos de encendido (n, es la velocidad de

Par resistente nominal T,.

Velocidad nl_

Figura 7.83.

Curvas par-velocidad

para diferentes ngulos de encendido.

CAPTULO

7.

ACCIONAMIENTOS ELCTRICOS

679

sincronismo nominal), que son similares a las de un motor de c.c. con excitacin en derivacin o tambin con excitacin independiente, en este caso para diversas tensiones aplicadas al inducido. Este mtodo de regulacin de velocidad se utiliza en mquinas de gran potencia (MW) en las que un ajuste de la velocidad en amplios rangos da lugar a una gran potencia de deslizamiento. Si el rango de variacin es pequeo, entonces el sistema Kramer tiene la ventaja de que la potencia de los convertidores electrnicos es mucho menor que la potencia del motor asncrono, ya que se disean solamente para la potencia de deslizamiento de la mquina; por ejemplo, si slo se desea regular la velocidad en el rango comprendido entre el 80 y 100 por 100 de la velocidad de sincronismo, la potencia de los convertidores electrnicos ser solamente el 20 por 100 de la potencia del motor (ya que el deslizamiento mximo previsto es del 20 por 100). El sistema Kramer esttico se puede modificar para que funcione por encima de la velocidad de sincronismo, lo que requiere que la potencia de deslizamiento pueda fluir en ambos sentidos, es decir, desde el rotor del motor a la red o desde la red al rotor. Para efectuar esta doble conversin, se debera sustituir el rectificador fijo de la Figura 7.82 por un rectificador controlado y de este modo ambos convertidores controlados de la Figura 7.82 pueden funcionar en rgimen de rectificacin o de inversin. Esta idea es difcil de implementar en la prctica por las diferentes frecuencias a las que deben trabajar ambos rectificadores controlados, que provoca problemas de sincronizacin de seales en las puertas de los tiristores, y es por ello que en la prctica se sustituyen ambos convertidores por un cicloconvertidor, de acuerdo con el esquema de la Figura 7.84. Este montaje se denomina accionamiento Scherbius esttico. El cicloconvertidor permite que la potencia de deslizamiento circule en ambos sentidos y de este modo se puede regular la velocidad de la mquina tanto en la zona de velocidades subsncronas como supersncronas.

EJEMPLO DE APLICACIN 7.15 Un motor asncrono trifsico de 4.000 kW, 5 kV, 50 Hz. JO polos de rotor devanado, se utiliza para mover una bomba centrfuga a velocidad variable con un sistema de accionamiento
R S
T

Red trifsica de c.a.

ANILLOS ROZANTES

~.....,

MOTOR ASNCRONO ROTOR DEVANADO

qMKP
4

O
r--- .......
SPa Flujo de potencia

o::: O el

~
C/)

<C

o:::
Z <C
f-

CICLOCONVERTIDOR

t~

O u,

o:::

Figura 7.84.

Accionamiento Scherbius esttico con cicloconvertidor.

680

MQUINAS ELCTRICAS

Kramer esttico anlogo al mostrado en la Figura 7.82. Tanto el esttor como el rotor del motor estn conectados en estrella. Cuando el motor est parado y se aplica al esttor la tensin nominal de 5 kV, se mide entre dos anillos del rotor una tensin de 2.000 V. VII transformador trifsico YyO de relacin 5.000/500 V se conecta entre la lnea y el inversor. Si \ el motor desarrolla una potencia mecnica de 1.200 kW a 480 r.p.m., calcular: a) potencia que devuelve el rotor a la red a travs del inversor; b) tensin entre los anillos del rotor; c) corriente lcc de la etapa de c.c.; d) ngulo de encendido del inversor; e) corrientes eficaces en el primario y secundario del transformador. NOTA: Suponer que el motor y el transformador son ideales y sin prdidas. SOLUCIN
a)

La velocidad de sincronismo del motor vale:


ni = --

60fl 6050 = -= 600 r.p.m. p 5

El deslizamiento del motor en las condiciones especificadas vale:


s=--=

nl-n ni

600-480 600

=20%

como quiera que la potencia mecnica interna del motor es igual a 1.200 kW, la potencia de entrehierro vale:
P'ni = Pa (1 - s)
y =>

1.200 = P, (l - 0,2)

=>

P = -a

1.200 = 1.500 kW 0,8

por consiguiente, la potencia de deslizamiento, que es la potencia que devuelve el motor a la red, es: sPa

= 0,2 . 1.500 = 300 kW

b)

La f.e.m. de lnea que se induce en el rotor cuando la mquina est parada es de 500 Y, por lo que la relacin de transformacin del motor asncrono vale:
El VI m=~=~=

5.000/)3 2.000/)3

E2

E2

=25

Como quiera que la tensin simple de la red es de 5.000/)3, (7.163), la tensin de c.c. que sale del rectificador vale:

y de acuerdo con

V c)

ce

3y' o sV = --- = 3j66


I

t:

0,2m n

5.000 3 )3:::::; 540,2 voltios

2,5

La potencia de deslizamiento, de acuerdo con (7.167), se puede escribir: sPa = VeJec


=>

300.000 = 540,2 . lcc

=> (,

300.000 = 555,36 amperios 540,2

CAPTULO

7.

ACCIONAMIENTOS ELCTRICOS

681

d)

Para calcular el ngulo de encendido de los tiristores aplicamos la expresin (7.166), resultando:
m s : mT cos rx= - -= - ---m 2,5

5.000 0,2.-5-00= -0,8


=>

s=- e)

mT

cos rx

=>

rx= 143,13

La corriente Icc de la etapa de c.c. se ha calculado en el apartado c) y vale 555,36 amperios. Para calcular la corriente en el lado de corriente alterna del inversor (que corresponde a la corriente eficaz en el secundario del transformador) debemos aplicar la expresin (7.69), resultando ser:

t, =

A A
i: =
----

553,36 = 453,17 amperios

y por consiguiente, la corriente primaria del transformador, supuesto ste ideal, ser: 453,17 5.000/500
= 45,317 amperios

7.12.6. Control vectorial de motores asncronos


El control vectorial (vector control en ingls), denominado tambin control por orientacin decampo (jield oriented control), constituye el mtodo de regulacin de velocidad ms sofisticadoy moderno de los motores asncronos. La base de este mtodo de regulacin es controlar tanto la magnitud como la fase del flujo magntico del motor asncrono para conseguir un funcionamientoanlogo al que tienen los motores de c.c. y que hasta fechas muy recientes hanrepresentado el motor por excelencia en los accionamientos elctricos de velocidad variable. De acuerdo con el epgrafe 6.8 del Captulo 6, en un motor de c.c. el valor del par electromagnticoproducido por esta mquina es de la forma: (7.170) De la ecuacin anterior se deduce que la regulacin del par de un motor de c.c. se puede realizarmodificando bien sea el flujo magntico ID,la corriente que atraviesa el inducido I, o de ambas magnitudes a la vez. Para mejorar la dinmica del sistema generalmente la mejor regulacinse realiza manteniendo constante el flujo magntico y variando la corriente que atraviesael inducido, en virtud de que la inductancia del devanado inducido es mucho menor quela del inductor, lo que significa que la constante de tiempo del inducido es menor que la delinductor, lo que se traduce en una respuesta ms rpida. Si se considera el esquema de la Figura 7.85a, en que se representa un motor de c.c. con excitacinseparada, se observa que en este motor se tiene:
a)

Un campo magntico estacionario producido por el devanado de excitacin o campo inductor que lleva una corriente le. Este devanado produce un flujo magntico IDa lo largo del eje de los polos, que se denomina eje directo o eje d del motor. b) Para generar un par electromagntico en los conductores del rotor, se debe hacer circular una corriente por el devanado inducido l tal como se seala en la Figura 7.85a. Debido a que el rotor lleva un colector de delgas ya la situacin del eje de escobillas que lleva el conmutador anterior y que se sita en la lnea neutra de la mquina, las corrientes del inducido producen una f.m.m. que tiene lugar en el eje de

682

MQUINAS ELCTRICAS

Eje q

Par electromagntico

Eje de la f.m.m. del inducido

Alimentacin del inductor


b) Fasores especiales de tlujo y f.m.m.

a) Esquema de devanados

Figura 7.85.
Q

Motor de c.c. con excitacin independiente.

escobillas, es decir, a 90 del eje magntico del inductor, y que se conoce como eje en cuadratura o eje q. En la Figura 7.85b se muestra para mayor claridad los fasores espaciales de flujo y fm.m. de un motor de c.c. y que conservan sus posiciones constantes en el espacio (es decir, realmente son vectores espaciales). De acuerdo con la expresin (7.170), la produccin del par en un motor de c.c. es proporcional al producto del flujo magntico del inductor, que a su vez es proporcional a la corriente de excitacin le Yde la corriente que atraviesa el inducido l, que a su vez es proporcional a la f.m.m. de este devanado. Debido a que la f.m.m. del inducido es perpendicular al flujo magntico del inductor, la variacin de la corriente que circula por este devanadono modificaro afectar al campo magnticoinductor(se considerandespreciableslos efectosde la reaccinde inducido);se dice por ello que las corrientesdel inductory del inducido estn desacopladas, de este modo manteniendola corrientede excitacinconstante,los cambios del par seguirn los cambios en la corriente aplicada al inducido instantneay exactamente. En un motor asncrono la situacin es mucho ms complicada que en un motor de c.c. En este caso hay un devanado trifsico en el esttor, y la f.m.m. producida por este arrollamiento es giratoria, es decir, se mueve a la velocidad angular de sincronismo w, = 21[f" siendoj, la frecuencia de las corrientes estatricas. El rotor gira a la velocidad angular mecnica wm = 2nn/60 (donde n se mide en r.p.m.), que corresponde a una velocidad angular elctrica pWm' siendo p el nmero de pares de polos del motor; pero teniendo en cuenta que la pulsacin de las corrientes del rotor es igual a w2 (que se denomina pulsacin de deslizamiento), la fm.m. del rotor gira a una velocidad total elctrica w2 + pWm que es igual a w,; de este modo las f.m.m.s. del esttor y del rotor estn enclavadas elctricamente (a esta igualdad la designan algunos autores como condicin de autopilotaje del motor asncrono). La interaccin de estas dos f.m.m.s. produce la f.m.m. resultante de imanacin, que crea a su vez el fujo mximo en el entrehierro de la mquina. En la Figura 7.86a se muestra la orientacin del campo magntico (flujo) y de las f.m.m.s. en un motor asncrono, que es una situacin espacial ms general que la estudiada en el epgrafe 4.16 del Captulo 4. Debe destacarse que el ngulo que forma el fasor espacial de fm.m. del esttor y del flujo en el entrehierro, y que se ha designado por b. en la Figura 7.86a, no es necesariamente de 90 como sucede en el motor de c.c. Con
1

CAPTULO

7.

ACCIONAMIENTOS ELCTRICOS

683

Red trifsica

Eje

:l'

~ w=o
al Fusores espaciales del motor asncrono

~Qe-

<l)n

~.__"""')1 ' Eje d del fasor espacial de corriente del estror

b) Componentes

Figura 7.86. Fasores espaciales de f.m.m. en un motor asncrono. objeto de identificar las posiciones que van siguiendo los fasores espaciales de f.m.m. del motor asncrono, se asigna en la Figura 7.86a un sistema de coordenadas estacionario ([J, x), de tal modo que el eje /J coincide con la horizontal mientras que el eje (X se sita 90 retrasado respecto del anterior. La produccin de un par electromagntico en el motor asncrono procede, como sabemos, del esfuerzo de alineacin de las f.m.m.s. de la mquina, o de otro modo ms visual, de la atraccin y repulsin de los polos equivalentes que se forman, y que se muestran en la Figura 7.86a. Recurdese, de acuerdo con la expresin (4.215) del Captulo 4, que el par electromagntico producido por un motor asncrono viene expresado por:
T = kr<1>",./, sen C),

(7.171)

donde se ha denominado ,J;' al fasor espacial representativo de la f.m.m. del esttor y (;, al ngulo que forma el fasor anterior con el de flujo magntico en el entrehierro. Como quiera que ,J;' es proporcional a la corriente del esttor 1" la ecuacin anterior se puede escribir de un modo equivalente: (7.172) De acuerdo con la Figura 7.86b, el fasor 1,puede descomponerse en dos partes: Id = (. cos (;, paralelo al eje de campo o eje directo d, que gira a la velocidad de sincronismo (J)I del campo giratorio, e 1'1 = 1, sen ()., perpendicular al eje anterior y denominado eje cuadratura o eje q *. A estas corrientes se las conoce como componentes del campo orientado. De este modo (7.172) se transforma en:
T=Kr<1>",1q (7.173)

En la Figura 7.86b se observa que la componente Id est en fase con el flujo magntico, por lo que representa la componente de la corriente del esttor responsable de producir el

* Se advierte al lector que la denominacin e incluso la situacin relativa de los ejes estacionarios ejes mviles q y d son muy dispares en la bibliografa tcnica y varan de unos textos a otros.

{i y

'l.

Yde los

684

MQUINAS ELCTRICAS

flujo magntico y es anloga a la corriente de excitacin le del motor de c.c. Por otro lado, la componente Iq de la corriente del esttor est en cuadratura con el flujo, y si se comparan las ecuaciones (7.170) Y (7.173), la corriente Iq es anloga a la corriente de inducido I del motor de c.c. que se utiliza para controlar el par del motor. Pese a esta analoga aparente, hay diferencias sustanciales entre el motor de c.c. y el motor asncrono que conviene destacar: en el motor de c.c. el eje de flujo es fijo (eje directo) y la f.m.m. del inducido tiene lugar a 90 respecto del flujo magntico (eje en cuadratura), en tanto que en el motor asncrono el flujo magntico es giratorio, por lo que los ejes directo y cuadratura de la Figura 7.86b se estn moviendo a la velocidad de sincronismo y el ngulo bs que forman la f.m.m. resultante del esttor .C;:; y el flujo mximo del entrehierro <1>m no es constante ni igual a 90, sino que depende del par que produce la mquina; por otro lado, en el motor de c.c. las corrientes del inductor y del inducido circulan por devanados distintos y accesibles desde el exterior, aspecto que no es posible conseguir en el motor de induccin, ya que no se puede actuar separadamente sobre las corrientes generadoras del par Iq y de flujo Id' En el diagrama de fasores espaciales de la Figura 7.86b se han sealado los ejes estacionarios p y IX, los ejes mviles q y d que giran a la velocidad de sincronismo (VI' la posicin del eje mecnico del rotor que gira a una velocidad angular elctrica P(Vm' el fasor I, equivalente del esttor y sus componentes (/p, la) en el sistema de referencia (P, IX) e (Iq, Id) en el sistema de referencia mvil (q, d). Tambin se indica la posicin espacial del flujo de entrehierro <1>m que define el eje d mvil (eje directo). Se han destacado en la Figura 7.86b los siguientes ngulos elctricos: 1) ngulo de deslizamiento (}2' que forma el eje sncrono q con el eje del rotor; 2) ngulo del rotor ()r' que forma el eje del rotor con el eje estacionario P del esttor; 3) ngulo del campo ()e' que es el formado por el eje d del flujo y el eje estacionario IX. La relacin entre estos ngulos es ()e = (}2 + ()r' Todos los ngulos anteriores dependen del tiempo. Se ha sealado tambin el ngulo de carga bs que forma I, Y <1>m' que es un ngulo que no depende del tiempo, pero que depende del par que desarrolla el motor, tal como seala la expresin (7.172). Como ya se ha indicado antes, la base del control vectorial es copiar el funcionamiento del motor de C.C., y para resolver el problema de control se deben mantener en cuadratura las componentes de imanacin Id y de par Iq de la corriente estatrica, desacoplando ambas componentes de forma que se puedan ajustar independientemente una de otra. En definitiva, en un sistema de control vectorial de motores asncronos hay que controlar en tiempo real la magnitud y fase de las corrientes de alimentacin del esttor, en respuesta a cambios en las demandas de velocidad y de par requeridas por el accionamiento. Es por ello que en el estudio del motor asncrono y de sus accionamientos se ha hecho necesario acudir a diversas transformaciones matemticas que relacionen las corrientes elctricas que se indican en la Figura 7.86. En los ejemplos de aplicacin 7.16 Y 7.17 se explican dos tipos de transformaciones: la primera de ellas se refiere a la matriz de transformacin T pa' que convierte un conjunto de devanados trifsico en otro bifsico, ambos estacionarios; en el segundo se explica la transformacin definida por la matriz Tf)' que convierte un sistema bifsico estacionario en otro rotativo que se mueve respecto al primero a la velocidad correspondiente al campo magntico giratorio del esttor (VI en rad/s elctricos. De acuerdo con el diagrama fasoria! de la Figura 7.86b, es evidente que para determinar las componentes Id e Iq debe conocerse con exactitud la posicin del fasor de flujo magntico en el entrehierro, para lo que se necesita disponer de una seal de realimentacin de la amplitud y la posicin instantnea del mismo flujo magntico, que puede realizarse de dos modos:

a)

Control vectorial directo, que incorpora dos transductores magnticos en el entrehierro basados en el efecto Hall y situados a 90 elctricos para averiguar la magnitud y posicin del flujo magnetico.

~'~~

CAPTULO

7.

ACCIONAMIENTOS

ELCTRICOS

685

b) Control vectorial indirecto, en el cual se calcula la amplitud y orientacin del flujo a partir de los parmetros del motor, y la posicin relativa instantnea del rotor se mide mediante un encder (transductor de posicin) situado en el rotor. El control vectorial directo, aunque tericamente tiene sus ventajas tcnicas, ya que incluye una medida directa del flujo y la situacin de su fasor espacial en el motor, tiene problemas de implementacin en un ambiente industrial, y es por ello que no suele utilizarse porque carece de la robustez necesaria (en definitiva, lo que sucede es que las ventajas del motor asncrono frente al motor de c.c. pierden de este modo su eficacia). Es por ello que en la prctica se utiliza un mtodo de control indirecto, midiendo la posicin y velocidad del motor con un encder y determinando a partir de los parmetros de la mquina la posicin del fasor espacial del flujo; sin embargo, este mtodo de estimacin delflujo es bastante sensible a los parmetros del motor, en concreto a los valores de la resistencia e inductancia de los devanados del rotor, que deben medirse con gran precisin. Desgraciadamente, los parmetros del motor usados en los clculos se miden en ensayos sin carga y no responden a los valores reales en el punto de funcionamiento de la mquina, ya que estos parmetros varan ampliamente con la saturacin del circuito magntico, temperatura y frecuencia de las corrientes rotricas. Es por ello que con este mtodo se deben calcular los parmetros de la mquina y se deben estimar sus variaciones conforme el motor cambia o modifica su punto de trabajo, pero hay que tener en cuenta que para que el comportamiento dinmico del accionamiento sea el adecuado, estas medidas y clculos deben realizarse en tiempo real y con extraordinaria rapidez para identificar sin errores la posicin del fasor espacial del flujo, lo que se consigue incorporando en el sistema de control microprocesadores rpidos y potentes, que incluyen DSP (digital signal processing), es decir, procesadores digitales de seales. Las funciones del DSP del sistema de control vectorial son diversas, entre las que destacamos las siguientes: a) Procesar las seales obtenidas del encder para determinar tanto la velocidad del rotor wm como el ngulo del rotor (),. b) Estimar el valor del flujo a partir de los datos de las tensiones, corrientes y velocidad del motor, incluyendo el modelo matemtico de la mquina. c) Identificar las dos componentes en eje directo y cuadratura de la corriente a partir de la informacin de las corrientes trifsicas. d) Implementar los lazos de control de velocidad y de corriente. e) Producir las seales de disparo del inversor, que definen la tensin y frecuencia de alimentacin del motor. f) Realizar las operaciones matemticas que requiere el modelo de la mquina y limitar las variables de control. En la Figura 7.87 se muestra un diagrama de bloques de un sistema de control vectorial indirecto que dispone de una etapa de potencia con inversor de corriente. El inversor de corriente dispone de una etapa intermedia de coriente continua que incluye una gran inductancia para mantener constante la corriente en el lado de c.c.; el sistema de potencia incluye lazos de comparacin de la corriente para mantener a sta en los valores requeridos, y es por ello que el inversor de corriente es m sencillo en su funcionamiento que el inversor de tensin. Obsrvese en la Figura 7.86b que para realizar el control vectorial debe determinarse con bastante precisin el valor del ngulo que permite localizar el fasor de flujo magntico. Este ngulo se obtiene como suma del ngulo que forma el eje del rotor con una referencia fija del esttor (), y que se obtiene del encder o transductor de posicin y del ngulo de deslizamiento e2 que se obtiene del modelo de mquina que se incluye en la memoria del microprocesadordel DSP. En la Figura 7.87 se tiene una entrada de velocidad de consigna w*, que se compara con la velocidad angular del motor wm para dar una seal de par de consigna T * a travs de un controlador proporcional-integral PI, cuya ganancia proporcional incluye el

e,

686

MQUINAS ELCTRICAS

Rectificador controlado
L

Inversor de corriente

MOTOR ASNCRONO

RED TRIFSICA 0------1 'V

Figura 7.87.

Control vectorial indirecto de la velocidad de un motor asncrono.

valor del nmero de pares de polos de la mquina para transformar la velocidad a radianes elctricos por segundo. Tambin, de acuerdo con la velocidad de giro real, se define el valor del flujo de entrehierro, de tal modo que por debajo de la velocidad base Jh se toma su valor nominal y por encima de ella la mquina trabaja por debilitamiento del campo. Del modelo de mquina se obtienen las componentes de corriente de consigna en los ejes mviles cuadratura y directo: i: e id* y el valor de la velocidad angular elctrica de deslizamiento J2*, que al integrarla da lugar al ngulo de deslizamiento de consigna e2~; al sumar este ngulo e'" al ngulo elctrico del rotor 0r se obtiene el ngulo de consigna del flujo magntico ee* (recurdese, segn seala la Figura 7.86b, que la relacin entre estos ngulos es 8e = 82 + e). Por otra parte, el DSP dispone de un bloque de transformacin de ejes mviles a fijos que convierte las corrientes iq* e id en i/J' e i;, respectivamente, mediante la matriz de transformacin Te-I [vase ecuacin (7.191) del ejemplo de aplicacin 7.17] y posteriormente se incluye otra matriz de transformacin [vase ecuacin (7.181) del ejemplo de aplicacin n." 7.16] que convierte las corrientes iir e i; en las corrientes trifsicas de consigna i~i<,i: e i/. Estas corrientes se comparan con las reales ia, ih e ir absorbidas por el motor para entrar en un bloque que genera los impulsos de disparo del inversor PWM que alimenta la mquina. Todas las funciones sealadas en el recuadro sombreado de la Figura 7.87 se realizan con un procesador digital de seales DSP. Los primeros DSP, que aparecieron en el mercado en la dcada de 1980, facilitaron el desarrollo de accionamientos de control vectorial, y se ha producido desde entonces un gran desarrollo de los mismos. En la actualidad existen nuevos controladores DSP, como los modelos TMS320x24x de Texas Instruments, que proporcionan

r:

CAPTULO 7.

ACCIONAMIENTOS ELCTRICOS

687

una gran capacidad de procesamiento y que incluyen perifricos especficos como generador de seales PWM, doble convertidor A/D (analgico-digital) y el QEP (Quadrature Encoder Pulse), que es el registro encargado de determinar la posicin y la velocidad del motor a travs del encder. Las dificultades inherentes al desarrollo de aplicaciones en tiempo real con DSP, ha hecho aconsejable el empleo de programas de simulacin de propsito general tipo MATLAB-SIMULINK, que se utilizan para comprobar el funcionamiento de los algoritmos de clculos empleados. Existen tambin paquetes de software especficos para aplicaciones de control digital de motores, como los desarrollados por la compaa Technosoft, con los que se consiguen resultados ms rpidos y fiables. Los ltimos avances en control vectorial de motores asncronos no incluyen ningn tipo de transductor (sensorless, sin sensores) para detectar la posicin del fasor espacial de flujo, y se estima la velocidad del motor nicamente a partir de medidas de tensiones y corrientes de la mquina. Los procedimientos empleados para ello utilizan tcnicas de la ingeniera de control moderna y del tratamiento de seales como: sistemas adaptativos, filtros de Kalman, observadores de velocidad tipo Luenberger, etc. Comprender el lector la complejidad que supone la implementacin de estos mtodos, que requieren del ingeniero una gran formacin multidisciplinar. Para el lector interesado en este tema se le remite al texto publicado por el Institute of Electrical and Electronics Engineers, IEEE (Instituto de' Ingenieros Elctricos y Electrnicos), editado por K. Rajashekara y que se menciona en la bibliografa de este captulo. EJEMPLO DE APLICACIN 7.16. TRANSFORMACIN DE CLARKE En la Figura 7.88 se muestra una mquina asncrona trifsica con tres devanados en el esttor, a, b y e, desfasados en el espacio 120 elctricos con N, espiras por polo y fase y factor de devanado kwque llevan, respectivamente, corrientes ia, ibe i, Se quiere sustituir el efecto de este devanado trifsico por otro bifsicoformado por dos devanados o: y fJ desfasados en el espacio 90 con el mismofactor de devanado kw'El nmero de espiras del conjunto de ambos devanados es el mismo, lo que significa que el nmero de espiras por polo y fase del devanado bifsico es 3N/2. El eje fJ del devanado bifsicoforma e grados elctricos respec-

Figura 7.88. Transformacinde Clarke.

688

MQUINAS ELCTRICAS

to del eje a del sistema trifsico. Si los dos sistemas deben producir la misma fm.m. en el entrehierrode la mquina,calcular:a) Los valoresde las comentes t; e i{J enfuncin de t; ib e ie' b) Contestar a la pregunta anterior en el caso de que () = 0 y determinar las expresiones de las corrientes i. e il! si las corrientes aplicadas al devanado trifsico son:

SOLUCIN
a)

En el esquema de la Figura 7.88 las f.m.m.s. producidas por el devanado trifsico proyectadas sobre los ejes ti. y f3 del sistema bifsico son:

(7.174) Mientras que el devanado bifsico, que tiene 3NJ2 espiras por polo y fase, produce en estos mismos ejes unas f.m.m.s.:

'~=;-2-1,

4 3Nskw .

(7.175)

al igualar las f.m.m.s. del mismo eje de las ecuaciones (7.174) y (7.175) se obtiene:
if3 = -[i 3

cos () + ib cos (8 - 120) + i. cos e

() + 120)]
(7.176)

Estas ecuaciones representan los valores de las corrientes que deben circular por el devanado bifsico para que produzcan las mismas f.m.m.s. que el sistema trifsico. Ahora bien, para que la transformacin sea biunvoca deben aparecer tres nuevas variables de corriente despus de la conversin. Como quiera que las corrientes i fl e t. calculadas del sistema bifsico producen la misma f.m.m. que el conjunto trifsico, habr que introducir una tercera variable que no contribuya a la creacin de f.m.m. en el entrehierro, y por este motivo es conveniente que esta tercera variable sea la corriente homopolar o de secuencia cero io que se emplea en el estudio de las componentes simtricas y que viene expresada por: (7.177) Tngase en cuenta que si la alimentacin trifsica original es a tres hilos (en estrella o tringulo), siempre se cumplir la igualdad: io + ib + ic = O,por lo que entonces la corriente io siempre valdr cero.

CAPTULO 7.

ACCIONAMIENTOS ELCTRICOS

689

De este modo, incluyendo (7.177) en (7.176) la ley de transformacin trifsica a bifsica vendr expresada en forma matricial por:
i} ~ =~ {

[COS

O cos (O - 120)

set
2

sen (e ~ 120")
2

(7.178)

donde la matriz de transformacin vale:

COS O

cos (O - 120) cos (O + 1200)] (7.179)

TpaO = ~ sen O sen (O - 120) sen (O + 120) 3 1 1 [ -1 2 2 2

que se denomina matriz de Clarke y que representa la transformacin de las variables a, b y e del esttor en las variables {J, a, O. Esta matriz de transformacin T fiaO se aplica no solamente a las corrientes del esttor, sino tambin a los flujos y a las tensiones de estos devanados. Por clculo matricial se puede demostrar que la transformacin inversa de Clarke viene expresada por:

(7.180)

b)

En el caso de que coincida el eje a del sistema trifsico con el eje {J del bifsico, entonces se cumple O = O, por lo que las matrices de transformacin anteriores se convierten en: -1/2 1/2
y si las corrientes trifsicas son:

-)3/2 )3/2
1/2

-1/2 ]

-)3/2 )3/2

:]

(7.181)

al aplicar (7.178), y teniendo en cuenta (7.181), resulta: -1/2 ifl} {


[

1
I~

:: = ~

-)3/2
1/2

(7.182)

690

MQUINAS ELCTRICAS

que operando da lugar a: (7.183) es decir, se obtienen, como era de esperar, dos corrientes bifsicas desafadas 90 en el tiempo. EJEMPLO DE APLICACIN 7.17. TRANSFORMACIN DE EJES FIJOS A EJES GIRATORIOS En la Figura 7.89 se muestra unjuego de dos devanados fijos a y {3, desfasados en el espacio 90 elctricos que tienen cada uno Ns espiras y que llevan, respectivamente, corrientes t, e ipSe quiere sustituir el efecto de estos dos devanados estticos por otro conjunto de dos devanados d y q situados entre s a 90 pero que se mueven a velocidad angular mi rad/s respecto del primero. El nmero de espiras del conjunto de ambos devanados es el mismo. El eje q del devanado mvil forma grados elctricos respecto del eje {3 del devanado fijo, siendo e = mlt + eo (donde eo representa la posicin inicial en t = O). Si ambos conjuntos deben producir la mismafm.m. en el entrehierro de la mquina, calcular los valores de las corrientes ide iqen funcin de t. e ip y determinar los valores correspondientes si estas ltimas son las obtenidas en (7.183), es decir: ip = 1m cos (mil t- (1.); i. = =L; sen (ml + (1.).

(I)~

Figura 7.89. Transformacinde ejes fijos a ejes giratorios.

SOLUCIN Igualando las f.m.m.s. que producen ambos conjuntos de devanados sobre los ejes resulta:
q y d

.~ = N,i = Ns(ip
q

cos

ee-

i, sen

e) ; .7;;:: NJd = N,Uf! e ;


id = isen

sen (J + i. cos

e)

(7.184)

de donde se obtiene la relacin de corrientes: iq = i{J cos


t, sen

e + i, cos e

(7.185)

CAPTULO 7.

ACCIONAMIENTOS ELCTRICOS

691

que se escribe en forma matricial: iq} = [COS { Id sen es decir, la matriz de transformacin vale:
T()=
COS [

e e

-sen cos

()J {~rl}
1,

(7.186)

O sen O

-sen 0J cos O

(7.187)

De la ecuacin (7.186) se deduce que si las corrientes ip e i, valen:


itl = 1m cos (Wlt +
IX) ;

t, = -1m sen (wlt +

IX)

(7.188)

y teniendo en cuenta que O = wJ + 00' los valores de las corrientes en el eje cuadratura y directo valen: iq} = [COS (wlt + eo) { id sen (Wlt + eo) cuyo resultado es: (7.190) Es importante darse cuenta del significado fsico que implica la transformacin anterior. En definitiva, se ha sustituido un sistema bifsico esttico de devanados por un sistema bifsico mvil. Las corrientes en el sistema bifsico fijo varan con respecto al tiempo segn seala (7.188), mientras que en el sistema mvil y de acuerdo con (7.190) las corrientes son constantes, como si fueran de corriente continua y solamente dependen de la fase inicial de la corriente IX y del ngulo inicial de giro 00 (de hecho, si se cumple que IX = 00 = O,el resultado (7.190) se convierte en iq = 1m; id = O.De este modo la f.m.m. del esttor parece estacionaria vista desde los devanados d y q mviles. Por otro lado, la matriz de transformacin inversa de (7.187) vale:
TI! =
_1 [ COS

(7.189)

-sen

sen cos

eJ
e

(7.191)

es decir, las relaciones inversas a (7.185) son:


itl = iq cos O + id sen O ;

t. = -iq sen () + id COS O

(7.192)

yen todas las ecuaciones anteriores se cumple e = wlt + Oo' Si se tienen en cuenta los resultados de este ejemplo de aplicacin y el anterior, se puede conseguir una transformacin conjunta que transforme un conjunto trifsico de devanados fijos Q, b Y e situados en el esttor por un sistema bifsico mvil d, q, O que se mueve a velocidad angular wl respecto a un referencial fijo. El lector puede demostrar que entonces las relaciones entre las corrientes respectivas dan lugar a la siguiente ecuacin matricial: cos (e sen (e
-

120) 120) (7.193)

1
2

692

MQUINAS ELCTRICAS

en la que 8 = wlt + 80. La transformacin correspondiente se denomina transformacin de Park y su expresin es:
COS

cos io - 120) cos (O + 1200)] (7.194)

TqdO = ~ sen O sen (O - 120) sen (O + 120) 3 l l 1 [ 2 2 2

Es importante darse cuenta del significado fsico que implica la transformacin de Park. En definitiva, se han sustituido los devanados reales de las fases a, b y e del esttor, con sus ejes magnticos fijos e inmviles, por dos arrollamientos ficticios q y d cuyos ejes magnticos se mueven a la velocidad wI del campo giratorio (es como si estuvieran en un rotor que se moviera a la velocidad de sincronismo). El arrollamiento de ndice O por el que circula la corriente homopolar io no interviene en el proceso. Si suponemos que las corrientes de los devanados reales, a, b y e, es un sistema de corrientes trifsicas equilibradas, sabemos que al estar desfasados en el espacio 120 producen un campo magntico giratorio que es sncrono con la pulsacin (1)1 de las corrientes. De este modo la f.m.m. del esttor parece estacionaria vista desde los devanados d y q mviles. Por otro lado, si se considera que las corrientes son trifsicas equilibradas de pulsacin (1)1 y que vienen expresadas por: ia

= 1m sen (I)I! + a)

; ib = 1m sen (I)l + t. = 1m sen (wI! + a + 120)

'X -

120) (7.195)

al aplicar la transformacin de Park a (7.195) se obtienen las siguientes corrientes: iq = 1m cos (80
-

a) ; id = 1m sen (00

'X)

io = O

(7.196)

que es el resultado previsto en (7.190). Es decir, la f.m.m. resultante que producen tres devanados desfasados en el espacio 120elctricos y alimentados por corrientes trifsicas es equivalente a la producida por dos devanados en cuadratura que se mueven a la velocidad de sincronismo pero alimentados con corriente continua. Es un gran descubrimiento que hizo R.H. Park en 1929 y que lo aplic al estudio de alternadores y que sera la base para un estudio generalizado de las mquinas elctricas que realizara ms tarde Gabriel Kron. La ventaja de la transformacin de Park es que las variables que dependen del tiempo en los modelos de mquinas elctricas: tensiones, corrientes, flujos e incluso los coeficientes de autoinduccin e induccin mutua entre devanados que dependen de la posicin del rotor se transforman en parmetros constantes, y de ah la gran simplificacin que se obtiene en el estudio de los modelos respectivos de mquinas elctricas. Este enfoque es muy til cuando se quieren estudiar las mquinas elctricas en rgimen dinmico y es imprescindible su aplicacin para un estudio riguroso de los accionamientos elctricos.

7.13. ACCIONAMIENTOS ELCTRICOS CON MOTORES DE C.A. SNCRONOS


La velocidad de un motor sncrono se puede cambiar de una forma sencilla modificando la frecuencia de alimentacin, ya que la velocidad de rotacin coincide con la de sincronismo

CAPTULO 7.

ACCIONAMIENTOS ELCTRICOS

693

del campo magntico giratorio n = 60f/p (en r.p.m.), dondefes la frecuencia de alimentacin y p el nmero de pares de polos de la mquina. Para cada frecuencia, la velocidad del motor permanecer constante a menos que la mquina pierda el sincronismo por haberse aplicado al eje un par resistente superior al mximo. Considrese por sencillez un motor sncrono trifsico de polos lisos (rotor cilndrico); la potencia mecnica que desarrolla el motor viene definida por la expresin (5.68) del Captulo 5, que es la siguiente: (7.197) donde Ea es la f.c.e.m. por fase, V la tensin aplicada al motor por fase, X, es la reactancia sncrona y b el ngulo de carga, que a su vez es el ngulo que forman los fasores Ea Y V. De acuerdo con la expresin anterior, el valor del par electromecnico producido por el motor sncrono si gira a la velocidad angular mecnica wm es igual a: (7.198)* donde se ha tenido en cuenta que la velocidad angular mecnica wm es igual a w/p, siendo p el nmero de pares de polos del motor y w la pulsacin elctrica, que a su vez es igual a 2nj, (f es la frecuencia de alimentacin del esttor). El valor de la f.c.e.m. del motor Ea est calculada en la expresin (5.5) del Captulo 5 y es igual a: (7.199) De acuerdo con la expresin anterior, si se mantiene constante la corriente de excitacin, el flujo magntico ser constante y por consiguiente la f.c.e.m. del motor ser directamente proporcional a la frecuencia de las corrientes del inducido, es decir: (7.200) Por otro lado, si se denomina L, a la inductancia sncrona, el valor de la reactancia sncrona a la frecuencia fes:

x, = col., = 2nfL,

(7.201)

y sustituyendo (7.200) Y(7.201) en (7.199) se obtiene la expresin del par siguiente: T

3pKJV
(2nf)
2

L,

sen b

=Kf

sen b

(7.202)

Si el motor es de polos salientes, se puede demostrar que la expresin correcta del par es:
3p V '. V'(x, - X,,) T =sen 15 + sen 215 ] (J) x, ,2XdXq

[Eo

donde Xd y Xq son, respectivamente, las reactancias sncronas de eje directo y cuadratura. El primer sumando es el par sncrono y el segundo el par de reluctancia. En mquinas con polos lisos se cumple: Xd = Xq X,.

694

MQUINAS ELCTRICAS

donde K es una constante que engloba todos los trminos invariantes. La expresin anterior se puede escribir de este modo: T

=Kf

sen

s = Tmx sen s ;

donde T mx = K

(7.203)

En la Figura 7.90 se muestra la curva par-ngulo de carga definida por la ecuacin anterior. La zona derecha de la curva (para (j > 0) corresponde al funcionamiento como motor, mientras que la zona izquierda (para (j < 0) corresponde al trabajo como generador en rgimen de frenado regenerativo. El par nominal T" se obtiene para un punto como el A, donde el ngulo de carga (jn es del orden de 25 a 30 elctricos, por lo cual el cociente TmjT" suele estar comprendido entre 2 y 2,5. Aumentando la excitacin del motor se eleva el valor de la f.c.e.m. Eo Ypor consiguiente se aumenta el par mximo, que en algunos casos puede llegar a valer de 3,5 a 4 veces el par nominal. Si se considera que el motor trabaja con un par resistente constante e igual al nominal, la mquina funciona en rgimen permanente en el punto A. Si en esta situacin aumenta el par resistente, se produce una ligera reduccin de velocidad, lo que provoca a su vez un aumento del ngulo de carga (j, elevndose de este modo el par del motor, y se alcanza una situacin de equilibrio para un ngulo de carga mayor que el original cuando el par motor se igual a al resistente y se consigue un nuevo enclavamiento entre los polos del rotor y el campo giratorio del esttor, girando la mquina nuevamente a la velocidad de sincronismo. Si se aumenta el par resistente hasta llegar al valor del par mximo, el punto de equilibrio sera el B de la la Figura 7.90; un posterior aumento del par resistente no se vera compensado con un aumento del par electromagntico, por lo que el motor se saldra del sincronismo.De este modo la zona OB de la curva par-velocidad es la zona de trabajo estable de la mquina, mientras que a la derecha de B se tiene un funcionamiento inestable. El punto B, para el cual (j = 90, representa el ngulo de carga mximo que define el lmite de estabilidad del motor. De un modo anlogo se puede analizar la zona correspondiente al funcionamiento de la mquina como generador o frenado regenerativo, en la que el punto e representa el lmite de estabilidad como freno. La regulacin de la velocidad de un motor sncrono se realiza controlando la frecuencia de alimentacin del inducido. Si se define una velocidad base nh para la cual la mquina trabaja con los valores nominales de tensin y frecuencia, si se mantiene constante el cocienT

Freno

Motor Lmite de estabilidad en rgimenpermanente

Par resistente
1800 900 900 1800

e --------------Trnx
Figura 7.90. Curva de par del motor sncrono en funcin del ngulo de carga.

CAPTULO

7.

ACCIONAMIENTOS ELCTRICOS

695

te VII, en todo el rango de velocidades desde cero hasta la velocidad base, el valor del par mximo, de acuerdo con (7.203), se mantendr tambin constante en este rango. Por encima de la velocidad base, la regulacin de velocidad se hace por debilitamiento del campo, manteniendo la tensin aplicada al motor en su valor nominal y siguiendo aumentando la frecuencia de alimentacin, lo que se traduce en una reduccin del par mximo en esta zona. En la Figura 7.91 se muestran las curvas par-velocidad correspondientes, que son rectas verticales de velocidad constante correspondientes a cada valor de la frecuencia aplicada. Si la carga tiene un par resistente constante igual al nominal, el motor ir aumentando su velocidad progresivamente desde cero hasta su valor mximo, pasando por los puntos de trabajo 1, 2, 3, 4, etc., mostrados en la Figura 7.91, conforme se eleva la frecuencia aplicada al inducido. Existen dos tipos de accionamientos para motores sncronos, a saber: a) en lazo abierto, en el que se controla directamente tanto la tensin como la frecuencia aplicada al motor; b) en lazo cerrado, en el que se regula directamente la tensin aplicada al inducido, mientras que el valor de la frecuencia de alimentacin va variando en funcin de la posicin del rotor.

7.13.1.

Regulacin de velocidad de motores sncronos en lazo abierto

En la Figura 7.92 se muestra un esquema bsico de control de velocidad de un motor sncrono en lazo abierto, que es un circuito similar al empleado en la regulacin de motores asncronos. Los bloques de electrnica de potencia consisten en un rectificador controlado situado a principio de lnea que alimenta a un bus de c.c. y un inversor al final de lnea que se aplica al inducido del motor sncrono. El inductor se alimenta a travs de un rectificador controlado mono o trifsico (no sealado en la Fig. 7.92) para ajustar la corriente de excitacin. Si se desea que el motor funcione en rgimen de frenado regenerativo se debe sustituir el rectificador controlado de principio de lnea por un sistema dual de dos rectificadores controlados conectados en antiparalelo, para que de este modo se pueda devolver energa elctrica a la red. En las aplicaciones de gran potencia se utilizan cicloconvertidores que transforman directamente la c.a. de la red en c.c. de tensin y frecuencia variable sin pasar por la etapa intermedia de c.c.
T
x

Aumento de frecuencia

-- --

....

-- -- --

.,.,
nb

Motor ,, Par resistente


"

C .-

2 3

~.(

6 7 8 ,'1' .{ ~.(

.-

Velocidad

--

--

--

, , r ,
t

, ,

, ,,

.'

Motor regenerativo (generador)

-- -- --

Figura 7.91. Curvas par-velocidad de un motor sncrono.

696

MQUINAS ELCTRICAS

e
i '<t: l:::
U

Rectificador controlado Filtro ---+

Inversor

MOTOR SNCRONO

<t:

t,

.... el
U.l

c:: c::

v,

+
Vee

v,

Generador impulsos de disparo del inversor

L..----,,-----I

Control de flujo (V/j) Velocidad de c0'lsigna n Limitador dn'/dl

Figura 7.92. Regulacin de velocidad de un motor sncrono en lazo abierto.

El motor arranca suavemente desde el reposo hasta la velocidad de consigna, y para ello dispone de un limitador de la derivada de la velocidad para que la frecuencia cambie gradualmente desde cero hasta el valor correspondiente a la velocidad de consigna. De acuerdo con esta velocidad n* se realiza el control de flujo, de tal modo que si n* < nb la regulacin se hace manteniendo el cociente VI! constante y para n* > nh la tensin se fija en el valor nominal y se va subiendo la frecuencia. La seal de tensin V controla el ngulo rt. de disparo del rectificador controlado, mientras que la seal! define la frecuencia de la c.a. que produce el inversor. En este circuito, si se producen variaciones lentas de par, aparecen oscilaciones en el rotor (penduleo) pero al cabo de un cierto tiempo se estabiliza. En el caso de que las variaciones del par sean muy fuertes, el ngulo de carga del rotor puede superar el valor lmite de 90 provocando la inestabilidad del motor, que se saldr del sincronismo.
0,

7.13.2. Regulacin de velocidad de motores sncronos en lazo cerrado. Motor sncrono autopilotado
Para evitar el inconveniente de prdida de sincronismo del motor sncrono cuando est sometido a variaciones bruscas de par, se debe utilizar una regulacin en lazo cerrado o con realimentacin. La idea de este procedimiento es hacer que las variaciones de la velocidad del rotor modifiquen automticamente la frecuencia del inversor; para ello se debe medir el movimiento del rotor con un transductor de posicin (encder), y esta informacin se utiliza para corregir la frecuencia aplicada al esttor haciendo que el motor conserve el sincronismo con la nueva frecuencia aplicada. De este modo, la velocidad del rotor corrige automticamente la frecuencia del esttor, y de ah que este accionamiento se denomine autocontrolado o autopilotado (self-controlled en ingls o autopilotage en francs). En la Figura 7.93 se muestra el esquema de control de un motor sncrono autopilotado, Se dispone de dos convertidores, uno al principio de lnea, que al igual que en el caso anterior es un rectificador controlado que alimenta a un bus intermedio de c.c. y al final del mismo se

CAPTULO 7.

ACCIONAMIENTOS ELCTRICOS

697

2
U (ji '<t: .

Rectificador controlado
L

Inversor de corriente Ji

MOTOR SNCRONO

<

~ ....
el .

~
IX

t
Velocidad de consigna

+
Vee

--+
lec

+ ~

n*o-~+X~.__~n~

_J

Figura 7.93. Motor sncrono autopilotado.

sita el inversor u ondulador que convierte la c.c. en c.a. trifsica de tensin y frecuencia variable. El encder, que se coloca en el eje del motor, da una seal indicativa de la posicin del rotor que se procesa en la lgica de control y se utiliza para generar los impulsos de disparo de los tiristores del inversor. De este modo, cualquier variacin en la velocidad del rotor debida a un cambio en la carga, modificar inmediatamente la frecuencia de encendido de los tiristores y ajustar la frecuencia de alimentacin del esttor al valor correcto para evitar la prdida de sincronismo del motor. En definitiva, de este modo se evitan las oscilaciones pendulares del motor y las inestabilidades asociadas con un cambio en el par o en la frecuencia El esquema de la Figura 7.93 incluye tambin un lazo de corriente que acta sobre los ngulos de encendido de los tiristores del rectificador controlado para mantener la corriente de la mquina en el valor de consigna estipulado. Para ello se toma como seal de referencia la corriente del lazo intermedio de c.c. lec' que es proporcional a la corriente del inducido del motor I. La corriente lec se compara con la de referencia para dar una seal de error que sirve para ajustar el ngulo de encendido r:t. de los tiristores del rectificador controlado. Nota informativa: Motor sncrono autopilotado en la traccin elctrica espaola. A VE Serie 100. Los trenes AVE Serie 100 (Alta Velocidad Espaola de la lnea Madrid-Sevilla, inaugurada en 1992) fueron construidos por GEC-Alsthom, grupo ferroviario que integr a las sociedades ATIENZA, MTM y MEINFESA. Diferentes industrias ferroviarias espaolas participaron tambin de forma importante en la construccin de los trenes: CAF, en sus talleres de Zaragoza y Beasan; SEPSA y STONE IBRICA, en sus factora cercanas a Madrid. Los trenes AVE son trenes de alta velocidad derivados del TGV Atlantique y son explotados en lneas RENFE, con la denominacin EUROMED. Tienen una composicin M-8R-M, que

700

MQUINAS ELCTRICAS

El ajuste del esfuerzo de traccin se logra variando en cada motor las intensidades tanto del esttor como del rotor, ajustando los tiempos de desbloqueo de los elementos electrnicos de control. El rotor se alimenta de una lnea de corriente continua de 500 V Ysu intensidad se controla mediante un chopper de excitacin, siendo la corriente proporcional a la que recorre el esttor, lo cual permite obtener caractersticas cercanas a las de un motor serie de e.e. En el frenado reosttico los motores de traccin funcionan como alternadores y, los onduladores como rectificadores. Los conjuntos motor y ondulador alimentan un restato: una parte de ste se conecta en paralelo con el chopper principal con objeto de variar la resistencia aparente de frenado. El esfuerzo de traccin se regula tambin variando, al mismo tiempo, la excitacin de los motores sncronos. Cuando desaparece la alimentacin de alta tensin (falta de tensin en lnea, apertura de disyuntores, etc.) la corriente de excitacin de los rotores es suministrada por la batera correspondiente y el esfuerzo de frenado es inferior en un 20 por 100 aproximadamente. La alimentacin de los equipos elctricos auxiliares se realiza a partir de un arrollamiento secundario auxiliar del transformador principal y de un puente rectificador (alimentacin a 25 kV) o a partir directamente de la catenaria (alimentacin a 3.000 V), Y por medio de choppers auxiliares reductores que suministran tensin continua de 500 V.

PROBLEMAS
7.1. Se dispone de una red monofsica con una tensin eficaz de 220 V, 50 Hz, que alimenta una carga que tiene una resistencia de 10 ohmios y una inductancia de 0,0318 henrios, a travs de un diodo rectificador de potencia (rectificacin media onda). Calcular: a) expresin de la corriente instantnea en la carga; b) tiempo de apagado del diodo, es decir, el tiempo correspondiente a la anulacin de la corriente de la carga (en el primer ciclo); e) tensin y corriente media en la carga. [Resp.: a) i(t) = 22[sen (on - n/4) + 0,707' e-314,47t]; (1) = 100 ti. b) 0,0125 segundos. e) 84,05 V; 8,405 A.] En el problema anterior se dispone de un diodo volante en paralelo con la carga.
a) Calcular la expresin de la corriente instantnea que circula por la misma. b) Con-

7.2.

testar a la pregunta anterior si el nuevo valor de la inductancia de la carga es de 0,5 henrios. e) Cul es entonces el valor de la potencia disipada en la resistencia de 10 ohmios? [Resp.: a) (t) = 9,9 + 11 sen (314, 16t - 45) - 2,95 sen (628,32t - 63,43) - 0,32 sen (1.256,64t - 75,96). b) (t) = 9,9 + 0,988 sen (314, 16t - 86,36) - 0,21 sen (628,32t- 88,18) - 0,021 sen (1.256,64t - 89,n]. e) La corriente eficaz en la carga es de 9,926 A, por lo que la potencia disipada es de 985,2 W.] 7.3. Un rectificador monofsico en puente de Graetz (rectificacin doble onda) alimenta una carga que tiene una resistencia de 20 n y una inductancia de 20 mH. La tensin de la red es de 220 V eficaces y 50 Hz. a) Calcular la expresin de la corriente instantnea que circular por la carga; b) Potencia disipada en la resistencia de 20 n. e) Contestar a la pregunta anterior si se considera la inductancia infinita. [Resp.: a) i(t) = 9,9 - 5,59 cos (cot - 32,14) - 0,822 cos (4(1)t- 51,49) - 0,265 cos (6(1)t- 62,05); (1) = 100 ti. b) 2.280 W. e) 1.960,2 W.] Una red trifsica de 380 V de lnea (valor eficaz) y 50 Hz alimenta una resistencia de 50 ohmios a travs de un puente rectificador trifsico en puente (doble onda). Calcular: a) tensin y corriente media en la resistencia de la carga; b) corriente media en

7.4.

APNDICE

l.

MQUINAS

ELCTRICAS:

ASPECTOS

HISTRICOS

719

primero que construy y patent este tipo de motores en octubre de 1887, Ypor lo que se le considera como el inventor de los mismos. Tesla public en la Revista AlEE Transactions de mayo de 1888 un artculo en el que describa tres tipos de motores inventados por l (ha sido lamentable para la ingeniera elctrica que un ingeniero de la talla de Tesla no publicara prcticamente nada sobre sus descubrimientos; este artculo probablemente sea uno de los pocos que dio a conocer al mundo cientfico); todos ellos disponan de un esttor en forma de anillo; el primer tipo tena un rotor con cuatro polos salientes, dando lugar a un motor de reluctancia que no posea cualidades de autoarranque, pero que giraba a la velocidad de sincronismo; el segundo motor era un verdadero motor asncrono: tena el rotor devanado, que poda arrancar pero que giraba a una velocidad por debajo de la correspondiente al sincronismo, y el tercero era motor sncrono, que funcionaba suministrando corriente continua al devanado del rotor. Debe destacarse que los primeros motores asncronos eran bifsicos y con polos salientes en el esttor, alimentados con dos corrientes desfasadas 90 en el tiempo y utilizando dos devanados desfasados 90 en el espacio. George Westinghouse compr las patentes de Tesla y utiliz a este ingeniero como consultor de su empresa; con la ayuda de C. F. Scott Y B. G. Lamrne, la empresa Westinghouse desarroll un motor bifsico con devanados distribuidos tanto en el esttor como en el rotor, logrndose un motor prctico alrededor de 1892. En la Feria Mundial de Chicago de 1893, la fbrica de Westinghouse present un motor bifsico de 300 CV, 12 polos a 220 V, que era una gran hazaa para esa poca; la alimentacin de este motor se lograba mediante dos alternadores monofsicos de 500 CV, 60 Hz, acoplados mecnicamente en el mismo eje, pero que estaban desplazados 90 elctricos en el espacio para poder generar una tensin bifsica. En 1891 la compaa americana Thomson-Houston comenz la construccin de motores de induccin trifsicos bajo la direccin de H. G. Reist y W. J. Foster. Por otra parte, en Europa, Dolivo-Dobrowolsky, ingeniero de la empresa alemana AEG, sugiri la utilizacin de circuitos trifsicos pero no independientes entre s, sino mutuamente conectados; la expresin alemana Verkettung der Phasen (encadenamiento de fases) traduce esta dependencia mutua de las tres corrientes que constituyen un sistema trifsico. Este sistema lo bautiz con el nombre Drehstrom (que significa corriente giratoria) alrededor de 1890.Para el ao 1893 Dolivo-Dobrowolsky haba construido motores asncronos de doble jaula de ardilla que mejoraban las cualidades de arranque de estos motores; tambin sugiri la construccin del motor de induccin con rotor devanado o con anillos deslizantes, para regular la velocidad del mismo, para lo cual es preciso conectar a los anillos un reostato de arranque y regulacin de un modo equivalente al de los motores de c.c. En EE.UU. se unieron las compaas Westinghouse y la Thornson-Houston para fabricar motores asncronos trifsicos; para ello result de gran utilidad en aquel momento el invento del ingeniero C. F. Scott, de la empresa Westingouse, para transformar un sistema bifsico en trifsico y poder alimentar estas mquinas. El rotor de jaula de ardilla, construido mediante barras de aluminio, fue patentado en 1916 por H. G. Reist y H. Maxwell, de la compaa General Electric. En cuanto a los motores asncronos monofsicos, en 1890 Maurice Leblanc demostr tericamente que se poda alimentar con una tensin monofsica un motor trifsico mediante un dispositivo especial de arranque. La demostracin de la descomposicin de un campo alternativo en dos giratorios se debe a l mismo y a los trabajos de Blondel; este ltimo, en 1894, hizo un estudio completo de la teora correspondiente. El motor de fase partida fue inventado por Tesla, empleando dos devanados con resistencias muy diferentes para poder lograr un alto par de arranque. La idea de utilizar condensadores especiales para el arranque de motores monofsicos se debe a Elihu Thomson en 1892 y tambin a Steinmetz; sin embar-

726

MQUINAS ELCTRICAS

controlados de silicio van aumentando, por lo que poco a poco se van imponiendo en la regulacin de todo tipo de mquinas elctricas. Los componentes modernos ms importantes que se utilizan en la regulacin de mquinas elctricas son: los GTO (gate turn-off thyristor, es decir. tiristores de apagado por puerta). los IGBT (insulated gate bipolar transistor. o transistor bipolar de puerta aislada), que son transistores que combinan las ventajas de los tipo bipolar, y los de tecnologa MOSFET (metal-xido semicondutor y de efecto de campo). En 1988 la General Electric anunci un dispositivo denominado MCT (MOS controlled thyristor o tiristor controlado MOS), al que los tcnicos auguran un gran futuro en las aplicaciones para el control de velocidad de mquinas elctricas. Aparte de los desarrollos de componentes electrnicos de alta potencia indicados en los prrafos anteriores, conviene destacar otros dispositivos y tcnicas electrnicas que intervienen en los lazos de regulacin de mquinas y cuya gnesis descansa en el descubrimiento del circuito integrado. Debe destacarse que la patente del primer circuito integrado la registr Jack Kilby, de la empresa Texas Instruments, en febrero de 1958 (su circuito integrado era de hecho dos circuitos construidos en una pieza de germanio). En octubre de 1958 la compaa mencionada construy un circuito integrado en el cual, por diversos procesos de difusin, se lograba obtener diversos componentes activos: diodos rectificadores y transistores y tambin elementos pasivos como resistencias y condensadores; el problema de este primer circuito integrado era que los diversos componentes del mismo tenan que interconectarse exacta y laboriosamente a mano. Poco despus, Robert Noyce, de la Fairchild, fabric en 1960 un circuito integrado completo que inclua las interconexiones sobre la misma pastilla utilizando un proceso denominado tecnologa planar. Esta tendencia a la miniaturizacin se fue acelerando en los aos siguientes, dando lugar a la creacin de nuevos microcircuitos y a una nueva rama tcnica conocida hoy da como microelectrnica (desde 1960,el nmero de componentes que se podan integrar en los chips individuales se ha duplicado aproximadamente cada ao, para aumentar desde las cifras bajas de 1960 a casi 100.000en 1980; esta duplicacin anual se ha denominado ley de Moore). En 1969 se dirigi a la empresa americana lntel una firma japonesa de calculadoras, la Busicom, con la intencin de que la Intelle fabricara una gama de chips de calculadora. La Busicom aport un diseo de contorno y una colaboracin de diseo. La Intel, y ms exactamente Ted Hoff, utilizando su preparacin en diseo de circuitos LSI (Large Integration Scale o integracin a gran escala), estudi el diseo del chip y pens que se poda construir uno mejor. Hoff pronto se convenci de que un chip programable nico podra ejecutar todas y an ms funciones que las que la Busicom exiga a sus varios modelos de calculadora. Despus de nueve meses de trabajo, el 15 de noviembre de 1971 la Intel construye el primer microprocesador del mundo (el 4004), que se llam entonces ordenador microprogramable en un chip y que ms tarde, en 1972, se le dar el nombre definitivo de microprocesador. El 4004 era capaz de procesar palabras de 4 bits de longitud (4 dgitos binarios) y contena, en el lenguaje peculiar de los ordenadores, una completa CPU integrada (Central Processing Unit, o unidad central de procesado de la informacin) con un sumador paralelo de 4 bits, 16registros, un acumulador y un registro de estado, todo en un chip. Esta CPU 4004 se uni con chips de memorias, ROM (Read Only Memory, es decir, memorias de slo lectura), RAM (Random Access Memory, es decir, memorias de acceso aleatorio. o de otro modo: memorias de lectura y escritura, ya que la informacin puede escribirse y borrarse a voluntad) y de registros de desplazamiento para constituir el sistema microordenador MCS-4. El microprocesador provoca una revolucin en la industria de la electrnica y los ordenadores. El microprocesador es ms que un simple componente, y su potencia de clculo, pe-

728

MQUINAS ELCTRICAS

ignitrones en la dcada de 1930, en las mquinas pequeas se sustituyeron los grupos WardLeonard rotativos por sistemas estticos que regulaban la tensin continua que se aplicaba a los inducidos de los motores. Al llegar el tiristor y conseguir un aumento en sus prestaciones (soportar mayores tensiones y corrientes) se ha conseguido realizar regulaciones de velocidad de motores de c.c. que llegan a potencias de hasta 10.000 kW. Fundamentalmente, el control de velocidad se realiza en la parte de potencia, por medio de convertidores a tiristores controlados por fase (cuando la alimentacin procede de c.a.) y tambin por medio de troceadores o choppers (cuando la alimentacin procede de c.c.). La regulacin puede realizarse a par constante o a potencia constante y se emplea, en el caso de que se utilice el control de fase, convertidores de cuatro cuadrantes o duales (formados por dos puentes trifsicos completos que transforman la tensin trifsica de entrada en una tensin continua variable en magnitud y signo), que permiten la operacin en sentido directo e inverso, aumentando la capacidad de frenado por recuperacin de energa, lo que es importante en aplicaciones como trenes de laminacin en aceras y papeleras y tambin en traccin elctrica. En casos ms simples, donde no se requiere inversin de velocidad, se emplean convertidores de uno o dos cuadrantes, y tales son las aplicaciones de regulacin de velocidad de ventiladores, bombas centrfugas, cintas transportadoras, mquinas herramientas, etc. Cuando se emplean troceadores, que son convertidores c.c. a C.C., la tensin constante de alimentacin se conmuta rpidamente por medio de tiristores, consiguiendo una seal de salida que se compone de trozos de la seal de entrada, lo que da lugar a una regulacin del valor medio de la tensin de alimentacin al motor. Cuando los tiempos de conduccin y no conduccin son variables, pero manteniendo constante el perodo total conduccin-no conducin, se dice que la regulacin se realiza por modulacin de la anchura de impulsos (PWM: Pulse Wdth Modulaton). Cuando el tiempo de conduccin es constante y el de no conduccin es variable, se dice que la regulacin se efecta por modulacin de la frecuencia. Este tipo de control de velocidad de motores de c.c. se realiza en los ferrocarriles metropolitanos, coches elctricos, trolebuses, carretillas elevadoras alimentadas por acumuladores, etc. Con este sistema de regulacin se ha logrado controlar mquinas de hasta 2.500 kW. El control de los impulsos de disparo de los tiristores es muy diverso; desde los casos ms simples, que han utilizado VJT, hasta el control digital moderno por microprocesador,pasando por osciladores tipo PLL (phase-locked loop, es decir, con lazo de amarre defase). La ventaja de la regulacin de velocidad en los motores de c.c. procede de que las fuerzas magnetomotrices del inductor y del inducido estn a 90 (en la terminologa habitual de estas mquinas se denomina al eje de los polos eje directo d, mientras que al eje de escobillas que se sita en la lnea neutra a 90 elctricos del anterior se le denomina eje en cuadratura q), es decir, estn prcticamente desacopladasy no interaccionanentre s, lo que permite un control independiente de ambas variables. Como quiera que el par de un motor de c.c. es proporcional al producto del flujo inductor por la corriente del inducido, al mantener el flujo inductor constante y modificar la corriente del inducido no se tendrn cambios en el campo y las variaciones en el par sern copia exacta de los cambios en la corriente del inducido. La desventaja de estos motores es que son ms caros que los de c.a. y menos robustos que stos en virtud de la existencia del colector de delgas, lo que se traduce en un mayor coste de mantenimiento.

6.2.2. Regulacin de velocidad de los motores de c.a.


Como se acaba de sealar, el motor de c.a. de induccin o asncrono es ms barato en su construccin que el motor de c.c. y no requiere apenas mantenimiento. Es por ello que los

744

MQUINAS ELCTRICAS

Si se consideran unas cantidades verdaderas Z(O), 1, Vy S, sus magnitudes expresadas por unidad sern: Z(O) Z(p.u.) =-

z.:

I I (p.u.) =-

i.:

V (p.u.) =-

Vhase

S S (p.u.) =-

(2)

Las ecuaciones anteriores nos permiten determinar las cantidades por unidad (p.u.) conociendo las magnitudes base y los valores verdaderos de las variables, o al contrario, si se conocen los valores p.u. se podrn calcular los valores verdaderos al haber fijado las magnitudes base. Supngase, por ejemplo, un circuito monofsico con los valores base siguientes:
Sbase

1.000 VA (1 p.u.) ; Vbase

= 200 V (1 p.u.)
Vhase 200 === 40 O (l p.u.)

(3)

Los valores base de la corriente y la impedancia sern segn (1):


Ihase = Shase

Vhase

= --

1.000 200

= 5 A (1 p.u.) ;

Zba.\e

I base

(4)

Por consiguiente, si se tienen unos valores verdaderos de tensin, corriente e impedancia de 150 V, 4 A Y 50 ohmios, respectivamente, correspondern a los valores por unidad siguientes: 150 4 50 V (p.u.) = = 0,75 ; I (p.u.) = - = 0,8 ; Z (p.u.) = ~ = 1,25 200 5 40 (5)

Como ya se ha indicado, el sistema por unidad tiene la ventaja de que con l hay menos posibilidades de equivocarse en los clculos. Por ejemplo, si les 0,8 p.u. (es decir, 4 A) Y Z es 1,5 p.u. (es decir, 60 0), la tensin ser: V = Z: 1= 1,5' 0,8 = 1,2 p.u. El valor de la tensin 1,2 p.u. con 200 V de tensin base representan: 1,2 . 200 = 240 V

(6)

(7)

que en magnitudes absolutas corresponden al producto de la impedancia de 60 ohmios por la corriente de 4 amperios. El sistema por unidad es muy til en el estudio de los sistemas elctricos de potencia. En la forma convencional en la que se emplean voltios y amperios, la solucin de una red de transporte con diferentes niveles de tensin requiere que todas las impedancias a sumar sean referidas a un mismo nivel de tensin. En el sistema por unidad, desaparecen completamente estos niveles y una red constituida por alternadores, transformadores y lneas de diferentes niveles de tensin se reduce a un sistema de impedancias simples. De hecho, en un sistema de potencia la tensin y la potencia base se seleccionan en un punto especfico del sistema. Los transformadores no afectan la magnitud de la potencia aparente base del sistema, puesto que en ellos idealmente la potencia aparente es igual en la entrada que en la salida. Sin embargo, la tensin s que cambia al pasar por un transformador, de tal suerte que la magnitud Vbase vara con la relacin de espiras de cada transformador instalado en la red. Como las magnitudes base cambian al pasar por un transformador, el proceso de referenciar las cantidades a un nico nivel de tensin se realiza automticamente durante la normalizacin. Considrese, por ejemplo, un transformador monofsico de 2.000 VA, relacin 200/400 V, con unas impedancias de primario y secundario de j 2 O Yj 6 O, respectivamente. En el lado de baja tensin (primario) se tendrn los siguientes valores base:

748

MQUINAS ELCTRICAS

d)

Para la lnea de transporte, la impedancia est referida aliado de 800 V, por lo que al aplicar (1) y (2) resulta un valor por unidad: 8002 = 5.000 ; = 50 + j200 . 8002 5.000 = 0,39 + } 1,56

Zh2

ZLpu2

El subndice 2 en la impedancia anterior significa en principio que est referida al secundario del transformador (aunque al expresarla por unidad ser independiente del lado del transfomador a que se refiera). Podemos hacer este clculo ms lentamente para corroborarlo, calculando la impedancia referida al primario y expresndola en valores por unidad, es decir:
ZL

250)2 (reducida al primario) = m2ZL = ( (50 + j200 ) = 4,88 + j 19,53 Q 800

y como quiera que la impedancia base referida al primario es, segn (1):

se obtendr, de acuerdo con (2): Z -_ ZL


Z'
b2

Lp.u.2 -

4,88 + j 19,53 __ =---_ = 0,39 + ji 52,5

,56

e)

que coincide con el valor calculado antes de una forma ms directa, sin necesidad de tener que reducir la impedancia de la lnea (situada en el secundario del transformador TA) del secundario al primario. Para el transformador TB se tiene:
ZBP.ul

=jO,08

Sbl

= 8.000 VA

Vbl = 1.000 V (primario)

y al aplicar (22) resulta:


ZBPu2

..

=jO,08

--2-

1.0002 5.000 -=jO,078 800 8.000

f)

Tngase en cuenta que para este transformador la tensin base Vh2 = 250 V se transforma en el lado de alta tensin en V2 = 800 V. La potencia de la carga en valores por unidad es: 2.500 S2p.u. =--=0,5 5.000 En la Figura A3.2 se representa el esquema unifilar del circuito referido al lado (base) de 250 V Ycon una potencia base de 5.000 VA. Reconocer el lector la enorme simplificacin que se obtiene de la red original. Este esquema admite una simplificacin mayor si se asocian las impedancias conectadas en serie.

NDICE ALFABTICO

755

inversor monofsico, 626 trifsico, 628 inversores, 626 jaula de ardilla, 150, 261 Kramer esttico, 676 Leblanc, teorema de, 126 lnea neutra, 90 locomotora S252, 668 magnetismo remanente, 7 magnitudes normalizadas, 743 mquina asncrona, 149, 259 de C.C., 147 de induccin, 149,259 sncrona, 146, 381 materiales magnticos, 2 MeT,564 metropolitano, 654 modo inversor, 608 modulacin de impulsos, 632 de un solo impulso, 630 senoidal de impulsos, 633 momento giratorio, 332 magntico, 3 Montsinger, ley de, 102 motor autopilotado, 696 motor con condensador, 344 espira de sombra, 347 motor de c.c., 514 de fase partida, 343 de induccin lineal, 360 de induccin monofsico, 337 motor paso-paso, 74 sncrono, 452 universal, 151 neutro inquieto, 213 ncleo, 162 onduladores, 626 ortogonalidad, 735 par de rotacin, 289 par electromagntico, 138 paramagnetismo, 3 Park, transformacin de, 692 paso diametral, 111 paso-paso, motor, 74

pndulo taquimtrico, 422 prdidas en el cobre, 100 en el hierro, 100 mecnicas, 100 por histresis, 23 permeancia, 13 pinzas amperimtricas, 239 placa de caractersticas, 105 polos auxiliares, 494 lisos, 383 salientes, 383 portadores mayoritarios, 533 minoritarios, 534 potencia asignada, 103 regulante, 426 Potier, tringulo de, 412 primario, 161 proteccin, indice de, 104 puente de Graetz, 580 puerta, 562 pulsacin de batido, 431 fundamental, 431 PWM,630 pyraleno, 166 reaccin de inducido, 488 reactancia de dispersin, 179 sncrona, 401 subtransitoria, 464 transitoria, 464 realimentacin, 655 recortadores, 617 rectificador dodecafsico, 590 monofsico doble onda, 578 monofsico media onda, 565 rectificadores controlados, 592 trifsicos, 585 red snubber, 559 regulacin primaria, 422 secundaria, 426 regulador con control de fase, 613 de induccin, 358 de tensin, 420 de velocidad, 420 on-off, 616 relacin de cortocircuito, 406 de transformacin, 174 rel, 44, 50 reluctancia, 10 reluctancia, motor de, 55 rendimiento de la rectificacin, 569 rendimiento, 107

También podría gustarte